Sie sind auf Seite 1von 791

“RECENT TRENDS IN POLITICAL LAW” (2006-2009)

Sec. of National Defense v. Manalo, 568 SCRA 42 (Writ of Amparo)----------------3

SJS v. Dangerous Drugs Board, 570 SCRA 410 (Random Drug Testing)-----------46

SJS v. Atienza, 545 SCRA 92 (Pandacan Oil Depot)----------------------------------71


League of Cities v. COMELEC, GR 176951, Nov. 18, 2008 (Conversion of Selected
Municipalities into Cities). See March 31, 2009 denying the Motion for Reconsideration
but see denial of Motion for Reconsideration of March 31 decision and later a December
21, 2009 decision reversing the initial pronouncement )----------------------------136

Chavez v. Gonzales, 545 SCRA 441 (Garci Tape in relation to Press Freedom)---157

**Province of North Cotabato v. GRP Panel, 568 SCRA 402 (Right to Information in
relation to Akbayan v. Aquino, GR 170516, July 16, 2008 (JPEPA); treaties; local
autonomy; constitutional amendment )--------------------------------------------187/260

Republic v. Eugenio, 545 SCRA 384 (AMLA and Ex Post Facto Law)-----------------301

Sema v. COMELEC, 558 SCRA 700 (Power of ARMM Regional Legislative Assembly to
Create Legislative District)--------------------------------------------------------------332

Neri v. Senate Committee, GR No. 180643, March 25, 2008 (Executive Privilege in
relation to Right to Information)-------------------------------------------------------342

Alcantara v. DENR, GR 161881 (IPRA in relation to FLGLA)--------------------------383

Secretary of DENR v. Mayor Yap, GR No. 167707, October 8, 2008 (Boracay Island Case
in relation to Regalian Doctrine)-------------------------------------------------------399

MMDA v. Concerned Citizens, GR 171947-48, December 18, 2008 (Clean-up of Manila


Bay)-------------------------------------------------------------------------------------430

Jacot v. Dal, GR 179848, November 27, 2008 (Dual Citizenship in relation to public
office)-----------------------------------------------------------------------------------459

Nicolas v. Romulo, GR 175888, February 11, 2009 (VFA)---------------------------474

Lozano v. Nograles, GR 187883, June 16, 2009 (Con-Ass.)--------------------------491

BANAT v. COMELEC, GR 179271, April 21, 2009 and July 8, 2009 (Party List; Allocation
of seats)----------------------------------------------------------------------------------498

1
Penera v. COMELEC, GR 181613, September 11, 2009 and Motion for Reconsideration of
November 25, 2009 reversing earlier decision (Premature Campaigning)----------536

KMU v. Director-General, GR 167930, April 19, 2006 (Privacy; National I.D. System)562

Pharmaceutical v. Health Secretary, GR No. 173034, Oct. 9, 2009 (Soft Law)-----588

Suplico v. NEDA, GR 178830, July 14, 2008 (Judicial Review)-----------------------638

Gov’t. of Hongkong v. Olalia, GR 153675, April 19, 2007 (Extradition)-------------658

Sabio v. Gordon, GR 174340, Oct. 17, 2006 (Inquiry in aid of legislation)---------672

David v. Arroyo, GR 171396, May 3, 2006 (State of Emergency)-------------------699

Quinto v. COMELEC, GR 189698, Dec. 1, 2009 (Appointive Officials running for election
in 2010; non-resignation) -------------------------------------------------------------767

2
Republic of the Philippines
Supreme Court
Manila

EN BANC

THE SECRETARY OF NATIONAL DEFENSE, THE CHIEF OF STAFF, G.R. No. 180906
ARMED FORCES OF THEPHILIPPINES,
Petitioners, Present:

PUNO, C.J.,
QUISUMBING,
YNARES-SANTIAGO,
CARPIO,
AUSTRIA-MARTINEZ,
- versus - CORONA,
CARPIO MORALES,
AZCUNA,
TINGA,
CHICO-NAZARIO,
VELASCO, JR.,
NACHURA,
RAYMOND MANALO and REYNALDO MANALO, REYES,
Respondents. LEONARDO-DE CASTRO,
and
BRION, JJ.

Promulgated:
October 7, 2008
x- - -- - - - - - - - - - - - - - - - - - - - - - - - - - - - - - - - - - - - - - - - - - - - - - - - -x

DECISION

PUNO, C.J.:

While victims of enforced disappearances are separated from the rest of the world behind
secret walls, they are not separated from the constitutional protection of their basic rights. The
constitution is an overarching sky that covers all in its protection. The case at bar involves the
rights to life, liberty and security in the first petition for a writ ofamparo filed before this Court.

This is an appeal via Petition for Review under Rule 45 of the Rules of Court in relation to
Section 19[1] of the Rule on the Writ of Amparo, seeking to reverse and set aside on both
questions of fact and law, the Decision promulgated by the Court of Appeals in C.A. G.R.

3
AMPARO No. 00001, entitled “Raymond Manalo and Reynaldo Manalo, petitioners, versus The
Secretary of National Defense, the Chief of Staff, Armed Forces of the Philippines, respondents.”

This case was originally a Petition for Prohibition, Injunction, and Temporary Restraining
Order (TRO)[2] filed before this Court by herein respondents (therein petitioners) on August 23,
2007 to stop herein petitioners (therein respondents) and/or their officers and agents from
depriving them of their right to liberty and other basic rights. Therein petitioners also sought
ancillary remedies, Protective Custody Orders, Appointment of Commissioner, Inspection and
Access Orders, and all other legal and equitable reliefs under Article VIII, Section 5(5)[3] of the
1987 Constitution and Rule 135, Section 6 of the Rules of Court. In our Resolution dated August
24, 2007, we (1) ordered the Secretary of the Department of National Defense and the Chief of
Staff of the AFP, their agents, representatives, or persons acting in their stead, including but not
limited to the Citizens Armed Forces Geographical Unit (CAFGU) to submit their Comment; and
(2) enjoined them from causing the arrest of therein petitioners, or otherwise restricting,
curtailing, abridging, or depriving them of their right to life, liberty, and other basic rights as
guaranteed under Article III, Section 1[4] of the 1987 Constitution.[5]

While the August 23, 2007 Petition was pending, the Rule on the Writ of Amparo took
effect on October 24, 2007. Forthwith, therein petitioners filed a Manifestation and Omnibus
Motion to Treat Existing Petition as Amparo Petition, to Admit Supporting Affidavits, and to Grant
Interim and Final Amparo Reliefs. They prayed that: (1) the petition be considered a Petition for
the Writ of Amparo under Sec. 26[6] of the Amparo Rule; (2) the Court issue the writ
commanding therein respondents to make a verified return within the period provided by law
and containing the specific matter required by law; (3) they be granted the interim reliefs
allowed by the Amparo Rule and all other reliefs prayed for in the petition but not covered by
the Amparo Rule; (4) the Court, after hearing, render judgment as required in Sec. 18[7] of
the Amparo Rule; and (5) all other just and equitable reliefs.[8]

On October 25, 2007, the Court resolved to treat the August 23, 2007 Petition as a
petition under the Amparo Rule and further resolved, viz:
WHEREFORE, let a WRIT OF AMPARO be issued to respondents requiring
them to file with the CA (Court of Appeals) a verified written return within five (5)
working days from service of the writ. We REMAND the petition to the CA and
designate the Division of Associate Justice Lucas P. Bersamin to conduct the
summary hearing on the petition on November 8, 2007 at 2:00 p.m. and decide the
petition in accordance with the Rule on the Writ of Amparo.[9]

On December 26, 2007, the Court of Appeals rendered a decision in favor of therein
petitioners (herein respondents), the dispositive portion of which reads, viz:
4
ACCORDINGLY, the PRIVILEGE OF THE WRIT OF AMPARO is GRANTED.

The respondents SECRETARY OF NATIONAL DEFENSE and AFP CHIEF OF


STAFF are hereby REQUIRED:

1. To furnish to the petitioners and to this Court within five days from notice of
this decision all official and unofficial reports of the investigation undertaken in
connection with their case, except those already on file herein;

2. To confirm in writing the present places of official assignment of M/Sgt


Hilario aka Rollie Castillo and Donald Caigas within five days from notice of this
decision.

3. To cause to be produced to this Court all medical reports, records and charts,
reports of any treatment given or recommended and medicines prescribed, if
any, to the petitioners, to include a list of medical and (sic) personnel (military
and civilian) who attended to them from February 14, 2006 until August 12,
2007 within five days from notice of this decision.

The compliance with this decision shall be made under the signature and
oath of respondent AFP Chief of Staff or his duly authorized deputy, the latter’s
authority to be express and made apparent on the face of the sworn compliance
with this directive.

SO ORDERED.[10]

Hence, this appeal. In resolving this appeal, we first unfurl the facts as alleged by herein
respondents:

Respondent Raymond Manalo recounted that about one or two weeks before February
14, 2006, several uniformed and armed soldiers and members of the CAFGU summoned to a
meeting all the residents of their barangay in San Idelfonso, Bulacan. Respondents were not
able to attend as they were not informed of the gathering, but Raymond saw some of the
soldiers when he passed by the barangay hall.[11]

On February 14, 2006, Raymond was sleeping in their house in Buhol na Mangga, San
Ildefonso, Bulacan. At past noon, several armed soldiers wearing white shirts, fatigue pants and
army boots, entered their house and roused him. They asked him if he was Bestre, but his
mother, Ester Manalo, replied that he was Raymond, not Bestre. The armed soldier slapped him
on both cheeks and nudged him in the stomach. He was then handcuffed, brought to the rear of
his house, and forced to the ground face down. He was kicked on the hip, ordered to stand and
face up to the light, then forcibly brought near the road. He told his mother to follow him, but
three soldiers stopped her and told her to stay.[12]

5
Among the men who came to take him, Raymond recognized brothers Michael de la Cruz,
Madning de la Cruz, “Puti” de la Cruz, and “Pula” de la Cruz, who all acted as lookout. They
were all members of the CAFGU and residing in Manuzon, San Ildefonso, Bulacan. He also
recognized brothers Randy Mendoza and Rudy Mendoza, also members of the CAFGU. While he
was being forcibly taken, he also saw outside of his house two barangay councilors, Pablo
Cunanan and Bernardo Lingasa, with some soldiers and armed men.[13]

The men forced Raymond into a white L300 van. Once inside, he was
blindfolded. Before being blindfolded, he saw the faces of the soldiers who took him. Later, in
his 18 months of captivity, he learned their names. The one who drove the van was Rizal Hilario
alias Rollie Castillo, whom he estimated was about 40 years of age or older. The leader of the
team who entered his house and abducted him was “Ganata.” He was tall, thin, curly-haired
and a bit old. Another one of his abductors was “George” who was tall, thin, white-skinned and
about 30 years old.[14]

The van drove off, then came to a stop. A person was brought inside the van and made
to sit beside Raymond. Both of them were beaten up. On the road, he recognized the voice of
the person beside him as his brother Reynaldo’s. The van stopped several times until they
finally arrived at a house. Raymond and Reynaldo were each brought to a different room. With
the doors of their rooms left open, Raymond saw several soldiers continuously hitting his brother
Reynaldo on the head and other parts of his body with the butt of their guns for about 15
minutes. After which, Reynaldo was brought to his (Raymond’s) room and it was his
(Raymond’s) turn to be beaten up in the other room. The soldiers asked him if he was a
member of the New People’s Army. Each time he said he was not, he was hit with the butt of
their guns. He was questioned where his comrades were, how many soldiers he had killed, and
how many NPA members he had helped. Each time he answered none, they hit him.[15]

In the next days, Raymond’s interrogators appeared to be high officials as the soldiers
who beat him up would salute them, call them “sir,” and treat them with respect. He was in
blindfolds when interrogated by the high officials, but he saw their faces when they arrived and
before the blindfold was put on. He noticed that the uniform of the high officials was different
from those of the other soldiers. One of those officials was tall and thin, wore white pants, tie,
and leather shoes, instead of combat boots. He spoke in Tagalog and knew much about his
parents and family, and a habeas corpus case filed in connection with the respondents’
abduction.[16] While these officials interrogated him, Raymond was not manhandled. But once
they had left, the soldier guards beat him up. When the guards got drunk, they also

6
manhandled respondents. During this time, Raymond was fed only at night, usually with left-
over and rotten food.[17]

On the third week of respondents’ detention, two men arrived while Raymond was
sleeping and beat him up. They doused him with urine and hot water, hit his stomach with a
piece of wood, slapped his forehead twice with a .45 pistol, punched him on the mouth, and
burnt some parts of his body with a burning wood. When he could no longer endure the torture
and could hardly breathe, they stopped. They then subjected Reynaldo to the same ordeal in
another room. Before their torturers left, they warned Raymond that they would come back the
next day and kill him.[18]

The following night, Raymond attempted to escape. He waited for the guards to get
drunk, then made noise with the chains put on him to see if they were still awake. When none of
them came to check on him, he managed to free his hand from the chains and jumped through
the window. He passed through a helipad and firing range and stopped near a fishpond where
he used stones to break his chains. After walking through a forested area, he came near a river
and an Iglesia ni Kristo church. He talked to some women who were doing the laundry, asked
where he was and the road to Gapan. He was told that he was in Fort Magsaysay.[19] He
reached the highway, but some soldiers spotted him, forcing him to run away. The soldiers
chased him and caught up with him. They brought him to another place near the entrance of
what he saw was FortMagsaysay. He was boxed repeatedly, kicked, and hit with chains until his
back bled. They poured gasoline on him. Then a so-called “Mam” or “Madam” suddenly called,
saying that she wanted to see Raymond before he was killed. The soldiers ceased the torture
and he was returned inside Fort Magsaysay where Reynaldo was detained.[20]

For some weeks, the respondents had a respite from all the torture. Their wounds were
treated. When the wounds were almost healed, the torture resumed, particularly when
respondents’ guards got drunk.[21]

Raymond recalled that sometime in April until May 2006, he was detained in a room
enclosed by steel bars. He stayed all the time in that small room measuring 1 x 2 meters, and
did everything there, including urinating, removing his bowels, bathing, eating and sleeping. He
counted that eighteen people[22] had been detained in that bartolina, including his brother
Reynaldo and himself.[23]

For about three and a half months, the respondents were detained
in Fort Magsaysay. They were kept in a small house with two rooms and a kitchen. One room
was made into the bartolina. The house was near the firing range, helipad and mango trees. At

7
dawn, soldiers marched by their house. They were also sometimes detained in what he only
knew as the “DTU.”[24]

At the DTU, a male doctor came to examine respondents. He checked their body and
eyes, took their urine samples and marked them. When asked how they were feeling, they
replied that they had a hard time urinating, their stomachs were aching, and they felt other
pains in their body. The next day, two ladies in white arrived. They also examined respondents
and gave them medicines, including orasol, amoxicillin and mefenamic acid. They brought with
them the results of respondents’ urine test and advised them to drink plenty of water and take
their medicine. The two ladies returned a few more times. Thereafter, medicines were sent
through the “master” of the DTU, “Master” Del Rosario alias Carinyoso at Puti. Respondents
were kept in the DTU for about two weeks. While there, he met a soldier named Efren who said
that Gen. Palparan ordered him to monitor and take care of them.[25]

One day, Rizal Hilario fetched respondents in a Revo vehicle. They, along with Efren and
several other armed men wearing fatigue suits, went to a detachment in Pinaud, San Ildefonso,
Bulacan. Respondents were detained for one or two weeks in a big two-storey house. Hilario
and Efren stayed with them. While there, Raymond was beaten up by Hilario’s men.[26]

From Pinaud, Hilario and Efren brought respondents to Sapang, San Miguel, Bulacan on
board the Revo. They were detained in a big unfinished house inside the compound of “Kapitan”
for about three months. When they arrived in Sapang, Gen. Palparan talked to them. They
were brought out of the house to a basketball court in the center of the compound and made to
sit. Gen. Palparan was already waiting, seated. He was about two arms’ length away from
respondents. He began by asking if respondents felt well already, to which Raymond replied in
the affirmative. He asked Raymond if he knew him. Raymond lied that he did not. He then
asked Raymond if he would be scared if he were made to face Gen. Palparan. Raymond
responded that he would not be because he did not believe that Gen. Palparan was an evil
man.[27]

Raymond narrated his conversation with Gen. Palparan in his affidavit, viz:
Tinanong ako ni Gen. Palparan, “Ngayon na kaharap mo na ako, di ka ba
natatakot sa akin?”

Sumagot akong, “Siyempre po, natatakot din…”

Sabi ni Gen. Palparan: “Sige, bibigyan ko kayo ng isang pagkakataon na


mabuhay, basta’t sundin n’yo ang lahat ng sasabihin ko… sabihin mo sa magulang
mo – huwag pumunta sa mga rali, sa hearing, sa Karapatan at sa Human Right

8
dahil niloloko lang kayo. Sabihin sa magulang at lahat sa bahay na huwag paloko
doon. Tulungan kami na kausapin si Bestre na sumuko na sa gobyerno.”[28]

Respondents agreed to do as Gen. Palparan told them as they felt they could not do
otherwise. At about 3:00 in the morning, Hilario, Efren and the former’s men - the same group
that abducted them - brought them to their parents’ house. Raymond was shown to his parents
while Reynaldo stayed in the Revo because he still could not walk. In the presence of Hilario and
other soldiers, Raymond relayed to his parents what Gen. Palparan told him. As they were
afraid, Raymond’s parents acceded. Hilario threatened Raymond’s parents that if they continued
to join human rights rallies, they would never see their children again. The respondents were
then brought back to Sapang.[29]

When respondents arrived back in Sapang, Gen. Palparan was about to leave. He was
talking with the four “masters” who were there: Arman, Ganata, Hilario and Cabalse.[30] When
Gen. Palparan saw Raymond, he called for him. He was in a big white vehicle. Raymond stood
outside the vehicle as Gen. Palparan told him to gain back his strength and be healthy and to
take the medicine he left for him and Reynaldo. He said the medicine was expensive at
Php35.00 each, and would make them strong. He also said that they should prove that they are
on the side of the military and warned that they would not be given another chance.[31] During
his testimony, Raymond identified Gen. Palparan by his picture.[32]

One of the soldiers named Arman made Raymond take the medicine left by Gen.
Palparan. The medicine, named “Alive,” was green and yellow. Raymond and Reynaldo were
each given a box of this medicine and instructed to take one capsule a day. Arman checked if
they were getting their dose of the medicine. The “Alive” made them sleep each time they took
it, and they felt heavy upon waking up.[33]

After a few days, Hilario arrived again. He took Reynaldo and left Raymond at
Sapang. Arman instructed Raymond that while in Sapang, he should introduce himself as
“Oscar,” a military trainee from Sariaya, Quezon, assigned in Bulacan. While there, he saw
again Ganata, one of the men who abducted him from his house, and got acquainted with other
military men and civilians.[34]

After about three months in Sapang, Raymond was brought to Camp Tecson under the
24th Infantry Battalion. He was fetched by three unidentified men in a big white vehicle. Efren
went with them. Raymond was then blindfolded. After a 30-minute ride, his blindfold was
removed. Chains were put on him and he was kept in the barracks.[35]

9
The next day, Raymond’s chains were removed and he was ordered to clean outside the
barracks. It was then he learned that he was in a detachment of the Rangers. There were many
soldiers, hundreds of them were training. He was also ordered to clean inside the barracks. In
one of the rooms therein, he met Sherlyn Cadapan from Laguna. She told him that she was a
student of the University of the Philippines and was abducted in Hagonoy, Bulacan. She
confided that she had been subjected to severe torture and raped. She was crying and longing
to go home and be with her parents. During the day, her chains were removed and she was
made to do the laundry.[36]

After a week, Reynaldo was also brought to Camp Tecson. Two days from his arrival, two
other captives, Karen Empeño and Manuel Merino, arrived. Karen and Manuel were put in the
room with “Allan” whose name they later came to know as Donald Caigas, called “master” or
“commander” by his men in the 24th Infantry Battalion. Raymond and Reynaldo were put in the
adjoining room. At times, Raymond and Reynaldo were threatened, and Reynaldo was beaten
up. In the daytime, their chains were removed, but were put back on at night. They were
threatened that if they escaped, their families would all be killed.[37]

On or about October 6, 2006, Hilario arrived in Camp Tecson. He told the detainees that
they should be thankful they were still alive and should continue along their “renewed
life.” Before the hearing of November 6 or 8, 2006, respondents were brought to their parents
to instruct them not to attend the hearing. However, their parents had already left
for Manila. Respondents were brought back to Camp Tecson. They stayed in that camp from
September 2006 to November 2006, and Raymond was instructed to continue using the name
“Oscar” and holding himself out as a military trainee. He got acquainted with soldiers of the
24th Infantry Battalion whose names and descriptions he stated in his affidavit.[38]

On November 22, 2006, respondents, along with Sherlyn, Karen, and Manuel, were
transferred to a camp of the 24th Infantry Battalion in Limay, Bataan. There were many huts in
the camp. They stayed in that camp until May 8, 2007. Some soldiers of the battalion stayed
with them. While there, battalion soldiers whom Raymond knew as “Mar” and “Billy” beat him
up and hit him in the stomach with their guns. Sherlyn and Karen also suffered enormous
torture in the camp. They were all made to clean, cook, and help in raising livestock.[39]

Raymond recalled that when “Operation Lubog” was launched, Caigas and some other
soldiers brought him and Manuel with them to take and kill all sympathizers of the NPA. They
were brought to Barangay Bayan-bayanan, Bataan where he witnessed the killing of an old man
doing kaingin. The soldiers said he was killed because he had a son who was a member of the
NPA and he coddled NPA members in his house.[40] Another time, in another “Operation Lubog,”
10
Raymond was brought to Barangay Orion in a house where NPA men stayed. When they
arrived, only the old man of the house who was sick was there. They spared him and killed only
his son right before Raymond’s eyes.[41]

From Limay, Raymond, Reynaldo, Sherlyn, Karen, and Manuel were transferred to
Zambales, in a safehouse near the sea. Caigas and some of his men stayed with them. A retired
army soldier was in charge of the house. Like in Limay, the five detainees were made to do
errands and chores. They stayed in Zambales from May 8 or 9, 2007 until June 2007.[42]

In June 2007, Caigas brought the five back to the camp in Limay. Raymond, Reynaldo,
and Manuel were tasked to bring food to detainees brought to the camp. Raymond narrated
what he witnessed and experienced in the camp, viz:
Isang gabi, sinabihan kami ni Donald (Caigas) na matulog na kami. Nakita
ko si Donald na inaayos ang kanyang baril, at nilagyan ng silenser. Sabi ni Donald
na kung mayroon man kaming makita o marinig, walang nangyari. Kinaumagahan,
nakita naming ang bangkay ng isa sa mga bihag na dinala sa kampo. Mayroong
binuhos sa kanyang katawan at ito’y sinunog. Masansang ang amoy.

Makaraan ang isang lingo, dalawang bangkay and ibinaba ng mga


unipormadong sundalo mula sa 6 x 6 na trak at dinala sa loob ng kampo. May
naiwang mga bakas ng dugo habang hinihila nila ang mga bangkay. Naamoy ko
iyon nang nililinis ang bakas.

Makalipas ang isa o dalawang lingo, may dinukot sila na dalawang


Ita. Itinali sila sa labas ng kubo, piniringan, ikinadena at labis na binugbog. Nakita
kong nakatakas ang isa sa kanila at binaril siya ng sundalo ngunit hindi siya
tinamaan. Iyong gabi nakita kong pinatay nila iyong isang Ita malapit sa Post 3;
sinilaban ang bangkay at ibinaon ito.

Pagkalipas ng halos 1 buwan, 2 pang bangkay ang dinala sa


kampo. Ibinaba ang mga bangkay mula sa pick up trak, dinala ang mga bangkay sa
labas ng bakod. Kinaumagahan nakita kong mayroong sinilaban, at
napakamasangsang ang amoy.

May nakilala rin akong 1 retiradong koronel at 1 kasama niya. Pinakain ko


sila. Sabi nila sa akin na dinukot sila sa Bataan. Iyong gabi, inilabas sila at hindi ko
na sila nakita.

xxx xxx xxx

Ikinadena kami ng 3 araw. Sa ikatlong araw, nilabas ni Lat si Manuel dahil


kakausapin daw siya ni Gen. Palparan. Nakapiring si Manuel, wala siyang suot
pang-itaas, pinosasan. Nilakasan ng mga sundalo ang tunog na galing sa istiryo ng
sasakyan. Di nagtagal, narinig ko ang hiyaw o ungol ni Manuel. Sumilip ako sa
isang haligi ng kamalig at nakita kong sinisilaban si Manuel.

11
Kinaumagahan, naka-kadena pa kami. Tinanggal ang mga kadena mga 3 o
4 na araw pagkalipas. Sinabi sa amin na kaya kami nakakadena ay dahil
pinagdedesisyunan pa ng mga sundalo kung papatayin kami o hindi.

Tinanggal ang aming kadena. Kinausap kami ni Donald. Tinanong kami


kung ano ang sabi ni Manuel sa amin. Sabi ni Donald huwag na raw naming
hanapin ang dalawang babae at si Manuel, dahil magkakasama na yung tatlo. Sabi
pa ni Donald na kami ni Reynaldo ay magbagong buhay at ituloy namin ni Reynaldo
ang trabaho. Sa gabi, hindi na kami kinakadena.[43]

On or about June 13, 2007, Raymond and Reynaldo were brought to Pangasinan,
ostensibly to raise poultry for Donald (Caigas). Caigas told respondents to also farm his land, in
exchange for which, he would take care of the food of their family. They were also told that
they could farm a small plot adjoining his land and sell their produce. They were no longer put in
chains and were instructed to use the names Rommel (for Raymond) and Rod (for Reynaldo)
and represent themselves as cousins from Rizal, Laguna.[44]

Respondents started to plan their escape. They could see the highway from where they
stayed. They helped farm adjoining lands for which they were paid Php200.00 or Php400.00 and
they saved their earnings. When they had saved Php1,000.00 each, Raymond asked a neighbor
how he could get a cellular phone as he wanted to exchange text messages with a girl who lived
nearby. A phone was pawned to him, but he kept it first and did not use it. They earned some
more until they had saved Php1,400.00 between them.

There were four houses in the compound. Raymond and Reynaldo were housed in one of
them while their guards lived in the other three. Caigas entrusted respondents to Nonong, the
head of the guards. Respondents’ house did not have electricity. They used a lamp. There was
no television, but they had a radio. In the evening of August 13, 2007, Nonong and his cohorts
had a drinking session. At about 1:00 a.m., Raymond turned up the volume of the radio. When
none of the guards awoke and took notice, Raymond and Reynaldo proceeded towards the
highway, leaving behind their sleeping guards and barking dogs. They boarded a bus bound
for Manila and were thus freed from captivity.[45]

Reynaldo also executed an affidavit affirming the contents of Raymond’s affidavit insofar
as they related to matters they witnessed together. Reynaldo added that when they were taken
from their house on February 14, 2006, he saw the faces of his abductors before he was
blindfolded with his shirt. He also named the soldiers he got acquainted with in the 18 months
he was detained. When Raymond attempted to escape from Fort Magsaysay, Reynaldo was
severely beaten up and told that they were indeed members of the NPA because Raymond

12
escaped. With a .45 caliber pistol, Reynaldo was hit on the back and punched in the face until
he could no longer bear the pain.

At one point during their detention, when Raymond and Reynaldo were in Sapang,
Reynaldo was separated from Raymond and brought to Pinaud by Rizal Hilario. He was kept in
the house of Kapitan, a friend of Hilario, in a mountainous area. He was instructed to use the
name “Rodel” and to represent himself as a military trainee from Meycauayan,
Bulacan. Sometimes, Hilario brought along Reynaldo in his trips. One time, he was brought to a
market in San Jose, del Monte, Bulacan and made to wait in the vehicle while Hilario was
buying. He was also brought to Tondo, Manila where Hilario delivered boxes of “Alive” in
different houses. In these trips, Hilario drove a black and red vehicle. Reynaldo was blindfolded
while still in Bulacan, but allowed to remove the blindfold once outside the province. In one of
their trips, they passed by FortMagsaysay and Camp Tecson where Reynaldo saw the sign board,
“Welcome to Camp Tecson.”[46]

Dr. Benito Molino, M.D., corroborated the accounts of respondents Raymond and
Reynaldo Manalo. Dr. Molino specialized in forensic medicine and was connected with the
Medical Action Group, an organization handling cases of human rights violations, particularly
cases where torture was involved. He was requested by an NGO to conduct medical
examinations on the respondents after their escape. He first asked them about their ordeal,
then proceeded with the physical examination. His findings showed that the scars borne by
respondents were consistent with their account of physical injuries inflicted upon them. The
examination was conducted on August 15, 2007, two days after respondents’ escape, and the
results thereof were reduced into writing. Dr. Molino took photographs of the scars. He testified
that he followed the Istanbul Protocol in conducting the examination.[47]

Petitioners dispute respondents’ account of their alleged abduction and torture. In


compliance with the October 25, 2007 Resolution of the Court, they filed a Return of the Writ
of Amparo admitting the abduction but denying any involvement therein, viz:
13. Petitioners Raymond and Reynaldo Manalo were not at any time
arrested, forcibly abducted, detained, held incommunicado, disappeared or under
the custody by the military. This is a settled issue laid to rest in the habeas
corpus case filed in their behalf by petitioners’ parents before the Court of Appeals
in C.A.-G.R. SP No. 94431 against M/Sgt. Rizal Hilario aka Rollie Castillo, as head of
the 24th Infantry Battalion; Maj. Gen. Jovito Palparan, as Commander of the
7th Infantry Division in Luzon; Lt. Gen. Hermogenes Esperon, in his capacity as the
Commanding General of the Philippine Army, and members of the Citizens Armed
Forces Geographical Unit (CAFGU), namely: Michael dela Cruz, Puti dela Cruz,
Madning dela Cruz, Pula dela Cruz, Randy Mendoza and Rudy Mendoza. The
respondents therein submitted a return of the writ… On July 4, 2006, the Court of
13
Appeals dropped as party respondents Lt. Gen. Hermogenes C. Esperon, Jr., then
Commanding General of the Philippine Army, and on September 19, 2006, Maj.
(sic) Jovito S. Palparan, then Commanding General, 7th Infantry Division, Philippine
Army, stationed at Fort Magsaysay, Palayan City, Nueva Ecija, upon a finding that
no evidence was introduced to establish their personal involvement in the taking of
the Manalo brothers. In a Decision dated June 27, 2007…, it exonerated M/Sgt.
Rizal Hilario aka Rollie Castillo for lack of evidence establishing his involvement in
any capacity in the disappearance of the Manalo brothers, although it held that the
remaining respondents were illegally detaining the Manalo brothers and ordered
them to release the latter.[48]

Attached to the Return of the Writ was the affidavit of therein respondent (herein
petitioner) Secretary of National Defense, which attested that he assumed office only on August
8, 2007 and was thus unaware of the Manalo brothers’ alleged abduction. He also claimed that:

7. The Secretary of National Defense does not engage in actual military


directional operations, neither does he undertake command directions of the AFP
units in the field, nor in any way micromanage the AFP operations. The principal
responsibility of the Secretary of National Defense is focused in providing
strategic policy direction to the Department (bureaus and agencies) including the
Armed Forces of the Philippines;

8. In connection with the Writ of Amparo issued by the Honorable Supreme


Court in this case, I have directed the Chief of Staff, AFP to institute immediate
action in compliance with Section 9(d) of theAmparo Rule and to submit report
of such compliance… Likewise, in a Memorandum Directive also dated October
31, 2007, I have issued a policy directive addressed to the Chief of Staff, AFP
that the AFP should adopt the following rules of action in the event the Writ
of Amparo is issued by a competent court against any members of the AFP:

(1) to verify the identity of the aggrieved party;

(2) to recover and preserve evidence related to the death or disappearance of


the person identified in the petition which may aid in the prosecution of the
person or persons responsible;

(3) to identify witnesses and obtain statements from them concerning the
death or disappearance;

(4) to determine the cause, manner, location and time of death or


disappearance as well as any pattern or practice that may have brought
about the death or disappearance;

(5) to identify and apprehend the person or persons involved in the death or
disappearance; and

(6) to bring the suspected offenders before a competent court.[49]

14
Therein respondent AFP Chief of Staff also submitted his own affidavit, attached to the
Return of the Writ, attesting that he received the above directive of therein respondent
Secretary of National Defense and that acting on this directive, he did the following:
3.1. As currently designated Chief of Staff, Armed Forces of the Philippines
(AFP), I have caused to be issued directive to the units of the AFP for the purpose of
establishing the circumstances of the alleged disappearance and the recent
reappearance of the petitioners.

3.2. I have caused the immediate investigation and submission of the result
thereof to Higher headquarters and/or direct the immediate conduct of the
investigation on the matter by the concerned unit/s, dispatching Radio Message on
November 05, 2007, addressed to the Commanding General, Philippine Army (Info:
COMNOLCOM, CG, 71D PA and CO 24 IB PA). A Copy of the Radio Message is
attached as ANNEX “3” of this Affidavit.

3.3. We undertake to provide result of the investigations conducted or to be


conducted by the concerned unit relative to the circumstances of the alleged
disappearance of the persons in whose favor the Writ of Amparo has been sought
for as soon as the same has been furnished Higher headquarters.

3.4. A parallel investigation has been directed to the same units relative to
another Petition for the Writ of Amparo (G.R. No. 179994) filed at the instance of
relatives of a certain Cadapan and Empeño pending before the Supreme Court.

3.5. On the part of the Armed Forces, this respondent will exert earnest
efforts to establish the surrounding circumstances of the disappearances of the
petitioners and to bring those responsible, including any military personnel if shown
to have participated or had complicity in the commission of the complained acts, to
the bar of justice, when warranted by the findings and the competent evidence that
may be gathered in the process.[50]

Also attached to the Return of the Writ was the affidavit of Lt. Col. Felipe Anontado, INF
(GSC) PA, earlier filed in G.R. No. 179994, another amparo case in this Court, involving
Cadapan, Empeño and Merino, which averred among others, viz:

10) Upon reading the allegations in the Petition implicating the 24th Infantry
Batallion detachment as detention area, I immediately went to the 24th IB
detachment in Limay, Bataan and found no untoward incidents in the area nor any
detainees by the name of Sherlyn Cadapan, Karen Empeño and Manuel Merino
being held captive;

11) There was neither any reports of any death of Manuel Merino in the
24th IB in Limay, Bataan;

12) After going to the 24th IB in Limay, Bataan, we made further inquiries
with the Philippine National Police, Limay, Bataan regarding the alleged detentions
or deaths and were informed that none was reported to their good office;
15
13) I also directed Company Commander 1st Lt. Romeo Publico to inquire
into the alleged beachhouse in Iba, Zambales also alleged to be a detention place
where Sherlyn Cadapan, Karen Empeño and Manuel Merino were detained. As per
the inquiry, however, no such beachhouse was used as a detention place found to
have been used by armed men to detain Cadapan, Empeño and Merino.[51]

It was explained in the Return of the Writ that for lack of sufficient time, the affidavits of
Maj. Gen Jovito S. Palparan (Ret.), M/Sgt. Rizal Hilario aka Rollie Castillo, and other persons
implicated by therein petitioners could not be secured in time for the submission of the Return
and would be subsequently submitted.[52]

Herein petitioners presented a lone witness in the summary hearings, Lt. Col. Ruben U.
Jimenez, Provost Marshall, 7th Infantry Division, Philippine Army, based
in FortMagsaysay, Palayan City, Nueva Ecija. The territorial jurisdiction of this Division covers
Nueva Ecija, Aurora, Bataan, Bulacan, Pampanga, Tarlac and a portion of Pangasinan.[53] The
24th Infantry Battalion is part of the 7th Infantry Division.[54]

On May 26, 2006, Lt. Col. Jimenez was directed by the Commanding General of the
7 Infantry Division, Maj. Gen. Jovito Palaran,[55] through his Assistant Chief of Staff,[56] to
th

investigate the alleged abduction of the respondents by CAFGU auxiliaries under his unit,
namely: CAA Michael de la Cruz; CAA Roman de la Cruz, aka Puti; CAA Maximo de la Cruz, aka
Pula; CAA Randy Mendoza; ex-CAA Marcelo de la Cruz aka Madning; and a civilian named Rudy
Mendoza. He was directed to determine: (1) the veracity of the abduction of Raymond and
Reynaldo Manalo by the alleged elements of the CAFGU auxiliaries; and (2) the administrative
liability of said auxiliaries, if any.[57] Jimenez testified that this particular investigation was
initiated not by a complaint as was the usual procedure, but because the Commanding General
saw news about the abduction of the Manalo brothers on the television, and he was concerned
about what was happening within his territorial jurisdiction.[58]

Jimenez summoned all six implicated persons for the purpose of having them execute
sworn statements and conducting an investigation on May 29, 2006.[59] The investigation
started at 8:00 in the morning and finished at 10:00 in the evening.[60] The investigating officer,
Technical Sgt. Eduardo Lingad, took the individual sworn statements of all six persons on that
day. There were no other sworn statements taken, not even of the Manalo family, nor were
there other witnesses summoned and investigated[61] as according to Jimenez, the directive to
him was only to investigate the six persons.[62]

Jimenez was beside Lingad when the latter took the statements.[63] The six persons were
not known to Jimenez as it was in fact his first time to meet them.[64] During the entire time

16
that he was beside Lingad, a subordinate of his in the Office of the Provost Marshall, Jimenez did
not propound a single question to the six persons.[65]

Jimenez testified that all six statements were taken on May 29, 2006, but Marcelo
Mendoza and Rudy Mendoza had to come back the next day to sign their statements as the
printing of their statements was interrupted by a power failure. Jimenez testified that the two
signed on May 30, 2006, but the jurats of their statements indicated that they were signed
on May 29, 2006.[66] When the Sworn Statements were turned over to Jimenez, he personally
wrote his investigation report. He began writing it in the afternoon ofMay 30, 2006 and finished
it on June 1, 2006.[67] He then gave his report to the Office of the Chief of Personnel.[68]

As petitioners largely rely on Jimenez’s Investigation Report dated June 1, 2006 for their
evidence, the report is herein substantially quoted:

III. BACKGROUND OF THE CASE


4. This pertains to the abduction of RAYMOND MANALO and REYNALDO
MANALO who were forcibly taken from their respective homes in Brgy. Buhol na
Mangga, San Ildefonso, Bulacan on 14 February 2006 by unidentified armed men
and thereafter were forcibly disappeared. After the said incident, relatives of the
victims filed a case for Abduction in the civil court against the herein suspects:
Michael dela Cruz, Madning dela Cruz, Puti Dela Cruz, Pula Dela Cruz, Randy
Mendoza and Rudy Mendoza as alleged members of the Citizen Armed Forces
Geographical Unit (CAFGU).

a) Sworn statement of CAA Maximo F. dela Cruz, aka Pula dated 29 May
2006 in (Exhibit “B”) states that he was at Sitio Mozon, Brgy. Bohol na Mangga,
San Ildefonso, Bulacan doing the concrete building of a church located nearby his
residence, together with some neighbor thereat. He claims that on 15 February
2006, he was being informed by Brgy. Kagawad Pablo Umayan about the abduction
of the brothers Raymond and Reynaldo Manalo. As to the allegation that he was
one of the suspects, he claims that they only implicated him because he was a
CAFGU and that they claimed that those who abducted the Manalo brothers are
members of the Military and CAFGU. Subject vehemently denied any participation
or involvement on the abduction of said victims.

b) Sworn statement of CAA Roman dela Cruz y Faustino Aka Puti dtd 29
May 2006 in (Exhibit “C”) states that he is a resident of Sitio Muzon, Brgy. Buhol na
Mangga, San Ildefonso, Bulacan and a CAA member based at Biak na Bato
Detachment, San Miguel, Bulacan. He claims that Raymond and Reynaldo Manalo
being his neighbors are active members/sympathizers of the CPP/NPA and he also
knows their elder Rolando Manalo @ KA BESTRE of being an NPA Leader operating
in their province. That at the time of the alleged abduction of the two (2) brothers
and for accusing him to be one of the suspects, he claims that on February 14,
2006, he was one of those working at the concrete chapel being constructed nearby
his residence. He claims further that he just came only to know about the incident
17
on other day (15 Feb 06) when he was being informed by Kagawad Pablo
Kunanan. That subject CAA vehemently denied any participation about the incident
and claimed that they only implicated him because he is a member of the CAFGU.

c) Sworn Statement of CAA Randy Mendoza y Lingas dated 29 May 2006 in


(Exhibit “O”) states that he is a resident of Brgy. Buhol na Mangga, San Ildefonso,
Bulacan and a member of CAFGU based at Biak na Bato Detachment. That being a
neighbor, he was very much aware about the background of the two (2) brothers
Raymond and Reynaldo as active supporters of the CPP NPA in their Brgy. and he
also knew their elder brother “KUMANDER BESTRE” TN: Rolando Manalo. Being one
of the accused, he claims that on 14 February 2006, he was at Brgy. Magmarate,
San Miguel, Bulacan in the house of his aunt and he learned only about the incident
when he arrived home in their place. He claims further that the only reason why
they implicated him was due to the fact that his mother has filed a criminal charge
against their brother Rolando Manalo @ KA BESTRE who is an NPA Commander who
killed his father and for that reason they implicated him in support of their
brother. Subject CAA vehemently denied any involvement on the abduction of said
Manalo brothers.

d) Sworn Statement of Rudy Mendoza y Lingasa dated May 29, 2006 in


(Exhibit “E”) states that he is a resident of Brgy. Marungko, Angat, Bulacan. He
claims that Raymond and Reynaldo Manalo are familiar to him being his barriomate
when he was still unmarried and he knew them since childhood. Being one of the
accused, he claims that on 14 February 2006, he was at his residence in Brgy.
Marungko, Angat, Bulacan. He claims that he was being informed only about the
incident lately and he was not aware of any reason why the two (2) brothers were
being abducted by alleged members of the military and CAFGU. The only reason he
knows why they implicated him was because there are those people who are angry
with their family particularly victims of summary execution (killing) done by their
brother @ KA Bestre Rolando Manalo who is an NPA leader. He claims further that
it was their brother @ KA BESTRE who killed his father and he was living witness to
that incident. Subject civilian vehemently denied any involvement on the abduction
of the Manalo brothers.

e) Sworn statement of Ex-CAA Marcelo dala Cruz dated 29 May 2006 in


(Exhibit “F”) states that he is a resident of Sitio Muzon, Brgy. Buhol na Mangga,
San Ildefonso, Bulacan, a farmer and a former CAA based at Biak na Bato, San
Miguel, Bulacan. He claims that Raymond and Reynaldo Manalo are familiar to him
being their barrio mate. He claims further that they are active supporters of
CPP/NPA and that their brother Rolando Manalo @ KA BESTRE is an NPA
leader. Being one of the accused, he claims that on 14 February 2006, he was in
his residence at Sitio Muzon, Brgy. Buhol na Mangga, San Ildefonso, Bulacan. That
he vehemently denied any participation of the alleged abduction of the two (2)
brothers and learned only about the incident when rumors reached him by his
barrio mates. He claims that his implication is merely fabricated because of his
relationship to Roman and Maximo who are his brothers.

f) Sworn statement of Michael dela Cruz y Faustino dated 29 May 2006 in


(Exhibit “G”) states that he is a resident of Sitio Muzon, Brgy. Buhol na Mangga,
San Ildefonso, Bulacan, the Chief of Brgy. Tanod and a CAFGU member based
18
at Biak na Bato Detachment, San Miguel, Bulacan. He claims that he knew very
well the brothers Raymond and Reynaldo Manalo in their barangay for having been
the Tanod Chief for twenty (20) years. He alleged further that they are active
supporters or sympathizers of the CPP/NPA and whose elder brother Rolando
Manalo @ KA BESTRE is an NPA leader operating within the area. Being one of the
accused, he claims that on 14 Feb 2006 he was helping in the construction of their
concrete chapel in their place and he learned only about the incident which is the
abduction of Raymond and Reynaldo Manalo when one of the Brgy. Kagawad in the
person of Pablo Cunanan informed him about the matter. He claims further that he
is truly innocent of the allegation against him as being one of the abductors and he
considers everything fabricated in order to destroy his name that remains loyal to
his service to the government as a CAA member.

IV. DISCUSSION

5. Based on the foregoing statements of respondents in this particular


case, the proof of linking them to the alleged abduction and disappearance of
Raymond and Reynaldo Manalo that transpired on 14 February 2006 at Sitio Muzon,
Brgy. Buhol na Mangga, San Ildefonso, Bulacan, is unsubstantiated. Their alleged
involvement theretofore to that incident is considered doubtful, hence, no basis to
indict them as charged in this investigation.

Though there are previous grudges between each families (sic) in the past
to quote: the killing of the father of Randy and Rudy Mendoza by @ KA BESTRE TN:
Rolando Manalo, this will not suffice to establish a fact that they were the ones who
did the abduction as a form of revenge. As it was also stated in the testimony of
other accused claiming that the Manalos are active sympathizers/supporters of the
CPP/NPA, this would not also mean, however, that in the first place, they were in
connivance with the abductors. Being their neighbors and as members of CAFGU’s,
they ought to be vigilant in protecting their village from any intervention by the
leftist group, hence inside their village, they were fully aware of the activities of
Raymond and Reynaldo Manalo in so far as their connection with the CPP/NPA is
concerned.

V. CONCLUSION

6. Premises considered surrounding this case shows that the alleged


charges of abduction committed by the above named respondents has not been
established in this investigation. Hence, it lacks merit to indict them for any
administrative punishment and/or criminal liability. It is therefore concluded that
they are innocent of the charge.

VI. RECOMMENDATIONS

7. That CAAs Michael F. dela Cruz, Maximo F. Dela Cruz, Roman dela Cruz,
Randy Mendoza, and two (2) civilians Maximo F. Dela Cruz and Rudy L. Mendoza be
exonerated from the case.

8. Upon approval, this case can be dropped and closed.[69]

19
In this appeal under Rule 45, petitioners question the appellate court’s assessment of the
foregoing evidence and assail the December 26, 2007 Decision on the following grounds, viz:

I.

THE COURT OF APPEALS SERIOUSLY AND GRIEVOUSLY ERRED IN BELIEVING AND


GIVING FULL FAITH AND CREDIT TO THE INCREDIBLE, UNCORROBORATED,
CONTRADICTED, AND OBVIOUSLY SCRIPTED, REHEARSED AND SELF-SERVING
AFFIDAVIT/TESTIMONY OF HEREIN RESPONDENT RAYMOND MANALO.

II.

THE COURT OF APPEALS SERIOUSLY AND GRIEVOUSLY ERRED IN REQUIRING


RESPONDENTS (HEREIN PETITIONERS) TO: (A) FURNISH TO THE MANALO
BROTHER(S) AND TO THE COURT OF APPEALS ALL OFFICIAL AND UNOFFICIAL
REPORTS OF THE INVESTIGATION UNDERTAKEN IN CONNECTION WITH THEIR
CASE, EXCEPT THOSE ALREADY IN FILE WITH THE COURT; (B) CONFIRM IN
WRITING THE PRESENT PLACES OF OFFICIAL ASSIGNMENT OF M/SGT. HILARIO
aka ROLLIE CASTILLO AND DONALD CAIGAS; AND (C) CAUSE TO BE PRODUCED
TO THE COURT OF APPEALS ALL MEDICAL REPORTS, RECORDS AND CHARTS, AND
REPORTS OF ANY TREATMENT GIVEN OR RECOMMENDED AND MEDICINES
PRESCRIBED, IF ANY, TO THE MANALO BROTHERS, TO INCLUDE A LIST OF
MEDICAL PERSONNEL (MILITARY AND CIVILIAN) WHO ATTENDED TO THEM FROM
FEBRUARY 14, 2006 UNTIL AUGUST 12, 2007.[70]

The case at bar is the first decision on the application of the Rule on the Writ
of Amparo (Amparo Rule). Let us hearken to its beginning.

The adoption of the Amparo Rule surfaced as a recurring proposition in the


recommendations that resulted from a two-day National Consultative Summit on Extrajudicial
Killings and Enforced Disappearances sponsored by the Court on July 16-17,
2007. The Summit was “envisioned to provide a broad and fact-based perspective on the issue
of extrajudicial killings and enforced disappearances,”[71] hence “representatives from all sides of
the political and social spectrum, as well as all the stakeholders in the justice
[72]
system” participated in mapping out ways to resolve the crisis.

On October 24, 2007, the Court promulgated the Amparo Rule “in light of the prevalence
of extralegal killing and enforced disappearances.”[73] It was an exercise for the first time of the
Court’s expanded power to promulgate rules to protect our people’s constitutional rights, which
made its maiden appearance in the 1987 Constitution in response to the Filipino experience of
the martial law regime.[74] As the Amparo Rule was intended to address the intractable problem
of “extralegal killings” and “enforced disappearances,” its coverage, in its present form, is
20
confined to these two instances or to threats thereof. “Extralegal killings” are “killings
committed without due process of law, i.e., without legal safeguards or judicial
proceedings.”[75] On the other hand, “enforced disappearances” are “attended by the following
characteristics: an arrest, detention or abduction of a person by a government official or
organized groups or private individuals acting with the direct or indirect acquiescence of the
government; the refusal of the State to disclose the fate or whereabouts of the person
concerned or a refusal to acknowledge the deprivation of liberty which places such persons
outside the protection of law.”[76]

The writ of amparo originated in Mexico. “Amparo” literally means “protection” in


Spanish.[77] In 1837, de Tocqueville’s Democracy in America became available inMexico and
stirred great interest. Its description of the practice of judicial review in the U.S. appealed to
many Mexican jurists.[78] One of them, Manuel Crescencio Rejón, drafted a constitutional
provision for his native state, Yucatan,[79] which granted judges the power to protect all persons
in the enjoyment of their constitutional and legal rights. This idea was incorporated into the
national constitution in 1847, viz:
The federal courts shall protect any inhabitant of the Republic in the exercise
and preservation of those rights granted to him by this Constitution and by laws
enacted pursuant hereto, against attacks by the Legislative and Executive powers
of the federal or state governments, limiting themselves to granting protection in
the specific case in litigation, making no general declaration concerning the statute
or regulation that motivated the violation.[80]

Since then, the protection has been an important part of Mexican constitutionalism.[81] If,
after hearing, the judge determines that a constitutional right of the petitioner is being violated,
he orders the official, or the official’s superiors, to cease the violation and to take the necessary
measures to restore the petitioner to the full enjoyment of the right in question. Amparo thus
combines the principles of judicial review derived from the U.S. with the limitations on judicial
power characteristic of the civil law tradition which prevails in Mexico. It enables courts to
enforce the constitution by protecting individual rights in particular cases, but prevents them
from using this power to make law for the entire nation.[82]

The writ of amparo then spread throughout the Western Hemisphere, gradually evolving
into various forms, in response to the particular needs of each country.[83] It became, in the
words of a justice of the Mexican Federal Supreme Court, one piece of Mexico’s self-attributed
“task of conveying to the world’s legal heritage that institution which, as a shield of human
dignity, her own painful history conceived.”[84] What began as a protection against acts or
omissions of public authorities in violation of constitutional rights later evolved for several

21
purposes: (1) amparo libertad for the protection of personal freedom, equivalent to the habeas
corpus writ; (2) amparo contra leyes for the judicial review of the constitutionality of statutes;
(3) amparo casacion for the judicial review of the constitutionality and legality of a judicial
decision; (4) amparo administrativo for the judicial review of administrative actions; and
(5) amparo agrario for the protection of peasants’ rights derived from the agrarian reform
process.[85]

In Latin American countries, except Cuba, the writ of amparo has been constitutionally
adopted to protect against human rights abuses especially committed in countries under military
juntas. In general, these countries adopted an all-encompassing writ to protect the whole
gamut of constitutional rights, including socio-economic rights.[86] Other countries
like Colombia, Chile, Germany and Spain, however, have chosen to limit the protection of the
writ of amparo only to some constitutional guarantees or fundamental rights.[87]

In the Philippines, while the 1987 Constitution does not explicitly provide for the writ
of amparo, several of the above amparo protections are guaranteed by our charter. The second
paragraph of Article VIII, Section 1 of the 1987 Constitution, the Grave Abuse Clause, provides
for the judicial power “to determine whether or not there has been a grave abuse of discretion
amounting to lack or excess of jurisdiction on the part of any branch or instrumentality of the
Government.” The Clause accords a similar general protection to human rights extended by
the amparo contra leyes, amparo casacion, and amparo administrativo. Amparo libertad is
comparable to the remedy of habeas corpus found in several provisions of the 1987
Constitution.[88] The Clause is an offspring of the U.S. common law tradition of judicial review,
which finds its roots in the 1803 case of Marbury v. Madison.[89]

While constitutional rights can be protected under the Grave Abuse Clause through
remedies of injunction or prohibition under Rule 65 of the Rules of Court and a petition
for habeas corpus under Rule 102,[90] these remedies may not be adequate to address the
pestering problem of extralegal killings and enforced disappearances. However, with the
swiftness required to resolve a petition for a writ of amparo through summary proceedings and
the availability of appropriate interim and permanent reliefs under the Amparo Rule, this hybrid
writ of the common law and civil law traditions - borne out of the Latin American and Philippine
experience of human rights abuses - offers a better remedy to extralegal killings and enforced
disappearances and threats thereof. The remedy provides rapid judicial relief as it partakes of a
summary proceeding that requires only substantial evidence to make the appropriate reliefs
available to the petitioner; it is not an action to determine criminal guilt requiring proof beyond
reasonable doubt, or liability for damages requiring preponderance of evidence, or

22
administrative responsibility requiring substantial evidence that will require full and exhaustive
proceedings.[91]

The writ of amparo serves both preventive and curative roles in addressing the problem
of extralegal killings and enforced disappearances. It is preventive in that it breaks the
expectation of impunity in the commission of these offenses; it is curative in that it facilitates
the subsequent punishment of perpetrators as it will inevitably yield leads to subsequent
investigation and action. In the long run, the goal of both the preventive and curative roles is to
deter the further commission of extralegal killings and enforced disappearances.

In the case at bar, respondents initially filed an action for “Prohibition, Injunction, and
Temporary Restraining Order”[92] to stop petitioners and/or their officers and agents from
depriving the respondents of their right to liberty and other basic rights on August 23,
2007,[93] prior to the promulgation of the Amparo Rule. They also sought ancillary remedies
including Protective Custody Orders, Appointment of Commissioner, Inspection and Access
Orders and other legal and equitable remedies under Article VIII, Section 5(5) of the 1987
Constitution and Rule 135, Section 6 of the Rules of Court. When the Amparo Rule came into
effect on October 24, 2007, they moved to have their petition treated as an amparo petition as it
would be more effective and suitable to the circumstances of the Manalo brothers’ enforced
disappearance. The Court granted their motion.

With this backdrop, we now come to the arguments of the petitioner. Petitioners’ first
argument in disputing the Decision of the Court of Appeals states, viz:
The Court of Appeals seriously and grievously erred in believing and giving
full faith and credit to the incredible uncorroborated, contradicted, and obviously
scripted, rehearsed and self-serving affidavit/testimony of herein respondent
Raymond Manalo.[94]

In delving into the veracity of the evidence, we need to mine and refine the ore of
petitioners’ cause of action, to determine whether the evidence presented is metal-strong to
satisfy the degree of proof required.

Section 1 of the Rule on the Writ of Amparo provides for the following causes of
action, viz:

Section 1. Petition. – The petition for a writ of amparo is a remedy available


to any person whose right to life, liberty and security is violated or threatened with
violation by an unlawful act or omission of a public official or employee, or of a
private individual or entity.

23
The writ shall cover extralegal killings and enforced disappearances or
threats thereof. (emphasis supplied)

Sections 17 and 18, on the other hand, provide for the degree of proof required, viz:
Sec. 17. Burden of Proof and Standard of Diligence Required. – The parties
shall establish their claims by substantial evidence.

xxx xxx xxx

Sec. 18. Judgment. – … If the allegations in the petition are proven by


substantial evidence, the court shall grant the privilege of the writ and such reliefs
as may be proper and appropriate; otherwise, the privilege shall be denied.
(emphases supplied)

Substantial evidence has been defined as such relevant evidence as a reasonable mind might
accept as adequate to support a conclusion.[95]

After careful perusal of the evidence presented, we affirm the findings of the Court of
Appeals that respondents were abducted from their houses in Sito Muzon, Brgy. Buhol na
Mangga, San Ildefonso, Bulacan on February 14, 2006 and were continuously detained until
they escaped on August 13, 2007. The abduction, detention, torture, and escape of the
respondents were narrated by respondent Raymond Manalo in a clear and convincing
manner. His account is dotted with countless candid details of respondents’ harrowing
experience and tenacious will to escape, captured through his different senses and etched in his
memory. A few examples are the following: “Sumilip ako sa isang haligi ng kamalig at nakita
kong sinisilaban si Manuel.”[96] “(N)ilakasan ng mga sundalo ang tunog na galing sa istiryo ng
sasakyan. Di nagtagal, narinig ko ang hiyaw o ungol ni Manuel.”[97] “May naiwang mga bakas
ng dugo habang hinihila nila ang mga bangkay. Naamoy ko iyon nang nililinis ang
bakas.”[98] “Tumigil ako sa may palaisdaan kung saan ginamit ko ang bato para tanggalin ang
mga kadena.”[99] “Tinanong ko sa isang kapit-bahay kung paano ako makakakuha ng cell phone;
sabi ko gusto kong i-text ang isang babae na nakatira sa malapit na lugar.”[100]

We affirm the factual findings of the appellate court, largely based on respondent
Raymond Manalo’s affidavit and testimony, viz:
…the abduction was perpetrated by armed men who were sufficiently identified by
the petitioners (herein respondents) to be military personnel and CAFGU
auxiliaries. Raymond recalled that the six armed men who barged into his house
through the rear door were military men based on their attire of fatigue pants and
army boots, and the CAFGU auxiliaries, namely: Michael de la Cruz, Madning de la
Cruz, Puti de la Cruz and Pula de la Cruz, all members of the CAFGU and residents of

24
Muzon, San Ildefonso, Bulacan, and the brothers Randy Mendoza and Rudy
Mendoza, also CAFGU members, served as lookouts during the abduction. Raymond
was sure that three of the six military men were Ganata, who headed the abducting
team, Hilario, who drove the van, and George. Subsequent incidents of their long
captivity, as narrated by the petitioners, validated their assertion of the participation
of the elements of the 7th Infantry Division, Philippine Army, and their CAFGU
auxiliaries.

We are convinced, too, that the reason for the abduction was the suspicion
that the petitioners were either members or sympathizers of the NPA, considering
that the abductors were looking for Ka Bestre, who turned out to be Rolando, the
brother of petitioners.

The efforts exerted by the Military Command to look into the abduction
were, at best, merely superficial. The investigation of the Provost Marshall of the
7th Infantry Division focused on the one-sided version of the CAFGU auxiliaries
involved. This one-sidedness might be due to the fact that the Provost Marshall
could delve only into the participation of military personnel, but even then the
Provost Marshall should have refrained from outrightly exculpating the CAFGU
auxiliaries he perfunctorily investigated…

Gen. Palparan’s participation in the abduction was also established. At the


very least, he was aware of the petitioners’ captivity at the hands of men in uniform
assigned to his command. In fact, he or any other officer tendered no controversion
to the firm claim of Raymond that he (Gen. Palparan) met them in person in a
safehouse in Bulacan and told them what he wanted them and their parents to do or
not to be doing. Gen. Palparan’s direct and personal role in the abduction might not
have been shown but his knowledge of the dire situation of the petitioners during
their long captivity at the hands of military personnel under his command bespoke of
his indubitable command policy that unavoidably encouraged and not merely
tolerated the abduction of civilians without due process of law and without probable
cause.

In the habeas proceedings, the Court, through the Former Special Sixth
Division (Justices Buzon, chairman; Santiago-Lagman, Sr., member; and Romilla-
Lontok, Jr., member/ponente.) found no clear and convincing evidence to establish
that M/Sgt. Rizal Hilario had anything to do with the abduction or the
detention. Hilario’s involvement could not, indeed, be then established after
Evangeline Francisco, who allegedly saw Hilario drive the van in which the
petitioners were boarded and ferried following the abduction, did not testify. (See
the decision of the habeas proceedings at rollo, p. 52)

However, in this case, Raymond attested that Hilario drove the white L-300
van in which the petitioners were brought away from their houses on February 14,
2006. Raymond also attested that Hilario participated in subsequent incidents
during the captivity of the petitioners, one of which was when Hilario fetched them
from Fort Magsaysay on board a Revo and conveyed them to a detachment in
Pinaud, San Ildefonso, Bulacan where they were detained for at least a week in a
house of strong materials (Exhibit D, rollo, p. 205) and then Hilario (along with
Efren) brought them to Sapang, San Miguel, Bulacan on board the Revo, to an
25
unfinished house inside the compound of Kapitan where they were kept for more or
less three months. (Exhibit D, rollo, p. 205) It was there where the petitioners came
face to face with Gen. Palparan. Hilario and Efren also brought the petitioners one
early morning to the house of the petitioners’ parents, where only Raymond was
presented to the parents to relay the message from Gen. Palparan not to join
anymore rallies. On that occasion, Hilario warned the parents that they would not
again see their sons should they join any rallies to denounce human rights
violations. (Exhibit D, rollo, pp. 205-206) Hilario was also among four Master
Sergeants (the others being Arman, Ganata and Cabalse) with whom Gen. Palparan
conversed on the occasion when Gen. Palparan required Raymond to take the
medicines for his health. (Exhibit D, rollo, p. 206) There were other occasions when
the petitioners saw that Hilario had a direct hand in their torture.

It is clear, therefore, that the participation of Hilario in the abduction and


forced disappearance of the petitioners was established. The participation of other
military personnel like Arman, Ganata, Cabalse and Caigas, among others, was
similarly established.

xxx xxx xxx

As to the CAFGU auxiliaries, the habeas Court found them personally


involved in the abduction. We also do, for, indeed, the evidence of their
participation is overwhelming.[101]
We reject the claim of petitioners that respondent Raymond Manalo’s statements were
not corroborated by other independent and credible pieces of evidence.[102] Raymond’s affidavit
and testimony were corroborated by the affidavit of respondent Reynaldo Manalo. The
testimony and medical reports prepared by forensic specialist Dr. Molino, and the pictures of the
scars left by the physical injuries inflicted on respondents,[103] also corroborate respondents’
accounts of the torture they endured while in detention. Respondent Raymond Manalo’s
familiarity with the facilities in Fort Magsaysay such as the “DTU,” as shown in his testimony and
confirmed by Lt. Col. Jimenez to be the “Division Training Unit,”[104] firms up respondents’ story
that they were detained for some time in said military facility.

In Ortiz v. Guatemala,[105] a case decided by the Inter-American Commission on Human


Rights, the Commission considered similar evidence, among others, in finding that complainant
Sister Diana Ortiz was abducted and tortured by agents of the Guatemalan government. In this
case, Sister Ortiz was kidnapped and tortured in early November 1989. The Commission’s
findings of fact were mostly based on the consistent and credible statements, written and oral,
made by Sister Ortiz regarding her ordeal.[106] These statements were supported by her
recognition of portions of the route they took when she was being driven out of the military
installation where she was detained.[107] She was also examined by a medical doctor whose
findings showed that the 111 circular second degree burns on her back and abrasions on her

26
cheek coincided with her account of cigarette burning and torture she suffered while in
detention.[108]

With the secret nature of an enforced disappearance and the torture perpetrated on the
victim during detention, it logically holds that much of the information and evidence of the
ordeal will come from the victims themselves, and the veracity of their account will depend on
their credibility and candidness in their written and/or oral statements. Their statements can be
corroborated by other evidence such as physical evidence left by the torture they suffered or
landmarks they can identify in the places where they were detained. Where powerful military
officers are implicated, the hesitation of witnesses to surface and testify against them comes as
no surprise.

We now come to the right of the respondents to the privilege of the writ
of amparo. There is no quarrel that the enforced disappearance of both respondents Raymond
and Reynaldo Manalo has now passed as they have escaped from captivity and surfaced. But
while respondents admit that they are no longer in detention and are physically free, they assert
that they are not “free in every sense of the word”[109] as their “movements continue to be
restricted for fear that people they have named in their Judicial Affidavits and testified against
(in the case of Raymond) are still at large and have not been held accountable in any
way. These people are directly connected to the Armed Forces of the Philippines and are, thus,
in a position to threaten respondents’ rights to life, liberty and security.”[110] (emphasis supplied)
Respondents claim that they are under threat of being once again abducted, kept captive or
even killed, which constitute a direct violation of their right to security of person.[111]

Elaborating on the “right to security, in general,” respondents point out that this right is
“often associated with liberty;” it is also seen as an “expansion of rights based on the prohibition
against torture and cruel and unusual punishment.” Conceding that there is no right to security
expressly mentioned in Article III of the 1987 Constitution, they submit that their rights “to be
kept free from torture and from incommunicado detention and solitary detention places[112] fall
under the general coverage of the right to security of person under the writ of Amparo.” They
submit that the Court ought to give an expansive recognition of the right to security of person in
view of the State Policy under Article II of the 1987 Constitution which enunciates that, “The
State values the dignity of every human person and guarantees full respect for human
rights.” Finally, to justify a liberal interpretation of the right to security of person, respondents
cite the teaching in Moncupa v. Enrile[113] that “the right to liberty may be made more
meaningful only if there is no undue restraint by the State on the exercise of that

27
liberty”[114] such as a requirement to “report under unreasonable restrictions that amounted to a
deprivation of liberty”[115] or being put under “monitoring and surveillance.”[116]

In sum, respondents assert that their cause of action consists in the threat to their right
to life and liberty, and a violation of their right to security.

Let us put this right to security under the lens to determine if it has indeed been violated
as respondents assert. The right to security or the right to security of person finds a textual
hook in Article III, Section 2 of the 1987 Constitution which provides, viz:
Sec. 2. The right of the people to be secure in their persons, houses, papers
and effects against unreasonable searches and seizures of whatever nature and for
any purpose shall beinviolable, and no search warrant or warrant of arrest shall
issue except upon probable cause to be determined personally by the judge…

At the core of this guarantee is the immunity of one’s person, including the extensions of
his/her person – houses, papers, and effects – against government intrusion. Section 2 not only
limits the state’s power over a person’s home and possessions, but more importantly, protects
the privacy and sanctity of the person himself.[117] The purpose of this provision was enunciated
by the Court in People v. CFI of Rizal, Branch IX, Quezon City, viz: [118]

The purpose of the constitutional guarantee against unreasonable searches and


seizures is to prevent violations of private security in person and property and
unlawful invasion of the security of the home by officers of the law acting under
legislative or judicial sanction and to give remedy against such usurpation when
attempted. (Adams v. New York, 192 U.S. 858; Alvero v. Dizon, 76 Phil. 637
[1946]). The right to privacy is an essential condition to the dignity and happiness
and to the peace and security of every individual, whether it be of home or of
persons and correspondence. (Tañada and Carreon, Political Law of the Philippines,
Vol. 2, 139 [1962]). The constitutional inviolability of this great fundamental right
against unreasonable searches and seizures must be deemed absolute as nothing is
closer to a man’s soul than the serenity of his privacy and the assurance of his
personal security. Any interference allowable can only be for the best causes and
reasons.[119] (emphases supplied)

While the right to life under Article III, Section 1[120] guarantees essentially the right to be
alive[121] - upon which the enjoyment of all other rights is preconditioned - the right to security
of person is a guarantee of the secure quality of this life, viz: “The life to which each person has
a right is not a life lived in fear that his person and property may be unreasonably violated by a
powerful ruler. Rather, it is a life lived with the assurance that the government he established
and consented to, will protect the security of his person and property. The ideal of security in life
and property… pervades the whole history of man. It touches every aspect of man’s

28
existence.”[122] In a broad sense, the right to security of person “emanates in a person’s legal
and uninterrupted enjoyment of his life, his limbs, his body, his health, and his reputation. It
includes the right to exist, and the right to enjoyment of life while existing, and it is invaded not
only by a deprivation of life but also of those things which are necessary to the enjoyment of life
according to the nature, temperament, and lawful desires of the individual.”[123]

A closer look at the right to security of person would yield various permutations of the
exercise of this right.

First, the right to security of person is “freedom from fear.” In its “whereas” clauses,
the Universal Declaration of Human Rights (UDHR) enunciates that “a world in which human
beings shall enjoy freedom of speech and belief and freedom from fear and want has been
proclaimed as the highest aspiration of the common people.” (emphasis supplied) Some scholars
postulate that “freedom from fear” is not only an aspirational principle, but essentially an
individual international human right.[124] It is the “right to security of person” as the word
“security” itself means “freedom from fear.”[125] Article 3 of the UDHR provides, viz:
Everyone has the right to life, liberty and security of person.[126] (emphasis supplied)

In furtherance of this right declared in the UDHR, Article 9(1) of the International
Covenant on Civil and Political Rights (ICCPR) also provides for the right to security of
person, viz:

1. Everyone has the right to liberty and security of person. No one shall be
subjected to arbitrary arrest or detention. No one shall be deprived of his liberty
except on such grounds and in accordance with such procedure as are established
by law. (emphasis supplied)

The Philippines is a signatory to both the UDHR and the ICCPR.

In the context of Section 1 of the Amparo Rule, “freedom from fear” is the right and
any threat to the rights to life, liberty or security is the actionable wrong. Fear is a state of mind,
a reaction; threat is a stimulus, a cause of action. Fear caused by the same stimulus can range
from being baseless to well-founded as people react differently. The degree of fear can vary
from one person to another with the variation of the prolificacy of their imagination, strength of
character or past experience with the stimulus. Thus, in the amparo context, it is more correct to
say that the “right to security” is actually the “freedom from threat.” Viewed in this light, the
“threatened with violation” Clause in the latter part of Section 1 of the Amparo Rule is a form of
violation of the right to security mentioned in the earlier part of the provision.[127]

Second, the right to security of person is a guarantee of bodily and psychological integrity
or security. Article III, Section II of the 1987 Constitution guarantees that, as a general rule,
29
one’s body cannot be searched or invaded without a search warrant.[128] Physical injuries
inflicted in the context of extralegal killings and enforced disappearances constitute more than a
search or invasion of the body. It may constitute dismemberment, physical disabilities, and
painful physical intrusion. As the degree of physical injury increases, the danger to life itself
escalates. Notably, in criminal law, physical injuries constitute a crime against persons because
they are an affront to the bodily integrity or security of a person.[129]

Physical torture, force, and violence are a severe invasion of bodily integrity. When
employed to vitiate the free will such as to force the victim to admit, reveal or fabricate
incriminating information, it constitutes an invasion of both bodily and psychological integrity as
the dignity of the human person includes the exercise of free will. Article III, Section 12 of the
1987 Constitution more specifically proscribes bodily and psychological invasion, viz:

(2) No torture, force, violence, threat or intimidation, or any other means


which vitiate the free will shall be used against him (any person under investigation
for the commission of an offense). Secret detention places,
solitary, incommunicado or other similar forms of detention are prohibited.

Parenthetically, under this provision, threat and intimidation that vitiate the free will - although
not involving invasion of bodily integrity - nevertheless constitute a violation of the right to
security in the sense of “freedom from threat” as afore-discussed.

Article III, Section 12 guarantees freedom from dehumanizing abuses of persons under
investigation for the commission of an offense. Victims of enforced disappearances who are not
even under such investigation should all the more be protected from these degradations.

An overture to an interpretation of the right to security of person as a right against


torture was made by the European Court of Human Rights (ECHR) in the recent case ofPopov
v. Russia.[130] In this case, the claimant, who was lawfully detained, alleged that the state
authorities had physically abused him in prison, thereby violating his right to security of
person. Article 5(1) of the European Convention on Human Rights provides, viz: “Everyone has
the right to liberty and security of person. No one shall be deprived of his liberty save in the
following cases and in accordance with a procedure prescribed by law ...” (emphases
supplied) Article 3, on the other hand, provides that “(n)o one shall be subjected to torture or
to inhuman or degrading treatment or punishment.” Although the application failed on the facts
as the alleged ill-treatment was found baseless, the ECHR relied heavily on the concept of
security in holding, viz:

...the applicant did not bring his allegations to the attention of domestic
authorities at the time when they could reasonably have been expected to take
30
measures in order to ensure his security and to investigate the circumstances in
question.

xxx xxx xxx

... the authorities failed to ensure his security in custody or to comply with the
procedural obligation under Art.3 to conduct an effective investigation into his
allegations.[131] (emphasis supplied)

The U.N. Committee on the Elimination of Discrimination against Women has also made a
statement that the protection of the bodily integrity of women may also be related to the right
to security and liberty, viz:

…gender-based violence which impairs or nullifies the enjoyment by women of


human rights and fundamental freedoms under general international law or under
specific human rights conventions is discrimination within the meaning of article 1 of
the Convention (on the Elimination of All Forms of Discrimination Against Women).
These rights and freedoms include . . . the right to liberty and security of person.[132]

Third, the right to security of person is a guarantee of protection of one’s rights by the
government. In the context of the writ of amparo, this right is built into the guarantees of the
right to life and liberty under Article III, Section 1 of the 1987 Constitution and the right to
security of person (as freedom from threat and guarantee of bodily and psychological integrity)
under Article III, Section 2. The right to security of person in this third sense is a corollary of
the policy that the State “guarantees full respect for human rights” under Article II, Section 11
of the 1987 Constitution.[133] As the government is the chief guarantor of order and security,
the Constitutional guarantee of the rights to life, liberty and security of person is rendered
ineffective if government does not afford protection to these rights especially when they are
under threat. Protection includes conducting effective investigations, organization of the
government apparatus to extend protection to victims of extralegal killings or enforced
disappearances (or threats thereof) and/or their families, and bringing offenders to the bar of
justice. The Inter-American Court of Human Rights stressed the importance of investigation in
the Velasquez Rodriguez Case,[134] viz:

(The duty to investigate) must be undertaken in a serious manner and not as


a mere formality preordained to be ineffective. An investigation must have an
objective and be assumed by the State as its own legal duty, not as a step taken by
private interests that depends upon the initiative of the victim or his family or upon
their offer of proof, without an effective search for the truth by the government.[135]

This third sense of the right to security of person as a guarantee of government protection
has been interpreted by the United Nations’ Human Rights Committee[136] in not a few cases

31
involving Article 9[137] of the ICCPR. While the right to security of person appears in conjunction
with the right to liberty under Article 9, the Committee has ruled that the right to security of
person can exist independently of the right to liberty. In other words, there need not necessarily
be a deprivation of liberty for the right to security of person to be invoked. In Delgado Paez v.
Colombia,[138] a case involving death threats to a religion teacher at a secondary school
in Leticia, Colombia, whose social views differed from those of the Apostolic Prefect of Leticia,
the Committee held, viz:

The first sentence of article 9 does not stand as a separate paragraph. Its
location as a part of paragraph one could lead to the view that the right to security
arises only in the context of arrest and detention. The travaux
préparatoires indicate that the discussions of the first sentence did indeed focus on
matters dealt with in the other provisions of article 9. The Universal Declaration of
Human Rights, in article 3, refers to the right to life, the right to liberty and the
right to security of the person. These elements have been dealt with in separate
clauses in the Covenant. Although in the Covenant the only reference to the right of
security of person is to be found in article 9, there is no evidence that it was
intended to narrow the concept of the right to security only to situations of formal
deprivation of liberty. At the same time, States parties have undertaken to
guarantee the rights enshrined in the Covenant. It cannot be the case that, as a
matter of law, States can ignore known threats to the life of persons under their
jurisdiction, just because that he or she is not arrested or otherwise detained.
States parties are under an obligation to take reasonable and appropriate measures
to protect them. An interpretation of article 9 which would allow a State party to
ignore threats to the personal security of non-detained persons within its
jurisdiction would render totally ineffective the guarantees of the
Covenant.[139] (emphasis supplied)

The Paez ruling was reiterated in Bwalya v. Zambia,[140] which involved a political activist
and prisoner of conscience who continued to be intimidated, harassed, and restricted in his
movements following his release from detention. In a catena of cases, the ruling of the
Committee was of a similar import: Bahamonde v. Equatorial Guinea,[141] involving
discrimination, intimidation and persecution of opponents of the ruling party in that
state; Tshishimbi v. Zaire,[142] involving the abduction of the complainant’s husband who was a
supporter of democratic reform in Zaire; Dias v. Angola,[143] involving the murder of
the complainant’s partner and the harassment he (complainant) suffered because of his
investigation of the murder; and Chongwe v. Zambia,[144] involving an assassination attempt on
the chairman of an opposition alliance.

Similarly, the European Court of Human Rights (ECHR) has interpreted the “right to
security” not only as prohibiting the State from arbitrarily depriving liberty, but imposing a
positive duty on the State to afford protection of the right to liberty.[145] The ECHR interpreted
the “right to security of person” under Article 5(1) of the European Convention of Human Rights
in the leading case on disappearance of persons, Kurt v. Turkey.[146] In this case, the claimant’s
32
son had been arrested by state authorities and had not been seen since. The family’s requests
for information and investigation regarding his whereabouts proved futile. The claimant
suggested that this was a violation of her son’s right to security of person. The ECHR ruled, viz:
... any deprivation of liberty must not only have been effected in conformity
with the substantive and procedural rules of national law but must equally be in
keeping with the very purpose of Article 5, namely to protect the individual from
arbitrariness... Having assumed control over that individual it is incumbent on the
authorities to account for his or her whereabouts. For this reason, Article 5 must
be seen as requiring the authorities to take effective measures to safeguard against
the risk of disappearance and to conduct a prompt effective investigation into an
arguable claim that a person has been taken into custody and has not been seen
since.[147] (emphasis supplied)

Applying the foregoing concept of the right to security of person to the case at bar, we
now determine whether there is a continuing violation of respondents’ right to security.

First, the violation of the right to security as freedom from threat to respondents’ life,
liberty and security.

While respondents were detained, they were threatened that if they escaped, their
families, including them, would be killed. In Raymond’s narration, he was tortured and poured
with gasoline after he was caught the first time he attempted to escape from Fort Magsaysay. A
call from a certain “Mam,” who wanted to see him before he was killed, spared him.

This time, respondents have finally escaped. The condition of the threat to be killed has
come to pass. It should be stressed that they are now free from captivity not because they
were released by virtue of a lawful order or voluntarily freed by their abductors. It ought to be
recalled that towards the end of their ordeal, sometime in June 2007 when respondents were
detained in a camp in Limay, Bataan, respondents’ captors even told them that they were still
deciding whether they should be executed. Respondent Raymond Manalo attested in his
affidavit, viz:
Kinaumagahan, naka-kadena pa kami. Tinanggal ang mga kadena mga 3 o 4 na
araw pagkalipas. Sinabi sa amin na kaya kami nakakadena ay dahil
pinagdedesisyunan pa ng mga sundalo kung papatayin kami o hindi.[148]

The possibility of respondents being executed stared them in the eye while they were in
detention. With their escape, this continuing threat to their life is apparent, moreso now that
they have surfaced and implicated specific officers in the military not only in their own abduction
and torture, but also in those of other persons known to have disappeared such as Sherlyn
Cadapan, Karen Empeño, and Manuel Merino, among others.
33
Understandably, since their escape, respondents have been under concealment and
protection by private citizens because of the threat to their life, liberty and security. The threat
vitiates their free will as they are forced to limit their movements or activities.[149] Precisely
because respondents are being shielded from the perpetrators of their abduction, they cannot be
expected to show evidence of overt acts of threat such as face-to-face intimidation or written
threats to their life, liberty and security. Nonetheless, the circumstances of respondents’
abduction, detention, torture and escape reasonably support a conclusion that there is an
apparent threat that they will again be abducted, tortured, and this time, even executed. These
constitute threats to their liberty, security, and life, actionable through a petition for a writ
of amparo.

Next, the violation of the right to security as protection by the government. Apart from
the failure of military elements to provide protection to respondents by themselves perpetrating
the abduction, detention, and torture, they also miserably failed in conducting an effective
investigation of respondents’ abduction as revealed by the testimony and investigation report of
petitioners’ own witness, Lt. Col. Ruben Jimenez, Provost Marshall of the 7th Infantry Division.

The one-day investigation conducted by Jimenez was very limited, superficial, and one-
sided. He merely relied on the Sworn Statements of the six implicated members of the CAFGU
and civilians whom he met in the investigation for the first time. He was present at the
investigation when his subordinate Lingad was taking the sworn statements, but he did not
propound a single question to ascertain the veracity of their statements or their credibility. He
did not call for other witnesses to test the alibis given by the six implicated persons nor for the
family or neighbors of the respondents.

In his affidavit, petitioner Secretary of National Defense attested that in a Memorandum


Directive dated October 31, 2007, he issued a policy directive addressed to the AFP Chief of
Staff, that the AFP should adopt rules of action in the event the writ of amparo is issued by a
competent court against any members of the AFP, which should essentially include verification
of the identity of the aggrieved party; recovery and preservation of relevant evidence;
identification of witnesses and securing statements from them; determination of the cause,
manner, location and time of death or disappearance; identification and apprehension of the
person or persons involved in the death or disappearance; and bringing of the suspected
offenders before a competent court.[150] Petitioner AFP Chief of Staff also submitted his own
affidavit attesting that he received the above directive of respondent Secretary of National
Defense and that acting on this directive, he immediately caused to be issued a directive to the
units of the AFP for the purpose of establishing the circumstances of the alleged disappearance

34
and the recent reappearance of the respondents, and undertook to provide results of the
investigations to respondents.[151] To this day, however, almost a year after the policy directive
was issued by petitioner Secretary of National Defense on October 31, 2007, respondents have
not been furnished the results of the investigation which they now seek through the instant
petition for a writ of amparo.

Under these circumstances, there is substantial evidence to warrant the conclusion that
there is a violation of respondents’ right to security as a guarantee of protection by the
government.

In sum, we conclude that respondents’ right to security as “freedom from threat” is


violated by the apparent threat to their life, liberty and security of person. Their right to
security as a guarantee of protection by the government is likewise violated by the ineffective
investigation and protection on the part of the military.

Finally, we come to the reliefs granted by the Court of Appeals, which petitioners
question.

First, that petitioners furnish respondents all official and unofficial reports of the
investigation undertaken in connection with their case, except those already in file with the
court.

Second, that petitioners confirm in writing the present places of official assignment of
M/Sgt. Hilario aka Rollie Castillo and Donald Caigas.

Third, that petitioners cause to be produced to the Court of Appeals all medical reports,
records and charts, and reports of any treatment given or recommended and medicines
prescribed, if any, to the Manalo brothers, to include a list of medical personnel (military and
civilian) who attended to them from February 14, 2006 until August 12, 2007.

With respect to the first and second reliefs, petitioners argue that the production order
sought by respondents partakes of the characteristics of a search warrant. Thus, they claim that
the requisites for the issuance of a search warrant must be complied with prior to the grant of
the production order, namely: (1) the application must be under oath or affirmation; (2) the
search warrant must particularly describe the place to be searched and the things to be seized;
(3) there exists probable cause with one specific offense; and (4) the probable cause must be
personally determined by the judge after examination under oath or affirmation of the
complainant and the witnesses he may produce.[152] In the case at bar, however, petitioners
point out that other than the bare, self-serving and vague allegations made by respondent
35
Raymond Manalo in his unverified declaration and affidavit, the documents respondents seek to
be produced are only mentioned generally by name, with no other supporting details. They also
argue that the relevancy of the documents to be produced must be apparent, but this is not true
in the present case as the involvement of petitioners in the abduction has not been shown.

Petitioners’ arguments do not hold water. The production order under the Amparo Rule
should not be confused with a search warrant for law enforcement under Article III, Section 2 of
the 1987 Constitution. This Constitutional provision is a protection of the people from the
unreasonable intrusion of the government, not a protection of the government from the demand
of the people such as respondents.

Instead, the amparo production order may be likened to the production of documents or
things under Section 1, Rule 27 of the Rules of Civil Procedure which provides in relevant
part, viz:

Section 1. Motion for production or inspection order.

Upon motion of any party showing good cause therefor, the court in which an
action is pending may (a) order any party to produce and permit the inspection and
copying or photographing, by or on behalf of the moving party, of any designated
documents, papers, books of accounts, letters, photographs, objects or tangible
things, not privileged, which constitute or contain evidence material to any matter
involved in the action and which are in his possession, custody or control…

In Material Distributors (Phil.) Inc. v. Judge Natividad,[153] the respondent judge, under
authority of Rule 27, issued a subpoena duces tecum for the production and inspection of among
others, the books and papers of Material Distributors (Phil.) Inc. The company questioned the
issuance of the subpoena on the ground that it violated the search and seizure clause. The
Court struck down the argument and held that the subpoena pertained to a civil procedure that
“cannot be identified or confused with unreasonable searches prohibited by the Constitution…”

Moreover, in his affidavit, petitioner AFP Chief of Staff himself undertook “to provide
results of the investigations conducted or to be conducted by the concerned unit relative to the
circumstances of the alleged disappearance of the persons in whose favor the Writ
of Amparo has been sought for as soon as the same has been furnished Higher headquarters.”

With respect to the second and third reliefs, petitioners assert that the disclosure of the
present places of assignment of M/Sgt. Hilario aka Rollie Castillo and Donald Caigas, as well as
the submission of a list of medical personnel, is irrelevant, improper, immaterial, and
unnecessary in the resolution of the petition for a writ of amparo. They add that it will
36
unnecessarily compromise and jeopardize the exercise of official functions and duties of military
officers and even unwittingly and unnecessarily expose them to threat of personal injury or even
death.

On the contrary, the disclosure of the present places of assignment of M/Sgt.


Hilario aka Rollie Castillo and Donald Caigas, whom respondents both directly implicated as
perpetrators behind their abduction and detention, is relevant in ensuring the safety of
respondents by avoiding their areas of territorial jurisdiction. Such disclosure would also help
ensure that these military officers can be served with notices and court processes in relation to
any investigation and action for violation of the respondents’ rights. The list of medical
personnel is also relevant in securing information to create the medical history of respondents
and make appropriate medical interventions, when applicable and necessary.

In blatant violation of our hard-won guarantees to life, liberty and security, these rights
are snuffed out from victims of extralegal killings and enforced disappearances. The writ
of amparo is a tool that gives voice to preys of silent guns and prisoners behind secret walls.

WHEREFORE, premises considered, the petition is DISMISSED. The Decision of the Court

of Appeals dated December 26, 2007 is affirmed.

37
SO ORDERED.

REYNATO S. PUNO
Chief Justice

WE CONCUR:

LEONARDO A. QUISUMBING
Associate Justice

CONSUELO YNARES-SANTIAGO ANTONIO T. CARPIO


Associate Justice Associate Justice

MA. ALICIA AUSTRIA-MARTINEZ RENATO C. CORONA


Associate Justice Associate Justice

CONCHITA CARPIO MORALES ADOLFO S. AZCUNA


Associate Justice Associate Justice

DANTE O. TINGA MINITA V. CHICO-NAZARIO


Associate Justice Associate Justice

38
PRESBITERO J. VELASCO, JR. ANTONIO EDUARDO B. NACHURA
Associate Justice Associate Justice

RUBEN T. REYES TERESITA J. LEONARDO-DE CASTRO


Associate Justice Associate Justice

ARTURO D. BRION
Associate Justice

CERTIFICATION

Pursuant to Section 13, Article VIII of the Constitution, I certify that the conclusions in
the above decision had been reached in consultation before the case was assigned to the writer
of the opinion of the Court.

REYNATO S. PUNO
Chief Justice

[1]
Sec. 19 of the Rule on the Writ of Amparo provides for appeal, viz:
Sec. 19. Appeal – Any party may appeal from the final judgment or order
to the Supreme Court under Rule 45. The appeal may raise questions of fact or law
or both.
The period of appeal shall be five (5) working days from the date of notice of
the adverse judgment.
The appeal shall be given the same priority as in habeas corpus cases.
[2]
G.R. No. 179095 filed on August 23, 2007.

39
[3]
1987 PHIL. CONST. Art. VIII, § 5(5) provides for the rule-making power of the Supreme
Court, viz:
Sec. 5. The Supreme Court shall have the following powers:
(5) Promulgate rules concerning the protection and enforcement of
constitutional rights…
[4]
1987 PHIL. CONST. Art. III, § 1 provides in relevant part, viz:
Sec. 1. No person shall be deprived of life, liberty…without due
process of law…
[5]
CA rollo, pp. 26-27.
[6]
Section 26 of the Rule on the Writ of Amparo provides, viz:
Sec. 26. Applicability to Pending Cases. – This Rule shall govern cases
involving extralegal killings and enforced disappearances or threats thereof pending
in the trial and appellate courts.
[7]
Section 18 of the Rule on the Writ of Amparo provides, viz:
Sec. 18. Judgment. – The court shall render judgment within ten (10) days
from the time the petition is submitted for decision. If the allegations in the
petition are proven by substantial evidence, the court shall grant the privilege of
the writ and such reliefs as may be proper and appropriate; otherwise, the privilege
shall be denied.
[8]
CA rollo, pp. 86-87.
[9]
Id. at 1-6.
[10]
Id. at 82-83.
[11]
Exhibit D (Sinumpaang Salaysay para sa Hukuman ni Raymond Manalo), CA rollo, pp. 200-
201; TSN, November 13, 2007, p. 47.
[12]
Exhibit D, CA rollo, pp. 200-201.
[13]
Id. at 201-202.
[14]
Id.
[15]
Id. at 202.
[16]
A Petition for Habeas Corpus was filed on May 12, 2006 in the Court of Appeals by the
relatives of herein respondents. (CA-G.R. SP. No. 94431). The petition alleged that military
personnel and CAFGU auxiliaries forcibly took petitioners from their homes in Bulacan on
February 14, 2006.
Impleaded as respondents were Lt. Gen. Hermogenes C. Esperon, then the
Commanding General of the Philippine Army; Maj. Gen. Jovito Palparan, then the Commanding
Officer, 7th Infantry Division, stationed in Luzon; M/Sgt. Rizal Hilario alias Rollie Castillo; and
civilians Michael dela Cruz, Madning dela Cruz, Puti dela Cruz, Pula dela Cruz, Randy Mendoza
and Rudy Mendoza, all CAFGU members.
Respondents denied any involvement in the petitioners’ abduction and disappearance.
After hearing, the Court of Appeals rendered a decision on June 27, 2007, viz:
WHEREFORE, in view of the foregoing, this Court holds that respondents
Madning de la Cruz, Puti de la Cruz, Pula de la Cruz, Rudy Mendoza and CAFGU
members Michael de la Cruz and Randy Mendoza are illegally detaining Raymond
and Reynaldo Manalo, and are hereby ordered to RELEASE said
victims Raymond Manalo and Reynaldo Manalo within ten (10) days from
receipt hereof; otherwise, they will be held in contempt of court. This is without
prejudice to any penalty that may be imposed should they be found later by any
other court of justice to be criminally, administratively, or civilly liable for any other
act/s against the persons of aforenamed victims. (CA rollo, pp. 60-61)

40
On July 18, 2007, the relatives of the petitioners appealed the decision to the
Supreme Court. (G.R. No. 178614). Respondents filed a motion for reconsideration in the Court
of Appeals.
On August 13, 2007, the petitioners escaped from captivity. Consequently, they filed
motions to withdraw the petition for habeas corpus in the CA and this Court as it had become
moot and academic. (CA rollo, p. 101; rollo, pp. 54-55)
[17]
Exhibit D, CA rollo, pp. 200-201.
[18]
Id. at 203.
[19]
TSN, November 13, 2007, p. 29.
[20]
Exhibit D, CA rollo, p. 203.
[21]
Id.
[22]
“Daniel Mendiola; Oscar Leuterio; mag-asawang Teresa at Vergel; isang nagngangalang Mang
Ipo at Ferdinand mula sa Nueva Ecija; isang taga-Bicol na ikinulong doon ng isa o dalawang
araw lamang (siya’y inilabas at hindi ko na nakitang muli); isang taga-Visayas (na ikinulong
doon ng isa o dalawang araw; siya’y inilabas at hindi ko na siya nakita); mga nagngangalang
Abel, Jojo at isa pa mula sa Nueva Ecija (na tumagal doon ng isang araw at isang gabi,
pagkatapos ay inilabas din); isang nagngangalang Bernard mula sa Hagonoy, Bulacan; ang
apelyido ni Bernard ay tila Majas ngunit hindi ako sigurado sa apelyido niya. Nang dinala doon
si Bernard, inilabas sina Mang Ipo at Ferdinand; dalawang lalaking may edad na, taga-Pinaud at
dinukot sa poultry (tumagal lang sila ng mga isang araw at tapos inilabas at hindi ko na nakita
uli).” (CA rollo, pp. 203-204)
[23]
Exhibit D, CA rollo, pp. 203-204.
[24]
Id. at 204.
[25]
Id. at 204-205.
[26]
Id. at 205.
[27]
Id.; TSN, November 13, 2007, pp. 36-38.
[28]
Exhibit D, CA rollo, p. 205.
[29]
Id.
[30]
Id.
[31]
Id. at 206.
[32]
TSN, November 13, 2007, p. 44; Exhibit F shows eights pictures of highest ranking officers of
the AFP and PNP in their uniforms; Exhibit F-1 is the picture of Gen. Palparan identified by
respondent Raymond Manalo, CA rollo, p. 214.
[33]
Exhibit D, CA rollo, p. 206.
[34]
Id. at 207.
[35]
Id.
[36]
Id. at 207-208.
[37]
Id. at 208.
[38]
Id.
[39]
Id. at 209.
[40]
Id.
[41]
Id.
[42]
Id.
[43]
Id. at 210-211.
[44]
Id. at 211.
[45]
Id.
[46]
Exhibit C (Sinumpaang Salaysay ni Reynaldo Manalo para sa Hukuman), CA rollo, pp. 196-
197.

41
[47]
TSN, November 13, 2007, pp. 85-90; Exhibit G is the background of the case of Raymond
and Reynaldo Manalo, CA rollo, p. 216; Exhibits G-1 to G-2 are the report proper for Reynaldo
Manalo containing a narration of his ordeal and complaints, and Dr. Molino’s physical findings,
analysis and recommendations, CA rollo, pp. 217-218; Exhibit G-3 are the pictures taken of
Reynaldo Manalo’s scars, CA rollo, p. 219; Exhibits G-4 to G-5 are the report proper for
Raymond Manalo with similar contents as Reynaldo’s report, CA rollo, pp. 220-221; Exhibits G-
6 to G-7 are the pictures of Raymond Manalo’s scars, CA rollo, pp. 222-223.
[48]
CA rollo, pp. 112-113; rollo, pp. 94-95.
[49]
CA rollo, pp. 122 and 171; rollo, pp. 28-29.
[50]
CA rollo, pp. 124-125; 177-178; rollo, pp. 29-31.
[51]
CA rollo, pp. 191-192; rollo, 106-107.
[52]
Id. at 107.
[53]
TSN, November 14, 2007, p. 25.
[54]
Id. at 84.
[55]
Id. at 36.
[56]
Id. at 40.
[57]
Id. at 41.
[58]
Id. at 92.
[59]
Id. at 46.
[60]
Id. at 44.
[61]
Id. at 46.
[62]
Id. at 80.
[63]
Id. at 28.
[64]
Id. at 50.
[65]
Id. at 55-56.
[66]
Id. at 57-61.
[67]
Id. at 61-63.
[68]
Id. at 63.
[69]
Exhibit 3-C, CA rollo, pp. 238-240.
[70]
Rollo, pp. 35-36.
[71]
Rule on the Writ of Amparo: The Rationale for the Writ of Amparo, p. 43.
[72]
Id.
[73]
Rule on the Writ of Amparo: Annotation, p. 47.
[74]
Id. Article VIII, § 5(5) of the 1987 Constitution provides for this rule-making power, viz:
Sec. 5. The Supreme Court shall have the following powers:
(5) Promulgate rules concerning the protection and enforcement of
constitutional rights…
[75]
Rule on the Writ of Amparo: Annotation, p. 48. This is the manner the term is used in United
Nations instruments.
[76]
Rule on the Writ of Amparo: Annotation, p. 48. This is the definition used in the Declaration
on the Protection of All Persons from Enforced Disappearances.
[77]
Barker, R., “Constitutionalism in the Americas: A Bicentennial Perspective,”
49 University of Pittsburgh Law Review (Spring, 1988) 891, 906.
[78]
Id., citing Zamudio, F., “A Brief Introduction to the Mexican Writ of Amparo,”
9 California Western International Law Journal (1979) 306, 309.
[79]
“At the time it adopted Rejón’s amparo, Yucatan had separated itself from Mexico. After a
few months, the secession ended and the state resumed its place in the union.” (Barker,
R., supra at 906.)

42
[80]
Acta de Reformas, art. 25 (1847) (amending Constitution of 1824).
[81]
Acta de Reformas, art. 25 (1847) (amending Constitution of 1824); CONST. of 1857, arts.
101, 102 (Mex.); CONST. art. 107 (Mex.).
[82]
Barker, R., supra at 906-907. See also Provost, R. “Emergency Judicial Relief for Human
Rights Violations in Canada and Argentina,” University of Miami Inter-American Law
Review (Spring/Summer, 1992) 693, 701-702.
[83]
Rule on the Writ of Amparo: Annotation, p. 45. See Article 107 of the Constitution of Mexico;
Article 28(15) of the Constitution of Ecuador; Article 77 of the Constitution of Paraguay; Article
43 of the Constitution of Argentina; Article 49 of the Constitution of Venezuela; Article 48 (3)
of the Constitution of Costa Rica; and Article 19 of the Constitution of Bolivia.
[84]
Provost, R., supra at 698, citing Ramirez, F., “The International Expansion of the Mexican
Amparo,” 1 Inter-American Law Review (1959) 163, 166.
[85]
Rule on the Writ of Amparo: Annotation, p. 45; see also Zagaris, B., “The Amparo Process
in Mexico,” 6 Mexico Law Journal (Spring 1998) 61, 66 and Provost, R., supra at 708-709.
[86]
Rule on the Writ of Amparo: Annotation, p. 45.
[87]
Brewer-Carias,
A., “The Latin American Amparo Proceeding and the Writ of Amparo in
the Philippines,” Second Distinguished Lecture, Series of 2007, Supreme Court, Philippine
Judicial Academy in coordination with the Philippine Association of Law Schools, March 7,
2008.
[88]
See 1987 PHIL. CONST. Art. III, §§ 13 & 15; Art. VII, § 18; Art. VIII, § 5(1).
[89]
5 U.S. 137 (1803). See Gormley, K. “Judicial Review in the Americas: Comments on
the United States and Mexico,” 45 Duquesne Law Review (Spring, 2007) 393.
[90]
Rule on the Writ of Amparo: Annotation, p. 47.
[91]
Deliberations of the Committee on the Revision of the Rules of Court, August 10, 2007;
August 24, 2007; August 31, 2007; and September 20, 2008.
[92]
G.R. No. 179095.
[93]
CA rollo, p. 3.
[94]
Rollo, p. 35.
[95]
Ferancullo v. Ferancullo, Jr., A.C. No. 7214, November 30, 2006, 509 SCRA 1.
[96]
CA rollo, p. 210.
[97]
Id.
[98]
Id.
[99]
Id. at 203.
[100]
Id. at 211.
[101]
Rollo, pp. 74-76.
[102]
Id. at 40.
[103]
CA rollo, pp. 219, 222-224.
[104]
TSN, November 14, 2007, p. 66.
[105]
Case 10.526, Report No. 31/96, Inter-Am.C.H.R.,OEA/Ser.L/V/II.95 Doc. 7 rev. at 332
(1997).
[106]
Id. at par. 49.
[107]
Id.
[108]
Id. at par. 50.
[109]
Rollo, p. 182.
[110]
Id.
[111]
Id. at 183.
[112]
Respondents cite 1987 PHIL. CONST. Art. III, § 12(2) which provides, viz:

43
(2) No torture, force, violence threat, intimidation, or any other means
which vitiate the free will shall be used against him (any person under investigation
for the commission of an offense). Secret detention places, solitary,
incommunicado, or other similar forms of detention are prohibited.
[113]
225 Phil. 191 (1986).
[114]
Rollo, pp. 182-183.
[115]
Id. at 183.
[116]
Id.
[117]
Bernas, THE 1987 CONSTITUTION OF THE REPUBLIC OF THE PHILIPPINES: A COMMENTARY
(2003) 162.
[118]
No. L-41686, November 17, 1980, 101 SCRA 86.
[119]
Id. at 100-101.
[120]
1987 PHIL. CONST. Art. III, § 1 provides, viz:
Sec. 1. No person shall be deprived of life, liberty, or property without due process
of law…
[121]
But see Bernas, supra at 110. “The constitutional protection of the right to life is not just a
protection of the right to be alive or to the security of one’s limb against physical harm.”
[122]
Separate Opinion of Chief Justice Reynato S. Puno in Republic v. Sandiganbayan, 454 Phil.
504 (2003).
[123]
Sandifer, D. and L. Scheman, THE FOUNDATION OF FREEDOM (1966), pp. 44-45.
[124]
Schmidt, C., “An International Human Right to Keep and Bear Arms,” 15 William and Mary
Bill of Rights Journal (February, 2007) 983, 1004.
[125]
Id., citing Webster’s Seventh New Collegiate Dictionary 780 (1971).
[126]
The U.N. Declaration on the Protection of All Persons from Enforced Disappearance also
provides for the right to security under Article 2, viz:
2. Any act of enforced disappearance places the persons subjected thereto
outside the protection of the law and inflicts severe suffering on them and their
families. It constitutes a violation of the rules of international law
guaranteeing, inter alia, the right to recognition as a person before the law,
the right to liberty and security of the person and the right not to be subjected to
torture and other cruel, inhuman or degrading treatment or punishment. It also
violates or constitutes a grave threat to the right to life. (emphasis supplied)
Various international human rights conventions and declarations affirm the “right to
security of person”, including the American Convention on Human Rights; European
Convention on Human Rights; African Charter; Inter-American Convention on the Prevention,
Punishment and Eradication of Violence Against Women; American Declaration of the Rights
and Duties of Man, African Women’s Protocol, and the U.N. Declaration on the Elimination of
Violence against Women.
[127]
Section 1 of the Rule on the Writ of Amparo provides, viz:
Section 1. Petition. – The petition for a writ of amparo is a remedy
available to any person whose right to life, liberty and security is violated or
threatened with violation by an unlawful act or omission of a public official or
employee, or of a private individual or entity. (emphasis supplied)
[128]
People v. Aruta, 351 Phil. 868 (1998).
[129]
Book Two, Title Eight, Crimes against Persons, of the Revised Penal Code consists of two
chapters: Chapter One – Destruction of Life, and Chapter Two – Physical Injuries.
[130]
(App. No.26853/04), ECtHR Judgment of July 13, 2006.
[131]
Id. at pars.196-197.

44
[132]
General Recommendation No. 19 on Violence against Women of the Committee on the
Elimination of Discrimination Against Women. Adoption of the Report, U.N. Committee on the
Elimination of Discrimination against Women, 11th Sess., Agenda Item 7, at para. 8, U.N.
Doc. CEDAW/C/1992/L.1/Add.15 (1992); see also Lai, S. and Ralph, R., “Female Sexual
Autonomy and Human Rights,” 8 Harvard Human Rights Journal (Spring, 1995) 201, 207-
208.
[133]
1987 PHIL. CONST. Art. II, § 11, provides, viz:
Sec. 11. The State values the dignity of every human person and
guarantees full respect for human rights.
[134]
I/A Court H.R. Velásquez Rodríguez Case, Judgment of July 29, 1988, Series C No. 4.
[135]
Id. at par. 177.
[136]
Created under Article 28 of the ICCPR as the treaty-based body charged with the
authoritative interpretation of the ICCPR. See Russell-Brown, S., “Out of the Crooked Timber
of Humanity: The Conflict Between South Africa’s Truth and Reconciliation Commission and
International Human Rights Norms Regarding ‘Effective Remedies’,” 26 Hastings International
and Comparative Law Review (Winter 2003) 227.
[137]
The ICCPR provides in Article 9(1), viz:
“1. Everyone has the right to liberty and security of person. No one shall
be subjected to arbitrary arrest or detention. No one shall be deprived of his liberty
except on such grounds and in accordance with such procedure as are established
by law.” (emphasis supplied)
[138]
Communication No. 195/1985, U. N. Doc. CCPR/C/39/D/195/1985 (1990).
[139]
Id. at , par. 5.5.
[140]
Communication No. 314/1988, U.N. Doc. CCPR/C/48/D/314/1988 (1993).
[141]
Communication No. 468/1991, U.N. Doc. CCPR/C/49/D/468/1991 (1993).
[142]
Communication No. 542/1993, U.N. Doc. CCPR/C/53/D/542/1993 (1996).
[143]
Communication No. 711/1996, U.N. Doc. CCPR/C/68/D/711/1996 (2000).
[144]
Communication No. 821/1998, U.N. Doc. CCPR/C/70/D/821/1998 (2000).
[145]
Powell, R., “The Right to Security of Person in European Court of Human Rights
Jurisprudence,” 6 European Human Rights Law Review (2007) 649, 652-653.
[146]
Kurt v. Turkey (1999) 27 E.H.R.R. 373.
[147]
Id. at pars. 122 and 123.
[148]
CA rollo, p. 210.
[149]
Rollo, p. 182
[150]
Rollo, pp. 28-29.
[151]
Rollo, pp. 29-31. The directives issued by the petitioners are in line with Article 13 of the
1992 U.N. Declaration on Enforced Disappearances which states that, “any person having
knowledge or legitimate interest who alleges that a person has been subjected to enforced
disappearance has the right to complain to a competent and independent state authority and
to have that complaint promptly, thoroughly and impartially investigated by the authority.”
[152]
Rollo, pp. 44-45.
[153]
84 Phil. 127 (1949).

45
Republic of the Philippines

SUPREME COURT

Manila

EN BANC

SOCIAL JUSTICE SOCIETY (SJS), G.R. No. 157870

Petitioner,

- versus -

DANGEROUS DRUGS BOARD and

PHILIPPINE DRUG ENFORCEMENT

AGENCY (PDEA),

Respondents.

x-----------------------------------------------x

ATTY. MANUEL J. LASERNA, JR., G.R. No. 158633

Petitioner,

- versus -

DANGEROUS DRUGS BOARD and

PHILIPPINE DRUG ENFORCEMENT

AGENCY,

Respondents.

46
x-----------------------------------------------x

AQUILINO Q. PIMENTEL, JR., G.R. No. 161658

Petitioner,

Present:

PUNO, C.J.,

QUISUMBING,

YNARES-SANTIAGO,

CARPIO,

AUSTRIA-MARTINEZ,
- versus - CORONA,

CARPIO MORALES,

AZCUNA,

TINGA,

CHICO-NAZARIO,

VELASCO, JR.,

NACHURA,

REYES,

LEONARDO-DE CASTRO, and

BRION, JJ.

COMMISSION ON ELECTIONS, Promulgated:

Respondent.

November 3, 2008

x-----------------------------------------------------------------------------------------x

DECISION

47
VELASCO, JR., J.:

In these kindred petitions, the constitutionality of Section 36 of Republic Act No. (RA)
9165, otherwise known as the Comprehensive Dangerous Drugs Act of 2002,insofar as it
requires mandatory drug testing of candidates for public office, students of secondary and
tertiary schools, officers and employees of public and private offices, and persons charged before
the prosecutor’s office with certain offenses, among other personalities, is put in issue.

As far as pertinent, the challenged section reads as follows:

SEC. 36. Authorized Drug Testing.—Authorized drug testing shall be done by


any government forensic laboratories or by any of the drug testing laboratories
accredited and monitored by the DOH to safeguard the quality of the test results. x
x x The drug testing shall employ, among others, two (2) testing methods, the
screening test which will determine the positive result as well as the type of drug
used and the confirmatory test which will confirm a positive screening test. x x
x The following shall be subjected to undergo drug testing:

xxxx

(c) Students of secondary and tertiary schools.—Students of secondary and


tertiary schools shall, pursuant to the related rules and regulations as contained in
the school’s student handbook and with notice to the parents, undergo a random
drug testing x x x;

(d) Officers and employees of public and private offices.—Officers and


employees of public and private offices, whether domestic or overseas, shall be
subjected to undergo a random drug test as contained in the company’s work rules
and regulations, x x x for purposes of reducing the risk in the workplace. Any
officer or employee found positive for use of dangerous drugs shall be dealt with
administratively which shall be a ground for suspension or termination, subject to
the provisions of Article 282 of the Labor Code and pertinent provisions of the Civil
Service Law;

48
xxxx

(f) All persons charged before the prosecutor’s office with a criminal offense
having an imposable penalty of imprisonment of not less than six (6) years and one
(1) day shall undergo a mandatory drug test;

(g) All candidates for public office whether appointed or elected both in
the national or local government shall undergo a mandatory drug test.

In addition to the above stated penalties in this Section, those found to be


positive for dangerous drugs use shall be subject to the provisions of Section 15 of
this Act.

G.R. No. 161658 (Aquilino Q. Pimentel, Jr. v. Commission on Elections)

On December 23, 2003, the Commission on Elections (COMELEC) issued Resolution No.
6486, prescribing the rules and regulations on the mandatory drug testing of candidates for
public office in connection with the May 10, 2004 synchronized national and local elections. The
pertinent portions of the said resolution read as follows:

WHEREAS, Section 36 (g) of Republic Act No. 9165 provides:

SEC. 36. Authorized Drug Testing.—x x x

49
xxxx

(g) All candidates for public office x x x both in the national or local
government shall undergo a mandatory drug test.

WHEREAS, Section 1, Article XI of the 1987 Constitution provides that


public officers and employees must at all times be accountable to the people, serve
them with utmost responsibility, integrity, loyalty and efficiency;

WHEREAS, by requiring candidates to undergo mandatory drug test, the


public will know the quality of candidates they are electing and they will be assured
that only those who can serve with utmost responsibility, integrity, loyalty, and
efficiency would be elected x x x.

NOW THEREFORE, The [COMELEC], pursuant to the authority vested in it


under the Constitution, Batas Pambansa Blg. 881 (Omnibus Election Code), [RA]
9165 and other election laws, RESOLVED to promulgate, as it hereby promulgates,
the following rules and regulations on the conduct of mandatory drug testing to
candidates for public office[:]

SECTION 1. Coverage.—All candidates for public office, both national and


local, in the May 10, 2004 Synchronized National and Local Elections shall undergo
mandatory drug test in government forensic laboratories or any drug testing
laboratories monitored and accredited by the Department of Health.

SEC. 3. x x x

On March 25, 2004, in addition to the drug certificates filed with their
respective offices, the Comelec Offices and employees concerned shall submit to
the Law Department two (2) separate lists of candidates. The first list shall consist
of those candidates who complied with the mandatory drug test while the second
list shall consist of those candidates who failed to comply x x x.

50
SEC. 4. Preparation and publication of names of candidates.—Before the
start of the campaign period, the [COMELEC] shall prepare two separate lists of
candidates. The first list shall consist of those candidates who complied with the
mandatory drug test while the second list shall consist of those candidates who
failed to comply with said drug test. x x x

SEC. 5. Effect of failure to undergo mandatory drug test and file drug test
certificate.—No person elected to any public office shall enter upon the duties of his
office until he has undergone mandatory drug test and filed with the offices
enumerated under Section 2 hereof the drug test certificate herein
required. (Emphasis supplied.)

Petitioner Aquilino Q. Pimentel, Jr., a senator of the Republic and a candidate for re-
election in the May 10, 2004 elections,[1] filed a Petition for Certiorari and Prohibition under Rule
65. In it, he seeks (1) to nullify Sec. 36(g) of RA 9165 and COMELEC Resolution No. 6486 dated
December 23, 2003 for being unconstitutional in that they impose a qualification for candidates
for senators in addition to those already provided for in the 1987 Constitution; and (2) to enjoin
the COMELEC from implementing Resolution No. 6486.

Pimentel invokes as legal basis for his petition Sec. 3, Article VI of the Constitution, which
states:

SECTION 3. No person shall be a Senator unless he is a natural-born citizen


of the Philippines, and, on the day of the election, is at least thirty-five years of
age, able to read and write, a registered voter, and a resident of the Philippines for
not less than two years immediately preceding the day of the election.

51
According to Pimentel, the Constitution only prescribes a maximum of five (5)
qualifications for one to be a candidate for, elected to, and be a member of the Senate. He says
that both the Congress and COMELEC, by requiring, via RA 9165 and Resolution No. 6486, a
senatorial aspirant, among other candidates, to undergo a mandatory drug test, create an
additional qualification that all candidates for senator must first be certified as drug free. He
adds that there is no provision in the Constitution authorizing the Congress or COMELEC to
expand the qualification requirements of candidates for senator.

G.R. No. 157870 (Social Justice Society v. Dangerous

Drugs Board and Philippine Drug Enforcement Agency)

In its Petition for Prohibition under Rule 65, petitioner Social Justice Society (SJS),
a registered political party, seeks to prohibit the Dangerous Drugs Board (DDB) and the
Philippine Drug Enforcement Agency (PDEA) from enforcing paragraphs (c), (d), (f), and (g) of
Sec. 36 of RA 9165 on the ground that they are constitutionally infirm. For one, the provisions
constitute undue delegation of legislative power when they give unbridled discretion to schools
and employers to determine the manner of drug testing. For another, the provisions trench in
the equal protection clause inasmuch as they can be used to harass a student or an employee
deemed undesirable. And for a third, a person’s constitutional right against unreasonable
searches is also breached by said provisions.

G.R. No. 158633 (Atty. Manuel J. Laserna, Jr. v. Dangerous


Drugs Board and Philippine Drug Enforcement Agency)

Petitioner Atty. Manuel J. Laserna, Jr., as citizen and taxpayer, also seeks in his Petition
for Certiorari and Prohibition under Rule 65 that Sec. 36(c), (d), (f), and (g) of RA 9165 be
struck down as unconstitutional for infringing on the constitutional right to privacy, the right
against unreasonable search and seizure, and the right against self-incrimination, and for being
contrary to the due process and equal protection guarantees.

52
The Issue on Locus Standi

First off, we shall address the justiciability of the cases at bench and the matter of the
standing of petitioners SJS and Laserna to sue. As respondents DDB and PDEA assert, SJS and
Laserna failed to allege any incident amounting to a violation of the constitutional rights
mentioned in their separate petitions.[2]

It is basic that the power of judicial review can only be exercised in connection with
a bona fide controversy which involves the statute sought to be reviewed.[3] But even with the
presence of an actual case or controversy, the Court may refuse to exercise judicial review
unless the constitutional question is brought before it by a party having the requisite standing to
challenge it.[4] To have standing, one must establish that he or she has suffered some actual or
threatened injury as a result of the allegedly illegal conduct of the government; the injury is
fairly traceable to the challenged action; and the injury is likely to be redressed by a favorable
action.[5]

The rule on standing, however, is a matter of procedure; hence, it can be relaxed for
non-traditional plaintiffs, like ordinary citizens, taxpayers, and legislators when the public
interest so requires, such as when the matter is of transcendental importance, of overarching
significance to society, or of paramount public interest.[6] There is no doubt that Pimentel, as
senator of the Philippines and candidate for the May 10, 2004 elections, possesses the requisite
standing since he has substantial interests in the subject matter of the petition, among other
preliminary considerations. Regarding SJS and Laserna, this Court is wont to relax the rule
on locus standi owing primarily to the transcendental importance and the paramount public
interest involved in the enforcement of Sec. 36 of RA 9165.

The Consolidated Issues

53
The principal issues before us are as follows:

(1) Do Sec. 36(g) of RA 9165 and COMELEC Resolution No. 6486 impose an additional
qualification for candidates for senator? Corollarily, can Congress enact a law prescribing
qualifications for candidates for senator in addition to those laid down by the Constitution? and

(2) Are paragraphs (c), (d), (f), and (g) of Sec. 36, RA 9165 unconstitutional?
Specifically, do these paragraphs violate the right to privacy, the right against unreasonable
searches and seizure, and the equal protection clause? Or do they constitute undue delegation of
legislative power?

Pimentel Petition

(Constitutionality of Sec. 36[g] of RA 9165 and

COMELEC Resolution No. 6486)

In essence, Pimentel claims that Sec. 36(g) of RA 9165 and COMELEC Resolution No.
6486 illegally impose an additional qualification on candidates for senator. He points out that,
subject to the provisions on nuisance candidates, a candidate for senator needs only to meet the
qualifications laid down in Sec. 3, Art. VI of the Constitution, to wit: (1) citizenship, (2) voter
registration, (3) literacy, (4) age, and (5) residency. Beyond these stated qualification
requirements, candidates for senator need not possess any other qualification to run for senator
and be voted upon and elected as member of the Senate. The Congress cannot validly amend or
otherwise modify these qualification standards, as it cannot disregard, evade, or weaken the
force of a constitutional mandate,[7] or alter or enlarge the Constitution.

Pimentel’s contention is well-taken. Accordingly, Sec. 36(g) of RA 9165 should be, as it is


hereby declared as, unconstitutional. It is basic that if a law or an administrative rule violates
any norm of the Constitution, that issuance is null and void and has no effect. The Constitution is
the basic law to which all laws must conform; no act shall be valid if it conflicts with the
Constitution.[8] In the discharge of their defined functions, the three departments of government

54
have no choice but to yield obedience to the commands of the Constitution. Whatever limits it
imposes must be observed.[9]

Congress’ inherent legislative powers, broad as they may be, are subject to certain
limitations. As early as 1927, in Government v. Springer, the Court has defined, in the abstract,
the limits on legislative power in the following wise:

Someone has said that the powers of the legislative department of the
Government, like the boundaries of the ocean, are unlimited. In constitutional
governments, however, as well as governments acting under delegated authority,
the powers of each of the departments x x x are limited and confined within the
four walls of the constitution or the charter, and each department can only exercise
such powers as are necessarily implied from the given powers. The Constitution is
the shore of legislative authority against which the waves of legislative enactment
may dash, but over which it cannot leap.[10]

Thus, legislative power remains limited in the sense that it is subject to substantive and
constitutional limitations which circumscribe both the exercise of the power itself and the
allowable subjects of legislation.[11] The substantive constitutional limitations are chiefly found in
the Bill of Rights[12] and other provisions, such as Sec. 3, Art. VI of the Constitution prescribing
the qualifications of candidates for senators.

In the same vein, the COMELEC cannot, in the guise of enforcing and administering
election laws or promulgating rules and regulations to implement Sec. 36(g), validly impose
qualifications on candidates for senator in addition to what the Constitution prescribes. If
Congress cannot require a candidate for senator to meet such additional qualification, the
COMELEC, to be sure, is also without such power. The right of a citizen in the democratic
process of election should not be defeated by unwarranted impositions of requirement not
otherwise specified in the Constitution.[13]

Sec. 36(g) of RA 9165, as sought to be implemented by the assailed COMELEC resolution,


effectively enlarges the qualification requirements enumerated in the Sec. 3, Art. VI of the

55
Constitution. As couched, said Sec. 36(g) unmistakably requires a candidate for senator to be
certified illegal-drug clean, obviously as a pre-condition to the validity of a certificate of
candidacy for senator or, with like effect, a condition sine qua non to be voted upon and, if
proper, be proclaimed as senator-elect. The COMELEC resolution completes the chain with the
proviso that “[n]o person elected to any public office shall enter upon the duties of his office
until he has undergone mandatory drug test.” Viewed, therefore, in its proper context, Sec.
36(g) of RA 9165 and the implementing COMELEC Resolution add another qualification layer to
what the 1987 Constitution, at the minimum, requires for membership in the Senate. Whether or
not the drug-free bar set up under the challenged provision is to be hurdled before or after
election is really of no moment, as getting elected would be of little value if one cannot assume
office for non-compliance with the drug-testing requirement.

It may of course be argued, in defense of the validity of Sec. 36(g) of RA 9165, that the
provision does not expressly state that non-compliance with the drug test imposition is a
disqualifying factor or would work to nullify a certificate of candidacy. This argument may be
accorded plausibility if the drug test requirement is optional. But the particular section of the
law, without exception, made drug-testing on those covered mandatory, necessarily suggesting
that the obstinate ones shall have to suffer the adverse consequences for not adhering to the
statutory command. And since the provision deals with candidates for public office, it stands to
reason that the adverse consequence adverted to can only refer to and revolve around the
election and the assumption of public office of the candidates. Any other construal would reduce
the mandatory nature of Sec. 36(g) of RA 9165 into a pure jargon without meaning and effect
whatsoever.

While it is anti-climactic to state it at this juncture, COMELEC Resolution No. 6486 is no


longer enforceable, for by its terms, it was intended to cover only the May 10, 2004
synchronized elections and the candidates running in that electoral event. Nonetheless, to
obviate repetition, the Court deems it appropriate to review and rule, as it hereby rules, on its
validity as an implementing issuance.

It ought to be made abundantly clear, however, that the unconstitutionality of Sec. 36(g)
of RA 9165 is rooted on its having infringed the constitutional provision defining the qualification
or eligibility requirements for one aspiring to run for and serve as senator.

56
SJS Petition

(Constitutionality of Sec. 36[c], [d], [f], and [g] of RA 9165)

The drug test prescribed under Sec. 36(c), (d), and (f) of RA 9165 for secondary and
tertiary level students and public and private employees, while mandatory, is a random and
suspicionless arrangement. The objective is to stamp out illegal drug and safeguard in the
process “the well being of [the] citizenry, particularly the youth, from the harmful effects of
dangerous drugs.” This statutory purpose, per the policy-declaration portion of the law, can be
achieved via the pursuit by the state of “an intensive and unrelenting campaign against the
trafficking and use of dangerous drugs x x x through an integrated system of planning,
implementation and enforcement of anti-drug abuse policies, programs and projects.”[14] The
primary legislative intent is not criminal prosecution, as those found positive for illegal drug use
as a result of this random testing are not necessarily treated as criminals. They may even be
exempt from criminal liability should the illegal drug user consent to undergo
rehabilitation. Secs. 54 and 55 of RA 9165 are clear on this point:

Sec. 54. Voluntary Submission of a Drug Dependent to Confinement,


Treatment and Rehabilitation.—A drug dependent or any person who violates
Section 15 of this Act may, by himself/herself or through his/her parent, [close
relatives] x x x apply to the Board x x x for treatment and rehabilitation of the drug
dependency. Upon such application, the Board shall bring forth the matter to the
Court which shall order that the applicant be examined for drug dependency. If the
examination x x x results in the certification that the applicant is a drug dependent,
he/she shall be ordered by the Court to undergo treatment and rehabilitation in a
Center designated by the Board x x x.

xxxx

Sec. 55. Exemption from the Criminal Liability Under the Voluntary
Submission Program.—A drug dependent under the voluntary submission program,

57
who is finally discharged from confinement, shall be exempt from the criminal
liability under Section 15 of this Act subject to the following conditions:

xxxx

School children, the US Supreme Court noted, are most vulnerable to the physical,
psychological, and addictive effects of drugs. Maturing nervous systems of the young are more
critically impaired by intoxicants and are more inclined to drug dependency. Their recovery is
also at a depressingly low rate.[15]

The right to privacy has been accorded recognition in this jurisdiction as a facet of the
right protected by the guarantee against unreasonable search and seizure[16] under Sec. 2, Art.
III[17] of the Constitution. But while the right to privacy has long come into its own, this case
appears to be the first time that the validity of a state-decreed search or intrusion through the
medium of mandatory random drug testing among students and employees is, in this
jurisdiction, made the focal point. Thus, the issue tendered in these proceedings is veritably one
of first impression.

US jurisprudence is, however, a rich source of persuasive jurisprudence. With respect to


random drug testing among school children, we turn to the teachings ofVernonia School
District 47J v. Acton (Vernonia) and Board of Education of Independent School District No. 92 of
Pottawatomie County, et al. v. Earls, et al. (Board of Education),[18] both fairly pertinent US
Supreme Court-decided cases involving the constitutionality of governmental search.

In Vernonia, school administrators in Vernonia, Oregon wanted to address the drug


menace in their respective institutions following the discovery of frequent drug use by school
athletes. After consultation with the parents, they required random urinalysis drug testing for
the school’s athletes. James Acton, a high school student, was denied participation in the
football program after he refused to undertake the urinalysis drug testing. Acton forthwith sued,

58
claiming that the school’s drug testing policy violated, inter alia, the Fourth Amendment[19] of
the US Constitution.

The US Supreme Court, in fashioning a solution to the issues raised in Vernonia,


considered the following: (1) schools stand in loco parentis over their students; (2) school
children, while not shedding their constitutional rights at the school gate, have less privacy
rights; (3) athletes have less privacy rights than non-athletes since the former observe
communal undress before and after sports events; (4) by joining the sports activity, the athletes
voluntarily subjected themselves to a higher degree of school supervision and regulation; (5)
requiring urine samples does not invade a student’s privacy since a student need not undress for
this kind of drug testing; and (6) there is need for the drug testing because of the dangerous
effects of illegal drugs on the young. The US Supreme Court held that the policy
constituted reasonable search under the Fourth[20] and 14th Amendments and declared the
random drug-testing policy constitutional.

In Board of Education, the Board of Education of a school


in Tecumseh, Oklahoma required a drug test for high school students desiring to join extra-
curricular activities. Lindsay Earls, a member of the show choir, marching band, and academic
team declined to undergo a drug test and averred that the drug-testing policy made to apply to
non-athletes violated the Fourth and 14th Amendments. As Earls argued, unlike athletes who
routinely undergo physical examinations and undress before their peers in locker rooms, non-
athletes are entitled to more privacy.

The US Supreme Court, citing Vernonia, upheld the constitutionality of drug testing even
among non-athletes on the basis of the school’s custodial responsibility and authority. In so
ruling, said court made no distinction between a non-athlete and an athlete. It ratiocinated that
schools and teachers act in place of the parents with a similar interest and duty of safeguarding
the health of the students. And in holding that the school could implement its random drug-
testing policy, the Court hinted that such a test was a kind of search in which even a reasonable
parent might need to engage.

59
In sum, what can reasonably be deduced from the above two cases and applied to this
jurisdiction are: (1) schools and their administrators stand in loco parentis with respect to their
students; (2) minor students have contextually fewer rights than an adult, and are subject to
the custody and supervision of their parents, guardians, and schools; (3) schools, acting in loco
parentis, have a duty to safeguard the health and well-being of their students and may adopt
such measures as may reasonably be necessary to discharge such duty; and (4) schools have
the right to impose conditions on applicants for admission that are fair, just, and non-
discriminatory.

Guided by Vernonia and Board of Education, the Court is of the view and so holds that the
provisions of RA 9165 requiring mandatory, random, and suspicionless drug testing of students
are constitutional. Indeed, it is within the prerogative of educational institutions to require, as a
condition for admission, compliance with reasonable school rules and regulations and
policies. To be sure, the right to enroll is not absolute; it is subject to fair, reasonable, and
equitable requirements.

The Court can take judicial notice of the proliferation of prohibited drugs in the country
that threatens the well-being of the people,[21] particularly the youth and school children who
usually end up as victims. Accordingly, and until a more effective method is conceptualized and
put in motion, a random drug testing of students in secondary and tertiary schools is not only
acceptable but may even be necessary if the safety and interest of the student population,
doubtless a legitimate concern of the government, are to be promoted and protected. To
borrow from Vernonia, “[d]eterring drug use by our Nation’s schoolchildren is as important as
enhancing efficient enforcement of the Nation’s laws against the importation of drugs”; the
necessity for the State to act is magnified by the fact that the effects of a drug-infested school
are visited not just upon the users, but upon the entire student body and faculty.[22] Needless to
stress, the random testing scheme provided under the law argues against the idea that the
testing aims to incriminate unsuspecting individual students.

Just as in the case of secondary and tertiary level students, the mandatory but random
drug test prescribed by Sec. 36 of RA 9165 for officers and employees of public and private
offices is justifiable, albeit not exactly for the same reason. The Court notes in this regard that
petitioner SJS, other than saying that “subjecting almost everybody to drug testing, without

60
probable cause, is unreasonable, an unwarranted intrusion of the individual right to
privacy,”[23] has failed to show how the mandatory, random, and suspicionless drug testing
under Sec. 36(c) and (d) of RA 9165 violates the right to privacy and constitutes unlawful and/or
unconsented search under Art. III, Secs. 1 and 2 of the Constitution.[24] Petitioner Laserna’s
lament is just as simplistic, sweeping, and gratuitous and does not merit serious consideration.
Consider what he wrote without elaboration:

The US Supreme Court and US Circuit Courts of Appeals have made various
rulings on the constitutionality of mandatory drug tests in the school and the
workplaces. The US courts have been consistent in their rulings that the mandatory
drug tests violate a citizen’s constitutional right to privacy and right against
unreasonable search and seizure. They are quoted extensively hereinbelow.[25]

The essence of privacy is the right to be left alone.[26] In context, the right to privacy
means the right to be free from unwarranted exploitation of one’s person or from intrusion into
one’s private activities in such a way as to cause humiliation to a person’s ordinary
sensibilities. [27] And while there has been general agreement as to the basic function of the
guarantee against unwarranted search, “translation of the abstract prohibition against
‘unreasonable searches and seizures’ into workable broad guidelines for the decision of particular
cases is a difficult task,” to borrow from C. Camara v. Municipal Court.[28] Authorities are agreed
though that the right to privacy yields to certain paramount rights of the public and defers to
the state’s exercise of police power.[29]

As the warrantless clause of Sec. 2, Art III of the Constitution is couched and as has been
held, “reasonableness” is the touchstone of the validity of a government search or
intrusion.[30] And whether a search at issue hews to the reasonableness standard is judged by
the balancing of the government-mandated intrusion on the individual’s privacy interest against
the promotion of some compelling state interest.[31] In the criminal context, reasonableness
requires showing of probable cause to be personally determined by a judge. Given that the drug-
testing policy for employees––and students for that matter––under RA 9165 is in the nature of
administrative search needing what was referred to inVernonia as “swift and informal disciplinary
procedures,” the probable-cause standard is not required or even practicable. Be that as it may,
the review should focus on the reasonableness of the challenged administrative search in
question.
61
The first factor to consider in the matter of reasonableness is the nature of the privacy
interest upon which the drug testing, which effects a search within the meaning of Sec. 2, Art.
III of the Constitution, intrudes. In this case, the office or workplace serves as the backdrop for
the analysis of the privacy expectation of the employees and the reasonableness of drug testing
requirement. The employees’ privacy interest in an office is to a large extent circumscribed by
the company’s work policies, the collective bargaining agreement, if any, entered into by
management and the bargaining unit, and the inherent right of the employer to maintain
discipline and efficiency in the workplace. Their privacy expectation in a regulated office
environment is, in fine, reduced; and a degree of impingement upon such privacy has been
upheld.

Just as defining as the first factor is the character of the intrusion authorized by the
challenged law. Reduced to a question form, is the scope of the search or intrusion clearly set
forth, or, as formulated in Ople v. Torres, is the enabling law authorizing a search “narrowly
drawn” or “narrowly focused”?[32]

The poser should be answered in the affirmative. For one, Sec. 36 of RA 9165 and its
implementing rules and regulations (IRR), as couched, contain provisions specifically directed
towards preventing a situation that would unduly embarrass the employees or place them under
a humiliating experience. While every officer and employee in a private establishment is under
the law deemed forewarned that he or she may be a possible subject of a drug test, nobody is
really singled out in advance for drug testing. The goal is to discourage drug use by not telling in
advance anyone when and who is to be tested. And as may be observed, Sec. 36(d) of RA 9165
itself prescribes what, inOple, is a narrowing ingredient by providing that the employees
concerned shall be subjected to “random drug test as contained in the company’s work rules and
regulations x x x for purposes of reducing the risk in the work place.”

For another, the random drug testing shall be undertaken under conditions calculated to
protect as much as possible the employee’s privacy and dignity. As to the mechanics of the test,
the law specifies that the procedure shall employ two testing methods, i.e., the screening test
and the confirmatory test, doubtless to ensure as much as possible the trustworthiness of the
results. But the more important consideration lies in the fact that the test shall be conducted by
62
trained professionals in access-controlled laboratories monitored by the Department of Health
(DOH) to safeguard against results tampering and to ensure an accurate chain of custody.[33] In
addition, the IRR issued by the DOH provides that access to the drug results shall be on the
“need to know” basis;[34] that the “drug test result and the records shall be [kept] confidential
subject to the usual accepted practices to protect the confidentiality of the test
[35]
results.” Notably, RA 9165 does not oblige the employer concerned to report to the
prosecuting agencies any information or evidence relating to the violation of
the Comprehensive Dangerous Drugs Act received as a result of the operation of the drug
testing. All told, therefore, the intrusion into the employees’ privacy, under RA 9165, is
accompanied by proper safeguards, particularly against embarrassing leakages of test results,
and is relatively minimal.

To reiterate, RA 9165 was enacted as a measure to stamp out illegal drug in the country
and thus protect the well-being of the citizens, especially the youth, from the deleterious effects
of dangerous drugs. The law intends to achieve this through the medium, among others, of
promoting and resolutely pursuing a national drug abuse policy in the workplace via a
mandatory random drug test.[36] To the Court, the need for drug testing to at least minimize
illegal drug use is substantial enough to override the individual’s privacy interest under the
premises. The Court can consider that the illegal drug menace cuts across gender, age group,
and social- economic lines. And it may not be amiss to state that the sale, manufacture, or
trafficking of illegal drugs, with their ready market, would be an investor’s dream were it not for
the illegal and immoral components of any of such activities. The drug problem has hardly
abated since the martial law public execution of a notorious drug trafficker. The state can no
longer assume a laid back stance with respect to this modern-day scourge. Drug enforcement
agencies perceive a mandatory random drug test to be an effective way of preventing and
deterring drug use among employees in private offices, the threat of detection by random
testing being higher than other modes. The Court holds that the chosen method is a reasonable
and enough means to lick the problem.

Taking into account the foregoing factors, i.e., the reduced expectation of privacy on the
part of the employees, the compelling state concern likely to be met by the search, and the well-
defined limits set forth in the law to properly guide authorities in the conduct of the random
testing, we hold that the challenged drug test requirement is, under the limited context of the
case, reasonable and, ergo, constitutional.

63
Like their counterparts in the private sector, government officials and employees also
labor under reasonable supervision and restrictions imposed by the Civil Service law and other
laws on public officers, all enacted to promote a high standard of ethics in the public
service.[37] And if RA 9165 passes the norm of reasonableness for private employees, the more
reason that it should pass the test for civil servants, who, by constitutional command, are
required to be accountable at all times to the people and to serve them with utmost
responsibility and efficiency.[38]

Petitioner SJS’ next posture that Sec. 36 of RA 9165 is objectionable on the ground of
undue delegation of power hardly commends itself for concurrence. Contrary to its position, the
provision in question is not so extensively drawn as to give unbridled options to schools and
employers to determine the manner of drug testing. Sec. 36 expressly provides how drug
testing for students of secondary and tertiary schools and officers/employees of public/private
offices should be conducted. It enumerates the persons who shall undergo drug testing. In the
case of students, the testing shall be in accordance with the school rules as contained in the
student handbook and with notice to parents. On the part of officers/employees, the testing shall
take into account the company’s work rules. In either case, the random procedure shall be
observed, meaning that the persons to be subjected to drug test shall be picked by chance or in
an unplanned way. And in all cases, safeguards against misusing and compromising the
confidentiality of the test results are established.

Lest it be overlooked, Sec. 94 of RA 9165 charges the DDB to issue, in consultation with
the DOH, Department of the Interior and Local Government, Department of Education, and
Department of Labor and Employment, among other agencies, the IRR necessary to enforce the
law. In net effect then, the participation of schools and offices in the drug testing scheme shall
always be subject to the IRR of RA 9165. It is, therefore, incorrect to say that schools and
employers have unchecked discretion to determine how often, under what conditions, and where
the drug tests shall be conducted.

The validity of delegating legislative power is now a quiet area in the constitutional
landscape.[39] In the face of the increasing complexity of the task of the government and the
increasing inability of the legislature to cope directly with the many problems demanding its
64
attention, resort to delegation of power, or entrusting to administrative agencies the power of
subordinate legislation, has become imperative, as here.

Laserna Petition (Constitutionality of Sec. 36[c], [d],

[f], and [g] of RA 9165)

Unlike the situation covered by Sec. 36(c) and (d) of RA 9165, the Court finds no valid
justification for mandatory drug testing for persons accused of crimes. In the case of students,
the constitutional viability of the mandatory, random, and suspicionless drug testing for students
emanates primarily from the waiver by the students of their right to privacy when they seek
entry to the school, and from their voluntarily submitting their persons to the parental authority
of school authorities. In the case of private and public employees, the constitutional soundness
of the mandatory, random, and suspicionless drug testing proceeds from the reasonableness of
the drug test policy and requirement.

We find the situation entirely different in the case of persons charged before the public
prosecutor’s office with criminal offenses punishable with six (6) years and one (1) day
imprisonment. The operative concepts in the mandatory drug testing are “randomness” and
“suspicionless.” In the case of persons charged with a crime before the prosecutor’s office, a
mandatory drug testing can never be random or suspicionless. The ideas of randomness and
being suspicionless are antithetical to their being made defendants in a criminal complaint. They
are not randomly picked; neither are they beyond suspicion. When persons suspected of
committing a crime are charged, they are singled out and are impleaded against their will. The
persons thus charged, by the bare fact of being haled before the prosecutor’s office and
peaceably submitting themselves to drug testing, if that be the case, do not necessarily consent
to the procedure, let alone waive their right to privacy.[40] To impose mandatory drug testing on
the accused is a blatant attempt to harness a medical test as a tool for criminal prosecution,
contrary to the stated objectives of RA 9165. Drug testing in this case would violate a persons’
right to privacy guaranteed under Sec. 2, Art. III of the Constitution. Worse still, the accused
persons are veritably forced to incriminate themselves.

65
WHEREFORE, the Court resolves to GRANT the petition in G.R. No. 161658 and
declares Sec. 36(g) of RA 9165 and COMELEC Resolution No. 6486 asUNCONSTITUTIONAL;
and to PARTIALLY GRANT the petition in G.R. Nos. 157870 and 158633 by declaring Sec.
36(c) and (d) of RA 9165 CONSTITUTIONAL, but declaring its Sec.
36(f) UNCONSTITUTIONAL. All concerned agencies are, accordingly, permanently enjoined
from implementing Sec. 36(f) and (g) of RA 9165. No costs.

SO ORDERED.

PRESBITERO J. VELASCO, JR.

Associate Justice

WE CONCUR:

REYNATO S. PUNO

Chief Justice

LEONARDO A. QUISUMBING CONSUELO YNARES-SANTIAGO

Associate Justice Associate Justice

66
ANTONIO T. CARPIO MA. ALICIA AUSTRIA-MARTINEZ

Associate Justice Associate Justice

RENATO C. CORONA CONCHITA CARPIO MORALES

Associate Justice Associate Justice

ADOLFO S. AZCUNA DANTE O. TINGA

Associate Justice Associate Justice

MINITA V. CHICO-NAZARIO ANTONIO EDUARDO B. NACHURA

Associate Justice Associate Justice

67
RUBEN T. REYES TERESITA J. LEONARDO-DE CASTRO

Associate Justice Associate Justice

ARTURO D. BRION

Associate Justice

CERTIFICATION

Pursuant to Section 13, Article VIII of the Constitution, I certify that the conclusions in
the above Decision had been reached in consultation before the case was assigned to the writer
of the opinion of the Court.

REYNATO S. PUNO

Chief Justice

68
[1]
Re-elected as senator in the 2004 elections.
[2]
Rollo (G.R. No. 158633), pp. 184-185.
[3]
Dumlao v. COMELEC, No. L-52245, January 22, 1980, 95 SCRA 392, 401.
[4]
Bernas, THE 1987 CONSTITUTION OF THE REPUBLIC OF THE PHILIPPINES: A
COMMENTARY 939 (2003).
[5]
Gonzales v. Narvasa, G.R. No. 140835, August 14, 2000, 337 SCRA 733, 740.
[6]
Tatad v. Secretary of the Department of Energy, G.R. Nos. 124360 & 127867,
November 5, 1997, 281 SCRA 330, 349; De Guia v. COMELEC, G.R. No. 104712, May 6, 1992,
208 SCRA 420, 422.
[7]
Palmer v. Board of Education, 276 NY 222 11 NE 2d 887.
[8]
Cruz, CONSTITUTIONAL LAW 4 (2000).
[9]
Mutuc v. Commission on Elections, No. L-32717, November 26, 1970, 36 SCRA
228, 234.
[10]
50 Phil. 259, 309 (1927).
[11]
J. Bernas, S.J., THE 1987 CONSTITUTION OF THE REPUBLIC OF
THE PHILIPPINES: A COMMENTARY 604 (1996).
[12]
Id.
[13]
See concurring opinion in Go v. Commision on Elections, G.R. No. 147741, May
10, 2001, 357 SCRA 739, 753.
[14]
RA 9165, Sec. 2.
[15]
Vernonia School District 47J v. Acton, 515 U.S. 646 (1995), 661.
[16]
Ople v. Torres, G.R. No. 127685, July 23, 1998, 293 SCRA 141, 169; citing Morfe
v. Mutuc, No. L-20387, January 31, 1968, 22 SCRA 424, 444-445.
[17]
Sec. 2. The right of the people to be secure in their persons, houses, papers, and
effects against unreasonable searches and seizures of whatever nature and for any purpose shall
be inviolable, and no search warrant or warrant of arrest shall issue except upon probable cause
to be determined personally by the judge after examination under oath or affirmation of the
complainant and the witnesses he may produce, and particularly describing the place to be
searched and the person or things to be seized.
[18]
536 U.S. 822 (2002); cited in 2 Bernas, CONSTITUTIONAL RIGHTS AND SOCIAL
DEMANDS 224-227 (2004).
[19]
The right of the people to be secure in their persons, houses, papers, and effects,
against unreasonable searches and seizures, shall not be violated, and no Warrants shall issue,
but upon probable cause, supported by Oath or affirmation, and particularly describing the place
to be searched, and the persons or things to be seized.
[20]
The Fourth Amendment is almost similar to Sec. 2, Art. III of the Constitution,
except that the latter limited the determination of probable cause to a judge after an
examination under oath of the complainant and his witnesses. Hence, pronouncements of the US
Federal Supreme Court and State Appellate Court may be considered doctrinal in this
jurisdiction, unless they are manifestly contrary to our Constitution. See Herrera, HANDBOOK
ON ARREST, SEARCH AND SEIZURE8 (2003).
[21]
Tolentino v. Alconcel, No. L-63400, March 18, 1983, 121 SCRA 92, 95-96.
[22]
Rollo (G.R. No. 158633), p. 204, respondents’ Consolidated Memorandum.
[23]
Rollo (G.R. No. 157870), p. 10.

69
[24]
Section 1. No person shall be deprived of life, liberty, or property without due
process of law, nor shall any person be denied the equal protection of the laws.
Sec. 2. The right of the people to be secure in their persons, houses, papers, and
effects against unreasonable searches and seizures of whatever nature and for any purpose shall
be inviolable, and no search warrant or warrant of arrest shall issue except upon probable cause
to be determined personally by the judge after examination under oath or affirmation of the
complainant and the witnesses he may produce, and particularly describing the place to be
searched and the person or things to be seized.
[25]
Rollo (G.R. No. 158633), p. 9.
[26]
Ople, supra note 16, at 153; citing Cooley on Torts, Sec. 135, Vol. 1, 4th ed., [1932].
[27]
62 Am. Jur. 2d, Privacy, Sec. 1.
[28]
387 U.S. 523; cited in 2 Bernas, supra note 18, at 232.
[29]
62 Am. Jur. 2d, Privacy, Sec. 17.
[30]
Vernonia & Board of Education, supra notes 15 & 18.
[31]
Skinner v. Railway Labor Executives Assn., 489 U.S. 602, 619 (1989); cited
in Vernonia, supra.
[32]
Supra note 16, at 166 & 169.
[33]
Under Sec. 7 [3] of the DOH IRR Governing Licensing and Accreditation of Drug
Laboratories, a laboratory is required to use documented chain of custody procedures to
maintain control and custody of specimens.
[34]
DOH IRR Governing Licensing and Accreditation of Drug Laboratories, Sec. 7
[10.3] provides that the original copy of the test results form shall be given to the client/donor,
copy furnished the DOH and the requesting agency.
[35]
Id., Sec. 7 [10.4].
[36]
Secs. 47 and 48 of RA 9165 charge the Department of Labor and Employment
with the duty to develop and promote a national drug prevention program and the necessary
guidelines in the work place, which shall include a mandatory drafting and adoption of policies to
achieve a drug-free workplace.
[37]
CODE OF CONDUCT AND ETHICAL STANDARDS FOR PUBLIC OFFICERS AND
EMPLOYEES, Sec. 2.
[38]
CONSTITUTION, Art. XI, Sec. 1.
[39]
Tatad, supra note 6, at 351.
[40]
Leona Pasion Viuda de Garcia v. Locsin, 65 Phil. 689, 695 (1938); citing
Cooley, CONST. LIM. 630 (8th ed.).

70
FIRST DIVISION

SOCIAL JUSTICE SOCIETY G.R. No. 156052

(SJS), VLADIMIR ALARIQUE T.

CABIGAO and BONIFACIO S.

TUMBOKON,

Petitioners, Present:

PUNO, C.J., Chairperson,

SANDOVAL-GUTIERREZ,

-versus- CORONA,

AZCUNA and

LEONARDO-DE CASTRO, JJ.

HON. JOSE L. ATIENZA, JR.,


in his capacity as Mayor of the
City of Manila,
Respondent.

x----------------------x

CHEVRON PHILIPPINES INC.,

PETRON CORPORATION and

PILIPINAS SHELL PETROLEUM

CORPORATION,

Movants-Intervenors.

71
x----------------------x

DEPARTMENT OF ENERGY,

Movant-Intervenor. Promulgated:

February 13, 2008

x- - - - - - - - - - - - - - - - - - - - - - - - - - - - - - - - - - - - - - - - - x

RESOLUTION

CORONA, J.:

After we promulgated our decision in this case on March 7, 2007, Chevron Philippines Inc.

(Chevron), Petron Corporation (Petron) and Pilipinas Shell Petroleum Corporation (Shell)

(collectively, the oil companies) and the Republic of the Philippines, represented by the

Department of Energy (DOE), filed their respective motions for leave to intervene and for

reconsideration of the decision.

Chevron[1] is engaged in the business of importing, distributing and marketing of

petroleum products in the Philippines while Shell and Petron are engaged in the business of

manufacturing, refining and likewise importing, distributing and marketing of petroleum


72
products in the Philippines.[2] The DOE is a governmental agency created under Republic Act

(RA) No. 7638[3] and tasked to prepare, integrate, coordinate, supervise and control all plans,

programs, projects and activities of the government relative to energy exploration,

development, utilization, distribution and conservation.[4]

The facts are restated briefly as follows:

Petitioners Social Justice Society, Vladimir Alarique T. Cabigao and Bonifacio S.

Tumbokon, in an original petition for mandamusunder Rule 65 of the Rules of Court, sought to

compel respondent Hon. Jose L. Atienza, Jr., then mayor of the City of Manila, to enforce

Ordinance No. 8027. This ordinance was enacted by the Sangguniang Panlungsod of Manila on

November 20, 2001,[5] approved by respondent Mayor on November 28, 2001,[6] and became

effective on December 28, 2001 after publication.[7] Sections 1 and 3 thereof state:

SECTION 1. For the purpose of promoting sound urban planning and ensuring
health, public safety, and general welfare of the residents of Pandacan and Sta. Ana
as well as its adjoining areas, the land use of [those] portions of land bounded by
the Pasig River in the north, PNR Railroad Track in the east, Beata St. in the south,
Palumpong St. in the southwest, and Estero de Pandacan in the west[,] PNR
Railroad in the northwest area, Estero de Pandacan in the [n]ortheast, Pasig River
in the southeast and Dr. M.L. Carreon in the southwest. The area of Punta, Sta.
Ana bounded by the Pasig River, Marcelino Obrero St., Mayo 28 St., and F. Manalo
Street, are hereby reclassified from Industrial II to Commercial I.

xxx xxx xxx

SEC. 3. Owners or operators of industries and other businesses, the operation of


which are no longer permitted under Section 1 hereof, are hereby given a period of
six (6) months from the date of effectivity of this Ordinance within which to cease
and desist from the operation of businesses which are hereby in consequence,
disallowed.

73
Ordinance No. 8027 reclassified the area described therein from industrial to commercial

and directed the owners and operators of businesses disallowed under the reclassification to

cease and desist from operating their businesses within six months from the date of effectivity of

the ordinance. Among the businesses situated in the area are the so-called “Pandacan

Terminals” of the oil companies.

On June 26, 2002, the City of Manila and the Department of Energy (DOE) entered into a

memorandum of understanding (MOU)[8]with the oil companies. They agreed that “the scaling

down of the Pandacan Terminals [was] the most viable and practicable option.” TheSangguniang

Panlungsod ratified the MOU in Resolution No. 97.[9] In the same resolution,

the Sanggunian declared that the MOU was effective only for a period of six months starting July

25, 2002.[10] Thereafter, on January 30, 2003, the Sanggunian adopted Resolution No.

13[11] extending the validity of Resolution No. 97 to April 30, 2003 and authorizing the mayor of

Manila to issue special business permits to the oil companies.[12]

This was the factual backdrop presented to the Court which became the basis of our

March 7, 2007 decision. We ruled that respondent had the ministerial duty under the Local

Government Code (LGC) to “enforce all laws and ordinances relative to the governance of the

city,”[13] including Ordinance No. 8027. We also held that we need not resolve the issue of

whether the MOU entered into by respondent with the oil companies and the subsequent

resolutions passed by the Sanggunian could amend or repeal Ordinance No. 8027 since the

resolutions which ratified the MOU and made it binding on the City of Manila expressly gave it

full force and effect only until April 30, 2003. We concluded that there was nothing that legally

hindered respondent from enforcing Ordinance No. 8027.

74
After we rendered our decision on March 7, 2007, the oil companies and DOE sought to

intervene and filed motions for reconsideration in intervention on March 12, 2007 and March 21,

2007 respectively. On April 11, 2007, we conducted the oral arguments in Baguio City to hear

petitioners, respondent and movants-intervenors oil companies and DOE.

The oil companies called our attention to the fact that on April 25, 2003, Chevron had filed

a complaint against respondent and the City of Manila in the Regional Trial Court (RTC) of

Manila, Branch 39, for the annulment of Ordinance No. 8027 with application for writs of

preliminary prohibitory injunction and preliminary mandatory injunction.[14] The case was

docketed as civil case no. 03-106377. On the same day, Shell filed a petition for prohibition

and mandamus likewise assailing the validity of Ordinance No. 8027 and with application for

writs of preliminary prohibitory injunction and preliminary mandatory injunction.[15] This was

docketed as civil case no. 03-106380. Later on, these two cases were consolidated and the RTC

of Manila, Branch 39 issued an order dated May 19, 2003 granting the applications for writs of

preliminary prohibitory injunction and preliminary mandatory injunction:

WHEREFORE, upon the filing of a total bond of TWO MILLION (Php


2,000,000.00) PESOS, let a Writ of Preliminary Prohibitory Injunction be issued
ordering [respondent] and the City of Manila, their officers, agents, representatives,
successors, and any other persons assisting or acting in their behalf, during the
pendency of the case, to REFRAIN from taking steps to enforce Ordinance No.
8027, and let a Writ of Preliminary Mandatory Injunction be issued ordering
[respondent] to issue [Chevron and Shell] the necessary Business Permits to
operate at the Pandacan Terminal.[16]

Petron likewise filed its own petition in the RTC of Manila, Branch 42, also attacking the

validity of Ordinance No. 8027 with prayer for the issuance of a writ of preliminary injunction

75
and/or temporary restraining order (TRO). This was docketed as civil case no. 03-106379. In

an order dated August 4, 2004, the RTC enjoined the parties to maintain the status quo.[17]

Thereafter, in 2006, the city council of Manila enacted Ordinance No. 8119, also known

as the Manila Comprehensive Land Use Plan and Zoning Ordinance of 2006.[18] This was

approved by respondent on June 16, 2006.[19]

Aggrieved anew, Chevron and Shell filed a complaint in the RTC of Manila, Branch 20,

asking for the nullification of Ordinance No. 8119.[20] This was docketed as civil case no. 06-

115334. Petron filed its own complaint on the same causes of action in the RTC of Manila,

Branch 41.[21] This was docketed as civil case no. 07-116700.[22] The court issued a TRO in

favor of Petron, enjoining the City of Manila and respondent from enforcing Ordinance No.

8119.[23]

Meanwhile, in civil case no. 03-106379, the parties filed a joint motion to withdraw

complaint and counterclaim on February 20, 2007.[24] In an order dated April 23, 2007, the

joint motion was granted and all the claims and counterclaims of the parties were

withdrawn.[25]

Given these additional pieces of information, the following were submitted as issues for

our resolution:

1. whether movants-intervenors should be allowed to intervene in this case;[26]

2. whether the following are impediments to the execution of our March 7, 2007

decision:

(a) Ordinance No. 8119, the enactment and existence of which were

not previously brought by the parties to the attention of the Court and

76
(b) writs of preliminary prohibitory injunction and preliminary

mandatory injunction and status quo order issued by the RTC of

Manila, Branches 39 and 42 and

3. whether the implementation of Ordinance No. 8027 will unduly encroach upon the

DOE’s powers and functions involving energy resources.

During the oral arguments, the parties submitted to this Court’s power to rule on the

constitutionality and validity of Ordinance No. 8027 despite the pendency of consolidated

cases involving this issue in the RTC.[27] The importance of settling this controversy as fully

and as expeditiously as possible was emphasized, considering its impact on public

interest. Thus, we will also dispose of this issue here. The parties were after all given ample

opportunity to present and argue their respective positions. By so doing, we will do away

with the delays concomitant with litigation and completely adjudicate an issue which will

most likely reach us anyway as the final arbiter of all legal disputes.

Before we resolve these issues, a brief review of the history of the Pandacan Terminals

is called for to put our discussion in the proper context.

HISTORY OF THE PANDACAN


OIL TERMINALS

Pandacan (one of the districts of the City of Manila) is situated along the banks of the

Pasig river. At the turn of the twentieth century, Pandacan was unofficially designated as the

industrial center of Manila. The area, then largely uninhabited, was ideal for various emerging
77
industries as the nearby river facilitated the transportation of goods and products. In the 1920s,

it was classified as an industrial zone.[28] Among its early industrial settlers were the oil

companies. Shell established its installation there on January 30, 1914.[29] Caltex (now

Chevron) followed suit in 1917 when the company began marketing its products in the

country.[30] In 1922, it built a warehouse depot which was later converted into a key distribution

terminal.[31] The corporate presence in the Philippines of Esso (Petron’s predecessor) became

more keenly felt when it won a concession to build and operate a refinery in Bataan in

1957.[32] It then went on to operate a state-of-the-art lube oil blending plant in the Pandacan

Terminals where it manufactures lubes and greases.[33]

On December 8, 1941, the Second World War reached the shores of the Philippine

Islands. Although Manila was declared an open city, the Americans had no interest in welcoming

the Japanese. In fact, in their zealous attempt to fend off the Japanese Imperial Army, the

United States Army took control of the Pandacan Terminals and hastily made plans to destroy

the storage facilities to deprive the advancing Japanese Army of a valuable logistics

weapon.[34] The U.S. Army burned unused petroleum, causing a frightening conflagration.

Historian Nick Joaquin recounted the events as follows:

After the USAFFE evacuated the City late in December 1941, all army fuel storage
dumps were set on fire. The flames spread, enveloping the City in smoke, setting
even the rivers ablaze, endangering bridges and all riverside buildings. … For one
week longer, the “open city” blazed—a cloud of smoke by day, a pillar of fire by
night.[35]

The fire consequently destroyed the Pandacan Terminals and rendered its network of depots and

service stations inoperative.[36]

78
After the war, the oil depots were reconstructed. Pandacan changed as Manila rebuilt

itself. The three major oil companies resumed the operation of their depots.[37] But the district

was no longer a sparsely populated industrial zone; it had evolved into a bustling, hodgepodge

community. Today, Pandacan has become a densely populated area inhabited by about 84,000

people, majority of whom are urban poor who call it home.[38] Aside from numerous industrial

installations, there are also small businesses, churches, restaurants, schools, daycare centers

and residences situated there.[39] Malacañang Palace, the official residence of the President of

the Philippines and the seat of governmental power, is just two kilometers away.[40] There is a

private school near the Petron depot. Along the walls of the Shell facility are shanties of informal

settlers.[41] More than 15,000 students are enrolled in elementary and high schools situated near

these facilities.[42] A university with a student population of about 25,000 is located directly

across the depot on the banks of the Pasig river.[43]

The 36-hectare Pandacan Terminals house the oil companies’ distribution terminals and

depot facilities.[44] The refineries of Chevron and Shell in Tabangao and Bauan, both in

Batangas, respectively, are connected to the Pandacan Terminals through a 114-

kilometer[45]underground pipeline system.[46] Petron’s refinery in Limay, Bataan, on the other

hand, also services the depot.[47] The terminals store fuel and other petroleum products and

supply 95% of the fuel requirements of Metro Manila,[48] 50% of Luzon’s consumption and 35%

nationwide.[49] Fuel can also be transported through barges along the Pasig river or tank trucks

via the South Luzon Expressway.

We now discuss the first issue: whether movants-intervenors should be allowed to

intervene in this case.

INTERVENTION OF THE OIL COMPANIES AND THE DOE


SHOULD BE ALLOWED IN THE INTEREST OF JUSTICE
79
Intervention is a remedy by which a third party, not originally impleaded in the

proceedings, becomes a litigant therein to enable him, her or it to protect or preserve a right

or interest which may be affected by such proceedings.[50] The pertinent rules are Sections 1

and 2, Rule 19 of the Rules of Court:

SEC. 1. Who may intervene. — A person who has a legal interest in the
matter in litigation, or in the success of either of the parties, or an interest against
both, or is so situated as to be adversely affected by a distribution or other
disposition of property in the custody of the court or of an officer thereof may, with
leave of court, be allowed to intervene in the action. The court shall consider
whether or not the intervention will unduly delay or prejudice the adjudication of
the rights of the original parties, and whether or not the intervenor’s rights may be
fully protected in a separate proceeding.

SEC. 2. Time to intervene. — The motion to intervene may be filed at any


time before rendition of judgment by the trial court. A copy of the pleading-in-
intervention shall be attached to the motion and served on the original parties.

Thus, the following are the requisites for intervention of a non-party:


(1) Legal interest

(a) in the matter in controversy; or

(b) in the success of either of the parties; or

I against both parties; or

(d) person is so situated as to be adversely affected by a distribution or


other disposition of property in the custody of the court or of an officer
thereof;

80
(2) Intervention will not unduly delay or prejudice the adjudication of rights of
original parties;

(3) Intervenor’s rights may not be fully protected in a separate


proceeding[51] and

(g)The motion to intervene may be filed at any time before rendition of judgment
by the trial court.

For both the oil companies and DOE, the last requirement is definitely absent. As a

rule, intervention is allowed “before rendition of judgment” as Section 2, Rule 19 expressly

provides. Both filed their separate motions after our decision was promulgated. In Republic of

the Philippines v. Gingoyon,[52] a recently decided case which was also an original action filed

in this Court, we declared that the appropriate time to file the motions-in-intervention was

before and not after resolution of the case.[53]

The Court, however, has recognized exceptions to Section 2, Rule 19 in the interest of

substantial justice:

The rule on intervention, like all other rules of procedure, is intended to


make the powers of the Court fully and completely available for justice. It is aimed
to facilitate a comprehensive adjudication of rival claims overriding technicalities on
the timeliness of the filing thereof.[54]

81
The oil companies assert that they have a legal interest in this case because the

implementation of Ordinance No. 8027 will directly affect their business and property

rights.[55]

[T]he interest which entitles a person to intervene in a suit between other


parties must be in the matter in litigation and of such direct and immediate
character that the intervenor will either gain or lose by direct legal operation and
effect of the judgment. Otherwise, if persons not parties to the action were allowed
to intervene, proceedings would become unnecessarily complicated, expensive and
interminable. And this would be against the policy of the law. The words “an
interest in the subject” means a direct interest in the cause of action as pleaded,
one that would put the intervenor in a legal position to litigate a fact alleged in the
complaint without the establishment of which plaintiff could not recover.[56]

We agree that the oil companies have a direct and immediate interest in the

implementation of Ordinance No. 8027. Their claim is that they will need to spend billions of

pesos if they are compelled to relocate their oil depots out of Manila. Considering that they

admitted knowing about this case from the time of its filing on December 4, 2002, they should

have intervened long before our March 7, 2007 decision to protect their interests. But they did

not.[57] Neither did they offer any worthy explanation to justify their late intervention.

Be that as it may, although their motion for intervention was not filed on time, we will

allow it because they raised and presented novel issues and arguments that were not

considered by the Court in its March 7, 2007 decision. After all, the allowance or disallowance

of a motion to intervene is addressed to the sound discretion of the court before which the

case is pending.[58] Considering the compelling reasons favoring intervention, we do not think

that this will unduly delay or prejudice the adjudication of rights of the original parties. In

fact, it will be expedited since their intervention will enable us to rule on the constitutionality

of Ordinance No. 8027 instead of waiting for the RTC’s decision.

82
The DOE, on the other hand, alleges that its interest in this case is also direct and

immediate as Ordinance No. 8027 encroaches upon its exclusive and national authority over

matters affecting the oil industry. It seeks to intervene in order to represent the interests of the

members of the public who stand to suffer if the Pandacan Terminals’ operations are

discontinued. We will tackle the issue of the alleged encroachment into DOE’s domain later

on. Suffice it to say at this point that, for the purpose of hearing all sides and considering the

transcendental importance of this case, we will also allow DOE’s intervention.

THE INJUNCTIVE WRITS ARE NOT IMPEDIMENTS TO


THE ENFORCEMENT OF ORDINANCE NO. 8027

Under Rule 65, Section 3[59] of the Rules of Court, a petition for mandamus may be filed

when any tribunal, corporation, board, officer or person unlawfully neglects the performance of

an act which the law specifically enjoins as a duty resulting from an office, trust or

station. According to the oil companies, respondent did not unlawfully fail or neglect to enforce

Ordinance No. 8027 because he was lawfully prevented from doing so by virtue of the injunctive

writs and status quo order issued by the RTC of Manila, Branches 39 and 42.

First, we note that while Chevron and Shell still have in their favor the writs of preliminary

injunction and preliminary mandatory injunction, the status quo order in favor of Petron is no

longer in effect since the court granted the joint motion of the parties to withdraw the complaint

and counterclaim.[60]

83
Second, the original parties failed to inform the Court about these injunctive writs.

Respondent (who was also impleaded as a party in the RTC cases) defends himself by saying

that he informed the court of the pendency of the civil cases and that a TRO was issued by the

RTC in the consolidated cases filed by Chevron and Shell. It is true that had the oil companies

only intervened much earlier, the Court would not have been left in the dark about these facts.

Nevertheless, respondent should have updated the Court, by way of manifestation, on such a

relevant matter.

In his memorandum, respondent mentioned the issuance of a TRO. Under Section 5 of

Rule 58 of the Rules of Court, a TRO issued by the RTC is effective only for a period of 20 days.

This is why, in our March 7, 2007 decision, we presumed with certainty that this had already

lapsed.[61] Respondent also mentioned the grant of injunctive writs in his rejoinder which the

Court, however, expunged for being a prohibited pleading. The parties and their counsels were

clearly remiss in their duties to this Court.

In resolving controversies, courts can only consider facts and issues pleaded by the

parties.[62] Courts, as well as magistrates presiding over them are not omniscient. They can only

act on the facts and issues presented before them in appropriate pleadings. They may not even

substitute their own personal knowledge for evidence. Nor may they take notice of matters

except those expressly provided as subjects of mandatory judicial notice.

We now proceed to the issue of whether the injunctive writs are legal impediments to the

enforcement of Ordinance No. 8027.

84
Section 3, Rule 58 of the Rules of Court enumerates the grounds for the issuance of a writ

of preliminary injunction:

SEC. 3. Grounds for issuance of preliminary injunction. ― A preliminary injunction


may be granted when it is established:

(a) That the applicant is entitled to the relief demanded, and the whole or
part of such relief consists in restraining the commission or continuance of the
act or acts complained of, or in requiring the performance of an act or acts,
either for a limited period or perpetually;

(b) That the commission, continuance or nonperformance of the act or acts


complained of during the litigation would probably work injustice to the
applicant; or

(g) IThat a party, court, agency or a person is doing, threatening, or is


attempting to do, or is procuring or suffering to be done, some act or
acts probably in violation of the rights of the applicant respecting the
subject of the action or proceeding, and tending to render the judgment
ineffectual.

There are two requisites for the issuance of a preliminary injunction: (1) the right to be

protected exists prima facie and (2) the acts sought to be enjoined are violative of that right. It

must be proven that the violation sought to be prevented will cause an irreparable injustice.

The act sought to be restrained here was the enforcement of Ordinance No. 8027. It is a

settled rule that an ordinance enjoys the presumption of validity and, as such, cannot be

restrained by injunction.[63] Nevertheless, when the validity of the ordinance is assailed, the

85
courts are not precluded from issuing an injunctive writ against its enforcement. However, we

have declared that the issuance of said writ is proper only when:

... the petitioner assailing the ordinance has made out a case of unconstitutionality
strong enough to overcome, in the mind of the judge, the presumption of validity,
in addition to a showing of a clear legal right to the remedy sought....[64] (Emphasis
supplied)

Judge Reynaldo G. Ros, in his order dated May 19, 2003, stated his basis for issuing the

injunctive writs:

The Court, in resolving whether or not a Writ of Preliminary Injunction or


Preliminary Mandatory Injunction should be issued, is guided by the following
requirements: (1) a clear legal right of the complainant; (2) a violation of that
right; and (3) a permanent and urgent necessity for the Writ to prevent serious
damage. The Court believes that these requisites are present in these cases.

There is no doubt that the plaintiff/petitioners have been


legitimately operating their business in the Pandacan Terminal for many years and
they have made substantial capital investment therein. Every year they were
issued Business Permits by the City of Manila. Its operations have not been
declared illegal or contrary to law or morals. In fact, because of its vital importance
to the national economy, it was included in the Investment Priorities Plan as
mandated under the “Downstream Oil Industry Deregulation Act of 1988 (R.A.
8479). As a lawful business, the plaintiff/petitioners have a right, therefore, to
continue their operation in the Pandacan Terminal and the right to protect their
investments. This is a clear and unmistakable right of the plaintiff/petitioners.

The enactment, therefore, of City Ordinance No. 8027 passed by the City
Council of Manila reclassifying the area where the Pandacan Terminal is located
from Industrial II to Commercial I and requiring the plaintiff/petitioners to cease
and desist from the operation of their business has certainly violated the rights of
the plaintiff/petitioners to continue their legitimate business in the Pandacan
Terminal and deprived them of their huge investments they put up therein. Thus,
86
before the Court, therefore, determines whether the Ordinance in question is valid
or not, a Writ of Preliminary Injunction and a Writ of Mandatory Injunction be
issued to prevent serious and irreparable damage to plaintiff/petitioners.[65]

Nowhere in the judge’s discussion can we see that, in addition to a showing of a clear

legal right of Chevron and Shell to the remedy sought, he was convinced that they had made out

a case of unconstitutionality or invalidity strong enough to overcome the presumption of validity

of the ordinance. Statutes and ordinances are presumed valid unless and until the courts

declare the contrary in clear and unequivocal terms.[66] The mere fact that the ordinance is

alleged to be unconstitutional or invalid will not entitle a party to have its enforcement

enjoined.[67] The presumption is all in favor of validity. The reason for this is obvious:

The action of the elected representatives of the people cannot be lightly set aside.
The councilors must, in the very nature of things, be familiar with the necessities of
their particular municipality and with all the facts and circumstances which surround
the subject and necessitate action. The local legislative body, by enacting the
ordinance, has in effect given notice that the regulations are essential to the well
being of the people . . . The Judiciary should not lightly set aside legislative action
when there is not a clear invasion of personal or property rights under the guise of
police regulation.[68]

X — x — x

...[Courts] accord the presumption of constitutionality to legislative enactments, not


only because the legislature is presumed to abide by the Constitution but also
because the judiciary[,] in the determination of actual cases and controversies[,]
must reflect the wisdom and justice of the people as expressed through their
representatives in the executive and legislative departments of the government.[69]

87
The oil companies argue that this presumption must be set aside when the invalidity or

unreasonableness appears on the face of the ordinance itself.[70] We see no reason to set aside

the presumption. The ordinance, on its face, does not at all appear to be unconstitutional. It

reclassified the subject area from industrial to commercial. Prima facie, this power is within the

power of municipal corporations:

The power of municipal corporations to divide their territory into industrial,


commercial and residential zones is recognized in almost all jurisdictions inasmuch
as it is derived from the police power itself and is exercised for the protection and
benefit of their inhabitants.[71]

X — x — x

There can be no doubt that the City of Manila has the power to
divide its territory into residential and industrial zones, and to prescribe that
offensive and unwholesome trades and occupations are to be established
exclusively in the latter zone.

Xxx xxx xxx

Likewise, it cannot be denied that the City of Manila has the


authority, derived from the police power, of forbidding the appellant to continue the
manufacture of toyoin the zone where it is now situated, which has been declared
residential....[72]

Courts will not invalidate an ordinance unless it clearly appears that it is unconstitutional.

There is no such showing here. Therefore, the injunctive writs issued in the Manila RTC’s May

19, 2003 order had no leg to stand on.

88
We are aware that the issuance of these injunctive writs is not being assailed as tainted

with grave abuse of discretion. However, we are confronted with the question of whether these

writs issued by a lower court are impediments to the enforcement of Ordinance No. 8027 (which

is the subject of the mandamus petition). As already discussed, we rule in the negative.

ORDINANCE NO. 8027 WAS NOT SUPERSEDED BY


ORDINANCE NO. 8119

The March 7, 2007 decision did not take into consideration the passage of Ordinance No.

8119 entitled “An Ordinance Adopting the Manila Comprehensive Land Use Plan and Zoning

Regulations of 2006 and Providing for the Administration, Enforcement and Amendment thereto”

which was approved by respondent on June 16, 2006. The simple reason was that the Court

was never informed about this ordinance.

While courts are required to take judicial notice of the laws enacted by Congress, the rule

with respect to local ordinances is different. Ordinances are not included in the enumeration of

matters covered by mandatory judicial notice under Section 1, Rule 129 of the Rules of Court.[73]

Although, Section 50 of RA 409[74] provides that:

SEC. 50 Judicial notice of ordinances. - All courts sitting in the city shall
take judicial notice of the ordinances passed by the [Sangguniang Panglungsod].

89
This cannot be taken to mean that this Court, since it has its seat in the City of Manila, should

have taken steps to procure a copy of the ordinance on its own, relieving the party of any duty

to inform the Court about it.

Even where there is a statute that requires a court to take judicial notice of municipal

ordinances, a court is not required to take judicial notice of ordinances that are not before it and

to which it does not have access. The party asking the court to take judicial notice is obligated to

supply the court with the full text of the rules the party desires it to have notice of.[75] Counsel

should take the initiative in requesting that a trial court take judicial notice of an ordinance even

where a statute requires courts to take judicial notice of local ordinances.[76]

The intent of a statute requiring a court to take judicial notice of a local ordinance is to

remove any discretion a court might have in determining whether or not to take notice of an

ordinance. Such a statute does not direct the court to act on its own in obtaining evidence for

the record and a party must make the ordinance available to the court for it to take notice.[77]

In its defense, respondent claimed that he did not inform the Court about the enactment of

Ordinance No. 8119 because he believed that it was different from Ordinance No. 8027 and that

the two were not inconsistent with each other.[78]

In the same way that we deem the intervenors’ late intervention in this case unjustified,

we find the failure of respondent, who was an original party here, inexcusable.

THE RULE ON JUDICIAL ADMISSIONS IS NOT


APPLICABLE AGAINST RESPONDENT

90
The oil companies assert that respondent judicially admitted that Ordinance No. 8027 was

repealed by Ordinance No. 8119 in civil case no. 03-106379 (where Petron assailed the

constitutionality of Ordinance No. 8027) when the parties in their joint motion to withdraw

complaint and counterclaim stated that “the issue ...has been rendered moot and academic by

virtue of the passage of [Ordinance No. 8119].”[79] They contend that such admission worked as

an estoppel against the respondent.

Respondent countered that this stipulation simply meant that Petron was recognizing the

validity and legality of Ordinance No. 8027 and that it had conceded the issue of said ordinance’s

constitutionality, opting instead to question the validity of Ordinance No. 8119.[80] The oil

companies deny this and further argue that respondent, in his answer in civil case no. 06-

115334 (where Chevron and Shell are asking for the nullification of Ordinance No. 8119),

expressly stated that Ordinance No. 8119 replaced Ordinance No. 8027:[81]

... Under Ordinance No. 8027, businesses whose uses are not in accord with the
reclassification were given six months to cease [their] operation. Ordinance No.
8119, which in effect, replaced Ordinance [No.] 8027, merely took note of the time
frame provided for in Ordinance No. 8119.... Ordinance No. 8119 thus provided for
an even longer term, that is[,] seven years;[82] (Emphasis supplied)

Rule 129, Section 4 of the Rules of Court provides:

Section 4. Judicial admissions. ― An admission, verbal or written, made by a


party in the course of the proceedings in the same case, does not require proof. The
admission may be contradicted only by showing that it was made through palpable
mistake or that no such admission was made. (Emphasis supplied)

91
While it is true that a party making a judicial admission cannot subsequently take a

position contrary to or inconsistent with what was pleaded,[83] the aforestated rule is not

applicable here. Respondent made the statements regarding the ordinances in civil case nos. 03-

106379 and 06-115334 which are not “the same” as this case before us.[84] To constitute a

judicial admission, the admission must be made in the same case in which it is offered.

Hence, respondent is not estopped from claiming that Ordinance No. 8119 did not

supersede Ordinance No. 8027. On the contrary, it is the oil companies which should be

considered estopped. They rely on the argument that Ordinance No. 8119 superseded Ordinance

No. 8027 but, at the same time, also impugn its (8119’s) validity. We frown on the adoption of

inconsistent positions and distrust any attempt at clever positioning under one or the other on

the basis of what appears advantageous at the moment. Parties cannot take vacillating or

contrary positions regarding the validity of a statute[85] or ordinance. Nonetheless, we will look

into the merits of the argument of implied repeal.

ORDINANCE NO. 8119 DID NOT IMPLIEDLY REPEAL


ORDINANCE NO. 8027

Both the oil companies and DOE argue that Ordinance No. 8119 repealed Ordinance No.

8027. They assert that although there was no express repeal[86] of Ordinance No. 8027,

Ordinance No. 8119 impliedly repealed it.

According to the oil companies, Ordinance No. 8119 reclassified the area covering the

Pandacan Terminals to “High Density Residential/Mixed Use Zone (R-3/MXD)”[87] whereas

Ordinance No. 8027 reclassified the same area from Industrial II to Commercial I:
92
SECTION 1. For the purpose of promoting sound urban planning and ensuring
health, public safety, and general welfare of the residents of Pandacan and Sta. Ana
as well as its adjoining areas, the land use of [those] portions of land bounded by
the Pasig River in the north, PNR Railroad Track in the east, Beata St. in the south,
Palumpong St. in the southwest, and Estero de Pancacan in the west[,] PNR
Railroad in the northwest area, Estero de Pandacan in the [n]ortheast, Pasig River
in the southeast and Dr. M.L. Carreon in the southwest. The area of Punta, Sta.
Ana bounded by the Pasig River, Marcelino Obrero St., Mayo 28 St., and F. Manalo
Street, are hereby reclassified from Industrial II to Commercial I. (Emphasis
supplied)

Moreover, Ordinance No. 8119 provides for a phase-out of seven years:

SEC. 72. Existing Non-Conforming Uses and Buildings. - The lawful use of any
building, structure or land at the time of the adoption of this Ordinance may be
continued, although such use does not conform with the provision of the Ordinance,
provided:

xxx xxx xxx

(g) In case the non-conforming use is an industrial use:

xxx xxx xxx

d. The land use classified as non-conforming shall program the phase-out


and relocation of the non-conforming use within seven (7) years from the
date of effectivity of this Ordinance. (Emphasis supplied)

This is opposed to Ordinance No. 8027 which compels affected entities to vacate the area within

six months from the effectivity of the ordinance:

93
SEC. 3. Owners or operators of industries and other businesses, the operation of
which are no longer permitted under Section 1 hereof, are hereby given a period of
six (6) months from the date of effectivity of this Ordinance within which to cease
and desist from the operation of businesses which are hereby in consequence,
disallowed.

Ordinance No. 8119 also designated the Pandacan oil depot area as a “Planned Unit

Development/Overlay Zone (O-PUD)”:

SEC. 23. Use Regulations in Planned Unit Development/Overlay Zone (O-PUD). –


O-PUD Zones are identified specific sites in the City of Manila wherein the project
site is comprehensively planned as an entity via unitary site plan which permits
flexibility in planning/ design, building siting, complementarily of building types and
land uses, usable open spaces and the preservation of significant natural land
features, pursuant to regulations specified for each particular PUD. Enumerated
below are identified PUD:

xxx xxx xxx

6. Pandacan Oil Depot Area

xxx xxx xxx

Enumerated below are the allowable uses:

1. all uses allowed in all zones where it is located

2. the [Land Use Intensity Control (LUIC)] under which zones are
located shall, in all instances be complied with

94
3. the validity of the prescribed LUIC shall only be [superseded] by the
development controls and regulations specified for each PUD as provided for
each PUD as provided for by the masterplan of respective
PUDs.[88] (Emphasis supplied)

Respondent claims that in passing Ordinance No. 8119, the Sanggunian did not intend to

repeal Ordinance No. 8027 but meant instead to carry over 8027’s provisions to 8119 for the

purpose of making Ordinance No. 8027 applicable to the oil companies even after the passage of

Ordinance No. 8119.[89] He quotes an excerpt from the minutes of the July 27, 2004 session of

the Sanggunian during the first reading of Ordinance No. 8119:

Member GARCIA: Your Honor, iyong patungkol po roon sa oil depot doon
sa amin sa Sixth District sa Pandacan, wala pong nakalagay eith sa ordinansa rito
na taliwas o kakaiba roon sa ordinansang ipinasa noong nakaraang Konseho, iyong
Ordinance No. 8027. So kung ano po ang nandirito sa ordinansa na ipinasa ninyo
last time, iyon lang po ang ni-lift eithe at inilagay eith. At eith eith ordinansang
…iyong naipasa ng huling Konseho, niri-classify [ninyo] from Industrial II to
Commercial C-1 ang area ng Pandacan kung nasaan ang oil depot. So ini-lift lang
po [eithe] iyong definition, density, at saka po yon pong … ng… noong ordinansa
ninyo na siya eith naming inilagay eith, iniba lang po naming iyong title. So wala
po kaming binago na taliwas o nailagay na taliwas doon sa ordinansang ipinasa
ninyo, ni-lift lang po [eithe] from Ordinance No. 8027.”[90] (Emphasis supplied)

We agree with respondent.

Repeal by implication proceeds on the premise that where a statute of later date clearly

reveals the intention of the legislature to abrogate a prior act on the subject, that intention must

be given effect.[91]

95
There are two kinds of implied repeal. The first is: where the provisions in the two acts on

the same subject matter are irreconcilably contradictory, the latter act, to the extent of the

conflict, constitutes an implied repeal of the earlier one.[92] The second is: if the later act covers

the whole subject of the earlier one and is clearly intended as a substitute, it will operate to

repeal the earlier law.[93] The oil companies argue that the situation here falls under the first

category.

Implied repeals are not favored and will not be so declared unless the intent of the

legislators is manifest.[94] As statutes and ordinances are presumed to be passed only after

careful deliberation and with knowledge of all existing ones on the subject, it follows that, in

passing a law, the legislature did not intend to interfere with or abrogate a former law relating to

the same subject matter.[95] If the intent to repeal is not clear, the later act should be construed

as a continuation of, and not a substitute for, the earlier act.[96]

These standards are deeply enshrined in our jurisprudence. We disagree that, in enacting

Ordinance No. 8119, there was any indication of the legislative purpose to repeal Ordinance No.

8027.[97] The excerpt quoted above is proof that there was never such an intent. While it is true

that both ordinances relate to the same subject matter, i.e. classification of the land use of the

area where Pandacan oil depot is located, if there is no intent to repeal the earlier enactment,

every effort at reasonable construction must be made to reconcile the ordinances so that both

can be given effect:

The fact that a later enactment may relate to the same subject matter as
that of an earlier statute is not of itself sufficient to cause an implied repeal of the
prior act, since the new statute may merely be cumulative or a continuation of the
old one. What is necessary is a manifest indication of legislative purpose to
repeal.[98]

96
For the first kind of implied repeal, there must be an irreconcilable conflict between the

two ordinances. There is no conflict between the two ordinances. Ordinance No. 8027

reclassified the Pandacan area from Industrial II to Commercial I. Ordinance No. 8119, in

Section 23, designated it as a “Planned Unit Development/Overlay Zone (O-PUD).” In its Annex

C which defined the zone boundaries,[99] the Pandacan area was shown to be within the “High

Density Residential/Mixed Use Zone (R-3/MXD).” These zone classifications in Ordinance No.

8119 are not inconsistent with the reclassification of the Pandacan area from Industrial to

Commercial in Ordinance No. 8027. The “O-PUD” classification merely made Pandacan a “project

site ... comprehensively planned as an entity via unitary site plan which permits flexibility in

planning/design, building siting, complementarity of building types and land uses, usable open

spaces and the preservation of significant natural land features....”[100] Its classification as “R-

3/MXD” means that it should “be used primarily for high-rise housing/dwelling purposes and

limited complementary/supplementary trade, services and business activities.”[101] There is no

conflict since both ordinances actually have a common objective, i.e., to shift the zoning

classification from industrial to commercial (Ordinance No. 8027) or mixed

residential/commercial (Ordinance No. 8119).

Moreover, it is a well-settled rule in statutory construction that a subsequent general law

does not repeal a prior special law on the same subject unless it clearly appears that the

legislature has intended by the latter general act to modify or repeal the earlier special

law.Generalia specialibus non derogant (a general law does not nullify a specific or special

law).[102] This is so even if the provisions of the general law are sufficiently comprehensive to

include what was set forth in the special act.[103] The special act and the general law must stand

together, one as the law of the particular subject and the other as the law of general

application.[104] The special law must be taken as intended to constitute an exception to, or a

qualification of, the general act or provision.[105]


97
The reason for this is that the legislature, in passing a law of special character,
considers and makes special provisions for the particular circumstances dealt with
by the special law. This being so, the legislature, by adopting a general law
containing provisions repugnant to those of the special law and without making any
mention of its intention to amend or modify such special law, cannot be deemed to
have intended an amendment, repeal or modification of the latter.[106]

Ordinance No. 8027 is a special law[107] since it deals specifically with a certain area

described therein (the Pandacan oil depot area) whereas Ordinance No. 8119 can be considered

a general law[108] as it covers the entire city of Manila.

The oil companies assert that even if Ordinance No. 8027 is a special law, the existence of

an all-encompassing repealing clause in Ordinance No. 8119 evinces an intent on the part of

the Sanggunian to repeal the earlier ordinance:

Sec. 84. Repealing Clause. – All ordinances, rules, regulations in conflict with the
provisions of this Ordinance are hereby repealed; PROVIDED, That the rights that
are vested upon the effectivity of this Ordinance shall not be impaired.

They cited Hospicio de San Jose de Barili, Cebu City v. Department of Agrarian Reform:[109]

The presence of such general repealing clause in a later statute clearly indicates the
legislative intent to repeal all prior inconsistent laws on the subject matter, whether
the prior law is a general law or a special law... Without such a clause, a later
general law will ordinarily not repeal a prior special law on the same subject. But
with such clause contained in the subsequent general law, the prior special law will

98
be deemed repealed, as the clause is a clear legislative intent to bring about that
result.[110]

This ruling in not applicable here. The repealing clause of Ordinance No. 8119 cannot be

taken to indicate the legislative intent to repeal all prior inconsistent laws on the subject matter,

including Ordinance No. 8027, a special enactment, since the aforequoted minutes (an official

record of the discussions in the Sanggunian) actually indicated the clear intent to preserve the

provisions of Ordinance No. 8027.

To summarize, the conflict between the two ordinances is more apparent than real. The

two ordinances can be reconciled. Ordinance No. 8027 is applicable to the area particularly

described therein whereas Ordinance No. 8119 is applicable to the entire City of Manila.

MANDAMUS LIES TO COMPEL RESPONDENT MAYOR TO ENFORCE


ORDINANCE NO. 8027

The oil companies insist that mandamus does not lie against respondent in

consideration of the separation of powers of the executive and judiciary.[111] This argument

is misplaced. Indeed,

[the] Courts will not interfere by mandamus proceedings with the legislative [or
executive departments] of the government in the legitimate exercise of its
powers, except to enforce mere ministerial acts required by law to be performed by
some officer thereof.[112] (Emphasis Supplied)

99
since this is the function of a writ of mandamus, which is the power to compel “the

performance of an act which the law specifically enjoins as a duty resulting from office, trust

or station.”[113]

They also argue that petitioners had a plain, speedy and adequate remedy to compel

respondent to enforce Ordinance No. 8027 which was to seek relief from the President of the

Philippines through the Secretary of the Department of Interior and Local Government (DILG)

by virtue of the President’s power of supervision over local government units. Again, we

disagree. A party need not go first to the DILG in order to compel the enforcement of an

ordinance. This suggested process would be unreasonably long, tedious and consequently

injurious to the interests of the local government unit (LGU) and its constituents whose

welfare is sought to be protected. Besides, petitioners’ resort to an original action

for mandamus before this Court is undeniably allowed by the Constitution.[114]

ORDINANCE NO. 8027 IS CONSTITUTIONAL AND


VALID

Having ruled that there is no impediment to the enforcement of Ordinance No. 8027,

we now proceed to make a definitive ruling on its constitutionality and validity.

The tests of a valid ordinance are well established. For an ordinance to be valid, it must

not only be within the corporate powers of the LGU to enact and be passed according to the

procedure prescribed by law, it must also conform to the following substantive requirements:

(1) must not contravene the Constitution or any statute; (2) must not be unfair or

oppressive; (3) must not be partial or discriminatory; (4) must not prohibit but may regulate

trade; (5) must be general and consistent with public policy and (6) must not be

unreasonable.[115]
100
THE CITY OF MANILA HAS THE POWER TO ENACT
ORDINANCE NO. 8027

Ordinance No. 8027 was passed by the Sangguniang Panlungsod of Manila in the

exercise of its police power. Police power is the plenary power vested in the legislature to

make statutes and ordinances to promote the health, morals, peace, education, good order

or safety and general welfare of the people.[116] This power flows from the recognition

that salus populi est suprema lex (the welfare of the people is the supreme law).[117] While

police power rests primarily with the national legislature, such power may be

delegated.[118] Section 16 of the LGC, known as the general welfare clause, encapsulates the

delegated police power to local governments:[119]

Section 16. General Welfare. ― Every local government unit shall exercise the
powers expressly granted, those necessarily implied therefrom, as well as powers
necessary, appropriate, or incidental for its efficient and effective governance, and
those which are essential to the promotion of the general welfare. Within their
respective territorial jurisdictions, local government units shall ensure and support,
among other things, the preservation and enrichment of culture, promote health
and safety, enhance the right of the people to a balanced ecology, encourage and
support the development of appropriate and self-reliant scientific and technological
capabilities, improve public morals, enhance economic prosperity and social justice,
promote full employment among their residents, maintain peace and order, and
preserve the comfort and convenience of their inhabitants.

101
LGUs like the City of Manila exercise police power through their respective legislative bodies, in

this case, the Sangguniang Panlungsod or the city council. Specifically, the Sanggunian can

enact ordinances for the general welfare of the city:

Section. 458. – Powers, Duties, Functions and Compensation. – (a)


The sangguniang panglungsod, as the legislative branch of the city, shall enact
ordinances, approve resolutions and appropriate funds for the general welfare of the
city and its inhabitants pursuant to Section 16 of this Code xxxx

This police power was also provided for in RA 409 or the Revised Charter of the City of Manila:

Section 18. Legislative powers. — The [City Council] shall have the following
legislative powers:

xxx xxx xxx

(g) To enact all ordinances it may deem necessary and proper for the
sanitation and safety, the furtherance of the prosperity, and the promotion of
the morality, peace, good order, comfort, convenience, and general welfare of
the city and its inhabitants, and such others as may be necessary to carry into
effect and discharge the powers and duties conferred by this chapter xxxx[120]

Specifically, the Sanggunian has the power to “reclassify land within the jurisdiction of
the city.”[121]

THE ENACTMENT OF ORDINANCE NO. 8027 IS A


LEGITIMATE EXERCISE OF POLICE POWER

102
As with the State, local governments may be considered as having properly exercised

their police power only if the following requisites are met: (1) the interests of the public

generally, as distinguished from those of a particular class, require its exercise and (2) the

means employed are reasonably necessary for the accomplishment of the purpose and not

unduly oppressive upon individuals. In short, there must be a concurrence of a lawful subject

and a lawful method.[122]

Ordinance No. 8027 was enacted “for the purpose of promoting sound urban planning,
ensuring health, public safety and general welfare”[123] of the residents of
Manila. The Sanggunian was impelled to take measures to protect the residents of Manila from
catastrophic devastation in case of a terrorist attack on the Pandacan Terminals. Towards this
objective, the Sanggunian reclassified the area defined in the ordinance from industrial to
commercial.

The following facts were found by the Committee on Housing, Resettlement and Urban
Development of the City of Manila which recommended the approval of the ordinance:

(1) the depot facilities contained 313.5 million liters of highly flammable and highly
volatile products which include petroleum gas, liquefied petroleum gas, aviation
fuel, diesel, gasoline, kerosene and fuel oil among others;
(2) the depot is open to attack through land, water or air;
(3) it is situated in a densely populated place and near Malacañang Palace and
(4) in case of an explosion or conflagration in the depot, the fire could spread to the
neighboring communities.[124]

The ordinance was intended to safeguard the rights to life, security and safety of all the
inhabitants of Manila and not just of a particular class.[125] The depot is perceived, rightly or
wrongly, as a representation of western interests which means that it is a terrorist target. As
long as it there is such a target in their midst, the residents of Manila are not safe. It therefore
became necessary to remove these terminals to dissipate the threat. According to respondent:

Such a public need became apparent after the 9/11 incident which showed that
what was perceived to be impossible to happen, to the most powerful country in the

103
world at that, is actually possible. The destruction of property and the loss of
thousands of lives on that fateful day became the impetus for a public need. In the
aftermath of the 9/11 tragedy, the threats of terrorism continued [such] that it
became imperative for governments to take measures to combat their effects.[126]

Wide discretion is vested on the legislative authority to determine not only what the

interests of the public require but also what measures are necessary for the protection of such

interests.[127] Clearly, the Sanggunian was in the best position to determine the needs of its

constituents.

In the exercise of police power, property rights of individuals may be subjected to

restraints and burdens in order to fulfill the objectives of the government.[128] Otherwise stated,

the government may enact legislation that may interfere with personal liberty, property, lawful

businesses and occupations to promote the general welfare.[129] However, the interference must

be reasonable and not arbitrary. And to forestall arbitrariness, the methods or means used to

protect public health, morals, safety or welfare must have a reasonable relation to the end in

view.[130]

The means adopted by the Sanggunian was the enactment of a zoning ordinance which

reclassified the area where the depot is situated from industrial to commercial. A zoning

ordinance is defined as a local city or municipal legislation which logically arranges, prescribes,

defines and apportions a given political subdivision into specific land uses as present and future

projection of needs.[131] As a result of the zoning, the continued operation of the businesses of

the oil companies in their present location will no longer be permitted. The power to establish

zones for industrial, commercial and residential uses is derived from the police power itself and

is exercised for the protection and benefit of the residents of a locality.[132] Consequently, the

104
enactment of Ordinance No. 8027 is within the power of theSangguniang Panlungsod of the City

of Manila and any resulting burden on those affected cannot be said to be unjust:

There can be no doubt that the City of Manila has the power to divide its territory
into residential and industrial zones, and to prescribe that offensive and
unwholesome trades and occupations are to be established exclusively in the latter
zone.

“The benefits to be derived by cities adopting such regulations (zoning)


may be summarized as follows: They attract a desirable and assure a permanent
citizenship; they foster pride in and attachment to the city; they promote happiness
and contentment; they stabilize the use and value of property and promote the
peace, [tranquility], and good order of the city. We do not hesitate to say that the
attainment of these objects affords a legitimate field for the exercise of the police
power. He who owns property in such a district is not deprived of its use by such
regulations. He may use it for the purposes to which the section in which it is
located is dedicated. That he shall not be permitted to use it to the desecration of
the community constitutes no unreasonable or permanent hardship and results in
no unjust burden.”

Xxx xxx xxx

“The 14th Amendment protects the citizen in his right to engage in any
lawful business, but it does not prevent legislation intended to regulate useful
occupations which, because of their nature or location, may prove injurious or
offensive to the public.”[133]

We entertain no doubt that Ordinance No. 8027 is a valid police power measure because

there is a concurrence of lawful subject and lawful method.

105
ORDINANCE NO. 8027 IS NOT UNFAIR, OPPRESSIVE OR
CONFISCATORY WHICH AMOUNTS TO TAKING WITHOUT
COMPENSATION

According to the oil companies, Ordinance No. 8027 is unfair and oppressive as it does

not only regulate but also absolutely prohibits them from conducting operations in the City of

Manila. Respondent counters that this is not accurate since the ordinance merely prohibits the

oil companies from operating their businesses in the Pandacan area.

Indeed, the ordinance expressly delineated in its title and in Section 1 what it pertained

to. Therefore, the oil companies’ contention is not supported by the text of the

ordinance. Respondent succinctly stated that:

The oil companies are not forbidden to do business in the City of Manila. They may
still very well do so, except that their oil storage facilities are no longer allowed in
the Pandacan area. Certainly, there are other places in the City of Manila where
they can conduct this specific kind of business. Ordinance No. 8027 did not render
the oil companies illegal. The assailed ordinance affects the oil companies business
only in so far as the Pandacan area is concerned.[134]

The oil companies are not prohibited from doing business in other appropriate zones in

Manila. The City of Manila merely exercised its power to regulate the businesses and industries

in the zones it established:

As to the contention that the power to regulate does not include the power to
prohibit, it will be seen that the ordinance copied above does not prohibit the
installation of motor engines within the municipality of Cabanatuan but only within
the zone therein fixed. If the municipal council of Cabanatuan is authorized to
establish said zone, it is also authorized to provide what kind of engines may be
106
installed therein. In banning the installation in said zone of all engines not excepted
in the ordinance, the municipal council of Cabanatuan did no more than regulate
their installation by means of zonification.[135]

The oil companies aver that the ordinance is unfair and oppressive because they have

invested billions of pesos in the depot.[136] Its forced closure will result in huge losses in income

and tremendous costs in constructing new facilities.

Their contention has no merit. In the exercise of police power, there is a limitation on or

restriction of property interests to promote public welfare which involves no compensable

taking. Compensation is necessary only when the state’s power of eminent domain is

exercised. In eminent domain, property is appropriated and applied to some public purpose.

Property condemned under the exercise of police power, on the other hand, is noxious or

intended for a noxious or forbidden purpose and, consequently, is not compensable.[137]The

restriction imposed to protect lives, public health and safety from danger is not a taking. It is

merely the prohibition or abatement of a noxious use which interferes with paramount rights of

the public.

Property has not only an individual function, insofar as it has to provide for the needs of

the owner, but also a social function insofar as it has to provide for the needs of the other

members of society.[138] The principle is this:

Police power proceeds from the principle that every holder of property, however
absolute and unqualified may be his title, holds it under the implied liability that his
use of it shall not be injurious to the equal enjoyment of others having an equal
right to the enjoyment of their property, nor injurious to the right of the
community. Rights of property, like all other social and conventional rights, are
subject to reasonable limitations in their enjoyment as shall prevent them from
107
being injurious, and to such reasonable restraints and regulations established by
law as the legislature, under the governing and controlling power vested in them by
the constitution, may think necessary and expedient.[139]

In the regulation of the use of the property, nobody else acquires the use or interest

therein, hence there is no compensable taking.[140] In this case, the properties of the oil

companies and other businesses situated in the affected area remain theirs. Only their use is

restricted although they can be applied to other profitable uses permitted in the commercial

zone.

ORDINANCE NO. 8027 IS NOT

PARTIAL AND DISCRIMINATORY

The oil companies take the position that the ordinance has discriminated against and

singled out the Pandacan Terminals despite the fact that the Pandacan area is congested with

buildings and residences that do not comply with the National Building Code, Fire Code and

Health and Sanitation Code.[141]

This issue should not detain us for long. An ordinance based on reasonable classification

does not violate the constitutional guaranty of the equal protection of the law.[142] The

requirements for a valid and reasonable classification are: (1) it must rest on substantial

distinctions; (2) it must be germane to the purpose of the law; (3) it must not be limited to

existing conditions only and (4) it must apply equally to all members of the same class.[143]
108
The law may treat and regulate one class differently from another class provided there are

real and substantial differences to distinguish one class from another.[144] Here, there is a

reasonable classification. We reiterate that what the ordinance seeks to prevent is a

catastrophic devastation that will result from a terrorist attack. Unlike the depot, the

surrounding community is not a high-value terrorist target. Any damage caused by fire or

explosion occurring in those areas would be nothing compared to the damage caused by a fire or

explosion in the depot itself. Accordingly, there is a substantial distinction. The enactment of the

ordinance which provides for the cessation of the operations of these terminals removes the

threat they pose. Therefore it is germane to the purpose of the ordinance. The classification is

not limited to the conditions existing when the ordinance was enacted but to future conditions as

well. Finally, the ordinance is applicable to all businesses and industries in the area it delineated.

ORDINANCE NO. 8027 IS NOT INCONSISTENT WITH RA


7638 AND RA 8479

The oil companies and the DOE assert that Ordinance No. 8027 is unconstitutional

because it contravenes RA 7638 (DOE Act of 1992)[145] and RA 8479 (Downstream Oil Industry

Deregulation Law of 1998).[146] They argue that through RA 7638, the national legislature

declared it a policy of the state “to ensure a continuous, adequate, and economic supply of

energy”[147] and created the DOE to implement this policy. Thus, under Section 5 I, DOE is

empowered to “establish and administer programs for the exploration, transportation,

marketing, distribution, utilization, conservation, stockpiling, and storage of energy

resources.” Considering that the petroleum products contained in the Pandacan Terminals are

major and critical energy resources, they conclude that their administration, storage, distribution

109
and transport are of national interest and fall under DOE’s primary and exclusive

jurisdiction.[148]

They further assert that the terminals are necessary for the delivery of immediate and

adequate supply of oil to its recipients in the most economical way.[149] Local legislation such as

Ordinance No. 8027 (which effectively calls for the removal of these terminals) allegedly

frustrates the state policy of ensuring a continuous, adequate, and economic supply of energy

expressed in RA 7638, a national law.[150] Likewise, the ordinance thwarts the determination of

the DOE that the terminals’ operations should be merely scaled down and not

discontinued.[151] They insist that this should not be allowed considering that it has a nationwide

economic impact and affects public interest transcending the territorial jurisdiction of the City of

Manila.[152]

According to them, the DOE’s supervision over the oil industry under RA 7638 was

subsequently underscored by RA 8479, particularly in Section 7 thereof:

SECTION 7. Promotion of Fair Trade Practices. ― The Department of Trade and


Industry (DTI) and DOE shall take all measures to promote fair trade and prevent
cartelization, monopolies, combinations in restraint of trade, and any unfair
competition in the Industry as defined in Article 186 of the Revised Penal Code, and
Articles 168 and 169 of Republic Act No. 8293, otherwise known as the “Intellectual
Property Rights Law”. The DOE shall continue to encourage certain practices in the
Industry which serve the public interest and are intended to achieve efficiency and
cost reduction, ensure continuous supply of petroleum products, and enhance
environmental protection. These practices may include borrow-and-loan
agreements, rationalized depot and manufacturing operations, hospitality
agreements, joint tanker and pipeline utilization, and joint actions on oil spill control
and fire prevention. (Emphasis supplied)

110
Respondent counters that DOE’s regulatory power does not preclude LGUs from

exercising their police power.[153]

Indeed, ordinances should not contravene existing statutes enacted by Congress. The

rationale for this was clearly explained inMagtajas vs. Pryce Properties Corp., Inc.:[154]

The rationale of the requirement that the ordinances should not contravene
a statute is obvious. Municipal governments are only agents of the national
government. Local councils exercise only delegated legislative powers conferred on
them by Congress as the national lawmaking body. The delegate cannot be superior
to the principal or exercise powers higher than those of the latter. It is a heresy to
suggest that the local government units can undo the acts of Congress, from which
they have derived their power in the first place, and negate by mere ordinance the
mandate of the statute.

“Municipal corporations owe their origin to, and derive their powers and
rights wholly from the legislature. It breathes into them the breath of life, without
which they cannot exist. As it creates, so it may destroy. As it may destroy, it may
abridge and control. Unless there is some constitutional limitation on the right, the
legislature might, by a single act, and if we can suppose it capable of so great a folly
and so great a wrong, sweep from existence all of the municipal corporations in the
State, and the corporation could not prevent it. We know of no limitation on the right
so far as to the corporation themselves are concerned. They are, so to phrase it, the
mere tenants at will of the legislature.”

This basic relationship between the national legislature and the local
government units has not been enfeebled by the new provisions in the Constitution
strengthening the policy of local autonomy. Without meaning to detract from that
policy, we here confirm that Congress retains control of the local government units
although in significantly reduced degree now than under our previous Constitutions.
The power to create still includes the power to destroy. The power to grant still
includes the power to withhold or recall. True, there are certain notable innovations
in the Constitution, like the direct conferment on the local government units of the
power to tax, which cannot now be withdrawn by mere statute. By and large,

111
however, the national legislature is still the principal of the local government units,
which cannot defy its will or modify or violate it.[155]

The question now is whether Ordinance No. 8027 contravenes RA 7638 and RA

8479. It does not.

Under Section 5 I of RA 7638, DOE was given the power to “establish and administer

programs for the exploration, transportation, marketing, distribution, utilization, conservation,

stockpiling, and storage of energy resources.” On the other hand, under Section 7 of RA 8749,

the DOE “shall continue to encourage certain practices in the Industry which serve the public

interest and are intended to achieve efficiency and cost reduction, ensure continuous supply of

petroleum products.” Nothing in these statutes prohibits the City of Manila from enacting

ordinances in the exercise of its police power.

The principle of local autonomy is enshrined in and zealously protected under the

Constitution. In Article II, Section 25 thereof, the people expressly adopted the following policy:

Section 25. The State shall ensure the autonomy of local governments.

An entire article (Article X) of the Constitution has been devoted to guaranteeing and promoting
the autonomy of LGUs. The LGC was specially promulgated by Congress to ensure the
autonomy of local governments as mandated by the Constitution:

Sec. 2. Declaration of Policy. ― (a) It is hereby declared the policy of the State that
the territorial and political subdivisions of the State shall enjoy genuine and
meaningful local autonomy to enable them to attain their fullest development as
self-reliant communities and make them more effective partners in the attainment of
national goals. Toward this end, the State shall provide for a more responsive and
accountable local government structure instituted through a system of
decentralization whereby local government units shall be given more powers,
authority, responsibilities, and resources. The process of decentralization shall
112
proceed from the National Government to the local government units. (Emphasis
supplied)

We do not see how the laws relied upon by the oil companies and DOE
stripped the City of Manila of its power to enact ordinances in the exercise of its
police power and to reclassify the land uses within its jurisdiction. To guide us, we
shall make a brief survey of our decisions where the police power measure of the
LGU clashed with national laws.

In Tan v. Pereña,[156] the Court ruled that Ordinance No. 7 enacted by the municipality of
Daanbantayan, Cebu allowing the operation of three cockpits was invalid for violating PD 449 (or
the Cockfighting Law of 1974) which permitted only one cockpit per municipality.

In Batangas CATV, Inc. v. Court of Appeals,[157] the Sangguniang Panlungsod of Batangas


City enacted Resolution No. 210 granting Batangas CATV, Inc. a permit to operate a cable
television (CATV) system in Batangas City. The Court held that the LGU did not have the
authority to grant franchises to operate a CATV system because it was the National
Telecommunications Commission (NTC) that had the power under EO Nos. 205 and 436 to
regulate CATV operations. EO 205 mandated the NTC to grant certificates of authority to CATV
operators while EO 436 vested on the NTC the power to regulate and supervise the CATV
industry.

In Lina, Jr. v. Paño,[158] we held that Kapasiyahan Bilang 508, Taon 1995 of
the Sangguniang Panlalawigan of Laguna could not be used as justification to prohibit lotto in the
municipality of San Pedro, Laguna because lotto was duly authorized by RA 1169, as amended
by BP 42. This law granted a franchise to the Philippine Charity Sweepstakes Office and allowed
it to operate lotteries.

In Magtajas v. Pryce Properties Corp., Inc.,[159] the Sangguniang Panlungsod of Cagayan


de Oro City passed Ordinance Nos. 3353 and 3375-93 prohibiting the operation of casinos in the
city. We ruled that these ordinances were void for contravening PD 1869 or the charter of the
Philippine Amusements and Gaming Corporation which had the power to operate casinos.

The common dominator of all of these cases is that the national laws were clearly and
expressly in conflict with the ordinances/resolutions of the LGUs. The inconsistencies were so
patent that there was no room for doubt. This is not the case here.

The laws cited merely gave DOE general powers to “establish and administer programs for the
exploration, transportation, marketing, distribution, utilization, conservation, stockpiling, and
storage of energy resources” and “to encourage certain practices in the [oil] industry which
serve the public interest and are intended to achieve efficiency and cost reduction, ensure
continuous supply of petroleum products.” These powers can be exercised without emasculating
the LGUs of the powers granted them. When these ambiguous powers are pitted against the
unequivocal power of the LGU to enact police power and zoning ordinances for the general
welfare of its constituents, it is not difficult to rule in favor of the latter. Considering that the
113
powers of the DOE regarding the Pandacan Terminals are not categorical, the doubt must be
resolved in favor of the City of Manila:

SECTION 5. Rules of Interpretation. ― In the interpretation of the provisions of


this Code, the following rules shall apply:

(a) Any provision on a power of a local government unit shall be liberally


interpreted in its favor, and in case of doubt, any question thereon shall be resolved
in favor of devolution of powers and of the lower local government unit. Any fair
and reasonable doubt as to the existence of the power shall be interpreted in favor
of the local government unit concerned;

xxx xxx xxx

(g) IThe general welfare provisions in this Code shall be liberally interpreted to
give more powers to local government units in accelerating economic development
and upgrading the quality of life for the people in the community xxxx

The least we can do to ensure genuine and meaningful local autonomy


is not to force an interpretation that negates powers explicitly granted to local
governments. To rule against the power of LGUs to reclassify areas within their
jurisdiction will subvert the principle of local autonomy guaranteed by the
Constitution.[160] As we have noted in earlier decisions, our national officials should
not only comply with the constitutional provisions on local autonomy but should
also appreciate the spirit and liberty upon which these provisions are based.[161]

THE DOE CANNOT EXERCISE THE POWER OF CONTROL


OVER LGUS

Another reason that militates against the DOE’s assertions is that Section 4 of Article
X of the Constitution confines the President’s power over LGUs to one of general supervision:

SECTION 4. The President of the Philippines shall exercise general supervision over
local governments. Xxxx

Consequently, the Chief Executive or his or her alter egos, cannot exercise the power of control

over them.[162] Control and supervision are distinguished as follows:

114
[Supervision] means overseeing or the power or authority of an officer to see that
subordinate officers perform their duties. If the latter fail or neglect to fulfill them,
the former may take such action or step as prescribed by law to make them perform
their duties. Control, on the other hand, means the power of an officer to alter or
modify or nullify or set aside what a subordinate officer ha[s] done in the
performance of his duties and to substitute the judgment of the former for that of
the latter.[163]

Supervisory power, when contrasted with control, is the power of mere oversight over

an inferior body; it does not include any restraining authority over such body.[164] It does not

allow the supervisor to annul the acts of the subordinate.[165] Here, what the DOE seeks to do is

to set aside an ordinance enacted by local officials, a power that not even its principal, the

President, has. This is because:

Under our present system of government, executive power is vested in the


President. The members of the Cabinet and other executive officials are merely alter
egos. As such, they are subject to the power of control of the President, at whose
will and behest they can be removed from office; or their actions and decisions
changed, suspended or reversed. In contrast, the heads of political subdivisions are
elected by the people. Their sovereign powers emanate from the electorate, to
whom they are directly accountable. By constitutional fiat, they are subject to the
President’s supervision only, not control, so long as their acts are exercised within
the sphere of their legitimate powers. By the same token, the President may not
withhold or alter any authority or power given them by the Constitution and the
law.[166]

Thus, the President and his or her alter egos, the department heads, cannot interfere with

the activities of local governments, so long as they act within the scope of their

authority. Accordingly, the DOE cannot substitute its own discretion for the discretion exercised

by thesanggunian of the City of Manila. In local affairs, the wisdom of local officials must prevail

as long as they are acting within the parameters of the Constitution and the law.[167]

ORDINANCE NO. 8027 IS NOT INVALID FOR FAILURE TO COMPLY WITH RA 7924 AND EO 72

115
The oil companies argue that zoning ordinances of LGUs are required to be submitted to

the Metropolitan Manila Development Authority (MMDA) for review and if found to be in

compliance with its metropolitan physical framework plan and regulations, it shall endorse the

same to the Housing and Land Use Regulatory Board (HLURB). Their basis is Section 3 (e) of RA

7924:[168]

SECTION 3. Scope of MMDA Services. ― Metro-wide services under the jurisdiction


of the MMDA are those services which have metro-wide impact and transcend local
political boundaries or entail huge expenditures such that it would not be viable for
said services to be provided by the individual [LGUs] comprising Metropolitan
Manila. These services shall include:

xxx xxx xxx

(g) Urban renewal, zoning, and land use planning, and shelter services
which include the formulation, adoption and implementation of policies, standards,
rules and regulations, programs and projects to rationalize and optimize urban land
use and provide direction to urban growth and expansion, the rehabilitation and
development of slum and blighted areas, the development of shelter and housing
facilities and the provision of necessary social services thereof. (Emphasis
supplied)

Reference was also made to Section 15 of its implementing rules:

Section 15. Linkages with HUDCC, HLURB, NHA, LGUs and Other National
Government Agencies Concerned on Urban Renewal, Zoning and Land Use Planning
and Shelter Services. Within the context of the National Housing and Urban
Development Framework, and pursuant to the national standards, guidelines and
regulations formulated by the Housing and Land Use Regulatory Board [HLURB] on
land use planning and zoning, the [MMDA] shall prepare a metropolitan physical
framework plan and regulations which shall complement and translate the socio-
economic development plan for Metro Manila into physical or spatial terms, and
provide the basis for the preparation, review, integration and implementation of local
land use plans and zoning, ordinance of cities and municipalities in the area.

116
Said framework plan and regulations shall contain, among others, planning
and zoning policies and procedures that shall be observed by local government units
in the preparation of their own plans and ordinances pursuant to Section 447 and
458 of RA 7160, as well as the identification of sites and projects that are considered
to be of national or metropolitan significance.

Cities and municipalities shall prepare their respective land use plans
and zoning ordinances and submit the same for review and integration by the
[MMDA] and indorsement to HLURB in accordance with Executive Order No. 72 and
other pertinent laws.

In the preparation of a Metropolitan Manila physical framework plan and


regulations, the [MMDA] shall coordinate with the Housing and Urban Development
Coordinating Council, HLURB, the National Housing Authority, Intramuros
Administration, and all other agencies of the national government which are
concerned with land use and zoning, urban renewal and shelter services. (Emphasis
supplied)

They also claim that EO 72[169] provides that zoning ordinances of cities and
municipalities of Metro Manila are subject to review by the HLURB to ensure compliance with
national standards and guidelines. They cite Section 1, paragraphs I, (e), (f) and (g):

SECTION 1. Plan formulation or updating. ―

xxx xxx xxx

(g) Cities and municipalities of Metropolitan Manila shall continue to


formulate or update their respective comprehensive land use plans, in
accordance with the land use planning and zoning standards and
guidelines prescribed by the HLURB pursuant to EO 392, S. of 1990, and
other pertinent national policies.

Xxx xxx xxx


117
(e) Pursuant to LOI 729, S. of 1978, EO 648, S. of 1981, and RA 7279,
the comprehensive land use plans of provinces, highly urbanized cities and
independent component cities shall be reviewed and ratified by the HLURB to
ensure compliance with national standards and guidelines.

(f) Pursuant to EO 392, S. of 1999, the comprehensive land use plans of


cities and municipalities of Metropolitan Manila shall be reviewed by the HLURB to
ensure compliance with national standards and guidelines.

(g) Said review shall be completed within three (3) months upon receipt
thereof otherwise, the same shall be deemed consistent with law, and,
therefore, valid. (Emphasis supplied)

They argue that because Ordinance No. 8027 did not go through this review process, it is

invalid.

The argument is flawed.

RA 7942 does not give MMDA the authority to review land use plans and zoning
ordinances of cities and municipalities. This was only found in its implementing rules which
made a reference to EO 72. EO 72 expressly refers to comprehensive land use plans (CLUPs)
only. Ordinance No. 8027 is admittedly not a CLUP nor intended to be one. Instead, it is a very
specific ordinance which reclassified the land use of a defined area in order to prevent the
massive effects of a possible terrorist attack. It is Ordinance No. 8119 which was explicitly
formulated as the “Manila [CLUP] and Zoning Ordinance of 2006.” CLUPs are the ordinances
which should be submitted to the MMDA for integration in its metropolitan physical framework
plan and approved by the HLURB to ensure that they conform with national guidelines and
policies.

Moreover, even assuming that the MMDA review and HLURB ratification are necessary, the

oil companies did not present any evidence to show that these were not complied with. In
118
accordance with the presumption of validity in favor of an ordinance, its constitutionality or

legality should be upheld in the absence of proof showing that the procedure prescribed by law

was not observed. The burden of proof is on the oil companies which already had notice that

this Court was inclined to dispose of all the issues in this case. Yet aside from their bare

assertion, they did not present any certification from the MMDA or the HLURB nor did they

append these to their pleadings. Clearly, they failed to rebut the presumption of validity of

Ordinance No. 8027.[170]

CONCLUSION

Essentially, the oil companies are fighting for their right to property. They allege that

they stand to lose billions of pesos if forced to relocate. However, based on the hierarchy of

constitutionally protected rights, the right to life enjoys precedence over the right to

property.[171] The reason is obvious: life is irreplaceable, property is not. When the state or

LGU’s exercise of police power clashes with a few individuals’ right to property, the former

should prevail.[172]

Both law and jurisprudence support the constitutionality and validity of Ordinance No.

8027. Without a doubt, there are no impediments to its enforcement and implementation. Any

delay is unfair to the inhabitants of the City of Manila and its leaders who have categorically

expressed their desire for the relocation of the terminals. Their power to chart and control their

own destiny and preserve their lives and safety should not be curtailed by the intervenors’

warnings of doomsday scenarios and threats of economic disorder if the ordinance is enforced.

Secondary to the legal reasons supporting the immediate implementation of Ordinance


No. 8027 are the policy considerations which drove Manila’s government to come up with such a
measure:

... [The] oil companies still were not able to allay the apprehensions of the city
regarding the security threat in the area in general. No specific action plan or
security measures were presented that would prevent a possible large-scale
119
terrorist or malicious attack especially an attack aimed at Malacañang. The
measures that were installed were more directed towards their internal security and
did not include the prevention of an external attack even on a bilateral level of
cooperation between these companies and the police and military.

Xxx xxx xxx

It is not enough for the city government to be told by these oil


companies that they have the most sophisticated fire-fighting equipments and have
invested millions of pesos for these equipments. The city government wants to be
assured that its residents are safe at any time from these installations, and in the
three public hearings and in their position papers, not one statement has been said
that indeed the absolute safety of the residents from the hazards posed by these
installations is assured.[173]

We are also putting an end to the oil companies’ determination to prolong their stay in

Pandacan despite the objections of Manila’s residents. As early as October 2001, the oil

companies signed a MOA with the DOE obliging themselves to:

... undertake a comprehensive and comparative study ... [which] shall include the
preparation of a Master Plan, whose aim is to determine the scope and timing of the
feasible location of the Pandacan oil terminals and all associated facilities and
infrastructure including government support essential for the relocation such as the
necessary transportation infrastructure, land and right of way acquisition,
resettlement of displaced residents and environmental and social acceptability
which shall be based on mutual benefit of the Parties and the public.[174]

Now that they are being compelled to discontinue their operations in the Pandacan Terminals,

they cannot feign unreadiness considering that they had years to prepare for this eventuality.

120
Just the same, this Court is not about to provoke a crisis by ordering the immediate

relocation of the Pandacan Terminals out of its present site. The enforcement of a decision of

this Court, specially one with far-reaching consequences, should always be within the bounds of

reason, in accordance with a comprehensive and well-coordinated plan, and within a time-frame

that complies with the letter and spirit of our resolution. To this end, the oil companies have no

choice but to obey the law.

A WARNING TO PETITIONERS’ COUNSEL

We draw the attention of the parties to a matter of grave concern to the legal profession.

Petitioners and their counsel, Atty. Samson Alcantara, submitted a four-page

memorandum that clearly contained either substance nor research. It is absolutely insulting to

this Court.

We have always tended towards judicial leniency, temperance and compassion to those

who suffer from a wrong perception of what the majesty of the law means. But for a member of

the bar, an officer of the court, to file in this Court a memorandum of such unacceptable quality

is an entirely different matter.

It is indicative less of a personal shortcoming or contempt of this Court and more of a

lawyer’s sorry descent from a high sense of duty and responsibility. As a member of the bar and

as an officer of the court, a lawyer ought to be keenly aware that the chief safeguard of the body

politic is respect for the law and its magistrates.

121
There is nothing more effective than the written word by which counsel can persuade this

Court of the righteousness of his cause. For if truth were self-evident, a memorandum would be

completely unnecessary and superfluous.

The inability of counsel to prepare a memorandum worthy of this Court’s consideration is

an ejemplo malo to the legal profession as it betrays no genuine interest in the cause he claims

to espouse. Or did counsel think he can earn his moment of glory without the hard work and

dedication called for by his petition?

A FINAL WORD

On Wednesday, January 23, 2008, a defective tanker containing 2,000 liters of gasoline

and 14,000 liters of diesel exploded in the middle of the street a short distance from the exit

gate of the Pandacan Terminals, causing death, extensive damage and a frightening

conflagration in the vicinity of the incident. Need we say anthing about what will happen if it is

the estimated 162 to 211 million liters[175]of petroleum products in the terminal complex which

blow up?

WHEREFORE, the motions for leave to intervene of Chevron Philippines Inc., Petron

Corporation and Pilipinas Shell Petroleum Corporation, and the Republic of the Philippines,

represented by the Department of Energy, are hereby GRANTED. Their respective motions for

reconsideration are hereby DENIED. The Regional Trial Court, Manila, Branch 39

is ORDERED to DISMISS the consolidated cases of Civil Case No. 03-106377 and Civil Case No.

03-106380.

122
We reiterate our order to respondent Mayor of the City of Manila to enforce Ordinance No.

8027. In coordination with the appropriate agencies and other parties involved, respondent

Mayor is hereby ordered to oversee the relocation and transfer of the Pandacan Terminals out of

its present site.

To ensure the orderly transfer, movement and relocation of assets and personnel, the

intervenors Chevron Philippines Inc., Petron Corporation and Pilipinas Shell Petroleum

Corporation shall, within a non-extendible period of ninety (90) days, submit to the Regional

Trial Court of Manila, Branch 39, the comprehensive plan and relocation schedule which have

allegedly been prepared. The presiding judge of Manila RTC, Branch 39 will monitor the strict

enforcement of this resolution.

Atty. Samson Alcantara is hereby ordered to explain within five (5) days from notice why

he should not be disciplined for his refusal, or inability, to file a memorandum worthy of the

consideration of this Court.

Treble costs against petitioners’ counsel, Atty. Samson Alcantara.

SO ORDERED.

Sgd.

RENATO C. CORONA
Associate Justice

123
WE CONCUR:

Sgd.

REYNATO S. PUNO
Chief Justice

Chairperson

Sgd. Sgd.

ANGELINA SANDOVAL-GUTIERREZ ADOLFO S. AZCUNA


Associate Justice Associate Justice

Sgd.

TERESITA J. LEONARDO-DE CASTRO

Associate Justice

CERTIFICATION

Pursuant to Section 13, Article VIII of the Constitution, I certify that the conclusions in the
above resolution had been reached in consultation before the case was assigned to the writer of
the opinion of the Court’s Division.

Sgd.

REYNATO S. PUNO

124
Chief Justice

[1]
Formerly known as Caltex (Philippines), Inc.
[2]
Rollo, p. 192.
[3]
Entitled “An Act Creating the Department of Energy, Rationalizing the Organization
and Functions of Government Agencies Related to Energy, and for Other Purposes” also
known as the “DOE Act of 1992” and approved on December 9, 1992. Prior to RA 7638, a
Department of Energy was established under Presidential Decree No. 1206, “Creating the
Department of Energy,” approved on October 6, 1977.
[4]
RA 7638, Section 4.
[5]
Entitled “Ordinance Reclassifying the Land Use of [Those] Portions of Land Bounded
by the Pasig River In The North[,] PNR Railroad Track in the East, Beata St. in the South,
Palumpong St. in the Southwest and Estero De Pandacan in the West, PNR Railroad in the
Northwest Area, Estero of Pandacan in the Northeast, Pasig River in the Southeast and Dr.
M. L. Carreon in the Southwest; the Area of Punta, Sta. Ana Bounded by the Pasig River,
Marcelino Obrero St.[,] Mayo 28 St. and the F. Manalo Street from Industrial II to
Commercial I.”
[6]
Rollo, p. 12.
[7]
Id., p. 6.
[8]
Id., pp. 16-18. This MOU modified the Memorandum of Agreement (MOA) executed
on October 12, 2001 by the oil companies and the DOE. This MOA called for close
coordination among the parties with a view of formulating appropriate measures to arrive
at the best possible option to ensure, maintain and at the same time harmonize the
interests of both government and the oil companies; id., pp. 413-415.
[9]
Entitled “Resolution Ratifying the Memorandum of Understanding (MOU) Entered
into by and among the Department of Energy, the City of Manila, Caltex (Philippines),
Inc., Petron Corporation and Pilipinas Shell Petroleum Corporation on 26 June 2002, and
Known as Document No. 60, Page No. 12, Book No. 1, Series of 2002 in the Notarial
Registry of Atty. Neil Lanson Salcedo, Notary Public for and in the City of Manila;” id., p.
36.
[10]
Id.
[11]
Entitled “Resolution Extending the Validity of Resolution 97, Series of 2002, to April
30, 2003, Thereby Authorizing his Honor Mayor Jose L. Atienza, Jr., to Issue Special
Business Permits to Caltex Phil., Inc., Petron Corporation and Pilipinas Shell Petroleum
Corporation Situated within the Pandacan Oil Terminal Covering the said Period;” id., p.
38.
[12]
Id.
[13]
Section 455 (b) (2).
[14]
Rollo, p. 280.
[15]
Id., p. 333.
[16]
Penned by Judge Reynaldo G. Ros, id., p. 388.
[17]
Penned by Judge Guillermo G. Purganan, id., p. 423.
125
[18]
Id., p. 458.
[19]
Id., p. 493.
[20]
Id., p. 495.
[21]
Petron tried to intervene in civil case no. 06-115334 but the court denied its motion;
id., pp. 692-694.
[22]
Memorandum of oil companies, p. 25.
[23]
Rollo, p. 238.
[24]
Id., p. 698.
[25]
Footnote no. 50 of memorandum of oil companies, id., p. 26.
[26]
According to the oil companies, they were informed that their and the DOE's motions
to intervene had already been granted (Memorandum of oil companies, p. 28). However,
this contention is not supported by the records.
[27]
Transcript of April 11, 2007 Oral Arguments, pp. 125, 192-195.
[28]
G. B. Bagayaua, Pandacan's Ring of Fire, NEWSBREAK 3(4): 12 (March 3, 2003).
[29]
Pandacan Installation Profile, <http://www.shell.com/home/framework?siteId=ph-
en&FC2=/ph-en/thml/iwgen/about_shell/> (visited March 11, 2007).
[30]
History: More than 85 years of Philippine Partnership,
<http://www.caltex.com/ph/en/ph_history. asp> (visited March 11, 2007).
[31]
Rollo, p. 300.
[32]
<http://www.fundinguniverse.com/company-histories/Petron-Corporation-Company-
History.html> (visited May 15, 2007).
[33]
<http://www.petron.com/about-leading.asp> (visited May 15, 2007).
[34]
United States v. Caltex, Phils., et. al., 344 U.S. 149 (1952).
[35]
N. JOAQUIN, ALMANAC FOR MANILEÑOS 97 (1979).
[36]
Supra note 30.
[37]
Pandacan oil depots: A disaster waiting to
happen <http://www.foe.co.uk/resource/reports/behind_shine.pdf> (visited May 15,
2007).
[38]
Id.
[39]
Safety and health risks in the
Philippines <http://www.foe.co.uk/resource/reports/failing_challenge.pdf> (visited May
15, 2007).
[40]
Supra note 37.
[41]
Supra note 28.
[42]
Supra note 37.
[43]
Id.
[44]
Rollo, p. 221.
[45]
Supra note 28 at 13.
[46]
Supra note 44.
[47]
Id., p. 223.
[48]
Id., p. 222.
[49]
Id., p. 731.
[50]
Hi-Tone Marketing Corporation v. Baikal Realty Corporation, G.R. No. 149992, 20
August 2004, 437 SCRA 121, 139, citing Manalo v. Court of Appeals, G.R. No. 141297, 8
October 2001, 366 SCRA 752, 766 (2001), which in turn citedFirst Philippine Holdings
Corporation v. Sandiganbayan, 253 SCRA 30 (1996).

126
[51]
See Ortega v. Court of Appeals, 359 Phil. 126, 139 (1998), citing the 1997 Rules of
Civil Procedure by Feria, pp. 71-72.
[52]
G.R. No. 166429, 1 February 2006, 481 SCRA 457.
[53]
Id., p. 470.
[54]
Pinlac v. Court of Appeals, 457 Phil. 527, 534 (2003), citing Director of Lands v.
Court of Appeals, G.R. No. L-45168, 25 September 1979, 93 SCRA 238, 246.
[55]
Rollo, p. 203.
[56]
Alfelor v. Halasan, G.R. No. 165987, 31 March 2006, 486 SCRA 451, 461,
citing Nordic Asia Ltd. v. Court of Appeals, 451 Phil. 482, 492-493 (2003).
[57]
To justify their late intervention, the oil companies explained that [they] were aware
of this Petition before the Honorable Court but they opted not to intervene then because
they believed that it was more proper to directly attack the validity of Ordinance No. 8027
(Memorandum, p. 22). They also said that they did not deem it necessary to intervene
then because they relied in good faith that respondent [Mayor] would, as a conscientious
litigant, interpose a fitting defense of the instant Petition. (Footnote no. 2, id., p. 3)
[58]
Hi-Tone Marketing Corporation v. Baikal Realty Corporation, supra note 50 at 140.
[59]
The full text reads:
SEC. 3. Petition for Mandamus. — When any tribunal, corporation, board,
officer or person unlawfully neglects the performance of an act which the law specifically
enjoins as a duty resulting from an office, trust, or station, or unlawfully excludes another
from the use and enjoyment of a right or office to which such other is entitled, and there
is no other plain, speedy and adequate remedy in the ordinary course of law, the person
aggrieved thereby may file a verified petition in the proper court, alleging the facts with
certainty and praying that judgment be rendered commanding the respondent,
immediately or at some other time to be specified by the court, to do the act required to
be done to protect the rights of the petitioner, and to pay the damages sustained by the
petitioner by reason of the wrongful acts of the respondent. xxx
[60]
Supra note 25.
[61]
Footnote no. 24, p. 9 of the decision.
[62]
Logronio v. Taleseo, 370 Phil. 907, 910 (1999).
[63]
Vera v. Hon. Judge Arca, 138 Phil. 369, 384 (1969).
[64]
Filipino Metals Corporation v. Secretary of Department of Trade and Industry, G.R.
No. 157498, 15 July 2005, 463 SCRA 616, 624 citing Valley Trading Co., Inc. v. CFI of
Isabela, Br. II, G.R. No. 49529, 31 March 1989, 171 SCRA 501, 508, in turn
citing Tablarin v. Gutierrez, G.R. No. L-78104, 31 July 1987, 52 SCRA 731, 737.
[65]
Rollo, pp. 387-388.
[66]
Valley Trading Co., Inc. v. CFI of Isabela, Br. II, supra note 64.
[67]
Id., citations omitted.
[68]
Ermita-Malate Hotel and Motel Operators Association, Inc. v. Hon. City Mayor of
Manila, 127 Phil. 306, 314-315 (1967), citing US v. Salaveria, 39 Phil. 102, 111 (1918).
[69]
Angara v. Electoral Commission, 63 Phil. 139, 158-159 (1936).
[70]
Memorandum of oil companies, p. 38, citing City of Manila v. Laguio, Jr., G.R. No.
118127, 12 April 2005, 455 SCRA 308, 358-359.
[71]
People v. De Guzman, et al., 90 Phil. 132, 135 (1951), citations omitted.
[72]
Seng Kee & Co. v. Earnshaw and Piatt, 56 Phil. 204, 212-213 (1931).

127
[73]
Sec. 1. Judicial notice, when mandatory. - A court shall take judicial notice, without
introduction of evidence, of the existence and territorial extent of states, their political
history, forms of government and symbols of nationality, the law of nations, the admiralty
and maritime courts of the world and their seals, the political constitution and history of
the Philippines, the official acts of the legislative, executive and judicial departments of
the Philippines, the laws of nature, the measure of time and the geographical divisions.
[74]
Revised Charter of the City of Manila.
[75]
29 AmJur 2d 156, Evidence, Section 126 citing Faustrum v. Board of Fire & Police
Comm’rs, (2d Dist) 240 III App 3d 947, 181 Ill Dec 567, 608 NE2d 640, app den 151 III
2d 563, 186 III Dec 380, 616 NE2d 333.
[76]
Id., citing Dream Mile Club, Inc. v. Tobyhanna Township Bd. of Supervisors, 150 Pa
Cmwlth 309, 615 A2d 931.
[77]
Id.
[78]
Transcript of April 11, 2007 Oral Arguments, p. 244.
[79]
Rollo, p. 698.
[80]
Memorandum of respondent, pp. 30-31.
[81]
Memorandum of oil companies, p. 26.
[82]
Answer, paragraphs 20.1 and 20.3, pp. 20-21.
[83]
Alfelor v. Halasan, supra note 56 at 460, citing Cunanan v. Amparo, 80 Phil. 227,
232 (1948), in turn citing McDaniel v. Apacible, 44 Phil. 248 (1922).
[84]
Republic Glass Corporation v. Qua, G.R. No. 144413, 30 July 2004, 435 SCRA 480,
492.
[85]
Republic of the Philippines v. Court of Appeals, 359 Phil. 530, 582 (1998), Romero,
J., separate opinion.
[86]
Sec. 84 of Ordinance No. 8119 provides:
“Repealing Clause. - All ordinances, rules or regulations in conflict with the provisions
of this Ordinance are hereby repealed; PROVIDED, That the rights that are vested upon
the effectivity of this Ordinance shall not be impaired.” (Rollo, p. 493.)
[87]
Memorandum of oil companies, pp. 44-45, citing Annex “C” of Ordinance No. 8119.
Annex “C” (Zone Boundaries) of Ordinance No. 8119 enumerates and specifies the areas
covered by the different zones:
“High Density Residential/Mixed Use Zone

R-3/MXD

Color: Yellow

District I

1. area covered by Smokey Mountain Development and Reclamation Project.

2. area bounded on the N by Manila–Navotas boundary, on the SW by Estero de


Maypajo, on the NW by Malaya, on the NE by Simeon de Jesus, and on the NW by
Taliba

3. area bounded on the N by Estero de Maypajo, on the SW by Estero de Sunog


Apog/Rodriguez, on the NW by Younger, and on the NE by Estaro de Maypajo

4. area occupied by a portion in Vitas Complex (as indicated in the Zoning Map)

128
5. area bounded on the SE by F. Varona, on the SW by Lallana, on the NW by Roxas,
and on the NE by Jacinto

6. area bounded on the E by Estero de Vitas, on the SW by C-2 Road, on the NW by


Velasquez, and on the NE by Osorio

7. area bounded on the SE by Varona, on the NW by Pitong Gatang, on the SW by


Lacson, on the S by Chesa, on the W by Quezon, on the NW by Liwayway, on the W by
Garcia, and on the NE by Harbosa (except the area covered by C-2/MXD Zone – area
bounded on the N by Bulacan, on the E by Magsaysay, on the S by Dandan, and on the
W by Garcia)

8. area bounded on the SE by Estero de Vitas, on the SW by Zamora, on the NW by


Herbosa, on the SW by Franco, on the NW by Concha/Nolasco, on the SE by Pavia, on
the NE Sta. Maria, on the SW by Perla, on the W by Varona, on the NE by Herbosa on
the NW by Velasquez, and on the NE by Inocencio (except the area covered by INS-G
– bounded on the SE by Dandan, on the SW by Sta. Maria, and on the NW by
Peñalosa/Sta. Maria)

9. area bounded on the SE by Corcuera/Estero dela Reina, on the NW by Pavia, and


on the NE by J. Luna

10. area bounded on the SE by a line parallel/extending from Arqueros, on the SW by


Dist. 1/Dist. II boundary on the NW by a line parallel/extending from Ricafort, and on
the NE by Dagupan Ext.

11. area bounded on the E by Dama de Noche, on the SW by Lakandula, on the SE by


Asuncion, on the SW by C.M. Recto, on the W by Del Pan, on the S by Zaragosa, on
the W by Kagitingan, and on the N/NE by Tuazon

Distinct II

1. area bounded on the N by Manila-Kalookan boundary, and on the E/S/W by Estero


de Maypajo

2. area bounded ion the N by Manila-Kalookan boundary, on the SW by J. Luna, on


the NW by Antipolo, and on the NE by Estero de Sunog Apog

3. area bounded on the SE by Avellana, on the SW by Dagupan, on the NW by


Bualcan, and on the NE by J. Luna

129
4. area bounded on the SE by Manila-Kalookan boundary, on the SW buy Rizal
Avenue, on the NW by Teodoro/Tabora/Estero de Maypajo and on the N/NW/NE by
Manila-Kalookan boundary

5. area bounded on the SE by Laguna, on the SW by Estero de San Lazaro, on the S


by Herrera, and on the NE by J. Abad Santos

6. area bounded on the SE by a line parallel/extending from Arqueros, on the SW by


A. Rivera, on the NW by a line parallel/extending from La Torre, and on the NE by Dist.
I – Dist. II boundary

District III

1. area bounded on the SE by Chu Chin Road, on the E by L. Rivera, on the NW by


Aurora Blvd., and on the NE by Liat Su

2. area bounded on the N by Laguna, on the E by T. Mapua, on the S by S. Herrera, and


on the W by Dist II – Dist. III boundary

District IV

1. area bounded on the SE by Manila-Quezon City boundary, on the SW by Piy Margal,


on the NW by Casañas, on the SW by Dapitan, on the NW by Ibarra, and on the NE by
Simoun

2. area bounded on the SE by PNR Railway, on the SW by Lardizabal, on the SE by M.


dela Fuente, on the NW by a lien parallel/extending from San Jose II, on the NW by
Loreto, on the NE by Tuazon, on the NW by M. dela Fuente, and on the NE by España

3. area bounded on the SE by Matimyas/Blumentritt, on the SE by Sobriedad Ext., on


the NW by Antipolo, and on the NE by S. Loyola, (except the area covered by Legarda
Elem. School)

4. area bounded on the SE Manila-Quezon City boundary, on the SW/NW by


Blumentritt, and on the NE by Matimyas

5. area bounded on the SE by Blumentritt, on the SW by Tuazon, on the NW by


Antipolo, and on the NE by Sobriedad (except the area bounded by Most Holy Trinity
Parish Church/Holy Trinity Academy)

6. area bounded on the SE by Manila-Quezon City boundary, on the S by Sociego, on


the E by Santol, on the S by one (1) block south of Escoda, on the W by one (1) block
west of Santol, on the S by one (10 block south of Tuazon, on the SE by Piña, on the S

130
by Vigan, on the E by Santiago, on the NW by PNR Railway, and on the NE by G.
Tuazon (except the area occupied by a portion of Burgos Elem. School)

District V

1. area occupied by an area in Baseco Compound (as indicated in the Zoning Map

2. area occupied by Engineering Island

3. area bounded on the SE by Estero de Pandacan, on the SW by Quirino Avenue,


on the S by Plaza Dilao, on the NW by Pres. Quirino Avenue, on the NE by the property
line of Grayline Phils. Inc. (except the area occupied by Plaza dela Virgen/M.A. Roxas
High School)

4. area bounded on the SE by Estero de Pandacan, on the SW by Estero Tripa de


Gallina/Pedro Gil, on the E by Onyx, on the SW by Estero Tripa de Gallina, on the
NW/NE by PNR Railway (except the area occupied by Concordia College)

5. area bounded on the NE by Pedro Gil, on the SE by Pasig Line, on the SW buy F.
Torres, and on the NW/W by Onyx

6. area bounded on the SE by one (1) block northwest of Tejeron, on the SW by F.


Torres, on the SE by Pasig Line, on the SW by Estrada, on the NW by Onyx, and on the
SW by A. Francisco

7. area bounded on the SE by Jimenez, on the SW by Franco, on the SE by


Alabastro, on the NE by road parallel/extending to Jade, on the SE by Topacio, on the
SW by Estrada, on the NW by PNR Railway, and on the NE by Estero Tripa de Gallina

8. area bounded on the SE by PNR Railway, on the SW by Estrada, on the SE by del


Pilar, on the SW by Don Pedro, on the SE by A. Aquino, on the SW by P. Ocampo, Sr.,
and on the NW by Diamante

9. area bounded on the NE by San Andres, on the SW by Diamante, on the S by


Zapanta, on the NW by Singalong, on the NE by Cong. A. Francisco, and on the NE by
Linao.

District VI

1. area bounded on the SE/SW by Manila-Quezon City boundary/San Juan River, on


the NW by PNR Railway, and on the N/NE by R. Magsaysay Blvd. (except the area
occupied by C-3/MXD – area bounded by R. Magsaysay and Santol Ext./area bounded
by R. Magsaysay Baldovino, Hintoloro, Road 2, Buenviaje, and V. Mapa)

131
2. area bounded on the SE by PNR Railway, on the SW by San Juan River, on the NE
by Dalisay, on the NW by Lubiran, and on the NE by Cordeleria

3. area bounded on the SE by San Juan River, on the SW by Manila-Mandaluyong


boundary/Panaderos, and on the NW/SW/NE by Pasig River

4. area bounded on the E/SW by Pres. Quirino Avenue, and on the NW/NE by Estero
de Pandacan

5. area bounded on the SE/E by Estero de Pandacan, on the W by Pres. Quirino


Avenue, and on the NE by Pasig River

6. area bounded on the SE by Pasig River, on the SW by PNR Railway, on the NW/SW
by Estero de Pandacan,/PNR rail tracks, on the NW by Pres. Quirino Avenue, and on
the NE by Estero de Pandacan

7. area bounded on the N by Estero de Pandacan, on the SW by PNR rail tracks, and
on the NW by Estero de Pandacan

8. area bounded on the SW by Kahilum/Felix, on the NW by Pedro Gil, on the NW by


Pedro Gil, on the NE by Estero Tripa de Gallina, on the NW by Estero de Pandacan, and
on the NE By Pres. Quirino Avenue

9. area bounded on the SE by Estero de Pandacan, on the SW/SE by Pasig River, on


the E by a line parallel/extending form Vista on the south side, on the SW by Pedro Gil,
on the NW by M. L. Carreon, and on the NE by PNR Railway

10. area bounded on the SE/SW by Pasig River/Manila-Makati boundary on the NW by


Tejeron, and on the NE by Pedro Gil/New Panaderos.”

Section 12 of Ordinance No. 8119 states the allowable uses of an R-3/MXD zone:
“Sec. 12. Use Regulations in [R-3/MXD]. - An R-3/MXD shall be used primarily for
high-rise housing/dwelling purposes and limited complementary/supplementary
trade, services and business activities. Enumerated below are the allowable uses:
xxx xxx xxx”
[88]
Rollo, pp. 742-744.
[89]
Memorandum of oil companies, p. 28.
[90]
Memorandum of respondent, p. 27.
[91]
Mecano v. Commission on Audit, G.R. No. 103982, 11 December 1992, 216 SCRA
500, 505, citation omitted.
[92]
Delfino v. St. James Hospital, Inc., G.R. No. 166735, 5 September 2006, 501 SCRA
97, 112, citing Mecano v. Commission on Audit, id., p. 506.
[93]
Id.
[94]
Tan v. Pereña, G.R. No. 149743, 18 February 2005, 452 SCRA 53, 68, citations
omitted.
[95]
Id.
[96]
Supra note 91.

132
[97]
See Villegas, etc., et al. v. Subido, 148-B Phil. 668, 676 (1971), citations omitted.
[98]
Supra note 91 at 507.
[99]
See Section 9; rollo, p. 460.
[100]
Section 23.
[101]
Section 12.
[102]
Leynes v. Commission on Audit, G.R. No. 143596, 11 December 2003, 418 SCRA
180, 196.
[103]
Supra note 97.
[104]
Philippine National Oil Company v. Court of Appeals, G.R. No. 109976, 26 April
2005, 457 SCRA 32, 80, citing Ex Parte United States, 226 U. S., 420; 57 L. ed., 281; Ex
Parte Crow Dog, 109 U. S., 556; 27 L. ed., 1030; Partee v. St. Louis & S. F. R. Co., 204
Fed. Rep., 970.
[105]
Id., citing Crane v. Reeder and Reeder, 22 Mich., 322, 334; University of Utah vs.
Richards, 77 Am. St. Rep., 928.
[106]
Supra note 102, citing De Villa v. Court of Appeals, 195 SCRA 722 (1991).
[107]
A special law is one which relates to particular persons or things of a class, or to a
particular portion or section of the state only; Leynes v. Commission on Audit, supra note
102, footnote no. 21, citing U.S. v. Serapio, 23 Phil 584 [1912].
[108]
A general law is one which affects all people of the state or all of a particular class of
persons in the state or embraces a class of subjects or places and does not omit any
subject or place naturally belonging to such class; id., footnote no. 22, citing U.S. v.
Serapio, id; Valera v. Tuason, 80 Phil 823 (1948) and Villegas v. Subido, supra note 97.
[109]
G.R. No. 140847, 23 September 2005, 470 SCRA 609.
[110]
Id., p. 623, citing R. AGPALO, STATUTORY CONSTRUCTION (2003), p. 411, in turn
citing Gaerlan v. Catubig, G.R. No. 23964, 1 June 1966, 17 SCRA 376.
[111]
Memorandum, p. 39, citing Mama, Jr. v. Court of Appeals, G.R. No. 86517, 30 April
1991, 196 SCRA 489, 496.
[112]
Suanes v. Chief Accountant of the Senate, 81 Phil. 877, 879 (1948), citing 55 C. J.,
S; sec. 130, p. 215; see also 34 Am. Jur., pp. 910-911; 95 A. L. R. 273, 277-278.
[113]
Rule 65, Section 3 of the Rules of Court.
[114]
Section 5 (1), Article VIII.
[115]
City of Manila v. Laguio, Jr., supra note 70 at 326, citing Tatel v. Municipality of
Virac, G.R. No. 40243, 11 March 1992, 207 SCRA 157, 161; Solicitor General v.
Metropolitan Manila Authority, G.R. No. 102782, 11 December 1991, 204 SCRA 837,
845; Magtajas v. Pryce Properties Corp., Inc., G.R. No. 111097, 20 July 1994, 234 SCRA
255, 268-267.
[116]
Metropolitan Manila Development Authority v. Viron Transportation Co., Inc., G.R.
No. 170656, 15 August 2007, citing Binay v. Domingo, G.R. No. 92389, September 11,
1991, 201 SCRA 508, 514; Presidential Commission on Good Government v. Peña, G.R.
No. L-77663, April 12, 1988, 159 SCRA 556, 574; Rubi v. Provincial Board of Mindoro, 39
Phil. 660, 708.
[117]
Id.
[118]
Id., citing Pangasinan Transportation Co., Inc. v. The Public Service Commission, 70
Phil. 221, 229 (1940) and Eastern Shipping Lines, Inc. v. Philippine Overseas Employment
Administration, G.R. No. L-76633, October 18, 1988, 166 SCRA 533, 544.
[119]
Roble Arrastre, Inc. v. Villaflor, G.R. No. 128509, 22 August 2006, 499 SCRA 434,
448.
[120]
Article III, Section 18 (kk).
[121]
Section 458 (a) (2) (viii).

133
[122]
Lucena Grand Central Terminal, Inc. v. Jac Liner, Inc., G.R. No. 148339, 23 February
2005, 452 SCRA 174, 185, citing Department of Education, Culture and Sports v. San
Diego, G.R. No. 89572, 21 December 1989, 180 SCRA 533, 537.
[123]
Section 1 thereof.
[124]
Rollo, pp. 982-985.
[125]
Id., p. 1004.
[126]
Id., p. 1006.
[127]
Fabie v. City of Manila, 21 Phil. 486, 492 (1912).
[128]
Didipio Earth-Savers' Multi-purpose Association, Incorporated (DESAMA) v. Gozun,
G.R. No. 157882, 30 March 2006, 485 SCRA 586, 604.
[129]
Patalinghug v. Court of Appeals, G.R. No. 104786, 27 January 1994, 229 SCRA 554,
559, citing Sangalang v. Intermediate Court, G.R. Nos. 71169, 76394, 74376 and 82281,
December 22, 1988, 168 SCRA 634; Ortigas & Co. Ltd. Partnership v. Feati Bank and
Trust Co., No. L-24670, December 14, 1989, 94 SCRA 533.
[130]
Balacuit v. Court of First Instance of Agusan del Norte and Butuan City, Branch II,
G.R. No. L-38429, 30 June 1988 163 SCRA 182, 191.
[131]
Sta. Rosa Realty Development Corporation v. Court of Appeals, G.R. No. 112526, 12
October 2001, 367 SCRA 175, 193, citing PD 449, Section 4 (b).
[132]
Tan Chat v. Municipality of Iloilo, 60 Phil. 465, 473 (1934).
[133]
Supra note 72.
[134]
Rollo, pp. 1010-1011.
[135]
People v. Cruz, 54 Phil. 24, 28 (1929).
[136]
Rollo, p. 300.
[137]
Association of Small Landowners in the Philippines, Inc. v. Secretary of Agrarian
Reform, G.R. No. 78742, 14 July 1989, 343 SCRA 175, 370.
[138]
Chief Justice Reynato S. Puno, “The Right to Property: Its Philosophical and Legal
Bases,” The Court Systems Journal, vol. 10, no. 4, December 2005, p. 6.
[139]
Telecommunications and Broadcast Attorneys of the Philippines, Inc. v.
Comelec, 352 Phil. 153 (1998), Dissenting Opinion of J. Romero, citing Cooley, Thomas
II Constitutional Limitations, 8th Ed., p. 1224 (1927). This is further reinforced by Section
6, Article XII of the Constitution: “The use of property bears a social function xxxx”
[140]
Didipio Earth-Savers' Multi-Purpose Association v. Gozun, supra note 128 at 605.
[141]
Rollo, p. 305.
[142]
Article III, Section 1 states:

Section 1. No person shall be deprived of life, liberty or property without due


process of law, nor shall any person be denied the equal protection of the laws.
[143]
Government Service Insurance System v. Montesclaros, G.R. No. 146494, 14 July
2004, 434 SCRA 441, 452.
[144]
Id., citations omitted.
[145]
Entitled “An Act Creating the Department of Energy, Rationalizing the Organization
and Functions of Government Agencies Related to Energy, and for Other Purposes”
approved on December 9, 1992.
[146]
Entitled “An Act Deregulating the Downstream Oil Industry, and for Other Purposes”
approved on February 10, 1998.
[147]
Section 1.
[148]
Rollo, p. 1443.
[149]
Id., p. 1444.
[150]
Id., p. 1470.

134
[151]
Id., p. 1480.
[152]
Id., p. 730.
[153]
Id., p. 1023.
[154]
Supra note 115.
[155]
Id., pp. 272-273, citation omitted.
[156]
G.R. No. 149743, 18 February 2005, 452 SCRA 53.
[157]
G.R. No. 138810, 29 September 2004, 439 SCRA 326.
[158]
416 Phil. 438 (2001).
[159]
Supra note 115.
[160]
Leynes v. Commission on Audit, supra note 102 at 199, citing Section 25, Article II
and Section 2, Article X of the Constitution.
[161]
Province of Batangas v. Romulo, G.R. No. 152774, 27 May 2004, 429 SCRA 736,
772, citing San Juan v. Civil Service Commission, G.R. No. 92299, 19 April 1991, 196
SCRA 69.
[162]
National Liga ng mga Barangay v. Paredes, G.R. Nos. 130775 and 131939, 27
September 2004, 439 SCRA 130.
[163]
Mondano v. Silvosa, 97 Phil. 143, 147-148 (1955).
[164]
Taule v. Santos, G.R. No. 90336, 12 August 1991, 200 SCRA 512, 522,
citing Hebron v. Reyes, 104 Phil. 175 (1958).
[165]
Municipality of Malolos v. Libangang Malolos, Inc., G.R. No. L-78592, 11
August 1988, 164 SCRA 290, 298 citing Hee Acusar v. IAC, G.R. Nos. L-72969-70, 17
December 1986, 146 SCRA 294, 300.
[166]
Pimentel, Jr. v. Aguirre, G.R. No. 132988, 19 July 2000, 336 SCRA 201, 215, citing
Sec. 1, Art. VII, 1987 Constitution and Joaquin G. Bernas, SJ, The 1987 Constitution of
the Republic of the Philippines: A Commentary, 1996 ed., p. 739.
[167]
See Dadole v. Commission on Audit, G.R. No. 125350, 3 December 2002, 393 SCRA
262, 271.
[168]
Entitled “An Act Creating the [MMDA], Defining its Powers and Functions, Providing
Funds therefor and for Other Purposes.”
[169]
Entitled “Providing for the Preparation and Implementation of the Comprehensive
Land Use Plans of Local Government Units Pursuant to the Local Government Code of
1991 and Other Pertinent Laws” issued on March 25, 1993.
[170]
Figuerres v. Court of Appeals, 364 Phil. 683, 692-693 (1999); Reyes v. Court of
Appeals, 378 Phil. 232, 239 (1999).
[171]
Secretary of Justice v. Hon. Lantion, 379 Phil. 165, 201 (2000).
[172]
Vda. de Genuino v. Court of Agrarian Relations, No. L-25035, 26 February 1968, 22
SCRA 792, 797.
[173]
Report of Committee on Housing, Resettlement and Urban Development of the City
of Manila's Sangguniang Panlungsod recommending the approval of Ordinance No.
8027; rollo, pp. 985, 989.
[174]
Even if this MOU was modified by the June 26, 2002 MOA; id., pp. 601-602.
[175]
Id., p. 1109. See also footnote 35.

135
EN BANC

LEAGUE OF CITIES OF THE G.R. No. 176951


PHILIPPINES (LCP) represented
by LCP National President
JERRY P. TREÑAS, CITY OF
ILOILO represented by
MAYOR JERRY P. TREÑAS,
CITY OF CALBAYOG
represented by MAYOR
MEL SENEN S. SARMIENTO,
and JERRY P. TREÑAS in his
personal capacity as taxpayer,
Petitioners,

- versus -

COMMISSION ON ELECTIONS;
MUNICIPALITY OF BAYBAY,
PROVINCE OF LEYTE;
MUNICIPALITY OF BOGO,
PROVINCE OF CEBU;
MUNICIPALITY OF CATBALOGAN,
PROVINCE OF WESTERN SAMAR;
MUNICIPALITY OF TANDAG,
PROVINCE OF SURIGAO DEL SUR;
MUNICIPALITY OF BORONGAN,
PROVINCE OF EASTERN SAMAR;
and MUNICIPALITY OF TAYABAS,
PROVINCE OF QUEZON,
Respondents.

CITY OF TARLAC, CITY OF SANTIAGO,


CITY OF IRIGA, CITY OF LIGAO,
CITY OF LEGAZPI, CITY OF
TAGAYTAY, CITY OF SURIGAO,
CITY OF BAYAWAN, CITY OF
SILAY, CITY OF GENERAL SANTOS,
CITY OF ZAMBOANGA, CITY OF
GINGOOG, CITY OF CAUAYAN,
CITY OF PAGADIAN, CITY OF
SAN CARLOS, CITY OF
SAN FERNANDO, CITY OF
TACURONG, CITY OF TANGUB,
CITY OF OROQUIETA, CITY OF
URDANETA, CITY OF VICTORIAS,
CITY OF CALAPAN, CITY OF
HIMAMAYLAN, CITY OF

136
BATANGAS, CITY OF BAIS,
CITY OF CADIZ, and
CITY OF TAGUM,
Petitioners-In-Intervention.
x-----------------------------x

LEAGUE OF CITIES OF THE G.R. No. 177499


PHILIPPINES (LCP) represented
by LCP National President
JERRY P. TREÑAS, CITY OF
ILOILO represented by
MAYOR JERRY P. TREÑAS,
CITY OF CALBAYOG
represented by MAYOR
MEL SENEN S. SARMIENTO,
and JERRY P. TREÑAS in his
personal capacity as taxpayer,
Petitioners,

- versus -

COMMISSION ON ELECTIONS;
MUNICIPALITY OF LAMITAN,
PROVINCE OF BASILAN;
MUNICIPALITY OF TABUK,
PROVINCE OF KALINGA;
MUNICIPALITY OF BAYUGAN,
PROVINCE OF AGUSAN DEL SUR;
MUNICIPALITY OF BATAC,
PROVINCE OF ILOCOS NORTE;
MUNICIPALITY OF MATI,
PROVINCE OF DAVAO ORIENTAL;
and MUNICIPALITY OF GUIHULNGAN,
PROVINCE OF NEGROS ORIENTAL,
Respondents.

CITY OF TARLAC, CITY OF


SANTIAGO, CITY OF IRIGA,
CITY OF LIGAO, CITY OF LEGAZPI,
CITY OF TAGAYTAY, CITY OF SURIGAO,
CITY OF BAYAWAN, CITY OF
SILAY, CITY OF GENERAL SANTOS,
CITY OF ZAMBOANGA, CITY OF
GINGOOG, CITY OF CAUAYAN,
CITY OF PAGADIAN, CITY OF
SAN CARLOS, CITY OF

137
SAN FERNANDO, CITY OF
TACURONG, CITY OF TANGUB,
CITY OF OROQUIETA, CITY OF
URDANETA, CITY OF VICTORIAS,
CITY OF CALAPAN, CITY OF
HIMAMAYLAN, CITY OF
BATANGAS, CITY OF BAIS,
CITY OF CADIZ, and
CITY OF TAGUM,
Petitioners-In-Intervention.
x - - - - - - - - - - - - - - - - - - - - - - - - - - --x

LEAGUE OF CITIES OF THE G.R. No. 178056


PHILIPPINES (LCP) represented
by LCP National President Present:
JERRY P. TREÑAS, CITY OF
ILOILO represented by PUNO, C.J.,
MAYOR JERRY P. TREÑAS, QUISUMBING,
CITY OF CALBAYOG YNARES-SANTIAGO,
represented by MAYOR CARPIO,
MEL SENEN S. SARMIENTO, AUSTRIA-MARTINEZ,
and JERRY P. TREÑAS in his CORONA,
personal capacity as taxpayer, CARPIO MORALES,
Petitioners, AZCUNA,
TINGA,
CHICO-NAZARIO,
- versus - VELASCO,
JR., NACHURA,
REYES,
LEONARDO-DE CASTRO, and
COMMISSION ON ELECTIONS; BRION, JJ.
MUNICIPALITY OF CABADBARAN,
PROVINCE OF AGUSAN
DEL NORTE; MUNICIPALITY
OF CARCAR, PROVINCE OF
CEBU; and MUNICIPALITY OF
EL SALVADOR, MISAMIS
ORIENTAL,
Respondents.

CITY OF TARLAC, CITY OF


SANTIAGO, CITY OF IRIGA,
CITY OF LIGAO, CITY OF LEGAZPI,
CITY OF TAGAYTAY, CITY OF SURIGAO,
CITY OF BAYAWAN, CITY OF SILAY,
CITY OF GENERAL SANTOS,
CITY OF ZAMBOANGA, CITY OF
GINGOOG, CITY OF CAUAYAN,

138
CITY OF PAGADIAN, CITY OF
SAN CARLOS, CITY OF
SAN FERNANDO, CITY OF
TACURONG, CITY OF TANGUB,
CITY OF OROQUIETA, CITY OF
URDANETA, CITY OF VICTORIAS,
CITY OF CALAPAN, CITY OF
HIMAMAYLAN, CITY OF
BATANGAS, CITY OF BAIS,
CITY OF CADIZ, and Promulgated:
CITY OF TAGUM,
Petitioners-In-Intervention. November 18, 2008
x--------------------------------------------------x

DECISION

CARPIO, J.:

The Case

These are consolidated petitions for prohibition[1] with prayer for the issuance of a writ of
preliminary injunction or temporary restraining order filed by the League of Cities of the
Philippines, City of Iloilo, City of Calbayog, and Jerry P. Treñas[2] assailing the constitutionality of
the subject Cityhood Laws and enjoining the Commission on Elections (COMELEC) and
respondent municipalities from conducting plebiscites pursuant to the Cityhood Laws.

The Facts

During the 11th Congress,[3] Congress enacted into law 33 bills converting 33
municipalities into cities. However, Congress did not act on bills converting 24 other
municipalities into cities.

During the 12th Congress,[4] Congress enacted into law Republic Act No. 9009 (RA
[5]
9009), which took effect on 30 June 2001. RA 9009 amended Section 450 of the Local
Government Code by increasing the annual income requirement for conversion of a municipality
into a city from P20 million to P100 million. The rationale for the amendment was to restrain, in
the words of Senator Aquilino Pimentel, “the mad rush” of municipalities to convert into cities
solely to secure a larger share in the Internal Revenue Allotment despite the fact that they are
incapable of fiscal independence.[6]

139
After the effectivity of RA 9009, the House of Representatives of the
12th Congress[7] adopted Joint Resolution No. 29,[8] which sought to exempt from the P100
million income requirement in RA 9009 the 24 municipalities whose cityhood bills were not
approved in the 11th Congress. However, the 12th Congress ended without the Senate
approving Joint Resolution No. 29.

During the 13th Congress,[9] the House of Representatives re-adopted Joint Resolution No.
29 as Joint Resolution No. 1 and forwarded it to the Senate for approval. However, the
Senate again failed to approve the Joint Resolution. Following the advice of Senator Aquilino
Pimentel, 16 municipalities filed, through their respective sponsors, individual cityhood bills. The
16 cityhood bills contained a common provision exempting all the 16 municipalities from
the P100 million income requirement in RA 9009.

On 22 December 2006, the House of Representatives approved the cityhood bills. The
Senate also approved the cityhood bills in February 2007, except that of Naga,Cebu which was
passed on 7 June 2007. The cityhood bills lapsed into law (Cityhood Laws[10]) on various dates
from March to July 2007 without the President’s signature.[11]

The Cityhood Laws direct the COMELEC to hold plebiscites to determine whether the
voters in each respondent municipality approve of the conversion of their municipality into a
city.

Petitioners filed the present petitions to declare the Cityhood Laws unconstitutional for
violation of Section 10, Article X of the Constitution, as well as for violation of the equal
protection clause.[12] Petitioners also lament that the wholesale conversion of municipalities into
cities will reduce the share of existing cities in the Internal Revenue Allotment because more
cities will share the same amount of internal revenue set aside for all cities under Section 285 of
the Local Government Code.[13]

The Issues

The petitions raise the following fundamental issues:

1. Whether the Cityhood Laws violate Section 10, Article X of the Constitution; and

140
2. Whether the Cityhood Laws violate the equal protection clause.

The Ruling of the Court

We grant the petitions.

The Cityhood Laws violate Sections 6 and 10, Article X of the Constitution, and are thus
unconstitutional.

First, applying the P100 million income requirement in RA 9009 to the present case is a
prospective, not a retroactive application, because RA 9009 took effect in 2001 while the
cityhood bills became law more than five years later.

Second, the Constitution requires that Congress shall prescribe all the criteria for the
creation of a city in the Local Government Code and not in any other law, including the Cityhood
Laws.

Third, the Cityhood Laws violate Section 6, Article X of the Constitution because they
prevent a fair and just distribution of the national taxes to local government units.

Fourth, the criteria prescribed in Section 450 of the Local Government Code, as amended
by RA 9009, for converting a municipality into a city are clear, plain and unambiguous, needing
no resort to any statutory construction.

Fifth, the intent of members of the 11th Congress to exempt certain municipalities from
the coverage of RA 9009 remained an intent and was never written into Section 450 of the Local
Government Code.

Sixth, the deliberations of the 11th or 12th Congress on unapproved bills or resolutions are
not extrinsic aids in interpreting a law passed in the 13th Congress.

Seventh, even if the exemption in the Cityhood Laws were written in Section 450 of the
Local Government Code, the exemption would still be unconstitutional for violation of the equal
protection clause.

141
Preliminary Matters

Prohibition is the proper action for testing the constitutionality of laws administered by the
COMELEC,[14] like the Cityhood Laws, which direct the COMELEC to hold plebiscites in
implementation of the Cityhood Laws. Petitioner League of Cities of the Philippines has legal
standing because Section 499 of the Local Government Code tasks the League with the “primary
purpose of ventilating, articulating and crystallizing issues affecting city government
administration and securing, through proper and legal means, solutions
[15] [16]
thereto.” Petitioners-in-intervention, which are existing cities, have legal standing
because their Internal Revenue Allotment will be reduced if the Cityhood Laws are declared
constitutional. Mayor Jerry P. Treñas has legal standing because as Mayor of Iloilo City and as a
taxpayer he has sufficient interest to prevent the unlawful expenditure of public funds, like the
release of more Internal Revenue Allotment to political units than what the law allows.

Applying RA 9009 is a Prospective Application of the Law

RA 9009 became effective on 30 June 2001 during the 11th Congress. This law specifically
amended Section 450 of the Local Government Code, which now provides:

Section 450. Requisites for Creation. — (a) A municipality or a cluster of


barangays may be converted into a component city if it has a locally
generated average annual income, as certified by the Department of Finance, of at
least One hundred million pesos (P100,000,000.00) for the last two (2) consecutive
years based on 2000 constant prices, and if it has either of the following requisites:

(i)

a contiguous territory
of at least one hundred (100) square kilometers, as certified by the Land
Management Bureau; or

(ii) a population of not less than one hundred fifty thousand (150,000)
inhabitants, as certified by the National Statistics Office.

The creation thereof shall not reduce the land area, population and income of the
original unit or units at the time of said creation to less than the minimum
requirements prescribed herein.

(b) The territorial jurisdiction of a newly-created city shall be properly identified


by metes and bounds. The requirement on land area shall not apply where the city

142
proposed to be created is composed of one (1) or more islands. The territory need
not be contiguous if it comprises two (2) or more islands.

(c) The average annual income shall include the income accruing to the general
fund, exclusive of special funds, transfers, and non-recurring income. (Emphasis
supplied)

Thus, RA 9009 increased the income requirement for conversion of a municipality into a city
from P20 million to P100 million. Section 450 of the Local Government Code, as amended by
RA 9009, does not provide any exemption from the increased income requirement.

Prior to the enactment of RA 9009, a total of 57 municipalities had cityhood bills pending
in Congress. Thirty-three cityhood bills became law before the enactment of RA 9009. Congress
did not act on 24 cityhood bills during the 11th Congress.

During the 12th Congress, the House of Representatives adopted Joint Resolution No. 29,
exempting from the income requirement of P100 million in RA 9009 the 24 municipalities whose
cityhood bills were not acted upon during the 11th Congress. This Resolution reached the
Senate. However, the 12th Congress adjourned without the Senate approving Joint Resolution
No. 29.

During the 13th Congress, 16 of the 24 municipalities mentioned in the unapproved Joint
Resolution No. 29 filed between November and December of 2006, through their respective
sponsors in Congress, individual cityhood bills containing a common provision, as follows:

Exemption from Republic Act No. 9009. — The City of x x x shall be exempted from
the income requirement prescribed under Republic Act No. 9009.

This common provision exempted each of the 16 municipalities from the income requirement
of P100 million prescribed in Section 450 of the Local Government Code, as amended by RA
9009. These cityhood bills lapsed into law on various dates from March to July 2007 after
President Gloria Macapagal-Arroyo failed to sign them.

Indisputably, Congress passed the Cityhood Laws long after the effectivity of RA 9009. RA
9009 became effective on 30 June 2001 or during the 11th Congress. The 13th Congress passed
in December 2006 the cityhood bills which became law only in 2007. Thus, respondent
municipalities cannot invoke the principle of non-retroactivity of laws.[17] This basic rule has no

143
application because RA 9009, an earlier law to the Cityhood Laws, is not being applied
retroactively but prospectively.

Congress Must Prescribe in the Local Government Code All Criteria

Section 10, Article X of the 1987 Constitution provides:

No province, city, municipality, or barangay shall be created, divided,


merged, abolished or its boundary substantially altered, except in accordance with
the criteria established in the local government code and subject to approval by a
majority of the votes cast in a plebiscite in the political units directly
affected. (Emphasis supplied)

The Constitution is clear. The creation of local government units must follow the criteria
established in the Local Government Code and not in any other law. There is only one Local
Government Code.[18] The Constitution requires Congress to stipulate in the Local Government
Code all the criteria necessary for the creation of a city, including the conversion of a
municipality into a city. Congress cannot write such criteria in any other law, like the Cityhood
Laws.

The criteria prescribed in the Local Government Code govern exclusively the creation of a
city. No other law, not even the charter of the city, can govern such creation. The clear intent
of the Constitution is to insure that the creation of cities and other political units must follow the
same uniform, non-discriminatory criteria found solely in the Local Government Code. Any
derogation or deviation from the criteria prescribed in the Local Government Code violates
Section 10, Article X of the Constitution.

RA 9009 amended Section 450 of the Local Government Code to increase the income
requirement from P20 million to P100 million for the creation of a city. This took effect on 30
June 2001. Hence, from that moment the Local Government Code required that any municipality
desiring to become a city must satisfy the P100 million income requirement. Section 450 of the
Local Government Code, as amended by RA 9009, does not contain any exemption from this
income requirement.

144
In enacting RA 9009, Congress did not grant any exemption to respondent municipalities,
even though their cityhood bills were pending in Congress when Congress passed RA 9009. The
Cityhood Laws, all enacted after the effectivity of RA 9009, explicitly exempt respondent
municipalities from the increased income requirement in Section 450 of the Local Government
Code, as amended by RA 9009. Such exemption clearly violates Section 10, Article X of the
Constitution and is thus patently unconstitutional. To be valid, such exemption must be written
in the Local Government Code and not in any other law, including the Cityhood Laws.

Cityhood Laws Violate Section 6, Article X of the Constitution

Uniform and non-discriminatory criteria as prescribed in the Local Government Code are
essential to implement a fair and equitable distribution of national taxes to all local government
units. Section 6, Article X of the Constitution provides:

Local government units shall have a just share, as determined by law, in the
national taxes which shall be automatically released to them. (Emphasis supplied)

If the criteria in creating local government units are not uniform and discriminatory, there can
be no fair and just distribution of the national taxes to local government units.

A city with an annual income of only P20 million, all other criteria being equal, should not
receive the same share in national taxes as a city with an annual income of P100 million or
more. The criteria of land area, population and income, as prescribed in Section 450 of the Local
Government Code, must be strictly followed because such criteria, prescribed by law, are
material in determining the “just share” of local government units in national taxes. Since the
Cityhood Laws do not follow the income criterion in Section 450 of the Local Government Code,
they prevent the fair and just distribution of the Internal Revenue Allotment in violation of
Section 6, Article X of the Constitution.

Section 450 of the Local Government Code is Clear,


Plain and Unambiguous

There can be no resort to extrinsic aids — like deliberations of Congress — if the language
of the law is plain, clear and unambiguous. Courts determine the intent of the law from the
literal language of the law, within the law’s four corners.[19] If the language of the law is plain,
145
clear and unambiguous, courts simply apply the law according to its express terms. If a literal
application of the law results in absurdity, impossibility or injustice, then courts may resort to
extrinsic aids of statutory construction like the legislative history of the law.[20]

Congress, in enacting RA 9009 to amend Section 450 of the Local Government Code, did
not provide any exemption from the increased income requirement, not even to respondent
municipalities whose cityhood bills were then pending when Congress passed RA 9009. Section
450 of the Local Government Code, as amended by RA 9009, contains no exemption
whatsoever. Since the law is clear, plain and unambiguous that any municipality desiring to
convert into a city must meet the increased income requirement, there is no reason to go
beyond the letter of the law in applying Section 450 of the Local Government Code, as amended
by RA 9009.

The 11th Congress’ Intent was not Written into the Local Government Code

True, members of Congress discussed exempting respondent municipalities from RA 9009,


as shown by the various deliberations on the matter during the 11th Congress. However,
Congress did not write this intended exemption into law. Congress could have easily included
such exemption in RA 9009 but Congress did not. This is fatal to the cause of respondent
municipalities because such exemption must appear in RA 9009 as an amendment to Section
450 of the Local Government Code. The Constitution requires that the criteria for the conversion
of a municipality into a city, including any exemption from such criteria, must all be written in
the Local Government Code. Congress cannot prescribe such criteria or exemption from such
criteria in any other law. In short, Congress cannot create a city through a law that does not
comply with the criteria or exemption found in the Local Government Code.

Section 10 of Article X is similar to Section 16, Article XII of the Constitution prohibiting
Congress from creating private corporations except by a general law. Section 16 of Article XII
provides:

The Congress shall not, except by general law, provide for the formation,
organization, or regulation of private corporations. Government-owned or controlled
corporations may be created or established by special charters in the interest of the
common good and subject to the test of economic viability. (Emphasis supplied)

146
Thus, Congress must prescribe all the criteria for the “formation, organization, or regulation” of
private corporations in a general law applicable to all without discrimination.[21] Congress
cannot create a private corporation through a special law or charter.

Deliberations of the 11th Congress on Unapproved Bills Inapplicable

Congress is not a continuing body.[22] The unapproved cityhood bills filed during the
11th Congress became mere scraps of paper upon the adjournment of the 11thCongress. All the
hearings and deliberations conducted during the 11th Congress on unapproved bills also became
worthless upon the adjournment of the 11th Congress. These hearings and deliberations cannot
be used to interpret bills enacted into law in the 13th or subsequent Congresses.

The members and officers of each Congress are different. All unapproved bills filed in one
Congress become functus officio upon adjournment of that Congress and must be re-filed anew
in order to be taken up in the next Congress. When their respective authors re-filed the cityhood
bills in 2006 during the 13th Congress, the bills had to start from square one again,
going through the legislative mill just like bills taken up for the first time, from the filing to the
approval. Section 123, Rule XLIV of the Rules of the Senate, on Unfinished Business, provides:

Sec. 123. x x x

All pending matters and proceedings shall terminate upon the expiration of
one (1) Congress, but may be taken by the succeeding Congress as if presented for
the first time. (Emphasis supplied)

Similarly, Section 78 of the Rules of the House of Representatives, on Unfinished


Business, states:

Section 78. Calendar of Business. The Calendar of Business shall consist of the
following:

a.

Unfinished Business. This is business being


considered by the House at the time of its last adjournment. Its consideration
shall be resumed until it is disposed of. The Unfinished Business at the end of a
session shall be resumed at the commencement of the next session as if no
adjournment has taken place. At the end of the term of a Congress, all
Unfinished Business are deemed terminated. (Emphasis supplied)
147
Thus, the deliberations during the 11th Congress on the unapproved cityhood bills, as well
as the deliberations during the 12th and 13th Congresses on the unapproved resolution
exempting from RA 9009 certain municipalities, have no legal significance. They do not qualify
as extrinsic aids in construing laws passed by subsequent Congresses.

Applicability of Equal Protection Clause

If Section 450 of the Local Government Code, as amended by RA 9009, contained an


exemption to the P100 million annual income requirement, the criteria for such exemption could
be scrutinized for possible violation of the equal protection clause. Thus, the criteria for the
exemption, if found in the Local Government Code, could be assailed on the ground of absence
of a valid classification. However, Section 450 of the Local Government Code, as amended by
RA 9009, does not contain any exemption. The exemption is contained in the Cityhood Laws,
which are unconstitutional because such exemption must be prescribed in the Local Government
Code as mandated in Section 10, Article X of the Constitution.

Even if the exemption provision in the Cityhood Laws were written in Section 450 of the
Local Government Code, as amended by RA 9009, such exemption would still be
unconstitutional for violation of the equal protection clause. The exemption provision merely
states, “Exemption from Republic Act No. 9009 ─ The City of x x x shall be exempted from the
income requirement prescribed under Republic Act No. 9009.” This one sentence exemption
provision contains no classification standards or guidelines differentiating the exempted
municipalities from those that are not exempted.

Even if we take into account the deliberations in the 11th Congress that municipalities with
pending cityhood bills should be exempt from the P100 million income requirement, there is still
no valid classification to satisfy the equal protection clause. The exemption will be based solely
on the fact that the 16 municipalities had cityhood bills pending in the 11th Congress when RA
9009 was enacted. This is not a valid classification between those entitled and those not
entitled to exemption from the P100 million income requirement.

To be valid, the classification in the present case must be based on substantial


distinctions, rationally related to a legitimate government objective which is the purpose of the

148
law,[23] not limited to existing conditions only, and applicable to all similarly situated. Thus, this
Court has ruled:

The equal protection clause of the 1987 Constitution permits a valid


classification under the following conditions:

1. The classification must rest on substantial distinctions;


2. The classification must be germane to the purpose of the law;
3. The classification must not be limited to existing conditions
only; and
4. The classification must apply equally to all members of the
same class.[24]

There is no substantial distinction between municipalities with pending cityhood bills in the
th
11 Congress and municipalities that did not have pending bills. The mere pendency of a
th
cityhood bill in the 11 Congress is not a material difference to distinguish one municipality from
another for the purpose of the income requirement. The pendency of a cityhood bill in the
11th Congress does not affect or determine the level of income of a municipality. Municipalities
with pending cityhood bills in the 11thCongress might even have lower annual income than
municipalities that did not have pending cityhood bills. In short, the classification
criterion − mere pendency of a cityhood bill in the 11th Congress − is not rationally related to
the purpose of the law which is to prevent fiscally non-viable municipalities from converting into
cities.

Municipalities that did not have pending cityhood bills were not informed that a pending
cityhood bill in the 11th Congress would be a condition for exemption from the increased P100
million income requirement. Had they been informed, many municipalities would have caused
the filing of their own cityhood bills. These municipalities, even if they have bigger annual
income than the 16 respondent municipalities, cannot now convert into cities if their income is
less than P100 million.

The fact of pendency of a cityhood bill in the 11th Congress limits the exemption to a
specific condition existing at the time of passage of RA 9009. That specific condition will never
happen again. This violates the requirement that a valid classification must not be limited to
existing conditions only. This requirement is illustrated inMayflower Farms, Inc. v. Ten
[25]
Eyck, where the challenged law allowed milk dealers engaged in business prior to a fixed date
to sell at a price lower than that allowed to newcomers in the same business. In Mayflower, the
U.S. Supreme Court held:
149
We are referred to a host of decisions to the effect that a regulatory law may
be prospective in operation and may except from its sweep those presently
engaged in the calling or activity to which it is directed. Examples are statutes
licensing physicians and dentists, which apply only to those entering the profession
subsequent to the passage of the act and exempt those then in practice, or zoning
laws which exempt existing buildings, or laws forbidding slaughterhouses within
certain areas, but excepting existing establishments. The challenged provision is
unlike such laws, since, on its face, it is not a regulation of a business or an activity
in the interest of, or for the protection of, the public, but an attempt to give an
economic advantage to those engaged in a given business at an arbitrary date as
against all those who enter the industry after that date. The appellees do not
intimate that the classification bears any relation to the public health or welfare
generally; that the provision will discourage monopoly; or that it was aimed at any
abuse, cognizable by law, in the milk business. In the absence of any such showing,
we have no right to conjure up possible situations which might justify the
discrimination. The classification is arbitrary and unreasonable and denies the
appellant the equal protection of the law. (Emphasis supplied)

In the same vein, the exemption provision in the Cityhood Laws gives the 16
municipalities a unique advantage based on an arbitrary date − the filing of their cityhood bills
before the end of the 11th Congress – as against all other municipalities that want to convert into
cities after the effectivity of RA 9009.

Furthermore, limiting the exemption only to the 16 municipalities violates the requirement
that the classification must apply to all similarly situated. Municipalities with the same income as
the 16 respondent municipalities cannot convert into cities, while the 16 respondent
municipalities can. Clearly, as worded the exemption provision found in the Cityhood Laws, even
if it were written in Section 450 of the Local Government Code, would still be unconstitutional for
violation of the equal protection clause.

WHEREFORE, we GRANT the petitions and declare UNCONSTITUTIONAL the Cityhood


Laws, namely: Republic Act Nos. 9389, 9390, 9391, 9392, 9393, 9394, 9398, 9404, 9405,
9407, 9408, 9409, 9434, 9435, 9436, and 9491.

SO ORDERED.

ANTONIO T. CARPIO
Associate Justice

150
WE CONCUR:

REYNATO S. PUNO
Chief Justice

(On leave)
LEONARDO A. QUISUMBING CONSUELO YNARES-SANTIAGO
Associate Justice Associate Justice

MA. ALICIA AUSTRIA-MARTINEZ RENATO C. CORONA


Associate Justice Associate Justice

CONCHITA CARPIO MORALES ADOLFO S. AZCUNA


Associate Justice Associate Justice

DANTE O. TINGA MINITA V. CHICO-NAZARIO


Associate Justice Associate Justice

PRESBITERO J. VELASCO, JR. ANTONIO EDUARDO B. NACHURA


Associate Justice Associate Justice

151
RUBEN T. REYES TERESITA J. LEONARDO-DE CASTRO
Associate Justice Associate Justice

ARTURO D. BRION
Associate Justice

CERTIFICATION

Pursuant to Section 13, Article VIII of the Constitution, I certify that the conclusions in the
above Decision had been reached in consultation before the case was assigned to the writer of
the opinion of the Court.

REYNATO S. PUNO
Chief Justice

[1]
Under Section 2, Rule 65 of the 1997 Rules of Civil Procedure.
[2]
As National President of the League of Cities of the Philippines, Mayor of Iloilo City,
and taxpayer.
[3]
June 1998 to June 2001.
[4]
June 2001 to June 2004.
[5]
Entitled AN ACT AMENDING SECTION 450 OF REPUBLIC ACT NO. 7160,
OTHERWISE KNOWN AS THE LOCAL GOVERNMENT CODE OF 1991, BY INCREASING
THE AVERAGE ANNUAL INCOME REQUIREMENT FOR A MUNICIPALITY OR CLUSTER
OF BARANGAYS TO BE CONVERTED INTO A COMPONENT CITY.
[6]
Sponsorship Speech of Senator Aquilino Pimentel, 5 October 2000.
[7]
June 2004 to June 2007.
[8]
Entitled Joint Resolution to Exempt Certain Municipalities Embodied in Bills Filed in
Congress before June 30, 2001 from the Coverage of Republic Act No. 9009.
[9]
June 2007 to June 2010.
[10]
The sixteen (16) Cityhood Laws are the following:

Republic Act No. 9389, entitled “An Act converting the Municipality of Baybay in
the Province of Leyte into a component city to be known as the City of Baybay.” Lapsed
into law on 15 March 2007;

Republic Act No. 9390, entitled “An Act converting


152
the Municipality of Bogo, Cebu Province into a component city to be known as the City
of Bogo.” Lapsed into law on 15 March 2007;

Republic Act No. 9391, entitled “An Act converting the Municipality of Catbalogan in
the Province of Samar into a component city to be known as the City ofCatbalogan.”
Lapsed into law on 15 March 2007;

Republic Act No. 9392, entitled “An Act converting the Municipality of Tandag in
the Province of Surigao del Sur into a component city to be known as the City ofTandag.”
Lapsed into law on 15 March 2007;

Republic Act No. 9394, entitled “An Act converting the Municipality of Borongan in
the Province of Eastern Samar into a component city to be known as the City
of Borongan.” Lapsed into law on 16 March 2007;

Republic Act No. 9398, entitled “An Act converting the Municipality of Tayabas in
the Province of Quezon into a component city to be known as the City ofTayabas.” Lapsed
into law on 18 March 2007;

Republic Act No. 9393, entitled “An Act converting the Municipality of Lamitan in
the Province of Basilan into a component city to be known as the City ofLamitan.” Lapsed
into law on 15 March 2007;

Rep
ublic Act No. 9404, entitled “An Act converting the Municipality of Tabuk into a component
city of the Province of Kalinga to be known as the City of Tabuk.” Lapsed into law on 23
March 2007;

Rep
ublic Act No. 9405, entitled “An Act converting the Municipality of Bayugan in
the Province of Agusan del Sur into a component city to be known as the City ofBayugan.”
Lapsed into law on 23 March 2007;

Rep
ublic Act No. 9407, entitled “An Act converting the Municipality of Batac in
the Province of Ilocos Norte into a component city to be known as the City of Batac.”
Lapsed into law on 24 March 2007;

Rep
ublic Act No. 9408, entitled “An Act converting the Municipality of Mati in
the Province of Davao Oriental into a component city to be known as the City of Mati.”
Lapsed into law on 24 March 2007;

153
Rep
ublic Act No. 9409, entitled “An Act converting the Municipality of Guihulngan in
the Province of Negros Oriental into a component city to be known as the City
ofGuihulngan.” Lapsed into law on 24 March 2007;

Rep
ublic Act No. 9434, entitled “An Act converting the Municipality of Cabadbaran into a
component city of the Province of Agusan Del Norte to be known as the City
of Cabadbaran.” Lapsed into law on 12 April 2007;

Rep
ublic Act No. 9436, entitled “An Act converting the Municipality of Carcar in
the Province of Cebu into a component city to be known as the City of Carcar.” Lapsed
into law on 15 April 2007;

Rep
ublic Act No. 9435, entitled “An Act converting the Municipality of El Salvador in
the Province of Misamis Oriental into a component city to be known as the City of El
Salvador.” Lapsed into law on 12 April 2007; and

Rep
ublic Act No. 9491, entitled “An Act converting the Municipality of Naga in
the Province of Cebu into a component city to be known as the City of Naga.” Lapsed into
law on 15 July 2007.
[11]
Section 27 (1), Article VI of the Constitution.
[12]
Section 1, Article III of the Constitution.
[13]
Section 285 of the Local Government Code provides: “Allocation to Local
Government Units. — The share of local government units in the internal revenue
allotment shall be allocated in the following manner:
(a) Provinces — Twenty-three percent (23%);
(b) Cities — Twenty-three percent (23%);
(c) Municipalities — Thirty-four percent (34%); and
(d) Barangays — Twenty percent (20%)
Provided, however, That the share of each province, city, and municipality shall
be determined on the basis of the following formula:
(a) Population — Fifty percent (50%);
(b) Land Area — Twenty-five percent (25%); and
(c) Equal sharing — Twenty-five percent (25%)
Provided, further, That the share of each barangay with a population of not less
than one hundred (100) inhabitants shall not be less than Eighty thousand (P80,000.00)
per annum chargeable against the twenty percent (20%) share of the barangay from the
internal revenue allotment, and the balance to be allocated on the basis of the following
formula:
(a) On the first year of the effectivity of this Code:
(1) Population — Forty percent (40%); and
(2) Equal Sharing — Sixty percent (60%)
(b) On the second year:
(1) Population — Fifty percent (50%); and

154
(2) Equal Sharing — Fifty percent (50%)
(c) On the third year and thereafter:
(1) Population — Sixty percent (60%); and
(2) Equal sharing — Forty percent (40%).
Provided, finally, That the financial requirements of barangays created by local
government units after the effectivity of this Code shall be the responsibility of the local
government unit concerned.”
[14]
Sema v. COMELEC, G.R. No. 177597, 16 July 2008; Social Weather Stations, Inc. v.
COMELEC, 409 Phil. 571, 592 (2001); Mutuc v. COMELEC, 146 Phil. 798 (1970).
[15]
Section 499 of the Local Government Code provides: “Purpose of Organization. —
There shall be an organization of all cities to be known as the League of Cities for the
primary purpose of ventilating, articulating and crystallizing issues affecting city government
administration, and securing, through proper and legal means, solutions thereto.

The league may form chapters at the provincial level for the component cities of a
province. Highly-urbanized cities may also form a chapter of the League. The National
League shall be composed of the presidents of the league of highly-urbanized cities and
the presidents of the provincial chapters of the league of component cities.”
[16]
The Court granted the interventions of the following cities: Santiago City, Iriga City,
Ligao City, Legazpi City, Tagaytay City, Surigao City, Bayawan City, Silay City, General
Santos City, Zamboanga City, Gingoog City, Cauayan City, Pagadian City, San Carlos City,
San Fernando City, Tacurong City, Tangub City, Oroquieta City, Urdaneta City, Victorias
City, Calapan City, Himamaylan City, Batangas City, Bais City, Tarlac City, Cadiz City, and
Tagum City.
[17]
Article 4 of the Civil Code provides: “Laws shall have no retroactive effect, unless the
contrary is provided.”
[18]
Republic Act No. 7160, as amended.
[19]
Ramirez v. Court of Appeals, G.R. No. 93833, 28 September 1995, 248 SCRA 590,
596; Security Bank and Trust Company v. RTC of Makati, Br. 61, G.R. No. 113926, 23
October 1996, 263 SCRA 483, 488.
[20]
Republic v. Court of Appeals, 359 Phil. 530, 559 (1998); Commissioner of Internal
Revenue v. Solidbank Corp., 462 Phil. 96, 129-131 (2003).
[21]
The Corporation Code of the Philippines (Batas Pambansa Blg. 68) is the general law
providing for the formation, organization and regulation of private corporations.
[22]
See Neri v. Senate Committee on Accountability of Public Officers and Investigations,
G.R. No. 180643, 25 March 2008, 549 SCRA 77, 135-136.
[23]
The rational basis test is the minimum level of scrutiny that all government actions
challenged under the equal protection clause must meet. The strict scrutiny test is used
in discriminations based on race or those which result in violations of fundamental rights.
Under the strict scrutiny test, to be valid the classification must promote a compelling
state interest. The intermediate scrutiny test is used in discriminations based on gender or
illegitimacy of children. Under the intermediate scrutiny test, the classification must
be substantially related to an important government objective. Laws not subject to the
strict or intermediate scrutiny test are evaluated under the rational basis test, which is the
easiest test to satisfy since the classification must only show a rational relationship to a
legitimate government purpose. See Erwin Chemerinsky, Constitutional Law, Principles
and Policies, 2nd Edition, pp. 645-646.

155
[24]
De Guzman, Jr. v. COMELEC, 391 Phil. 70, 79 (2000); Tiu v. Court of Tax Appeals,
361 Phil. 229, 242 (1999).
[25]
297 U.S. 266 (1936).

156
EN BANC

FRANCISCO CHAVEZ, G.R. No. 168338


Petitioner, x--------------
Present: ----------------
----------------
PUNO, C.J., ----------------
QUISUMBING, ----------------
YNARES-SANTIAGO, -------x
- versus - SANDOVAL-GUTIERREZ,
CARPIO,
AUSTRIA-MARTINEZ, DECISIO
CORONA, N
CARPIO MORALES,
AZCUNA,
TINGA, PUNO, C.J.:
CHICO-NAZARIO,
RAUL M. GONZALES, VELASCO, JR.,
A. Precis
in his capacity as the NACHURA,
Secretary of the REYES, and
Department of Justice; LEONARDO-DE CASTRO, JJ. In this
and NATIONAL TELECOMMUNICATIONS
jurisdiction, it
COMMISSION (NTC), Promulgated:
Respondents. is established
February 15, 2008 that freedom
of the press is
crucial and so
inextricably
woven into
the right to free speech and free expression, that any attempt to restrict it must be met with an
examination so critical that only a danger that is clear and present would be allowed to curtail
it.

Indeed, we have not wavered in the duty to uphold this cherished freedom. We have
struck down laws and issuances meant to curtail this right, as in Adiong v. COMELEC,[1]Burgos v.
Chief of Staff,[2] Social Weather Stations v. COMELEC,[3] and Bayan v. Executive Secretary
Ermita.[4] When on its face, it is clear that a governmental act is nothing more than a naked
means to prevent the free exercise of speech, it must be nullified.

B. The Facts

157
1. The case originates from events that occurred a year after the 2004 national and local
elections. On June 5, 2005, Press Secretary Ignacio Bunye told reporters that the
opposition was planning to destabilize the administration by releasing an audiotape of a
mobile phone conversation allegedly between the President of the Philippines, Gloria
Macapagal Arroyo, and a high-ranking official of the Commission on Elections
[5]
(COMELEC). The conversation was audiotaped allegedly through wire-tapping. Later, in
a Malacañang press briefing, Secretary Bunye produced two versions of the tape, one
supposedly the complete version, and the other, a spliced, “doctored” or altered version,
which would suggest that the President had instructed the COMELEC official to manipulate
the election results in the President’s favor. [6] It seems that Secretary Bunye admitted
that the voice was that of President Arroyo, but subsequently made a retraction. [7]

2. On June 7, 2005, former counsel of deposed President Joseph Estrada, Atty. Alan
Paguia, subsequently released an alleged authentic tape recording of the
wiretap. Included in the tapes were purported conversations of the President, the First
Gentleman Jose Miguel Arroyo, COMELEC Commissioner Garcillano, and the late Senator
Barbers.[8]

3. On June 8, 2005, respondent Department of Justice (DOJ) Secretary Raul Gonzales


warned reporters that those who had copies of the compact disc (CD) and those
broadcasting or publishing its contents could be held liable under the Anti-Wiretapping
Act. These persons included Secretary Bunye and Atty. Paguia. He also stated that
persons possessing or airing said tapes were committing a continuing offense, subject to
arrest by anybody who had personal knowledge if the crime was committed or was being
committed in their presence.[9]

4. On June 9, 2005, in another press briefing, Secretary Gonzales ordered the National
Bureau of Investigation (NBI) to go after media organizations “found to have caused the
spread, the playing and the printing of the contents of a tape” of an alleged wiretapped
conversation involving the President about fixing votes in the 2004 national
elections. Gonzales said that he was going to start with Inq7.net, a joint venture between
the Philippine Daily Inquirer and GMA7 television network, because by the very nature of
the Internet medium, it was able to disseminate the contents of the tape more widely. He
then expressed his intention of inviting the editors and managers of Inq7.net and GMA7 to
a probe, and supposedly declared, “I [have] asked the NBI to conduct a tactical
[10]
interrogation of all concerned.”

158
5. On June 11, 2005, the NTC issued this press release: [11]

NTC GIVES FAIR WARNING TO RADIO AND TELEVISION


OWNERS/OPERATORS TO OBSERVE ANTI-WIRETAPPING LAW AND
PERTINENT CIRCULARS ON PROGRAM STANDARDS

xxx xxx xxx

Taking into consideration the country’s unusual situation, and in order not to
unnecessarily aggravate the same, the NTC warns all radio stations and
television network owners/operators that the conditions of the authorization
and permits issued to them by Government like the Provisional Authority
and/or Certificate of Authority explicitly provides that said companies shall not
use [their] stations for the broadcasting or telecasting of false information or
willful misrepresentation. Relative thereto, it has come to the attention of the
[NTC] that certain personalities are in possession of alleged taped
conversations which they claim involve the President of the Philippines and a
Commissioner of the COMELEC regarding supposed violation of election laws.

These personalities have admitted that the taped conversations are products
of illegal wiretapping operations.

Considering that these taped conversations have not been duly authenticated
nor could it be said at this time that the tapes contain an accurate or truthful
representation of what was recorded therein, it is the position of the [NTC]
that the continuous airing or broadcast of the said taped conversations by
radio and television stations is a continuing violation of the Anti-Wiretapping
Law and the conditions of the Provisional Authority and/or Certificate of
Authority issued to these radio and television stations. It has been
subsequently established that the said tapes are false and/or fraudulent after
a prosecution or appropriate investigation, the concerned radio and television
companies are hereby warned that their broadcast/airing of such false
information and/or willful misrepresentation shall be just cause for the
suspension, revocation and/or cancellation of the licenses or authorizations
issued to the said companies.

In addition to the above, the [NTC] reiterates the pertinent NTC circulars on
program standards to be observed by radio and television stations. NTC
Memorandum Circular 111-12-85 explicitly states, among others, that “all
radio broadcasting and television stations shall, during any broadcast or
telecast, cut off from the air the speech, play, act or scene or other matters
being broadcast or telecast the tendency thereof is to disseminate false
information or such other willful misrepresentation, or to propose and/or
incite treason, rebellion or sedition.” The foregoing directive had been
reiterated by NTC Memorandum Circular No. 22-89, which, in addition
thereto, prohibited radio, broadcasting and television stations from using their
stations to broadcast or telecast any speech, language or scene disseminating
159
false information or willful misrepresentation, or inciting, encouraging or
assisting in subversive or treasonable acts.

The [NTC] will not hesitate, after observing the requirements of due process,
to apply with full force the provisions of said Circulars and their accompanying
sanctions on erring radio and television stations and their
owners/operators.

6. On June 14, 2005, NTC held a dialogue with the Board of Directors of the Kapisanan ng
mga Brodkaster sa Pilipinas (KBP). NTC allegedly assured the KBP that the press release
did not violate the constitutional freedom of speech, of expression, and of the press, and
the right to information. Accordingly, NTC and KBP issued aJoint Press Statement which
states, among others, that: [12]

 NTC respects and will not hinder freedom of the press and the right to
information on matters of public concern. KBP & its members have always
been committed to the exercise of press freedom with high sense of
responsibility and discerning judgment of fairness and honesty.

 NTC did not issue any MC [Memorandum Circular] or Order constituting a


restraint of press freedom or censorship. The NTC further denies and does
not intend to limit or restrict the interview of members of the opposition or
free expression of views.

 What is being asked by NTC is that the exercise of press freedom [be] done
responsibly.

 KBP has program standards that KBP members will observe in the
treatment of news and public affairs programs. These include verification of
sources, non-airing of materials that would constitute inciting to sedition
and/or rebellion.

 The KBP Codes also require that no false statement or willful


misrepresentation is made in the treatment of news or commentaries.

 The supposed wiretapped tapes should be treated with sensitivity and


handled responsibly giving due consideration to the process being
undertaken to verify and validate the authenticity and actual content of the
same.”

C. The Petition

Petitioner Chavez filed a petition under Rule 65 of the Rules of Court against respondents
Secretary Gonzales and the NTC, “praying for the issuance of the writs ofcertiorari and
160
prohibition, as extraordinary legal remedies, to annul void proceedings, and to prevent the
unlawful, unconstitutional and oppressive exercise of authority by the respondents.”[13]

Alleging that the acts of respondents are violations of the freedom on expression and of
the press, and the right of the people to information on matters of public concern,[14] petitioner
specifically asked this Court:

[F]or [the] nullification of acts, issuances, and orders of respondents committed or


made since June 6, 2005 until the present that curtail the public’s rights to freedom
of expression and of the press, and to information on matters of public concern
specifically in relation to information regarding the controversial taped conversion
of President Arroyo and for prohibition of the further commission of such acts, and
making of such issuances, and orders by respondents. [15]

Respondents[16] denied that the acts transgress the Constitution, and questioned
petitioner’s legal standing to file the petition. Among the arguments they raised as to the
validity of the “fair warning” issued by respondent NTC, is that broadcast media enjoy lesser
constitutional guarantees compared to print media, and the warning was issued pursuant to the
NTC’s mandate to regulate the telecommunications industry. [17] It was also stressed that “most
of the [television] and radio stations continue, even to this date, to air the tapes, but of late
within the parameters agreed upon between the NTC and KBP.” [18]

D. THE PROCEDURAL THRESHOLD: LEGAL STANDING

To be sure, the circumstances of this case make the constitutional challenge peculiar.
Petitioner, who is not a member of the broadcast media, prays that we strike down the acts and
statements made by respondents as violations of the right to free speech, free expression and a
free press. For another, the recipients of the press statements have not come forward—neither
intervening nor joining petitioner in this action. Indeed, as a group, they issued a joint
statement with respondent NTC that does not complain about restraints on freedom of the press.

It would seem, then, that petitioner has not met the requisite legal standing, having failed
to allege “such a personal stake in the outcome of the controversy as to assure that concrete
adverseness which sharpens the presentation of issues upon which the Court so largely depends
for illumination of difficult constitutional questions.” [19]

But as early as half a century ago, we have already held that where serious constitutional
questions are involved, “the transcendental importance to the public of these cases demands
161
that they be settled promptly and definitely, brushing aside if we must, technicalities of
[20]
procedure.” Subsequently, this Court has repeatedly and consistently refused to wield
procedural barriers as impediments to its addressing and resolving serious legal questions that
greatly impact on public interest,[21] in keeping with the Court's duty under the 1987
Constitution to determine whether or not other branches of government have kept themselves
within the limits of the Constitution and the laws and that they have not abused the discretion
given to them.

Thus, in line with the liberal policy of this Court on locus standi when a case involves an issue of
overarching significance to our society,[22] we therefore brush aside technicalities of procedure
and take cognizance of this petition,[23] seeing as it involves a challenge to the most exalted of
all the civil rights, the freedom of expression. The petition raises other issues like the extent of
the right to information of the public. It is fundamental, however, that we need not address all
issues but only the most decisive one which in the case at bar is whether the acts of the
respondents abridge freedom of speech and of the press.

But aside from the primordial issue of determining whether free speech and freedom of the
press have been infringed, the case at bar also gives this Court the opportunity: (1) to distill the
essence of freedom of speech and of the press now beclouded by the vagaries of motherhood
statements; (2) to clarify the types of speeches and their differing restraints allowed by law; (3)
to discuss the core concepts of prior restraint, content-neutral and content-based regulations
and their constitutional standard of review; (4) to examine the historical difference in the
treatment of restraints between print and broadcast media and stress the standard of review
governing both; and (5) to call attention to the ongoing blurring of the lines of distinction
between print and broadcast media.

E. RE-EXAMINING THE LAW ON FREEDOM OF SPEECH,


OF EXPRESSION AND OF THE PRESS

No law shall be passed abridging the freedom of speech, of expression, or of the


press, or the right of the people peaceably to assemble and petition the
government for redress of grievances.[24]

Freedom of expression has gained recognition as a fundamental principle of every


democratic government, and given a preferred right that stands on a higher level than
substantive economic freedom or other liberties. The cognate rights codified by Article III,
Section 4 of the Constitution, copied almost verbatim from the First Amendment of the U.S. Bill
162
of Rights,[25] were considered the necessary consequence of republican institutions and the
complement of free speech.[26] This preferred status of free speech has also been codified at the
international level, its recognition now enshrined in international law as a customary norm that
binds all nations.[27]

In the Philippines, the primacy and high esteem accorded freedom of expression is a
fundamental postulate of our constitutional system. [28] This right was elevated to constitutional
status in the 1935, the 1973 and the 1987 Constitutions, reflecting our own lesson of history,
both political and legal, that freedom of speech is an indispensable condition for nearly every
other form of freedom.[29] Moreover, our history shows that the struggle to protect the freedom
of speech, expression and the press was, at bottom, the struggle for the indispensable
preconditions for the exercise of other freedoms.[30] For it is only when the people have
unbridled access to information and the press that they will be capable of rendering enlightened
judgments. In the oft-quoted words of Thomas Jefferson, we cannot both be free and ignorant.

E.1. ABSTRACTION OF FREE SPEECH

Surrounding the freedom of speech clause are various concepts that we have adopted as
part and parcel of our own Bill of Rights provision on this basic freedom.[31]What is embraced
under this provision was discussed exhaustively by the Court in Gonzales v. Commission on
Elections, [32] in which it was held:

…At the very least, free speech and free press may be identified with the liberty to
discuss publicly and truthfully any matter of public interest without censorship and
punishment. There is to be no previous restraint on the communication of views
or subsequent liability whether in libel suits, prosecution for sedition, or action for
damages, or contempt proceedings unless there be a clear and present danger of
substantive evil that Congress has a right to prevent. [33]

Gonzales further explained that the vital need of a constitutional democracy for freedom
of expression is undeniable, whether as a means of assuring individual self-fulfillment; of
attaining the truth; of assuring participation by the people in social, including political, decision-
making; and of maintaining the balance between stability and change.[34] As early as the 1920s,
the trend as reflected in Philippine and American decisions was to recognize the broadest scope
and assure the widest latitude for this constitutional guarantee. The trend represents a profound
commitment to the principle that debate on public issue should be uninhibited, robust, and wide-
open. [35]

163
Freedom of speech and of the press means something more than the right to approve
existing political beliefs or economic arrangements, to lend support to official measures, and to
take refuge in the existing climate of opinion on any matter of public consequence.[36] When
atrophied, the right becomes meaningless.[37] The right belongs as well -- if not more – to those
who question, who do not conform, who differ.[38] The ideas that may be expressed under this
freedom are confined not only to those that are conventional or acceptable to the majority. To
be truly meaningful, freedom of speech and of the press should allow and even encourage the
articulation of the unorthodox view, though it be hostile to or derided by others; or though such
view “induces a condition of unrest, creates dissatisfaction with conditions as they are, or even
stirs people to anger.”[39] To paraphrase Justice Holmes, it is freedom for the thought that we
hate, no less than for the thought that agrees with us. [40]

The scope of freedom of expression is so broad that it extends protection to nearly all
forms of communication. It protects speech, print and assembly regarding secular as well as
political causes, and is not confined to any particular field of human interest. The protection
covers myriad matters of public interest or concern embracing all issues, about which
information is needed or appropriate, so as to enable members of society to cope with the
exigencies of their period. The constitutional protection assures the broadest possible exercise of
free speech and free press for religious, political, economic, scientific, news, or informational
ends, inasmuch as the Constitution's basic guarantee of freedom to advocate ideas is not
confined to the expression of ideas that are conventional or shared by a majority.

The constitutional protection is not limited to the exposition of ideas. The protection
afforded free speech extends to speech or publications that are entertaining as well as
instructive or informative. Specifically, in Eastern Broadcasting Corporation (DYRE) v.
[41]
Dans, this Court stated that all forms of media, whether print or broadcast, are entitled to the
broad protection of the clause on freedom of speech and of expression.

While all forms of communication are entitled to the broad protection of freedom of
expression clause, the freedom of film, television and radio broadcasting is somewhat lesser in
scope than the freedom accorded to newspapers and other print media, as will be subsequently
discussed.

E.2. DIFFERENTIATION: THE LIMITS & RESTRAINTS OF FREE SPEECH

164
From the language of the specific constitutional provision, it would appear that the right to
free speech and a free press is not susceptible of any limitation. But the realities of life in a
complex society preclude a literal interpretation of the provision prohibiting the passage of a law
that would abridge such freedom. For freedom of expression is not an absolute, [42] nor is it an
“unbridled license that gives immunity for every possible use of language and prevents the
punishment of those who abuse this freedom.”

Thus, all speech are not treated the same. Some types of speech may be subjected to
some regulation by the State under its pervasive police power, in order that it may not be
injurious to the equal right of others or those of the community or society.[43] The difference in
treatment is expected because the relevant interests of one type of speech, e.g., political
speech, may vary from those of another, e.g., obscene speech. Distinctions have therefore been
made in the treatment, analysis, and evaluation of the permissible scope of restrictions on
various categories of speech. [44] We have ruled, for example, that in our jurisdiction slander or
libel, lewd and obscene speech, as well as “fighting words” are not entitled to constitutional
protection and may be penalized.[45]
Moreover, the techniques of reviewing alleged restrictions on speech (overbreadth,
vagueness, and so on) have been applied differently to each category, either consciously or
unconsciously. [46] A study of free speech jurisprudence—whether here or abroad—will reveal
that courts have developed different tests as to specific types or categories of speech in
concrete situations; i.e., subversive speech; obscene speech; the speech of the broadcast media
and of the traditional print media; libelous speech; speech affecting associational rights; speech
before hostile audiences; symbolic speech; speech that affects the right to a fair trial; and
speech associated with rights of assembly and petition. [47]

Generally, restraints on freedom of speech and expression are evaluated by either or a


combination of three tests, i.e., (a) the dangerous tendency doctrine which permits limitations
on speech once a rational connection has been established between the speech restrained and
the danger contemplated; [48] (b) the balancing of interests tests, used as a standard when
courts need to balance conflicting social values and individual interests, and requires a conscious
and detailed consideration of the interplay of interests observable in a given situation of type of
situation; [49] and (c) the clear and present danger rule which rests on the premise that speech
may be restrained because there is substantial danger that the speech will likely lead to an evil
the government has a right to prevent. This rule requires that the evil consequences sought to
be prevented must be substantive, “extremely serious and the degree of imminence extremely
high.” [50]

165
As articulated in our jurisprudence, we have applied either the dangerous tendency
doctrine or clear and present danger test to resolve free speech challenges. More recently, we
have concluded that we have generally adhered to the clear and present danger test. [51]

E.3. IN FOCUS: FREEDOM OF THE PRESS

Much has been written on the philosophical basis of press freedom as part of the larger
right of free discussion and expression. Its practical importance, though, is more easily grasped.
It is the chief source of information on current affairs. It is the most pervasive and perhaps most
powerful vehicle of opinion on public questions. It is the instrument by which citizens keep their
government informed of their needs, their aspirations and their grievances. It is the sharpest
weapon in the fight to keep government responsible and efficient. Without a vigilant press, the
mistakes of every administration would go uncorrected and its abuses unexposed. As Justice
Malcolm wrote in United States v. Bustos:[52]

The interest of society and the maintenance of good government demand a full
discussion of public affairs. Complete liberty to comment on the conduct of public
men is a scalpel in the case of free speech. The sharp incision of its probe relieves
the abscesses of officialdom. Men in public life may suffer under a hostile and
unjust accusation; the wound can be assuaged with the balm of clear conscience.

Its contribution to the public weal makes freedom of the press deserving of extra
protection. Indeed, the press benefits from certain ancillary rights. The productions of writers
are classified as intellectual and proprietary. Persons who interfere or defeat the freedom to
write for the press or to maintain a periodical publication are liable for damages, be they private
individuals or public officials.

E.4. ANATOMY OF RESTRICTIONS: PRIOR RESTRAINT, CONTENT-NEUTRAL AND CONTENT-


BASED REGULATIONS

Philippine jurisprudence, even as early as the period under the 1935 Constitution, has
recognized four aspects of freedom of the press. These are (1) freedom from prior restraint; (2)
freedom from punishment subsequent to publication; [53] (3) freedom of access to
information; [54] and (4) freedom of circulation.[55]

166
Considering that petitioner has argued that respondents’ press statement constitutes a form
of impermissible prior restraint, a closer scrutiny of this principle is in order, as well as its sub-
specie of content-based (as distinguished from content-neutral) regulations.

At this point, it should be noted that respondents in this case deny that their acts
constitute prior restraints. This presents a unique tinge to the present challenge, considering
that the cases in our jurisdiction involving prior restrictions on speech never had any issue of
whether the governmental act or issuance actually constituted prior restraint. Rather, the
determinations were always about whether the restraint was justified by the Constitution.

Be that as it may, the determination in every case of whether there is an impermissible


restraint on the freedom of speech has always been based on the circumstances of each case,
including the nature of the restraint. And in its application in our jurisdiction, the parameters of
this principle have been etched on a case-to-case basis, always tested by scrutinizing the
governmental issuance or act against the circumstances in which they operate, and then
determining the appropriate test with which to evaluate.

Prior restraint refers to official governmental restrictions on the press or other forms of
expression in advance of actual publication or dissemination.[56] Freedom from prior restraint is
largely freedom from government censorship of publications, whatever the form of censorship,
and regardless of whether it is wielded by the executive, legislative or judicial branch of the
government. Thus, it precludes governmental acts that required approval of a proposal to
publish; licensing or permits as prerequisites to publication including the payment of license
taxes for the privilege to publish; and even injunctions against publication. Even the closure of
the business and printing offices of certain newspapers, resulting in the discontinuation of
their printing and publication, are deemed as previous restraint or censorship. [57] Any law or
official that requires some form of permission to be had before publication can be made,
commits an infringement of the constitutional right, and remedy can be had at the courts.

Given that deeply ensconced in our fundamental law is the hostility against all prior
restraints on speech, and any act that restrains speech is presumed invalid,[58] and “any act
that restrains speech is hobbled by the presumption of invalidity and should be greeted with
furrowed brows,” [59] it is important to stress not all prior restraints on speech are
invalid. Certain previous restraints may be permitted by the Constitution, but determined only
upon a careful evaluation of the challenged act as against the appropriate test by which it should
be measured against.

167
Hence, it is not enough to determine whether the challenged act constitutes some form of
restraint on freedom of speech. A distinction has to be made whether the restraint is (1)
a content-neutral regulation, i.e., merely concerned with the incidents of the speech, or one that
merely controls the time, place or manner, and under well defined standards;[60] or (2)
a content-based restraint or censorship, i.e., the restriction is based on the subject matter of
the utterance or speech. [61] The cast of the restriction determines the test by which the
challenged act is assayed with.

When the speech restraints take the form of a content-neutral regulation, only a substantial
governmental interest is required for its validity.[62] Because regulations of this type are not
designed to suppress any particular message, they are not subject to the strictest form of
judicial scrutiny but an intermediate approach—somewhere between the mere rationality that is
required of any other law and the compelling interest standard applied to content-based
restrictions.[63] The test is called intermediate because the Court will not merely rubberstamp
the validity of a law but also require that the restrictions be narrowly-tailored to promote an
important or significant governmental interest that is unrelated to the suppression of
expression. The intermediate approach has been formulated in this manner:

A governmental regulation is sufficiently justified if it is within the constitutional


power of the Government, if it furthers an important or substantial governmental
interest; if the governmental interest is unrelated to the suppression of free
expression; and if the incident restriction on alleged [freedom of speech &
expression] is no greater than is essential to the furtherance of that interest. [64]

On the other hand, a governmental action that restricts freedom of speech or of the
press based on content is given the strictest scrutiny in light of its inherent and invasive
impact. Only when the challenged act has overcome the clear and present danger rule will it
pass constitutional muster,[65] with the government having the burden of overcoming the
presumed unconstitutionality.

Unless the government can overthrow this presumption, the content-based restraint will be
struck down.[66]
With respect to content-based restrictions, the government must also show the type of
harm the speech sought to be restrained would bring about— especially the gravity and the
imminence of the threatened harm – otherwise the prior restraint will be invalid. Prior restraint
on speech based on its content cannot be justified by hypothetical fears, “but only by showing a
substantive and imminent evil that has taken the life of a reality already on ground.”[67] As
168
formulated, “the question in every case is whether the words used are used in such
circumstances and are of such a nature as
to create a clear and present danger that they will bring about the substantive evils
that Congress has a right to prevent. It is a question of proximity and degree.”[68]

The regulation which restricts the speech content must also serve an important or
substantial government interest, which is unrelated to the suppression of free expression.[69]

Also, the incidental restriction on speech must be no greater than what is essential to the
furtherance of that interest. [70] A restriction that is so broad that it encompasses more than
what is required to satisfy the governmental interest will be invalidated. [71] The regulation,
therefore, must be reasonable and narrowly drawn to fit the regulatory purpose, with the least
restrictive means undertaken. [72]

Thus, when the prior restraint partakes of a content-neutral regulation, it is subjected to an


intermediate review. A content-based regulation,[73] however, bears a heavy presumption of
invalidity and is measured against the clear and present danger rule. The latter will pass
constitutional muster only if justified by a compelling reason, and the restrictions imposed are
neither overbroad nor vague. [74]

Applying the foregoing, it is clear that the challenged acts in the case at bar need to be
subjected to the clear and present danger rule, as they are content-basedrestrictions. The acts
of respondents focused solely on but one object—a specific content— fixed as these were on the
alleged taped conversations between the President and a COMELEC official. Undoubtedly these
did not merely provide regulations as to the time, place or manner of the dissemination of
speech or expression.

E.5. Dichotomy of Free Press: Print v. Broadcast Media

Finally, comes respondents’ argument that the challenged act is valid on the ground that
broadcast media enjoys free speech rights that are lesser in scope to that of print media. We
next explore and test the validity of this argument, insofar as it has been invoked to validate a
content-based restriction on broadcast media.

169
The regimes presently in place for each type of media differ from one other. Contrasted
with the regime in respect of books, newspapers, magazines and traditional printed matter,
broadcasting, film and video have been subjected to regulatory schemes.

The dichotomy between print and broadcast media traces its origins in the United States.
There, broadcast radio and television have been held to have limited First Amendment
protection,[75] and U.S. Courts have excluded broadcast media from the application of the “strict
scrutiny” standard that they would otherwise apply to content-based restrictions.[76] According
to U.S. Courts, the three major reasons why broadcast media stands apart from print media are:
(a) the scarcity of the frequencies by which the medium operates [i.e., airwaves are physically
limited while print medium may be limitless]; [77] (b) its “pervasiveness” as a medium; and (c)
its unique accessibility to children.[78] Because cases involving broadcast media need not follow
“precisely the same approach that [U.S. courts] have applied to other media,” nor go “so far as
to demand that such regulations serve ‘compelling’ government interests,”[79] they are decided
on whether the “governmental restriction” is narrowly tailored to further a substantial
governmental interest,”[80] or the intermediate test.

As pointed out by respondents, Philippine jurisprudence has also echoed a differentiation


in treatment between broadcast and print media. Nevertheless, a review of Philippine case law
on broadcast media will show that—as we have deviated with the American conception of the Bill
of Rights[81]— we likewise did not adopt en masse the U.S. conception of free speech as it relates
to broadcast media, particularly as to which test would govern content-based prior restraints.

Our cases show two distinct features of this dichotomy. First, the difference in treatment,
in the main, is in the regulatory scheme applied to broadcast media that is not imposed on
traditional print media, and narrowly confined to unprotected speech (e.g., obscenity,
pornography, seditious and inciting speech), or is based on a compelling government interest
that also has constitutional protection, such as national security or the electoral process.

Second, regardless of the regulatory schemes that broadcast media is subjected to, the
Court has consistently held that the clear and present danger test applies to content-based
restrictions on media, without making a distinction as to traditional print or broadcast media.

The distinction between broadcast and traditional print media was first enunciated
in Eastern Broadcasting Corporation (DYRE) v. Dans,[82] wherein it was held that “[a]ll forms of
media, whether print or broadcast, are entitled to the broad protection of the freedom of speech

170
and expression clause. The test for limitations on freedom of expression continues to be the
clear and present danger rule…”[83]

Dans was a case filed to compel the reopening of a radio station which had been
summarily closed on grounds of national security. Although the issue had become moot and
academic because the owners were no longer interested to reopen, the Court still proceeded to
do an analysis of the case and made formulations to serve as guidelines for all inferior courts
and bodies exercising quasi-judicial functions. Particularly, the Court made a detailed exposition
as to what needs be considered in cases involving broadcast media. Thus:[84]

xxx xxx xxx

(3) All forms of media, whether print or broadcast, are entitled to the broad
protection of the freedom of speech and expression clause. The test for
limitations on freedom of expression continues to be the clear and present
danger rule, that words are used in such circumstances and are of such a
nature as to create a clear and present danger that they will bring about the
substantive evils that the lawmaker has a right to prevent, In
his Constitution of the Philippines (2nd Edition, pp. 569-570) Chief Justice
Enrique M. Fernando cites at least nine of our decisions which apply the test.
More recently, the clear and present danger test was applied in J.B.L. Reyes
in behalf of theAnti-Bases Coalition v. Bagatsing. (4) The clear and present
danger test, however, does not lend itself to a simplistic and all embracing
interpretation applicable to all utterances in all forums.
Broadcasting has to be licensed. Airwave frequencies have to be allocated
among qualified users. A broadcast corporation cannot simply appropriate a
certain frequency without regard for government regulation or for the rights
of others.
All forms of communication are entitled to the broad protection of the
freedom of expression clause. Necessarily, however, the freedom of
television and radio broadcasting is somewhat lesser in scope than the
freedom accorded to newspaper and print media.
The American Court in Federal Communications Commission v. Pacifica
Foundation (438 U.S. 726), confronted with a patently offensive and indecent
regular radio program, explained why radio broadcasting, more than other
forms of communications, receives the most limited protection from the free
expression clause. First, broadcast media have established a uniquely
pervasive presence in the lives of all citizens, Material presented over the
airwaves confronts the citizen, not only in public, but in the privacy of his
home. Second, broadcasting is uniquely accessible to children. Bookstores
and motion picture theaters may be prohibited from making certain material
available to children, but the same selectivity cannot be done in radio or
television, where the listener or viewer is constantly tuning in and out.

171
Similar considerations apply in the area of national security.
The broadcast media have also established a uniquely pervasive presence in
the lives of all Filipinos. Newspapers and current books are found only in
metropolitan areas and in the poblaciones of municipalities accessible to fast
and regular transportation. Even here, there are low income masses who find
the cost of books, newspapers, and magazines beyond their humble means.
Basic needs like food and shelter perforce enjoy high priorities.
On the other hand, the transistor radio is found everywhere. The television
set is also becoming universal. Their message may be simultaneously
received by a national or regional audience of listeners including the
indifferent or unwilling who happen to be within reach of a blaring radio or
television set. The materials broadcast over the airwaves reach every person
of every age, persons of varying susceptibilities to persuasion, persons of
different I.Q.s and mental capabilities, persons whose reactions to
inflammatory or offensive speech would be difficult to monitor or predict. The
impact of the vibrant speech is forceful and immediate. Unlike readers of the
printed work, the radio audience has lesser opportunity to cogitate analyze,
and reject the utterance.
(5) The clear and present danger test, therefore, must take the particular
circumstances of broadcast media into account. The supervision of radio
stations-whether by government or through self-regulation by the industry
itself calls for thoughtful, intelligent and sophisticated handling.
The government has a right to be protected against broadcasts which incite
the listeners to violently overthrow it. Radio and television may not be used
to organize a rebellion or to signal the start of widespread uprising. At the
same time, the people have a right to be informed. Radio and television
would have little reason for existence if broadcasts are limited to bland,
obsequious, or pleasantly entertaining utterances. Since they are the most
convenient and popular means of disseminating varying views on public
issues, they also deserve special protection.
(6) The freedom to comment on public affairs is essential to the vitality of a
representative democracy. In the 1918 case of United States v. Bustos (37
Phil. 731) this Court was already stressing that.
The interest of society and the maintenance of good government demand a
full discussion of public affairs. Complete liberty to comment on the conduct
of public men is a scalpel in the case of free speech. The sharp incision of its
probe relieves the abscesses of officialdom. Men in public life may suffer
under a hostile and an unjust accusation; the wound can be assuaged with
the balm of a clear conscience. A public officer must not be too thin-skinned
with reference to comment upon his official acts. Only thus can the
intelligence and dignity of the individual be exalted.
(7) Broadcast stations deserve the special protection given to all forms of
media by the due process and freedom of expression clauses of the
Constitution. [Citations omitted]

172
It is interesting to note that the Court in Dans adopted the arguments found in U.S.
jurisprudence to justify differentiation of treatment (i.e., the scarcity, pervasiveness and
accessibility to children), but only after categorically declaring that “the test for limitations on
freedom of expression continues to be the clear and present danger rule,” for all forms of media,
whether print or broadcast. Indeed, a close reading of the above-quoted provisions would show
that the differentiation that the Court in Dansreferred to was narrowly restricted to what is
otherwise deemed as “unprotected speech” (e.g., obscenity, national security, seditious and
inciting speech), or to validate a licensing or regulatory scheme necessary to allocate the limited
broadcast frequencies, which is absent in print media. Thus, when this Court declared
in Dans that the freedom given to broadcast media was “somewhat lesser in scope than the
freedom accorded to newspaper and print media,” it was not as to what test should be applied,
but the context by which requirements of licensing, allocation of airwaves, and application of
[85]
norms to unprotected speech.

In the same year that the Dans case was decided, it was reiterated in Gonzales v.
Katigbak,[86] that the test to determine free expression challenges was the clear and present
danger, again without distinguishing the media.[87] Katigbak, strictly speaking, does not treat of
broadcast media but motion pictures. Although the issue involved obscenity standards as applied
to movies,[88] the Court concluded its decision with the following obiter dictum that a less liberal
approach would be used to resolve obscenity issues in television as opposed to motion pictures:

All that remains to be said is that the ruling is to be limited to the concept of
obscenity applicable to motion pictures. It is the consensus of this Court that
where television is concerned, a less liberal approach calls for observance. This is
so because unlike motion pictures where the patrons have to pay their way,
television reaches every home where there is a set. Children then will likely be
among the avid viewers of the programs therein shown…..It cannot be denied
though that the State as parens patriae is called upon to manifest an attitude of
caring for the welfare of the young.

More recently, in resolving a case involving the conduct of exit polls and dissemination of
the results by a broadcast company, we reiterated that the clear and present danger rule is the
test we unquestionably adhere to issues that involve freedoms of speech and of the press.[89]

This is not to suggest, however, that the clear and present danger rule has been applied to
all cases that involve the broadcast media. The rule applies to all media, including broadcast,
but only when the challenged act is a content-based regulation that infringes on free speech,
expression and the press. Indeed, in Osmena v. COMELEC,[90] which also involved broadcast
173
media, the Court refused to apply the clear and present danger rule to a COMELEC regulation of
time and manner of advertising of political advertisements because the challenged restriction
was content-neutral.[91] And in a case involving due process and equal protection issues, the
Court inTelecommunications and Broadcast Attorneys of the Philippines v. COMELEC[92] treated
a restriction imposed on a broadcast media as a reasonable condition for the grant of the
media’s franchise, without going into which test would apply.

That broadcast media is subject to a regulatory regime absent in print media is observed
also in other jurisdictions, where the statutory regimes in place over broadcast media include
elements of licensing, regulation by administrative bodies, and censorship. As explained by a
British author:

The reasons behind treating broadcast and films differently from the print media
differ in a number of respects, but have a common historical basis. The stricter
system of controls seems to have been adopted in answer to the view that owing
to their particular impact on audiences, films, videos and broadcasting require a
system of prior restraints, whereas it is now accepted that books and other
printed media do not. These media are viewed as beneficial to the public in a
number of respects, but are also seen as possible sources of harm.[93]

Parenthetically, these justifications are now the subject of debate. Historically, the scarcity
of frequencies was thought to provide a rationale. However, cable and satellite television have
enormously increased the number of actual and potential channels. Digital technology will
further increase the number of channels available. But still, the argument persists that
broadcasting is the most influential means of communication, since it comes into the home, and
so much time is spent watching television. Since it has a unique impact on people and affects
children in a way that the print media normally does not, that regulation is said to be necessary
in order to preserve pluralism. It has been argued further that a significant main threat to free
expression—in terms of diversity—comes not from government, but from private corporate
bodies. These developments show a need for a reexamination of the traditional notions of the
scope and extent of broadcast media regulation. [94]

The emergence of digital technology -- which has led to the convergence of broadcasting,
telecommunications and the computer industry -- has likewise led to the question of whether the
regulatory model for broadcasting will continue to be appropriate in the converged
[95]
environment. Internet, for example, remains largely unregulated, yet the Internet and the
broadcast media share similarities, [96] and the rationales used to support broadcast regulation
apply equally to the Internet.[97] Thus, it has been argued that courts, legislative bodies and the
174
government agencies regulating media must agree to regulate both, regulate neither or develop
a new regulatory framework and rationale to justify the differential treatment. [98]

F. The Case At Bar

Having settled the applicable standard to content-based restrictions on broadcast media,


let us go to its application to the case at bar. To
recapitulate, a governmental action that restricts freedom of speech
or of the press based on content is given the strictest
scrutiny, with the government having the burden of overcoming the
presumed unconstitutionality by the clear and present danger rule. This rule applies equally
to all kinds of media, including broadcast media.

This outlines the procedural map to follow in cases like the one at bar as it spells out the
following: (a) the test; (b) the presumption; (c) the burden of proof; (d) the party to discharge
the burden; and (e) the quantum of evidence necessary. On the basis of the records of the case
at bar, respondents who have the burden to show that these acts do not abridge freedom of
speech and of the press failed to hurdle the clear and present danger test. It appears that
the great evil which government wants to prevent is the airing of a tape recording in alleged
violation of the anti-wiretapping law. The records of the case at bar, however, are confused and
confusing, and respondents’ evidence falls short of satisfying the clear and present danger
test. Firstly, the various statements of the Press Secretary obfuscate the identity of the voices
in the tape recording. Secondly, the integrity of the taped conversation is also suspect. The
Press Secretary showed to the public two versions, one supposed to be a “complete” version and
the other, an “altered” version. Thirdly, the evidence of the respondents on the who’s and the
how’s of the wiretapping act is ambivalent, especially considering the tape’s different versions.
The identity of the wire-tappers, the manner of its commission and other related and relevant
proofs are some of the invisibles of this case. Fourthly, given all these unsettled facets of the
tape, it is even arguable whether its airing would violate the anti-wiretapping law.

We rule that not every violation of a law will justify straitjacketing the exercise of freedom
of speech and of the press. Our laws are of different kinds and doubtless, some of them provide
norms of conduct which even if violated have only an adverse effect on a person’s private
comfort but does not endanger national security. There are laws of great significance but their
violation, by itself and without more, cannot support suppression of free speech and free press.
In fine, violation of law is just a factor, a vital one to be sure, which should be

175
weighed in adjudging whether to restrain freedom of speech and of the press. The totality of
the injurious effects of the violation to private and public interest must be calibrated in light of
the preferred status accorded by the Constitution and by related international covenants
protecting freedom of speech and of the press. In calling for a careful and calibrated
measurement of the circumference of all these factors to determine compliance with the clear
and present danger test, the Court should not be misinterpreted as devaluing violations of
law. By all means, violations of law should be vigorously prosecuted by the
State for they breed their own evil consequence. But to repeat, the need to prevent their
violation cannot per se trump the exercise of free speech and free press, a preferred right
whose breach can lead to greater evils. For this failure of the respondents alone to offer proof
to satisfy the clear and present danger test, the Court has no option but to uphold the exercise
of free speech and free press. There is no showing that the feared violation of the anti-
wiretapping law clearly endangers the national security of the State.

This is not all the faultline in the stance of the respondents. We slide to the issue of
whether the mere press statements of the Secretary of Justice and of the NTC in question
constitute a form of content-based prior restraint that has transgressed the Constitution. In
resolving this issue, we hold that it is not decisive that the press statements made by
respondents were not reduced in or followed up with formal orders or circulars. It is sufficient
that the press statements were made by respondents while in the exercise of their official
functions. Undoubtedly, respondent Gonzales made his statements as Secretary of Justice, while
the NTC issued its statement as the regulatory body of media. Any act done, such as a speech
uttered, for and on behalf of the government in an official capacity is covered by the rule on
prior restraint. The concept of an “act” does not limit itself to acts already converted to a formal
order or official circular. Otherwise, the non formalization of an act into an official order or
circular will result in the easy circumvention of the prohibition on prior restraint. The press
statements at bar are acts that should be struck down as they constitute impermissible forms of
prior restraints on the right to free speech and press.

There is enough evidence of chilling effect of the complained acts on


record. The warnings given to media came from no less the NTC, a regulatory agency that can
cancel the Certificate of Authority of the radio and broadcast media. They also came from the
Secretary of Justice, the alter ego of the Executive, who wields the awesome power to prosecute
those perceived to be violating the laws of the land. After the warnings, the KBP inexplicably
joined the NTC in issuing an ambivalent Joint Press Statement. After the warnings, petitioner
Chavez was left alone to fight this battle for freedom of speech and of the press. This silence on

176
the sidelines on the part of some media practitioners is too deafening to be the subject of
misinterpretation.

The constitutional imperative for us to strike down unconstitutional acts should always be
exercised with care and in light of the distinct facts of each case. For there are no hard and fast
rules when it comes to slippery constitutional questions, and the limits and construct of relative
freedoms are never set in stone. Issues revolving on their construct must be decided on a case
to case basis, always based on the peculiar shapes and shadows of each case. But in cases
where the challenged acts are patent invasions of a constitutionally protected right, we should
be swift in striking them down as nullities per se. A blow too soon struck for freedom is preferred
than a blow too late.

In VIEW WHEREOF, the petition is GRANTED. The writs of certiorari and prohibition are
hereby issued, nullifying the official statements made by respondents on June 8, and 11, 2005
warning the media on airing the alleged wiretapped conversation between the President and
other personalities, for constituting unconstitutional prior restraint on the exercise of freedom of
speech and of the press

SO ORDERED.

REYNATO S. PUNO
Chief Justice

WE CONCUR:

LEONARDO A. QUISUMBING CONSUELO YNARES-SANTIAGO


Associate Justice Associate Justice

ANGELINA SANDOVAL-GUTIERREZ ANTONIO T. CARPIO


Associate Justice Associate Justice

177
MA. ALICIA AUSTRIA-MARTINEZ RENATO C. CORONA
Associate Justice Associate Justice

CONCHITA CARPIO MORALES ADOLFO S. AZCUNA


Associate Justice Associate Justice

DANTE O. TINGA MINITA V. CHICO-NAZARIO


Associate Justice Associate Justice

PRESBITERO J. VELASCO, JR. ANTONIO EDUARDO B. NACHURA


Associate Justice Associate Justice

RUBEN T. REYES TERESITA LEONARDO-DE CASTRO


Associate Justice Associate Justice

178
CERTIFICATION

Pursuant to Section 13, Article VIII of the Constitution, I certify that the conclusions in the
above decision had been reached in consultation before the case was assigned to the writer of
the opinion of the Court.

REYNATO S. PUNO
Chief Justice

[1]
G.R. No. 103956, March 31, 1992, 207 SCRA 712.
[2]
218 Phil. 754 (1984).
[3]
G.R. No. 147571, May 5, 2001, 357 SCRA 496.
[4]
G.R. No. 169838, April 25, 2006, 488 SCRA 226.
[5]
Rollo, pp. 6-7 (citing the Philippine Daily Inquirer (PDI), June 7, 2005, pp. A1, A18; PDI,
June 14, 2005, p. A1); and p. 58.
[6]
Id. at 7-8 (citing the Manila Standard, June 10, 2005, p. A2); and 58.
[7]
Id. at 7-8 and 59.
[8]
Id.
[9]
Id. at 8-9 and 59.
[10]
Id. at 9.
[11]
Id. at 10-12, 43-44, 60-62.
[12]
Id. at 62-63, 86-87.
[13]
Id. at 6.
[14]
Respondents have “committed blatant violations of the freedom of expression and of the
press and the right of the people to information on matters of public concern enshrined in
Article III, Sections 4 and 7 of the 1987 Constitution. Id. at 18. Petitioner also argued that
respondent NTC acted beyond its powers when it issued the press release of June 11, 2005.
Id.
[15]
Id. at 6.
[16]
Through the Comment filed by the Solicitor-General. Id. at 56-83.
[17]
Id. at 71-73.
[18]
Id. at 74-75.
[19]
The Court will exercise its power of judicial review only if the case is brought before it by
a party who has the legal standing to raise the constitutional or legal question. “Legal
standing” means a personal and substantial interest in the case such that the party has
sustained or will sustain direct injury as a result of the government act that is being
challenged. The term “interest” is material interest, an interest in issue and to be affected
by the decree, as distinguished from mere interest in the question involved, or a mere
incidental interest. Pimentel v. Executive Secretary, G.R. No. 158088, July 6, 2005, 462

179
SCRA 622, citing Joya vs. Presidential Commission on Good Government, G.R. No. 96541,
August 24, 1993, 225 SCRA 568. See Kilosbayan, Inc. v. Morato, G.R. No. 118910, July 17,
1995, 246 SCRA 540, 562–563; and Agan v. PIATCO(Decision), 450 Phil. 744 (2003).
[20]
Araneta v. Dinglasan, 84 Phil. 368, 373 (1949), cited in Osmeña v. COMELEC, G.R. No.
100318, July 30, 1991, 199 SCRA 750.
[21]
See Agan v. PIATCO (Decision), 450 Phil. 744 (2003).
[22]
Philconsa v. Jimenez, 122 Phil. 894 (1965); Civil Liberties Union v. Executive Secretary,
G.R. No. 83896, February 22, 1991, 194 SCRA 317; Guingona v. Carague, G.R. No. 94571,
April 22, 1991, 196 SCRA 221; Osmeña v. COMELEC, G.R. No. 100318, July 30, 1991, 199
SCRA 750; Basco v. PAGCOR, 274 Phil. 323 (1991); Carpio v. Executive Secretary, G.R.
No. 96409, February 14, 1992, 206 SCRA 290; Del Mar v. PAGCOR, 400 Phil. 307 (2000).
[23]
Basco v. PAGCOR, 274 Phil. 323 (1991), citing Kapatiran ng mga Naglilingkod sa
Pamahalaan ng Pilipinas Inc. v. Tan, G.R. No. L-81311, June 30, 1988, 163 SCRA 371.
[24]
1987 PHIL. CONST. Art. III, §4.
[25]
U.S. Bill of Rights, First Amendment. (“Congress shall make no law…abridging the
freedom of speech, or of the press, or the right of the people peaceably to assemble, and
to petition the Government for a redress of grievances.”)
[26]
The First Amendment was so crafted because the founders of the American government
believed -- as a matter of history and experience -- that the freedom to express personal
opinions was essential to a free government. See LARRY KRAMER, THE PEOPLE
THEMSELVES: POPULAR CONSTITUTION AND JUDICIAL REVIEW (2004).
[27]
Article 19 of the 1948 Universal Declaration on Human Rights (UDHR) states:
“Everyone has the right to freedom of opinion and expression; this right includes the
right to hold
opinions without interference and to seek, receive and impart information and ideas
through any media and regardless of frontiers.” Although the UDHR is not binding as a
treaty, many of its provisions have acquired binding status on States and are now part of
customary international law. Article 19 forms part of the UDHR principles that have been
transformed into binding norms. Moreover, many of the rights in the UDHR were included
in and elaborated on in the International Covenant on Civil and Political Rights (ICCPR), a
treaty ratified by over 150 States, including the Philippines. The recognition of freedom of
expression is also found in regional human rights instruments, namely, the European
Convention on Human Rights (Article 10), the American Convention on Human Rights
(Article 10), and the African Charter on Human and Peoples’ Rights (Article 9).
[28]
Gonzales v. COMELEC, 137 Phil. 471, 492 (1969).
[29]
Salonga v. Cruz-Pano, G.R. 59524, February 18, 1985, 134 SCRA 458-459; Gonzales v.
COMELEC, 137 Phil. 489, 492-3 (1969); Philippine Blooming Mills Employees Organization
v. Philippine Blooming Mills Co., 151-A Phil. 676-677 (1973); National Press Club v.
COMELEC, G.R. No. 102653, March 5, 1992, 207 SCRA 1, 9; Adiong v. COMELEC, G.R. No.
103956, March 31, 1992, 207 SCRA 712, 715.
[30]
Indeed, the struggle that attended the recognition of the value of free expression was
discussed by Justice Malcolm in the early case United States v. Bustos, 37 Phil. 731, 739
(1918). Justice Malcolm generalized that the freedom of speech as cherished in democratic
countries was unknown in the Philippine Islands before 1900. Despite the presence of
pamphlets and books early in the history of the Philippine Islands, the freedom of speech
was alien to those who were used to obeying the words of barangay lords and, ultimately,
the colonial monarchy. But ours was a history of struggle for that specific right: to be able
to express ourselves especially in the governance of this country. Id.
[31]
Id.
180
[32]
137 Phil. 471, 492 (1969).
[33]
Id.
[34]
Id. at 493, citing Thomas I. Emerson, Toward a General Theory of the First Amendment,
72 Yale Law Journal 877 (1963).
[35]
Id. citing New York Times Co. v. Sullivan, 376 US 254, 270 (1964).
[36]
Id.
[37]
Id.
[38]
Id.
[39]
Id. citing Terminiello v. City of Chicago, 337 US 1, 4 (1949).
[40]
Id. citing U.S. v. Schwimmer, 279 US 644, 655 (1929).
[41]
G.R. No. L-59329, July 19, 1985, 137 SCRA 628.
[42]
Gonzales v. COMELEC, 137 Phil. 471, 494(1969).
[43]
HECTOR S. DE LEON, I PHILIPPINE CONSTITUTIONAL LAW: PRINCIPLES AND CASES 485
(2003) [Hereinafter DE LEON, CONSTITUTIONAL LAW].
[44]
See JOHN E. NOWAK & RONALD D. ROTUNDA, CONSTITUTIONAL LAW §16.1,
1131 (7th ed.2000 [Hereinafter NOWAK & ROTUNDA, CONSTITUTIONAL LAW].
[45]
DE LEON, CONSTITUTIONAL LAW at 485. Laws have also limited the freedom of speech
and of the press, or otherwise affected the media and freedom of expression. The
Constitution itself imposes certain limits (such as Article IX on the Commission on
Elections, and Article XVI prohibiting foreign media ownership); as do
the Revised Penal Code (with provisions on national security, libel and obscenity),
the Civil Code (which contains two articles on privacy), the Rules of Court (on the fair
administration of justice and contempt) and certain presidential decrees. There is also a
“shield law,” or Republic Act No. 53, as amended by Republic Act No. 1477. Section 1 of
this law provides protection for non-disclosure of sources of information, without
prejudice to one’s liability under civil and criminal laws. The publisher, editor, columnist
or duly accredited reporter of a newspaper, magazine or periodical of general circulation
cannot be compelled to reveal the source of any information or news report appearing in
said publication, if the information was released in confidence to such publisher, editor or
reporter unless the court or a Committee of Congress finds that such revelation is
demanded by the security of the state.
[46]
See NOWAK & ROTUNDA, CONSTITUTIONAL LAW §16.1, 1131 (7th ed.2000).
[47]
Id.
[48]
Cabansag v. Fernandez, 102 Phil. 151 (1957); Gonzales v. COMELEC, 137 Phil. 471
(1969). See People v. Perez, 4 Phil. 599 (1905); People v. Nabong, 57 Phil. 455
(1933); People v. Feleo, 57 Phil. 451 (1933).
[49]
This test was used by J. Ruiz-Castro in his Separate Opinion in Gonzales v.
COMELEC, 137 Phil. 471, 532-537 (1969).
[50]
Cabansag v. Fernandez, 102 Phil. 151 (1957).
[51]
ABS-CBN Broadcasting Corp. v. COMELEC, 380 Phil. 780, 794 (2000).
[52]
See U.S. v. Bustos, 37 Phil. 731 (1918).
[53]
The aspect of freedom from liability subsequent to publication precludes liability for
completed publications of views traditionally held innocent. Otherwise, the prohibition on
prior restraint would be meaningless, as the unrestrained threat of subsequent punishment,
by itself, would be an effective prior restraint. Thus, opinions on public issues cannot be
punished when published, merely because the opinions are novel or controversial, or
because they clash with current doctrines. This fact does not imply that publishers and
editors are never liable for what they print. Such freedom gives no immunity from laws
punishing scandalous or obscene matter, seditious or disloyal writings, and libelous or
181
insulting words. As classically expressed, the freedom of the press embraces at the very
least the freedom to discuss truthfully and publicly matters of public concern, without
previous restraint or fear of subsequent punishment. For discussion to be innocent, it must
be truthful, must concern something in which people in general take a healthy interest, and
must not endanger some important social end that the government by law protects.
See JOAQUIN G. BERNAS, S.J., THE 1987 CONSTITUTION OF THE REPUBLIC OF THE
PHILIPPINES: A COMMENTARY, 225 (2003 ed.).
[54]
Freedom of access to information regarding matters of public interest is kept real in
several ways. Official papers, reports and documents, unless held confidential and secret by
competent authority in the public interest, are public records. As such, they are open and
subject to reasonable regulation, to the scrutiny of the inquiring reporter or editor.
Information obtained confidentially may be printed without specification of the source; and
that source is closed to official inquiry, unless the revelation is deemed by the courts, or by
a House or committee of Congress, to be vital to the security of the State. Id.
[55]
Freedom of circulation refers to the unhampered distribution of newspapers and other
media among customers and among the general public. It may be interfered with in several
ways. The most important of these is censorship. Other ways include requiring a permit or
license for the distribution of media and penalizing dissemination of copies made without
it;[55] and requiring the payment of a fee or tax, imposed either on the publisher or on the
distributor, with the intent to limit or restrict circulation. These modes of interfering with
the freedom to circulate have been constantly stricken down as unreasonable limitations on
press freedom. Thus, imposing a license tax measured by gross receipts for the privilege of
engaging in the business of advertising in any newspaper, or charging license fees for the
privilege of selling religious books are impermissible restraints on the freedom of
expression. Id. citing Grosjean v. American Press Co., 297 U.S. 233 (1936); Murdock v.
Pennsylvania, 319 U.S. 105 (1943), and American Bible Society v. City of Manila, 101 Phil.
386 (1957). It has been held, however, even in the Philippines, that publishers and
distributors of newspapers and allied media cannot complain when required to pay ordinary
taxes such as the sales tax. The exaction is valid only when the obvious and immediate
effect is to restrict oppressively the distribution of printed matter.
[56]
Id at 225.
[57]
Burgos v. Chief of Staff, 218 Phil. 754 (1984).
[58]
Gonzales v. COMELEC, 137 Phil. 471 (1969); ABS-CBN Broadcasting Corp. v. COMELEC,
380 Phil. 780, 795 (2000) (“Doctrinally, the Court has always ruled in favor of the freedom
of expression, and any restriction is treated an exemption.”); Social Weather Stations v.
COMELEC, G.R. No. 147571, May 5, 2001, 357 SCRA 496 (“[A]ny system of prior restraint
comes to court bearing a heavy burden against its constitutionality. It is the government
which must show justification for enforcement of the restraint.”). See also Iglesia ni Cristo
v. Court of Appeals, 328 Phil. 893 (1996) (religious speech falls within the protection of
free speech).
[59]
Iglesia ni Cristo v. CA, 328 Phil. 893, 928 (1996), citing Near v. Minnesota, 283 US 697
(1931); Bantam Books Inc. v. Sullivan, 372 US 58 (1963); New York Times v. United
States, 403 US 713 (1971).
[60]
See J.B.L. Reyes v. Bagatsing, 210 Phil. 457 (1983), Navarro v. Villegas, G.R. No. L-
31687, February 18, 1970, 31 SCRA 730; Ignacio v. Ela, 99 Phil. 346 (1956); Primicias
v. Fugosa, 80 Phil. 71 (1948).
[61]
Determining if a restriction is content-based is not always obvious. A regulation may be
content-neutral on its face but partakes of a content-based restriction in its application, as
when it can be shown that the government only enforces the restraint as to prohibit one
182
type of content or viewpoint. In this case, the restriction will be treated as a content-based
regulation. The most important part of the time, place, or manner standard is the
requirement that the regulation be content-neutral both as written and applied.
See NOWAK & ROTUNDA, CONSTITUTIONAL LAW §16.1, 1133 (7th ed.2000).
[62]
See Osmeña v. COMELEC, 351 Phil. 692, 718 (1998). The Court looked to Adiong
v. COMELEC, G.R. No. 103456, March 31, 1992, 207 SCRA 712, which had cited a U.S.
doctrine, viz. “A governmental regulation is sufficiently justified if it is within the
constitutional power of the Government, if it furthers an important or substantial
governmental interest; if the governmental interest is unrelated to the suppression of free
expression; and if the incident restriction on alleged [freedom of speech & expression] is no
greater than is essential to the furtherance of that interest.”
[63]
NOWAK & ROTUNDA, CONSTITUTIONAL LAW §16.1, 1133 (7th ed.2000). This was also
called a “deferential standard of review” in Osmeña v. COMELEC, 351 Phil. 692, 718
(1998). It was explained that the clear and present danger rule is not a sovereign remedy
for all free speech problems, and its application to content-neutral regulations would be
tantamount to “using a sledgehammer to drive a nail when a regular hammer is all that is
needed.” Id. at 478.
[64]
Osmeña v. COMELEC, 351 Phil. 692, 717, citing Adiong v. COMELEC, G.R. No. 103956,
March 31, 1992, 207 SCRA 712. It was noted that the test was actually formulated
in United States v. O’Brien, 391 U.S. 367 (1968), which was deemed appropriate for
restrictions on speech which are content-neutral.
[65]
Iglesia ni Cristo v. Court of Appeals, 328 Phil. 893 (1996). In this case, it was found that
the act of respondent Board of Review for Motion Pictures and Television of rating a TV
program with “X”— on the ground that it “offend[s] and constitute[s] an attack against
other religions which is expressly prohibited by law”— was a form of prior restraint and
required the application of the clear and present danger rule.
[66]
Iglesia ni Cristo v. Court of Appeals, 328 Phil. 893 (1996); Gonzales v. COMELEC, 137
Phil. 471 (1969); ABS-CBN Broadcasting Corp. v. COMELEC, 380 Phil. 780 (2000); Social
Weather Stations v. COMELEC, G.R. No. 147571, May 5, 2001, 357 SCRA 496.
[67]
Iglesia ni Cristo v. Court of Appeals, 328 Phil. 893 (1996).
[68]
Schenke v. United States, 249 U.S. 47, 52 (19191), cited in Cabansag v. Fernandez, 102
Phil. 151 (1957); and ABS-CBN Broadcasting Corp. v. COMELEC, 380 Phil. 780, 794
(2000).
[69]
Adiong v. COMELEC, G.R. No. 103956, March 31, 1992, 207 SCRA 712, cited in ABS-CBN
Broadcasting Corp. v. COMELEC, 380 Phil. 780, 795 (2000).
[70]
See Adiong v. COMELEC, G.R. No. 103956, March 31, 1992, 207 SCRA 712,
and Gonzales v. COMELEC, 137 Phil. 471 (1969), cited in ABS-CBN Broadcasting Corp. v.
COMELEC, 380 Phil. 780, 795 (2000).
[71]
See Adiong v. COMELEC, G.R. No. 103956, March 31, 1992, 207 SCRA 712.
[72]
See Osmeña v. COMELEC, 351 Phil. 692 (1998).
[73]
Parenthetically, there are two types of content-based restrictions. First, the government
may be totally banning some type of speech for content (total ban). Second, the
government may be requiring individuals who wish to put forth certain types of speech to
certain times or places so that the type of speech does not adversely affect its
environment. See NOWAK & ROTUNDA, CONSTITUTIONAL LAW §16.1,
1131 (7thed.2000). Both types of conten-based regulations are subject to strict scrutiny
and the clear and present danger rule.

183
[74]
Iglesia ni Cristo v. Court of Appeals, 328 Phil. 893 (1996); Gonzales v. COMELEC, 137
Phil. 471 (1969); ABS-CBN Broadcasting Corp. v. COMELEC, 380 Phil. 780 (2000); Social
Weather Stations v. COMELEC, G.R. No. 147571, May 5, 2001, 357 SCRA 496.
[75]
This is based on a finding that “broadcast regulation involves unique considerations,” and
that “differences in the characteristics of new media justify differences in the First
Amendment standards applied to them.”Red Lion Broad. Co. v. Federal Communications
Commission [FCC], 395 U.S. 367, 386 (1969). See generally National Broadcasting Co. v.
United States, 319 U.S. 190, 219 (1943) (noting that the public interest standard denoted
to the FCC is an expansive power).
[76]
See Federal Communications Commission [FCC] v. Pacifica Foundation, 438 U.S. 726
(1978); Sable Communications v. FCC, 492 U.S. 115 (1989); and Reno v. American Civil
Liberties Union [ACLU], 521 U.S. 844, 874 (1997). In these cases, U.S. courts disregarded
the argument that the offended listener or viewer could simply turn the dial and avoid the
unwanted broadcast [thereby putting print and broadcast media in the same footing],
reasoning that because the broadcast audience is constantly tuning in and out, prior
warnings cannot protect the listener from unexpected program content.
[77]
Red Lion Broad. Co. v. FCC, 395 U.S. 367, 386 (1969). Red Lion involved the application
of the fairness doctrine and whether someone personally attacked had the right to respond
on the broadcast medium within the purview of FCC regulation. The court sustained the
regulation. The Court in Red Lion reasoned that because there are substantially more
individuals who want to broadcast than there are frequencies available, this “scarcity of the
spectrum” necessitates a stricter standard for broadcast media, as opposed to newspapers
and magazines. See generally National Broadcasting v. United States, 319 U.S. 190, 219
(1943) (noting that the public interest standard denoted to the FCC is an expansive
power).
[78]
See Federal Communications Commission v. Pacifica Foundation, 438 U.S. 726
(1978); Sable Communications v. FCC, 492 U.S. 115 (1989); and Reno v. American Civil
Liberties Union [ACLU], 521 U.S. 844, 874 (1997). In FCC v. Pacifica Foundation, involving
an FCC decision to require broadcasters to channel indecent programming away from times
of the day when there is a reasonable risk that children may be in the audience, the U.S.
Court found that the broadcast medium was an intrusive and pervasive one. In reaffirming
that this medium should receive the most limited of First Amendment protections, the U.S.
Court held that the rights of the public to avoid indecent speech trump those of the
broadcaster to disseminate such speech. The justifications for this ruling were two-fold.
First, the regulations were necessary because of the pervasive presence of broadcast media
in American life, capable of injecting offensive material into the privacy of the home, where
the right "to be left alone plainly outweighs the First Amendment rights of an intruder."
Second, the U.S. Court found that broadcasting "is uniquely accessible to children, even
those too young to read." The Court dismissed the argument that the offended listener or
viewer could simply turn the dial and avoid the unwanted broadcast, reasoning that
because the broadcast audience is constantly tuning in and out, prior warnings cannot
protect the listener from unexpected program content.
[79]
FCC v. League of Women Voters, 468 U.S. 364, 376 (1984).
[80]
Id. at 380.
[81]
See Estrada v. Escritor (Resolution), A.M. No. P-02-1651, June 22, 2006 (free exercise of
religion); and Osmeña v. COMELEC, 351 Phil. 692, 718 (1998) (speech restrictions to
promote voting rights). The Court inOsmeña v. COMELEC, for example, noted that it is a
foreign notion to the American Constitution that the government may restrict the speech of
some in order to enhance the relative voice of others [the idea being that voting is a form
184
of speech]. But this Court then declared that the same does not hold true of the Philippine
Constitution, the notion “being in fact an animating principle of that document.” 351 Phil.
692, 718 (1998).
[82]
G.R. No. L-59329, July 19, 1985, 137 SCRA 628.
[83]
Id.
[84]
Id. at 634-637.
[85]
There is another case wherein the Court had occasion to refer to the differentiation
between traditional print media and broadcast media, but of limited application to the case
at bar inasmuch as the issues did not invoke a free-speech challenge, but due process and
equal protection. See Telecommunications and Broadcast Attorneys of the Philippines, Inc.
v. COMELEC, 352 Phil. 153 (1998) (challenge to legislation requiring broadcast stations to
provide COMELEC Time free of charge).
[86]
G.R. No. L-69500, July 22, 1985, 137 SCRA 717. In this case, the classification of a
movie as “For Adults Only” was challenged, with the issue focused on obscenity as basis for
the alleged invasion of the right to freedom on artistic and literary expression embraced in
the free speech guarantees of the Constitution. The Court held that the test to determine
free expression was the clear and present danger rule. The Court found there was an abuse
of discretion, but did not get enough votes to rule it was grave. The decision specifically
stated that the ruling in the case was limited to concept of obscenity applicable to motion
pictures. Id. at 723-729.
[87]
Id. at 725.
[88]
Id.
[89]
ABS-CBN Broadcasting Corp. v. COMELEC, 380 Phil. 780, 794 (COMELEC Resolution
restraining ABS-CBN, a corporation engaged in broadcast media of television and radio,
from conducting exit surveys after the 1998 elections). Although the decision was
rendered after the 1998 elections, the Court proceeded to rule on the case to rule on the
issue of the constitutionality of holding exit polls and the dissemination of data derived
therefrom. The Court ruled that restriction on exit polls must be tested against the clear
and present danger rule, the rule we “unquestionably” adhere to. The framing of the
guidelines issued by the Court clearly showed that the issue involved not only the conduct
of the exit polls but also its dissemination by broadcast media. And yet, the Court did not
distinguish, and still applied the clear and present danger rule.
[90]
351 Phil. 692 (1998) (challenge to legislation which sought to equalize media access
through regulation).
[91]
Id. at 718.
[92]
Telecommunications and Broadcast Attorneys of the Philippines, Inc. v. COMELEC, 352
Phil. 153 (1998) (challenge to legislation requiring broadcast stations to provide COMELEC
Time free of charge).
[93]
HELEN FENWICK, CIVIL LIBERTIES AND HUMAN RIGHTS 296 (3rd ed. 2002).
[94]
Id.
[95]
Stephen J. Shapiro, How Internet Non-Regulation Undermines The Rationales Used To
Support Broadcast Regulation, 8-FALL MEDIA L. & POL'Y 1, 2 (1999).
[96]
Technological advances, such as software that facilitates the delivery of live, or real-
time, audio and video over the Internet, have enabled Internet content providers to offer
the same services as broadcasters. Indeed, these advancements blur the distinction
between a computer and a television. Id. at 13.
[97]
Id.
[98]
The current rationales used to support regulation of the broadcast media become
unpersuasive in light of the fact that the unregulated Internet and the regulated broadcast
185
media share many of the same features. Id.In other words, as the Internet and broadcast
media become identical, for all intents and purposes, it makes little sense to regulate one
but not the other in an effort to further First Amendment principles. Indeed, as Internet
technologies advance, broadcasters will have little incentive to continue developing
broadcast programming under the threat of regulation when they can disseminate the
same content in the same format through the unregulated Internet. In conclusion, "the
theory of partial regulation, whatever its merits for the circumstances of the last fifty years,
will be unworkable in the media landscape of the future." Id. at 23.

186
EN BANC

THE PROVINCE OF NORTH COTABATO, G.R. No. 183591


duly represented by GOVERNOR JESUS
SACDALAN and/or VICE-GOVERNOR Present:
EMMANUEL PIÑOL, for and in his own
behalf, PUNO, C.J.,
Petitioners, QUISUMBING,
YNARES-SANTIAGO,
CARPIO,
- versus - AUSTRIA-MARTINEZ,
CORONA,
CARPIO MORALES,
THE GOVERNMENT OF THE REPUBLIC OF AZCUNA,
THE PHILIPPINES PEACE PANEL ON TINGA,
ANCESTRAL DOMAIN (GRP), represented CHICO-NAZARIO,
by SEC. RODOLFO GARCIA, ATTY. LEAH VELASCO, JR.,
ARMAMENTO, ATTY. SEDFREY NACHURA,
CANDELARIA, MARK RYAN SULLIVAN REYES,
and/or GEN. HERMOGENES ESPERON, JR., LEONARDO-DE CASTRO, &
the latter in his capacity as the present BRION, JJ.
and duly-appointed Presidential Adviser on
the Peace Process (OPAPP) or the so-called Promulgated:
Office of the Presidential Adviser on the
Peace Process, October 14, 2008
Respondents.
x--------------------------------------------x
CITY GOVERNMENT OF ZAMBOANGA, as
represented by HON. CELSO L. LOBREGAT,
City Mayor of Zamboanga, and in his
personal capacity as resident of the City of
Zamboanga, Rep. MA. ISABELLE G.
CLIMACO, District 1, and Rep. ERICO G.R. No. 183752
BASILIO A. FABIAN, District 2, City
ofZamboanga,
Petitioners,

- versus -

THE GOVERNMENT OF THE REPUBLIC OF


THE PHILIPPINES PEACE NEGOTIATING
PANEL (GRP), as represented by RODOLFO
C. GARCIA, LEAH ARMAMENTO, SEDFREY
CANDELARIA, MARK RYAN SULLIVAN and
HERMOGENES ESPERON, in his capacity as
the Presidential Adviser on Peace Process,
Respondents.

187
x--------------------------------------------x
THE CITY OF ILIGAN, duly represented by
CITY MAYOR LAWRENCE LLUCH CRUZ,
Petitioner,

- versus –

THE GOVERNMENT OF THE REPUBLIC OF


THE PHILIPPINES PEACE PANEL ON
ANCESTRAL DOMAIN (GRP), represented
by SEC. RODOLFO GARCIA, ATTY. LEAH G.R. No. 183893
ARMAMENTO, ATTY. SEDFREY
CANDELARIA, MARK RYAN SULLIVAN;
GEN. HERMOGENES ESPERON, JR., in his
capacity as the present and duly appointed
Presidential Adviser on the Peace Process;
and/or SEC. EDUARDO ERMITA, in his
capacity as Executive Secretary.
Respondents.
x--------------------------------------------x
THE PROVINCIAL GOVERNMENT OF
ZAMBOANGA DEL NORTE, as represented
by HON. ROLANDO E. YEBES, in his
capacity as Provincial Governor, HON.
FRANCIS H. OLVIS, in his capacity as Vice-
Governor and Presiding Officer of the
Sangguniang Panlalawigan,HON. CECILIA
JALOSJOS CARREON, Congresswoman,
1stCongressional District, HON. CESAR G.
JALOSJOS, Congressman,
3rd Congressional District, and Members of
the Sangguniang Panlalawigan of the
Province of Zamboanga del Norte, namely,
HON. SETH FREDERICK P. JALOSJOS,
HON. FERNANDO R. CABIGON, JR., HON.
ULDARICO M. MEJORADA II, HON.
EDIONAR M. ZAMORAS, HON. EDGAR J. G.R. No. 183951
BAGUIO, HON. CEDRIC L. ADRIATICO,
HON.FELIXBERTO C. BOLANDO, HON.
JOSEPH BRENDO C. AJERO, HON.
NORBIDEIRI B. EDDING, HON. ANECITO S.
DARUNDAY, HON. ANGELICA J. CARREON
and HON. LUZVIMINDA E. TORRINO,
Petitioners,

- versus -

188
THE GOVERNMENT OF THE REPUBLIC OF
THE PHILIPPINES PEACE NEGOTIATING
PANEL [GRP], as represented by HON.
RODOLFO C. GARCIA and HON.
HERMOGENES ESPERON, in his capacity as
the Presidential Adviser of Peace Process,
Respondents.
x--------------------------------------------x
ERNESTO M. MACEDA, JEJOMAR C. BINAY,
and AQUILINO L. PIMENTEL III,
Petitioners,

- versus -

THE GOVERNMENT OF THE REPUBLIC OF


THE PHILIPPINES PEACE NEGOTIATING
PANEL, represented by its Chairman
RODOLFO C. GARCIA, and the MORO
ISLAMIC LIBERATION FRONT PEACE
NEGOTIATING PANEL, represented by its
Chairman MOHAGHER IQBAL,
Respondents.
x--------------------------------------------x
FRANKLIN M. DRILON and ADEL ABBAS
TAMANO,
Petitioners-in-Intervention.
x--------------------------------------------x
SEN. MANUEL A. ROXAS,
Petitioners-in-Intervention.
x--------------------------------------------x
MUNICIPALITY OF LINAMON duly
represented by its Municipal Mayor NOEL
N. DEANO,
Petitioners-in-Intervention, G.R. No. 183962
x--------------------------------------------x
THE CITY OF ISABELA,BASILAN PROVINCE,
represented by MAYOR CHERRYLYN P.
SANTOS-AKBAR,
Petitioners-in-Intervention.
x--------------------------------------------x
THE PROVINCE OF SULTAN KUDARAT, rep.
by HON. SUHARTO T. MANGUDADATU, in
his capacity as Provincial Governor and a
resident of the Province of Sultan Kudarat,
Petitioner-in-Intervention.

189
x-------------------------------------------x

RUY ELIAS LOPEZ, for and in his own


behalf and on behalf of Indigenous Peoples
in Mindanao Not Belonging to the MILF,
Petitioner-in-Intervention.
x--------------------------------------------x
CARLO B. GOMEZ, GERARDO S. DILIG,
NESARIO G. AWAT, JOSELITO C. ALISUAG
and RICHALEX G. JAGMIS, as citizens and
residents of Palawan,
Petitioners-in-Intervention.
x--------------------------------------------x
MARINO RIDAO and KISIN BUXANI,
Petitioners-in-Intervention.
x--------------------------------------------x
MUSLIM LEGAL ASSISTANCE
FOUNDATION, INC (MUSLAF),
Respondent-in-Intervention.
x--------------------------------------------x
MUSLIM MULTI-SECTORAL MOVEMENT
FOR PEACE & DEVELOPMENT (MMMPD),
Respondent-in-Intervention.
x--------------------------------------------x

x - - - - - - - - - - - - - - - - - - - - - - - - - - - - - - - - - - - - - - - - - - - - - - - - - - -x

DECISION

CARPIO MORALES, J.:

Subject of these consolidated cases is the extent of the powers of the President in
pursuing the peace process. While the facts surrounding this controversy center on the armed
conflict in Mindanao between the government and the Moro Islamic Liberation Front (MILF), the
legal issue involved has a bearing on all areas in the country where there has been a long-
standing armed conflict. Yet again, the Court is tasked to perform a delicate balancing act. It
must uncompromisingly delineate the bounds within which the President may lawfully exercise
her discretion, but it must do so in strict adherence to the Constitution, lest its ruling unduly
restricts the freedom of action vested by that same Constitution in the Chief Executive precisely
to enable her to pursue the peace process effectively.
I. FACTUAL ANTECEDENTS OF THE PETITIONS

190
On August 5, 2008, the Government of the Republic of the Philippines (GRP) and the
MILF, through the Chairpersons of their respective peace negotiating panels, were scheduled to
sign a Memorandum of Agreement on the Ancestral Domain (MOA-AD) Aspect of the GRP-MILF
Tripoli Agreement on Peace of 2001 in Kuala Lumpur,Malaysia.

The MILF is a rebel group which was established in March 1984 when, under the
leadership of the late Salamat Hashim, it splintered from the Moro National Liberation Front
(MNLF) then headed by Nur Misuari, on the ground, among others, of what Salamat perceived to
be the manipulation of the MNLF away from an Islamic basis towards Marxist-Maoist
orientations.[1]

The signing of the MOA-AD between the GRP and the MILF was not to materialize,
however, for upon motion of petitioners, specifically those who filed their cases before the
scheduled signing of the MOA-AD, this Court issued a Temporary Restraining Order enjoining the
GRP from signing the same.

The MOA-AD was preceded by a long process of negotiation and the concluding of several
prior agreements between the two parties beginning in 1996, when the GRP-MILF peace
negotiations began. On July 18, 1997, the GRP and MILF Peace Panels signed the Agreement on
General Cessation of Hostilities. The following year, they signed the General Framework of
Agreement of Intent on August 27, 1998.

The Solicitor General, who represents respondents, summarizes the MOA-AD by stating
that the same contained, among others, the commitment of the parties to pursue peace
negotiations, protect and respect human rights, negotiate with sincerity in the resolution and
pacific settlement of the conflict, and refrain from the use of threat or force to attain undue
advantage while the peace negotiations on the substantive agenda are on-going.[2]

Early on, however, it was evident that there was not going to be any smooth sailing in
the GRP-MILF peace process. Towards the end of 1999 up to early 2000, the MILF attacked a
number of municipalities in Central Mindanao and, in March 2000, it took control of the town
hall of Kauswagan, Lanao del Norte.[3] In response, then President Joseph Estrada declared and
carried out an “all-out-war” against the MILF.

When President Gloria Macapagal-Arroyo assumed office, the military offensive against
the MILF was suspended and the government sought a resumption of the peace talks. The MILF,

191
according to a leading MILF member, initially responded with deep reservation, but when
President Arroyo asked the Government of Malaysia through Prime Minister Mahathir Mohammad
to help convince the MILF to return to the negotiating table, the MILF convened its Central
Committee to seriously discuss the matter and, eventually, decided to meet with the GRP.[4]

The parties met in Kuala Lumpur on March 24, 2001, with the talks being facilitated by
the Malaysian government, the parties signing on the same date the Agreement on the General
Framework for the Resumption of Peace Talks Between the GRP and the MILF. The MILF
[5]
thereafter suspended all its military actions.

Formal peace talks between the parties were held in Tripoli, Libya from June 20-22, 2001,
the outcome of which was the GRP-MILF Tripoli Agreement on Peace (Tripoli Agreement 2001)
containing the basic principles and agenda on the following aspects of the
negotiation: Security Aspect, Rehabilitation Aspect, and Ancestral DomainAspect. With regard to
the Ancestral Domain Aspect, the parties in Tripoli Agreement 2001 simply agreed “that the
same be discussed further by the Parties in their next meeting.”

A second round of peace talks was held in Cyberjaya, Malaysia on August 5-7,
2001 which ended with the signing of the Implementing Guidelines on the Security Aspectof the
Tripoli Agreement 2001 leading to a ceasefire status between the parties. This was followed by
the Implementing Guidelines on the Humanitarian Rehabilitation and Development Aspects of
the Tripoli Agreement 2001, which was signed on May 7,
2002 at Putrajaya, Malaysia. Nonetheless, there were many incidence of violence between
government forces and the MILF from 2002 to 2003.

Meanwhile, then MILF Chairman Salamat Hashim passed away on July 13, 2003 and he
was replaced by Al Haj Murad, who was then the chief peace negotiator of the MILF. Murad’s
position as chief peace negotiator was taken over by Mohagher Iqbal.[6]

In 2005, several exploratory talks were held between the parties in Kuala Lumpur,
eventually leading to the crafting of the draft MOA-AD in its final form, which, as mentioned, was
set to be signed last August 5, 2008.

II. STATEMENT OF THE PROCEEDINGS

192
Before the Court is what is perhaps the most contentious “consensus” ever embodied in
an instrument – the MOA-AD which is assailed principally by the present petitions bearing docket
numbers 183591, 183752, 183893, 183951 and 183962.

Commonly impleaded as respondents are the GRP Peace Panel on Ancestral


[7]
Domain and the Presidential Adviser on the Peace Process (PAPP) Hermogenes Esperon, Jr.

On July 23, 2008, the Province of North Cotabato[8] and Vice-Governor Emmanuel Piñol
filed a petition, docketed as G.R. No. 183591, for Mandamus and Prohibition with Prayer for the
Issuance of Writ of Preliminary Injunction and Temporary Restraining Order.[9] Invoking the
right to information on matters of public concern, petitioners seek to compel respondents to
disclose and furnish them the complete and official copies of the MOA-AD including its
attachments, and to prohibit the slated signing of the MOA-AD, pending the disclosure of the
contents of the MOA-AD and the holding of a public consultation thereon. Supplementarily,
petitioners pray that the MOA-AD be declared unconstitutional.[10]

This initial petition was followed by another one, docketed as G.R. No. 183752, also for
Mandamus and Prohibition[11] filed by the City of Zamboanga,[12] Mayor Celso Lobregat, Rep. Ma.
Isabelle Climaco and Rep. Erico Basilio Fabian who likewise pray for similar injunctive
reliefs. Petitioners herein moreover pray that the City ofZamboanga be excluded from the
Bangsamoro Homeland and/or Bangsamoro Juridical Entity and, in the alternative, that the MOA-
AD be declared null and void.

By Resolution of August 4, 2008, the Court issued a Temporary Restraining


Order commanding and directing public respondents and their agents to cease and desist from
formally signing the MOA-AD.[13] The Court also required the Solicitor General to submit to the
Court and petitioners the official copy of the final draft of the MOA-AD,[14] to which she
complied.[15]

Meanwhile, the City of Iligan[16] filed a petition for Injunction and/or Declaratory Relief,
docketed as G.R. No. 183893, praying that respondents be enjoined from signing the MOA-AD
or, if the same had already been signed, from implementing the same, and that the MOA-AD be
declared unconstitutional. Petitioners herein additionally implead Executive Secretary Eduardo
Ermita as respondent.

193
The Province of Zamboanga del Norte,[17] Governor Rolando Yebes, Vice-Governor Francis
Olvis, Rep. Cecilia Jalosjos-Carreon, Rep. Cesar Jalosjos, and the members[18] of
the Sangguniang Panlalawigan of Zamboanga del Norte filed on August 15, 2008 a petition for
Certiorari, Mandamus and Prohibition,[19] docketed as G.R. No. 183951. They pray, inter alia,
that the MOA-AD be declared null and void and without operative effect, and that respondents
be enjoined from executing the MOA-AD.

On August 19, 2008, Ernesto Maceda, Jejomar Binay, and Aquilino Pimentel III filed a
petition for Prohibition,[20] docketed as G.R. No. 183962, praying for a judgment prohibiting and
permanently enjoining respondents from formally signing and executing the MOA-AD and or any
other agreement derived therefrom or similar thereto, and nullifying the MOA-AD for being
unconstitutional and illegal. Petitioners herein additionally implead as respondent the MILF
Peace Negotiating Panel represented by its Chairman Mohagher Iqbal.
Various parties moved to intervene and were granted leave of court to file their petitions-
/comments-in-intervention. Petitioners-in-Intervention include Senator Manuel A. Roxas, former
Senate President Franklin Drilon and Atty. Adel Tamano, the City of Isabela[21] and Mayor
Cherrylyn Santos-Akbar, the Province of Sultan Kudarat[22] and Gov. Suharto Mangudadatu, the
Municipality of Linamon in Lanao del Norte,[23] Ruy Elias Lopez of Davao City and of the Bagobo
tribe, Sangguniang Panlungsod member Marino Ridao and businessman Kisin Buxani, both of
Cotabato City; and lawyers Carlo Gomez, Gerardo Dilig, Nesario Awat, Joselito Alisuag, Richalex
Jagmis, all of Palawan City. The Muslim Legal Assistance Foundation, Inc. (Muslaf) and the
Muslim Multi-Sectoral Movement for Peace and Development (MMMPD) filed their respective
Comments-in-Intervention.

By subsequent Resolutions, the Court ordered the consolidation of the


petitions. Respondents filed Comments on the petitions, while some of petitioners submitted
their respective Replies.

Respondents, by Manifestation and Motion of August 19, 2008, stated that the Executive
Department shall thoroughly review the MOA-AD and pursue further negotiations to address the
issues hurled against it, and thus moved to dismiss the cases. In the succeeding exchange of
pleadings, respondents’ motion was met with vigorous opposition from petitioners.

The cases were heard on oral argument on August 15, 22 and 29, 2008 that tackled the
following principal issues:

1. Whether the petitions have become moot and academic


194
(i) insofar as the mandamus aspect is concerned, in view of the disclosure of
official copies of the final draft of the Memorandum of Agreement (MOA);
and

(ii) insofar as the prohibition aspect involving the Local Government Units is
concerned, if it is considered that consultation has become fait accompli with
the finalization of the draft;

2. Whether the constitutionality and the legality of the MOA is ripe for
adjudication;

3. Whether respondent Government of the Republic of the Philippines Peace Panel


committed grave abuse of discretion amounting to lack or excess of jurisdiction
when it negotiated and initiated the MOA vis-à-vis ISSUES Nos. 4 and 5;

4. Whether there is a violation of the people’s right to information on matters of


public concern (1987 Constitution, Article III, Sec. 7) under a state policy of full
disclosure of all its transactions involving public interest (1987 Constitution,
Article II, Sec. 28) including public consultation under Republic Act No. 7160
(LOCAL GOVERNMENT CODE OF 1991)[;]

If it is in the affirmative, whether prohibition under Rule 65 of the 1997 Rules of


Civil Procedure is an appropriate remedy;

5. Whether by signing the MOA, the Government of the Republic of


the Philippines would be BINDING itself

a) to create and recognize the Bangsamoro Juridical Entity (BJE) as a separate


state, or a juridical, territorial or political subdivision not recognized by law;

b) to revise or amend the Constitution and existing laws to conform to the


MOA;

c) to concede to or recognize the claim of the Moro Islamic Liberation Front for
ancestral domain in violation of Republic Act No. 8371 (THE
INDIGENOUS PEOPLES RIGHTS ACT OF1997), particularly Section 3(g) &
Chapter VII (DELINEATION, RECOGNITION OF ANCESTRAL DOMAINS)[;]

If in the affirmative, whether the Executive Branch has the authority to so bind
the Government of the Republic of the Philippines;

6. Whether the inclusion/exclusion of the Province of North Cotabato, Cities of


Zamboanga, Iligan and Isabela, and the Municipality of Linamon, Lanao del
Norte in/from the areas covered by the projected Bangsamoro Homeland is a
justiciable question; and

7. Whether desistance from signing the MOA derogates any prior valid
commitments of the Government of the Republic of the Philippines.[24]

195
The Court, thereafter, ordered the parties to submit their respective Memoranda. Most of
the parties submitted their memoranda on time.

III. OVERVIEW OF THE MOA-AD

As a necessary backdrop to the consideration of the objections raised in the subject five
petitions and six petitions-in-intervention against the MOA-AD, as well as the two comments-in-
intervention in favor of the MOA-AD, the Court takes an overview of the MOA.

The MOA-AD identifies the Parties to it as the GRP and the MILF.

Under the heading “Terms of Reference” (TOR), the MOA-AD includes not only four earlier
agreements between the GRP and MILF, but also two agreements between the GRP and the
MNLF: the 1976 Tripoli Agreement, and the Final Peace Agreement on the Implementation of the
1976 Tripoli Agreement, signed on September 2, 1996 during the administration of President
Fidel Ramos.

The MOA-AD also identifies as TOR two local statutes – the organic act for the
Autonomous Region in Muslim Mindanao (ARMM)[25] and the Indigenous Peoples Rights Act
(IPRA),[26] and several international law instruments – the ILO Convention No. 169 Concerning
Indigenous and Tribal Peoples in Independent Countries in relation to the UN Declaration on the
Rights of the Indigenous Peoples, and the UN Charter, among others.

The MOA-AD includes as a final TOR the generic category of “compact rights
entrenchment emanating from the regime of dar-ul-mua’hada (or territory undercompact)
and dar-ul-sulh (or territory under peace agreement) that partakes the nature of a treaty
device.”

During the height of the Muslim Empire, early Muslim jurists tended to see the world
through a simple dichotomy: there was the dar-ul-Islam (the Abode of Islam) anddar-ul-
harb (the Abode of War). The first referred to those lands where Islamic laws held sway, while
the second denoted those lands where Muslims were persecuted or where Muslim laws were
outlawed or ineffective.[27] This way of viewing the world, however, became more complex
through the centuries as the Islamic world became part of the international community of
nations.

196
As Muslim States entered into treaties with their neighbors, even with distant States and
inter-governmental organizations, the classical division of the world into dar-ul-Islam and dar-ul-
harb eventually lost its meaning. New terms were drawn up to describe novel ways of
perceiving non-Muslim territories. For instance, areas like dar-ul-mua’hada (land of compact)
and dar-ul-sulh (land of treaty) referred to countries which, though under a secular regime,
maintained peaceful and cooperative relations with Muslim States, having been bound to each
other by treaty or agreement. Dar-ul-aman (land of order), on the other hand, referred to
countries which, though not bound by treaty with Muslim States, maintained freedom of religion
for Muslims.[28]

It thus appears that the “compact rights entrenchment” emanating from the regime
of dar-ul-mua’hada and dar-ul-sulh simply refers to all other agreements between the MILF and
the Philippine government – the Philippines being the land of compact and peace agreement –
that partake of the nature of a treaty device, “treaty” being broadly defined as “any solemn
agreement in writing that sets out understandings, obligations, and benefits for both parties
which provides for a framework that elaborates the principles declared in the [MOA-AD].”[29]

The MOA-AD states that the Parties “HAVE AGREED AND ACKNOWLEDGED AS FOLLOWS,”
and starts with its main body.

The main body of the MOA-AD is divided into four


strands, namely, Concepts and Principles, Territory,
Resources, and Governance.

A. CONCEPTS AND PRINCIPLES

This strand begins with the statement that it is “the birthright of all Moros and all
Indigenous peoples of Mindanao to identify themselves and be accepted as ‘Bangsamoros.’” It
defines “Bangsamoro people” as the natives or original inhabitants of Mindanao and its adjacent
islands including Palawan and the Sulu archipelago at the time of conquest or colonization, and
their descendants whether mixed or of full blood, including their spouses.[30]

Thus, the concept of “Bangsamoro,” as defined in this strand of the MOA-AD, includes not
only “Moros” as traditionally understood even by Muslims,[31] but
allindigenous peoples of Mindanao and its adjacent islands. The MOA-AD adds that the freedom
of choice of indigenous peoples shall be respected. What this freedom of choice consists in has
not been specifically defined.

197
The MOA-AD proceeds to refer to the “Bangsamoro homeland,” the ownership of which is
vested exclusively in the Bangsamoro people by virtue of their prior rights of
[32]
occupation. Both parties to the MOA-AD acknowledge that ancestral domain does not form
part of the public domain.[33]

The Bangsamoro people are acknowledged as having the right to self-governance, which
right is said to be rooted on ancestral territoriality exercised originally under the suzerain
authority of their sultanates and the Pat a Pangampong ku Ranaw. The sultanates were
described as states or “karajaan/kadatuan” resembling a body politic endowed with all the
elements of a nation-state in the modern sense.[34]

The MOA-AD thus grounds the right to self-governance of the Bangsamoro people on the
past suzerain authority of the sultanates. As gathered, the territory defined as the Bangsamoro
homeland was ruled by several sultanates and, specifically in the case of the Maranao, by
the Pat a Pangampong ku Ranaw, a confederation of independent principalities (pangampong)
each ruled by datus and sultans, none of whom was supreme over the others.[35]

The MOA-AD goes on to describe the Bangsamoro people as “the ‘First Nation’ with
defined territory and with a system of government having entered into treaties of amity and
commerce with foreign nations.”
The term “First Nation” is of Canadian origin referring to the indigenous peoples of that
territory, particularly those known as Indians. In Canada, each of these indigenous peoples is
equally entitled to be called “First Nation,” hence, all of them are usually described collectively
by the plural “First Nations.”[36] To that extent, the MOA-AD, by identifying the Bangsamoro
people as “the First Nation” – suggesting its exclusive entitlement to that designation – departs
from the Canadian usage of the term.

The MOA-AD then mentions for the first time the “Bangsamoro Juridical Entity” (BJE) to
which it grants the authority and jurisdiction over the Ancestral Domain andAncestral Lands of
the Bangsamoro.[37]

B. TERRITORY

198
The territory of the Bangsamoro homeland is described as the land mass as well as the
maritime, terrestrial, fluvial and alluvial domains, including the aerial domain and the
atmospheric space above it, embracing the Mindanao-Sulu-Palawan geographic region.[38]

More specifically, the core of the BJE is defined as the present geographic area of the
ARMM – thus constituting the following areas: Lanao del Sur, Maguindanao, Sulu, Tawi-Tawi,
Basilan, and Marawi City. Significantly, this core also includes certain municipalities of Lanao del
Norte that voted for inclusion in the ARMM in the 2001 plebiscite.[39]

Outside of this core, the BJE is to cover other provinces, cities, municipalities and
barangays, which are grouped into two categories, Category A and Category B. Each of these
areas is to be subjected to a plebiscite to be held on different dates, years apart from each
other. Thus, Category A areas are to be subjected to a plebiscite not later than twelve (12)
months following the signing of the MOA-AD.[40] Category B areas, also called “Special
Intervention Areas,” on the other hand, are to be subjected to a plebiscite twenty-five (25)
years from the signing of a separate agreement – the Comprehensive Compact.[41]

The Parties to the MOA-AD stipulate that the BJE shall have jurisdiction over all natural
resources within its “internal waters,” defined as extending fifteen (15) kilometers from the
coastline of the BJE area;[42] that the BJE shall also have “territorial waters,” which shall stretch
beyond the BJE internal waters up to the baselines of the Republic of the Philippines (RP) south
east and south west of mainland Mindanao; and that within these territorial waters, the BJE and
the “Central Government” (used interchangeably with RP) shall exercise joint jurisdiction,
authority and management over all natural resources.[43] Notably, the jurisdiction over
the internal waters is not similarly described as “joint.”

The MOA-AD further provides for the sharing of minerals on the territorial waters between
the Central Government and the BJE, in favor of the latter, through production sharing and
economic cooperation agreement.[44] The activities which the Parties are allowed to conduct on
the territorial waters are enumerated, among which are the exploration and utilization of natural
resources, regulation of shipping and fishing activities, and the enforcement of police and safety
measures.[45] There is no similar provision on the sharing of minerals and allowed activities with
respect to the internal waters of the BJE.

C. RESOURCES

199
The MOA-AD states that the BJE is free to enter into any economic cooperation and trade
relations with foreign countries and shall have the option to establish trade missions in those
countries. Such relationships and understandings, however, are not to include aggression
against the GRP. The BJE may also enter into environmental cooperation agreements.[46]

The external defense of the BJE is to remain the duty and obligation of the Central
Government. The Central Government is also bound to “take necessary steps to ensure the
BJE’s participation in international meetings and events” like those of the ASEAN and the
specialized agencies of the UN. The BJE is to be entitled to participate in Philippine official
missions and delegations for the negotiation of border agreements or protocols for
environmental protection and equitable sharing of incomes and revenues involving the bodies of
water adjacent to or between the islands forming part of the ancestral domain.[47]

With regard to the right of exploring for, producing, and obtaining all potential sources of
energy, petroleum, fossil fuel, mineral oil and natural gas, the jurisdiction and control thereon is
to be vested in the BJE “as the party having control within its territorial jurisdiction.” This right
carries the proviso that, “in times of national emergency, when public interest so requires,” the
Central Government may, for a fixed period and under reasonable terms as may be agreed upon
by both Parties, assume or direct the operation of such resources.[48]

The sharing between the Central Government and the BJE of total production pertaining
to natural resources is to be 75:25 in favor of the BJE.[49]
The MOA-AD provides that legitimate grievances of the Bangsamoro people arising from
any unjust dispossession of their territorial and proprietary rights, customary land tenures, or
their marginalization shall be acknowledged. Whenever restoration is no longer possible,
reparation is to be in such form as mutually determined by the Parties.[50]

The BJE may modify or cancel the forest concessions, timber licenses, contracts or
agreements, mining concessions, Mineral Production and Sharing Agreements (MPSA), Industrial
Forest Management Agreements (IFMA), and other land tenure instruments granted by the
Philippine Government, including those issued by the present ARMM.[51]

D. GOVERNANCE

The MOA-AD binds the Parties to invite a multinational third-party to observe and monitor
the implementation of the Comprehensive Compact. This compact is to embody the “details for
the effective enforcement” and “the mechanisms and modalities for the actual implementation”
200
of the MOA-AD. The MOA-AD explicitly provides that the participation of the third party shall not
in any way affect the status of the relationship between the Central Government and the
BJE.[52]

The “associative” relationship


between the Central Government
and the BJE

The MOA-AD describes the relationship of the Central Government and the BJE as
“associative,” characterized by shared authority and responsibility. And it states that the
structure of governance is to be based on executive, legislative, judicial, and administrative
institutions with defined powers and functions in the Comprehensive Compact.

The MOA-AD provides that its provisions requiring “amendments to the existing legal
framework” shall take effect upon signing of the Comprehensive Compact and upon effecting the
aforesaid amendments, with due regard to the non-derogation of prior agreements and within
the stipulated timeframe to be contained in the Comprehensive Compact. As will be discussed
later, much of the present controversy hangs on the legality of this provision.

The BJE is granted the power to build, develop and maintain its own institutions inclusive
of civil service, electoral, financial and banking, education, legislation, legal, economic, police
and internal security force, judicial system and correctional institutions, the details of which shall
be discussed in the negotiation of the comprehensive compact.

As stated early on, the MOA-AD was set to be signed on August 5, 2008 by Rodolfo
Garcia and Mohagher Iqbal, Chairpersons of the Peace Negotiating Panels of theGRP and the
MILF, respectively. Notably, the penultimate paragraph of the MOA-AD identifies the signatories
as “the representatives of the Parties,” meaning the GRP and MILF themselves, and not merely
of the negotiating panels.[53] In addition, the signature page of the MOA-AD states that it is
“WITNESSED BY” Datuk Othman Bin Abd Razak, Special Adviser to the Prime Minister of
Malaysia, “ENDORSED BY” Ambassador Sayed Elmasry, Adviser to Organization of the Islamic
Conference (OIC) Secretary General and Special Envoy for Peace Process in Southern
Philippines, and SIGNED “IN THE PRESENCE OF” Dr. Albert G. Romulo, Secretary of Foreign
Affairs of RP and Dato’ Seri Utama Dr. Rais Bin Yatim, Minister of Foreign Affairs, Malaysia, all of
whom were scheduled to sign the Agreement last August 5, 2008.

201
Annexed to the MOA-AD are two documents containing the respective lists cum maps of
the provinces, municipalities, and barangays under Categories A and B earlier mentioned in the
discussion on the strand on TERRITORY.

IV. PROCEDURAL ISSUES

A. RIPENESS

The power of judicial review is limited to actual cases or controversies.[54] Courts decline
to issue advisory opinions or to resolve hypothetical or feigned problems, or mere academic
questions.[55] The limitation of the power of judicial review to actual cases and
controversies defines the role assigned to the judiciary in a tripartite allocation of power, to
assure that the courts will not intrude into areas committed to the other branches of
government.[56]

An actual case or controversy involves a conflict of legal rights, an assertion of opposite


legal claims, susceptible of judicial resolution as distinguished from a hypothetical or abstract
difference or dispute. There must be a contrariety of legal rights that can be interpreted and
enforced on the basis of existing law and jurisprudence.[57] The Court can decide the
constitutionality of an act or treaty only when a proper case between opposing parties is
submitted for judicial determination.[58]

Related to the requirement of an actual case or controversy is the requirement of


ripeness. A question is ripe for adjudication when the act being challenged has had a direct
adverse effect on the individual challenging it.[59] For a case to be considered ripe for
adjudication, it is a prerequisite that something had then been accomplished or performed by
either branch before a court may come into the picture,[60] and the petitioner must allege the
existence of an immediate or threatened injury to itself as a result of the challenged
action.[61] He must show that he has sustained or is immediately in danger of sustaining some
direct injury as a result of the act complained of.[62]

The Solicitor General argues that there is no justiciable controversy that is ripe for judicial
review in the present petitions, reasoning that

The unsigned MOA-AD is simply a list of consensus points subject to further


negotiations and legislative enactments as well as constitutional processes aimed at
attaining a final peaceful agreement. Simply put, the MOA-AD remains to be
202
a proposal that does not automatically create legally demandable rights and
obligations until the list of operative acts required have been duly complied with. x
xx

xxxx

In the cases at bar, it is respectfully submitted that this Honorable Court has
no authority to pass upon issues based on hypothetical or feigned constitutional
problems or interests with no concrete bases. Considering
the preliminary character of the MOA-AD, there are no concrete acts that could
possibly violate petitioners’ and intervenors’ rights since the acts complained of
are mere contemplated steps toward the formulation of a final peace
agreement. Plainly, petitioners and intervenors’ perceived injury, if at all, is merely
imaginary and illusory apart from being unfounded and based on mere
conjectures. (Underscoring supplied)

The Solicitor General cites[63] the following provisions of the MOA-AD:

TERRITORY

xxxx

2. Toward this end, the Parties enter into the following stipulations:
xxxx

d. Without derogating from the requirements of prior agreements, the


Government stipulates to conduct and deliver, using all possible
legal measures, within twelve (12) months following the signing of
the MOA-AD, a plebiscite covering the areas as enumerated in the
list and depicted in the map as Category A attached herein (the
“Annex”). The Annex constitutes an integral part of this framework
agreement. Toward this end, the Parties shall endeavor to
complete the negotiations and resolve all outstanding issues on the
Comprehensive Compact within fifteen (15) months from the
signing of the MOA-AD.

xxxx

GOVERNANCE

xxxx

7. The Parties agree that mechanisms and modalities for the actual
implementation of this MOA-AD shall be spelt out in the Comprehensive
Compact to mutually take such steps to enable it to occur effectively.

Any provisions of the MOA-AD requiring amendments to the existing


legal framework shall come into force upon the signing of a
203
Comprehensive Compact and upon effecting the necessary changes to the
legal framework with due regard to non-derogation of prior
agreements and within the stipulated timeframe to be contained in the
Comprehensive Compact.[64] (Underscoring supplied)

The Solicitor General’s arguments fail to persuade.

Concrete acts under the MOA-AD are not necessary to render the present controversy
ripe. In Pimentel, Jr. v. Aguirre,[65] this Court held:

x x x [B]y the mere enactment of the questioned law or the approval of the
challenged action, the dispute is said to have ripened into a judicial controversy
even without any other overt act. Indeed, even a singular violation of the
Constitution and/or the law is enough to awaken judicial duty.

xxxx

By the same token, when an act of the President, who in our constitutional
scheme is a coequal of Congress, is seriously alleged to have infringed the
Constitution and the laws x x x settling the dispute becomes the duty and the
responsibility of the courts.[66]

In Santa Fe Independent School District v. Doe,[67] the United States Supreme Court held
that the challenge to the constitutionality of the school’s policy allowing student-led prayers and
speeches before games was ripe for adjudication, even if no public prayer had yet been led
under the policy, because the policy was being challenged as unconstitutional on its face.[68]

That the law or act in question is not yet effective does not negate ripeness. For
example, in New York v. United States,[69] decided in 1992, the United States Supreme Court
held that the action by the State of New York challenging the provisions of the Low-Level
Radioactive Waste Policy Act was ripe for adjudication even if the questioned provision was not
to take effect until January 1, 1996, because the parties agreed that New York had to take
immediate action to avoid the provision's consequences.[70]

The present petitions pray for Certiorari,[71] Prohibition, and Mandamus. Certiorari and
Prohibition are remedies granted by law when any tribunal, board or officer has acted, in the
case of certiorari, or is proceeding, in the case of prohibition, without or in excess of its
jurisdiction or with grave abuse of discretion amounting to lack or excess of
[72]
jurisdiction. Mandamus is a remedy granted by law when any tribunal, corporation, board,

204
officer or person unlawfully neglects the performance of an act which the law specifically enjoins
as a duty resulting from an office, trust, or station, or unlawfully excludes another from the use
or enjoyment of a right or office to which such other is entitled.[73] Certiorari, Mandamus and
Prohibition are appropriate remedies to raise constitutional issues and to review and/or
prohibit/nullify, when proper, acts of legislative and executive officials.[74]

The authority of the GRP Negotiating Panel is defined by Executive Order No. 3 (E.O. No.
3), issued on February 28, 2001.[75] The said executive order requires that “[t]he government's
policy framework for peace, including the systematic approach and the administrative structure
for carrying out the comprehensive peace process x x x be governed by this Executive Order.”[76]

The present petitions allege that respondents GRP Panel and PAPP Esperon drafted the
terms of the MOA-AD without consulting the local government units or communities affected, nor
informing them of the proceedings. As will be discussed in greater detail later, such omission,
by itself, constitutes a departure by respondents from their mandate under E.O. No. 3.

Furthermore, the petitions allege that the provisions of the MOA-AD violate the
Constitution. The MOA-AD provides that “any provisions of the MOA-AD requiring amendments
to the existing legal framework shall come into force upon the signing of a Comprehensive
Compact and upon effecting the necessary changes to the legal framework,” implying an
amendment of the Constitution to accommodate the MOA-AD. This stipulation, in
effect, guaranteed to the MILF the amendment of the Constitution. Such act constitutes another
violation of its authority. Again, these points will be discussed in more detail later.
As the petitions allege acts or omissions on the part of respondent that exceed their
authority, by violating their duties under E.O. No. 3 and the provisions of the Constitution and
statutes, the petitions make a prima facie case for Certiorari, Prohibition, and Mandamus, and an
actual case or controversy ripe for adjudication exists. When an act of a branch of government is
seriously alleged to have infringed the Constitution, it becomes not only the right but in fact the
duty of the judiciary to settle the dispute.[77]

B. LOCUS STANDI

For a party to have locus standi, one must allege “such a personal stake in the outcome
of the controversy as to assure that concrete adverseness which sharpens the presentation of
issues upon which the court so largely depends for illumination of difficult constitutional
questions.”[78]

205
Because constitutional cases are often public actions in which the relief sought is likely to
affect other persons, a preliminary question frequently arises as to this interest in the
constitutional question raised.[79]

When suing as a citizen, the person complaining must allege that he has been or is about
to be denied some right or privilege to which he is lawfully entitled or that he is about to be
subjected to some burdens or penalties by reason of the statute or act complained of.[80] When
the issue concerns a public right, it is sufficient that the petitioner is a citizen and has an interest
in the execution of the laws.[81]

For a taxpayer, one is allowed to sue where there is an assertion that public funds are
illegally disbursed or deflected to an illegal purpose, or that there is a wastage of public funds
through the enforcement of an invalid or unconstitutional law.[82] The Court retains discretion
whether or not to allow a taxpayer’s suit.[83]

In the case of a legislator or member of Congress, an act of the Executive that injures the
institution of Congress causes a derivative but nonetheless substantial injury that can be
questioned by legislators. A member of the House of Representatives has standing to maintain
inviolate the prerogatives, powers and privileges vested by the Constitution in his office.[84]

An organization may be granted standing to assert the rights of its members,[85] but the
mere invocation by the Integrated Bar of the Philippines or any member of the legal
profession of the duty to preserve the rule of law does not suffice to clothe it with standing.[86]

As regards a local government unit (LGU), it can seek relief in order to protect or
vindicate an interest of its own, and of the other LGUs.[87]

Intervenors, meanwhile, may be given legal standing upon showing of facts that satisfy
the requirements of the law authorizing intervention,[88] such as a legal interest in the matter in
litigation, or in the success of either of the parties.

In any case, the Court has discretion to relax the procedural technicality on locus
standi, given the liberal attitude it has exercised, highlighted in the case of David v. Macapagal-
Arroyo,[89] where technicalities of procedure were brushed aside, the constitutional issues raised
being of paramount public interest or of transcendental importance deserving the attention of

206
the Court in view of their seriousness, novelty and weight as precedents.[90] The Court’s
forbearing stance on locus standi on issues involving constitutional issues has for its purpose the
protection of fundamental rights.

In not a few cases, the Court, in keeping with its duty under the Constitution to
determine whether the other branches of government have kept themselves within the limits of
the Constitution and the laws and have not abused the discretion given them, has brushed aside
technical rules of procedure.[91]

In the petitions at bar, petitioners Province of North Cotabato (G.R. No. 183591) Province
of Zamboanga del Norte (G.R. No. 183951), City of Iligan (G.R. No. 183893) and City of
Zamboanga (G.R. No. 183752) and petitioners-in-intervention Province of Sultan Kudarat, City
of Isabela and Municipality of Linamon have locus standi in view of the direct and substantial
injury that they, as LGUs, would suffer as their territories, whether in whole or in part, are to be
included in the intended domain of the BJE. These petitioners allege that they did not vote for
their inclusion in the ARMM which would be expanded to form the BJE territory. Petitioners’ legal
standing is thus beyond doubt.

In G.R. No. 183962, petitioners Ernesto Maceda, Jejomar Binay and Aquilino
Pimentel III would have no standing as citizens and taxpayers for their failure to specify that
they would be denied some right or privilege or there would be wastage of public funds. The
fact that they are a former Senator, an incumbent mayor of MakatiCity, and a resident of
Cagayan de Oro, respectively, is of no consequence. Considering their invocation of the
transcendental importance of the issues at hand, however, the Court grants them standing.

Intervenors Franklin Drilon and Adel Tamano, in alleging their standing as taxpayers,
assert that government funds would be expended for the conduct of an illegal and
unconstitutional plebiscite to delineate the BJE territory. On that score alone, they can be given
legal standing. Their allegation that the issues involved in these petitions are of “undeniable
transcendental importance” clothes them with added basis for their personality to intervene in
these petitions.

With regard to Senator Manuel Roxas, his standing is premised on his being a member of
the Senate and a citizen to enforce compliance by respondents of the public’s constitutional right
to be informed of the MOA-AD, as well as on a genuine legal interest in the matter in litigation,

207
or in the success or failure of either of the parties. He thus possesses the requisite standing as
an intervenor.

With respect to Intervenors Ruy Elias Lopez, as a former congressman of the 3rd district
of Davao City, a taxpayer and a member of the Bagobo tribe; Carlo B. Gomez, et al., as
members of the IBP Palawan chapter, citizens and taxpayers; Marino Ridao, as taxpayer,
resident and member of the Sangguniang Panlungsod of Cotabato City; and Kisin Buxani, as
taxpayer, they failed to allege any proper legal interest in the present petitions. Just the same,
the Court exercises its discretion to relax the procedural technicality on locus standi given the
paramount public interest in the issues at hand.

Intervening respondents Muslim Multi-Sectoral Movement for Peace and Development, an


advocacy group for justice and the attainment of peace and prosperity in Muslim Mindanao;
and Muslim Legal Assistance Foundation Inc., a non-government organization of Muslim lawyers,
allege that they stand to be benefited or prejudiced, as the case may be, in the resolution of the
petitions concerning the MOA-AD, and prays for the denial of the petitions on the grounds
therein stated. Such legal interest suffices to clothe them with standing.

B. MOOTNESS

Respondents insist that the present petitions have been rendered moot with the
satisfaction of all the reliefs prayed for by petitioners and the subsequent pronouncement of the
Executive Secretary that “[n]o matter what the Supreme Court ultimately decides[,] the
government will not sign the MOA.”[92]

In lending credence to this policy decision, the Solicitor General points out that the
President had already disbanded the GRP Peace Panel.[93]

In David v. Macapagal-Arroyo,[94] this Court held that the “moot and academic” principle
not being a magical formula that automatically dissuades courts in resolving a case, it will decide
cases, otherwise moot and academic, if it finds that (a) there is a grave violation of the
Constitution;[95] (b) the situation is of exceptional character and paramount public interest is
involved;[96] (c) the constitutional issue raised requires formulation of controlling principles to
guide the bench, the bar, and the public;[97] and (d) the case is capable of repetition yet evading
review.[98]

208
Another exclusionary circumstance that may be considered is where there is
a voluntary cessation of the activity complained of by the defendant or doer. Thus, once a suit
is filed and the doer voluntarily ceases the challenged conduct, it does not automatically deprive
the tribunal of power to hear and determine the case and does not render the case moot
especially when the plaintiff seeks damages or prays for injunctive relief against the possible
recurrence of the violation.[99]

The present petitions fall squarely into these exceptions to thus thrust them into the
domain of judicial review. The grounds cited above in David are just as applicable in the
present cases as they were, not only in David, but also in Province of Batangas v.
Romulo[100] and Manalo v. Calderon[101] where the Court similarly decided them on the merits,
supervening events that would ordinarily have rendered the same moot notwithstanding.

Petitions not mooted

Contrary then to the asseverations of respondents, the non-signing of the MOA-AD and
the eventual dissolution of the GRP Peace Panel did not moot the present petitions. It bears
emphasis that the signing of the MOA-AD did not push through due to the Court’s issuance of a
Temporary Restraining Order.

Contrary too to respondents’ position, the MOA-AD cannot be considered a mere “list of
consensus points,” especially given its nomenclature, the need to have it signed or initialed by
all the parties concerned on August 5, 2008, and the far-reaching Constitutional implications of
these “consensus points,” foremost of which is the creation of the BJE.

In fact, as what will, in the main, be discussed, there is a commitment on the part of
respondents to amend and effect necessary changes to the existing legal framework for certain
provisions of the MOA-AD to take effect. Consequently, the present petitions are not confined to
the terms and provisions of the MOA-AD, but to other on-going and future negotiations and
agreements necessary for its realization. The petitions have not, therefore, been rendered moot
and academic simply by the public disclosure of the MOA-AD,[102] the manifestation that it will
not be signed as well as the disbanding of the GRP Panel not withstanding.

Petitions are imbued with paramount public interest

209
There is no gainsaying that the petitions are imbued with paramount public interest,
involving a significant part of the country’s territory and the wide-ranging political modifications
of affected LGUs. The assertion that the MOA-AD is subject to further legal enactments
including possible Constitutional amendments more than ever provides impetus for the Court
to formulate controlling principles to guide the bench, the bar, the public and, in this case, the
government and its negotiating entity.

Respondents cite Suplico v. NEDA, et al.[103] where the Court did not “pontificat[e] on
issues which no longer legitimately constitute an actual case or controversy [as this] will do
more harm than good to the nation as a whole.”

The present petitions must be differentiated from Suplico. Primarily, in Suplico, what was
assailed and eventually cancelled was a stand-alone government procurement contract for a
national broadband network involving a one-time contractual relation between two parties—the
government and a private foreign corporation. As the issues therein involved specific
government procurement policies and standard principles on contracts, the majority opinion
in Suplico found nothing exceptional therein, the factual circumstances being peculiar only to the
transactions and parties involved in the controversy.
The MOA-AD is part of a series of agreements

In the present controversy, the MOA-AD is a significant part of a series of


agreements necessary to carry out the Tripoli Agreement 2001. The MOA-AD which dwells on
the Ancestral Domain Aspect of said Tripoli Agreement is the third such component to be
undertaken following the implementation of the Security Aspect in August 2001 and
the Humanitarian, Rehabilitation and Development Aspect in May 2002.

Accordingly, even if the Executive Secretary, in his Memorandum of August 28, 2008 to
the Solicitor General, has stated that “no matter what the Supreme Court ultimately decides[,]
the government will not sign the MOA[-AD],” mootness will not set in in light of the terms of
the Tripoli Agreement 2001.

Need to formulate principles-guidelines

Surely, the present MOA-AD can be renegotiated or another one will be drawn up to carry
out the Ancestral Domain Aspect of the Tripoli Agreement 2001, in another or in any form, which
could contain similar or significantly drastic provisions. While the Court notes the word of the

210
Executive Secretary that the government “is committed to securing an agreement that is both
constitutional and equitable because that is the only way that long-lasting peace can be
assured,” it is minded to render adecision on the merits in the present petitions to formulate
controlling principles to guide the bench, the bar, the public and, most especially, the
government in negotiating with the MILF regarding Ancestral Domain.

Respondents invite the Court’s attention to the separate opinion of then Chief Justice
Artemio Panganiban in Sanlakas v. Reyes[104] in which he stated that the doctrine of “capable of
repetition yet evading review” can override mootness, “provided the party raising it in a proper
case has been and/or continue to be prejudiced or damaged as a direct result of their
issuance.” They contend that the Court must have jurisdiction over the subject matter for the
doctrine to be invoked.

The present petitions all contain prayers for Prohibition over which this Court exercises
original jurisdiction. While G.R. No. 183893 (City of Iligan v. GRP) is a petition for Injunction
and Declaratory Relief, the Court will treat it as one for Prohibition as it has far reaching
implications and raises questions that need to be resolved.[105] At all events, the Court has
jurisdiction over most if not the rest of the petitions.

Indeed, the present petitions afford a proper venue for the Court to again apply the
doctrine immediately referred to as what it had done in a number of landmark cases.[106] There
is a reasonable expectation that petitioners, particularly the Provinces of North Cotabato,
Zamboanga del Norte and Sultan Kudarat, the Cities of Zamboanga, Iligan and Isabela, and the
Municipality of Linamon, will again be subjected to the same problem in the future as
respondents’ actions are capable of repetition, in another or any form.

It is with respect to the prayers for Mandamus that the petitions have become moot,
respondents having, by Compliance of August 7, 2008, provided this Court and petitioners with
official copies of the final draft of the MOA-AD and its annexes. Too, intervenors have been
furnished, or have procured for themselves, copies of the MOA-AD.
V. SUBSTANTIVE ISSUES

As culled from the Petitions and Petitions-in-Intervention, there are basically two
SUBSTANTIVE issues to be resolved, one relating to the manner in which the MOA-AD was
negotiated and finalized, the other relating to its provisions, viz:

211
1. Did respondents violate constitutional and statutory provisions on public consultation
and the right to information when they negotiated and later initialed the MOA-AD?

2. Do the contents of the MOA-AD violate the Constitution and the laws?

ON THE FIRST SUBSTANTIVE ISSUE

Petitioners invoke their constitutional right to information on matters of public concern, as


provided in Section 7, Article III on the Bill of Rights:

Sec. 7. The right of the people to information on matters of public concern


shall be recognized. Access to official records, and to documents, and papers
pertaining to official acts, transactions, or decisions, as well as to government
research data used as basis for policy development, shall be afforded the citizen,
subject to such limitations as may be provided by law.[107]

As early as 1948, in Subido v. Ozaeta,[108] the Court has recognized the statutory right to
examine and inspect public records, a right which was eventually accorded constitutional status.

The right of access to public documents, as enshrined in both the 1973 Constitution and
the 1987 Constitution, has been recognized as a self-executory constitutional right.[109]

In the 1976 case of Baldoza v. Hon. Judge Dimaano,[110] the Court ruled that access to
public records is predicated on the right of the people to acquire information on matters of public
concern since, undoubtedly, in a democracy, the pubic has a legitimate interest in matters of
social and political significance.

x x x The incorporation of this right in the Constitution is a recognition of


the fundamental role of free exchange of information in a democracy. There can be
no realistic perception by the public of the nation’s problems, nor a meaningful
democratic decision-making if they are denied access to information of general
interest. Information is needed to enable the members of society to cope with the
exigencies of the times. As has been aptly observed: “Maintaining the flow of such
information depends on protection for both its acquisition and its dissemination
since, if either process is interrupted, the flow inevitably ceases.” x x x[111]

In the same way that free discussion enables members of society to cope with the
exigencies of their time, access to information of general interest aids the people in democratic

212
decision-making by giving them a better perspective of the vital issues confronting the
nation[112] so that they may be able to criticize and participate in the affairs of the government in
a responsible, reasonable and effective manner. It is by ensuring an unfettered and uninhibited
exchange of ideas among a well-informed public that a government remains responsive to the
changes desired by the people.[113]

The MOA-AD is a matter of public concern

That the subject of the information sought in the present cases is a matter of public
concern[114] faces no serious challenge. In fact, respondents admit that the MOA-AD is indeed of
public concern.[115] In previous cases, the Court found that the regularity of real estate
transactions entered in the Register of Deeds,[116] the need for adequate notice to the public of
the various laws,[117] the civil service eligibility of a public employee,[118] the proper management
of GSIS funds allegedly used to grant loans to public officials,[119] the recovery of the Marcoses’
alleged ill-gotten wealth,[120] and the identity of party-list nominees,[121] among others, are
matters of public concern. Undoubtedly, the MOA-AD subject of the present cases is of public
concern, involving as it does the sovereignty and territorial integrity of the State, which directly
affects the lives of the public at large.

Matters of public concern covered by the right to information include steps and
negotiations leading to the consummation of the contract. In not distinguishing as to the
executory nature or commercial character of agreements, the Court has categorically ruled:

x x x [T]he right to information “contemplates inclusion of negotiations


leading to the consummation of the transaction.” Certainly, a consummated
contract is not a requirement for the exercise of the right to information. Otherwise,
the people can never exercise the right if no contract is consummated, and if one is
consummated, it may be too late for the public to expose its defects.

Requiring a consummated contract will keep the public in the dark until the
contract, which may be grossly disadvantageous to the government or even illegal,
becomes fait accompli. This negates the State policy of full transparency on matters
of public concern, a situation which the framers of the Constitution could not have
intended. Such a requirement will prevent the citizenry from participating in the
public discussion of any proposed contract, effectively truncating a basic right
enshrined in the Bill of Rights. We can allow neither an emasculation of a
constitutional right, nor a retreat by the State of its avowed “policy of full disclosure
of all its transactions involving public interest.”[122] (Emphasis and italics in the
original)

213
Intended as a “splendid symmetry”[123] to the right to information under the Bill of Rights
is the policy of public disclosure under Section 28, Article II of the Constitution reading:

Sec. 28. Subject to reasonable conditions prescribed by law, the State


adopts and implements a policy of full public disclosure of all its transactions
involving public interest.[124]

The policy of full public disclosure enunciated in above-quoted Section


28 complements the right of access to information on matters of public concern found in the Bill
of Rights. The right to information guarantees the right of the people to demand information,
while Section 28 recognizes the duty of officialdom to give information even if nobody
demands.[125]

The policy of public disclosure establishes a concrete ethical principle for the conduct of
public affairs in a genuinely open democracy, with the people’s right to know as the
centerpiece. It is a mandate of the State to be accountable by following such policy.[126] These
provisions are vital to the exercise of the freedom of expression and essential to hold public
officials at all times accountable to the people.[127]

Whether Section 28 is self-executory, the records of the deliberations of the


Constitutional Commission so disclose:

MR. SUAREZ. And since this is not self-executory, this policy will not be
enunciated or will not be in force and effect until after Congress shall have provided
it.

MR. OPLE. I expect it to influence the climate of public ethics immediately


but, of course, the implementing law will have to be enacted by Congress, Mr.
Presiding Officer.[128]

The following discourse, after Commissioner Hilario Davide, Jr., sought clarification on the
issue, is enlightening.

MR. DAVIDE. I would like to get some clarifications on this. Mr. Presiding
Officer, did I get the Gentleman correctly as having said that this is not a self-
executing provision? It would require a legislation by Congress to implement?

214
MR. OPLE. Yes. Originally, it was going to be self-executing, but I accepted
an amendment from Commissioner Regalado, so that the safeguards on national
interest are modified by the clause “as may be provided by law”

MR. DAVIDE. But as worded, does it not mean that this will immediately take
effect and Congress may provide for reasonable safeguards on the sole ground
national interest?

MR. OPLE. Yes. I think so, Mr. Presiding Officer, I said earlier that it should
immediately influence the climate of the conduct of public affairs but, of course,
Congress here may no longer pass a law revoking it, or if this is approved, revoking
this principle, which is inconsistent with this policy.[129] (Emphasis supplied)

Indubitably, the effectivity of the policy of public disclosure need not await the passing of
a statute. As Congress cannot revoke this principle, it is merely directed to provide for
“reasonable safeguards.” The complete and effective exercise of the right to information
necessitates that its complementary provision on public disclosure derive the same self-
executory nature. Since both provisions go hand-in-hand, it is absurd to say that the
broader[130] right to information on matters of public concern is already enforceable while the
correlative duty of the State to disclose its transactions involving public interest is not
enforceable until there is an enabling law. Respondents cannot thus point to the absence of an
implementing legislation as an excuse in not effecting such policy.

An essential element of these freedoms is to keep open a continuing dialogue or process


of communication between the government and the people. It is in the interest of the State that
the channels for free political discussion be maintained to the end that the government may
perceive and be responsive to the people’s will.[131] Envisioned to becorollary to the twin rights
to information and disclosure is the design for feedback mechanisms.

MS. ROSARIO BRAID. Yes. And lastly, Mr. Presiding Officer, will the people
be able to participate? Will the government provide feedback mechanisms so that
the people can participate and can react where the existing media facilities are not
able to provide full feedback mechanisms to the government? I suppose this will be
part of the government implementing operational mechanisms.

MR. OPLE. Yes. I think through their elected representatives and that is how
these courses take place. There is a message and a feedback, both ways.

xxxx

MS. ROSARIO BRAID. Mr. Presiding Officer, may I just make one last
sentence?

215
I think when we talk about the feedback network, we are not talking about
public officials but also network of private business o[r] community-based
organizations that will be reacting. As a matter of fact, we will put more credence
or credibility on the private network of volunteers and voluntary community-based
organizations. So I do not think we are afraid that there will be another OMA in the
making.[132] (Emphasis supplied)

The imperative of a public consultation, as a species of the right to information, is evident


in the “marching orders” to respondents. The mechanics for the duty to disclose information and
to conduct public consultation regarding the peace agenda and process is manifestly provided by
E.O. No. 3.[133] The preambulatory clause of E.O. No. 3 declares that there is a need to further
enhance the contribution of civil society to the comprehensive peace process by institutionalizing
the people’s participation.

One of the three underlying principles of the comprehensive peace process is that it
“should be community-based, reflecting the sentiments, values and principles important to all
Filipinos” and “shall be defined not by the government alone, nor by the different contending
groups only, but by all Filipinos as one community.”[134] Included as a component of the
comprehensive peace process is consensus-building and empowerment for peace, which includes
“continuing consultations on both national and local levels to build consensus for a peace agenda
and process, and the mobilization and facilitation of people’s participation in the peace
process.”[135]

Clearly, E.O. No. 3 contemplates not just the conduct of a plebiscite to effectuate
“continuing” consultations, contrary to respondents’ position that plebiscite is “more than
sufficient consultation.”[136]

Further, E.O. No. 3 enumerates the functions and responsibilities of the PAPP, one of
which is to “[c]onduct regular dialogues with the National Peace Forum (NPF) and other peace
partners to seek relevant information, comments, recommendations as well as to render
appropriate and timely reports on the progress of the comprehensive peace process.”[137] E.O.
No. 3 mandates the establishment of the NPF to be “the principal forum for the PAPP to consult
with and seek advi[c]e from the peace advocates, peace partners and concerned sectors of
society on both national and local levels, on the implementation of the comprehensive peace
process, as well as for government[-]civil society dialogue and consensus-building on peace
agenda and initiatives.”[138]

216
In fine, E.O. No. 3 establishes petitioners’ right to be consulted on the peace agenda, as a
corollary to the constitutional right to information and disclosure.

PAPP Esperon committed grave abuse of discretion

The PAPP committed grave abuse of discretion when he failed to carry out the pertinent
consultation. The furtive process by which the MOA-AD was designed and crafted runs contrary
to and in excess of the legal authority, and amounts to a whimsical, capricious, oppressive,
arbitrary and despotic exercise thereof.

The Court may not, of course, require the PAPP to conduct the consultation in a particular
way or manner. It may, however, require him to comply with the law and discharge the
functions within the authority granted by the President.[139]

Petitioners are not claiming a seat at the negotiating table, contrary to respondents’
retort in justifying the denial of petitioners’ right to be consulted. Respondents’ stance manifests
the manner by which they treat the salient provisions of E.O. No. 3 on people’s
participation. Such disregard of the express mandate of the President is not much different from
superficial conduct toward token provisos that border on classic lip service.[140] It illustrates a
gross evasion of positive duty and a virtual refusal to perform the duty enjoined.

As for respondents’ invocation of the doctrine of executive privilege, it is not tenable


under the premises. The argument defies sound reason when contrasted with E.O. No. 3’s
explicit provisions on continuing consultation and dialogue on both national and local levels. The
executive order even recognizes the exercise of the public’s righteven before the GRP makes its
official recommendations or before the government proffers its definite propositions.[141] It bear
emphasis that E.O. No. 3 seeks to elicit relevant advice, information, comments and
recommendations from the people through dialogue.

AT ALL EVENTS, respondents effectively waived the defense of executive privilege in view
of their unqualified disclosure of the official copies of the final draft of the MOA-AD. By
unconditionally complying with the Court’s August 4, 2008 Resolution, without a prayer for the
document’s disclosure in camera, or without a manifestation that it was complying therewith ex
abundante ad cautelam.

217
Petitioners’ assertion that the Local Government Code (LGC) of 1991 declares it a State
policy to “require all national agencies and offices to conduct periodic consultations with
appropriate local government units, non-governmental and people's organizations, and other
concerned sectors of the community before any project or program is implemented in their
respective jurisdictions”[142] is well-taken. The LGC chapter on intergovernmental relations puts
flesh into this avowed policy:

Prior Consultations Required. – No project or program shall be


implemented by government authorities unless the consultations mentioned in
Sections 2 (c) and 26 hereof are complied with, and prior approval of the
sanggunian concerned is obtained: Provided, That occupants in areas where such
projects are to be implemented shall not be evicted unless appropriate relocation
sites have been provided, in accordance with the provisions of the
Constitution.[143] (Italics and underscoring supplied)

In Lina, Jr. v. Hon. Paño,[144] the Court held that the above-stated policy and above-
quoted provision of the LGU apply only to national programs or projects which are to be
implemented in a particular local community. Among the programs and projects covered are
those that are critical to the environment and human ecology including those that may call for
the eviction of a particular group of people residing in the locality where these will be
implemented.[145] The MOA-AD is one peculiar program that unequivocally and unilaterally vests
ownership of a vast territory to the Bangsamoro people,[146] which could pervasively and
drastically result to the diaspora or displacement of a great number of inhabitants from their
total environment.

With respect to the indigenous cultural communities/indigenous peoples (ICCs/IPs),


whose interests are represented herein by petitioner Lopez and are adversely affected by the
MOA-AD, the ICCs/IPs have, under the IPRA, the right to participate fully at all levels of
decision-making in matters which may affect their rights, lives and destinies.[147] The MOA-AD,
an instrument recognizing ancestral domain, failed to justify its non-compliance with the clear-
cut mechanisms ordained in said Act,[148] which entails, among other things, the observance of
the free and prior informed consent of the ICCs/IPs.
Notably, the IPRA does not grant the Executive Department or any government agency
the power to delineate and recognize an ancestral domain claim by mere agreement or
compromise. The recognition of the ancestral domain is the raison d’etre of the MOA-AD,
without which all other stipulations or “consensus points” necessarily must fail. In proceeding to
make a sweeping declaration on ancestral domain, without complying with the IPRA, which is

218
cited as one of the TOR of the MOA-AD,respondents clearly transcended the boundaries of their
authority. As it seems, even the heart of the MOA-AD is still subject to necessary changes to
the legal framework. While paragraph 7 on Governance suspends the effectivity of all provisions
requiring changes to the legal framework, such clause is itself invalid, as will be discussed in the
following section.

Indeed, ours is an open society, with all the acts of the government subject to public
scrutiny and available always to public cognizance. This has to be so if the country is to remain
democratic, with sovereignty residing in the people and all government authority emanating
from them.[149]

ON THE SECOND SUBSTANTIVE ISSUE

With regard to the provisions of the MOA-AD, there can be no question that they cannot
all be accommodated under the present Constitution and laws. Respondents have admitted as
much in the oral arguments before this Court, and the MOA-AD itself recognizes the need to
amend the existing legal framework to render effective at least some of its
provisions. Respondents, nonetheless, counter that the MOA-AD is free of any legal infirmity
because any provisions therein which are inconsistent with the present legal framework will not
be effective until the necessary changes to that framework are made. The validity of this
argument will be considered later. For now, the Court shall pass upon how

The MOA-AD is inconsistent with the Constitution and


laws as presently worded.

In general, the objections against the MOA-AD center on the extent of the powers
conceded therein to the BJE. Petitioners assert that the powers granted to the BJE exceed those
granted to any local government under present laws, and even go beyond those of the present
ARMM. Before assessing some of the specific powers that would have been vested in the BJE,
however, it would be useful to turn first to a general idea that serves as a unifying link to the
different provisions of the MOA-AD, namely, the international law concept
of association. Significantly, the MOA-AD explicitly alludes to this concept, indicating that the
Parties actually framed its provisions with it in mind.

219
Association is referred to in paragraph 3 on TERRITORY, paragraph 11 on RESOURCES,
and paragraph 4 on GOVERNANCE. It is in the last mentioned provision, however, that the
MOA-AD most clearly uses it to describe the envisioned relationship between the BJE and the
Central Government.

4. The relationship between the Central Government and the Bangsamoro juridical
entity shall be associative characterized by shared authority and responsibility with
a structure of governance based on executive, legislative, judicial and
administrative institutions with defined powers and functions in the comprehensive
compact. A period of transition shall be established in a comprehensive peace
compact specifying the relationship between the Central Government and the BJE.
(Emphasis and underscoring supplied)

The nature of the “associative” relationship may have been intended to be defined more
precisely in the still to be forged Comprehensive Compact. Nonetheless, given that there is a
concept of “association” in international law, and the MOA-AD – by its inclusion of international
law instruments in its TOR– placed itself in an international legal context, that concept of
association may be brought to bear in understanding the use of the term “associative” in the
MOA-AD.

Keitner and Reisman state that

[a]n association is formed when two states of unequal power voluntarily establish
durable links. In the basic model, one state, the associate, delegates certain
responsibilities to the other, the principal, while maintaining its international status
as a state. Free associations represent a middle ground between integration and
independence. x x x[150] (Emphasis and underscoring supplied)

For purposes of illustration, the Republic of the Marshall Islands and the Federated
States of Micronesia (FSM), formerly part of the U.S.-administered Trust Territory of the Pacific
Islands,[151] are associated states of the U.S. pursuant to a Compact of Free Association. The
currency in these countries is the U.S. dollar, indicating their very close ties with the U.S., yet
they issue their own travel documents, which is a mark of their statehood. Their international
legal status as states was confirmed by the UN Security Council and by their admission to UN
membership.

According to their compacts of free association, the Marshall Islands and


the FSM generally have the capacity to conduct foreign affairs in their own name and right, such
capacity extending to matters such as the law of the sea, marine resources, trade, banking,

220
postal, civil aviation, and cultural relations. The U.S. government, when conducting its foreign
affairs, is obligated to consult with the governments of the Marshall Islands or the FSM on
matters which it (U.S. government) regards as relating to or affecting either government.

In the event of attacks or threats against the Marshall Islands or the FSM, the U.S.
government has the authority and obligation to defend them as if they were part of U.S.
territory. The U.S. government, moreover, has the option of establishing and using military
areas and facilities within these associated states and has the right to bar the military personnel
of any third country from having access to these territories for military purposes.

It bears noting that in U.S. constitutional and international practice, free association is
understood as an international association between sovereigns. The Compact of Free
Association is a treaty which is subordinate to the associated nation’s national constitution, and
each party may terminate the association consistent with the right ofindependence. It has been
said that, with the admission of the U.S.-associated states to the UN in 1990, the UN recognized
that the American model of free association is actually based on an underlying status
of independence.[152]

In international practice, the “associated state” arrangement has usually been used as
a transitional device of former colonies on their way to full independence. Examples of states
that have passed through the status of associated states as a transitional phase are Antigua, St.
Kitts-Nevis-Anguilla, Dominica, St. Lucia, St. Vincent andGrenada. All have since become
[153]
independent states.

Back to the MOA-AD, it contains many provisions which are consistent with the
international legal concept of association, specifically the following: the BJE’s capacity to enter
into economic and trade relations with foreign countries, the commitment of the Central
Government to ensure the BJE’s participation in meetings and events in the ASEAN and the
specialized UN agencies, and the continuing responsibility of the Central Government over
external defense. Moreover, the BJE’s right to participate in Philippine official missions bearing
on negotiation of border agreements, environmental protection, and sharing of revenues
pertaining to the bodies of water adjacent to or between the islands forming part of the
ancestral domain, resembles the right of the governments of FSM and the Marshall Islands to be
consulted by the U.S. government on any foreign affairs matter affecting them.

221
These provisions of the MOA indicate, among other things, that the Parties aimed to vest
in the BJE the status of an associated state or, at any rate, a status closely approximating it.

The concept
of association is not recognized under the present
Constitution

No province, city, or municipality, not even the ARMM, is recognized under our laws as
having an “associative” relationship with the national government. Indeed, the concept implies
powers that go beyond anything ever granted by the Constitution to any local or regional
government. It also implies the recognition of the associated entity as a state. The
Constitution, however, does not contemplate any state in this jurisdiction other than the
Philippine State, much less does it provide for a transitory status that aims to prepare any part
of Philippine territory for independence.

Even the mere concept animating many of the MOA-AD’s provisions, therefore, already
requires for its validity the amendment of constitutional provisions, specifically the following
provisions of Article X:

SECTION 1. The territorial and political subdivisions of the Republic of


the Philippines are the provinces, cities, municipalities, and barangays. There shall
be autonomous regions in Muslim Mindanao and the Cordilleras as hereinafter
provided.

SECTION 15. There shall be created autonomous regions in Muslim Mindanao and in
the Cordilleras consisting of provinces, cities, municipalities, and geographical areas
sharing common and distinctive historical and cultural heritage, economic and social
structures, and other relevant characteristics within the framework of this
Constitution and the national sovereignty as well as territorial integrity of the
Republic of the Philippines.

The BJE is a far more powerful


entity than the autonomous region
recognized in the Constitution

It is not merely an expanded version of the ARMM, the status of its relationship with the
national government being fundamentally different from that of the ARMM. Indeed, BJE is a
state in all but name as it meets the criteria of a state laid down in the Montevideo

222
Convention,[154] namely, a permanent population, a defined territory, a government, and
a capacity to enter into relations with other states.

Even assuming arguendo that the MOA-AD would not necessarily sever any portion of
Philippine territory, the spirit animating it – which has betrayed itself by its use of the concept
of association – runs counter to the national sovereignty and territorial integrity of the
Republic.

The defining concept underlying the relationship between the national government and
the BJE being itself contrary to the present Constitution, it is not surprising that many of the
specific provisions of the MOA-AD on the formation and powers of the BJE are in conflict with the
Constitution and the laws.

Article X, Section 18 of the Constitution provides that “[t]he creation of the


autonomous region shall be effective when approved by a majority of the votes cast by
the constituent units in a plebiscite called for the purpose, provided that only provinces, cities,
and geographic areas voting favorably in such plebiscite shall be included in the autonomous
region.” (Emphasis supplied)

As reflected above, the BJE is more of a state than an autonomous region. But even
assuming that it is covered by the term “autonomous region” in the constitutional provision just
quoted, the MOA-AD would still be in conflict with it. Under paragraph 2(c) on TERRITORY in
relation to 2(d) and 2(e), the present geographic area of the ARMM and, in addition, the
municipalities of Lanao del Norte which voted for inclusion in the ARMM during the 2001
plebiscite – Baloi, Munai, Nunungan, Pantar, Tagoloan and Tangkal – are automatically part of
the BJE without need of another plebiscite, in contrast to the areas under Categories A and B
mentioned earlier in the overview. That the present components of the ARMM and the above-
mentioned municipalities voted for inclusion therein in 2001, however, does not render another
plebiscite unnecessary under the Constitution, precisely because what these areas voted for then
was their inclusion in the ARMM, not the BJE.

The MOA-AD, moreover, would not


comply with Article X, Section 20 of
the Constitution

since that provision defines the powers of autonomous regions as follows:

223
SECTION 20. Within its territorial jurisdiction and subject to the provisions of this
Constitution and national laws, the organic act of autonomous regions shall provide
for legislative powers over:

(1) Administrative organization;


(2) Creation of sources of revenues;
(3) Ancestral domain and natural resources;
(4) Personal, family, and property relations;
(5) Regional urban and rural planning development;
(6) Economic, social, and tourism development;
(7) Educational policies;
(8) Preservation and development of the cultural heritage; and
(9) Such other matters as may be authorized by law for the promotion of the
general welfare of the people of the region. (Underscoring supplied)

Again on the premise that the BJE may be regarded as an autonomous region, the MOA-
AD would require an amendment that would expand the above-quoted provision. The mere
passage of new legislation pursuant to sub-paragraph No. 9 of said constitutional provision
would not suffice, since any new law that might vest in the BJE the powers found in the MOA-AD
must, itself, comply with other provisions of the Constitution. It would not do, for instance, to
merely pass legislation vesting the BJE with treaty-making power in order to accommodate
paragraph 4 of the strand on RESOURCES which states: “The BJE is free to enter into any
economic cooperation and trade relations with foreign countries: provided, however, that such
relationships and understandings do not include aggression against the Government of the
Republic of the Philippines x x x.” Under our constitutional system, it is only the President who
has that power. Pimentel v. Executive Secretary[155] instructs:

In our system of government, the President, being the head of state, is


regarded as the sole organ and authority in external relations and is the country's
sole representative with foreign nations. As the chief architect of foreign policy, the
President acts as the country's mouthpiece with respect to international affairs.
Hence, the President is vested with the authority to deal with foreign states and
governments, extend or withhold recognition, maintain diplomatic relations, enter
into treaties, and otherwise transact the business of foreign relations. In the realm
of treaty-making, the President has the sole authority to negotiate with other
states. (Emphasis and underscoring supplied)

Article II, Section 22 of the Constitution must also be amended if the scheme envisioned
in the MOA-AD is to be effected. That constitutional provision states: “The State recognizes and
promotes the rights of indigenous cultural communities within the framework of national

224
unity and development.” (Underscoring supplied) Anassociative arrangement does not uphold
national unity. While there may be a semblance of unity because of the associative ties between
the BJE and the national government,the act of placing a portion of Philippine territory in a
status which, in international practice, has generally been a preparation for independence, is
certainly not conducive tonational unity.

Besides being irreconcilable with the Constitution, the


MOA-AD is also inconsistent with prevailing statutory law,
among which are R.A. No. 9054[156] or the Organic Act of
the ARMM, and the IPRA.[157]

Article X, Section 3 of the Organic Act of the ARMM is a bar to the adoption of the
definition of “Bangsamoro people” used in the MOA-AD. Paragraph 1 onCONCEPTS AND
PRINCIPLES states:

1. It is the birthright of all Moros and all Indigenous peoples of Mindanao to identify
themselves and be accepted as “Bangsamoros”. The Bangsamoro people refers to
those who are natives or original inhabitants of Mindanao and its adjacent
islands including Palawan and the Sulu archipelago at the time of conquest or
colonization of its descendants whether mixed or of full blood. Spouses and their
descendants are classified as Bangsamoro. The freedom of choice of the Indigenous
people shall be respected. (Emphasis and underscoring supplied)

This use of the term Bangsamoro sharply contrasts with that found in the Article X,
Section 3 of the Organic Act, which, rather than lumping together the identities of the
Bangsamoro and other indigenous peoples living in Mindanao, clearly distinguishes between
Bangsamoro people and Tribal peoples, as follows:

“As used in this Organic Act, the phrase “indigenous cultural community” refers
to Filipino citizens residing in the autonomous region who are:

(a) Tribal peoples. These are citizens whose social, cultural and economic
conditions distinguish them from other sectors of the national community; and

(b) Bangsa Moro people. These are citizens who are believers in Islam and who
have retained some or all of their own social, economic, cultural, and political
institutions.”

Respecting the IPRA, it lays down the prevailing procedure for the delineation and
recognition of ancestral domains. The MOA-AD’s manner of delineating the ancestral domain of

225
the Bangsamoro people is a clear departure from that procedure. By paragraph 1
of TERRITORY, the Parties simply agree that, subject to the delimitations in the agreed
Schedules, “[t]he Bangsamoro homeland and historic territory refer to the land mass as well as
the maritime, terrestrial, fluvial and alluvial domains, and the aerial domain, the atmospheric
space above it, embracing the Mindanao-Sulu-Palawan geographic region.”

Chapter VIII of the IPRA, on the other hand, lays down a detailed procedure, as
illustrated in the following provisions thereof:

SECTION 52. Delineation Process. — The identification and delineation of ancestral


domains shall be done in accordance with the following procedures:

xxxx

b) Petition for Delineation. — The process of delineating a specific perimeter


may be initiated by the NCIP with the consent of the ICC/IP concerned, or through
a Petition for Delineation filed with the NCIP, by a majority of the members of the
ICCs/IPs;

c) Delineation Proper. — The official delineation of ancestral domain


boundaries including census of all community members therein, shall be
immediately undertaken by the Ancestral Domains Office upon filing of the
application by the ICCs/IPs concerned. Delineation will be done in coordination with
the community concerned and shall at all times include genuine involvement and
participation by the members of the communities concerned;

d) Proof Required. — Proof of Ancestral Domain Claims shall include the


testimony of elders or community under oath, and other documents directly or
indirectly attesting to the possession or occupation of the area since time
immemorial by such ICCs/IPs in the concept of owners which shall be any one (1)
of the following authentic documents:

1) Written accounts of the ICCs/IPs customs and traditions;

2) Written accounts of the ICCs/IPs political structure and institution;

3) Pictures showing long term occupation such as those of old improvements,


burial grounds, sacred places and old villages;

4) Historical accounts, including pacts and agreements concerning boundaries


entered into by the ICCs/IPs concerned with other ICCs/IPs;

5) Survey plans and sketch maps;

6) Anthropological data;

226
7) Genealogical surveys;

8) Pictures and descriptive histories of traditional communal forests and


hunting grounds;

9) Pictures and descriptive histories of traditional landmarks such as


mountains, rivers, creeks, ridges, hills, terraces and the like; and

10) Write-ups of names and places derived from the native dialect of the
community.

e) Preparation of Maps. — On the basis of such investigation and the findings


of fact based thereon, the Ancestral Domains Office of the NCIP shall prepare a
perimeter map, complete with technical descriptions, and a description of the
natural features and landmarks embraced therein;

f) Report of Investigation and Other Documents. — A complete copy of the


preliminary census and a report of investigation, shall be prepared by the Ancestral
Domains Office of the NCIP;

g) Notice and Publication. — A copy of each document, including a translation


in the native language of the ICCs/IPs concerned shall be posted in a prominent
place therein for at least fifteen (15) days. A copy of the document shall also be
posted at the local, provincial and regional offices of the NCIP, and shall be
published in a newspaper of general circulation once a week for two (2) consecutive
weeks to allow other claimants to file opposition thereto within fifteen (15) days
from date of such publication: Provided, That in areas where no such newspaper
exists, broadcasting in a radio station will be a valid substitute: Provided, further,
That mere posting shall be deemed sufficient if both newspaper and radio station
are not available;

h) Endorsement to NCIP. — Within fifteen (15) days from publication, and of


the inspection process, the Ancestral Domains Office shall prepare a report to the
NCIP endorsing a favorable action upon a claim that is deemed to have sufficient
proof. However, if the proof is deemed insufficient, the Ancestral Domains Office
shall require the submission of additional evidence: Provided, That the Ancestral
Domains Office shall reject any claim that is deemed patently false or fraudulent
after inspection and verification: Provided, further, That in case of rejection, the
Ancestral Domains Office shall give the applicant due notice, copy furnished all
concerned, containing the grounds for denial. The denial shall be appealable to the
NCIP: Provided, furthermore, That in cases where there are conflicting claims
among ICCs/IPs on the boundaries of ancestral domain claims, the Ancestral
Domains Office shall cause the contending parties to meet and assist them in
coming up with a preliminary resolution of the conflict, without prejudice to its full
adjudication according to the section below.

xxxx

227
To remove all doubts about the irreconcilability of the MOA-AD with the present legal
system, a discussion of not only the Constitution and domestic statutes, but also of international
law is in order, for

Article II, Section 2 of the Constitution states that the


Philippines “adopts the generally accepted principles of
international law as part of the law of the land.”

Applying this provision of the Constitution, the Court, in Mejoff v. Director of Prisons,[158] held
that the Universal Declaration of Human Rights is part of the law of the land on account of which
it ordered the release on bail of a detained alien of Russian descent whose deportation order had
not been executed even after two years. Similarly, the Court in Agustin v. Edu[159] applied the
aforesaid constitutional provision to the 1968 Vienna Convention on Road Signs and Signals.

International law has long recognized the right to self-determination of “peoples,”


understood not merely as the entire population of a State but also a portion thereof. In
considering the question of whether the people of Quebec had a right to unilaterally secede from
Canada, the Canadian Supreme Court in REFERENCE RE SECESSION OF QUEBEC[160] had
occasion to acknowledge that “the right of a people to self-determination is now so widely
recognized in international conventions that the principle has acquired a status beyond
‘convention’ and is considered a general principle of international law.”

Among the conventions referred to are the International Covenant on Civil and Political
[161]
Rights and the International Covenant on Economic, Social and Cultural Rights[162] which
state, in Article 1 of both covenants, that all peoples, by virtue of the right of self-determination,
“freely determine their political status and freely pursue their economic, social, and cultural
development.”

The people’s right to self-determination should not, however, be understood as extending


to a unilateral right of secession. A distinction should be made between the right of internal and
external self-determination. REFERENCE RE SECESSION OF QUEBEC is again
instructive:

“(ii) Scope of the Right to Self-determination

126. The recognized sources of international law establish that the right to
self-determination of a people is normally fulfilled through internal self-
determination – a people’s pursuit of its political, economic, social and cultural
228
development within the framework of an existing state. A right to external self-
determination (which in this case potentially takes the form of the assertion of a
right to unilateral secession) arises in only the most extreme of cases and, even
then, under carefully defined circumstances. x x x

External self-determination can be defined as in the following statement from


the Declaration on Friendly Relations, supra, as

The establishment of a sovereign and independent State, the free association or


integration with an independent State or the emergence into any other political
status freely determined by a people constitute modes of implementing the right of
self-determination by that people. (Emphasis added)

127. The international law principle of self-determination has evolved


within a framework of respect for the territorial integrity of existing states. The
various international documents that support the existence of a people’s right to
self-determination also contain parallel statements supportive of the conclusion that
the exercise of such a right must be sufficiently limited to prevent threats to an
existing state’s territorial integrity or the stability of relations between sovereign
states.

x x x x (Emphasis, italics and underscoring supplied)

The Canadian Court went on to discuss the exceptional cases in which the right
to external self-determination can arise, namely, where a people is under colonial rule, is subject
to foreign domination or exploitation outside a colonial context, and – less definitely but asserted
by a number of commentators – is blocked from the meaningful exercise of its right
to internal self-determination. The Court ultimately held that the population of Quebec had no
right to secession, as the same is not under colonial rule or foreign domination, nor is it being
deprived of the freedom to make political choices and pursue economic, social and cultural
development, citing that Quebec is equitably represented in legislative, executive and judicial
institutions within Canada, even occupying prominent positions therein.

The exceptional nature of the right of secession is further exemplified in the REPORT OF
THE INTERNATIONAL COMMITTEE OF JURISTS ON THE LEGAL ASPECTS OF THE
[163]
AALAND ISLANDS QUESTION. There, Sweden presented to the Council of the League of
Nations the question of whether the inhabitants of theAaland Islands should be authorized to
determine by plebiscite if the archipelago should remain under Finnish sovereignty or be
incorporated in the kingdom of Sweden. The Council, before resolving the question, appointed
an International Committee composed of three jurists to submit an opinion on the preliminary

229
issue of whether the dispute should, based on international law, be entirely left to the domestic
jurisdiction of Finland. The Committee stated the rule as follows:

x x x [I]n the absence of express provisions in international treaties, the


right of disposing of national territory is essentially an attribute of the sovereignty
of every State. Positive International Law does not recognize the right of national
groups, as such, to separate themselves from the State of which they form part by
the simple expression of a wish, any more than it recognizes the right of other
States to claim such a separation. Generally speaking, the grant or refusal of the
right to a portion of its population of determining its own political fate by plebiscite
or by some other method, is, exclusively, an attribute of the sovereignty of every
State which is definitively constituted. A dispute between two States concerning
such a question, under normal conditions therefore, bears upon a question which
International Law leaves entirely to the domestic jurisdiction of one of the States
concerned. Any other solution would amount to an infringement of sovereign rights
of a State and would involve the risk of creating difficulties and a lack of stability
which would not only be contrary to the very idea embodied in term “State,” but
would also endanger the interests of the international community. If this right is
not possessed by a large or small section of a nation, neither can it be held by the
State to which the national group wishes to be attached, nor by any other
State. (Emphasis and underscoring supplied)

The Committee held that the dispute concerning the Aaland Islands did not refer to a question
which is left by international law to the domestic jurisdiction of Finland, thereby applying the
exception rather than the rule elucidated above. Its ground for departing from the general rule,
however, was a very narrow one, namely, the Aaland Islands agitation originated at a time
when Finland was undergoing drastic political transformation. The internal situation
of Finland was, according to the Committee, so abnormal that, for a considerable time, the
conditions required for the formation of a sovereign State did not exist. In the midst of
revolution, anarchy, and civil war, the legitimacy of the Finnish national government was
disputed by a large section of the people, and it had, in fact, been chased from the capital and
forcibly prevented from carrying out its duties. The armed camps and the police were divided
into two opposing forces. In light of these circumstances, Finland was not, during the relevant
time period, a “definitively constituted” sovereign state. The Committee, therefore, found that
Finland did not possess the right to withhold from a portion of its population the option to
separate itself – a right which sovereign nations generally have with respect to their own
populations.

Turning now to the more specific category of indigenous peoples, this term has been
used, in scholarship as well as international, regional, and state practices, to refer to groups with

230
distinct cultures, histories, and connections to land (spiritual and otherwise) that have been
forcibly incorporated into a larger governing society. These groups are regarded as “indigenous”
since they are the living descendants of pre-invasion inhabitants of lands now dominated by
others. Otherwise stated, indigenous peoples, nations, or communities are culturally distinctive
groups that find themselves engulfed by settler societies born of the forces of empire and
conquest.[164] Examples of groups who have been regarded as indigenous peoples are the Maori
of New Zealand and the aboriginal peoples of Canada.

As with the broader category of “peoples,” indigenous peoples situated within states do
not have a general right to independence or secession from those states under international
law,[165] but they do have rights amounting to what was discussed above as the right
to internal self-determination.

In a historic development last September 13, 2007, the UN General Assembly adopted
the United Nations Declaration on the Rights of Indigenous Peoples (UN DRIP) through General
Assembly Resolution 61/295. The vote was 143 to 4, the Philippines being included among
those in favor, and the four voting against being Australia, Canada,New Zealand, and
the U.S. The Declaration clearly recognized the right of indigenous peoples to self-
determination, encompassing the right to autonomy or self-government, to wit:

Article 3

Indigenous peoples have the right to self-determination. By virtue of that right they
freely determine their political status and freely pursue their economic, social and
cultural development.

Article 4

Indigenous peoples, in exercising their right to self-determination, have the right


to autonomy or self-government in matters relating to their internal and local
affairs, as well as ways and means for financing their autonomous functions.

Article 5

Indigenous peoples have the right to maintain and strengthen their distinct political,
legal, economic, social and cultural institutions, while retaining their right to
participate fully, if they so choose, in the political, economic, social and cultural life
of the State.

Self-government, as used in international legal discourse pertaining to indigenous


peoples, has been understood as equivalent to “internal self-determination.”[166] The extent of
231
self-determination provided for in the UN DRIP is more particularly defined in its subsequent
articles, some of which are quoted hereunder:
Article 8
1. Indigenous peoples and individuals have the right not to be subjected to
forced assimilation or destruction of their culture.
2. States shall provide effective mechanisms for prevention of, and redress for:
(a) Any action which has the aim or effect of depriving them of their
integrity as distinct peoples, or of their cultural values or ethnic identities;
(b) Any action which has the aim or effect of dispossessing them of their
lands, territories or resources;
(c) Any form of forced population transfer which has the aim or effect of
violating or undermining any of their rights;
(d) Any form of forced assimilation or integration;

(e) Any form of propaganda designed to promote or incite racial or ethnic


discrimination directed against them.
Article 21

1. Indigenous peoples have the right, without discrimination, to the


improvement of their economic and social conditions, including, inter alia, in
the areas of education, employment, vocational training and retraining,
housing, sanitation, health and social security.
2. States shall take effective measures and, where appropriate, special
measures to ensure continuing improvement of their economic and social
conditions. Particular attention shall be paid to the rights and special needs of
indigenous elders, women, youth, children and persons with disabilities.

Article 26

1. Indigenous peoples have the right to the lands, territories and resources
which they have traditionally owned, occupied or otherwise used or acquired.
2. Indigenous peoples have the right to own, use, develop and control the lands,
territories and resources that they possess by reason of traditional ownership
or other traditional occupation or use, as well as those which they have
otherwise acquired.
3. States shall give legal recognition and protection to these lands, territories
and resources. Such recognition shall be conducted with due respect to the
customs, traditions and land tenure systems of the indigenous peoples
concerned.

Article 30

1. Military activities shall not take place in the lands or territories of indigenous
peoples, unless justified by a relevant public interest or otherwise freely
agreed with or requested by the indigenous peoples concerned.

232
2. States shall undertake effective consultations with the indigenous peoples
concerned, through appropriate procedures and in particular through their
representative institutions, prior to using their lands or territories for military
activities.

Article 32

1. Indigenous peoples have the right to determine and develop priorities and
strategies for the development or use of their lands or territories and other
resources.

2. States shall consult and cooperate in good faith with the indigenous peoples
concerned through their own representative institutions in order to obtain their
free and informed consent prior to the approval of any project affecting their
lands or territories and other resources, particularly in connection with the
development, utilization or exploitation of mineral, water or other resources.

3. States shall provide effective mechanisms for just and fair redress for any
such activities, and appropriate measures shall be taken to mitigate adverse
environmental, economic, social, cultural or spiritual impact.

Article 37

1. Indigenous peoples have the right to the recognition, observance and


enforcement of treaties, agreements and other constructive arrangements
concluded with States or their successors and to have States honour and
respect such treaties, agreements and other constructive arrangements.

2. Nothing in this Declaration may be interpreted as diminishing or eliminating


the rights of indigenous peoples contained in treaties, agreements and other
constructive arrangements.

Article 38

States in consultation and cooperation with indigenous peoples, shall take the
appropriate measures, including legislative measures, to achieve the ends of this
Declaration.

Assuming that the UN DRIP, like the Universal Declaration on Human Rights, must now
be regarded as embodying customary international law – a question which the Court need not
definitively resolve here – the obligations enumerated therein do not strictly require the Republic
to grant the Bangsamoro people, through the instrumentality of the BJE, the particular rights
and powers provided for in the MOA-AD. Even the more specific provisions of the UN DRIP are
general in scope, allowing for flexibility in its application by the different States.

233
There is, for instance, no requirement in the UN DRIP that States now guarantee
indigenous peoples their own police and internal security force. Indeed, Article 8 presupposes
that it is the State which will provide protection for indigenous peoples against acts like the
forced dispossession of their lands – a function that is normally performed by police officers. If
the protection of a right so essential to indigenous people’s identity is acknowledged to be the
responsibility of the State, then surely the protection of rights less significant to them as such
peoples would also be the duty of States. Nor is there in the UN DRIP an acknowledgement of
the right of indigenous peoples to the aerial domain and atmospheric space. What it upholds, in
Article 26 thereof, is the right of indigenous peoples to the lands, territories and resources which
they have traditionally owned, occupied or otherwise used or acquired.

Moreover, the UN DRIP, while upholding the right of indigenous peoples to autonomy,
does not obligate States to grant indigenous peoples the near-independent status of an
associated state. All the rights recognized in that document are qualified in Article 46 as follows:

1. Nothing in this Declaration may be interpreted as implying for any State,


people, group or person any right to engage in any activity or to perform any
act contrary to the Charter of the United Nations or construed as authorizing or
encouraging any action which would dismember or impair, totally or in part,
the territorial integrity or political unity of sovereign and independent States.

Even if the UN DRIP were considered as part of the law of the land pursuant to Article II,
Section 2 of the Constitution, it would not suffice to uphold the validity of the MOA-AD so as to
render its compliance with other laws unnecessary.

It is, therefore, clear that the MOA-AD contains numerous provisions that cannot be
reconciled with the Constitution and the laws as presently worded. Respondents proffer,
however, that the signing of the MOA-AD alone would not have entailed any violation of law or
grave abuse of discretion on their part, precisely because it stipulates that the provisions thereof
inconsistent with the laws shall not take effect until these laws are amended. They cite
paragraph 7 of the MOA-AD strand on GOVERNANCE quoted earlier, but which is reproduced
below for convenience:

7. The Parties agree that the mechanisms and modalities for the actual
implementation of this MOA-AD shall be spelt out in the Comprehensive Compact to
mutually take such steps to enable it to occur effectively.
Any provisions of the MOA-AD requiring amendments to the existing legal
framework shall come into force upon signing of a Comprehensive Compact and

234
upon effecting the necessary changes to the legal framework with due regard to
non derogation of prior agreements and within the stipulated timeframe to be
contained in the Comprehensive Compact.

Indeed, the foregoing stipulation keeps many controversial provisions of the MOA-AD from
coming into force until the necessary changes to the legal framework are effected. While the
word “Constitution” is not mentioned in the provision now under consideration or anywhere else
in the MOA-AD, the term “legal framework” is certainly broad enough to include the Constitution.

Notwithstanding the suspensive clause, however, respondents, by their mere act of


incorporating in the MOA-AD the provisions thereof regarding the associative relationship
between the BJE and the Central Government, have already violated the Memorandum of
Instructions From The President dated March 1, 2001, which states that the “negotiations shall
be conducted in accordance with x x x the principles of the sovereignty and territorial integrity of
the Republic of the Philippines.” (Emphasis supplied) Establishing an associative relationship
between the BJE and the Central Government is, for the reasons already discussed, a
preparation for independence, or worse, an implicit acknowledgment of an independent status
already prevailing.

Even apart from the above-mentioned Memorandum, however, the MOA-AD is defective
because the suspensive clause is invalid, as discussed below.

The authority of the GRP Peace Negotiating Panel to negotiate with the MILF is founded
on E.O. No. 3, Section 5(c), which states that there shall be established Government Peace
Negotiating Panels for negotiations with different rebel groups to be “appointed by the President
as her official emissaries to conduct negotiations, dialogues, and face-to-face discussions with
rebel groups.” These negotiating panels are to report to the President, through the PAPP on the
conduct and progress of the negotiations.

It bears noting that the GRP Peace Panel, in exploring lasting solutions to the Moro
Problem through its negotiations with the MILF, was not restricted by E.O. No. 3 only to those
options available under the laws as they presently stand. One of the components of a
comprehensive peace process, which E.O. No. 3 collectively refers to as the “Paths to Peace,” is
the pursuit of social, economic, and political reforms which may require new legislation or even
constitutional amendments. Sec. 4(a) of E.O. No. 3, which reiterates Section 3(a), of E.O. No.
125,[167] states:

235
SECTION 4. The Six Paths to Peace. – The components of the comprehensive peace
process comprise the processes known as the “Paths to Peace”. These component
processes are interrelated and not mutually exclusive, and must therefore be
pursued simultaneously in a coordinated and integrated fashion. They shall include,
but may not be limited to, the following:

a. PURSUIT OF SOCIAL, ECONOMIC AND POLITICAL REFORMS. This component


involves the vigorous implementation of various policies, reforms, programs
and projects aimed at addressing the root causes of internal armed conflicts
and social unrest. This may require administrative action, new legislation or
even constitutional amendments.

x x x x (Emphasis supplied)

The MOA-AD, therefore, may reasonably be perceived as an attempt of respondents to


address, pursuant to this provision of E.O. No. 3, the root causes of the armed conflict
in Mindanao. The E.O. authorized them to “think outside the box,” so to speak. Hence, they
negotiated and were set on signing the MOA-AD that included various social, economic, and
political reforms which cannot, however, all be accommodated within the present legal
framework, and which thus would require new legislation and constitutional amendments.

The inquiry on the legality of the “suspensive clause,” however, cannot stop here,
because it must be asked

whether the President herself may exercise the power


delegated to the GRP Peace Panel under E.O. No. 3, Sec.
4(a).
The President cannot delegate a power that she herself does not possess. May the
President, in the course of peace negotiations, agree to pursue reforms that would require new
legislation and constitutional amendments, or should the reforms be restricted only to those
solutions which the present laws allow? The answer to this question requires a discussion of

the extent of the President’s power to conduct peace


negotiations.

That the authority of the President to conduct peace negotiations with rebel groups is not
explicitly mentioned in the Constitution does not mean that she has no such authority.
In Sanlakas v. Executive Secretary,[168] in issue was the authority of the President to declare a
state of rebellion – an authority which is not expressly provided for in the Constitution. The
Court held thus:
236
“In her ponencia in Marcos v. Manglapus, Justice Cortes put her thesis into
jurisprudence. There, the Court, by a slim 8-7 margin, upheld the President's
power to forbid the return of her exiled predecessor. The rationale for the majority's
ruling rested on the President's

. . . unstated residual powers which are implied from the grant


of executive power and which are necessary for her to comply with her
duties under the Constitution.The powers of the President are not
limited to what are expressly enumerated in the article on the
Executive Department and in scattered provisions of the
Constitution.This is so, notwithstanding the avowed intent of the
members of the Constitutional Commission of 1986 to limit the powers
of the President as a reaction to the abuses under the regime of Mr.
Marcos, for the result was a limitation of specific powers of the
President, particularly those relating to the commander-in-chief clause,
but not a diminution of the general grant of executive power.

Thus, the President's authority to declare a state of rebellion springs in the


main from her powers as chief executive and, at the same time, draws strength
from her Commander-in-Chief powers. x x x (Emphasis and underscoring
supplied)
Similarly, the President’s power to conduct peace negotiations is implicitly included in her
powers as Chief Executive and Commander-in-Chief. As Chief Executive, the President has the
general responsibility to promote public peace, and as Commander-in-Chief, she has the more
specific duty to prevent and suppress rebellion and lawless violence.[169]

As the experience of nations which have similarly gone through internal armed conflict
will show, however, peace is rarely attained by simply pursuing a military solution. Oftentimes,
changes as far-reaching as a fundamental reconfiguration of the nation’s constitutional structure
is required. The observations of Dr. Kirsti Samuels are enlightening, to wit:

x x x [T]he fact remains that a successful political and governance transition


must form the core of any post-conflict peace-building mission. As we have
observed in Liberia and Haiti over the last ten years, conflict cessation without
modification of the political environment, even where state-building is undertaken
through technical electoral assistance and institution- or capacity-building, is
unlikely to succeed. On average, more than 50 percent of states emerging from
conflict return to conflict. Moreover, a substantial proportion of transitions have
resulted in weak or limited democracies.

The design of a constitution and its constitution-making process can play an


important role in the political and governance transition. Constitution-making after
conflict is an opportunity to create a common vision of the future of a state and a
road map on how to get there. The constitution can be partly a peace agreement

237
and partly a framework setting up the rules by which the new democracy will
operate.[170]

In the same vein, Professor Christine Bell, in her article on the nature and legal status of
peace agreements, observed that the typical way that peace agreements establish or confirm
mechanisms for demilitarization and demobilization is by linking them to new constitutional
structures addressing governance, elections, and legal and human rights institutions.[171]

In the Philippine experience, the link between peace agreements and constitution-making
has been recognized by no less than the framers of the Constitution. Behind the provisions of
[172]
the Constitution on autonomous regions is the framers’ intention to implement a particular
peace agreement, namely, the Tripoli Agreement of 1976 between the GRP and the MNLF,
signed by then Undersecretary of National Defense Carmelo Z. Barbero and then MNLF Chairman
Nur Misuari.

MR. ROMULO. There are other speakers; so, although I have some more
questions, I will reserve my right to ask them if they are not covered by the other
speakers. I have only two questions.
I heard one of the Commissioners say that local autonomy already exists in
the Muslim region; it is working very well; it has, in fact, diminished a great deal of
the problems. So, my question is: since that already exists, why do we have to go
into something new?

MR. OPLE. May I answer that on behalf of Chairman Nolledo. Commissioner


Yusup Abubakar is right that certain definite steps have been taken to implement
the provisions of theTripoli Agreement with respect to an autonomous region
in Mindanao. This is a good first step, but there is no question that this is merely a
partial response to the Tripoli Agreement itself and to the fuller standard of regional
autonomy contemplated in that agreement, and now by state
policy.[173] (Emphasis supplied)

The constitutional provisions on autonomy and the statutes enacted pursuant to them
have, to the credit of their drafters, been partly successful. Nonetheless, the Filipino people are
still faced with the reality of an on-going conflict between the Government and the MILF. If the
President is to be expected to find means for bringing this conflict to an end and to achieve
lasting peace in Mindanao, then she must be given the leeway to explore, in the course of peace
negotiations, solutions that may require changes to the Constitution for their
implementation. Being uniquely vested with the power to conduct peace negotiations with rebel
groups, the President is in a singular position to know the precise nature of their grievances
which, if resolved, may bring an end to hostilities.
238
The President may not, of course, unilaterally implement the solutions that she considers
viable, but she may not be prevented from submitting them as recommendations to Congress,
which could then, if it is minded, act upon them pursuant to the legal procedures for
constitutional amendment and revision. In particular, Congress would have the option, pursuant
to Article XVII, Sections 1 and 3 of the Constitution, to propose the recommended amendments
or revision to the people, call a constitutional convention, or submit to the electorate the
question of calling such a convention.

While the President does not possess constituent powers – as those powers may be
exercised only by Congress, a Constitutional Convention, or the people through initiative and
referendum – she may submit proposals for constitutional change to Congress in a manner that
does not involve the arrogation of constituent powers.

In Sanidad v. COMELEC,[174] in issue was the legality of then President Marcos’ act of
directly submitting proposals for constitutional amendments to a referendum, bypassing the
interim National Assembly which was the body vested by the 1973 Constitution with the power
to propose such amendments. President Marcos, it will be recalled, never convened the interim
National Assembly. The majority upheld the President’s act, holding that “the urges of absolute
necessity” compelled the President as the agent of the people to act as he did, there being no
interim National Assembly to propose constitutional amendments. Against this ruling, Justices
Teehankee and Muñoz Palma vigorously dissented. The Court’s concern at present, however, is
not with regard to the point on which it was then divided in that controversial case, but on that
which was not disputed by either side.

Justice Teehankee’s dissent,[175] in particular, bears noting. While he disagreed that the
President may directly submit proposed constitutional amendments to a referendum, implicit in
his opinion is a recognition that he would have upheld the President’s action along with the
majority had the President convened the interim National Assembly and coursed his proposals
through it. Thus Justice Teehankee opined:

“Since the Constitution provides for the organization of the essential


departments of government, defines and delimits the powers of each and prescribes
the manner of the exercise of such powers, and the constituent power has not been
granted to but has been withheld from the President or Prime Minister, it follows
that the President’s questioned decrees proposing and submitting constitutional
amendments directly to the people (without the intervention of the interim National

239
Assembly in whom the power is expressly vested) are devoid of constitutional and
legal basis.”[176] (Emphasis supplied)

From the foregoing discussion, the principle may be inferred that the President – in the
course of conducting peace negotiations – may validly consider implementing even those policies
that require changes to the Constitution, but she may not unilaterally implement them without
the intervention of Congress, or act in any way as if the assent of that body were assumed as a
certainty.

Since, under the present Constitution, the people also have the power to directly propose
amendments through initiative and referendum, the President may also submit her
recommendations to the people, not as a formal proposal to be voted on in a plebiscite similar to
what President Marcos did in Sanidad, but for their independent consideration of whether these
recommendations merit being formally proposed through initiative.

These recommendations, however, may amount to nothing more than the President’s
suggestions to the people, for any further involvement in the process of initiative by the Chief
Executive may vitiate its character as a genuine “people’s initiative.” The only initiative
recognized by the Constitution is that which truly proceeds from the people. As the Court stated
in Lambino v. COMELEC:[177]

“The Lambino Group claims that their initiative is the ‘people's voice.’
However, the Lambino Group unabashedly states in ULAP Resolution No. 2006-02,
in the verification of their petition with the COMELEC, that ‘ULAP maintains its
unqualified support to the agenda of Her Excellency President Gloria Macapagal-
Arroyo for constitutional reforms.’ The Lambino Group thus admits that their
‘people's’ initiative is an ‘unqualified support to the agenda’ of the incumbent
President to change the Constitution. This forewarns the Court to be wary of
incantations of ‘people's voice’ or ‘sovereign will’ in the present initiative.”

It will be observed that the President has authority, as stated in her oath of
[178]
office, only to preserve and defend the Constitution. Such presidential power does not,
however, extend to allowing her to change the Constitution, but simply to recommend proposed
amendments or revision. As long as she limits herself to recommending these changes and
submits to the proper procedure for constitutional amendments and revision, her mere
recommendation need not be construed as an unconstitutional act.

240
The foregoing discussion focused on the President’s authority to
propose constitutional amendments, since her authority to propose new legislation is not in
controversy. It has been an accepted practice for Presidents in this jurisdiction to propose new
legislation. One of the more prominent instances the practice is usually done is in the yearly
State of the Nation Address of the President to Congress. Moreover, the annual general
appropriations bill has always been based on the budget prepared by the President, which – for
all intents and purposes – is a proposal for new legislation coming from the President.[179]

The “suspensive clause” in the MOA-AD viewed in light of


the above-discussed standards

Given the limited nature of the President’s authority to propose constitutional


amendments, she cannot guarantee to any third party that the required amendments will
eventually be put in place, nor even be submitted to a plebiscite. The most she could do is
submit these proposals as recommendations either to Congress or the people, in whom
constituent powers are vested.

Paragraph 7 on Governance of the MOA-AD states, however, that all provisions thereof
which cannot be reconciled with the present Constitution and laws “shall come into force upon
signing of a Comprehensive Compact and upon effecting the necessary changes to the legal
framework.” This stipulation does not bear the marks of a suspensive condition – defined in civil
law as a future and uncertain event – but of a term. It is not a question of whether the
necessary changes to the legal framework will be effected, but when. That there is no
uncertainty being contemplated is plain from what follows, for the paragraph goes on to state
that the contemplated changes shall be “with due regard to non derogation of prior
agreements and within the stipulated timeframe to be contained in the Comprehensive
Compact.”

Pursuant to this stipulation, therefore, it is mandatory for the GRP to effect the changes to
the legal framework contemplated in the MOA-AD – which changes would include constitutional
amendments, as discussed earlier. It bears noting that,

By the time these changes are put in place, the MOA-AD


itself would be counted among the “prior agreements”
from which there could be no derogation.

241
What remains for discussion in the Comprehensive Compact would merely be the implementing
details for these “consensus points” and, notably, the deadline for effecting the contemplated
changes to the legal framework.

Plainly, stipulation-paragraph 7 on GOVERNANCE is inconsistent with the limits of the


President’s authority to propose constitutional amendments, it being a virtual guarantee that the
Constitution and the laws of the Republic of the Philippines will certainly be adjusted to conform
to all the “consensus points” found in the MOA-AD. Hence, it must be struck down
as unconstitutional.

A comparison between the “suspensive clause” of the MOA-AD with a similar provision
appearing in the 1996 final peace agreement between the MNLF and the GRP is most instructive.

As a backdrop, the parties to the 1996 Agreement stipulated that it would be implemented
in two phases. Phase I covered a three-year transitional period involving the putting up of new
administrative structures through Executive Order, such as the Special Zone of Peace and
Development (SZOPAD) and the Southern Philippines Council for Peace and Development
(SPCPD), while Phase II covered the establishment of the new regional autonomous
government through amendment or repeal of R.A. No. 6734, which was then the Organic Act of
the ARMM.

The stipulations on Phase II consisted of specific agreements on the structure of the


expanded autonomous region envisioned by the parties. To that extent, they are similar to the
provisions of the MOA-AD. There is, however, a crucial difference between the two
agreements. While the MOA-AD virtually guarantees that the “necessary changes to the legal
framework” will be put in place, the GRP-MNLF final peace agreement states thus: “Accordingly,
these provisions [on Phase II] shall berecommended by the GRP to Congress for incorporation in
the amendatory or repealing law.”

Concerns have been raised that the MOA-AD would have given rise to a binding
international law obligation on the part of the Philippines to change its Constitution in conformity
thereto, on the ground that it may be considered either as a binding agreement under
international law, or a unilateral declaration of the Philippine government to the international
community that it would grant to the Bangsamoro people all the concessions therein
stated. Neither ground finds sufficient support in international law,
however.

242
The MOA-AD, as earlier mentioned in the overview thereof, would have included foreign
dignitaries as signatories. In addition, representatives of other nations were invited to witness
its signing in Kuala Lumpur. These circumstances readily lead one to surmise that the MOA-AD
would have had the status of a binding international agreement had it been signed. An
examination of the prevailing principles in international law, however, leads to the contrary
conclusion.

The Decision on CHALLENGE TO JURISDICTION: LOMÉ ACCORD AMNESTY[180] (the Lomé


Accord case) of the Special Court of Sierra Leone is enlightening. The Lomé Accord was a peace
agreement signed on July 7, 1999 between the Government of Sierra Leone and the
Revolutionary United Front (RUF), a rebel group with which the Sierra Leone Government had
been in armed conflict for around eight years at the time of signing. There were non-contracting
signatories to the agreement, among which were the Government of the Togolese Republic,
the Economic Community of West African States, and the UN.

On January 16, 2002, after a successful negotiation between the UN Secretary-General


and the Sierra Leone Government, another agreement was entered into by the UN and that
Government whereby the Special Court of Sierra Leone was established. The sole purpose of
the Special Court, an international court, was to try persons who bore the greatest responsibility
for serious violations of international humanitarian law and Sierra Leonean law committed in the
territory of Sierra Leone since November 30, 1996.

Among the stipulations of the Lomé Accord was a provision for the full pardon of the
members of the RUF with respect to anything done by them in pursuit of their objectives as
members of that organization since the conflict began.

In the Lomé Accord case, the Defence argued that the Accord created an internationally
binding obligation not to prosecute the beneficiaries of the amnesty provided therein, citing,
among other things, the participation of foreign dignitaries and international organizations in the
finalization of that agreement. The Special Court, however, rejected this argument, ruling that
the Lome Accord is not a treaty and that it can only create binding obligations and rights
between the parties in municipal law, not in international law. Hence, the Special Court held, it
is ineffective in depriving an international court like it of jurisdiction.

“37. In regard to the nature of a negotiated settlement of an internal armed


conflict it is easy to assume and to argue with some degree of plausibility, as
243
Defence counsel for the defendants seem to have done, that the mere fact that
in addition to the parties to the conflict, the document formalizing the
settlement is signed by foreign heads of state or their representatives and
representatives of international organizations, means the agreement of the
parties is internationalized so as to create obligations in international law.

xxxx

40. Almost every conflict resolution will involve the parties to the conflict and the
mediator or facilitator of the settlement, or persons or bodies under whose
auspices the settlement took place but who are not at all parties to the
conflict, are not contracting parties and who do not claim any obligation from
the contracting parties or incur any obligation from the settlement.

41. In this case, the parties to the conflict are the lawful authority of the State
and the RUF which has no status of statehood and is to all intents and
purposes a faction within the state. The non-contracting signatories of the
Lomé Agreement were moral guarantors of the principle that, in the terms of
Article XXXIV of the Agreement, “this peace agreement is implemented with
integrity and in good faith by both parties”. The moral guarantors assumed no
legal obligation. It is recalled that the UN by its representative appended,
presumably for avoidance of doubt, an understanding of the extent of the
agreement to be implemented as not including certain international crimes.

42. An international agreement in the nature of a treaty must create rights and
obligations regulated by international law so that a breach of its terms will be a
breach determined under international law which will also provide principle
means of enforcement. The Lomé Agreement created neither rights nor
obligations capable of being regulated by international law. An agreement
such as the Lomé Agreement which brings to an end an internal armed conflict
no doubt creates a factual situation of restoration of peace that the
international community acting through the Security Council may take note
of. That, however, will not convert it to an international agreement which
creates an obligation enforceable in international, as distinguished from
municipal, law. A breach of the terms of such a peace agreement resulting in
resumption of internal armed conflict or creating a threat to peace in the
determination of the Security Council may indicate a reversal of the factual
situation of peace to be visited with possible legal consequences arising from
the new situation of conflict created. Such consequences such as action by the
Security Council pursuant to Chapter VII arise from the situation and not from
the agreement, nor from the obligation imposed by it. Such action cannot be
regarded as a remedy for the breach. A peace agreement which settles
an internal armed conflict cannot be ascribed the same status as one which
settles an international armed conflict which, essentially, must be between two
or more warring States. The Lomé Agreement cannot be characterised as an
international instrument. x x x” (Emphasis, italics and underscoring supplied)

244
Similarly, that the MOA-AD would have been signed by representatives of States and
international organizations not parties to the Agreement would not have sufficed to vest in it a
binding character under international law.

In another vein, concern has been raised that the MOA-AD would amount to a unilateral
declaration of the Philippine State, binding under international law, that it would comply with all
the stipulations stated therein, with the result that it would have to amend its Constitution
accordingly regardless of the true will of the people. Cited as authority for this view is Australia
v. France,[181] also known as the Nuclear Tests Case, decided by the International Court of
Justice (ICJ).

In the Nuclear Tests Case, Australia challenged before the ICJ the legality of France’s
nuclear tests in the South Pacific. France refused to appear in the case, but public statements
from its President, and similar statements from other French officials including its Minister of
Defence, that its 1974 series of atmospheric tests would be its last, persuaded the ICJ to dismiss
the case.[182] Those statements, the ICJ held, amounted to a legal undertaking addressed to
the international community, which required no acceptance from other States for it to become
effective.

Essential to the ICJ ruling is its finding that the French government intended to be bound
to the international community in issuing its public statements, viz:

43. It is well recognized that declarations made by way of unilateral acts,


concerning legal or factual situations, may have the effect of creating legal
obligations. Declarations of this kind may be, and often are, very
specific. When it is the intention of the State making the declaration that it
should become bound according to its terms, that intention confers on the
declaration the character of a legal undertaking, the State being thenceforth
legally required to follow a course of conduct consistent with the
declaration. An undertaking of this kind, if given publicly, and with an intent to
be bound, even though not made within the context of international
negotiations, is binding. In these circumstances, nothing in the nature of a
quid pro quo nor any subsequent acceptance of the declaration, nor even any
reply or reaction from other States, is required for the declaration to take
effect, since such a requirement would be inconsistent with the strictly
unilateral nature of the juridical act by which the pronouncement by the State
was made.

44. Of course, not all unilateral acts imply obligation; but a State may choose to
take up a certain position in relation to a particular matter with the intention of
being bound–the intention is to be ascertained by interpretation of the

245
act. When States make statements by which their freedom of action is to be
limited, a restrictive interpretation is called for.

xxxx

51. In announcing that the 1974 series of atmospheric tests would be the last,
the French Government conveyed to the world at large, including the
Applicant, its intention effectively to terminate these tests. It was bound to
assume that other States might take note of these statements and rely on
their being effective. The validity of these statements and their legal
consequences must be considered within the general framework of the security
of international intercourse, and the confidence and trust which are so
essential in the relations among States. It is from the actual substance of
these statements, and from the circumstances attending their making, that the
legal implications of the unilateral act must be deduced. The objects of these
statements are clear and they were addressed to the international community
as a whole, and the Court holds that they constitute an undertaking possessing
legal effect. The Court considers *270 that the President of the Republic, in
deciding upon the effective cessation of atmospheric tests, gave an
undertaking to the international community to which his words were
addressed. x x x (Emphasis and underscoring supplied)

As gathered from the above-quoted ruling of the ICJ, public statements of a state
representative may be construed as a unilateral declaration only when the following conditions
are present: the statements were clearly addressed to the international community, the state
intended to be bound to that community by its statements, and that not to give legal effect to
those statements would be detrimental to the security of international intercourse. Plainly,
unilateral declarations arise only in peculiar circumstances.

The limited applicability of the Nuclear Tests Case ruling was recognized in a later case
decided by the ICJ entitled Burkina Faso v. Mali,[183] also known as the Case Concerning the
Frontier Dispute. The public declaration subject of that case was a statement made by the
President of Mali, in an interview by a foreign press agency, that Maliwould abide by the decision
to be issued by a commission of the Organization of African Unity on a frontier dispute then
pending between Mali and Burkina Faso.
Unlike in the Nuclear Tests Case, the ICJ held that the statement of Mali’s President was
not a unilateral act with legal implications. It clarified that its ruling in the Nuclear Tests case
rested on the peculiar circumstances surrounding the French declaration subject thereof, to wit:

40. In order to assess the intentions of the author of a unilateral act, account
must be taken of all the factual circumstances in which the act occurred. For
example, in the Nuclear Tests cases, the Court took the view that since the
applicant States were not the only ones concerned at the possible continuance
246
of atmospheric testing by the French Government, that Government's
unilateral declarations had ‘conveyed to the world at large, including the
Applicant, its intention effectively to terminate these tests‘ (I.C.J. Reports
1974, p. 269, para. 51; p. 474, para. 53). In the particular circumstances of
those cases, the French Government could not express an intention to be
bound otherwise than by unilateral declarations. It is difficult to see how it
could have accepted the terms of a negotiated solution with each of the
applicants without thereby jeopardizing its contention that its conduct was
lawful. The circumstances of the present case are radically different. Here,
there was nothing to hinder the Parties from manifesting an intention to accept
the binding character of the conclusions of the Organization of African Unity
Mediation Commission by the normal method: a formal agreement on the
basis of reciprocity. Since no agreement of this kind was concluded between
the Parties, the Chamber finds that there are no grounds to interpret the
declaration made by Mali's head of State on 11 April 1975 as a unilateral act
with legal implications in regard to the present case. (Emphasis and
underscoring supplied)

Assessing the MOA-AD in light of the above criteria, it would not have amounted to a
unilateral declaration on the part of the Philippine State to the international community. The
Philippine panel did not draft the same with the clear intention of being bound thereby to the
international community as a whole or to any State, but only to the MILF. While there were
States and international organizations involved, one way or another, in the negotiation and
projected signing of the MOA-AD, they participated merely as witnesses or, in the case
of Malaysia, as facilitator. As held in the Lomé Accord case, the mere fact that in addition to the
parties to the conflict, the peace settlement is signed by representatives of states and
international organizations does not mean that the agreement is internationalized so as to create
obligations in international law.

Since the commitments in the MOA-AD were not addressed to States, not to give legal
effect to such commitments would not be detrimental to the security of international intercourse
– to the trust and confidence essential in the relations among States.

In one important respect, the circumstances surrounding the MOA-AD are closer to that
of Burkina Faso wherein, as already discussed, the Mali President’s statement was not held to be
a binding unilateral declaration by the ICJ. As in that case, there was also nothing to hinder the
Philippine panel, had it really been its intention to be bound to other States, to manifest that
intention by formal agreement. Here, that formal agreement would have come about by the
inclusion in the MOA-AD of a clear commitment to be legally bound to the international
community, not just the MILF, and by an equally clear indication that the signatures of the

247
participating states-representatives would constitute an acceptance of that
commitment. Entering into such a formal agreement would not have resulted in a loss of face
for the Philippine government before the international community, which was one of the
difficulties that prevented the French Government from entering into a formal agreement with
other countries. That the Philippine panel did not enter into such a formal agreement suggests
that it had no intention to be bound to the international community. On that ground, the MOA-
AD may not be considered a unilateral declaration under international law.

The MOA-AD not being a document that can bind the Philippines under international law
notwithstanding, respondents’ almost consummated act of guaranteeing amendments to the
legal framework is, by itself, sufficient to constitute grave abuse of discretion. The grave abuse
lies not in the fact that they considered, as a solution to the Moro Problem, the creation of a
state within a state, but in their brazen willingness to guarantee that Congress and the
sovereign Filipino people would give their imprimatur to their solution. Upholding such an act
would amount to authorizing a usurpation of the constituent powers vested only in Congress, a
Constitutional Convention, or the people themselves through the process of initiative, for the
only way that the Executive can ensure the outcome of the amendment process is through an
undue influence or interference with that process.

The sovereign people may, if it so desired, go to the extent of giving up a portion of its
own territory to the Moros for the sake of peace, for it can change the Constitution in any it
wants, so long as the change is not inconsistent with what, in international law, is known as Jus
Cogens.[184] Respondents, however, may not preempt it in that decision.

SUMMARY

The petitions are ripe for adjudication. The failure of respondents to consult the local
government units or communities affected constitutes a departure by respondents from their
mandate under E.O. No. 3. Moreover, respondents exceeded their authority by the mere act of
guaranteeing amendments to the Constitution. Any alleged violation of the Constitution by any
branch of government is a proper matter for judicial review.

As the petitions involve constitutional issues which are of paramount public interest or of
transcendental importance, the Court grants the petitioners, petitioners-in-intervention and
intervening respondents the requisite locus standi in keeping with the liberal stance adopted
in David v. Macapagal-Arroyo.
248
Contrary to the assertion of respondents that the non-signing of the MOA-AD and the
eventual dissolution of the GRP Peace Panel mooted the present petitions, the Court finds that
the present petitions provide an exception to the “moot and academic” principle in view of (a)
the grave violation of the Constitution involved; (b) the exceptional character of the situation
and paramount public interest; (c) the need to formulate controlling principles to guide the
bench, the bar, and the public; and (d) the fact that the case is capable of repetition yet evading
review.

The MOA-AD is a significant part of a series of agreements necessary to carry out the
GRP-MILF Tripoli Agreement on Peace signed by the government and the MILF back in June
2001. Hence, the present MOA-AD can be renegotiated or another one drawn up that could
contain similar or significantly dissimilar provisions compared to the original.

The Court, however, finds that the prayers for mandamus have been rendered moot in
view of the respondents’ action in providing the Court and the petitioners with the official copy of
the final draft of the MOA-AD and its annexes.

The people’s right to information on matters of public concern under Sec. 7, Article III of
the Constitution is in splendid symmetry with the state policy of full public disclosure of all its
transactions involving public interest under Sec. 28, Article II of the Constitution. The right to
information guarantees the right of the people to demand information, while Section 28
recognizes the duty of officialdom to give information even if nobody demands. The complete
and effective exercise of the right to information necessitates that its complementary provision
on public disclosure derive the same self-executory nature, subject only to reasonable
safeguards or limitations as may be provided by law.

The contents of the MOA-AD is a matter of paramount public concern involving public
interest in the highest order. In declaring that the right to information contemplates steps and
negotiations leading to the consummation of the contract, jurisprudence finds no distinction as
to the executory nature or commercial character of the agreement.

An essential element of these twin freedoms is to keep a continuing dialogue or process of


communication between the government and the people. Corollary to these twin rights is the
design for feedback mechanisms. The right to public consultation was envisioned to be a species
of these public rights.

249
At least three pertinent laws animate these constitutional imperatives and justify the
exercise of the people’s right to be consulted on relevant matters relating to the peace agenda.

One, E.O. No. 3 itself is replete with mechanics for continuing consultations on both
national and local levels and for a principal forum for consensus-building. In fact, it is the duty
of the Presidential Adviser on the Peace Process to conduct regular dialogues to seek relevant
information, comments, advice, and recommendations from peace partners and concerned
sectors of society.

Two, Republic Act No. 7160 or the Local Government Code of 1991 requires all national
offices to conduct consultations before any project or program critical to the environment and
human ecology including those that may call for the eviction of a particular group of people
residing in such locality, is implemented therein. The MOA-AD is one peculiar program that
unequivocally and unilaterally vests ownership of a vast territory to the Bangsamoro people,
which could pervasively and drastically result to the diaspora or displacement of a great number
of inhabitants from their total environment.

Three, Republic Act No. 8371 or the Indigenous Peoples Rights Act of 1997 provides for
clear-cut procedure for the recognition and delineation of ancestral domain, which entails,
among other things, the observance of the free and prior informed consent of the Indigenous
Cultural Communities/Indigenous Peoples. Notably, the statute does not grant the Executive
Department or any government agency the power to delineate and recognize an ancestral
domain claim by mere agreement or compromise.

The invocation of the doctrine of executive privilege as a defense to the general right to
information or the specific right to consultation is untenable. The various explicit legal
provisions fly in the face of executive secrecy. In any event, respondents effectively waived
such defense after it unconditionally disclosed the official copies of the final draft of the MOA-AD,
for judicial compliance and public scrutiny.

IN SUM, the Presidential Adviser on the Peace Process committed grave abuse of
discretion when he failed to carry out the pertinent consultation process, as mandated by E.O.
No. 3, Republic Act No. 7160, and Republic Act No. 8371. The furtive process by which the
MOA-AD was designed and crafted runs contrary to and in excess of the legal authority, and

250
amounts to a whimsical, capricious, oppressive, arbitrary and despotic exercise thereof. It
illustrates a gross evasion of positive duty and a virtual refusal to perform the duty enjoined.

The MOA-AD cannot be reconciled with the present Constitution and laws. Not only
its specific provisions but the very concept underlying them, namely, the associative relationship
envisioned between the GRP and the BJE, are unconstitutional, for the concept presupposes that
the associated entity is a state and implies that the same is on its way to independence.

While there is a clause in the MOA-AD stating that the provisions thereof inconsistent with
the present legal framework will not be effective until that framework is amended, the same
does not cure its defect. The inclusion of provisions in the MOA-AD establishing an associative
relationship between the BJE and the Central Government is, itself, a violation of the
Memorandum of Instructions From The President dated March 1, 2001, addressed to the
government peace panel. Moreover, as the clause is worded, it virtually guarantees that the
necessary amendments to the Constitution and the laws will eventually be put in place. Neither
the GRP Peace Panel nor the President herself is authorized to make such a
guarantee. Upholding such an act would amount to authorizing a usurpation of the constituent
powers vested only in Congress, a Constitutional Convention, or the people themselves through
the process of initiative, for the only way that the Executive can ensure the outcome of the
amendment process is through an undue influence or interference with that process.

While the MOA-AD would not amount to an international agreement or unilateral


declaration binding on the Philippines under international law, respondents’ act of guaranteeing
amendments is, by itself, already a constitutional violation that renders the MOA-AD fatally
defective.

WHEREFORE, respondents’ motion to dismiss is DENIED. The main and intervening


petitions are GIVEN DUE COURSE and hereby GRANTED.

The Memorandum of Agreement on the Ancestral Domain Aspect of the GRP-MILF Tripoli
Agreement on Peace of 2001 is declared CONTRARY TO LAW AND THE CONSTITUTION.

SO ORDERED.

CONCHITA CARPIO MORALES


Associate Justice

251
WE CONCUR:

REYNATO S. PUNO
Chief Justice

LEONARDO A. QUISUMBING CONSUELO YNARES- SANTIAGO


Associate Justice Associate Justice

ANTONIO T. CARPIO MA. ALICIA AUSTRIA-MARTINEZ


Associate Justice Associate Justice

RENATO C. CORONA ADOLFO S. AZCUNA


Associate Justice Associate Justice

DANTE O. TINGA MINITA V. CHICO-NAZARIO


Associate Justice Associate Justice

PRESBITERO J. VELASCO, JR. ANTONIO EDUARDO B. NACHURA


Associate Justice Associate Justice

252
TERESITA J. LEONARDO-DE CASTRO
RUBEN T. REYES Associate Justice
Associate Justice

ARTURO D. BRION
Associate Justice

CERTIFICATION

Pursuant to Article VIII, Section 13 of the Constitution, I certify that the conclusions in
the above Decision had been reached in consultation before the case was assigned to the writer
of the opinion of the Court.

REYNATO S. PUNO
Chief Justice

[1]
Eric Gutierrez and Abdulwahab Guialal, THE UNFINISHED JIHAD: THE MORO ISLAMIC
LIBERATION FRONT AND PEACE IN MINDANAO IN REBELS, WARLORDS AND ULAMA: A
READER ON MUSLIM SEPARATISM AND THE WAR IN SOUTHERN PHILIPPINES 275 (1999).
[2]
Memorandum of Respondents dated September 24, 2008, p. 10.
[3]
Memorandum of Respondents dated September 24, 2008, pp. 10-11.
[4]
Vide Salah Jubair, THE LONG ROAD TO PEACE: INSIDE THE GRP-MILF PEACE
PROCESS 35-36 (2007).
[5]
Memorandum of Respondents dated September 24, 2008, p. 12.
[6]
Vide Salah Jubair, THE LONG ROAD TO PEACE: INSIDE THE GRP-MILF PEACE
PROCESS 40-41 (2007).
[7]
Composed of its Chairperson, Sec. Rodolfo Garcia, and members, Atty. Leah Armamento,
Atty. Sedfrey Candelaria, with Mark Ryan Sullivan as Secretariat head.
[8]
Represented by Governor Jesus Sacdalan and/or Vice-Governor Emmanuel Piñol.
[9]
Rollo (G.R. No. 183591), pp. 3-33.
[10]
Supplement to Petition (with motion for leave) of August 11, 2008, rollo (G.R. No.
183591), pp. 143-162.
253
[11]
Rollo (G.R. No. 183752), pp. 3-28.
[12]
Represented by Mayor Celso L. Lobregat.
[13]
Rollo (G.R. No. 183591), pp. 132-135; rollo (G.R. No. 183752), pp. 68-71.
[14]
Rollo (G.R. No. 183591), pp. 130-131; rollo (G.R. No. 183752), pp. 66-67.
[15]
Rollo (G.R. No. 183752), pp. 173-246.
[16]
Represented by Mayor Lawrence Lluch Cruz.
[17]
Represented by Governor Rolando Yebes.
[18]
Namely, Seth Frederick Jaloslos, Fernando Cabigon, Jr., Uldarico Mejorada II, Edionar
Zamoras, Edgar Baguio, Cedric Adriatico, Felixberto Bolando, Joseph Brendo Ajero, Norbideiri
Edding, Anecito Darunday, Angelica Carreon, and Luzviminda Torrino.
[19]
Rollo (G.R. No. 183951), pp. 3-33.
[20]
Rollo (G.R. No. 183962), pp. 3- 20.
[21]
Represented by Mayor Cherrylyn Santos-Akbar.
[22]
Represented by Gov. Suharto Mangudadatu.
[23]
Represented by Mayor Noel Deano.
[24]
Rollo (G.R. No. 183591), pp. 451-453.
[25]
R.A. No. 6734, as amended by R.A. 9054 entitled AN ACT TO STRENGTHEN AND EXPAND
THE ORGANIC ACT FOR THE AUTONOMOUS REGION IN MUSLIM MINDANAO, AMENDING
FOR THE PURPOSE REPUBLIC ACT NO. 6734, ENTITLED AN ACT OF PROVIDING FOR THE
AUTONOMOUS REGION IN MUSLIM MINDANAO, AS AMENDED.
[26]
R.A. No. 8371, AN ACT TO RECOGNIZE, PROTECT AND PROMOTE THE RIGHTS OF
INDIGENOUS CULTURAL COMMUNITIES/INDIGENOUS PEOPLES, CREATING A NATIONAL
COMMISSION ON INDIGENOUS PEOPLES, ESTABLISHING IMPLEMENTING MECHANISMS,
APPROPRIATING FUNDS THEREFOR, AND FOR OTHER PURPOSES, October 29, 1997.
[27]
Cesar Adib Majul, THE GENERAL NATURE OF ISLAMIC LAW AND ITS APPLICATION IN THE
PHILIPPINES, lecture delivered as part of the Ricardo Paras Lectures, a series jointly
sponsored by the Commission on Bar Integration of the Supreme Court, the Integrated Bar of
the Philippines and the U.P. Law Center, September 24, 1977.
[28]
Ibid., vide M.A. Muqtedar Khan Ph.D., IMMIGRANT AMERICAN MUSLIMS AND THE MORAL
DILEMMAS OF CITIZENSHIP, http://www.islamfortoday.com/khan04.htm, visited on
September 18, 2008, and Syed Shahabuddin, MUSLIM WORLD AND THE CONTEMPORARY
IJMA' ON RULES OF GOVERNANCE - II, http://www.milligazette.com/Archives/2004/01-
15May04-Print-Edition/0105200471.htm, visited on September 18, 2008.
[29]
MOA-AD Terms of Reference.
[30]
MOA-AD, Concepts and Principles, par. 1.
[31]
A traditional Muslim historical account of the acts of Shariff Kabungsuwan is quoted by
historian Cesar Adib Majul in his book, MUSLIMS IN THE PHILIPPINES (1973):

After a time it came to pass that Mamalu, who was the chief man next to
Kabungsuwan, journeyed to Cotabato. He found there that many of the people had
ceased to regard the teachings of the Koran and had fallen into evil ways. Mamamlu
sent to Kabungsuwan word of these things.
Kabungsuwan with a portion of his warriors went from Malabang to Cotabato and
found that the word sent to him by Mamamlu was true. Then he assembled together
all the people. Those of them, who had done evilly and disregarded the teachings of
the Koran thenceforth, he drove out of the town into the hills, with their wives and
children.
Those wicked one who were thus cast out were the beginnings of the tribes of the
Tirurais and Manobos, who live to the east of Cotabato in the country into which their

254
evil forefathers were driven. And even to this day they worship not God; neither do
they obey the teachings of the Koran . . . But the people of Kabungsuwan, who
regarded the teachings of the Koran and lived in fear of God, prospered and increased,
and we Moros of today are their descendants. (Citation omitted, emphasis supplied).
[32]
Id., par. 2.
[33]
Id., par. 3.
[34]
Id., par. 4.
[35]
Francisco L. Gonzales, SULTANS OF A VIOLENT LAND, in Rebels, Warlords and Ulama: A
Reader on Muslim Separatism and the War in Southern Philippines 99, 103 (1999).
[36]
The Charter of the Assembly of First Nations, the leading advocacy group for the
indigenous peoples of Canada, adopted in 1985, begins thus:
“WE THE CHIEFS OF THE INDIAN FIRST NATIONS IN CANADA HAVING DECLARED:
THAT our peoples are the original peoples of this land having been put here by the Creator;
x x x.”
[37]
Id., par. 6.
[38]
MOA-AD, Territory, par. 1.
[39]
Id., par. 2(c).
[40]
Id., par. 2(d).
[41]
Id., par. 2(e).
[42]
Id., par. 2(f).
[43]
Id., par, 2(g)(1).
[44]
Id., par. 2(h).
[45]
Id., par. 2(i).
[46]
MOA-AD, Resources, par. 4.
[47]
Ibid.
[48]
Id., par. 5.
[49]
Id., par. 6.
[50]
Id., par. 7.
[51]
Id., par. 9.
[52]
MOA-AD, Governance, par. 3.
[53]
“IN WITNESS WHEREOF, the undersigned, being the representatives of the Parties[,]
hereby affix their signatures.”
[54]
Vide 1987 CONSTITUTION, Article VIII, Section 1.
[55]
Vide Muskrat v. US, 219 US 346 (1911).
[56]
Flast v. Cohen, 88 S.Ct. 1942, 1950 (1968).
[57]
Didipio Earth Savers’ Multi-Purpose Association, Incorporated (DESAMA) v. Gozun, G.R.
No. 157882, March 30, 2006, 485 SCRA 286.
[58]
Vide U.S. v. Muskrat, 219 U.S. 346, 357 (1902).
[59]
Guingona, Jr. v. Court of Appeals, 354 Phil. 415, 427-428 (1998).
[60]
Francisco, Jr. v. House of Representatives, 460 Phil. 830, 901-902 (2003) (citation
omitted).
[61]
Vide Warth v. Seldin, 422 US 490, 511 (1975).
[62]
Vide id. at 526.
[63]
Solicitor General’s Comment to G.R. No. 183752, pp. 9-11.
[64]
MOA-AD, pp. 3-7, 10.
[65]
391 Phil. 43 (2000).
[66]
Id. at 107-108.
[67]
530 US 290 (2000).

255
[68]
Id. at 292.
[69]
505 U.S. 144 (1992).
[70]
Id. at 175.
[71]
Although only one petition is denominated a petition for certiorari, most petitions pray that
the MOA-AD be declared unconstitutional/null and void.
[72]
Vide RULES OF COURT, Rule 65, Secs. 1 and 2.
[73]
Vide RULES OF COURT, Rule 65, Sec. 3.
[74]
Tañada v. Angara, 338 Phil. 546, 575 (1997).
[75]
Entitled DEFINING POLICY AND ADMINISTRATIVE STRUCTURE FOR GOVERNMENT’S
PEACE EFFORTS which reaffirms and reiterates Executive Order No. 125 of September 15,
1993.
[76]
E.O. No. 3, (2001), Sec. 1.
[77]
Vide Tañada v. Angara, supra note 74.
[78]
Baker v. Carr, 369 U.S. 186 (1962).
[79]
Vicente V. Mendoza , JUDICIAL REVIEW OF CONSTITUTIONAL QUESTIONS 137 (2004).
[80]
Francisco, Jr. v. The House of Representatives, 460 Phil. 830, 896 (2003).
[81]
David v. Macapagal-Arroyo, G.R. No. 171396, May 3, 2006, 489 SCRA 160, 223.
[82]
Kilosbayan, Inc. v. Morato, 320 Phil. 171 (1995).
[83]
Macasiano v. NHA, G.R. No. 107921, July 1, 1993, 224 SCRA 236.
[84]
Del Mar v. Phil. Amusement and Gaming Corp., 400 Phil. 307, 328-329 (2000) citing Phil.
Constitution Ass’n., Inc. v. Mathay, et al., 124 Phil. 890 (1966).
[85]
Vide NAACP v. Alabama, 357 U.S. 449 (1958).
[86]
Francisco, Jr. v. The House of Representatives, supra note 80.
[87]
Province of Batangas v. Romulo, G.R. No. 152774, May 27, 2004, 429 SCRA 736.
[88]
Firestone Ceramics, Inc. v. Court of Appeals, 372 Phil. 401 (1999) citing Gibson v. Judge
Revilla, 180 Phil. 645 (1979).
[89]
Supra note 81.
[90]
Integrated Bar of the Phils. v. Hon. Zamora, 392 Phil. 618 (2000).
[91]
Tatad v. Secretary of Energy, 346 Phil. 321 (1997).
[92]
Vide Compliance of September 1, 2008 of respondents.
[93]
Vide Manifestation of September 4, 2008 of respondents.
[94]
Supra note 81.
[95]
Id. citing Province of Batangas v. Romulo, supra note 87.
[96]
Id. citing Lacson v. Perez, 410 Phil. 78 (2001).
[97]
Id. citing Province of Batangas v. Romulo, supra note 87.
[98]
Id. citing Albaña v. Comelec, 478 Phil. 941 (2004); Chief Supt. Acop v. Guingona Jr., 433
Phil. 62 (2002); SANLAKAS v. Executive Secretary Reyes, 466 Phil. 482 (2004).
[99]
US v. W.T. Grant Co., 345 U.S. 629 (1953); US v. Trans-Missouri Freight Assn, 166 U.S.
290, 308-310 (1897); Walling v. Helmerich & Payne, Inc., 323 U.S. 37, 43 (1944); Gray v.
Sanders, 372 U.S. 368, 376 (1963); Defunis v. Odegaard, 416 U.S. 312 (1974).
[100]
Supra note 87.
[101]
G.R. No. 178920, October 15, 2007, 536 SCRA 290.
[102]
Chavez v. PCGG, 366 Phil. 863, 871 (1999).
[103]
G.R. No. 178830, July 14, 2008.
[104]
Supra note 98.
[105]
Ortega v. Quezon City Government, G.R. No. 161400, September 2, 2005, 469 SCRA 388.
[106]
Alunan III v. Mirasol, 342 Phil. 476 (1997); Viola v. Alunan III, 343 Phil. 184
(1997); Chief Superintendent Acop v. Guingona, Jr., supra note 98; Roble Arrastre, Inc. v.
Villaflor, G.R. No. 128509, August 22, 2006, 499 SCRA 434, 447.

256
[107]
CONSTITUTION, Article III, Sec. 7.
[108]
80 Phil. 383 (1948).
[109]
Legaspi v. Civil Service Commission, G.R. No. L-72119, May 29, 1987, 150 SCRA 530.
[110]
162 Phil. 868 (1976).
[111]
Baldoza v. Dimaano, supra at 876.
[112]
Legaspi v. Civil Service Commission, supra note 109.
[113]
Chavez v. PCGG, 360 Phil 133, 164 (1998).
[114]
In Legaspi v. Civil Service Commission, supra note 109 at 541, it was held that:
In determining whether or not a particular information is of public concern there is no
rigid test which can be applied. `Public concern' like `public interest' is a term that eludes
exact definition. Both terms embrace a broad spectrum of subjects which the public may
want to know, either because these directly affect their lives, or simply because such matters
naturally arouse the interest of an ordinary citizen. In the final analysis, it is for the courts to
determine on a case by case basis whether the matter at issue is of interest or importance,
as it relates to or affects the public.
[115]
Respondents’ Comment of August 4, 2008, p. 9.
[116]
Subido v. Ozaeta, supra note 108.
[117]
Tañada, et al. v. Hon. Tuvera, et al., 220 Phil. 422 (1985); Tañada, v. Hon. Tuvera, 230
Phil. 528 (1986).
[118]
Legaspi v. Civil Service Commission, supra note 109.
[119]
Valmonte v. Belmonte, Jr., G.R. No. 74930, February 13, 1989, 170 SCRA 256.
[120]
Chavez v. PCGG, supra note 113; Chavez v. PCGG, supra note 102.
[121]
Bantay Republic Act or BA-RA 7941 v. Commission on Elections, G.R. 177271, May 4,
2007, 523 SCRA 1.
[122]
Chavez v. Public Estates Authority, 433 Phil. 506, 532-533 (2002).
[123]
Vide V RECORD, CONSTITUTIONAL COMMISSION 26-28 (September 24, 1986) which is
replete with such descriptive phrase used by Commissioner Blas Ople.
[124]
CONSTITUTION, Article II, Sec. 28.
[125]
Bernas, Joaquin, THE 1987 CONSTITUTION OF THE REPUBLIC OF THE PHILIPPINES: A
COMMENTARY 100 (2003).
[126]
Vide Bernas, Joaquin, THE INTENT OF THE 1986 CONSTITUTION WRITERS 155 (1995).
[127]
Vide Chavez v. Public Estates Authority, supra note 122.
[128]
V RECORD, CONSTITUTIONAL COMMISSION 25 (September 24, 1986).
[129]
V RECORD, CONSTITUTIONAL COMMISSION 28-29 (September 24, 1986). The phrase
“safeguards on national interest” that may be provided by law was subsequently replaced by
“reasonable conditions,” as proposed by Commissioner Davide
[vide V RECORD, CONSTITUTIONAL COMMISSION 30 (September 24, 1986)].
[130]
In Chavez v. National Housing Authority, G.R. No. 164527, August 15, 2007, 530 SCRA
235, 331, the Court stated:
x x x The duty to disclose covers only transactions involving public interest, while the
duty to allow access has a broader scope of information which embraces not only
transactions involving public interest, but any matter contained in official communications
and public documents of the government agency. (Underscoring supplied)
[131]
Valmonte v. Belmonte, Jr., supra note 119.
[132]
V RECORD, CONSTITUTIONAL COMMISSION 28, 30 (September 24, 1986).
[133]
Supra note 55.
[134]
EXECUTIVE ORDER No. 3 (2001), Sec. 3 (a).
[135]
EXECUTIVE ORDER No. 3 (2001), Sec. 4 (b).
[136]
Respondents’ Memorandum of September 24, 2008, p. 44.

257
[137]
EXECUTIVE ORDER No. 3 (2001), Sec. 5 (b), par. 6.
[138]
EXECUTIVE ORDER No. 3 (2001), Sec. 8, see also Sec. 10.
[139]
Cf. Garcia v. Board of Investments, G.R. No. 88637, September 7, 1989, 177 SCRA 374,
382-384 where it was held that the Omnibus Investment Code of 1987 mandates the holding
of consultations with affected communities, whenever necessary, on the acceptability of
locating the registered enterprise within the community.
[140]
In their Memorandum, respondents made allegations purporting to show that
consultations were conducted on August 30, 2001 in Marawi City and Iligan City,
on September 20, 2001 in Midsayap, Cotabato, and on January 18-19, 2002 in Metro Manila.
(Memorandum of September 24, 2008, p. 13)
[141]
Cf. Chavez v. Public Estates Authority, supra note 120.
[142]
REPUBLIC ACT No. 7160, Sec. 2(c).
[143]
REPUBLIC ACT No. 7160, Sec. 27.
[144]
416 Phil. 438 (2001).
[145]
Id.; vide Alvarez v. PICOP Resources, Inc., G.R. No. 162243, November 29, 2006, 508
SCRA 498; Cf. Bangus Fry Fisherfolk v. Lanzanas, 453 Phil. 479 (2002).
[146]
Vide MOA-AD “Concepts and Principles,” pars. 2 & 7 in relation to “Resources,” par. 9
where vested property rights are made subject to the cancellation, modification and review
by the Bangsamoro Juridical Entity.
[147]
REPUBLIC ACT No. 8371 or “THE INDIGENOUS PEOPLES RIGHTS ACT OF 1997,” Sec. 16.
[148]
Id., Sec. 3 (g), Chapter VIII, inter alia.
[149]
Tañada v. Tuvera, No. L-63915, December 29, 1986, 146 SCRA 446, 456.
[150]
C.I. Keitner and W.M. Reisman, FREE ASSOCIATION: THE UNITED STATES
EXPERIENCE, 39 Tex. Int'l L.J. 1 (2003).
[151]
“The former Trust Territory of the Pacific Islands is made up of the Caroline Islands,
the Marshall Islands, and the Northern Mariana Islands, which extend east of
the Philippines and northeast of Indonesia in the North Pacific Ocean.” (Ibid.)
[152]
H. Hills, FREE ASSOCIATION FOR MICRONESIA AND THE MARSHALL ISLANDS: A
POLITICAL STATUS MODEL, 27 U. Haw. L. Rev. 1 (2004).
[153]
Henkin, et al., INTERNATIONAL LAW: CASES AND MATERIALS, 2nd ed., 274 (1987).
[154]
Convention on Rights and Duties of States, Dec. 26, 1933, 49 Stat. 3097, 165 L.N.T.S. 19.
[155]
G.R. No. 158088, July 6, 2005, 462 SCRA 622, 632.
[156]
AN ACT TO STRENGTHEN AND EXPAND THE ORGANIC ACT FOR THE AUTONOMOUS
REGION IN MUSLIM MINDANAO, AMENDING FOR THE PURPOSE REPUBLIC ACT NO. 6734,
ENTITLED ‘AN ACT PROVIDING FOR THE AUTONOMOUS REGION IN MUSLIM MINDANAO,’ AS
AMENDED, March 31, 2001.
[157]
AN ACT TO RECOGNIZE, PROTECT AND PROMOTE THE RIGHTS OF INDIGENOUS CULTURAL
COMMUNITIES/INDIGENOUS PEOPLES, CREATING A NATIONAL COMMISSION ON
INDIGENOUS PEOPLES, ESTABLISHING IMPLEMENTING MECHANISMS, APPROPRIATING
FUNDS THEREFOR, AND FOR OTHER PURPOSES, October 29, 1997.
[158]
90 Phil. 70, 73-74 (1951).
[159]
177 Phil. 160, 178-179 (1979).
[160]
2 S.C.R. 217 (1998).
[161]
999 U.N.T.S. 171 (March 23, 1976).
[162]
993 U.N.T.S. 3 (January 3, 1976).
[163]
League of Nations Official Journal, Special Supp. No. 3 (October 1920).
[164]
Lorie M. Graham, RESOLVING INDIGENOUS CLAIMS TO SELF-DETERMINATION, 10 ILSA J.
Int'l & Comp. L. 385 (2004). Vide S. James Anaya, SUPERPOWER ATTITUDES TOWARD
INDIGENOUS PEOPLES AND GROUP RIGHTS, 93 Am. Soc'y Int'l L. Proc. 251 (1999): “In

258
general, the term indigenous is used in association with groups that maintain a continuity of
cultural identity with historical communities that suffered some form of colonial invasion, and
that by virtue of that continuity of cultural identity continue to distinguish themselves from
others.”
[165]
Catherine J. Iorns, INDIGENOUS PEOPLES AND SELF DETERMINATION: CHALLENGING
STATE SOVEREIGNTY, 24 Case W. Res. J. Int'l L. 199 (1992).
[166]
Federico Lenzerini, “SOVEREIGNTY REVISITED: INTERNATIONAL LAW AND PARALLEL
SOVEREIGNTY OF INDIGENOUS PEOPLES,” 42 Tex. Int'l L.J. 155 (2006). Vide Christopher J.
Fromherz, INDIGENOUS PEOPLES' COURTS: EGALITARIAN JURIDICAL PLURALISM, SELF-
DETERMINATION, AND THE UNITED NATIONS DECLARATION ON
THE RIGHTS OF INDIGENOUS PEOPLES, 156 U. Pa. L. Rev. 1341 (2008): “While Australia
and the United States made much of the distinction between ‘self-government’ and ‘self-
determination’ on September 13, 2007, the U.S. statement to the UN on May 17, 2004,
seems to use these two concepts interchangeably. And, indeed, under the DRIP [Declaration
on the Rights of Indigenous Peoples], all three terms should be considered virtually
synonymous. Self-determination under the DRIP means ‘internal self-determination’ when
read in conjunction with Article 46, and ‘self-government,’ articulated in Article 4, is the core
of the ‘self-determination.’”
[167]
DEFINING THE APPROACH AND ADMINISTRATIVE STRUCTURE FOR GOVERNMENT’S
COMPREHENSIVE PEACE EFFORTS, September 15, 1993.
[168]
466 Phil. 482, 519-520 (2004).
[169]
CONSTITUTION, Article VII, Sec. 18.
[170]
Kirsti Samuels, POST-CONFLICT PEACE-BUILDING AND CONSTITUTION-MAKING, 6 Chi. J.
Int'l L. 663 (2006).
[171]
Christine Bell, PEACE AGREEMENTS: THEIR NATURE AND LEGAL STATUS, 100 Am. J. Int'l
L. 373 (2006).
[172]
CONSTITUTION, Article X, Sections 15-21.
[173]
III Record, Constitutional Commission, 180 (August 11, 1986).
[174]
165 Phil. 303 (1976).
[175]
Id. at 412.
[176]
Id. at 413.
[177]
G.R. No. 174153, October 25, 2006, 505 SCRA 160, 264-265.
[178]
CONSTITUTION, Art. VII, Sec. 5.
[179]
Article VI, Section 25 (1) of the Constitution states as follows: “The Congress may not
increase the appropriations recommended by the President for the operation of the
Government as specified in the budget. The form, content, and manner of preparation of the
budget shall be prescribed by law.”
[180]
Prosecutor v. Kallon and Kamara [Case No. SCSL-2004-15-AR72(E), SCSL-2004-16-
AR72(E), March 13, 2004].
[181]
1974 I.C.J. 253, 1974 WL 3 (I.C.J.).
[182]
M. Janis and J. Noyes, INTERNATIONAL LAW, CASES AND COMMENTARY, 3rd ed. 280
(2006).
[183]
1986 I.C.J. 554, 1986 WL 15621 (I.C.J.), December 22, 1986.
[184]
Planas v. COMELEC, 151 Phil. 217, 249 (1973).

259
EN BANC

AKBAYAN CITIZENS ACTION PARTY (“AKBAYAN”), PAMBANSANG


KATIPUNAN NG MGA SAMAHAN SA KANAYUNAN (“PKSK”), G.R. No. 170516
ALLIANCE OF PROGRESSIVE LABOR (“APL”), VICENTE A. FABE,
ANGELITO R. MENDOZA, MANUEL P. QUIAMBAO, ROSE BEATRIX Present:
CRUZ-ANGELES, CONG. LORENZO R. TANADA III, CONG. MARIO
JOYO AGUJA, CONG. LORETA ANN P. ROSALES, CONG. ANA
THERESIA HONTIVEROS-BARAQUEL, AND CONG. EMMANUEL PUNO, C.J.,
JOEL J. VILLANUEVA, QUISUMBING,
Petitioners, YNARES-SANTIAGO,
CARPIO,
- versus – AUSTRIA-MARTINEZ,
CORONA,
CARPIO MORALES,
THOMAS G. AQUINO, in his capacity as Undersecretary of the AZCUNA,
Department of Trade and Industry (DTI) and Chairman and Chief TINGA,
Delegate of the Philippine Coordinating Committee (PCC) for the CHICO-NAZARIO,
Japan-Philippines Economic Partnership Agreement, EDSEL T. VELASCO, JR.,
CUSTODIO, in his capacity as Undersecretary of the Department NACHURA,
of Foreign Affairs (DFA) and Co-Chair of the PCC for the JPEPA, REYES,
EDGARDO ABON, in his capacity as Chairman of the Tariff LEONARDO-DE CASTRO, &
Commission and lead negotiator for Competition Policy and BRION, JJ.
Emergency Measures of the JPEPA, MARGARITA SONGCO, in her
capacity as Assistant Director-General of the National Economic
Development Authority (NEDA) and lead negotiator for Trade in
Services and Cooperation of the JPEPA, MALOU MONTERO, in her
capacity as Foreign Service Officer I, Office of the Undersecretary
for International Economic Relations of the DFA and lead
negotiator for the General and Final Provisions of the JPEPA,
ERLINDA ARCELLANA, in her capacity as Director of the Board of
Investments and lead negotiator for Trade in Goods (General
Rules) of the JPEPA, RAQUEL ECHAGUE, in her capacity as lead
negotiator for Rules of Origin of the JPEPA, GALLANT SORIANO,
in his official capacity as Deputy Commissioner of the Bureau of Promulgated:
Customs and lead negotiator for Customs Procedures and
Paperless Trading of the JPEPA, MA. LUISA GIGETTE IMPERIAL,
in her capacity as Director of the Bureau of Local Employment of July 16, 2008
the Department of Labor and Employment (DOLE) and lead
negotiator for Movement of Natural Persons of the JPEPA,
PASCUAL DE GUZMAN, in his capacity as Director of the Board of
Investments and lead negotiator for Investment of the JPEPA,
JESUS MOTOOMULL, in his capacity as Director for the Bureau of
Product Standards of the DTI and lead negotiator for Mutual
Recognition of the JPEPA, LOUIE CALVARIO, in his capacity as
lead negotiator for Intellectual Property of the JPEPA, ELMER H.
DORADO, in his capacity as Officer-in-Charge of the Government
Procurement Policy Board Technical Support Office, the
government agency that is leading the negotiations on

260
Government Procurement of the JPEPA, RICARDO V. PARAS, in
his capacity as Chief State Counsel of the Department of Justice
(DOJ) and lead negotiator for Dispute Avoidance and Settlement
of the JPEPA, ADONIS SULIT, in his capacity as lead negotiator
for the General and Final Provisions of the JPEPA, EDUARDO
R. ERMITA, in his capacity as Executive Secretary, and ALBERTO
ROMULO, in his capacity as Secretary of the
*
DFA,

Respondents.

x - - - - - - - - - - - - - - - - - - - - - - - - - - - - - - - - - - - - - - - - - - - - - - - - - - -x

DECISION

CARPIO MORALES, J.:

Petitioners – non-government organizations, Congresspersons, citizens and taxpayers –


seek via the present petition for mandamus and prohibition to obtain from respondents the full
text of the Japan-Philippines Economic Partnership Agreement (JPEPA) including the Philippine
and Japanese offers submitted during the negotiation process and all pertinent attachments and
annexes thereto.

Petitioners Congressmen Lorenzo R. Tañada III and Mario Joyo Aguja filed on January 25,
2005 House Resolution No. 551 calling for an inquiry into the bilateral trade agreements then
being negotiated by the Philippine government, particularly the JPEPA. The Resolution became
the basis of an inquiry subsequently conducted by the House Special Committee on Globalization
(the House Committee) into the negotiations of the JPEPA.

In the course of its inquiry, the House Committee requested herein respondent
Undersecretary Tomas Aquino (Usec.Aquino), Chairman of the Philippine Coordinating

261
Committee created under Executive Order No. 213 (“CREATION OF A PHILIPPINE
COORDINATING COMMITTEE TO STUDY THE FEASIBILITY OF THE JAPAN-PHILIPPINES
ECONOMIC PARTNERSHIP AGREEMENT”)[1] to study and negotiate the proposed JPEPA, and to
furnish the Committee with a copy of the latest draft of the JPEPA. Usec. Aquino did not heed
the request, however.

Congressman Aguja later requested for the same document, but Usec. Aquino, by letter
of November 2, 2005, replied that the Congressman shall be provided with a copy thereof “once
the negotiations are completed and as soon as a thorough legal review of the proposed
agreement has been conducted.”

In a separate move, the House Committee, through Congressman Herminio G. Teves,


requested Executive Secretary Eduardo Ermita to furnish it with “all documents on the subject
including the latest draft of the proposed agreement, the requests and offers etc.”[2] Acting on
the request, Secretary Ermita, by letter of June 23, 2005, wrote Congressman Teves as follows:

In its letter dated 15 June 2005 (copy enclosed), [the] D[epartment of]
F[oreign] A[ffairs] explains that the Committee’s request to be furnished all
documents on the JPEPA may be difficult to accomplish at this time, since the
proposed Agreement has been a work in progress for about three years. A copy of
the draft JPEPA will however be forwarded to the Committee as soon as the text
thereof is settled and complete. (Emphasis supplied)

Congressman Aguja also requested NEDA Director-General Romulo Neri and Tariff
Commission Chairman EdgardoAbon, by letter of July 1, 2005, for copies of the latest text of the
JPEPA.

Chairman Abon replied, however, by letter of July 12, 2005 that the Tariff Commission
does not have a copy of the documents being requested, albeit he was certain
that Usec. Aquino would provide the Congressman with a copy “once the negotiation is
completed.” And by letter of July 18, 2005, NEDA Assistant Director-General Margarita
R. Songco informed the Congressman that his request addressed to Director-General Neri had
been forwarded to Usec. Aquino who would be “in the best position to respond” to the request.

In its third hearing conducted on August 31, 2005, the House Committee resolved to
issue a subpoena for the most recent draft of the JPEPA, but the same was not pursued because
by Committee Chairman Congressman Teves’ information, then House Speaker Jose

262
de Venecia had requested him to hold in abeyance the issuance of the subpoena until the
President gives her consent to the disclosure of the documents.[3]

Amid speculations that the JPEPA might be signed by the Philippine government within
December 2005, the present petition was filed on December 9, 2005.[4] The agreement was to
be later signed on September 9, 2006 by President GloriaMacapagal-Arroyo and Japanese Prime
Minister Junichiro Koizumi in Helsinki, Finland, following which the President endorsed it to the
Senate for its concurrence pursuant to Article VII, Section 21 of the Constitution. To date, the
JPEPA is still being deliberated upon by the Senate.

The JPEPA, which will be the first bilateral free trade agreement to be entered into by the
Philippines with another country in the event the Senate grants its consent to it, covers a broad
range of topics which respondents enumerate as follows: trade in goods, rules of origin, customs
procedures, paperless trading, trade in services, investment, intellectual property rights,
government procurement, movement of natural persons, cooperation, competition policy,
mutual recognition, dispute avoidance and settlement, improvement of the business
[5]
environment, and general and final provisions.

While the final text of the JPEPA has now been made accessible to the public
since September 11, 2006,[6] respondents do not dispute that, at the time the petition was filed
up to the filing of petitioners’ Reply – when the JPEPA was still being negotiated – the initial
drafts thereof were kept from public view.

Before delving on the substantive grounds relied upon by petitioners in support of the
petition, the Court finds it necessary to first resolve some material procedural issues.

Standing

For a petition for mandamus such as the one at bar to be given due course, it must be
instituted by a party aggrieved by the alleged inaction of any tribunal, corporation, board or
person which unlawfully excludes said party from the enjoyment of a legal right.[7] Respondents
deny that petitioners have such standing to sue. “[I]n the interest of a speedy and definitive
resolution of the substantive issues raised,” however, respondents consider it sufficient to cite a
portion of the ruling in Pimentel v. Office of Executive Secretary[8] which emphasizes the need
for a “personal stake in the outcome of the controversy” on questions of standing.

263
In a petition anchored upon the right of the people to information on matters of public
concern, which is a public right by its very nature, petitioners need not show that they have any
legal or special interest in the result, it being sufficient to show that they are citizens and,
therefore, part of the general public which possesses the right.[9] As the present petition is
anchored on the right to information and petitioners are all suing in their capacity as citizens and
groups of citizens including petitioners-members of the House of Representatives who
additionally are suing in their capacity as such, the standing of petitioners to file the present suit
is grounded in jurisprudence.

Mootness

Considering, however, that “[t]he principal relief petitioners are praying for is the
disclosure of the contents of the JPEPAprior to its finalization between the two States
parties,”[10] public disclosure of the text of the JPEPA after its signing by the President, during
the pendency of the present petition, has been largely rendered moot and academic.

With the Senate deliberations on the JPEPA still pending, the agreement as it now stands
cannot yet be considered as final and binding between the two States. Article 164 of the JPEPA
itself provides that the agreement does not take effect immediately upon the signing
thereof. For it must still go through the procedures required by the laws of each country for its
entry into force,viz:

Article 164
Entry into Force

This Agreement shall enter into force on the thirtieth day after the date on which
the Governments of the Parties exchange diplomatic notes informing each
other that their respective legal procedures necessary for entry into force of this
Agreement have been completed. It shall remain in force unless terminated as
provided for in Article 165.[11] (Emphasis supplied)

President Arroyo’s endorsement of the JPEPA to the Senate for concurrence is part of the
legal procedures which must be met prior to the agreement’s entry into force.

The text of the JPEPA having then been made accessible to the public, the petition has
become moot and academic to the extent that it seeks the disclosure of the “full text” thereof.

264
The petition is not entirely moot, however, because petitioners seek to obtain, not merely
the text of the JPEPA, but also the Philippine and Japanese offers in the course of the
negotiations.[12]

A discussion of the substantive issues, insofar as they impinge on petitioners’ demand for
access to the Philippine and Japanese offers, is thus in order.

Grounds relied upon by petitioners

Petitioners assert, first, that the refusal of the government to disclose the documents
bearing on the JPEPA negotiations violates their right to information
[13]
on matters of public concern and contravenes other constitutional provisions on
transparency, such as that on the policy of full public disclosure of all transactions involving
public interest.[14] Second, they contend that non-disclosure of the same documents undermines
their right to effective and reasonable participation in all levels of social, political, and economic
decision-making.[15] Lastly, they proffer that divulging the contents of the JPEPA only after the
agreement has been concluded will effectively make the Senate into a mere rubber stamp of the
Executive, in violation of the principle of separation of powers.

Significantly, the grounds relied upon by petitioners for the disclosure of the latest text of
the JPEPA are, except for the last, the same as those cited for the disclosure of the Philippine
and Japanese offers.

The first two grounds relied upon by petitioners which bear on the merits of respondents’
claim of privilege shall be discussed. The last, being purely speculatory given that the Senate is
still deliberating on the JPEPA, shall not.

The JPEPA is a matter of public concern

To be covered by the right to information, the information sought must meet the
threshold requirement that it be a matter of public concern. Apropos is the teaching
of Legaspi v. Civil Service Commission:

265
In determining whether or not a particular information is of public concern
there is no rigid test which can be applied. ‘Public concern’ like ‘public interest’ is a
term that eludes exact definition. Both terms embrace a broad spectrum of
subjects which the public may want to know, either because these directly affect
their lives, or simply because such matters naturally arouse the interest of an
ordinary citizen. In the final analysis, it is for the courts to determine on a case by
case basis whether the matter at issue is of interest or importance, as it relates to
or affects the public.[16] (Underscoring supplied)

From the nature of the JPEPA as an international trade agreement, it is evident that the
Philippine and Japanese offers submitted during the negotiations towards its execution are
matters of public concern. This, respondents do not dispute. They only claim that diplomatic
negotiations are covered by the doctrine of executive privilege, thus constituting an exception to
the right to information and the policy of full public disclosure.

Respondents’ claim of privilege

It is well-established in jurisprudence that neither the right to information nor the policy
of full public disclosure is absolute, there being matters which, albeit of public concern or public
interest, are recognized as privileged in nature. The types of information which may be
considered privileged have been elucidated in Almonte v. Vasquez,[17] Chavez v.
[18] [19] [20]
PCGG, Chavez v. Public Estate’s Authority, and most recently in Senate v. Ermita where
the Court reaffirmed the validity of the doctrine of executive privilege in this jurisdiction and
dwelt on its scope.

Whether a claim of executive privilege is valid depends on the ground invoked to justify it
and the context in which it is made.[21] In the present case, the ground for respondents’ claim
of privilege is set forth in their Comment, viz:

x x x The categories of information that may be considered privileged


includes matters of diplomatic character and under negotiation and review. In this
case, the privileged character of the diplomatic negotiations has been categorically
invoked and clearly explained by respondents particularly respondent DTI Senior
Undersecretary.

The documents on the proposed JPEPA as well as the text which is subject
to negotiations and legal review by the parties fall under the exceptions to the right
of access to information on matters of public concern and policy of public
disclosure. They come within the coverage of executive privilege. At the
time when the Committee was requesting for copies of such documents, the
negotiations were ongoing as they are still now and the text of the proposed JPEPA
266
is still uncertain and subject to change. Considering the status and nature of such
documents then and now, these are evidently covered by executive privilege
consistent with existing legal provisions and settled jurisprudence.

Practical and strategic considerations likewise counsel against the


disclosure of the “rolling texts” which may undergo radical change or portions of
which may be totally abandoned. Furthermore, the negotiations of the
representatives of the Philippines as well as of Japan must be allowed to explore
alternatives in the course of the negotiations in the same manner as judicial
deliberations and working drafts of opinions are accorded strict
confidentiality.[22] (Emphasis and underscoring supplied)

The ground relied upon by respondents is thus not simply that the information sought
involves a diplomatic matter, but that it pertains to diplomatic negotiations then in progress.

Privileged character of diplomatic negotiations

The privileged character of diplomatic negotiations has been recognized in this


jurisdiction. In discussing valid limitations on the right to information, the Court in Chavez v.
PCGG held that “information on inter-government exchanges prior to the conclusion of treaties
and executive agreements may be subject to reasonable safeguards for the sake of national
interest.”[23]Even earlier, the same privilege was upheld in People’s Movement for Press Freedom
(PMPF) v. Manglapus[24] wherein the Court discussed the reasons for the privilege in more
precise terms.

In PMPF v. Manglapus, the therein petitioners were seeking information from the
President’s representatives on the state of the then on-going negotiations of the RP-US Military
Bases Agreement.[25] The Court denied the petition, stressing that “secrecy of negotiations with
foreign countries is not violative of the constitutional provisions of freedom of speech or of the
press nor of the freedom of access to information.” The Resolution went on to state, thus:

The nature of diplomacy requires centralization of authority and expedition


of decision which are inherent in executive action. Another essential characteristic
of diplomacy is its confidential nature. Although much has been said about “open”
and “secret” diplomacy, with disparagement of the latter, Secretaries of State
Hughes and Stimson have clearly analyzed and justified the practice. In the words
of Mr. Stimson:

“A complicated negotiation . . . cannot be carried through


without many, many private talks and discussion, man to man; many
tentative suggestions and proposals. Delegates from other countries

267
come and tell you in confidence of their troubles at home and of their
differences with other countries and with other delegates; they tell you
of what they would do under certain circumstances and would not do
under other circumstances. . . If these reports . . . should become
public . . . who would ever trust American Delegations in another
conference? (United States Department of State, Press Releases, June
7, 1930, pp. 282-284.).”

xxxx

There is frequent criticism of the secrecy in which negotiation with foreign


powers on nearly all subjects is concerned. This, it is claimed, is incompatible with
the substance of democracy. As expressed by one writer, “It can be said that there
is no more rigid system of silence anywhere in the world.” (E.J. Young, Looking
Behind the Censorship, J. B. Lippincott Co., 1938) President Wilson in starting his
efforts for the conclusion of the World War declared that we must have “open
covenants, openly arrived at.” He quickly abandoned his thought.

No one who has studied the question believes that such a method of
publicity is possible. In the moment that negotiations are started, pressure groups
attempt to “muscle in.” An ill-timed speech by one of the parties or a frank
declaration of the concession which are exacted or offered on both sides would
quickly lead to widespread propaganda to block the negotiations. After a treaty has
been drafted and its terms are fully published, there is ample opportunity for
discussion before it is approved. (The New American Government and Its Works,
James T. Young, 4th Edition, p. 194) (Emphasis and underscoring supplied)

Still in PMPF v. Manglapus, the Court adopted the doctrine in U.S. v. Curtiss-Wright Export
[26]
Corp. that the President is the sole organ of the nation in its negotiations with foreign
countries, viz:

“x x x In this vast external realm, with its important, complicated, delicate


and manifold problems, the President alone has the power to speak or listen as a
representative of the nation. He makes treaties with the advice and consent of the
Senate; but he alone negotiates. Into the field of negotiation the Senate cannot
intrude; and Congress itself is powerless to invade it. As Marshall said in his great
argument of March 7, 1800, in the House of Representatives, “The President is the
sole organ of the nation in its external relations, and its sole representative with
foreign nations.” Annals, 6th Cong., col. 613. . . (Emphasis supplied; underscoring in
the original)

Applying the principles adopted in PMPF v. Manglapus, it is clear that while the final text
of the JPEPA may not be kept perpetually confidential – since there should be “ample
opportunity for discussion before [a treaty] is approved” – the offersexchanged by the parties
268
during the negotiations continue to be privileged even after the JPEPA is published. It is
reasonable to conclude that the Japanese representatives submitted their offers with the
understanding that “historic confidentiality”[27]would govern the same. Disclosing these offers
could impair the ability of the Philippines to deal not only with Japan but with other foreign
governments in future negotiations.

A ruling that Philippine offers in treaty negotiations should now be open to public scrutiny
would discourage future Philippine representatives from frankly expressing their views during
negotiations. While, on first impression, it appears wise to deter Philippine representatives from
entering into compromises, it bears noting that treaty negotiations, or any negotiation for that
matter, normally involve a process of quid pro quo, and oftentimes negotiators have to be willing
to grant concessions in an area of lesser importance in order to obtain more favorable terms in
an area of greater national interest. Aproposare the following observations of Benjamin S.
Duval, Jr.:

x x x [T]hose involved in the practice of negotiations appear to be in


agreement that publicity leads to “grandstanding,” tends to freeze negotiating
positions, and inhibits the give-and-take essential to successful
negotiation. As Sissela Bok points out, if “negotiators have more to gain from being
approved by their own sides than by making a reasoned agreement with
competitors or adversaries, then they are inclined to 'play to the gallery . . .'' In
fact, the public reaction may leave them little option. It would be a brave, or
foolish, Arab leader who expressed publicly a willingness for peace with Israel that
did not involve the return of the entire West Bank, or Israeli leader who stated
publicly a willingness to remove Israel's existing settlements from Judea and
Samaria in return for peace.[28] (Emphasis supplied)

Indeed, by hampering the ability of our representatives to compromise, we may be


jeopardizing higher national goals for the sake of securing less critical ones.
Diplomatic negotiations, therefore, are recognized as privileged in this jurisdiction, the
JPEPA negotiations constituting no exception. It bears emphasis, however, that such privilege is
only presumptive. For as Senate v. Ermita holds, recognizing a type of information as privileged
does not mean that it will be considered privileged in all instances. Only after a consideration of
the context in which the claim is made may it be determined if there is a public interest that
calls for the disclosure of the desired information, strong enough to overcome its traditionally
privileged status.

269
Whether petitioners have established the presence of such a public interest shall be
discussed later. For now, the Court shall first pass upon the arguments raised by petitioners
against the application of PMPF v. Manglapus to the present case.

Arguments proffered by petitioners against the application of PMPF v. Manglapus

Petitioners argue that PMPF v. Manglapus cannot be applied in toto to the present case,
there being substantial factual distinctions between the two.

To petitioners, the first and most fundamental distinction lies in the nature of the treaty
involved. They stress that PMPF v. Manglapus involved the Military Bases Agreement which
necessarily pertained to matters affecting national security; whereas the present case
involves an economic treaty that seeks to regulate trade and commerce between
the Philippines and Japan, matters which, unlike those covered by the Military Bases Agreement,
are not so vital to national security to disallow their disclosure.

Petitioners’ argument betrays a faulty assumption that information, to be considered


privileged, must involve national security. The recognition in Senate v. Ermita[29] that executive
privilege has encompassed claims of varying kinds, such that it may even be more accurate to
speak of “executive privileges,” cautions against such generalization.

While there certainly are privileges grounded on the necessity of safeguarding national
security such as those involving military secrets, not all are founded thereon. One example
is the “informer’s privilege,” or the privilege of the Government not to disclose the identity of a
person or persons who furnish information of violations of law to officers charged with the
enforcement of that law.[30] The suspect involved need not be so notorious as to be a threat to
national security for this privilege to apply in any given instance. Otherwise, the privilege would
be inapplicable in all but the most high-profile cases, in which case not only would this be
contrary to long-standing practice. It would also be highly prejudicial to law enforcement efforts
in general.

Also illustrative is the privilege accorded to presidential communications, which are


presumed privileged without distinguishing between those which involve matters of national
security and those which do not, the rationale for the privilege being that

270
x x x [a] frank exchange of exploratory ideas and assessments, free from
the glare of publicity and pressure by interested parties, is essential to protect the
independence of decision-making of those tasked to exercise Presidential,
Legislative and Judicial power. x x x[31] (Emphasis supplied)

In the same way that the privilege for judicial deliberations does not depend on the
nature of the case deliberated upon, so presidential communications are privileged whether they
involve matters of national security.
It bears emphasis, however, that the privilege accorded to presidential communications
is not absolute, one significant qualification being that “the Executive cannot, any more than the
other branches of government, invoke a general confidentiality privilege to shield its officials and
employees from investigations by the proper governmental institutions into possible criminal
wrongdoing.” [32] This qualification applies whether the privilege is being invoked in the context
of a judicial trial or a congressional investigation conducted in aid of legislation.[33]

Closely related to the “presidential communications” privilege is the deliberative process


privilege recognized in theUnited States. As discussed by the U.S. Supreme Court in NLRB v.
Sears, Roebuck & Co,[34] deliberative process coversdocuments reflecting advisory opinions,
recommendations and deliberations comprising part of a process by which governmental
decisions and policies are formulated. Notably, the privileged status of such documents
rests, not on the need to protect national security but, on the “obvious realization that officials
will not communicate candidly among themselves if each remark is a potential item of discovery
and front page news,” the objective of the privilege being to enhance the quality of agency
decisions. [35]

The diplomatic negotiations privilege bears a close resemblance to the deliberative


process and presidential communications privilege. It may be readily perceived that the
rationale for the confidential character of diplomatic negotiations, deliberative process, and
presidential communications is similar, if not identical.
The earlier discussion on PMPF v. Manglapus[36] shows that the privilege for diplomatic
negotiations is meant to encourage a frank exchange of exploratory ideas between the
negotiating parties by shielding such negotiations from public view. Similar to the privilege for
presidential communications, the diplomatic negotiations privilege seeks, through the same
means, to protect the independence in decision-making of the President, particularly in its
capacity as “the sole organ of the nation in its external relations, and its sole representative with
foreign nations.” And, as with the deliberative process privilege, the privilege accorded to
diplomatic negotiations arises, not on account of the content of the information per se, but
271
because the information is part of a process of deliberation which, in pursuit of the public
interest, must be presumed confidential.

The decision of the U.S. District Court, District of Columbia in Fulbright & Jaworski v.
Department of the Treasury[37]enlightens on the close relation between diplomatic negotiations
and deliberative process privileges. The plaintiffs in that case sought access to notes taken by a
member of the U.S. negotiating team during the U.S.-French tax treaty negotiations. Among the
points noted therein were the issues to be discussed, positions which the French and U.S. teams
took on some points, the draft language agreed on, and articles which needed to be
amended. Upholding the confidentiality of those notes, Judge Green ruled, thus:

Negotiations between two countries to draft a treaty represent a true


example of a deliberative process. Much give-and-take must occur for the countries
to reach an accord. A description of the negotiations at any one point would not
provide an onlooker a summary of the discussions which could later be relied on as
law. It would not be “working law” as the points discussed and positions agreed on
would be subject to change at any date until the treaty was signed by the
President and ratified by the Senate.

The policies behind the deliberative process privilege support non-


disclosure. Much harm could accrue to the negotiations process if these notes were
revealed. Exposure of the pre-agreement positions of the French negotiators might
well offend foreign governments and would lead to less candor by the U. S. in
recording the events of the negotiations process. As several months pass in
between negotiations, this lack of record could hinder readily the U. S. negotiating
team. Further disclosure would reveal prematurely adopted policies. If these
policies should be changed, public confusion would result easily.

Finally, releasing these snapshot views of the negotiations would be


comparable to releasing drafts of the treaty, particularly when the notes state the
tentative provisions and language agreed on. As drafts of regulations typically are
protected by the deliberative process privilege, Arthur Andersen & Co. v. Internal
Revenue Service, C.A. No. 80-705 (D.C.Cir., May 21, 1982), drafts of
treaties should be accorded the same protection. (Emphasis and underscoring
supplied)

Clearly, the privilege accorded to diplomatic negotiations follows as a logical consequence


from the privileged character of the deliberative process.

The Court is not unaware that in Center for International Environmental Law (CIEL), et al.
v. Office of U.S. Trade Representative[38] – where the plaintiffs sought information
272
relating to the just-completed negotiation of a United States-Chile Free Trade Agreement – the
same district court, this time under Judge Friedman, consciously refrained from applying the
doctrine in Fulbright and ordered the disclosure of the information being sought.

Since the factual milieu in CIEL seemed to call for the straight application of the doctrine
in Fulbright, a discussion of why the district court did not apply the same would help illumine
this Court’s own reasons for deciding the present case along the lines of Fulbright.

In both Fulbright and CIEL, the U.S. government cited a statutory basis for withholding
information, namely, Exemption 5 of the Freedom of Information Act (FOIA).[39] In order to
qualify for protection under Exemption 5, a document must satisfy two conditions: (1) it must
be either inter-agency or intra-agency in nature, and (2) it must be both pre-decisional and part
of the agency's deliberative or decision-making process.[40]

Judge Friedman, in CIEL, himself cognizant of a “superficial similarity of context” between


the two cases, based his decision on what he perceived to be a significant distinction: he found
the negotiator’s notes that were sought in Fulbright to be “clearly internal,” whereas the
documents being sought in CIEL were those produced by or exchanged with an outside
party, i.e. Chile. The documents subject of Fulbright being clearly internal in character, the
question of disclosure therein turned not on the threshold requirement of Exemption 5 that the
document be inter-agency, but on whether the documents were part of the agency's pre-
decisional deliberative process. On this basis, Judge Friedman found that “Judge Green's
discussion [in Fulbright] of the harm that could result from disclosure therefore is
irrelevant, since the documents at issue [in CIEL] are not inter-agency, and the Court does not
reach the question of deliberative process.” (Emphasis supplied)

In fine, Fulbright was not overturned. The court in CIEL merely found the same to be
irrelevant in light of its distinct factual setting. Whether this conclusion was valid – a question
on which this Court would not pass – the ruling in Fulbright that “[n]egotiations between two
countries to draft a treaty represent a true example of a deliberative process” was left standing,
since the CIEL court explicitly stated that it did not reach the question of deliberative process.

273
Going back to the present case, the Court recognizes that the information sought by
petitioners includes documents produced and communicated by a party external to the
Philippine government, namely, the Japanese representatives in the JPEPA negotiations, and to
that extent this case is closer to the factual circumstances of CIEL than those of Fulbright.

Nonetheless, for reasons which shall be discussed shortly, this Court echoes the principle
articulated in Fulbright that the public policy underlying the deliberative process privilege
requires that diplomatic negotiations should also be accorded privileged status, even if the
documents subject of the present case cannot be described as purely internal in character.

It need not be stressed that in CIEL, the court ordered the disclosure of information
based on its finding that the first requirement of FOIA Exemption 5 – that the documents be
inter-agency – was not met. In determining whether the government may validly refuse
disclosure of the exchanges between the U.S. and Chile, it necessarily had to deal with this
requirement, it being laid down by a statute binding on them.

In this jurisdiction, however, there is no counterpart of the FOIA, nor is there any
statutory requirement similar to FOIA Exemption 5 in particular. Hence, Philippine courts, when
assessing a claim of privilege for diplomatic negotiations, are more free to focus directly on the
issue of whether the privilege being claimed is indeed supported by public policy, without having
to consider – as the CIEL court did – if these negotiations fulfill a formal requirement of being
“inter-agency.” Important though that requirement may be in the context of domestic
negotiations, it need not be accorded the same significance when dealing with international
negotiations.

There being a public policy supporting a privilege for diplomatic negotiations for the
reasons explained above, the Court sees no reason to modify, much less abandon, the doctrine
in PMPF v. Manglapus.

A second point petitioners proffer in their attempt to differentiate PMPF


v. Manglapus from the present case is the fact that the petitioners therein consisted entirely of

274
members of the mass media, while petitioners in the present case include members of the House
of Representatives who invoke their right to information not just as citizens but as members of
Congress.

Petitioners thus conclude that the present case involves the right of members of Congress
to demand information on negotiations of international trade agreements from the Executive
branch, a matter which was not raised in PMPF v. Manglapus.

While indeed the petitioners in PMPF v. Manglapus consisted only of members of the mass
media, it would be incorrect to claim that the doctrine laid down therein has no bearing on a
controversy such as the present, where the demand for information has come from members of
Congress, not only from private citizens.

The privileged character accorded to diplomatic negotiations does not ipso facto lose all
force and effect simply because the same privilege is now being claimed under different
circumstances. The probability of the claim succeeding in the new context might differ, but to
say that the privilege, as such, has no validity at all in that context is another matter
altogether.

The Court’s statement in Senate v. Ermita that “presidential refusals to furnish


information may be actuated by any of at least three distinct kinds of considerations [state
secrets privilege, informer’s privilege, and a generic privilege for internal deliberations], and may
be asserted, with differing degrees of success, in the context of either judicial or legislative
investigations,”[41] implies that a privilege, once recognized, may be invoked under different
procedural settings. That this principle holds true particularly with respect to diplomatic
negotiations may be inferred from PMPF v. Manglapus itself, where the Court held that it is the
President alone who negotiates treaties, and not even the Senate or the House of
Representatives,unless asked, may intrude upon that process.

Clearly, the privilege for diplomatic negotiations may be invoked not only against citizens’
demands for information, but also in the context of legislative investigations.

275
Hence, the recognition granted in PMPF v. Manglapus to the privileged character of
diplomatic negotiations cannot be considered irrelevant in resolving the present case, the
contextual differences between the two cases notwithstanding.

As third and last point raised against the application of PMPF v. Manglapus in this case,
petitioners proffer that “the socio-political and historical contexts of the two cases are worlds
apart.” They claim that the constitutional traditions and concepts prevailing at the time PMPF
v. Manglapus came about, particularly the school of thought that the requirements of foreign
policy and the ideals of transparency were incompatible with each other or the “incompatibility
hypothesis,” while valid when international relations were still governed by power, politics and
wars, are no longer so in this age of international cooperation.[42]

Without delving into petitioners’ assertions respecting the “incompatibility hypothesis,”


the Court notes that the ruling inPMPF v. Manglapus is grounded more on the nature of treaty
negotiations as such than on a particular socio-political school of thought. If petitioners are
suggesting that the nature of treaty negotiations have so changed that “[a]n ill-timed speech by
one of the parties or a frank declaration of the concession which are exacted or offered on both
sides” no longer “lead[s] to widespread propaganda to block the negotiations,” or that parties in
treaty negotiations no longer expect their communications to be governed by historic
confidentiality, the burden is on them to substantiate the same. This petitioners failed to
discharge.

Whether the privilege applies only at certain stages of the negotiation process

Petitioners admit that “diplomatic negotiations on the JPEPA are entitled to a reasonable
amount of confidentiality so as not to jeopardize the diplomatic process.” They argue, however,
that the same is privileged “only at certain stages of the negotiating process, after which such
information must necessarily be revealed to the public.”[43] They add that the duty to disclose
this information was vested in the government when the negotiations moved from the
formulation and exploratory stage to the firming up of definite propositions or official
recommendations, citing Chavez v. PCGG[44] and Chavez v. PEA.[45]

The following statement in Chavez v. PEA, however, suffices to show that the doctrine in
both that case and Chavez v. PCGG with regard to the duty to disclose “definite propositions of
the government” does not apply to diplomatic negotiations:

276
We rule, therefore, that the constitutional right to information includes
official information on on-going negotiations before a final contract.The information,
however, must constitute definite propositions by the government and should not
cover recognized exceptions like privileged information, military and diplomatic
secrets and similar matters affecting national security and public
order. x xx[46] (Emphasis and underscoring supplied)

It follows from this ruling that even definite propositions of the government may not be
disclosed if they fall under “recognized exceptions.” The privilege for diplomatic negotiations is
clearly among the recognized exceptions, for the footnote to the immediately quoted ruling
cites PMPF v. Manglapus itself as an authority.

Whether there is sufficient public interest to overcome the claim of privilege

It being established that diplomatic negotiations enjoy a presumptive privilege against


disclosure, even against the demands of members of Congress for information, the Court shall
now determine whether petitioners have shown the existence of a public interest sufficient to
overcome the privilege in this instance.

To clarify, there are at least two kinds of public interest that must be taken into
account. One is the presumed public interest in favor of keeping the subject information
confidential, which is the reason for the privilege in the first place, and the other is the public
interest in favor of disclosure, the existence of which must be shown by the party asking for
information.[47]

The criteria to be employed in determining whether there is a sufficient public interest in


favor of disclosure may be gathered from cases such as U.S. v. Nixon,[48] Senate Select
Committee on Presidential Campaign Activities v. Nixon,[49] andIn re Sealed Case.[50]

U.S. v. Nixon, which involved a claim of the presidential communications privilege against
the subpoena duces tecum of a district court in a criminal case, emphasized the need to balance
such claim of privilege against the constitutional duty of courts to ensure a fair administration
of criminal justice.

x x x the allowance of the privilege to withhold evidence that


is demonstrably relevant in a criminal trial would cut deeply into the guarantee of
due process of law and gravely impair the basic function of the courts. A
President’s acknowledged need for confidentiality in the communications of his
277
office is general in nature, whereas the constitutional need for production of
relevant evidence in a criminal proceeding is specific and central to the fair
adjudication of a particular criminal case in the administration of justice. Without
access to specific facts a criminal prosecution may be totally frustrated. The
President’s broad interest in confidentiality of communications will not be vitiated
by disclosure of a limited number of conversations preliminarily shown to have
some bearing on the pending criminal cases. (Emphasis, italics and underscoring
supplied)

Similarly, Senate Select Committee v. Nixon,[51] which involved a claim of the presidential
communications privilege against the subpoena duces tecum of a Senate committee, spoke
of the need to balance such claim with the duty of Congress to perform its legislative functions.

The staged decisional structure established in Nixon v. Sirica was designed


to ensure that the President and those upon whom he directly relies in the
performance of his duties could continue to work under a general assurance that
their deliberations would remain confidential. So long as the presumption that the
public interest favors confidentiality can be defeated only by a strong showing of
need by another institution of government- a showing that the responsibilities of
that institution cannot responsibly be fulfilled without access to records of the
President's deliberations- we believed in Nixon v. Sirica, and continue to believe,
that the effective functioning of the presidential office will not be impaired. x x x

xxxx

The sufficiency of the Committee's showing of need has come to depend,


therefore, entirely on whether the subpoenaed materials are critical to the
performance of its legislative functions. x x x (Emphasis and underscoring supplied)

In re Sealed Case[52] involved a claim of the deliberative process and presidential


communications privileges against a subpoena duces tecum of a grand jury. On the claim of
deliberative process privilege, the court stated:

The deliberative process privilege is a qualified privilege and can be


overcome by a sufficient showing of need. This need determination is to be made
flexibly on a case-by-case, ad hoc basis. "[E]ach time [the deliberative process
privilege] is asserted the district court must undertake a fresh balancing of the
competing interests," taking into account factors such as "the relevance of the
evidence," "the availability of other evidence," "the seriousness of the litigation,"
"the role of the government," and the "possibility of future timidity by government
employees. x x x (Emphasis, italics and underscoring supplied)

278
Petitioners have failed to present the strong and “sufficient showing of need” referred to
in the immediately cited cases. The arguments they proffer to establish their entitlement to the
subject documents fall short of this standard.

Petitioners go on to assert that the non-involvement of the Filipino people in the JPEPA
negotiation process effectively results in the bargaining away of their economic and property
rights without their knowledge and participation, in violation of the due process clause of the
Constitution. They claim, moreover, that it is essential for the people to have access to the
initial offers exchanged during the negotiations since only through such disclosure can their
constitutional right to effectively participate in decision-making be brought to life in the context
of international trade agreements.

Whether it can accurately be said that the Filipino people were not involved in the JPEPA
negotiations is a question of fact which this Court need not resolve. Suffice it to state that
respondents had presented documents purporting to show that public consultations were
conducted on the JPEPA. Parenthetically, petitioners consider these “alleged consultations”
as “woefully selective and inadequate.”[53]

AT ALL EVENTS, since it is not disputed that the offers exchanged by the Philippine and
Japanese representatives have not been disclosed to the public, the Court shall pass upon the
issue of whether access to the documents bearing on them is, as petitioners claim, essential to
their right to participate in decision-making.

The case for petitioners has, of course, been immensely weakened by the disclosure of
the full text of the JPEPA to the public since September 11, 2006, even as it is still being
deliberated upon by the Senate and, therefore, not yet binding on the Philippines. Were the
Senate to concur with the validity of the JPEPA at this moment, there has already been, in the
words ofPMPF v. Manglapus, “ample opportunity for discussion before [the treaty] is approved.”

The text of the JPEPA having been published, petitioners have failed to convince this
Court that they will not be able to meaningfully exercise their right to participate in decision-
making unless the initial offers are also published.

It is of public knowledge that various non-government sectors and private citizens have
already publicly expressed their views on the JPEPA, their comments not being limited to general
observations thereon but on its specific provisions. Numerous articles and statements critical of

279
the JPEPA have been posted on the Internet.[54] Given these developments, there is no basis
for petitioners’ claim that access to the Philippine and Japanese offers is essential to the exercise
of their right to participate in decision-making.

Petitioner-members of the House of Representatives additionally anchor their claim to


have a right to the subject documents on the basis of Congress’ inherent power to regulate
commerce, be it domestic or international. They allege that Congress cannot meaningfully
exercise the power to regulate international trade agreements such as the JPEPA without being
given copies of the initial offers exchanged during the negotiations thereof. In the same vein,
they argue that the President cannot exclude Congress from the JPEPA negotiations since
whatever power and authority the President has to negotiate international trade agreements is
derived only by delegation of Congress, pursuant to Article VI, Section 28(2) of the Constitution
and Sections 401 and 402 of Presidential Decree No. 1464.[55]

The subject of Article VI Section 28(2) of the Constitution is not the power to negotiate
treaties and international agreements, but the power to fix tariff rates, import and export
quotas, and other taxes. Thus it provides:

(2) The Congress may, by law, authorize the President to fix within
specified limits, and subject to such limitations and restrictions as it may impose,
tariff rates, import and export quotas, tonnage and wharfage dues, and other duties
or imposts within the framework of the national development program of the
Government.

As to the power to negotiate treaties, the constitutional basis thereof is Section


21 of Article VII – the article on the Executive Department – which states:

No treaty or international agreement shall be valid and effective unless


concurred in by at least two-thirds of all the Members of the Senate.

The doctrine in PMPF v. Manglapus that the treaty-making power is exclusive to


the President, being the sole organ of the nation in its external relations, was echoed in BAYAN
v. Executive Secretary[56] where the Court held:

280
By constitutional fiat and by the intrinsic nature of his office, the President,
as head of State, is the sole organ and authority in the external affairs of the
country. In many ways, the President is the chief architect of the nation's foreign
policy; his "dominance in the field of foreign relations is (then) conceded." Wielding
vast powers and influence, his conduct in the external affairs of the nation,
as Jefferson describes, is “executive altogether.”

As regards the power to enter into treaties or international agreements, the


Constitution vests the same in the President,subject only to the concurrence of at
least two thirds vote of all the members of the Senate. In this light, the negotiation
of the VFA and the subsequent ratification of the agreement are exclusive acts
which pertain solely to the President, in the lawful exercise of his vast executive
and diplomatic powers granted him no less than by the fundamental law itself. Into
the field of negotiation the Senate cannot intrude, and Congress itself is powerless
to invade it. x x x (Italics in the original; emphasis and underscoring supplied)

The same doctrine was reiterated even more recently in Pimentel v. Executive
Secretary[57] where the Court ruled:

In our system of government, the President, being the head of state, is


regarded as the sole organ and authority in external relations and is the country's
sole representative with foreign nations. As the chief architect of foreign policy, the
President acts as the country's mouthpiece with respect to international affairs.
Hence, the President is vested with the authority to deal with foreign states and
governments, extend or withhold recognition, maintain diplomatic relations, enter
into treaties, and otherwise transact the business of foreign relations. In the realm
of treaty-making, the President has the sole authority to negotiate with other
states.

Nonetheless, while the President has the sole authority to negotiate and
enter into treaties, the Constitution provides a limitation to his power by requiring
the concurrence of 2/3 of all the members of the Senate for the validity of the
treaty entered into by him. x x x (Emphasis and underscoring supplied)

While the power then to fix tariff rates and other taxes clearly belongs to Congress, and is
exercised by the President only by delegation of that body, it has long been recognized that the
power to enter into treaties is vested directly and exclusively in the President, subject only to
the concurrence of at least two-thirds of all the Members of the Senate for the validity of the
treaty. In this light, the authority of the President to enter into trade agreements with foreign
nations provided under P.D. 1464[58] may be interpreted as an acknowledgment of a power
already inherent in its office. It may not be used as basis to hold the President or its
representatives accountable to Congress for the conduct of treaty negotiations.

281
This is not to say, of course, that the President’s power to enter into treaties is unlimited
but for the requirement of Senate concurrence, since the President must still ensure that all
treaties will substantively conform to all the relevant provisions of the Constitution.

It follows from the above discussion that Congress, while possessing vast legislative
powers, may not interfere in the field of treaty negotiations. While Article VII, Section 21
provides for Senate concurrence, such pertains only to the validity of the treaty under
consideration, not to the conduct of negotiations attendant to its conclusion. Moreover, it is not
even Congress as a whole that has been given the authority to concur as a means of checking
the treaty-making power of the President, but only the Senate.

Thus, as in the case of petitioners suing in their capacity as private citizens, petitioners-
members of the House of Representatives fail to present a “sufficient showing of need” that the
information sought is critical to the performance of the functions of Congress, functions that do
not include treaty-negotiation.

Respondents’ alleged failure to timely claim executive privilege

On respondents’ invocation of executive privilege, petitioners find the same


defective, not having been done seasonably as it was raised only in their Comment to the
present petition and not during the House Committee hearings.
That respondents invoked the privilege for the first time only in their Comment to the
present petition does not mean that the claim of privilege should not be credited. Petitioners’
position presupposes that an assertion of the privilege should have been made during the House
Committee investigations, failing which respondents are deemed to have waived it.

When the House Committee and petitioner-Congressman Aguja requested respondents


for copies of the documents subject of this case, respondents replied that the negotiations were
still on-going and that the draft of the JPEPA would be released once the text thereof is settled
and complete. There was no intimation that the requested copies are confidential in nature by
reason of public policy. The response may not thus be deemed a claim of privilege by the
standards of Senate v. Ermita, which recognizes as claims of privilege only those which are
accompanied by precise and certain reasons for preserving theconfidentiality of the information
being sought.

282
Respondents’ failure to claim the privilege during the House Committee hearings may not,
however, be construed as a waiver thereof by the Executive branch. As the immediately
preceding paragraph indicates, what respondents received from the House Committee and
petitioner-Congressman Aguja were mere requests for information. And as priorly stated, the
House Committee itself refrained from pursuing its earlier resolution to issue a
subpoena duces tecum on account of then Speaker Jose de Venecia’s alleged request to
Committee Chairperson Congressman Teves to hold the same in abeyance.

While it is a salutary and noble practice for Congress to refrain from issuing subpoenas to
executive officials – out of respect for their office – until resort to it becomes necessary, the fact
remains that such requests are not a compulsory process. Being mere requests, they do not
strictly call for an assertion of executive privilege.

The privilege is an exemption to Congress’ power of inquiry.[59] So long as Congress itself


finds no cause to enforce such power, there is no strict necessity to assert the privilege. In this
light, respondents’ failure to invoke the privilege during the House Committee investigations did
not amount to a waiver thereof.

The Court observes, however, that the claim of privilege appearing in respondents’
Comment to this petition fails to satisfy in full the requirement laid down in Senate
v. Ermita that the claim should be invoked by the President or through the Executive Secretary
“by order of the President.”[60] Respondents’ claim of privilege is being sustained, however, its
flaw notwithstanding, because of circumstances peculiar to the case.

The assertion of executive privilege by the Executive Secretary, who is one of the
respondents herein, without him adding the phrase “by order of the President,” shall be
considered as partially complying with the requirement laid down in Senate v.Ermita. The
requirement that the phrase “by order of the President” should accompany the Executive
Secretary’s claim of privilege is a new rule laid down for the first time in Senate v. Ermita, which
was not yet final and executory at the time respondents filed their Comment to the
petition.[61] A strict application of this requirement would thus be unwarranted in this case.

Response to the Dissenting Opinion of the Chief Justice

283
We are aware that behind the dissent of the Chief Justice lies a genuine zeal to protect
our people’s right to information against any abuse of executive privilege. It is a zeal that We
fully share.

The Court, however, in its endeavor to guard against the abuse of executive privilege,
should be careful not to veer towards the opposite extreme, to the point that it would strike
down as invalid even a legitimate exercise thereof.

We respond only to the salient arguments of the Dissenting Opinion which have not yet
been sufficiently addressed above.

1. After its historical discussion on the allocation of power over international trade
agreements in the United States, the dissent concludes that “it will be turning somersaults with
history to contend that the President is the sole organ for external relations” in that
jurisdiction. With regard to this opinion, We make only the following observations:

There is, at least, a core meaning of the phrase “sole organ of the nation in its external
relations” which is not being disputed, namely, that the power to directly negotiate treaties and
international agreements is vested by our Constitution only in the Executive. Thus, the dissent
states that “Congress has the power to regulate commerce with foreign nations but
does nothave the power to negotiate international agreements directly.”[62]

What is disputed is how this principle applies to the case at bar.

The dissent opines that petitioner-members of the House of Representatives, by asking


for the subject JPEPA documents, are not seeking to directly participate in the negotiations of
the JPEPA, hence, they cannot be prevented from gaining access to these documents.

On the other hand, We hold that this is one occasion where the following ruling in Agan v.
PIATCO[63] – and in other cases both before and since – should be applied:

This Court has long and consistently adhered to the legal maxim that those
that cannot be done directly cannot be done indirectly. To declare the PIATCO
contracts valid despite the clear statutory prohibition against a direct government
guaranteewould not only make a mockery of what the BOT Law seeks to prevent --
which is to expose the government to the risk of incurring a monetary obligation
resulting from a contract of loan between the project proponent and its lenders and
to which the Government is not a party to -- but would also render the BOT Law

284
useless for what it seeks to achieve –- to make use of the resources of the private
sector in the “financing, operation and maintenance of infrastructure and
development projects” which are necessary for national growth and development
but which the government, unfortunately, could ill-afford to finance at this point in
time.[64]

Similarly, while herein petitioners-members of the House of Representatives may not


have been aiming to participate in the negotiations directly, opening the JPEPA negotiations to
their scrutiny – even to the point of giving them access to the offers exchanged between the
Japanese and Philippine delegations – would have made a mockery of what the Constitution
sought to prevent and rendered it useless for what it sought to achieve when it vested the power
of direct negotiation solely with the President.

What the U.S. Constitution sought to prevent and aimed to achieve in defining the
treaty-making power of the President, which our Constitution similarly defines, may be gathered
from Hamilton’s explanation of why the U.S. Constitution excludes the House of Representatives
from the treaty-making process:

x x x The fluctuating, and taking its future increase into account, the
multitudinous composition of that body, forbid us to expect in it those qualities
which are essential to the proper execution of such a trust. Accurate and
comprehensive knowledge of foreign politics; a steady and systematic adherence to
the same views; a nice and uniform sensibility to national character,
decision, secrecy and dispatch; are incompatible with a body so variable and so
numerous. The very complication of the business by introducing a necessity of the
concurrence of so many different bodies, would of itself afford a solid
objection. The greater frequency of the calls upon the house of representatives,
and the greater length of time which it would often be necessary to keep them
together when convened, to obtain their sanction in the progressive stages of a
treaty, would be source of so great inconvenience and expense, as alone ought to
condemn the project.[65]

These considerations a fortiori apply in this jurisdiction, since the Philippine


Constitution, unlike that of the U.S., does not even grant the Senate the power to advise the
Executive in the making of treaties, but only vests in that body the power to concur in the
validity of the treaty after negotiations have been concluded.[66] Much less, therefore, should it
be inferred that the House of Representatives has this power.

Since allowing petitioner-members of the House of Representatives access to the subject


JPEPA documents would set a precedent for future negotiations, leading to the contravention of
285
the public interests articulated above which the Constitution sought to protect, the subject
documents should not be disclosed.

2. The dissent also asserts that respondents can no longer claim the diplomatic secrets
privilege over the subject JPEPA documents now that negotiations have been concluded, since
their reasons for nondisclosure cited in the June 23, 2005 letter of Sec. Ermita, and later in their
Comment, necessarily apply only for as long as the negotiations were still pending;

In their Comment, respondents contend that “the negotiations of the representatives of


the Philippines as well as of Japanmust be allowed to explore alternatives in the course of the
negotiations in the same manner as judicial deliberations and working drafts of opinions are
accorded strict confidentiality.” That respondents liken the documents involved in the JPEPA
negotiations to judicial deliberations and working drafts of opinions evinces, by itself, that they
were claiming confidentiality not only until, but even after, the conclusion of the negotiations.

Judicial deliberations do not lose their confidential character once a decision has been
promulgated by the courts. The same holds true with respect to working drafts of opinions,
which are comparable to intra-agency recommendations. Such intra-agency recommendations
are privileged even after the position under consideration by the agency has developed into a
definite proposition, hence, the rule in this jurisdiction that agencies have the duty to
disclose only definite propositions, and not the inter-agency and intra-agency communications
during the stage when common assertions are still being formulated.[67]

3. The dissent claims that petitioner-members of the House of Representatives have


sufficiently shown their need for the same documents to overcome the privilege. Again, We
disagree.

The House Committee that initiated the investigations on the JPEPA did not pursue its
earlier intention to subpoena the documents. This strongly undermines the assertion that access
to the same documents by the House Committee is critical to the performance of its legislative
functions. If the documents were indeed critical, the House Committee should have, at the very
least, issued a subpoena duces tecum or, like what the Senate did in Senate v. Ermita, filed the
present petition as a legislative body, rather than leaving it to the discretion of individual
Congressmen whether to pursue an action or not. Such acts would have served as strong indicia
that Congress itself finds the subject information to be critical to its legislative functions.

286
Further, given that respondents have claimed executive privilege, petitioner-members of
the House of Representatives should have, at least, shown how its lack of access to the
Philippine and Japanese offers would hinder the intelligent crafting of legislation. Mere assertion
that the JPEPA covers a subject matter over which Congress has the power to legislate would not
suffice. As Senate Select Committee v. Nixon[68] held, the showing required to overcome the
presumption favoring confidentiality turns, not only on the nature and appropriateness of the
function in the performance of which the material was sought, but also the degree to which the
material was necessary to its fulfillment. This petitioners failed to do.

Furthermore, from the time the final text of the JPEPA including its annexes and
attachments was published, petitioner-members of the House of Representatives have been free
to use it for any legislative purpose they may see fit. Since such publication, petitioners’ need, if
any, specifically for the Philippine and Japanese offers leading to the final version of the JPEPA,
has become even less apparent.

In asserting that the balance in this instance tilts in favor of disclosing the JPEPA
documents, the dissent contends that the Executive has failed to show how disclosing
them after the conclusion of negotiations would impair the performance of its functions. The
contention, with due respect, misplaces the onus probandi. While, in keeping with the general
presumption of transparency, the burden is initially on the Executive to provide precise and
certain reasons for upholding its claim of privilege, once the Executive is able to show that the
documents being sought are covered by a recognized privilege, the burden shifts to the party
seeking information to overcome the privilege by a strong showing of need.

When it was thus established that the JPEPA documents are covered by the privilege for
diplomatic negotiations pursuant to PMPF v. Manglapus, the presumption arose that their
disclosure would impair the performance of executive functions. It was then incumbent on
petitioner- requesting parties to show that they have a strong need for the information sufficient
to overcome the privilege. They have not, however.

4. Respecting the failure of the Executive Secretary to explicitly state that he is claiming
the privilege “by order of the President,” the same may not be strictly applied to the privilege
claim subject of this case.

When the Court in Senate v. Ermita limited the power of invoking the privilege to the
President alone, it was laying down a new rule for which there is no counterpart even in

287
the United States from which the concept of executive privilege was adopted. As held in the
2004 case of Judicial Watch, Inc. v. Department of Justice,[69] citing In re Sealed Case,[70] “the
issue of whether a President must personally invoke the [presidential communications] privilege
remains an open question.” U.S. v. Reynolds,[71] on the other hand, held that “[t]here must be a
formal claim of privilege, lodged by the head of the department which has control over the
matter, after actual personal consideration by that officer.”

The rule was thus laid down by this Court, not in adherence to any established precedent,
but with the aim of preventing the abuse of the privilege in light of its highly exceptional
nature. The Court’s recognition that the Executive Secretary also bears the power to invoke the
privilege, provided he does so “by order of the President,” is meant to avoid laying down too
rigid a rule, the Court being aware that it was laying down a new restriction on executive
privilege. It is with the same spirit that the Court should not be overly strict with applying the
same rule in this peculiar instance, where the claim of executive privilege occurred before the
judgment in Senate v. Ermita became final.

5. To show that PMPF v. Manglapus may not be applied in the present case, the dissent
implies that the Court therein erred in citing US v. Curtiss Wright[72] and the book entitled The
New American Government and Its Work[73] since these authorities, so the dissent claims, may
not be used to calibrate the importance of the right to information in the Philippine setting.

The dissent argues that since Curtiss-Wright referred to a conflict between the executive
and legislative branches of government, the factual setting thereof was different from that
of PMPF v. Manglapus which involved a collision between governmental power over the conduct
of foreign affairs and the citizen’s right to information.

That the Court could freely cite Curtiss-Wright – a case that upholds the secrecy of
diplomatic negotiations againstcongressional demands for information – in the course of laying
down a ruling on the public right to information only serves to underscore the principle
mentioned earlier that the privileged character accorded to diplomatic negotiations does not ipso
factolose all force and effect simply because the same privilege is now being claimed under
different circumstances.

PMPF v. Manglapus indeed involved a demand for information from private citizens and
not an executive-legislative conflict, but so did Chavez v. PEA[74] which held that “the
[public’s] right to information . . . does not extend to matters recognized as privileged

288
information under the separation of powers.” What counts as privileged information in an
executive-legislative conflict is thus also recognized as such in cases involving the public’s right
to information.

Chavez v. PCGG[75] also involved the public’s right to information, yet the Court
recognized as a valid limitation to that right the same privileged information based on separation
of powers – closed-door Cabinet meetings, executive sessions of either house of Congress, and
the internal deliberations of the Supreme Court.

These cases show that the Court has always regarded claims of privilege, whether in the
context of an executive-legislative conflict or a citizen’s demand for information, as closely
intertwined, such that the principles applicable to one are also applicable to the other.

The reason is obvious. If the validity of claims of privilege were to be assessed by


entirely different criteria in each context, this may give rise to the absurd
result where Congress would be denied access to a particular information because of a claim of
executive privilege, but the general public would have access to the same information, the claim
of privilege notwithstanding.

Absurdity would be the ultimate result if, for instance, the Court adopts the “clear and
present danger” test for the assessment of claims of privilege against citizens’ demands for
information. If executive information, when demanded by a citizen, is privileged only when
there is a clear and present danger of a substantive evil that the State has a right to prevent, it
would be very difficult for the Executive to establish the validity of its claim in each instance. In
contrast, if the demand comes from Congress, the Executive merely has to show that the
information is covered by a recognized privilege in order to shift the burden on Congress to
present a strong showing of need. This would lead to a situation where it would be more difficult
for Congress to access executive information than it would be for private citizens.

We maintain then that when the Executive has already shown that an information is
covered by executive privilege, the party demanding the information must present a “strong
showing of need,” whether that party is Congress or a private citizen.

The rule that the same “showing of need” test applies in both these contexts, however,
should not be construed as a denial of the importance of analyzing the context in which an
executive privilege controversy may happen to be placed. Rather, it affirms it, for it means that

289
the specific need being shown by the party seeking information in every particular instance is
highly significant in determining whether to uphold a claim of privilege. This “need” is, precisely,
part of the context in light of which every claim of privilege should be assessed.

Since, as demonstrated above, there are common principles that should be applied to
executive privilege controversies across different contexts, the Court in PMPF v. Manglapus did
not err when it cited the Curtiss-Wright case.

The claim that the book cited in PMPF v. Manglapus entitled The New American
Government and Its Work could not have taken into account the expanded statutory right to
information in the FOIA assumes that the observations in that book in support of the
confidentiality of treaty negotiations would be different had it been written after the FOIA. Such
assumption is, with due respect, at best, speculative.

As to the claim in the dissent that “[i]t is more doubtful if the same book be used to
calibrate the importance of the right of access to information in the Philippine setting considering
its elevation as a constitutional right,” we submit that the elevation of such right as a
constitutional right did not set it free from the legitimate restrictions of executive privilege which
is itselfconstitutionally-based.[76] Hence, the comments in that book which were cited in PMPF
v. Manglapus remain valid doctrine.

6. The dissent further asserts that the Court has never used “need” as a test to uphold or
allow inroads into rights guaranteed under the Constitution. With due respect, we assert
otherwise. The Court has done so before, albeit without using the term “need.”

In executive privilege controversies, the requirement that parties present a “sufficient


showing of need” only means, in substance, that they should show a public interest in favor of
disclosure sufficient in degree to overcome the claim of privilege.[77] Verily, the Court in such
cases engages in a balancing of interests. Such a balancing of interests is certainly not new in
constitutional adjudication involving fundamental rights. Secretary of Justice
[78]
v. Lantion, which was cited in the dissent, applied just such a test.

Given that the dissent has clarified that it does not seek to apply the “clear and present
danger” test to the present controversy, but the balancing test, there seems to be no substantial
dispute between the position laid down in this ponencia and that reflected in the dissent as to

290
what test to apply. It would appear that the only disagreement is on the results of applying that
test in this instance.

The dissent, nonetheless, maintains that “it suffices that information is of public concern
for it to be covered by the right, regardless of the public’s need for the information,” and that
the same would hold true even “if they simply want to know it because it interests them.” As
has been stated earlier, however, there is no dispute that the information subject of this case is
a matter of public concern. The Court has earlier concluded that it is a matter of public
concern, not on the basis of any specific need shown by petitioners, but from the very nature of
the JPEPA as an international trade agreement.

However, when the Executive has – as in this case – invoked the privilege, and it has
been established that the subject information is indeed covered by the privilege being claimed,
can a party overcome the same by merely asserting that the information being demanded is a
matter of public concern, without any further showing required? Certainly not, for that would
render the doctrine of executive privilege of no force and effect whatsoever as a limitation on the
right to information, because then the sole test in such controversies would be whether an
information is a matter of public concern.

Moreover, in view of the earlier discussions, we must bear in mind that, by disclosing the
documents of the JPEPA negotiations, the Philippine government runs the grave risk of betraying
the trust reposed in it by the Japanese representatives, indeed, by the Japanese government
itself. How would the Philippine government then explain itself when that happens? Surely, it
cannot bear to say that it just had to release the information because certain persons simply
wanted to know it “because it interests them.”

Thus, the Court holds that, in determining whether an information is covered by the right
to information, a specific “showing of need” for such information is not a relevant consideration,
but only whether the same is a matter of public concern. When, however, the government has
claimed executive privilege, and it has established that the information is indeed covered by the
same, then the party demanding it, if it is to overcome the privilege, must show that that the
information is vital, not simply for the satisfaction of its curiosity, but for its ability to effectively
and reasonably participate in social, political, and economic decision-making.[79]

7. The dissent maintains that “[t]he treaty has thus entered the ultimate stage where the
people can exercise their right to participate in the discussion whether the Senate should concur

291
in its ratification or not.” (Emphasis supplied) It adds that this right “will be diluted unless the
people can have access to the subject JPEPA documents”. What, to the dissent, is a dilution of
the right to participate in decision-making is, to Us, simply a recognition of the qualified nature
of the public’s right to information. It is beyond dispute that the right to information is not
absolute and that the doctrine of executive privilege is a recognized limitation on that right.

Moreover, contrary to the submission that the right to participate in decision-making


would be diluted, We reiterate that our people have been exercising their right to participate in
the discussion on the issue of the JPEPA, and they have been able to articulate their different
opinions without need of access to the JPEPA negotiation documents.

Thus, we hold that the balance in this case tilts in favor of executive privilege.

8. Against our ruling that the principles applied in U.S. v. Nixon, the Senate Select
Committee case, and In re Sealed Case, are similarly applicable to the present controversy, the
dissent cites the caveat in the Nixon case that the U.S. Court was there addressing only the
President’s assertion of privilege in the context of a criminal trial, not a civil litigation nor a
congressional demand for information. What this caveat means, however, is only that courts
must be careful not to hastily apply the ruling therein to other contexts. It does not, however,
absolutely mean that the principles applied in that case may never be applied in such contexts.

Hence, U.S. courts have cited U.S. v. Nixon in support of their rulings on claims of
executive privilege in contexts other than a criminal trial, as in the case of Nixon v.
Administrator of General Services[80] – which involved former President Nixon’s invocation of
executive privilege to challenge the constitutionality of the “Presidential Recordings and
Materials Preservation Act”[81] – and the above-mentioned In re Sealed Case which involved a
claim of privilege against a subpoena duces tecumissued in a grand jury investigation.

Indeed, in applying to the present case the principles found in U.S. v. Nixon and in the
other cases already mentioned, We are merely affirming what the Chief Justice stated in his
Dissenting Opinion in Neri v. Senate Committee on Accountability[82] – a case involving an
executive-legislative conflict over executive privilege. That dissenting opinion stated that,
while Nixon was not concerned with the balance between the President’s generalized interest in
confidentiality and congressional demands for information, “[n]onetheless the [U.S.] Court laid
down principles and procedures that can serve as torch lights to illumine us on the scope and
use of Presidential communication privilege in the case at bar.”[83] While the Court was divided

292
in Neri, this opinion of the Chief Justice was not among the points of disagreement, and We
similarly hold now that theNixon case is a useful guide in the proper resolution of the present
controversy, notwithstanding the difference in context.

Verily, while the Court should guard against the abuse of executive privilege, it should
also give full recognition to the validity of the privilege whenever it is claimed within the proper
bounds of executive power, as in this case. Otherwise, the Court would undermine its own
credibility, for it would be perceived as no longer aiming to strike a balance, but seeking merely
to water down executive privilege to the point of irrelevance.

Conclusion

To recapitulate, petitioners’ demand to be furnished with a copy of the full text of the
JPEPA has become moot and academic, it having been made accessible to the public
since September 11, 2006. As for their demand for copies of the Philippine and
Japanese offers submitted during the JPEPA negotiations, the same must be denied,
respondents’ claim of executive privilege being valid.

Diplomatic negotiations have, since the Court promulgated its Resolution in PMPF
v. Manglapus on September 13, 1988, been recognized as privileged in this jurisdiction and the
reasons proffered by petitioners against the application of the ruling therein to the present case
have not persuaded the Court. Moreover, petitioners – both private citizens and members of the
House of Representatives – have failed to present a “sufficient showing of need” to overcome the
claim of privilege in this case.

That the privilege was asserted for the first time in respondents’ Comment to the present
petition, and not during the hearings of the House Special Committee on Globalization, is of no
moment, since it cannot be interpreted as a waiver of the privilege on the part of the Executive
branch.

For reasons already explained, this Decision shall not be interpreted as departing from the
ruling in Senate v. Ermita that executive privilege should be invoked by the President or through
the Executive Secretary “by order of the President.”

WHEREFORE, the petition is DISMISSED.


293
SO ORDERED.

CONCHITA CARPIO
MORALES
Associate Justice

WE CONCUR:

REYNATO S. PUNO
Chief Justice

LEONARDO A. QUISUMBING CONSUELO YNARES- SANTIAGO


Associate Justice Associate Justice

ANTONIO T. CARPIO MA. ALICIA AUSTRIA-MARTINEZ


Associate Justice Associate Justice

RENATO C. CORONA ADOLFO S. AZCUNA


Associate Justice Associate Justice

DANTE O. TINGA MINITA V. CHICO-NAZARIO


Associate Justice Associate Justice

294
PRESBITERO J. VELASCO, JR. ANTONIO EDUARDO B. NACHURA
Associate Justice Associate Justice

RUBEN T. REYES TERESITA J. LEONARDO-DE CASTRO


Associate Justice Associate Justice

ARTURO D. BRION
Associate Justice

CERTIFICATION

Pursuant to Section 13, Article VIII of the Constitution, I certify that the conclusions in
the above Decision had been reached in consultation before the case was assigned to the writer
of the opinion of the Court.

REYNATO S. PUNO
Chief Justice

*
In the case title as indicated in the petition, only the name of Usec. Thomas
G. Aquino appears in the portion for “Respondents,” to wit: “HON. THOMAS G. AQUINO, in
his capacity as Chairman and Chief Delegate of the Philippine Coordinating Committee for the
Japan-Philippines Economic Partnership Agreement, et al.” (Underscoring supplied) The
other respondents are enumerated in the body of the petition. (Rollo, pp. 20-23) The

295
Court motu proprio included the names of these other respondents in the case title to
conform to Sec. 1, par. 2, Rule 7 of the Rules of Civil Procedure, as well as the capacities in
which they are being sued. Moreover, it inserted therein that respondent Usec.Aquino, as
stated in the petition, is also being sued in his capacity as DTI Undersecretary.
[1]
Effective May 28, 2003.
[2]
Annex “F” of Petition, rollo, p. 95.
[3]
The Petition quoted the following statement of Congressman Teves appearing in the
transcript of the Committee hearing held on October 12, 2005:
THE CHAIRPERSON. Now I call on Usec. Aquino to furnish us a copy of the
draft JPEPA and enunciate to this body the positive as well as the negative impact of said
agreement. Is this the draft that the government will sign in December or this will still be
subjected to revisions in the run-up to its signing? x x x We requested also to subpoena
this but then the Speaker requested me to hold in abeyance because he wanted to get a
(sic) consent of the President before we can x x x the department can furnish us a copy of
this agreement. (Rollo, p. 32)
[4]
Id. at 16.
[5]
Annex “A,” Comment, rollo, p. 207.
[6]
Respondents’ Manifestation dated September 12, 2007; vide “Business Philippines: A
Department of Trade and Industry Website” at www.business.gov.ph,
particularlywww.business.gov.ph/DTI_News.php?contentID=136 (visited August 9, 2007).
[7]
Legaspi v. Civil Service Commission, G.R. No. L-72119, May 29, 1987; 150 SCRA 530,
535.
[8]
G.R. No. 158088, July 6, 2005; 462 SCRA 622, 630-631.
[9]
Supra note 7 at 536.
[10]
Reply to the Comment of the Solicitor General, rollo, p. 319 (underscoring supplied).
[11]
Business Philippines: A Department of Trade and Industry
Website, http://www.business.gov.ph/filedirectory/JPEPA.pdf, accessed on June 12, 2007.
[12]
By Resolution dated August 28, 2007, this Court directed the parties to manifest whether
the Philippine and Japanese offers have been made accessible to the public just like the full
text of the JPEPA and, if not, whether petitioners still intend to pursue their prayer to be
provided with copies thereof. In compliance, petitioners manifested that the offers have not
yet been made public and reiterated their prayer that respondents be compelled to provide
them with copies thereof, including all pertinent attachments and annexes thereto
(Manifestation and Motion dated September 17, 2007). Respondents, on the other hand,
asserted that the offers have effectively been made accessible to the public since September
11, 2006 (Manifestation dated September 12, 2007). Respondents’ claim does not persuade,
however. By their own manifestation, the documents posted on the DTI website on that date
were only the following: (1) Joint Statement on the Occasion of the Signing of the Agreement
between Japan and the Republic of the Philippines, (2) the full text of the JPEPA itself and its
annexes, (3) the JPEPA implementing Agreement, and (4) “resource materials on the JPEPA
including presentations of the [DTI] during the hearings of the Senate’s Committee on Trade
and Commerce and Committee on Economic Affairs.” While these documents no doubt
provide very substantial information on the JPEPA, the publication thereof still falls short of
addressing the prayer of petitioners to be provided with copies of the Philippine and
Japanese offers. Thus, the petition, insofar as it prays for access to these offers, has not
become moot.
[13]
CONSTITUTION, Art. III, Sec. 7.
[14]
Id. at Art. II, Sec. 28.
[15]
Id. at Art. XIII, Sec. 16.

296
[16]
Supra note 7 at 541.
[17]
314 Phil. 150 (1995).
[18]
360 Phil. 133 (1998).
[19]
433 Phil. 506 (2002).
[20]
G.R. No. 169777, April 20, 2006, 488 SCRA 1.
[21]
Id. at 51.
[22]
Rollo, pp. 191-192.
[23]
360 Phil. 133, 764 (1998), citing V RECORD OF THE CONSTITUTIONAL COMMISSION 25
(1986).
[24]
G.R. No. 84642, Resolution of the Court En Banc dated September 13, 1988.
[25]
Specifically, petitioners therein asked that the Court order respondents to (1) open to
petitioners their negotiations/sessions with the U.S. counterparts on the agreement; (2)
reveal and/or give petitioners access to the items which they have already agreed upon; and
(3) reveal and/or make accessible the respective positions on items they have not agreed
upon, particularly the compensation package for the continued use by the U.S. of their
military bases and facilities in the Philippines.
[26]
299 U.S. 304 (1936).
[27]
Vide Xerox Corp. v. U.S. (12 Cl.Ct. 93). Against the claim of a taxpayer for the production
of a letter from the Inland Revenue of the United Kingdom to the associate commissioner of
the Internal Revenue Service (IRS), defendant asserted a claim of privilege, relying on the
affidavit of Lawrence B. Gibbs, Commissioner of IRS, which stated that the production of the
letter “would impair the United States government's ability to deal with the tax authorities of
foreign governments * * * by breaching the historicconfidentiality of negotiations between
the United States and foreign sovereigns * * *.” (Emphasis supplied) The U.S. court therein
ruled thus: “Given the context in which the letter in question was written, it is reasonable to
conclude that frank and honest expression of views on the treaty language in issue were
expressed, views that ostensibly were expressed in the belief that “historic confidentiality”
would govern such expressions.” (Underscoring supplied)
[28]
B. DuVal, Jr., Project Director, American Bar Foundation. B.A.,
1958, University of Virginia; J.D., 1961, Yale University, THE OCCASIONS OF SECRECY (47 U.
Pitt. L. Rev. 579).
[29]
Supra note 20 at 46.
[30]
Ibid.
[31]
Supra note 19 at 189.
[32]
Senate Select Committee on Presidential Campaign Activities v. Nixon, 498 F.2d 725,
162 U.S.App.D.C. 183.
[33]
Vide Arnault v. Nazareno, 87 PHIL. 29, 46 (1950): “In the present case the jurisdiction of
the Senate, thru the Special Committee created by it, to investigate
the Buenavistaand Tambobong estates deal is not challenged by the petitioner; and we
entertain no doubt as to the Senate’s authority to do so and as to the validity of Resolution
No. 8 hereinabove quoted. The transaction involved a questionable and allegedly
unnecessary and irregular expenditure of no less than P5,000,000 of public funds, of which
Congress is the constitutional guardian. x x x”
[34]
421 U.S., at 150, 95 S.Ct. 1504, reiterated in Department of the Interior and Bureau of
Indian Affairs v. Klamath Water Users Protective Association, 532 U.S. 1, 121 S.Ct. 1060.
[35]
Id. at 151, 95 S.Ct. 1504 (emphasis supplied).
[36]
Supra note 24.
[37]
545 F.Supp. 615, May 28, 1982.

297
[38]
237 F.Supp.2d 17.
[39]
5 U.S.C. 552(b)(5).
[40]
CIEL v. Office of U.S. Trade Representative, 237 F.Supp.2d 17. Vide Department of the
Interior and Bureau of Indian Affairs v. Klamath Water Users Protective Association, 532 U.S.
1, 121 S.Ct. 1060: “Exemption 5 protects from disclosure “inter-agency or intra-agency
memorandums or letters which would not be available by law to a party other than an agency
in litigation with the agency.” 5 U.S.C. § 552(b)(5). To qualify, a document must thus satisfy
two conditions: its source must be a Government agency, and it must fall within the ambit of
a privilege against discovery under judicial standards that would govern litigation against the
agency that holds it.”
[41]
Supra note 20 at 46 (emphasis supplied).
[42]
Petitioners expound as follows:
“It has been 18 years since the PMPF v. Manglapus case, and the world
has changed considerably in that span of time. The Berlin Wall fell in 1989, bringing down
with it the Cold War and its attendant hostilities, and ushering in a new era of
globalization and international economic cooperation as we know it. The Philippinesnow
finds itself part of an international economic community as a member of both the ASEAN
Free Trade Area (AFTA) and the World Trade Organization (WTO). Domestically, this
Honorable Court has repeatedly upheld the people’s right to information on matters of
public concern, allowing ordinary Filipino citizens to inquire into various government
actions such as GSIS loans to public officials, settlement of Marcos ill-gotten wealth, and
sale of reclaimed land to foreign corporations.” (Rollo, p. 326)
[43]
Rollo, pp. 50-51.
[44]
Supra note 18.
[45]
Supra note 19.
[46]
433 Phil. 506, 534 (2002), citing PMPF v. Manglapus, supra note 24 and Chavez v. PCGG,
supra note 18.
[47]
In re Sealed Case (121 F.3d 729, 326 U.S.App.D.C. 276 [1997]) states thus: “Nixon,
GSA, Sirica, and the other Nixon cases all employed a balancing methodology in analyzing
whether, and in what circumstances, the presidential communications privilege can be
overcome. Under this methodology, these opinions balanced the public interests served by
protecting the President's confidentiality in a particular context with those furthered by
requiring disclosure.” (Emphasis supplied)
[48]
418 U.S. 683 (1974).
[49]
Supra note 31.
[50]
Supra note 47.
[51]
Supra note 32
[52]
Supra note 47.
[53]
Rollo, p. 349.
[54]
For a small sampling, vide “Primer sa Japan-Philippine Economic Partnership Agreement”
(JPEPA) at www.bayan.ph/downloads/Primer%20on%20jpepa.pdf; “A RESOLUTION
EXPRESSING SUPPORT TO THE CALLS FOR THE SENATE TO REJECT THE JAPAN-PHILIPPINES
PARTNERSHIP AGREEMENT (JPEPA)”
at www.nccphilippines.org/indexfiles/Page1562.htm; “JPEPA Ratification: Threat Economics”
at http://www.aer.ph/index.php?option/=com_content&task=view&id=632&Itemid=63 (all
sites visited on February 2, 2008).
[55]
Entitled “A DECREE TO CONSOLIDATE AND CODIFY ALL THE TARIFF AND CUSTOMS LAWS
OF THE PHILIPPINES,” promulgated June 11, 1978. In light of the arguments of petitioners,
the most salient portion of the provisions cited by them is Section 402(1) which states, in
298
part: “For the purpose of expanding foreign markets x x x in establishing and maintaining
better relations between the Philippines and other countries, the President is authorized from
time to time:
(1.1) To enter into trade agreements with foreign governments or
instrumentalities thereof; x x x”
[56]
396 Phil. 623, 663 (2000).
[57]
G.R. No. 158088, July 6, 2005, 462 SCRA 622, 632-633.
[58]
Supra note 55.
[59]
G.R. No. 169777, April 20, 2006, 488 SCRA 1, 44.
[60]
Id. at 68.
[61]
According to the records of this Court, the judgment in Senate v. Ermita was entered
on July 21, 2006. Respondents filed their Comment on May 15, 2006.
[62]
Revised Dissenting Opinion, p. 15 (Emphasis and underscoring supplied).
[63]
450 PHIL. 744 (2003), penned by then Associate Justice Puno.
[64]
Id., at 833 (Italics in the original, emphasis and underscoring supplied)
[65]
The Federalist, No. 75 (Italics in the original, emphasis and underscoring supplied).
[66]
Article II Section 2 of the U.S. Constitution states: “He [the President] shall have
Power, by and with the Advice and Consent of the Senate, to make Treaties, provided two
thirds of the Senators present concur x x x”. (Emphasis and underscoring supplied) On the
other hand, Article VII Section 21 of the Philippine Constitution states: “No treaty or
international agreement shall be valid and effective unless concurred in by at least two-thirds
of all the Members of the Senate.”
[67]
Supra note 18.
[68]
162 U.S. App.D.C. 183, 189.
[69]
365 F.3d 1108, 361 U.S.App.D.C. 183 (2004).
[70]
Supra note 47.
[71]
345 U.S. 1, 73 S.Ct. 528 (1953)
[72]
Supra at note 63.
[73]
Supra at note 64.
[74]
Supra note 19.
[75]
Supra at note 18.
[76]
U.S. v. Nixon (418 U.S. 683) states: “Nowhere in the Constitution x x x is there any
explicit reference to a privilege of confidentiality, yet to the extent this interest relates to
theeffective discharge of a President’s powers, it is constitutionally based.” (Emphasis, italics
and underscoring supplied)
[77]
In re Sealed Case (121 F.3d 729) states thus: “Nixon, GSA, Sirica, and the
other Nixon cases all employed a balancing methodology in analyzing whether, and in what
circumstances, the presidential communications privilege can be overcome. Under this
methodology, these opinions balanced the public interests served by protecting the
President’s confidentiality in a particular context with those furthered by requiring
disclosure.” (Emphasis and underscoring supplied)
[78]
G.R. No. 139465, October 17, 2000, penned by then Associate Justice Reynato S. Puno.
In that case, respondent Mark Jimenez claimed under the due process clause the right
to notice and hearing in the extradition proceedings against him. Consider the
following enlightening disquisition of the Court:
“In the case at bar, on one end of the balancing pole is the private
respondent’s claim to due process predicated on Section 1, Article III of the

299
Constitution, which provides that “No person shall be deprived of life, liberty, or
property without due process of law…” Without a bubble of a doubt, procedural due
process of law lies at the foundation of a civilized society which accords paramount
importance to justice and fairness. It has to be accorded the weight it deserves.
“This brings us to the other end of the balancing pole. Petitioner avers
that the Court should give more weight to our national commitment under the RP-
US Extradition Treaty to expedite the extradition to the United States of persons
charged with violation of some of its laws. Petitioner also emphasizes the need to
defer to the judgment of the Executive on matters relating to foreign affairs in
order not to weaken if not violate the principle of separation of powers.
“Considering that in the case at bar, the extradition proceeding is only at
its evaluation stage, the nature of the right being claimed by the private respondent
is nebulous and the degree of prejudice he will allegedly suffer is weak, we
accord greater weight to the interests espoused by the government thru the
petitioner Secretary of Justice. x x x (Emphasis, italics, and underscoring supplied)
[79]
Constitution, Art. XIII, Sec. 16.
[80]
433 U.S. 425.
[81]
88 Stat. 1695.
[82]
G.R. No. 180643, March 25, 2008.
[83]
Emphasis supplied.

300
SECOND DIVISION

REPUBLIC OF THE PHILIPPINES, G.R. No. 174629


Represented by THE ANTI-MONEY
LAUNDERING COUNCIL (AMLC), Present:
Petitioner,

QUISUMBING, J.,
Chairperson,
- versus - AUSTRIA MARTINEZ,*
CARPIO MORALES,
TINGA, and
HON. ANTONIO M. EUGENIO, VELASCO, JR., JJ.
JR., AS PRESIDING JUDGE OF
RTC, MANILA, BRANCH 34,
PANTALEON ALVAREZ and Promulgated:
LILIA CHENG,
Respondents. February 14, 2008

x ---------------------------------------------------------------------------------------x

DECISION

TINGA, J.:

The present petition for certiorari and prohibition under Rule 65 assails the orders and
resolutions issued by two different courts in two different cases. The courts and cases in
question are the Regional Trial Court of Manila, Branch 24, which heard SP Case No. 06-
114200[1] and the Court of Appeals, Tenth Division, which heared CA-G.R. SP No. 95198.[2] Both
cases arose as part of the aftermath of the ruling of this Court in Agan v. PIATCO[3] nullifying the
concession agreement awarded to the Philippine International Airport Terminal Corporation
(PIATCO) over the Ninoy Aquino International Airport – International Passenger Terminal 3
(NAIA 3) Project.

I.

Following the promulgation of Agan, a series of investigations concerning the award of


the NAIA 3 contracts to PIATCO were undertaken by the Ombudsman and the Compliance and
Investigation Staff (CIS) of petitioner Anti-Money Laundering Council (AMLC). On 24 May 2005,
301
the Office of the Solicitor General (OSG) wrote the AMLC requesting the latter’s assistance “in
obtaining more evidence to completely reveal the financial trail of corruption surrounding the
[NAIA 3] Project,” and also noting that petitioner Republic of the Philippines was presently
defending itself in two international arbitration cases filed in relation to the NAIA 3 Project.[4] The
CIS conducted an intelligence database search on the financial transactions of certain individuals
involved in the award, including respondent Pantaleon Alvarez (Alvarez) who had been the
Chairman of the PBAC Technical Committee, NAIA-IPT3 Project.[5] By this time, Alvarez had
already been charged by the Ombudsman with violation of Section 3(j) of R.A. No. 3019.[6] The
search revealed that Alvarez maintained eight (8) bank accounts with six (6) different banks.[7]

On 27 June 2005, the AMLC issued Resolution No. 75, Series of 2005,[8] whereby the
Council resolved to authorize the Executive Director of the AMLC “to sign and verify an
application to inquire into and/or examine the [deposits] or investments of Pantaleon Alvarez,
Wilfredo Trinidad, Alfredo Liongson, and Cheng Yong, and their related web of accounts
wherever these may be found, as defined under Rule 10.4 of the Revised Implementing Rules
and Regulations;” and to authorize the AMLC Secretariat “to conduct an inquiry into subject
accounts once the Regional Trial Court grants the application to inquire into and/or examine the
bank accounts” of those four individuals.[9] The resolution enumerated the particular bank
accounts of Alvarez, Wilfredo Trinidad (Trinidad), Alfredo Liongson (Liongson) and Cheng
Yong which were to be the subject of the inquiry.[10] The rationale for the said resolution was
founded on the cited findings of the CIS that amounts were transferred from a Hong Kong bank
account owned by Jetstream Pacific Ltd. Account to bank accounts in the Philippines maintained
by Liongson and Cheng Yong.[11] The Resolution also noted that “[b]y awarding the contract to
PIATCO despite its lack of financial capacity, Pantaleon Alvarez caused undue injury to the
government by giving PIATCO unwarranted benefits, advantage, or preference in the discharge
of his official administrative functions through manifest partiality, evident bad faith, or gross
inexcusable negligence, in violation of Section 3(e) of Republic Act No. 3019.”[12]

Under the authority granted by the Resolution, the AMLC filed an application to inquire
into or examine the deposits or investments of Alvarez, Trinidad, Liongson and Cheng Yong
before the RTC of Makati, Branch 138, presided by Judge (now Court of Appeals Justice) Sixto
Marella, Jr. The application was docketed as AMLC No. 05-005.[13] The Makati RTC heard the
302
testimony of the Deputy Director of the AMLC, Richard David C. Funk II, and received the
documentary evidence of the AMLC.[14] Thereafter, on 4 July 2005, the Makati RTC rendered an
Order (Makati RTC bank inquiry order) granting the AMLC the authority to inquire and examine
the subject bank accounts of Alvarez, Trinidad, Liongson and Cheng Yong, the trial court being
satisfied that there existed “[p]robable cause [to] believe that the deposits in various bank
accounts, details of which appear in paragraph 1 of the Application, are related to the offense of
violation of Anti-Graft and Corrupt Practices Act now the subject of criminal prosecution before
the Sandiganbayan as attested to by the Informations, Exhibits C, D, E, F, and G.”[15] Pursuant
to the Makati RTC bank inquiry order, the CIS proceeded to inquire and examine the deposits,
investments and related web accounts of the four.[16]

Meanwhile, the Special Prosecutor of the Office of the Ombudsman, Dennis Villa-Ignacio,
wrote a letter dated 2 November 2005, requesting the AMLC to investigate the accounts of
Alvarez, PIATCO, and several other entities involved in the nullified contract. The letter adverted
to probable cause to believe that the bank accounts “were used in the commission of unlawful
activities that were committed” in relation to the criminal cases then pending before the
Sandiganbayan.[17] Attached to the letter was a memorandum “on why the investigation of the
[accounts] is necessary in the prosecution of the above criminal cases before the
Sandiganbayan.”[18]

In response to the letter of the Special Prosecutor, the AMLC promulgated on 9 December
2005 Resolution No. 121 Series of 2005,[19] which authorized the executive director of the AMLC
to inquire into and examine the accounts named in the letter, including one maintained by
Alvarez with DBS Bank and two other accounts in the name of Cheng Yong with Metrobank. The
Resolution characterized the memorandum attached to the Special Prosecutor’s letter as
“extensively justif[ying] the existence of probable cause that the bank accounts of the persons
and entities mentioned in the letter are related to the unlawful activity of violation of Sections
3(g) and 3(e) of Rep. Act No. 3019, as amended.”[20]

Following the December 2005 AMLC Resolution, the Republic, through the AMLC, filed an
application[21] before the Manila RTC to inquire into and/or examine thirteen (13) accounts and

303
two (2) related web of accounts alleged as having been used to facilitate corruption in the NAIA
3 Project. Among said accounts were the DBS Bank account of Alvarez and the Metrobank
accounts of Cheng Yong. The case was raffled to Manila RTC, Branch 24, presided by respondent
Judge Antonio Eugenio, Jr., and docketed as SP Case No. 06-114200.

On 12 January 2006, the Manila RTC issued an Order (Manila RTC bank inquiry order)
granting the Ex Parte Application expressing therein “[that] the allegations in said application to
be impressed with merit, and in conformity with Section 11 of R.A. No. 9160, as amended,
otherwise known as the Anti-Money Laundering Act (AMLA) of 2001 and Rules 11.1 and 11.2 of
the Revised Implementing Rules and Regulations.”[22] Authority was thus granted to the AMLC to
inquire into the bank accounts listed therein.

On 25 January 2006, Alvarez, through counsel, entered his appearance[23] before the
Manila RTC in SP Case No. 06-114200 and filed an Urgent Motion to Stay Enforcement of Order
of January 12, 2006.[24] Alvarez alleged that he fortuitously learned of the bank inquiry order,
which was issued following an ex parte application, and he argued that nothing in R.A. No. 9160
authorized the AMLC to seek the authority to inquire into bank accounts ex parte.[25] The day
after Alvarez filed his motion, 26 January 2006, the Manila RTC issued an Order[26] staying the
enforcement of its bank inquiry order and giving the Republic five (5) days to respond to
Alvarez’s motion.

The Republic filed an Omnibus Motion for Reconsideration[27] of the 26 January


2006 Manila RTC Order and likewise sought to strike out Alvarez’s motion that led to the
issuance of said order. For his part, Alvarez filed a Reply and Motion to Dismiss[28] the
application for bank inquiry order. On 2 May 2006, the Manila RTC issued an Omnibus
Order[29] granting the Republic’s Motion for Reconsideration, denying Alvarez’s motion to dismiss
and reinstating “in full force and effect” the Order dated 12 January 2006. In the omnibus order,
the Manila RTC reiterated that the material allegations in the application for bank inquiry order
filed by the Republic stood as “the probable cause for the investigation and examination of the
bank accounts and investments of the respondents.”[30]

304
Alvarez filed on 10 May 2006 an Urgent Motion[31] expressing his apprehension that the
AMLC would immediately enforce the omnibus order and would thereby render the motion for
reconsideration he intended to file as moot and academic; thus he sought that the Republic be
refrained from enforcing the omnibus order in the meantime. Acting on this motion, the Manila
RTC, on 11 May 2006, issued an Order[32] requiring the OSG to file a comment/opposition and
reminding the parties that judgments and orders become final and executory upon the
expiration of fifteen (15) days from receipt thereof, as it is the period within which a motion for
reconsideration could be filed. Alvarez filed his Motion for Reconsideration[33] of the omnibus
order on 15 May 2006, but the motion was denied by the Manila RTC in an Order[34] dated 5
July 2006.

On 11 July 2006, Alvarez filed an Urgent Motion and Manifestation[35] wherein he


manifested having received reliable information that the AMLC was about to implement the
Manila RTC bank inquiry order even though he was intending to appeal from it. On the premise
that only a final and executory judgment or order could be executed or implemented, Alvarez
sought that the AMLC be immediately ordered to refrain from enforcing the Manila RTC bank
inquiry order.

On 12 July 2006, the Manila RTC, acting on Alvarez’s latest motion, issued an
Order[36] directing the AMLC “to refrain from enforcing the order dated January 12, 2006 until
the expiration of the period to appeal, without any appeal having been filed.” On the same day,
Alvarez filed a Notice of Appeal[37] with the Manila RTC.

On 24 July 2006, Alvarez filed an Urgent Ex Parte Motion for Clarification.[38] Therein, he
alleged having learned that the AMLC had began to inquire into the bank accounts of the other
persons mentioned in the application for bank inquiry order filed by the Republic.[39] Considering
that the Manila RTC bank inquiry order was issued ex parte, without notice to those other
persons, Alvarez prayed that the AMLC be ordered to refrain from inquiring into any of the other
bank deposits and alleged web of accounts enumerated in AMLC’s application with the RTC; and
that the AMLC be directed to refrain from using, disclosing or publishing in any proceeding or
venue any information or document obtained in violation of the 11 May 2006 RTC Order.[40]

305
On 25 July 2006, or one day after Alvarez filed his motion, the Manila RTC issued an
Order[41] wherein it clarified that “the Ex Parte Order of this Court dated January 12, 2006 can
not be implemented against the deposits or accounts of any of the persons enumerated in the
AMLC Application until the appeal of movant Alvarez is finally resolved, otherwise, the appeal
would be rendered moot and academic or even nugatory.”[42] In addition, the AMLC was ordered
“not to disclose or publish any information or document found or obtained in [v]iolation of
the May 11, 2006 Order of this Court.”[43] The Manila RTC reasoned that the other persons
mentioned in AMLC’s application were not served with the court’s 12 January 2006 Order. This25
July 2006 Manila RTC Order is the first of the four rulings being assailed through this petition.

In response, the Republic filed an Urgent Omnibus Motion for Reconsideration[44] dated 27
July 2006, urging that it be allowed to immediately enforce the bank inquiry order against
Alvarez and that Alvarez’s notice of appeal be expunged from the records since appeal from an
order of inquiry is disallowed under the Anti money Laundering Act (AMLA).

Meanwhile, respondent Lilia Cheng filed with the Court of Appeals a Petition for Certiorari,
Prohibition and Mandamus with Application for TRO and/or Writ of Preliminary
[45]
Injunction dated 10 July 2006, directed against the Republic of the Philippines through the
AMLC, Manila RTC Judge Eugenio, Jr. and Makati RTC Judge Marella, Jr.. She identified herself as
the wife of Cheng Yong[46] with whom she jointly owns a conjugal bank account with Citibank
that is covered by the Makati RTC bank inquiry order, and two conjugal bank accounts with
Metrobank that are covered by the Manila RTC bank inquiry order. Lilia Cheng imputed grave
abuse of discretion on the part of the Makati and Manila RTCs in granting AMLC’s ex
parteapplications for a bank inquiry order, arguing among others that the ex parte applications
violated her constitutional right to due process, that the bank inquiry order under the AMLA can
only be granted in connection with violations of the AMLA and that the AMLA can not apply to
bank accounts opened and transactions entered into prior to the effectivity of the AMLA or to
bank accounts located outside the Philippines.[47]

On 1 August 2006, the Court of Appeals, acting on Lilia Cheng’s petition, issued a
Temporary Restraining Order[48]enjoining the Manila and Makati trial courts from implementing,

306
enforcing or executing the respective bank inquiry orders previously issued, and the AMLC from
enforcing and implementing such orders. On even date, the Manila RTC issued an
Order[49] resolving to hold in abeyance the resolution of the urgent omnibus motion for
reconsideration then pending before it until the resolution of Lilia Cheng’s petition for certiorari
with the Court of Appeals. The Court of Appeals Resolution directing the issuance of the
temporary restraining order is the second of the four rulings assailed in the present petition.

The third assailed ruling[50] was issued on 15 August 2006 by the Manila RTC, acting on
the Urgent Motion for Clarification[51] dated 14 August 2006 filed by Alvarez. It appears that the
1 August 2006 Manila RTC Order had amended its previous 25 July 2006 Order by deleting the
last paragraph which stated that the AMLC “should not disclose or publish any information or
document found or obtained in violation of the May 11, 2006 Order of this Court.”[52] In this new
motion, Alvarez argued that the deletion of that paragraph would allow the AMLC to implement
the bank inquiry orders and publish whatever information it might obtain thereupon even before
the final orders of the Manila RTC could become final and executory.[53] In the 15 August
2006 Order, the Manila RTC reiterated that the bank inquiry order it had issued could not be
implemented or enforced by the AMLC or any of its representatives until the appeal therefrom
was finally resolved and that any enforcement thereof would be unauthorized.[54]

The present Consolidated Petition[55] for certiorari and prohibition under Rule 65 was filed
on 2 October 2006, assailing the two Orders of the Manila RTC dated 25 July and 15 August
2006 and the Temporary Restraining Order dated 1 August 2006of the Court of Appeals.
Through an Urgent Manifestation and Motion[56] dated 9 October 2006, petitioner informed the
Court that on 22 September 2006, the Court of Appeals hearing Lilia Cheng’s petition had
granted a writ of preliminary injunction in her favor.[57] Thereafter, petitioner sought as well the
nullification of the 22 September 2006 Resolution of the Court of Appeals, thereby constituting
the fourth ruling assailed in the instant petition.[58]

The Court had initially granted a Temporary Restraining Order[59] dated 6 October
2006 and later on a Supplemental Temporary Restraining Order[60] dated 13 October 2006 in
petitioner’s favor, enjoining the implementation of the assailed rulings of the Manila RTC and the

307
Court of Appeals. However, on respondents’ motion, the Court, through a Resolution[61]dated 11
December 2006, suspended the implementation of the restraining orders it had earlier issued.

Oral arguments were held on 17 January 2007. The Court consolidated the issues for
argument as follows:

1. Did the RTC-Manila, in issuing the Orders dated 25 July 2006 and 15
August 2006 which deferred the implementation of its Order dated 12 January
2006, and the Court of Appeals, in issuing its Resolution dated 1 August 2006,
which ordered the status quo in relation to the 1 July 2005 Order of the RTC-
Makati and the 12 January 2006 Order of the RTC-Manila, both of which
authorized the examination of bank accounts under Section 11 of Rep. Act No.
9160 (AMLA), commit grave abuse of discretion?

(a) Is an application for an order authorizing inquiry into or


examination of bank accounts or investments under Section 11 of the
AMLA ex-parte in nature or one which requires notice and hearing?

(b) What legal procedures and standards should be observed in


the conduct of the proceedings for the issuance of said order?

(c) Is such order susceptible to legal challenges and judicial


review?

2. Is it proper for this Court at this time and in this case to inquire into and
pass upon the validity of the 1 July 2005 Order of the RTC-Makati and the 12
January 2006 Order of the RTC-Manila, considering the pendency of CA G.R. SP
No. 95-198 (Lilia Cheng v. Republic) wherein the validity of both orders was
challenged?[62]

After the oral arguments, the parties were directed to file their respective memoranda,
which they did,[63] and the petition was thereafter deemed submitted for resolution.

II.

Petitioner’s general advocacy is that the bank inquiry orders issued by the Manila and
Makati RTCs are valid and immediately enforceable whereas the assailed rulings, which
effectively stayed the enforcement of the Manila and Makati RTCs bank inquiry orders, are
sullied with grave abuse of discretion. These conclusions flow from the posture that a bank
inquiry order, issued upon a finding of probable cause, may be issued ex parte and, once issued,
308
is immediately executory. Petitioner further argues that the information obtained following the
bank inquiry is necessarily beneficial, if not indispensable, to the AMLC in discharging its
awesome responsibility regarding the effective implementation of the AMLA and that any
restraint in the disclosure of such information to appropriate agencies or other judicial fora
would render meaningless the relief supplied by the bank inquiry order.

Petitioner raises particular arguments questioning Lilia Cheng’s right to seek injunctive
relief before the Court of Appeals, noting that not one of the bank inquiry orders is directed
against her. Her “cryptic assertion” that she is the wife of Cheng Yong cannot, according to
petitioner, “metamorphose into the requisite legal standing to seek redress for an imagined
injury or to maintain an action in behalf of another.” In the same breath, petitioner argues that
Alvarez cannot assert any violation of the right to financial privacy in behalf of other persons
whose bank accounts are being inquired into, particularly those other persons named in the
Makati RTC bank inquiry order who did not take any step to oppose such orders before the
courts.

Ostensibly, the proximate question before the Court is whether a bank inquiry order
issued in accordance with Section 10 of the AMLA may be stayed by injunction. Yet in arguing
that it does, petitioner relies on what it posits as the final and immediately executory character
of the bank inquiry orders issued by the Manila and Makati RTCs. Implicit in that position is the
notion that the inquiry orders are valid, and such notion is susceptible to review and validation
based on what appears on the face of the orders and the applications which triggered their
issuance, as well as the provisions of the AMLA governing the issuance of such orders. Indeed,
to test the viability of petitioner’s argument, the Court will have to be satisfied that the subject
inquiry orders are valid in the first place. However, even from a cursory examination of the
applications for inquiry order and the orders themselves, it is evident that the orders are not in
accordance with law.

III.

A brief overview of the AMLA is called for.

309
Money laundering has been generally defined by the International Criminal Police
Organization (Interpol) `as “any act or attempted act to conceal or disguise the identity of
illegally obtained proceeds so that they appear to have originated from legitimate
sources.”[64] Even before the passage of the AMLA, the problem was addressed by the Philippine
government through the issuance of various circulars by the Bangko Sentral ng Pilipinas. Yet
ultimately, legislative proscription was necessary, especially with the inclusion of
the Philippines in the Financial Action Task Force’s list of non-cooperative countries and
territories in the fight against money laundering.[65] The original AMLA, Republic Act (R.A.) No.
9160, was passed in 2001. It was amended by R.A. No. 9194 in 2003.

Section 4 of the AMLA states that “[m]oney laundering is a crime whereby the proceeds of
an unlawful activity as [defined in the law] are transacted, thereby making them appear to have
originated from legitimate sources.”[66] The section further provides the three modes through
which the crime of money laundering is committed. Section 7 creates the AMLC and defines its
powers, which generally relate to the enforcement of the AMLA provisions and the initiation of
legal actions authorized in the AMLA such as civil forefeiture proceedings and complaints for the
prosecution of money laundering offenses.[67]

In addition to providing for the definition and penalties for the crime of money laundering,
the AMLA also authorizes certain provisional remedies that would aid the AMLC in the
enforcement of the AMLA. These are the “freeze order” authorized under Section 10, and the
“bank inquiry order” authorized under Section 11.

Respondents posit that a bank inquiry order under Section 11 may be obtained only upon
the pre-existence of a money laundering offense case already filed before the courts.[68] The
conclusion is based on the phrase “upon order of any competent court in cases of violation of
this Act,” the word “cases” generally understood as referring to actual cases pending with the
courts.

We are unconvinced by this proposition, and agree instead with the then Solicitor General
who conceded that the use of the phrase “in cases of” was unfortunate, yet submitted that it
should be interpreted to mean “in the event there are violations” of the AMLA, and not that
310
there are already cases pending in court concerning such violations.[69] If the contrary position is
adopted, then the bank inquiry order would be limited in purpose as a tool in aid of litigation of
live cases, and wholly inutile as a means for the government to ascertain whether there is
sufficient evidence to sustain an intended prosecution of the account holder for violation of the
AMLA. Should that be the situation, in all likelihood the AMLC would be virtually deprived of its
character as a discovery tool, and thus would become less circumspect in filing complaints
against suspect account holders. After all, under such set-up the preferred strategy would be to
allow or even encourage the indiscriminate filing of complaints under the AMLA with the hope or
expectation that the evidence of money laundering would somehow surface during the
trial. Since the AMLC could not make use of the bank inquiry order to determine whether there
is evidentiary basis to prosecute the suspected malefactors, not filing any case at all would not
be an alternative. Such unwholesome set-up should not come to pass. Thus Section 11 cannot
be interpreted in a way that would emasculate the remedy it has established and encourage the
unfounded initiation of complaints for money laundering.

Still, even if the bank inquiry order may be availed of without need of a pre-existing case
under the AMLA, it does not follow that such order may be availed of ex parte. There are several
reasons why the AMLA does not generally sanction ex parteapplications and issuances of the
bank inquiry order.

IV.

It is evident that Section 11 does not specifically authorize, as a general rule, the
issuance ex parte of the bank inquiry order. We quote the provision in full:

SEC. 11. Authority to Inquire into Bank Deposits. ― Notwithstanding the


provisions of Republic Act No. 1405, as amended, Republic Act No. 6426, as
amended, Republic Act No. 8791, and other laws, the AMLC may inquire into or
examine any particular deposit or investment with any banking institution or non
bank financial institution upon order of any competent court in cases of violation of
this Act, when it has been established that there is probable cause that the deposits
or investments are related to an unlawful activity as defined in Section 3(i) hereof
or a money laundering offense under Section 4 hereof, except that no court order
shall be required in cases involving unlawful activities defined in Sections 3(i)1, (2)
and (12).
311
To ensure compliance with this Act, the Bangko Sentral ng Pilipinas (BSP)
may inquire into or examine any deposit of investment with any banking institution
or non bank financial institution when the examination is made in the course of a
periodic or special examination, in accordance with the rules of examination of the
BSP.[70] (Emphasis supplied)

Of course, Section 11 also allows the AMLC to inquire into bank accounts without having
to obtain a judicial order in cases where there is probable cause that the deposits or investments
are related to kidnapping for ransom,[71] certain violations of the Comprehensive Dangerous
Drugs Act of 2002,[72] hijacking and other violations under R.A. No. 6235, destructive arson and
murder. Since such special circumstances do not apply in this case, there is no need for us to
pass comment on this proviso. Suffice it to say, the proviso contemplates a situation distinct
from that which presently confronts us, and for purposes of the succeeding discussion, our
reference to Section 11 of the AMLA excludes said proviso.

In the instances where a court order is required for the issuance of the bank inquiry
order, nothing in Section 11 specifically authorizes that such court order may be issued ex
parte. It might be argued that this silence does not preclude the ex parte issuance of the bank
inquiry order since the same is not prohibited under Section 11. Yet this argument falls when
the immediately preceding provision, Section 10, is examined.

SEC. 10. Freezing of Monetary Instrument or Property. ― The Court of


Appeals, upon application ex parte by the AMLC and after determination
that probable cause exists that any monetary instrument or property is in any way
related to an unlawful activity as defined in Section 3(i) hereof, may issue a freeze
order which shall be effective immediately. The freeze order shall be for a period of
twenty (20) days unless extended by the court.[73]

Although oriented towards different purposes, the freeze order under Section 10 and the
bank inquiry order under Section 11 are similar in that they are extraordinary provisional reliefs
which the AMLC may avail of to effectively combat and prosecute money laundering offenses.
Crucially, Section 10 uses specific language to authorize an ex parte application for the
provisional relief therein, a circumstance absent in Section 11. If indeed the legislature had
intended to authorize ex parte proceedings for the issuance of the bank inquiry order, then it

312
could have easily expressed such intent in the law, as it did with the freeze order under Section
10.

Even more tellingly, the current language of Sections 10 and 11 of the AMLA was crafted
at the same time, through the passage of R.A. No. 9194. Prior to the amendatory law, it was
the AMLC, not the Court of Appeals, which had authority to issue a freeze order, whereas a bank
inquiry order always then required, without exception, an order from a competent court.[74]It
was through the same enactment that ex parte proceedings were introduced for the first time
into the AMLA, in the case of the freeze order which now can only be issued by the Court of
Appeals. It certainly would have been convenient, through the same amendatory law, to allow a
similar ex parte procedure in the case of a bank inquiry order had Congress been so minded. Yet
nothing in the provision itself, or even the available legislative record, explicitly points to an ex
parte judicial procedure in the application for a bank inquiry order, unlike in the case of the
freeze order.

That the AMLA does not contemplate ex parte proceedings in applications for bank inquiry
orders is confirmed by the present implementing rules and regulations of the AMLA,
promulgated upon the passage of R.A. No. 9194. With respect to freeze orders under Section
10, the implementing rules do expressly provide that the applications for freeze orders be
filed ex parte,[75] but no similar clearance is granted in the case of inquiry orders under Section
11.[76] These implementing rules were promulgated by the Bangko Sentral ng Pilipinas, the
Insurance Commission and the Securities and Exchange Commission,[77]and if it was the true
belief of these institutions that inquiry orders could be issued ex parte similar to freeze orders,
language to that effect would have been incorporated in the said Rules. This is stressed not
because the implementing rules could authorize ex parte applications for inquiry orders despite
the absence of statutory basis, but rather because the framers of the law had no intention to
allow such ex parte applications.

Even the Rules of Procedure adopted by this Court in A.M. No. 05-11-04-SC[78] to enforce
the provisions of the AMLA specifically authorize ex parte applications with respect to freeze
orders under Section 10[79] but make no similar authorization with respect to bank inquiry orders
under Section 11.
313
The Court could divine the sense in allowing ex parte proceedings under Section 10 and in
proscribing the same under Section 11. A freeze order under Section 10 on the one hand is
aimed at preserving monetary instruments or property in any way deemed related to unlawful
activities as defined in Section 3(i) of the AMLA. The owner of such monetary instruments or
property would thus be inhibited from utilizing the same for the duration of the freeze order. To
make such freeze order anteceded by a judicial proceeding with notice to the account
holder would allow for or lead to the dissipation of such funds even before the order could be
issued.

On the other hand, a bank inquiry order under Section 11 does not necessitate any form
of physical seizure of property of the account holder. What the bank inquiry order authorizes is
the examination of the particular deposits or investments in banking institutions or non-bank
financial institutions. The monetary instruments or property deposited with such banks or
financial institutions are not seized in a physical sense, but are examined on particular details
such as the account holder’s record of deposits and transactions. Unlike the assets subject of the
freeze order, the records to be inspected under a bank inquiry order cannot be physically seized
or hidden by the account holder. Said records are in the possession of the bank and therefore
cannot be destroyed at the instance of the account holder alone as that would require the
extraordinary cooperation and devotion of the bank.

Interestingly, petitioner’s memorandum does not attempt to demonstrate before the


Court that the bank inquiry order under Section 11 may be issued ex parte, although the
petition itself did devote some space for that argument. The petition argues that the bank
inquiry order is “a special and peculiar remedy, drastic in its name, and made necessary
because of a public necessity… [t]hus, by its very nature, the application for an order or inquiry
must necessarily, be ex parte.” This argument is insufficient justification in light of the clear
disinclination of Congress to allow the issuance ex parte of bank inquiry orders under Section
11, in contrast to the legislature’s clear inclination to allow the ex parte grant of freeze orders
under Section 10.

314
Without doubt, a requirement that the application for a bank inquiry order be done with
notice to the account holder will alert the latter that there is a plan to inspect his bank account
on the belief that the funds therein are involved in an unlawful activity or money laundering
offense.[80] Still, the account holder so alerted will in fact be unable to do anything to conceal or
cleanse his bank account records of suspicious or anomalous transactions, at least not without
the whole-hearted cooperation of the bank, which inherently has no vested interest to aid the
account holder in such manner.

V.

The necessary implication of this finding that Section 11 of the AMLA does not generally
authorize the issuance ex parteof the bank inquiry order would be that such orders cannot be
issued unless notice is given to the owners of the account, allowing them the opportunity to
contest the issuance of the order. Without such a consequence, the legislated distinction
between ex parte proceedings under Section 10 and those which are not ex parte under Section
11 would be lost and rendered useless.

There certainly is fertile ground to contest the issuance of an ex parte order. Section 11
itself requires that it be established that “there is probable cause that the deposits or
investments are related to unlawful activities,” and it obviously is the court which stands as
arbiter whether there is indeed such probable cause. The process of inquiring into the existence
of probable cause would involve the function of determination reposed on the trial court.
Determination clearly implies a function of adjudication on the part of the trial court, and not a
mechanical application of a standard pre-determination by some other body. The word
"determination" implies deliberation and is, in normal legal contemplation, equivalent to "the
decision of a court of justice."[81]

The court receiving the application for inquiry order cannot simply take the AMLC’s word
that probable cause exists that the deposits or investments are related to an unlawful activity. It
will have to exercise its

315
own determinative function in order to be convinced of such fact. The account holder would be
certainly capable of contesting such probable cause if given the opportunity to be apprised of the
pending application to inquire into his account; hence a notice requirement would not be an
empty spectacle. It may be so that the process of obtaining the inquiry order may become more
cumbersome or prolonged because of the notice requirement, yet we fail to see any
unreasonable burden cast by such circumstance. After all, as earlier stated, requiring notice to
the account holder should not, in any way, compromise the integrity of the bank records subject
of the inquiry which remain in the possession and control of the bank.

Petitioner argues that a bank inquiry order necessitates a finding of probable cause, a
characteristic similar to a search warrant which is applied to and heard ex parte. We have
examined the supposed analogy between a search warrant and a bank inquiry order yet we
remain to be unconvinced by petitioner.

The Constitution and the Rules of Court prescribe particular requirements attaching to
search warrants that are not imposed by the AMLA with respect to bank inquiry orders. A
constitutional warrant requires that the judge personally examine under oath or affirmation the
complainant and the witnesses he may produce,[82] such examination being in the form of
searching questions and answers.[83] Those are impositions which the legislative did not
specifically prescribe as to the bank inquiry order under the AMLA, and we cannot find sufficient
legal basis to apply them to Section 11 of the AMLA. Simply put, a bank inquiry order is not a
search warrant or warrant of arrest as it contemplates a direct object but not the seizure of
persons or property.

Even as the Constitution and the Rules of Court impose a high procedural standard for the
determination of probable cause for the issuance of search warrants which Congress chose not
to prescribe for the bank inquiry order under the AMLA, Congress nonetheless disallowed ex
parte applications for the inquiry order. We can discern that in exchange for these procedural
standards normally applied to search warrants, Congress chose instead to legislate a right to
notice and a right to be heard— characteristics of judicial proceedings which are not ex parte.
Absent any demonstrable constitutional infirmity, there is no reason for us to dispute such
legislative policy choices.
316
VI.

The Court’s construction of Section 11 of the AMLA is undoubtedly influenced by right to


privacy considerations. If sustained, petitioner’s argument that a bank account may be inspected
by the government following an ex parte proceeding about which the depositor would know
nothing would have significant implications on the right to privacy, a right innately cherished by
all notwithstanding the legally recognized exceptions thereto. The notion that the government
could be so empowered is cause for concern of any individual who values the
right to privacy which, after all, embodies even the right to be “let
alone,” the most comprehensive of rights and the right most valued by civilized people.[84]

One might assume that the constitutional dimension of the right to privacy, as applied to
bank deposits, warrants our present inquiry. We decline to do so. Admittedly, that question has
proved controversial in American jurisprudence. Notably, the United States Supreme Court
in U.S. v. Miller[85] held that there was no legitimate expectation of privacy as to the bank
records of a depositor.[86] Moreover, the text of our Constitution has not bothered with the
triviality of allocating specific rights peculiar to bank deposits.

However, sufficient for our purposes, we can assert there is a right to privacy governing
bank accounts in the Philippines, and that such right finds application to the case at bar. The
source of such right is statutory, expressed as it is in R.A. No. 1405 otherwise known as the
Bank Secrecy Act of 1955. The right to privacy is enshrined in Section 2 of that law, to wit:

SECTION 2. All deposits of whatever nature with banks or banking


institutions in the Philippines including investments in bonds issued by the
Government of the Philippines, its political subdivisions and its instrumentalities,
are hereby considered as of an absolutely confidential nature and may not be
examined, inquired or looked into by any person, government official, bureau or
office, except upon written permission of the depositor, or in cases of
impeachment, or upon order of a competent court in cases of bribery or
dereliction of duty of public officials, or in cases where the money deposited or
invested is the subject matter of the litigation. (Emphasis supplied)

317
Because of the Bank Secrecy Act, the confidentiality of bank deposits remains a basic
state policy in the Philippines.[87]Subsequent laws, including the AMLA, may have added
exceptions to the Bank Secrecy Act, yet the secrecy of bank deposits still lies as the general
rule. It falls within the zones of privacy recognized by our laws.[88] The framers of the 1987
Constitution likewise recognized that bank accounts are not covered by either the right to
information[89] under Section 7, Article
III or under the requirement of full public disclosure[90] under
Section 28, Article II.[91] Unless the Bank Secrecy Act is repealed or
amended, the legal order is obliged to conserve the absolutely confidential nature of Philippine
bank deposits.

Any exception to the rule of absolute confidentiality must be specifically legislated. Section
2 of the Bank Secrecy Act itself prescribes exceptions whereby these bank accounts may be
examined by “any person, government official, bureau or office”; namely when: (1) upon written
permission of the depositor; (2) in cases of impeachment; (3) the examination of bank accounts
is upon order of a competent court in cases of bribery or dereliction of duty of public officials;
and (4) the money deposited or invested is the subject matter of the litigation. Section 8 of R.A.
Act No. 3019, the Anti-Graft and Corrupt Practices Act, has been recognized by this Court as
constituting an additional exception to the rule of absolute confidentiality,[92] and there have
been other similar recognitions as well.[93]

The AMLA also provides exceptions to the Bank Secrecy Act. Under Section 11, the AMLC
may inquire into a bank account upon order of any competent court in cases of violation of the
AMLA, it having been established that there is probable cause that the deposits or investments
are related to unlawful activities as defined in Section 3(i) of the law, or a money laundering
offense under Section 4 thereof. Further, in instances where there is probable cause that the
deposits or investments are related to kidnapping for ransom,[94] certain violations of the
Comprehensive Dangerous Drugs Act of 2002,[95] hijacking and other violations under R.A. No.
6235, destructive arson and murder, then there is no need for the AMLC to obtain a court order
before it could inquire into such accounts.

318
It cannot be successfully argued the proceedings relating to the bank inquiry order under
Section 11 of the AMLA is a “litigation” encompassed in one of the exceptions to the Bank
Secrecy Act which is when “the money deposited or invested is the subject matter of the
litigation.” The orientation of the bank inquiry order is simply to serve as a provisional relief or
remedy. As earlier stated, the application for such does not entail a full-blown trial.

Nevertheless, just because the AMLA establishes additional exceptions to the Bank
Secrecy Act it does not mean that the later law has dispensed with the general principle
established in the older law that “[a]ll deposits of whatever nature with banks or banking
institutions in the Philippines x x x are hereby considered as of an absolutely confidential
nature.”[96] Indeed, by force of statute, all bank deposits are absolutely confidential, and that
nature is unaltered even by the legislated exceptions referred to above. There is disfavor
towards construing these exceptions in such a manner that would authorize unlimited discretion
on the part of the government or of any party seeking to enforce those exceptions and inquire
into bank deposits. If there are doubts in upholding the absolutely confidential nature of bank
deposits against affirming the authority to inquire into such accounts, then such doubts must be
resolved in favor of the former. Such a stance would persist unless Congress passes a law
reversing the general state policy of preserving the absolutely confidential nature of Philippine
bank accounts.

The presence of this statutory right to privacy addresses at least one of the arguments
raised by petitioner, that Lilia Cheng had no personality to assail the inquiry orders before the
Court of Appeals because she was not the subject of said orders. AMLC Resolution No. 75, which
served as the basis in the successful application for the Makati inquiry order, expressly adverts
to Citibank Account No. 88576248 “owned by Cheng Yong and/or Lilia G. Cheng with Citibank
N.A.,”[97] whereas Lilia Cheng’s petition before the Court of Appeals is accompanied by a
certification from Metrobank that Account Nos. 300852436-0 and 700149801-7, both of which
are among the subjects of the Manila inquiry order, are accounts in the name of “Yong Cheng or
Lilia Cheng.”[98] Petitioner does not specifically deny that Lilia Cheng holds rights of ownership
over the three said accounts, laying focus instead on the fact that she was not named as a
subject of either the Makati or Manila RTC inquiry orders. We are reasonably convinced that Lilia
Cheng has sufficiently demonstrated her joint ownership of the three accounts, and such
319
conclusion leads us to acknowledge that she has the standing to assail via certiorari the inquiry
orders authorizing the examination of her bank accounts as the orders interfere with her
statutory right to maintain the secrecy of said accounts.

While petitioner would premise that the inquiry into Lilia Cheng’s accounts finds root in
Section 11 of the AMLA, it cannot be denied that the authority to inquire under Section 11 is
only exceptional in character, contrary as it is to the general rule preserving the secrecy of bank
deposits. Even though she may not have been the subject of the inquiry orders, her bank
accounts nevertheless were, and she thus has the standing to vindicate the right to secrecy that
attaches to said accounts and their owners. This statutory right to privacy will not prevent the
courts from authorizing the inquiry anyway upon the fulfillment of the requirements set forth
under Section 11 of the AMLA or Section 2 of the Bank Secrecy Act; at the same time, the owner
of the accounts have the right to challenge whether the requirements were indeed complied
with.

VII.

There is a final point of concern which needs to be addressed. Lilia Cheng argues that the
AMLA, being a substantive penal statute, has no retroactive effect and the bank inquiry order
could not apply to deposits or investments opened prior to the effectivity of Rep. Act No. 9164,
or on 17 October 2001. Thus, she concludes, her subject bank accounts, opened between 1989
to 1990, could not be the subject of the bank inquiry order lest there be a violation of the
constitutional prohibition against ex post facto laws.

No ex post facto law may be enacted,[99] and no law may be construed in such fashion as
to permit a criminal prosecution offensive to the ex post facto clause. As applied to the AMLA, it
is plain that no person may be prosecuted under the penal provisions of the AMLA for acts
committed prior to the enactment of the law on 17 October 2001. As much was understood by
the lawmakers since they deliberated upon the AMLA, and indeed there is no serious dispute on
that point.

320
Does the proscription against ex post facto laws apply to the interpretation of Section 11,
a provision which does not provide for a penal sanction but which merely authorizes the
inspection of suspect accounts and deposits? The answer is in the affirmative. In this jurisdiction,
we have defined an ex post facto law as one which either:

(1) makes criminal an act done before the passage of the law and
which was innocent when done, and punishes such an act;
(2) aggravates a crime, or makes it greater than it was, when
committed;
(3) changes the punishment and inflicts a greater punishment than the
law annexed to the crime when committed;
(4) alters the legal rules of evidence, and authorizes conviction upon
less or different testimony than the law required at the time of the commission of
the offense;
(5) assuming to regulate civil rights and remedies only, in effect
imposes penalty or deprivation of a right for something which when done was
lawful; and
(6) deprives a person accused of a crime of some lawful protection to
which he has become entitled, such as the protection of a former conviction or
acquittal, or a proclamation of amnesty. (Emphasis supplied)[100]

Prior to the enactment of the AMLA, the fact that bank accounts or deposits were involved
in activities later on enumerated in Section 3 of the law did not, by itself, remove such accounts
from the shelter of absolute confidentiality. Prior to the AMLA, in order that bank accounts could
be examined, there was need to secure either the written permission of the depositor or a court
order authorizing such examination, assuming that they were involved in cases of bribery or
dereliction of duty of public officials, or in a case where the money deposited or invested was
itself the subject matter of the litigation. The passage of the AMLA stripped another layer off the
rule on absolute confidentiality that provided a measure of lawful protection to the account
holder. For that reason, the application of the bank inquiry order as a means of inquiring into
records of transactions entered into prior to the passage of the AMLA would be constitutionally
infirm, offensive as it is to the ex post facto clause.

Still, we must note that the position submitted by Lilia Cheng is much broader than what
we are willing to affirm. She argues that the proscription against ex post facto laws goes as far
as to prohibit any inquiry into deposits or investments included in bank accounts opened prior to

321
the effectivity of the AMLA even if the suspect transactions were entered into when the law had
already taken effect. The Court recognizes that if this argument were to be affirmed, it would
create a horrible loophole in the AMLA that would in turn supply the means to fearlessly engage
in money laundering in the Philippines; all that the criminal has to do is to make sure that the
money laundering activity is facilitated through a bank account opened prior to 2001. Lilia Cheng
admits that “actual money launderers could utilize the ex post facto provision of the Constitution
as a shield” but that the remedy lay with Congress to amend the law. We can hardly presume
that Congress intended to enact a self-defeating law in the first place, and the courts are
inhibited from such a construction by the cardinal rule that “a law should be interpreted with a
view to upholding rather than destroying it.”[101]

Besides, nowhere in the legislative record cited by Lilia Cheng does it appear that there
was an unequivocal intent to exempt from the bank inquiry order all bank accounts opened prior
to the passage of the AMLA. There is a cited exchange between Representatives Ronaldo Zamora
and Jaime Lopez where the latter confirmed to the former that “deposits are supposed to be
exempted from scrutiny or monitoring if they are already in place as of the time the law is
enacted.”[102] That statement does indicate that transactions already in place when the AMLA
was passed are indeed exempt from scrutiny through a bank inquiry order, but it cannot yield
any interpretation that records of transactions undertaken after the enactment of the AMLA are
similarly exempt. Due to the absence of cited authority from the legislative record that
unqualifiedly supports respondent Lilia Cheng’s thesis, there is no cause for us to sustain her
interpretation of the AMLA, fatal as it is to the anima of that law.

IX.

We are well aware that Lilia Cheng’s petition presently pending before the Court of
Appeals likewise assails the validity of the subject bank inquiry orders and precisely seeks the
annulment of said orders. Our current declarations may indeed have the effect of preempting
that0 petition. Still, in order for this Court to rule on the petition at bar which insists on the

322
enforceability of the said bank inquiry orders, it is necessary for us to consider and rule on the
same question which after all is a pure question of law.

WHEREFORE, the PETITION is DISMISSED. No pronouncement as to costs.

SO ORDERED.

DANTE O. TINGA
Associate Justice

323
WE CONCUR:

LEONARDO A. QUISUMBING
Associate Justice
Chairperson

MA. ALICIA AUSTRIA MARTINEZ CONCHITA CARPIO MORALES


Associate Justice Associate Justice

PRESBITERO J. VELASCO, JR.


Associate Justice

ATTESTATION

I attest that the conclusions in the above Decision were reached in consultation before
the case was assigned to the writer of the opinion of the Court’s Division.

LEONARDO A. QUISUMBING
Associate Justice
Chairperson, Second Division

CERTIFICATION

Pursuant to Section 13, Article VIII of the Constitution, and the Division Chairperson’s
Attestation, it is hereby certified that the conclusions in the above Decision were reached in
consultation before the case was assigned to the writer of the opinion of the Court’s Division.

324
REYNATO S. PUNO
Chief Justice

*
As replacement of Justice Antonio T. Carpio who inhibited himself per Administrative
Circular No. 84-2007.
[1]
Entitled “In the Matter of the Application for An Order Allowing An Inquiry Into Certain
Bank Accounts or Investments and Related Web of Accounts, The Republic of
the Philippines Represented by the Anti-Money Laundering Council, Applicant.”
[2]
Entitled “Lilia Cheng v. Republic of the Philippines represented by the Anti-Money
Laundering Council (AMLC), Hon. Antonio M. Eugenio, As Presiding Judge of the RTC Manila, Br.
24; Hon. Sixto Marella, Jr., as Presiding Judge of RTC, Makati City, Br. 38; and John Does.”
[3]
G.R. No. 155001.
[4]
Rollo, p. 96.
[5]
Id. at 97.
[6]
Sec. 3. Corrupt practices of public officers. - In addition to acts or omissions of public
officers already penalized by existing law, the following shall constitute corrupt practices of any
public officer and are hereby declared to be unlawful:

(j) Knowingly approving or granting any license, permit, privilege or


benefit in favor of any person not qualified for or not legally entitled to such
license, permit, privilege or advantage, or of a mere representative or dummy of
one who is not so qualified or entitled.
[7]
Rollo, p. 98.
[8]
Id. at 96-100.
[9]
Id. at 99-100.
[10]
Id. at 98.

325
[11]
Id.
[12]
Id. at 99.
[13]
Id. at 101.
[14]
Id.
[15]
Id.
[16]
Id. at 27.
[17]
Id. at 104.
[18]
Id.
[19]
Id. at 105-107.
[20]
Id. at 106.
[21]
See id. at 109-110.
[22]
Id. at 109.
[23]
Id. at 111.
[24]
Id. at 111-117.

[25]
Id. at 111.
[26]
Id. at 118.
[27]
Id. at 119-130.
[28]
Id. at 131-141.
[29]
Id. at 142-147.
[30]
Id. at 146.
[31]
Id. at 148-149.

[32]
Id. at 150.
[33]
Id. at 151-158.
[34]
Id. at 167.

326
[35]
Id. at 168-169.
[36]
Id. at 171.
.
[37]
Id. at 172-173.
[38]
Id. at 174-175.
[39]
Id. at 174.
[40]
Id. at 175.
[41]
Id. at 68-69.
[42]
Id. at 69.

[43]
Id.
[44]
Id. at 176-186.
[45]
Id. at 187-249.
[46]
Id. at 189.
[47]
Id. at 200-201.
[48]
Id. at 73-77.
[49]
Id. at 78.
[50]
Order dated 15 August 2006, see id. at 71.
[51]
Id. at 285-287.
[52]
Id. at 285-286.
[53]
Id. at 286.
[54]
Id. at 71.
[55]
Id. at 6-65.
[56]
Id. at 299-304.
[57]
See id. at 310.

327
[58]
Id. at 302.
[59]
Id. at 297-298.
[60]
Id. at 312-313.
[61]
Id. at 549-551.
[62]
Id. at 752-753.
[63]
See rollo, pp. 786-828; 867-910; 913-936.
[64]
See Funds derived from criminal activities (FOPAC),
(http://www.interpol.int/Public/ FinancialCrime/MoneyLaundering/default.asp, last visited 8
December 2007). See also J.M.B. TIROL, THE ANTI-MONEY LAUNDERING LAW OF
THE PHILIPPINES Annotated (2nd ed., 2007), at 3.
[65]
TIROL, supra note 64, at 4-6. The Financial Action Task Force was established in 1989
by the so-called Group of 7 countries to formulate and encourage the adoption of international
standards and measures to fight money laundering and related activities. Id. at 28.
[66]
Republic Act No. 9160 (2002), Sec. 4.
[67]
Republic Act No. 9160 (2002), Secs. 7(3) and (4).
[68]
See rollo, pp. 809-810, 932.
[69]
Id. at 600-601.
[70]
Republic Act No. 9194 (2003), Sec. 11.
[71]
Under Article 267 of the Revised Penal Code.
[72]
Particularly Sections 4, 5, 6, 8, 9, 10, 12, 13, 14, 15, and 16 thereof.
[73]
Republic Act No. 9194 (2003), Sec. 10.
[74]
Unlike in the present law which authorizes the issuance without need of judicial order
when there is probable cause that the deposits are involved in such specifically enumerated
crimes as kidnapping, hijacking, destructive arson and murder, and violations of some provisions
of the Dangerous Drugs Act of 2002. See Sec. 11, R.A. No. 9194, in connection with Section
3(i).
[75]
“Rule 10.1. When the AMLC may apply for the freezing of any monetary instrument or
property.

(a) after an investigation conducted by the AMLC and upon determination


that probable cause exists that a monetary instrument or property is in any way
related to any unlawful activity as defined under section 3(i). The AMLC may file

328
an ex-parte application before the the Court of Appeals for the issuance of a freeze
order on any monetary instrument or property subject thereof prior to the
institution or in the course of, the criminal proceedings involving the unlawful
activity to which said monetary instrument or property is any way related.” Rule
10.1, Revised Implementing Rules And Regulations R.A. No. 9160, As Amended By
R.A. No. 9194. (Emphasis supplied)
[76]
See Rule 11.1, Revised Implementing Rules And Regulations R.A. No. 9160, As
Amended By R.A. No. 9194. “Rule 11.1. Authority to Inquire into Bank Deposits With Court
Order. Notwithstanding the provisions of Republic Act No. 1405, as amended; Republic Act No.
6426, as amended; Republic Act No. 8791, and other laws, the AMLC may inquire into or
examine any particular deposit or investment with any banking institution or non-bank financial
institution AND THEIR SUBSIDIARIES AND AFFILIATES uponorder of any competent court in
cases of violation of this Act, when it has been established that there is probable cause that the
deposits or investments involved are related to an unlawful activity as defined in Section 3(j)
hereof or a money laundering offense under Section 4 hereof; except in cases as provided under
Rule 11.2.”
[77]
Republic Act No. 9160 (See Section 18, AMLA).
[78]
Effective 15 December 2005.
[79]
See Title VIII, Sec. 44, Rule Of Procedure In Cases Of Civil Forfeiture, Asset
Preservation, And Freezing Of Monetary Instrument, Property, Or Proceeds Representing,
Involving, Or Relating To An Unlawful Activity Or Money Laundering Offense Under Republic Act
No. 9160, As Amended.
[80]
Republic Act No. 9160 (2002), Sec. 11.
[81]
See J. Tinga, Concurring and Dissenting, Gonzales v. Abaya, G.R. No. 164007, 10
August 2006, 498 SCRA 445, 501; citing 12 Words and Phrases (1954 ed.), p. 478-479 and 1
BOUVIER'S LAW DICTIONARY (8th ed., 1914), p. 858.
[82]
CONST., Art. III, Sec. 2.
[83]
2000 RULES OF CRIMINAL PROCEDURE, Rule 126, Sec. 5.
[84]
Perhaps the prophecy of Justice Brandeis, dissenting in Olmstead v. U.S.,
227 U.S. 438, 473 (1928), has come to pass: "[T]ime works changes, brings into existence new
conditions and purposes." Subtler and more far-reaching means of invading privacy have
become available to the Government. Discovery and invention have made it possible for the
Government, by means far more effective than stretching upon the rack, to obtain disclosure in
court of what is whispered in the closet…Moreover, "in the application of a constitution, our
contemplation cannot be only of what has, been but of what may be." The progress of science in
furnishing the Government with means of espionage is not likely to stop with wiretapping. Ways
may someday be developed by which the Government, without removing papers from secret
drawers, can reproduce them in court, and by which it will be enabled to expose to a jury the
most intimate occurrences of the home.” Id. at 473-474.

329
[85]
425 U.S. 435 (1976).
[86]
“Even if we direct our attention to the original checks and deposit slips, rather than to
the microfilm copies actually viewed and obtained by means of the subpoena, we perceive no
legitimate "expectation of privacy" in their contents. The checks are not confidential
communications but negotiable instruments to be used in commercial transactions. All of the
documents obtained, including financial statements and deposit slips, contain only information
voluntarily conveyed to the banks and exposed to their employees in the ordinary course of
business. The lack of any legitimate expectation of privacy concerning the information kept in
bank records was assumed by Congress in enacting the Bank Secrecy Act, the expressed
purpose of which is to require records to be maintained because they "have a high degree of
usefulness in criminal, tax, and regulatory investigations and proceedings." Ibid. The passage
by the U.S. Congress in 1978 of the Right to Financial Privacy Act was essentially in reaction to
the Miller ruling. TIROL, supra note 64, at 155.
[87]
See TIROL, supra note 64, citing GABRIEL SINGSON, LAW AND JURISPRUDENCE ON
SECRECY OF BANK DEPOSITS, 46 Ateneo Law Journal 670, 682.
[88]
See Ople v. Torres, 354 Phil. 948 (1998).
[89]
“The right of the people to information on matters of public concern shall be
recognized. Access to official records, and to documents, and papers pertaining to official acts,
transactions, or decisions, as well as to government research data used as basis for policy
development, shall be afforded the citizen, subject to such limitations as may be provided by
law."
[90]
“Subject to reasonable conditions prescribed by law, the State adopts and implements
a policy of full public disclosure of all its transactions involving public interest."
[91]
Chavez v. PCGG, 360 Phil. 133, 161, citing V Record of the Constitutional Commission
25 (1986).
[92]
See Phil. National Bank v. Gancayco, et al., 122 Phil. 503, 506-507 (1965).
[93]
Section 8 of R.A. Act No. 6770, or the Ombudsman Act of 1989 empowers the
Ombudsman to “[a]dminister oaths, issue subpoena and subpoena duces tecum and take
testimony in any investigation or inquiry, including the power to examine and have access to
bank accounts and records.” See Sec. 8, Rep. Act No. 6770 (1989). In Marquez v. Hon. Desierto,
412 Phil. 387 (2001), the Court, interpreted this provision in line with the “absolutely
confidential” nature of bank deposits under the Bank Secrecy Act, infra, and mandated: “there
must be a pending case before a court of competent jurisdiction[;] the account must be clearly
identified, the inspection limited to the subject matter of the pending case before the court of
competent jurisdiction[;] the bank personnel and the account holder must be notified to be
present during the inspection, and such inspection may cover only the account identified in the
pending case.” Id. at 397. With respect to the Ombudsman’s power of inquiry into bank
deposits, Marquez remains good law. See Ejercito v. Sandiganbayan, G.R. Nos. 157294-95, 30
November 2006, 509 SCRA 190, 224 and 226.
[94]
Under Article 267 of the Revised Penal Code.

330
[95]
Particularly Sections 4, 5, 6, 8, 9, 10, 12, 13, 14, 15, and 16 thereof.
[96]
Republic Act No. 1405 (1955), Sec. 2.
[97]
Rollo, p. 98.
[98]
A copy of such certification was attached to Cheng’s Comment as Annex “2”. See id.
at 421.
[99]
CONST., Art. III, Sec. 22.
[100]
In the Matter of the Petition for the Declaration of the Petitioner’s Rights and Duties
under Sec. 8 of R.A. No. 6132, 146 Phil. 429, 431-432 (1970). See also Tan v. Barrios, G.R.
Nos. 85481-82, 18 October 1990, 703.
[101]
Interpretate fienda est ut res valeat quam pereat.
[102]
Rollo, p. 818, citing House Committee Deliberations on 26 September 2001.

331
EN BANC

[G.R. No. 134163-64. December 13, 2000]

MUSLIMIN SEMA, petitioner, vs. COMMISSION ON ELECTIONS and RODEL


MAÑARA, respondents.

[G.R. No. 141249-50. December 13, 2000]

RODEL MAÑARA, petitioner, vs. COMMISSION ON ELECTIONS and MUSLIMIN


SEMA, respondents.

[G.R. No. 141534-35. December 13, 2000]

RODEL MAÑARA, petitioner, vs. COMMISSION ON ELECTIONS and MUSLIMIN


SEMA, respondents.

DECISION
KAPUNAN, J.:

Muslimin Sema*and Rodel Mañara were two (2) of the eleven (11) candidates for city mayor
of Cotabato City during the May 11, 1998 elections.
During the canvassing of the election returns from the three hundred sixty-two (362)
precincts of Cotabato City by the City Board of Canvassers (CBC), numerous petitions for
exclusion of election returns were filed. For his part, Sema objected to thirty (30) election
returns from the following precincts, namely: Precinct Nos. 295A/A1, 274A/275A, 46A2,
262A/263A, 218A/219A, 178A, 255A/256A, 158A/158A1, 214A/214A1, 104A/104A1,
154A/154A1, 92A, 212A/212A1, 109A/109A1, 184A1; 175A1, 168A/168A1, 233A/233A1,
209A/209A1, 121A1, 275A, 198A/198A1, 237A/237A1, 176A, 213A1/213A2, 241A, 167A, 180A,
103A, 264A/265A/266A and thereafter, filed a petition for exclusion of such returns with the CBC
on the ground that the same contained material defects, were allegedly tampered with or
falsified, prepared under duress, threat, coercion, and intimidation, or substituted with
fraudulent ones. If the 30 election returns were to be excluded, Sema and Mañara would obtain
13,338 and 12,484 votes, respectively. Including the 30 election returns, the votes of Sema and
Mañara would be 13,713 and 15,442, respectively.

332
On May 22, 1998, the CBC issued an order dismissing one hundred-sixteen (116) petitions
for exclusion of election returns including the petitions for exclusion filed by Sema with respect
to thirteen (13) of the thirty (30) contested returns he filed.
On May 23, 1998, the CBC issued another order dismissing fifty-five (55) petitions for
exclusion of election returns including Sema’s petitions for exclusion with respect to fifteen (15)
of the remaining seventeen (17) contested returns. In effect then, only two (2) election returns
remained contested.
No appeal was taken from these orders.
Nonetheless, on May 30, 1998, the CBC issued another order dated May 29, 1998, this time
granting Sema’s petition for exclusion of the thirty (30) election returns. A copy of this order
was actually served upon Mañara in the morning of May 31, 1998. On May 30, 1998, counsel
for Mañara, already aware of the existence of the May 29, 1998 order, questioned the illegal
proceedings of the CBC saying that it had previously ruled upon the inclusion of twenty-eight
(28) of said thirty (30) contested returns. In addition, Mañara questioned the composition of the
CBC, the legality of its proceedings and the capacity of the board to act fairly and
judiciously. The latter did not rule on his objection.
Upon the resumption of the canvassing in the evening of May 31, 1998, counsel for Mañara
again called the CBC’s attention to the fact that it had already ruled upon and dismissed the
petition for exclusion of Sema in its Orders dated May 22, and 23, 1998, and which orders had
already become final and executory because no appeal was taken therefrom. But the CBC
ignored the manifestation, explaining that the previous orders did not include Sema’s
objections. Consequently, Mañara’s counsel manifested his intent to appeal from the May 29,
1998 order of the CBC.
On May 31, 1998, Sema and the other winning candidates for the City of Cotabato were
proclaimed by the CBC. Notably, said proclamation was based on the canvass of only three
hundred thirty-two (332) election returns, thirty (30) returns having been excluded from the
total of three hundred sixty-two (362) returns pursuant to the CBC’s Order of May 29, 1998.
On June 2, 1998, Mañara filed his written notice of appeal with the CBC.
On June 5, 1998, Mañara filed his appeal with the Commission on Elections (COMELEC),
docketed as SPC No. 98-240, questioning the exclusion of the thirty (30) election returns in the
canvass and the proceeding of the CBC in promulgating the May 29, 1998 order which he
claimed to be illegal. The appeal was anchored on the following grounds: (1) the CBC exceeded
its authority in its ruling of May 29, 1998 excluding from the canvass the 30 election returns,
considering that the CBC had already dismissed the petition for exclusion in its orders dated May
22 and 23, 1998; (2) the CBC was illegally constituted when it issued its ruling of May 29, 1998
because it was presided by Casan Macadatu, who had already been replaced by Atty. Lintang
Bidol effective May 25, 1998; and (3) the CBC’s proclamation of Sema as Mayor of the City of
Cotabato was invalid as it was only on May 31, 1998 that the CBC completed the canvassing of
362 election returns and it was in the morning of the same day when the Board was not in
session that Mañara was furnished with a copy of the CBC’s questioned ruling of May 29, 1998;
such being the case, it was only at nightfall of May 31, 1998 when the Board held session that
Mañara was able to manifest his intent to appeal from the said ruling.
On June 9, 1998, Mañara filed with the COMELEC a petition for annulment of the
proclamation of Sema which was docketed as SPC No. 98-262.

333
Initially, the hearing of Mañara’s appeal and petition for annulment of proclamation was set
on June 19, 1998 before the COMELEC en banc. However, the hearing was cancelled and the
two (2) cases were referred to the First Division of the COMELEC instead.
Said cases were then heard on June 27, 1998 by the First Division of the COMELEC
afterwhich they were submitted for resolution.
On June 29, 1998, the First Division of the COMELEC issued an order which reads as follows:

Without prejudice to the issuance at a later time of a formal Resolution in these cases, but based
on the pleadings, the evidence adduced by the parties during the hearing on June 27, 1998, and
the facts established therein, the effects and consequences of the proclamation for the position
of City Mayor per Certificate of Canvass of Votes and Proclamation dated May 31, 1998 issued by
the City Board of Canvassers of Cotabato City is HEREBY SUSPENDED.

Respondent Muslimin Sema is directed to cease and desist from taking his oath of office as City
Mayor and/or from discharging the functions of said office.

The Clerk of the Commission is directed to furnish a copy of this order to the Hon. Secretary of
the Department of Interior and Local Government, and Land Bank of the Philippines, thru its
branch at Cotabato City.

The Regional Election Director, Atty. Hector Masna shall serve immediately to the parties a copy
of this Order.[1]

Despite the above order of the COMELEC, Sema assumed the office of the city mayor of
Cotabato and commenced to discharge the functions of said office.
On July 3, 1998, Sema filed a petition for certiorari and prohibition before this Court to annul
the order of the COMELEC dated June 29, 1998 with prayer for the issuance of a temporary
restraining order and/or writ of preliminary injunction directing the COMELEC to desist from
enforcing the questioned order. The case was docketed as G.R. Nos. 134163-64. As grounds
therefor, Sema alleged that:

The pendency of the appeal which was apparently filed out of time and the petition for
annulment of proclamation is not a sufficient basis to enjoin petitioner from discharging the
functions of the contested office especially where, as here, he had already taken his oath of
office and assumed the same in accordance with law.

The evidence submitted by the private respondent before the Comelec proved beyond doubt
that his appeal (SPC No. 98-240) was filed out of time and that he failed to comply with the
requirements of a pre-proclamation controversy.[2]

On July 14, 1998, this Court issued a resolution directing the parties to maintain the status
quo prevailing at the time of the issuance of the order of the COMELEC dated June 29, 1998 in
SPC Nos. 98-240 and 98-262.[3]
On August 17, 1999, this Court directed the COMELEC to resolve SPC Nos. 98-240 and 98-
262 within thirty (30) days from receipt of the resolution and to make a report of the same to
the Court within five (5) days from its promulgation.[4]

334
On September 17, 1999, the COMELEC filed a manifestation and motion for extension of
time to resolve SPC Nos. 98-240 and 98-262 stating that the Commissioner to whom the cases
were raffled to and assigned for writing of the Commission’s opinion was out of the country and
would be back on October 8, 1999. Consequently, it asked for a period of thirty (30) days from
October 8, 1999 or until November 7, 1999 to resolve the said cases.[5]
On October 19, 1999, the Court noted the manifestation and granted the motion for
extension of time to resolve the said cases.[6]
On October 27, 1999, the COMELEC submitted its compliance[7] to the Court’s Resolution of
August 17, 1999 and attached therewith a copy of the resolution of the First Division of the
COMELEC dated October 18, 1999[8] denying due course to SPC No. 98-240 for having been filed
out of time, dismissing SPC No. 98-262 for lack of merit and affirming the proclamation of Sema
as mayor of Cotabato City.
On November 5, 1999, Mañara filed his comment on the COMELEC’s compliance stating,
among other things, that the same is premature since it is only a resolution of the First Division
and not a final resolution of the Commission en banc.[9]
Consequently, on December 7, 1999 this Court issued a resolution directing the COMELEC en
banc to resolve SPC Nos. 98-240 and 98-262 with finality within a non-extendible period of
thirty (30) days from receipt of the resolution and to forthwith make a report thereon to the
Court within five (5) days from the promulgation of the resolution.
On January 19, 2000, respondent COMELEC submitted its compliance and reported that it
issued a resolution[10] on January 2, 2000 denying the motion for reconsideration filed by
Mañara.[11]
On January 24, 2000, Sema filed a manifestation with motion to consider G.R. Nos. 134163-
64 closed and terminated.
On January 27, 2000, Mañara filed a counter-manifestation with motion to resolve the
petition in G.R. Nos. 134163-64 on the merits.
On February 14, 2000, Sema filed a motion for leave of court to file an incorporated reply to
Mañara’s counter-manifestation.
All motions were noted by this Court.
Previously, or on January 13, 2000, Mañara filed a petition for certiorari captioned
“petition ex abundanti cautela”, docketed as G.R. Nos. 141249-50, questioning the COMELEC
Resolutions dated October 18, 1999 and January 2, 2000. He prayed that the proclamation of
Sema as mayor of Cotabato City be annulled and that the COMELEC be ordered to canvass the
thirty (30) election returns excluded by the CBC. This petition, however, was dismissed by the
Court on January 25, 2000 for lack of a verified statement on material dates. An addendum to
said petition was noted without action by the Court on February 8, 1999.[12]
On February 7, 2000, Mañara seasonably[13] filed a petition for certiorari, docketed as G.R.
Nos. 141534-35.[14] Aside from questioning the non-observance of the COMELEC of its own Rules
of Procedure, the petition challenges the illegal proclamation of Sema which proceeded from the
illegal proceedings of the CBC in excluding thirty (30) election returns in the canvassing of votes
for mayor in the City of Cotabato when it had earlier ruled for the inclusion of twenty-eight (28)
of said returns. It ascribes to the COMELEC the following errors, viz:
A
335
WHETHER OR NOT THE COMMISSION ACTED WITH JURISDICTION OR SOUND DISCRETION,
OR WITHOUT OR IN EXCESS OF JURISDICTION, OR WITH GRAVE ABUSE OF DISCRETION
AMOUNTING TO LACK OR IN EXCESS OF DISCRETION IN ALLOWING ONE AND THE SAME
COMMISSIONER AS PONENTE FOR BOTH THE CHALLENGED RESOLUTIONS;

WHETHER OR NOT THE COMMISSION ACTED WITH JURISDICTION OR SOUND DISCRETION,


OR WITHOUT OR IN EXCESS OF JURISDICTION, OR WITH GRAVE ABUSE OF DISCRETION
AMOUNTING TO LACK OR IN EXCESS OF JURISDICTION IN ALLOWING A COMMISSIONER
OF THE SECOND DIVISION TO SIGN AS MEMBER OF THE FIRST DIVISION ON THE
CHALLENGED SPLIT RESOLUTION, ANNEX "A" HEREOF, NOTWITHSTANDING THE FACT THAT
THERE WAS NO VACANCY IN THE COMPOSITION OF THE FIRST DIVISION;

WHETHER OR NOT THE COMMISSION ACTED WITH SOUND DISCRETION OR WITH GRAVE
ABUSE OF DISCRETION AMOUNTING TO LACK OR IN EXCESS OF JURISDICTION IN NOT
PRIORLY RESOLVING THE CRISES IN QUORUM OF THE COMMISSION EN BANC SO AS TO
ACCORD TO PETITIONER THE EQUAL PROTECTION OF THE LAW;

WHETHER OR NOT THE COMMISSION ACTED WITH SOUND DISCRETION OR WITH GRAVE
ABUSE OF DISCRETION AMOUNTING TO LACK OR IN EXCESS OF JURISDICTION IN BY-
PASSING IN ITS CHALLENGED SPLIT RESOLUTION THE ISSUES RAISED BY PETITIONER IN
HIS SPC NO. 98-240 AND SPC NO. 98-262;

WHETHER OR NOT THE COMMISSION ACTED WITH SOUND DISCRETION OR WITH GRAVE
ABUSE OF DISCRETION AMOUNTING TO LACK OR IN EXCESS OF JURISDICTION IN
DISTORTING THE MATERIAL FACTS OF THE CASE, WHICH THE HONORABLE COURT, IN ITS
RESOLUTION OF AUGUST 17, 1999, HAS ALREADY ASCERTAINED;

WHETHER OR NOT THE COMMISSION ACTED WITH JURISDICTION OR SOUND DISCRETION,


OR WITHOUT OR IN EXCESS OF JURISDICTION, OR WITH GRAVE ABUSE OF DISCRETION
AMOUNTING TO LACK OR IN EXCESS OF JURISDICTION IN UPHOLDING THE BOARD IN ITS
ILLEGAL PROCLAMATION OF PRIVATE RESPONDENT MUSLIMEN SEMA AS THE ELECTED
MAYOR OF COTABATO CITY WHICH PETITIONER IN SPC NO. 98-240 AND SPC NO. 98-262
CHALLENGED AND QUESTIONED FOR BEING NULL AND VOID AB INITIO.

WHETHER OR NOT THE COMMISSION ACTED WITH SOUND DISCRETION OR WITH GRAVE
ABUSE OF DISCRETION AMOUNTING TO LACK OR IN EXCESS OF JURISDICTION IN NOT
GIVING DUE COURSE TO PETITIONER'S SPC NO. 98-240 AND SPC NO. 98-262, ON THE
336
TECHNICALITY GROUND THAT THE SAME WERE ALLEGEDLY FILED OUT OF TIME, WHICH IS
NOT IN ACCORD WITH LAW AND CONTRARY TO THE SETTLED DECISION OF THE
HONORABLE COURT.[15]

I
The pivotal issue in this case is whether or not the order of the CBC of Cotabato City dated
May 29, 1998 granting Sema’s 28 petitions for exclusion of the 30 contested election returns is
null and void for having been issued after its earlier ruling embodied in its orders of May 22 and
23, 1998 directing the exclusion of the same returns had already become final.
It may be recalled that on May 22, 1998, the CBC issued an order dismissing 13 of the 30
petitions for exclusion filed by Sema. On May 23, 1998, the CBC issued another order
dismissing 15 of the remaining 17 petitions for exclusion he filed. He did not appeal from these
orders within the reglementary period, consequently, the same already became final. However,
the CBC issued another order on May 29, 1998, this time granting Sema’s petitions for exclusion
of 30 election returns, among which were the 28 election returns already ordered included for
canvass.
It was blatantly absurd for the CBC to rationalize that the May 22 and 23, 1998 orders
dismissing the petitions for exclusions refer only to candidates Guiani’s and Leyretana’s petitions
and not Sema’s. The wordings of the May 23, 1998 order is plain and unequivocal. It says: “all
petitions/cases against the hereunder contested precincts are hereby being DISMISSED for lack
of merit xxx.” If all petitions/cases were dismissed, then, these necessarily included Sema’s
petition. Furthermore, there was nothing in the aforementioned orders which would indicate
that the CBC reserved its right to rule on Sema’s petition at a later time. Neither do the minutes
of the board even intimate such a reservation.
II
Even assuming arguendo that the orders of the CBC of May 22 and 23, 1998 had not
become final and executory, we are not persuaded by the COMELEC’s pronouncement that
Mañara belatedly filed his appeal from the May 29, 1998 ruling of the CBC on June 5,
1998. According to the COMELEC's First Division in its Resolution dated October 18, 1999:

Records show that the ruling which aggrieved appellant was issued on or about 4:00 p.m. of May
30, 1998. Following the instructions of the provisions above-cited, Mañara had, until 4:00 p.m.
of June 1, 1998 to file his notice of appeal with the Board. As it was, he filed said notice only on
June 2, 1998. In this regard alone, appellant had already committed a procedural lapse. He
aggravated his errors when he filed his appeal before the Commission on June 5, 1998, a full
day beyond the 5-day reglementary period. The law, we have to stress, specifically ruled out
any extension of the five-day period. It is most unfortunate that in committing not only one but
two fatal lapses, appellant disregarded a procedure which according to COMELEC Resolution No.
2962 “is mandatory and shall be strictly observed by the Board of Canvassers”. It cost him his
appeal because the same had, for all intents and purposes prescribed. The May 30, 1998 ruling
of the City Board of Canvassers of Cotabato City, not having been seasonably questioned can no
longer be disturbed.[16]

It would appear that the May 29, 1998 ruling of the CBC was received by Mañara only on
May 31, 1998 which was the same date the CBC declared that it had completed the canvassing
of 362 returns. It was also in the evening of May 31, 1998 while the CBC was in session that

337
Mañara manifested his intent to appeal from said ruling. The appeal was therefore filed with the
COMELEC on June 5, 1998 within the period prescribed in Section 20 (e) and (f) of R.A. No.
7166. [17]
Further assuming that the reckoning date for appeal was May 30, 1998 and not May 31,
1998, it bears stressing that the petition brought by Mañara to the COMELEC on June 5, 1998,
docketed as SPC No. 98-240, in effect challenged the composition of the CBC and the legality of
its proceedings. If such be the situation, the proceedings would be governed by Section 19 of
R.A. No. 7166 and Section 8, Rule 27 of the COMELEC Rules of Procedure, to wit:
Section 19 of R.A. No. 7166 reads:

SEC. 19. Contested Composition or Proceedings of the Board; Period to Appeal; Decision by the
Commission.—Parties adversely affected by a ruling of the board of canvassers on questions
affecting the composition or proceedings of the board may appeal the matter to the Commission
within three (3) days from a ruling thereon. The Commission shall summarily decide the case
within five (5) days from the filing thereof.

Section 8, Rule 27 of the COMELEC Rules of Procedure provides:

SEC. 8. Procedure Before the Board of Canvassers When Composition or Proceedings of Board
are Contested.— (a) When the composition or proceeding of the board of canvassers are
contested, the board of canvassers shall, within twenty-four (24) hours, make a ruling thereon
with notice to the contestant who, if adversely affected, may appeal the matter to the
Commission within three (3) days after the ruling with proper notice to the board of
canvassers. The Commission en banc shall summarily decide the case within five (5) days from
the filing thereof.

(b) Upon receipt of such appeal, the Clerk of Court concerned shall immediately set the case for
hearing, with due notice to the parties, by the Commission en banc.

(c) During the pendency of the appeal, the board of canvassers shall immediately suspend the
canvass until the Commission orders the continuation or resumption thereof.

Pursuant to the foregoing provisions, the party adversely affected by a ruling of the board
must take an appeal within three (3) days from the date of the ruling. In this case, the facts
would suggest that the CBC adjourned its proceedings on May 30 and 31, 1998 without making
any ruling on Mañara’s objections to the CBC’s proceedings. When Mañara filed his appeal in
SPC No. 98-240 on June 5, 1998, it cannot be correctly argued that the 3-day period set by law
for its submission had expired because the CBC never ruled on his objections to the board’s
proceedings. The failure or refusal of the CBC to rule on Mañara’s objections should not prevent
his right to elevate the matter to the COMELEC for proper review.[18] On this score, we find
persuasive and logical the dissent of Commissioner Teresita Dy-Liacco Flores to the First
Division’s ruling, thus:

Mañara filed the present Appeal on 5 June 1998. Whether it is within the three day period to
file, nobody knows, because the Board never issued any ruling from 30 May 1998 when the
proceedings of the board was challenged up to the time it adjourned on 31 May 1998. It never
made a ruling at all even after that. The board’s adjournment without making any written and
express ruling thereon means that the Board has not complied with its duty to rule thereon. The
338
absence of any ruling makes it impossible for Mañara to file his appeal within the prescribed
period because there was no ruling to appeal from in the first place. The absence of compliance
of the duty by the board makes it legally unjustifiable for this Commission to dismiss the present
appeal because the three-day period within which to appeal must be counted from the time the
ruling was made which in the case at bar is absent.

Mañara has every right to expect a ruling from the Board on its objection over the latter’s
proceedings. Up to this time, however, the Board has not complied with its statutory
responsibility to come up with a ruling thereon. The failure of the Board to discharge this
obligation should not in any way prejudice Mañara’s right to elevate the matter to this
Commission on appeal. Otherwise, all that a partial board can do to favor a party is to refuse to
make a ruling on the latter’s opponent’s objections effectively preventing its review by this
Commission. (Abella vs. Larrazabal 180 SCRA 509). It is in this light that the instant appeal
must be considered seasonably filed. This Commission must assume jurisdiction, entertain the
allegations raised and resolve the issues involved in SPC No. 98-240.[19]

It is clear that the CBC acted without authority when it issued its May 29, 1998
ruling. Consequently, the COMELEC acted without or in excess of its jurisdiction and with grave
abuse of discretion when it rendered the questioned resolution of October 18, 1999 denying due
course to SPC No. 98-240 for allegedly having been filed out of time and affirming the
proclamation of Sema as Mayor of Cotabato City; and the resolution of January 2, 2000 denying
Mañara’s motion for reconsideration of the October 18, 1999 resolution.
Accordingly, the proclamation of Sema is null and void as it was based on an incomplete
canvass. An incomplete canvass is illegal and cannot be the basis of a valid proclamation.[20] A
proclamation made where the contested returns set aside will affect the result of the election
and the board of canvassers proceeded to proclaim without the authority from the COMELEC is
null and void.[21]
WHEREFORE, the petition for certiorari, docketed as G.R. Nos. 141534-35 is hereby
GRANTED. The Resolutions of the Commission on Elections dated October 18, 1999 and January
2, 2000 are hereby REVERSED and SET ASIDE. The COMELEC is ORDERED to direct the City
Board of Canvassers of Cotabato City to reconvene within ten (10) days from receipt of this
decision for the purpose of completing the canvass of votes and proclaiming the winner. The
petition for certiorari in G.R. Nos. 134163-64 is deemed CLOSED and TERMINATED. The status
quo order dated July 14, 1998 is hereby ordered LIFTED.
SO ORDERED.
Davide, Jr., C.J., Bellosillo, Melo, Puno, Vitug, Mendoza, Panganiban, Quisumbing, Buena,
Gonzaga-Reyes Santiago, and De Leon, Jr., JJ., concur.
Pardo J., no part. was Comelec chair at the time.

*
Sometimes referred to in the pleadings and/or COMELEC resolutions as Muslimen
Sema or Muslemin Sema.

339
[1]
Rollo of G.R. Nos. 134163-64, pp. 24-25.
[2]
Id., at 12 and 14.
[3]
Id., at 124.
[4]
Id., at 425.
[5]
Id., at 429-430.
[6]
Id., at 432.
[7]
Id., at 438-440.
[8]
Id., at 443-471. Commissioners Manolo B. Gorospe and Luzviminda G. Tancangco voted to
deny due course to SPC No. 98-240 and to dismiss SPC No. 98-262 for lack of merit while
Commissioner Teresita Dy-Liacco Flores dissented.
[9]
Id., at 472-477.
[10]
The voting was as follows:
(f) Commissioners Manolo B. Gorospe and Luzviminda G. Tancangco voted to deny the MR;
(f) Chairperson Harriet O. Demetriou and Commissioner Teresita Dy-Liacco Flores dissented;
while
(f) Commissioners Julio Desamito and Japal Guiani inhibited themselves.
[11]
Id., at 521-527.
[12]
Rollo of G.R. Nos. 141249-50, pp. 3-108.
[13]
Mañara received a copy of the COMELEC resolution denying his motion for reconsideration
on January 7, 2000. Consequently, he had until February 6, 2000 to file a petition for
certiorari. Since February 6, 2000 fell on a Sunday, he had until February 7, 2000 to do so, and
he did. He filed the petition docketed as G.R. Nos. 141534-35 on February 7, 2000.
[14]
Rollo of G.R. Nos. 141534-35, pp. 3-62.
[15]
Id., at 21-23.
[16]
Id., at 66.
[17]
Sec. 20. Procedure in Disposition of Contested Election Returns.-
xxx
(e) Any party adversely affected by the ruling of the board shall immediately inform the board if
he intends to appeal said ruling. The board shall enter said information in the minutes of the
canvass, set aside the returns and proceed to consider the other returns.
(f) After all the uncontested returns have been canvassed and the contested returns ruled upon
by it, the board shall suspend the canvass. Within forty-eight (48) hours therefrom, any party
adversely affected by the ruling may file with the board a written and verified notice of appeal;
and within an unextendible period of five (5) days thereafter, an appeal may be taken to the
Commission.
xxx.

340
[18]
Abella v. Larrazabal, 180 SCRA 509 (1989).
[19]
Id., at 131-132.
[20]
Jamil v. COMELEC, 283 SCRA 349 (1997).
[21]
Sections 238 and 245, Batas Pambansa Blg. 881; Vide: Jamil v. COMELEC, supra.

341
Republic of the Philippines
Supreme Court
Manila
---

EN BANC

ROMULO L. NERI, G.R. No. 180643


Petitioner,

Present:

- versus - PUNO, C.J.,

QUISUMBING,

YNARES-SANTIAGO,

SENATE COMMITTEE ON CARPIO,


ACCOUNTABILITY OF PUBLIC
OFFICERS AND AUSTRIA-MARTINEZ,
INVESTIGATIONS, SENATE
COMMITTEE ON TRADE AND CORONA,
COMMERCE, AND SENATE
COMMITTEE ON NATIONAL CARPIO MORALES,
DEFENSE AND SECURITY,
AZCUNA,
Respondents.
TINGA,

CHICO-NAZARIO,

VELASCO, JR.,

NACHURA,

REYES,

LEONARDO-DE CASTRO, and

BRION, JJ.

342
Promulgated:

March 25, 2008

x----------------------------------------------------------------------------------------------------------
----------x

DECISION

LEONARDO-DE CASTRO, J.:

At bar is a petition for certiorari under Rule 65 of the Rules of Court assailing the show
[1]
cause Letter dated November 22, 2007 and contempt Order[2] dated January 30,
2008 concurrently issued by respondent
Senate Committees on Accountability of Public Officers and Investigations,[3] Trade
and Commerce,[4] and National Defense and Security[5] against petitioner Romulo L.
Neri, former Director General of the National Economic and Development Authority (NEDA).

The facts, as culled from the pleadings, are as follows:

On April 21, 2007, the Department of Transportation and Communication (DOTC)


entered into a contract with Zhong Xing Telecommunications Equipment (ZTE) for the supply of
equipment and services for the National Broadband Network (NBN) Project in the amount of U.S.
$ 329,481,290 (approximately P16 Billion Pesos). The Project was to be financed by the
People’s Republic of China.

In connection with this NBN Project, various Resolutions were introduced in the Senate,
as follows:

343
(1) P.S. Res. No. 127, introduced by Senator Aquilino Q. Pimentel, Jr.,
entitled RESOLUTION DIRECTING THE BLUE RIBBON COMMITTEE AND THE
COMMITTEE ON TRADE AND INDUSTRY TO INVESTIGATE, IN AID OF LEGISLATION,
THE CIRCUMSTANCES LEADING TO THE APPROVAL OF THE BROADBAND
CONTRACT WITH ZTE AND THE ROLE PLAYED BY THE OFFICIALS CONCERNED IN
GETTING IT CONSUMMATED AND TO MAKE RECOMMENDATIONS TO HALE TO THE
COURTS OF LAW THE PERSONS RESPONSIBLE FOR ANY ANOMALY IN CONNECTION
THEREWITH AND TO PLUG THE LOOPHOLES, IF ANY IN THE BOT LAW AND OTHER
PERTINENT LEGISLATIONS.

(2) P.S. Res. No. 144, introduced by Senator Mar Roxas, entitled Á
RESOLUTION URGING PRESIDENT GLORIA MACAPAGAL ARROYO TO DIRECT THE
CANCELLATION OF THE ZTE CONTRACT

(3) P.S. Res. No. 129, introduced by Senator Panfilo M. Lacson, entitled
RESOLUTION DIRECTING THE COMMITTEE ON NATIONAL DEFENSE AND SECURITY
TO CONDUCT AN INQUIRY IN AID OF LEGISLATION INTO THE NATIONAL SECURITY
IMPLICATIONS OF AWARDING THE NATIONAL BROADBAND NETWORK CONTRACT
TO THE CHINESE FIRM ZHONG XING TELECOMMUNICATIONS EQUIPMENT
COMPANY LIMITED (ZTE CORPORATION) WITH THE END IN VIEW OF PROVIDING
REMEDIAL LEGISLATION THAT WILL PROTECT OUR NATIONAL SOVEREIGNTY,
SECURITY AND TERRITORIAL INTEGRITY.

(4) P.S. Res. No. 136, introduced by Senator Miriam Defensor Santiago,
entitled RESOLUTION DIRECTING THE PROPER SENATE COMMITTEE TO CONDUCT
AN INQUIRY, IN AID OF LEGISLATION, ON THE LEGAL AND ECONOMIC
JUSTIFICATION OF THE NATIONAL BROADBAND NETWORK (NBN) PROJECT OF THE
NATIONAL GOVERNMENT.

At the same time, the investigation was claimed to be relevant to the consideration of
three (3) pending bills in the Senate, to wit:

1. Senate Bill No. 1793, introduced by Senator Mar Roxas, entitled AN ACT
SUBJECTING TREATIES, INTERNATIONAL OR EXECUTIVE AGREEMENTS
INVOLVING FUNDING IN THE PROCUREMENT OF INFRASTRUCTURE PROJECTS,
GOODS, AND CONSULTING SERVICES TO BE INCLUDED IN THE SCOPE AND
APPLICATION OF PHILIPPINE PROCUREMENT LAWS, AMENDING FOR THE
PURPOSE REPUBLIC ACT NO. 9184, OTHERWISE KNOWN AS THE GOVERNMENT
PROCUREMENT REFORM ACT, AND FOR OTHER PURPOSES;

2. Senate Bill No. 1794, introduced by Senator Mar Roxas, entitled AN ACT
IMPOSING SAFEGUARDS IN CONTRACTING LOANS CLASSIFIED AS OFFICIAL
DEVELOPMENT ASSISTANCE, AMENDING FOR THE PURPOSE REPUBLIC ACT NO.
8182, AS AMENDED BY REPUBLIC ACT NO. 8555, OTHERWISE KNOWN AS THE
OFFICIAL DEVELOPMENT ASSISTANCE ACT OF 1996, AND FOR OTHER
PURPOSES; and

344
3. Senate Bill No. 1317, introduced by Senator Miriam Defensor Santiago,
entitled AN ACT MANDATING CONCURRENCE TO INTERNATIONAL AGREEMENTS
AND EXECUTIVE AGREEMENTS.

Respondent Committees initiated the investigation by sending invitations to certain


personalities and cabinet officials involved in the NBN Project. Petitioner was among those
invited. He was summoned to appear and testify on September 18, 20, and 26 and October 25,
2007. However, he attended only the September 26 hearing, claiming he was “out of town”
during the other dates.

In the September 18, 2007 hearing, businessman Jose de Venecia III testified that
several high executive officials and power brokers were using their influence to push the
approval of the NBN Project by the NEDA. It appeared that the Project was initially approved as
a Build-Operate-Transfer (BOT) project but, on March 29, 2007, the NEDA acquiesced to convert
it into a government-to-government project, to be financed through a loan from the Chinese
Government.

On September 26, 2007, petitioner testified before respondent Committees for eleven
(11) hours. He disclosed that then Commission on Elections (COMELEC) Chairman Benjamin
Abalos offered him P200 Million in exchange for his approval of the NBN Project. He
further narrated that he informed President Arroyo about the bribery attempt and that she
instructed him not to accept the bribe. However, when probed further on what they discussed
about the NBN Project, petitioner refused to answer, invoking “executive privilege”. In particular,
he refused to answer the questions on (a) whether or not President Arroyo followed up the NBN
Project,[6] (b) whether or not she directed him to prioritize it,[7] and (c) whether or not she
directed him to approve.[8]

Unrelenting, respondent Committees issued a Subpoena Ad Testificandum to petitioner,


requiring him to appear and testify on November 20, 2007.

However, in the Letter dated November 15, 2007, Executive Secretary Eduardo R. Ermita
requested respondent Committees to dispense with petitioner’s testimony on the ground
of executive privilege. The pertinent portion of the letter reads:

With reference to the subpoena ad testificandum issued to Secretary


Romulo Neri to appear and testify again on 20 November 2007 before the Joint
Committees you chair, it will be recalled that Sec. Neri had already testified and
exhaustively discussed the ZTE / NBN project, including his conversation with the
President thereon last 26 September 2007.

345
Asked to elaborate further on his conversation with the President, Sec. Neri
asked for time to consult with his superiors in line with the ruling of the Supreme
Court in Senate v. Ermita, 488 SCRA 1 (2006).

Specifically, Sec. Neri sought guidance on the possible invocation of


executive privilege on the following questions, to wit:

a) Whether the President followed up the (NBN) project?


b) Were you dictated to prioritize the ZTE?
c) Whether the President said to go ahead and approve the project after
being told about the alleged bribe?

Following the ruling in Senate v. Ermita, the foregoing questions fall under
conversations and correspondence between the President and public officials which
are considered executive privilege (Almonte v. Vasquez, G.R. 95637, 23 May
1995; Chavez v. PEA, G.R. 133250, July 9, 2002). Maintaining the confidentiality
of conversations of the President is necessary in the exercise of her executive and
policy decision making process. The expectation of a President to the
confidentiality of her conversations and correspondences, like the value which we
accord deference for the privacy of all citizens, is the necessity for protection of the
public interest in candid, objective, and even blunt or harsh opinions in Presidential
decision-making. Disclosure of conversations of the President will have a chilling
effect on the President, and will hamper her in the effective discharge of her duties
and responsibilities, if she is not protected by the confidentiality of her
conversations.

The context in which executive privilege is being invoked is that the


information sought to be disclosed might impair our diplomatic as well as economic
relations with the People’s Republic of China. Given the confidential nature in which
these information were conveyed to the President, he cannot provide the
Committee any further details of these conversations, without disclosing the very
thing the privilege is designed to protect.

In light of the above considerations, this Office is constrained to invoke the


settled doctrine of executive privilege as refined in Senate v. Ermita, and has
advised Secretary Neri accordingly.

Considering that Sec. Neri has been lengthily interrogated on the subject in
an unprecedented 11-hour hearing, wherein he has answered all questions
propounded to him except the foregoing questions involving executive privilege, we
therefore request that his testimony on 20 November 2007 on the ZTE / NBN
project be dispensed with.

On November 20, 2007, petitioner did not appear before respondent Committees. Thus,
on November 22, 2007, the latter issued the show cause Letter requiring him to explain why he
should not be cited in contempt. The Letter reads:

346
Since you have failed to appear in the said hearing, the Committees on
Accountability of Public Officers and Investigations (Blue Ribbon), Trade and
Commerce and National Defense and Security require you to show cause why you
should not be cited in contempt under Section 6, Article 6 of the Rules of the
Committee on Accountability of Public Officers and Investigations (Blue Ribbon).

The Senate expects your explanation on or before 2 December 2007.

On November 29, 2007, petitioner replied to respondent Committees, manifesting that it


was not his intention to ignore the Senate hearing and that he thought the only remaining
questions were those he claimed to be covered by executive privilege, thus:

It was not my intention to snub the last Senate hearing. In fact, I have
cooperated with the task of the Senate in its inquiry in aid of legislation as shown
by my almost 11 hours stay during the hearing on 26 September 2007. During
said hearing, I answered all the questions that were asked of me, save for those
which I thought was covered by executive privilege, and which was confirmed by
the Executive Secretary in his Letter 15 November 2007. In good faith, after that
exhaustive testimony, I thought that what remained were only the three questions,
where the Executive Secretary claimed executive privilege. Hence, his
request that my presence be dispensed with.

Be that as it may, should there be new matters that were not yet taken up
during the 26 September 2007 hearing, may I be furnished in advance as to what
else I need to clarify, so that as a resource person, I may adequately prepare
myself.

In addition, petitioner submitted a letter prepared by his counsel, Atty. Antonio R.


Bautista, stating, among others that: (1)his (petitioner) non-appearance was upon the order of
the President; and (2) his conversation with President Arroyo dealt with delicate and sensitive
national security and diplomatic matters relating to the impact of the bribery scandal involving
high government officials and the possible loss of confidence of foreign investors and lenders in
the Philippines. The letter ended with a reiteration of petitioner’s request that he “be furnished
in advance” as to what else he needs to clarify so that he may adequately prepare for the
hearing.

In the interim, on December 7, 2007, petitioner filed with this Court the present petition
for certiorari assailing the show cause Letter dated November 22, 2007.

Respondent Committees found petitioner’s explanations unsatisfactory. Without


responding to his request for advance notice of the matters that he should still clarify, they
issued the Order dated January 30, 2008, citing him in contempt of respondent Committees and
ordering his arrest and detention at the Office of the Senate Sergeant-At-Arms until such time
that he would appear and give his testimony. The said Order states:

ORDER
347
For failure to appear and testify in the Committee’s hearing on Tuesday,
September 18, 2007; Thursday, September 20, 2007; Thursday, October 25, 2007;
and Tuesday, November 20, 2007, despite personal notice and Subpoenas Ad
Testificandum sent to and received by him, which thereby delays, impedes and
obstructs, as it has in fact delayed, impeded and obstructed the inquiry into the
subject reported irregularities, AND for failure to explain satisfactorily why he
should not be cited for contempt (Neri letter of 29 November 2007), herein
attached) ROMULO L. NERI is hereby cited in contempt of this (sic) Committees and
ordered arrested and detained in the Office of the Senate Sergeant-At-Arms until
such time that he will appear and give his testimony.

The Sergeant-At-Arms is hereby directed to carry out and implement this


Order and make a return hereof within twenty four (24) hours from its
enforcement.

SO ORDERED.

On the same date, petitioner moved for the reconsideration of the above Order.[9] He
insisted that he has not shown “any contemptible conduct worthy of contempt and arrest.” He
emphasized his willingness to testify on new matters, however, respondent Committees did not
respond to his request for advance notice of questions. He also mentioned the petition
forcertiorari he filed on December 7, 2007. According to him, this should restrain respondent
Committees from enforcing the show cause Letter “through the issuance of declaration of
contempt” and arrest.

In view of respondent Committees’ issuance of the contempt Order, petitioner filed on


February 1, 2008 a Supplemental Petition for Certiorari (With Urgent Application for
TRO/Preliminary Injunction), seeking to restrain the implementation of the said contempt Order.

On February 5, 2008, the Court issued a Status Quo Ante Order (a) enjoining
respondent Committees from implementing their contempt Order, (b) requiring the parties to
observe the status quo prevailing prior to the issuance of the assailed order, and (c) requiring
respondent Committees to file their comment.

Petitioner contends that respondent Committees’ show cause Letter and


contempt Order were issued with grave abuse of discretion
amounting to lack or excess of jurisdiction. He stresses that his conversations with
President Arroyo are “candid discussions meant to explore options in making policy decisions.”
According to him, these discussions “dwelt on the impact of the bribery scandal involving high
government officials on the country’s diplomatic relations and economic and military affairs and
the possible loss of confidence of foreign investors and lenders in thePhilippines.” He also
emphasizes that his claim of executive privilege is upon the order of the President and within the
348
parameters laid down in Senate v. Ermita[10] and United States v. Reynolds.[11] Lastly, he argues
that he is precluded from disclosing communications made
to him in official confidence under Section 7[12] of Republic Act No. 6713,
otherwise known as Code of Conduct and Ethical Standards for Public Officials and
Employees, and Section 24[13] (e) of Rule 130 of the Rules of Court.

Respondent Committees assert the contrary. They argue that (1) petitioner’s
testimony is material and pertinent in the investigation conducted in aid of legislation; (2) there
is no valid justification for petitioner to claim executive privilege; (3)there is no abuse of their
authority to order petitioner’s arrest; and (4) petitioner has not come to court with clean
hands.

In the oral argument held last March 4, 2008, the following issues were ventilated:

1. What communications between the President and petitioner Neri are covered
by the principle of ‘executive privilege’?

1.a Did Executive Secretary Ermita correctly invoke the principle of executive
privilege, by order of the President, to cover (i) conversations
of the President in the exercise of her executive and policy decision-making
and (ii) information, which might impair our diplomatic as well as economic
relations with the People’s Republic of China?

1.b. Did petitioner Neri correctly invoke executive privilege to avoid testifying on
his conversations with the President on the NBN contract on his assertions
that the said conversations “dealt with delicate and sensitive national
security and diplomatic matters relating to the impact of bribery scandal
involving high government officials and the possible loss of confidence of
foreign investors and lenders in the Philippines” x x x within the principles
laid down in Senate v. Ermita (488 SCRA 1 [2006])?

1.c Will the claim of executive privilege in this case violate the
following provisions of the Constitution:

Sec. 28, Art. II (Full public disclosure of all transactions involving public
interest)

349
Sec. 7, Art. III (The right of the people to information on matters of public
concern)

Sec. 1, Art. XI (Public office is a public trust)

Sec. 17, Art. VII (The President shall ensure that the laws be faithfully
executed)

and the due process clause and the principle of separation of powers?

2. What is the proper procedure to be followed in invoking executive privilege?

3. Did the Senate Committees gravely abuse their discretion in ordering the
arrest of petitioner for non-compliance with the subpoena?

After the oral argument, the parties were directed to manifest to the Court within twenty-
four (24) hours if they are amenable to the Court’s proposal of allowing petitioner to
immediately resume his testimony before the Senate Committees to answer the other questions
of the Senators without prejudice to the decision on the merits of this pending petition. It was
understood that petitioner may invoke executive privilege in the course of the Senate
Committees proceedings, and if the respondent Committees disagree thereto, the unanswered
questions will be the subject of a supplemental pleading to be resolved along with the three (3)
questions subject of the present petition.[14] At the same time, respondent Committees were
directed to submit several pertinent documents.[15]

The Senate did not agree with the proposal for the reasons stated in the Manifestation
dated March 5, 2008. As to the required documents, the Senate and respondent
Committees manifested that they would not be able to submit the latter’s “Minutes of all
meetings” and the “Minute Book” because it has never been the “historical and traditional

350
legislative practice to keep them.”[16] They instead submitted the Transcript of Stenographic
Notes of respondent Committees’ joint public hearings.

On March 17, 2008, the Office of the Solicitor General (OSG) filed a Motion for Leave to
Intervene and to Admit Attached Memorandum, founded on the following arguments:

(1) The communications between petitioner and the President are covered by the
principle of “executive privilege.”

(2) Petitioner was not summoned by respondent Senate Committees in


accordance with the law-making body’s power to conduct inquiries in aid of
legislation as laid down in Section 21, Article VI of the Constitution and Senate
v. Ermita.

(3) Respondent Senate Committees gravely abused its discretion for alleged non-
compliance with the Subpoena dated November 13, 2007.

The Court granted the OSG’s motion the next day, March 18, 2008.

As the foregoing facts unfold, related events transpired.

On March 6, 2008, President Arroyo issued Memorandum Circular No. 151, revoking
Executive Order No. 464 and Memorandum Circular No. 108. She advised executive officials
and employees to follow and abide by the Constitution, existing laws and jurisprudence,
including, among others, the case of Senate v. Ermita[17] when they are invited to legislative
inquiries in aid of legislation.

At the core of this controversy are the two (2) crucial queries, to wit:

First, are the communications elicited by the subject three (3) questions covered by
executive privilege?
351
And second, did respondent Committees commit grave abuse of discretion in issuing the
contempt Order?

We grant the petition.

At the outset, a glimpse at the landmark case of Senate v. Ermita[18] becomes

imperative. Senate draws in bold strokes the distinction between

the legislative and oversight powers of the Congress, as embodied under Sections 21 and 22,

respectively, of Article VI of the Constitution, to wit:

SECTION 21. The Senate or the House of Representatives or any of

its respective committees may conduct inquiries in aid of legislation in

accordance with its duly published rules of procedure. The rights of persons

appearing in or affected by such inquiries shall be respected.

SECTION 22. The heads of department may upon their own initiative, with

the consent of the President, or upon the request of either House, or as the rules of

each House shall provide, appear before and be heard by such House on any matter

pertaining to their departments. Written questions shall be submitted to the

President of the Senate or the Speaker of the House of Representatives at least

three days before their scheduled appearance. Interpellations shall not be limited

to written questions, but may cover matters related thereto. When the security of

the state or the public interest so requires and the President so states in writing,

the appearance shall be conducted in executive session.

352
Senate cautions that while the above provisions are closely related and complementary to
each other, they should not be considered as pertaining to the same power of
Congress. Section 21 relates to the power to conduct inquiries in aid of legislation. Its aim is
to elicit information that may be used for legislation. On the other hand, Section 22 pertains to
the power to conduct a question hour, the objective of which is to obtain information in pursuit
of Congress’ oversight function.[19] Simply stated, while both powers allow Congress or any of
its committees to conduct inquiry, their objectives are different.

This distinction gives birth to another distinction with regard to the use of compulsory
process. Unlike in Section 21, Congress cannot compel the appearance of executive officials
under Section 22. The Court’s pronouncement in Senate v. Ermita[20] is clear:

When Congress merely seeks to be informed on how department heads are

implementing the statutes which it has issued, its right to such information is not as

imperative as that of the President to whom, as Chief Executive, such department

heads must give a report of their performance as a matter of duty. In such

instances, Section 22, in keeping with the separation of powers, states that

Congress may only requesttheir appearance. Nonetheless, when the inquiry in

which Congress requires their appearance is ‘in aid of legislation’ under Section 21,

the appearance is mandatory for the same reasons stated in Arnault.

In fine, the oversight function of Congress may be facilitated by compulsory

process only to the extent that it is performed in pursuit of legislation. This is

consistent with the intent discerned from the deliberations of the Constitutional

Commission

353
Ultimately, the power of Congress to compel the appearance of executive

officials under section 21 and the lack of it under Section 22 find their basis in the

principle of separation of powers. While the executive branch is a co-equal branch of

the legislature, it cannot frustrate the power of Congress to legislate by refusing to

comply with its demands for information. (Emphasis supplied.)

The availability of the power of judicial review to resolve the issues raised in this case has
also been settled in Senate v. Ermita, when it held:

As evidenced by the American experience during the so-called “McCarthy

era,” however, the right of Congress to conduct inquiries in aid of legislation is, in

theory, no less susceptible to abuse than executive or judicial power. It may thus be

subjected to judicial review pursuant to the Court’s certiorari powers under Section

1, Article VIII of the Constitution.

Hence, this decision.

The Communications Elicited by the Three (3) Questions are Covered


by Executive Privilege

We start with the basic premises where the parties have conceded.

354
The power of Congress to conduct inquiries in aid of legislation is broad. This is based on
the proposition that a legislative body cannot legislate wisely or effectively in the absence of
information respecting the conditions which the legislation is intended to affect or
change.[21] Inevitably, adjunct thereto is the compulsory process to enforce it. But, the power,
broad as it is, has limitations. To be valid, it is imperative that it is done in accordance with the
Senate or House duly published rules of procedure and that the rights of the persons appearing
in or affected by such inquiries be respected.

The power extends even to executive officials and the only way for them to be exempted
is through a valid claim of executive privilege.[22] This directs us to the consideration of the
question -- is there a recognized claim of executive privilege despite the revocation of E.O. 464?

A- There is a Recognized Claim

of Executive Privilege Despite the

Revocation of E.O. 464

At this juncture, it must be stressed that the revocation of E.O. 464 does not in any way
diminish our concept of executive privilege. This is because this concept has Constitutional
underpinnings. Unlike the United States which has further accorded the concept with statutory
status by enacting the Freedom of Information Act[23] and the Federal Advisory Committee
Act,[24] the Philippines has retained its constitutional origination, occasionally interpreted only
by this Court in various cases. The most recent of these is the case of Senate v. Ermita where
this Court declared unconstitutional substantial portions of E.O. 464. In this regard, it is worthy
to note that Executive Ermita’s Letter dated November 15, 2007 limits its bases for the claim of
executive privilege to Senate v. Ermita, Almonte v. Vasquez,[25] and Chavez v. PEA.[26] There
was never a mention of E.O. 464.

While these cases, especially Senate v. Ermita,[27] have comprehensively discussed the
concept of executive privilege, we deem it imperative to explore it once more in view of the

355
clamor for this Court to clearly define the communications covered by executive
privilege.

The Nixon and post-Watergate cases established the broad contours of the presidential
communications privilege.[28] In United States v. Nixon,[29] the U.S. Court recognized a great
public interest in preserving “the confidentiality of conversations that take place in the
President’s performance of his official duties.” It thus considered presidential communications
as “presumptively privileged.” Apparently, the presumption is founded on the “President’s
generalized interest in confidentiality.” The privilege is said to be necessary to guarantee the
candor of presidential advisors and to provide “the President and those who assist him…
with freedom to explore alternatives in the process of shaping policies and making decisions
and to do so in a way many would be unwilling to express except privately.”

In In Re: Sealed Case,[30] the U.S. Court of Appeals delved deeper. It ruled that there
are two (2) kinds of executive privilege; one is the presidential communications privilege and,
the other is the deliberative process privilege. The former pertains to “communications,
documents or other materials that reflect presidential decision-making and deliberations and
that the President believes should remain confidential.” The latter includes ‘advisory opinions,
recommendations and deliberations comprising part of a process by which governmental
decisions and policies are formulated.”

Accordingly, they are characterized by marked distinctions. Presidential communications


privilege applies to decision-making of the President while, the deliberative process privilege,
to decision-making of executive officials. The firstis rooted in the constitutional principle of
separation of power and the President’s unique constitutional
role; the second on common law privilege. Unlike the deliberative process
privilege, the presidential communications privilege applies to documents in their entirety, and
covers final and post-decisional materials as well as pre-deliberative ones[31] As a consequence,
congressional or judicial negation of thepresidential communications privilege is always subject
to greater scrutiny than denial of the deliberative process privilege.

356
Turning on who are the officials covered by the presidential communications privilege, In
Re: Sealed Case confines the privilege only to White House Staff that has “operational proximity”
to direct presidential decision-making. Thus, the privilege is meant to encompass only those
functions that form the core of presidential authority, involving what the court characterized as
“quintessential and non-delegable Presidential power,” such as commander-in-chief power,
appointment and removal power, the power to grant pardons and reprieves, the sole-authority
to receive ambassadors and other public officers, the power to negotiate treaties, etc.[32]

The situation in Judicial Watch, Inc. v. Department of Justice[33] tested the In Re: Sealed
Case principles. There, while the presidential decision involved is the exercise of the President’s
pardon power, a non-delegable, core-presidential function, the Deputy Attorney General and the
Pardon Attorney were deemed to be too remote from the President and his
senior White House advisors to be protected. The Court conceded that

functionally those officials were performing a task directly related to the President’s pardon
power, but concluded that an organizational test was more appropriate for confining the
potentially broad sweep that would result from the In Re: Sealed Case’s functional test. The
majority concluded that, the lesser protections of the deliberative process privilege would
suffice. That privilege was, however, found insufficient to justify the confidentiality of the 4,341
withheld documents.

But more specific classifications of communications covered by executive privilege are


made in older cases. Courts ruled early that the Executive has a right to withhold documents
that might reveal military or state secrets,[34] identity of government informers in some
circumstances,,[35] and information related to pending investigations.[36] An area where the
privilege is highly revered is in foreign relations. In United States v. Curtiss-Wright Export
Corp.[37] the U.S. Court, citing President George Washington, pronounced:

The nature of foreign negotiations requires caution, and their success must

often depend on secrecy, and even when brought to a conclusion, a full disclosure

357
of all the measures, demands, or eventual concessions which may have been

proposed or contemplated would be extremely impolitic, for this might have a

pernicious influence on future negotiations or produce immediate inconveniences,

perhaps danger and mischief, in relation to other powers. The necessity of such

caution and secrecy was one cogent reason for vesting the power of making

treaties in the President, with the advice and consent of the Senate, the principle

on which the body was formed confining it to a small number of members. To

admit, then, a right in the House of Representatives to demand and to have as a

matter of course all the papers respecting a negotiation with a foreign power would

be to establish a dangerous precedent.

Majority of the above jurisprudence have found their way in our jurisdiction. In Chavez
v. PCGG[38], this Court held that there is a “governmental privilege against public disclosure with
respect to state secrets regarding military, diplomatic and other security matters.” In Chavez v.
PEA,[39] there is also a recognition of the confidentiality of Presidential conversations,
correspondences, and discussions in closed-door Cabinet meetings. In Senate v. Ermita, the
concept of presidential communications privilege is fully discussed.

As may be gleaned from the above discussion, the claim of executive privilege is highly
recognized in cases where the subject of inquiry relates to a power textually committed by the
Constitution to the President, such as the area of military and foreign relations. Under our
Constitution, the President is the repository of the commander-in-
chief,[40] appointing,[41]pardoning,[42] and diplomatic[43] powers. Consistent with the doctrine of
separation of powers, the information relating to these powers may enjoy greater confidentiality
than others.

358
The above cases, especially, Nixon, In Re Sealed Case and Judicial Watch, somehow
provide the elements ofpresidential communications privilege, to wit:

1) The protected communication must relate to a “quintessential and non-

delegable presidential power.”

2) The communication must be authored or “solicited and received” by a close

advisor of the President or the President himself. The judicial test is that an

advisor must be in “operational proximity” with the President.

3) The presidential communications privilege remains a qualified privilege that

may be overcome by a showing of adequate need, such that the information

sought “likely contains important evidence” and by the unavailability of the

information elsewhere by an appropriate investigating authority.[44]

In the case at bar, Executive Secretary Ermita premised his claim of executive privilege
on the ground that the communications elicited by the three (3) questions “fall under
conversation and correspondence between the President and public officials” necessary in “her
executive and policy decision-making process” and, that “the information sought to be disclosed
might impair our diplomatic as well as economic relations with the People’s Republic of
China.” Simply put, the bases are presidential communications privilege and executive privilege
on matters relating to diplomacy or foreign relations.

Using the above elements, we are convinced that, indeed, the communications elicited by
the three (3) questions are covered by the presidential communications privilege. First, the
communications relate to a “quintessential and non-delegable power” of the President, i.e. the
power to enter into an executive agreement with other countries. This authority of the President
to enter into executive agreements without the concurrence of the Legislature has traditionally
been recognized in Philippine jurisprudence.[45] Second, the communications are “received” by
a close advisor of the President. Under the “operational proximity” test, petitioner can be
considered a close advisor, being a member of President Arroyo’s cabinet. Andthird, there is no

359
adequate showing of a compelling need that would justify the limitation of the privilege and of
theunavailability of the information elsewhere by an appropriate investigating authority.

The third element deserves a lengthy discussion.

United States v. Nixon held that a claim of executive privilege is subject to balancing
against other interest. In other words, confidentiality in executive privilege is not
absolutely protected by the Constitution. The U.S. Court held:

[N]either the doctrine of separation of powers, nor the need for confidentiality of

high-level communications, without more, can sustain an absolute, unqualified

Presidential privilege of immunity from judicial process under all circumstances.

The foregoing is consistent with the earlier case of Nixon v. Sirica,[46] where it was held
that presidential communications are presumptively privileged and that the presumption can be
overcome only by mere showing of public need by the branch seeking access to conversations.
The courts are enjoined to resolve the competing interests of the political branches of the
government “in the manner that preserves the essential functions of each Branch.”[47] Here, the
record is bereft of any categorical explanation from respondent Committees to show a
compelling or citical need for the answers to the three (3) questions in the enactment of a
law. Instead, the questions veer more towards the exercise of the legislative oversight function
under Section 22 of Article VI rather than Section 21 of the same Article. Senate v.
Ermita ruled that the “the oversight function of Congress may be facilitated by compulsory
process only to the extent that it is performed in pursuit of legislation.” It is conceded that
it is difficult to draw the line between an inquiry in aid of legislation and an inquiry in the
exercise of oversight function of Congress. In this regard, much will depend on the content of
the questions and the manner the inquiry is conducted.

360
Respondent Committees argue that a claim of executive privilege does not guard against
a possible disclosure of a crime or wrongdoing. We see no dispute on this. It is settled in United
States v. Nixon[48] that “demonstrated, specific need for evidence in pending criminal trial”
outweighs the President’s “generalized interest in confidentiality.” However, the present
case’s distinction with the Nixon case is very
evident. In Nixon, there is a pending criminal proceeding where the information is
requested and it is the demands of due process of law and the fair administration of criminal
justice that the information be disclosed. This is the reason why the U.S.
Court was quick to “limit the scope of its decision.” It stressed that it is “not concerned here
with the balance between the President’s generalized interest in confidentiality x x x and
congressional demands for information.” Unlike in Nixon, the information here is elicited, not in
a criminal proceeding, but in a legislative inquiry. In this regard, Senate v. Ermita stressed that
the validity of the claim of executive privilege depends not only on the ground invoked but, also,
on the procedural setting or the context in which the claim is made. Furthermore, inNixon, the
President did not interpose any claim of need to protect military, diplomatic or sensitive national
security secrets. In the present case, Executive Secretary Ermita categorically claims executive
privilege on the grounds of presidential communications privilege in relation to her executive and
policy decision-making process and diplomatic secrets.

The respondent Committees should cautiously tread into the investigation of matters
which may present a conflict of interest that may provide a ground to inhibit the Senators
participating in the inquiry if later on an impeachment proceeding is initiated on the same
subject matter of the present Senate inquiry. Pertinently, in Senate Select Committee on
Presidential Campaign Activities v. Nixon,[49] it was held that since an impeachment proceeding
had been initiated by a House Committee, the Senate Select Committee’s immediate oversight
need for five presidential tapes should give way to the House Judiciary Committee which has the
constitutional authority to inquire into presidential impeachment. The Court expounded on this
issue in this wise:

It is true, of course, that the Executive cannot, any more than the other
branches of government, invoke a general confidentiality privilege to shield its

361
officials and employees from investigations by the proper governmental institutions
into possible criminal wrongdoing. The Congress learned this as to its own
privileges in Gravel v. United States, as did the judicial branch, in a sense, in Clark
v. United States, and the executive branch itself in Nixon v. Sirica. But under Nixon
v. Sirica, the showing required to overcome the presumption favoring confidentiality
turned, not on the nature of the presidential conduct that the subpoenaed material
might reveal, but, instead, on the nature and appropriateness of the function in the
performance of which the material was sought, and the degree to which the
material was necessary to its fulfillment. Here also our task requires and our
decision implies no judgment whatever concerning possible presidential
involvement in culpable activity. On the contrary, we think the sufficiency of the
Committee's showing must depend solely on whether the subpoenaed evidence is
demonstrably critical to the responsible fulfillment of the Committee's functions.

In its initial briefs here, the Committee argued that it has shown exactly this.
It contended that resolution, on the basis of the subpoenaed tapes, of the conflicts
in the testimony before it ‘would aid in a determination whether legislative
involvement in political campaigns is necessary’ and ‘could help engender the public
support needed for basic reforms in our electoral system.’ Moreover, Congress has,
according to the Committee, power to oversee the operations of the executive
branch, to investigate instances of possible corruption and malfeasance in office,
and to expose the results of its investigations to public view. The Committee says
that with respect to Watergate-related matters, this power has been delegated to it
by the Senate, and that to exercise its power responsibly, it must have access to
the subpoenaed tapes.

We turn first to the latter contention. In the circumstances of this case, we


need neither deny that the Congress may have, quite apart from its legislative
responsibilities, a general oversight power, nor explore what the lawful reach of
that power might be under the Committee's constituent resolution. Since passage
of that resolution, the House Committee on the Judiciary has begun an inquiry into
presidential impeachment. The investigative authority of the Judiciary Committee
with respect to presidential conduct has an express constitutional source. x x x We
have been shown no evidence indicating that Congress itself attaches any particular
value to this interest. In these circumstances, we think the need for the tapes
premised solely on an asserted power to investigate and inform cannot justify
enforcement of the Committee's subpoena.

The sufficiency of the Committee's showing of need has come to depend,

therefore, entirely on whether the subpoenaed materials are critical to the

performance of its legislative functions. There is a clear difference between

362
Congress' legislative tasks and the responsibility of a grand jury, or any institution

engaged in like functions. While fact-finding by a legislative committee is

undeniably a part of its task, legislative judgments normally depend more on the

predicted consequences of proposed legislative actions and their political

acceptability, than on precise reconstruction of past events; Congress frequently

legislates on the basis of conflicting information provided in its hearings. In

contrast, the responsibility of the grand jury turns entirely on its ability to

determine whether there is probable cause to believe that certain named individuals

did or did not commit specific crimes. If, for example, as in Nixon v. Sirica, one of

those crimes is perjury concerning the content of certain conversations, the grand

jury's need for the most precise evidence, the exact text of oral statements

recorded in their original form, is undeniable. We see no comparable need in the

legislative process, at least not in the circumstances of this case.Indeed, whatever

force there might once have been in the Committee's argument that the

subpoenaed materials are necessary to its legislative judgments has been

substantially undermined by subsequent events. (Emphasis supplied)

Respondent Committees further contend that the grant of petitioner’s claim of executive
privilege violates the constitutional provisions on the right of the people to information on
matters of public concern.[50] We might have agreed with such contention if petitioner did not
appear before them at all. But petitioner made himself available to them during the September
26 hearing, where he was questioned for eleven (11) hours. Not only that, he expressly
manifested his willingness to answer more questions from the Senators, with the exception only
of those covered by his claim of executive privilege.

The right to public information, like any other right, is subject to limitation. Section 7 of
Article III provides:

The right of the people to information on matters of public concern shall be


recognized. Access to official records, and to documents, and papers pertaining to
official acts, transactions, or decisions, as well as to government research data used
363
as basis for policy development, shall be afforded the citizen, subject to such
limitations as may be provided by law.

The provision itself expressly provides the limitation, i.e. as may be provided by
law. Some of these laws are Section 7 of Republic Act (R.A.) No.
6713,[51] Article 229[52] of the Revised Penal Code, Section 3 (k)[53] of R.A.
No. 3019, and Section 24(e)[54] of Rule 130 of the Rules of
Court. These are in addition to what our body of jurisprudence classifies as confidential[55] and
what our Constitution considers as belonging to the larger concept of executive
privilege. Clearly, there is a recognized public interest in the confidentiality of certain
information. We find the information subject of this case belonging to such kind.

More than anything else, though, the right of Congress or any of its Committees to obtain
information in aid of legislationcannot be equated with the people’s right to public
information. The former cannot claim that every legislative inquiry is an exercise of the
people’s right to information. The distinction between such rights is laid down in Senate v.
Ermita:

There are, it bears noting, clear distinctions between the right of Congress to
information which underlies the power of inquiry and the right of people to
information on matters of public concern. For one, the demand of a citizen for the
production of documents pursuant to his right to information does not have the
same obligatory force as a subpoena duces tecum issued by Congress. Neither does
the right to information grant a citizen the power to exact testimony from
government officials. These powers belong only to Congress, not to an individual
citizen.

Thus, while Congress is composed of representatives elected by the people, it does


not follow, except in a highly qualified sense, that in every exercise of its power of
inquiry, the people are exercising their right to information.

The members of respondent Committees should not invoke as justification in their


exercise of power a right properly belonging to the people in general. This is because when they
discharge their power, they do so as public officials and members of Congress. Be that as it
may, the right to information must be balanced with and should give way, in appropriate cases,
to constitutional precepts particularly those pertaining to delicate interplay of executive-
legislative powers and privileges which is the subject of careful review by numerous decided
cases.

364
B- The Claim of Executive Privilege

is Properly Invoked

We now proceed to the issue -- whether the claim is properly invoked by the
President. Jurisprudence teaches that for the claim to be properly invoked, there must be a
formal claim of privilege, lodged by the head of the department which has control over the
matter.”[56] A formal and proper claim of executive privilege requires a “precise and certain
reason” for preserving their confidentiality.[57]

The Letter dated November 17, 2007 of Executive Secretary Ermita satisfies the
requirement. It serves as the formal claim of privilege. There, he expressly states that “this
Office is constrained to invoke the settled doctrine of executive privilege as refined in Senate v.
Ermita, and has advised Secretary Neri accordingly.” Obviously, he is referring to the Office of
the President. That is more than enough compliance. In Senate v. Ermita, a less categorical
letter was even adjudged to be sufficient.

With regard to the existence of “precise and certain reason,” we find the grounds
relied upon by Executive Secretary Ermita specific enough so as not “to leave respondent
Committees in the dark on how the requested information could be classified as privileged.” The
case of Senate v. Ermita only requires that an allegation be made “whether the information
demanded involves military or diplomatic secrets, closed-door Cabinet meetings, etc.” The
particular ground must only be specified. The enumeration is not even intended to be
comprehensive.”[58] The following statement of grounds satisfies the requirement:

The context in which executive privilege is being invoked is that the information
sought to be disclosed might impair our diplomatic as well as economic relations
with the People’s Republic of China. Given the confidential nature in which these
information were conveyed to the President, he cannot provide the Committee any
further details of these conversations, without disclosing the very thing the privilege
is designed to protect.

At any rate, as held further in Senate v. Ermita, [59] the Congress must not require the
executive to state the reasons for the claim with such particularity as to compel disclosure of the
information which the privilege is meant to protect. This is a matter of respect to a coordinate
and co-equal department.

II
365
Respondent Committees Committed Grave Abuse of Discretion in

Issuing the Contempt Order

Grave abuse of discretion means “such capricious and whimsical exercise of judgment as
is equivalent to lack of jurisdiction, or, in other words where the power is exercised in an
arbitrary or despotic manner by reason of passion or personal hostility and it must be so patent
and gross as to amount to an evasion of positive duty or to a virtual refusal to perform the
duty enjoined or to act at all in contemplation of law.”[60]

It must be reiterated that when respondent Committees issued the show


cause Letter dated November 22, 2007, petitioner replied immediately, manifesting that it was
not his intention to ignore the Senate hearing and that he thought the only remaining questions
were the three (3) questions he claimed to be covered by executive privilege. In addition
thereto, he submitted Atty. Bautista’s letter, stating that his non-appearance was upon the
order of the President and specifying the reasons why his conversations with President Arroyo
are covered by executive privilege. Both correspondences include an expression of his
willingness to testify again, provided he “be furnished in advance” copies of the
questions. Without responding to his request for advance list of questions, respondent
Committees issued the Order dated January 30, 2008, citing him in contempt of respondent
Committees and ordering his arrest and detention at the Office of the Senate Sergeant-At-Arms
until such time that he would appear and give his testimony. Thereupon, petitioner filed a
motion for reconsideration, informing respondent Committees that he had filed the present
petition for certiorari.

Respondent Committees committed grave abuse of discretion in issuing the


contempt Order in view of five (5) reasons.

First, there being a legitimate claim of executive privilege, the issuance of the contempt
Order suffers from constitutional infirmity.

Second, respondent Committees did not comply with the requirement laid down
in Senate v. Ermita that the invitations should contain the “possible needed statute which
prompted the need for the inquiry,” along with “the usual indication of the subject of inquiry
and the questions relative to and in furtherance thereof.” Compliance with this requirement is
366
imperative, both under Sections 21 and 22 of Article VI of the Constitution. This must be so to
ensure that the rights of both personsappearing in or affected by such inquiry are respected as
mandated by said Section 21 and by virtue of the express language of Section
22. Unfortunately, despite petitioner’s repeated demands, respondent Committees did not send
him an advance list of questions.

Third, a reading of the transcript of respondent Committees’ January 30,


2008 proceeding reveals that only a minority of the members of the Senate Blue Ribbon
Committee was present during the deliberation. [61] Section 18 of the Rules of Procedure
Governing Inquiries in Aid of Legislation provides that:

“The Committee, by a vote of majority of all its members, may punish for

contempt any witness before it who disobeys any order of the Committee or refuses

to be sworn or to testify or to answer proper questions by the Committee or any of

its members.”

Clearly, the needed vote is a majority of all the members of the Committee. Apparently,
members who did not actually participate in the deliberation were made to sign the contempt
Order. Thus, there is a cloud of doubt as to the validity of the contempt Order dated January
30, 2008. We quote the pertinent portion of the transcript, thus:

THE CHAIRMAN (SEN. CAYETANO, A). For clarification. x x x The Chair


will call either a caucus or will ask the Committee on Rules if there is a problem.
Meaning, if we do not have the sufficient numbers. But if we have a sufficient
number, we will just hold a caucus to be able to implement that right away
because…Again, our Rules provide that any one held in contempt and ordered
arrested, need the concurrence of a majority of all members of the said committee
and we have three committees conducting this.

So thank you very much to the members…


367
SEN. PIMENTEL. Mr. Chairman.

THE CHAIRMAN (SEN. CAYETANO,A). May I recognize the Minority Leader


and give him the floor, Senator Pimentel.

SEN. PIMENTEL. Mr. Chairman, there is no problem, I think, with consulting


the other committees. But I am of the opinion that the Blue Ribbon Committee is
the lead committee, and therefore, it should have preference in enforcing its own
decisions. Meaning to say, it is not something that is subject to consultation with
other committees. I am not sure that is the right interpretation. I think that once
we decide here, we enforce what we decide, because otherwise, before we know it,
our determination is watered down by delay and, you know, the so-called
“consultation” that inevitably will have to take place if we follow the premise that
has been explained.

So my suggestion, Mr. Chairman, is the Blue Ribbon Committee should not


forget it’s the lead committee here, and therefore, the will of the lead committee
prevails over all the other, you, know reservations that other committees might
have who are only secondary or even tertiary committees, Mr. Chairman.

THE CHAIRMAN (SEN. CAYETANO, A.) Thank you very much to the Minority
Leader. And I agree with the wisdom of his statements. I was merely mentioning
that under Section 6 of the Rules of the Committee and under Section 6, “The
Committee by a vote of a majority of all its members may punish for contempt any
witness before it who disobeys any order of the Committee.”

So the Blue Ribbon Committee is more than willing to take that


responsibility. But we only have six members here today, I am the seventh as
chair and so we have not met that number. So I am merely stating that, sir, that
when we will prepare the documentation, if a majority of all members sign and I am
following the Sabio v. Gordon rule wherein I do believe, if I am not mistaken,
Chairman Gordon prepared the documentation and then either in caucus or in
session asked the other members to sign. And once the signatures are obtained,
solely for the purpose that Secretary Neri or Mr. Lozada will not be able to legally
question our subpoena as being insufficient in accordance with law.
368
SEN. PIMENTEL. Mr. Chairman, the caution that the chair is suggesting
is very well-taken. But I’d like to advert to the fact that the quorum of the
committee is only two as far as I remember. Any two-member senators attending
a Senate committee hearing provide that quorum, and therefore there is more than
a quorum demanded by our Rules as far as we are concerned now, and acting as
Blue Ribbon Committee, as Senator Enrile pointed out. In any event, the
signatures that will follow by the additional members will only tend to strengthen
the determination of this Committee to put its foot forward – put down on what is
happening in this country, Mr. Chairman, because it really looks terrible if the
primary Committee of the Senate, which is the Blue Ribbon Committee, cannot
even sanction people who openly defy, you know, the summons of this
Committee. I know that the Chair is going through an agonizing moment here. I
know that. But nonetheless, I think we have to uphold, you know, the institution
that we are representing because the alternative will be a disaster for all of us, Mr.
Chairman. So having said that, I’d like to reiterate my point.

THE CHAIRMAN (SEN. CAYETANO, A.) First of all, I agree 100


percent with the intentions of the Minority Leader. But let me very respectfully
disagree with the legal requirements. Because, yes, we can have a hearing if we
are only two but both under Section 18 of the Rules of the Senate and under
Section 6 of the Rules of the Blue Ribbon Committee, there is a need for a majority
of all members if it is a case of contempt and arrest. So, I am simply trying to
avoid the court rebuking the Committee, which will instead of strengthening will
weaken us. But I do agree, Mr. Minority Leader, that we should push for this and
show the executive branch that the well-decided – the issue has been decided upon
the Sabio versus Gordon case. And it’s very clear that we are all allowed to call
witnesses. And if they refure or they disobey not only can we cite them in
contempt and have them arrested. x x x [62]

Fourth, we find merit in the argument of the OSG that respondent Committees likewise
violated Section 21 of Article VI of the Constitution, requiring that the inquiry be in
accordance with the “duly published rules of procedure.” We quote the OSG’s explanation:

369
The phrase ‘duly published rules of procedure’ requires the Senate of every

Congress to publish its rules of procedure governing inquiries in aid of legislation

because every Senate is distinct from the one before it or after it. Since Senatorial

elections are held every three (3) years for one-half of the Senate’s membership,

the composition of the Senate also changes by the end of each term. Each Senate

may thus enact a different set of rules as it may deem fit. Not having published

its Rules of Procedure, the subject hearings in aid of legislation conducted by the

14th Senate, are therefore, procedurally infirm.

And fifth, respondent Committees’ issuance of the contempt Order is arbitrary and
precipitate. It must be pointed out that respondent Committees did not first pass upon the
claim of executive privilege and inform petitioner of their ruling. Instead, they curtly dismissed
his explanation as “unsatisfactory” and simultaneously issued the Order citing him in
contempt and ordering his immediate arrest and detention.

A fact worth highlighting is that petitioner is not an unwilling witness. He manifested


several times his readiness to testify before respondent Committees. He refused to answer the
three (3) questions because he was ordered by the President to claim executive privilege. It
behooves respondent Committees to first rule on the claim of executive privilege and inform
petitioner of their finding thereon, instead of peremptorily dismissing his explanation as
“unsatisfactory.” Undoubtedly,
respondent Committees’ actions constitute grave abuse of discretion for
being arbitrary and for denying petitioner due process of law. The same
quality afflicted their conduct when they (a) disregarded petitioner’s motion for
reconsideration alleging that he had filed the present petition before this Court and (b) ignored
petitioner’s repeated request for an advance list of questions, if there be any aside from the
three (3) questions as to which he claimed to be covered by executive privilege.

370
Even the courts are repeatedly advised to exercise the power of contempt judiciously and
sparingly with utmost self-restraint with the end in view of utilizing the same for correction and
preservation of the dignity of the court, not for retaliation or vindication.[63] Respondent
Committees should have exercised the same restraint, after all petitioner is not even an ordinary
witness. He holds a high position in a co-equal branch of government.

In this regard, it is important to mention that many incidents of judicial review could have
been avoided if powers are discharged with circumspection and deference. Concomitant with the
doctrine of separation of powers is the mandate to observe respect to a co-equal branch of the
government.

One last word.

The Court was accused of attempting to abandon its constitutional duty when it required
the parties to consider a proposal that would lead to a possible compromise. The accusation is
far from the truth. The Court did so, only to test a tool that other jurisdictions find to be
effective in settling similar cases, to avoid a piecemeal consideration of the questions for
review and to avert a constitutional crisis between the executive and legislative branches of
government.

In United States v. American Tel. & Tel Co.,[64] the court refrained from deciding the case
because of its desire to avoid a resolution that might disturb the balance of power between the
two branches and inaccurately reflect their true needs. Instead, it remanded the record to the
District Court for further proceedings during which the parties are required to negotiate a
settlement. In the subsequent case of United States v. American Tel. &Tel Co.,[65] it was held
that “much of this spirit of compromise is reflected in the generality of language found in the
Constitution.” It proceeded to state:

371
Under this view, the coordinate branches do not exist in an exclusively adversary

relationship to one another when a conflict in authority arises. Rather each branch

should take cognizance of an implicit constitutional mandate to seek optimal

accommodation through a realistic evaluation of the needs of the conflicting

branches in the particular fact situation.

It thereafter concluded that: “The Separation of Powers often impairs efficiency, in terms
of dispatch and the immediate functioning of government. It is the long-term staying power
of government that is enhanced by the mutual accommodation required by the separation of
powers.”

In rendering this decision, the Court emphasizes once more that the basic principles
of constitutional law cannot be subordinated to the needs of a
particular situation. As magistrates, our mandate is to rule
objectively and dispassionately, always mindful of Mr. Justice Holmes’ warning on the
dangers inherent in cases of this nature, thus:

“some accident of immediate and overwhelming interest…appeals to the


feelings and distorts the judgment. These immediate interests exercise a kind of
hydraulic pressure which makes what previously was clear seem doubtful, and
before which even well settled principles of law will bend.”[66]

In this present crusade to “search for truth,” we should turn to the


fundamental constitutional principles which underlie our tripartite system of
government, where the Legislature enacts the law, the Judiciary interprets it and the
Executive implements it. They are considered separate, co-
equal, coordinate and supreme within their respective spheres but, imbued with a system of
checks and balances to prevent unwarranted exercise of power. The Court’s mandate is to
preserve these constitutional principles at all times to keep the political branches of
government within constitutional bounds in the exercise of their respective powers and
prerogatives, even if it be in the search for truth. This is the only way we can preserve the
stability of our democratic institutions and uphold the Rule of Law.
372
WHEREFORE, the petition is hereby GRANTED. The subject Order dated January 30,
2008, citing petitioner Romulo L. Neri in contempt of the Senate Committees and directing his
arrest and detention, is hereby nullified.

SO ORDERED.

TERESITA J. LEONARDO DE CASTRO


Associate Justice

WE CONCUR:

REYNATO S. PUNO

Chief Justice

CONSUELO YNARES-SANTIAGO

LEONARDO A. QUISUMBING Associate Justice


Associate Justice

373
ANTONIO T. CARPIO MA. ALICIA AUSTRIA-MARTINEZ

Associate Justice Associate Justice

RENATO C. CORONA CONCHITA CARPIO MORALES


Associate Justice
Associate Justice

ADOLFO S. AZCUNA
DANTE O. TINGA
Associate Justice
Associate Justice

MINITA V. CHICO-NAZARIO PRESBITERO J. VELASCO, JR.

Associate Justice Associate Justice

ANTONIO EDUARDO B. NACHURA RUBEN T. REYES

374
Associate Justice Associate Justice

ARTURO D. BRION

Associate Justice

CERTIFICATION

Pursuant to Section 13, Article VIII of the Constitution, I certify that the conclusions in
the above decision had been reached in consultation before the case was assigned to the writer
of the opinion of the Court.

REYNATO S. PUNO

Chief Justice

[1]
Rollo, pp. 12-14.
[2]
Rollo, pp. 85-86. Through the Supplemental Petition for Certiorari (With Urgent
Application for Temporary Restraining Order/Preliminary Injunction).
[3]
Chaired by Hon. Senator Alan Peter S. Cayetano.
375
[4]
Chaired by Hon. Senator Manuel A. Roxas II.
[5]
Chaired by Hon. Senator Rodolfo G. Biazon.
[6]
Transcript of the September 26, 2007 Hearing of the respondent
Committees, pp.91-92.
[7]
Id., pp. 114-115.
[8]
Id., pp. 276-277.
[9]
See Letter dated January 30, 2008.
[10]
488 SCRA 1 (2006).
[11]
345 U.S. 1 (1953).
[12]
Section 7. Prohibited Acts and Transactions. – In addition to acts and omissions of
public officials and employees now prescribed in the Constitution and existing laws, the following
shall constitute prohibited acts and transactions of any public official and employee and are
hereby declared to be unlawful: x x x

(c) Disclosure and/or misuse of confidential information. -


Public officials and employees shall not use or divulge, confidential or classified
information officially known to them by reason of their office and not made available to
the public, either:
(1) To further their private interests, or give undue advantage to anyone; or
(2) To prejudice the public interest.
[13]
SEC. 24. Disqualification by reason of privileged communication. – The following
persons cannot testify as to matters learned in confidence in the following cases. (e) A public
officer cannot be examined during his term of office or afterwards, as to communications made
to him in official confidence, when the court finds that the public interest would suffer by
disclosure.
[14]
TSN of the Oral Argument, March 4, 2008, p. 455.
[15]
(1) Minutes of all meetings of the three (3) committees held in January and February,
2008; (2) Notice for joint meeting of three (3) committees held on 30 January 2008 duly
received by the members of the committees; (3) Minute Books of the three (3)
committees; (4) Composition of the three (3) committees; and (5) Other documents required of
them in the course of the oral argument.
[16]
See Manifestation, rollo, pp.170-174.
[17]
Supra..
[18]
Supra.
[19]
Ibid.
[20]
Ibid.
[21]
Arnault v. Nazareno, 87 Phil 32 (1950)
[22]
Senate v. Ermita, p. 58.
[23]
5 U.S. C. § 552
[24]
51 U.S. C. app.
[25]
433 Phil. 506 (2002).
[26]
G.R. No. 130716, December 9, 1998, (360 SCRA 132 ).
[27]
Supra.
[28]
CRS Report for Congress, Presidential Claims of Executive Privilege: History,
Law, Practice and Recent Developments at p. 2.

376
[29]
418 U.S. 683.
[30]
In Re: Sealed Case No. 96-3124, June 17, 1997.
[31]
Id.
[32]
CRS Report for Congress, Presidential Claims of Executive Privilege: History,
Law, Practice and Recent Developments at pp. 18-19.
[33]
365 F.3d 1108, 361 U.S.App.D.C. 183, 64 Fed. R. Evid. Serv. 141.
[34]
See United States v. Reynolds, 345 U.S. 1, 6-8 (1953); Chicago v. Airlines, Inc. v.
Waterman Steamship Corp., 333 U.S. 103, 111; Totten v. United States, 92 U.S. 105, 106-
107 (1875).
[35]
Roviaro v. United States, 353 U.S. 53, 59-61.
[36]
See Friedman v. Bache Halsey Stuart Shields, Inc. 738 F. 2d 1336,1341-43 (D.C.
Cir. 1984).
[37]
14 F. Supp. 230, 299 U.S. 304 (1936).
[38]
360 Phil. 133 (1998).
[39]
Supra.
[40]
Section 18, Article VII.
[41]
Section 16, Article VII.
[42]
Section 19, Article VII.
[43]
Section 20 and 21, Article VII.
[44]
CRS Report for Congress, Presidential Claims of Executive Privilege: History, Law
Practice and Recent Developments, supra..
[45]
Bernas, S.J., The 1987 Constitution of the Republic of the Philippines, A
Commentary, 2003 Ed. p. 903.
[46]
159 U.S. App. DC. 58, 487 F. 2d 700 (D.C. Cir. 1973).
[47]
U.S. v. Nixon, 418 U.S. 683 (1974)
[48]
Supra.
[49]
498 F. 2d 725 (D.C. Cir.1974).
[50]
Citing Section 7, Article 3 of the Constitution.
[51]
Section 7. Prohibited Acts and Transactions. – In addition to acts and omissions of
public officials and employees now prescribed in the Constitution and existing laws, the following
shall constitute prohibited acts and transactions of any public official and employee and are
hereby declared to be unlawful: x x x

( c) Disclosure and/or misuse of confidential information. - Public officials and


employees shall not use or divulge, confidential or classified information officially known
to them by reason of their office and not made available to the public, either:
(1) To further their private interests, or give undue advantage to anyone; or
(2) To prejudice the public interest.
[52]
Article 229. Revelation of secrets by an officer. – Any public officer who shall reveal
any secret known to him by reason of his official capacity, or shall wrongfully deliver papers or
copies of papers of which he may have charge and which should not be published, shall suffer
the penalties of prision correccional in its medium and maximum periods, perpetual special
disqualification and a fine not exceeding 2,000 pesos if the revelation of such secrets or the
delivery of such papers shall have caused serious damage to the public interest; otherwise, the
penalties of prision correccional in its minimum period, temporary special disqualification and a
fine not exceeding 500 pesos shall be imposed.

377
[53]
Section 3. Corrupt practices of public officers. – In addition to acts or omissions of
public officers already penalized by existing law, the following shall constitute corrupt
practices of any public officer and are hereby declared to be unlawful:
(k) Divulging valuable information of a confidential character, acquired by his office or by
him on account of his official position to unauthorized persons, or releasing such
information in advance of its authorized release date.
[54]
Sec. 24. Disqualification by reason of privileged communications. – The following

persons cannot testify as to matters learned in confidence in the following case: x x

(a) A public officer cannot be examined during his term of office or


afterwards, as to communications made to him in official confidence, when the court finds
that the public interest would suffer by the disclosure.
[55]
In Chavez v. Public Estates Authority, supra., the Supreme Court recognized matters
which the Court has long considered as confidential such as “information on military and
diplomatic secrets, information affecting national security, and information on investigations of
crimes by law enforcement agencies before the prosecution of the accused.” It also stated that
“presidential conversations, correspondences, or discussions during close-door cabinet meetings
which, like internal deliberations of the Supreme Court or other collegiate courts, or executive
sessions of either House of Congress, are recognized as confidential. Such information cannot
be pried-open by a co-equal branch of government.
[56]
United States v. Reynolds, supra..
[57]
Unites States v. Article of Drug, 43 F.R.D. at 190.
[58]
Senate v. Ermita, supra., p. 63.
[59]
Id., citing U.S. v. Reynolds, 345 U.S. 1, 73 S. Ct. 528, 97 L. Ed. 727, 32 A.L. R. 2d
382 (1953).
[60]
Freedom from Debt Coalition v. Energy Regulatory Commission, G.R. No.
161113. June 15, 2004.
[61]
Trancript of the January 30, 2008 proceedings, p. 29.
[62]
Trancript of the January 30, 2008 Proceeding of the respondent Senate Committees,
pp. 26-31.
[63]
Rodriguez v. Judge Bonifacio, A.M. No. RTJ-99-1510, November 6, 2000, 344 SCRA
519.
[64]
179 U.S. App. Supp. D.C. 198, 551 F 2d. 384 (1976).

[65]
567 F 2d 121 (1977).
[66]
Northern Securities Co. v. United States, 193 U.S. 197, 48 L. Ed. 679, 24 S Ct. 436
(1904).

378
EN BANC

G.R. No. 168056 --- ABAKADA Guro Party List (Formerly AASJAS) Officers Samson
S. Alcantara and Ed Vincent S. Albano, Petitioners, versus The
Honorable Executive Secretary Eduardo Ermita, et
al., Respondents.

G.R. No. 168207 --- Aquilino Q. Pimentel, Jr., et al., Petitioners, versus Executive
Secretary Eduardo R. Ermita, et al., Respondents.

G.R. No. 168461 --- Association of Pilipinas Shell Dealers, Inc., et


al., Petitioners, versus Cesar V. Purisima, et al., Respondents.

G.R. No. 168463 --- Francis Joseph G. Escudero, et al., Petitioners, versus Cesar V.
Purisima, et al.,Respondents.

G.R. No. 168730 --- Bataan Governor Enrique T. Garcia, Jr., et


al., Petitioner, versus Hon. Eduardo R. Ermita, et
al., Respondents.

Promulgated:
September 1, 2005

x ---------------------------------------------------------------------------------------- x

CONCURRING AND DISSENTING OPINION

YNARES-SANTIAGO, J.:

The ponencia states that under the provisions of the Rules of the House of
Representatives and the Senate Rules, the Bicameral Conference Committee is mandated to
settle differences between the disagreeing provisions in the House bill and Senate bill. However,
the ponencia construed the term “settle” as synonymous to “reconcile” and “harmonize,” and as
such, the Bicameral Conference Committee may either (a) adopt the specific provisions of either
the House bill or Senate bill, (b) decide that neither provisions in the House bill or the provisions
in the Senate bill would be carried into the final form of the bill, and/or(c) try to arrive at a
compromise between the disagreeing provisions.

I beg to differ on the third proposition.

379
Indeed, Section 16(3), Article VI of the 1987 Constitution explicitly allows each House to
determine the rules of its proceedings. However, the rules must not contravene constitutional
provisions. The rule-making power of Congress should take its bearings from the Constitution.
If in the exercise of this rule-making power, Congress failed to set parameters in the functions of
the committee and allowed the latter unbridled authority to perform acts which Congress itself is
prohibited, like the passage of a law without undergoing the requisite three-reading and the so-
called no-amendment rule, then the same amount to grave abuse of discretion which this Court
is empowered to correct under its expanded certiorari jurisdiction. Notwithstanding the doctrine
of separation of powers, therefore, it is the duty of the Court to declare as void a legislative
enactment, either from want of constitutional power to enact or because the constitutional forms
or conditions have not been observed.[1] When the Court declares as unconstitutional a law or a
specific provision thereof because procedural requirements for its passage were not complied,
the Court is by no means asserting its ascendancy over the Legislature, but simply affirming the
supremacy of the Constitution as repository of the sovereign will.[2] The judicial branch must
ensure that constitutional norms for the exercise of powers vested upon the two other branches
are properly observed. This is the very essence of judicial authority conferred upon the Court
under Section 1, Article VII of the 1987 Constitution.

The Rules of the House of Representatives and the Rules of the Senate provide that in the
event there is disagreement between the provisions of the House and Senate bills, the
differences shall be settled by a bicameral conference committee.

By this, I fully subscribe to the theory advanced in the Dissenting Opinion of Chief Justice
Hilario G. Davide, Jr. inTolentino v. Secretary of Finance[3] that the authority of the bicameral
conference committee was limited to the reconciliation of disagreeing provisions or the resolution
of differences or inconsistencies. Thus, it could only either (a) restore, wholly or partly, the
specific provisions of the House bill amended by the Senate bill, (b) sustain, wholly or partly, the
Senate’s amendments, or (c) by way of a compromise, to agree that neither provisions in the
House bill amended by the Senate nor the latter’s amendments thereto be carried into the final
form of the former.

Otherwise stated, the Bicameral Conference Committee is authorized only to adopt either
the version of the House bill or the Senate bill, or adopt neither. It cannot, as
the ponencia proposed, “try to arrive at a compromise”, such as introducing provisions not
included in either the House or Senate bill, as it would allow a mere ad hoc committee to
substitute the will of the entire Congress and without undergoing the requisite three-reading,
which are both constitutionally proscribed. To allow the committee unbridled discretion to
380
overturn the collective will of the whole Congress defies logic considering that the bills are
passed presumably after study, deliberation and debate in both houses. A lesser body like the
Bicameral Conference Committee should not be allowed to substitute its judgment for that of the
entire Congress, whose will is expressed collectively through the passed bills.

When the Bicameral Conference Committee goes beyond its limited function by
substituting its own judgment for that of either of the two houses, it violates the internal rules of
Congress and contravenes material restrictions imposed by the Constitution, particularly on the
passage of law. While concededly, the internal rules of both Houses do not explicitly limit the
Bicameral Conference Committee to a consideration only of conflicting provisions, it is
understood that the provisions of the Constitution should be read into these rules as imposing
limits on what the committee can or cannot do. As such, it cannot perform its delegated
function in violation of the three-reading requirement and the no-amendment rule.

Section 26(2) of Article VI of the 1987 Constitution provides that:

(2) No bill shall be passed by either House shall become a law unless it
has passed three readings on separate days, and printed copies thereof in its final
form have been distributed to its Members three days before its passage, except
when the President certifies to the necessity of its immediate enactment to meet a
public calamity or emergency. Upon the last reading of a bill, no amendment
hereto shall be allowed, and the vote thereon shall be taken immediately
thereafter, and the yeas and nays entered in the Journal.

Thus, before a bill becomes a law, it must pass three readings. Hence,
the ponencia’s submission that despite its limited authority, the Bicameral Conference
Committee could “compromise the disagreeing provisions” by substituting it with its own version
– clearly violate the three-reading requirement, as the committee’s version would no longer
undergo the same since it would be immediately put into vote by the respective houses. In
effect, it is not a bill that was passed by the entire Congress but by the members of the ad hoc
committee only, which of course is constitutionally infirm.

I disagree that the no-amendment rule referred only to “the procedure to be followed by
each house of Congress with regard to bills initiated in each of said respective houses” because
it would relegate the no-amendment rule to a mere rule of procedure. To my mind, the no-
amendment rule should be construed as prohibiting the Bicameral Conference Committee from
introducing amendments and modifications to non-disagreeing provisions of the House and

381
Senate bills. In sum, the committee could only either adopt the version of the House bill or the
Senate bill, or adopt neither. As Justice Reynato S. Puno said in his Dissenting Opinion
in Tolentino v. Secretary of Finance,[4] there is absolutely no legal warrant for the bold
submission that a Bicameral Conference Committee possesses the power to add/delete
provisions in bills already approved on third reading by both Houses or an ex post veto power.

In view thereof, it is my submission that the amendments introduced by the Bicameral


Conference Committee which are not found either in the House or Senate versions of the VAT
reform bills, but are inserted merely by the Bicameral Conference Committee and thereafter
included in Republic Act No. 9337, should be declared unconstitutional. The insertions and
deletions made do not merely settle conflicting provisions but materially altered the bill, thus
giving rise to the instant petitions.

I, therefore, join the concurring and dissenting opinion of Mr. Justice Reynato S. Puno.

CONSUELO YNARES-SANTIAGO
Associate Justice

[1]
Cooley on Constitutional Limitations, 8th Ed., Vol. I, p. 332.
[2]
Angara v. Electoral Commission, 63 Phil. 139, 158 [1936].
[3]
G.R. Nos. 115455, 115525, 115543, 115544, 115754, 115781, 115852, 115873, 115931, 25
August 1994, 235 SCRA 630, 750.
[4]
Supra, p. 811.

382
Republic of the Philippines
Supreme Court
Manila

THIRD DIVISION

NICASIO I. ALCANTARA, G.R. No. 161881


Petitioner,

- versus -

DEPARTMENT OF ENVIRONMENT
and NATURAL RESOURCES, DENR Present:
SECRETARY ELISEA G. GOZUN,
REGIONAL EXECUTIVE DIRECTOR YNARES-SANTIAGO, J.,
MUSA C. SARUANG, DENR CENRO Chairperson,
ANDREW B. PATRICIO, and AUSTRIA-MARTINEZ,
ROLANDO PAGLANGAN, ET AL., CHICO-NAZARIO,
Respondents. REYES, and
DE CASTRO,* JJ.
HEIRS OF DATU ABDUL B.
PENDATUN, represented by DATU
NASSER B. PENDATUN, AL HAJ,
HEIRS OF SABAL MULA and GAWAN
CLAN, represented by TRIBAL CHIEF-
TAIN LORETO GAWAN, Promulgated:
Respondents-Intervenors, July 31, 2008
x----------------------------------------------------------x

DECISION

AUSTRIA-MARTINEZ, J.:

Before the Court is a Petition for Review on Certiorari under Rule 45 of the Rules of Court, seeking
a reversal of the Decision[1] of the Court of Appeals (CA) dated September 24, 2003 which affirmed the
orders of the Department of Environment and Natural Resources (DENR), cancelling the Forest Land
Grazing Lease Agreement (FLGLA) with Nicasio A.Alcantara (petitioner), ordering him to vacate the land
subject of the cancelled FLGLA and directing the installation of members of a group composed
of B’laan and Maguindanaoans, represented by Rolando Paglangan (private respondents) in the area; as
well as the CA Resolution[2] dated January 23, 2004 denying petitioner's Motion for Reconsideration.

The antecedent facts are as follows:

383
Petitioner is a lessee under FLGLA No. 542, issued by the DENR, of nine hundred twenty-three
(923) hectares of public forest land[3] (subject land) located in the vicinity of Sitio Lanton,
Barrio Apopong, General Santos City.[4]

The subject land, however, is being claimed as the ancestral land of the
indigenous B'laan and Maguindanao people, who maintain that they and their predecessors have been
cultivating, possessing and occupying it since time immemorial.[5] They claim that Christian settlers
(settlers) started occupying the area only after World War II. As a result, there was constant friction
between the indigenous inhabitants and the settlers, with the disputes, at times, erupting in
violence. Overpowered, the indigenous people eventually lost physical control of much of the land.[6]

Petitioner, a son of one of the settlers, used to hold a pasture permit over the subject land, which
was later on converted into FLGLA No. 542 covering the subject property.[7] Petitioner claims that FLGLA
No. 542 has been subsisting since 1983.[8]

On April 10, 1990, private respondents, representing the B'laan and Maguindanao tribes, filed a
complaint[9] against petitioner before the Commission on the Settlement of Land Problems (COSLAP)
seeking the cancellation of FLGLA No. 542 and the reversion of the land to the indigenous
communities.[10]

Private respondents, the Heirs of Datu Abdul B. Pendatun and the Heirs of
the Sabal Mula Gawan Clan (respondents-intervenors), claiming to represent
the B’laan and Maguindanaoan tribes, aver that they have always possessed the land until the first settlers
occupied the area.[11] They claim that among those who took the land by force was petitioner’s
predecessor, Conrado Alcantara. They narrate that in 1962, some of their tribal leaders tried to re-take
the land, but failed because the well-armed settlers repelled them.[12] The incident, in fact, led to the
killing of two of their leaders.[13]

Petitioner filed an answer to the complaint questioning the authority of the COSLAP and alleged
that it was the secretary of the DENR who should have jurisdiction to administer and dispose of public
lands.[14] Petitioner also contended that the COSLAP should suspend the hearing of the case, as the DENR
was then hearing a similar controversy.[15]

In 1993, despite the pendency of the COSLAP case, and despite opposition from private
respondents, petitioner was able to renew FLGLA No. 542 when it expired that year.[16] The renewal given
to petitioner was for another 25 years, or untilDecember 31, 2018.[17]

384
Meanwhile, on October 29, 1997, Congress passed Republic Act No. 8371, or the Indigenous
People's Rights Act (IPRA), which was intended to recognize and promote all the rights of the country's
Indigenous Cultural Communities/Indigenous Peoples (ICCs/IPs) within the framework of the
Constitution.[18]

On August 3, 1998, the COSLAP rendered its decision, the dispositive portion of which reads as
follows:

WHEREFORE, the foregoing considered, judgment is hereby RENDERED in favour


of the complainants and against the Respondents as follows:

1. Recommends to the Hon. Secretary of DENR the cancellation of


respondent’s renewed Forest Land Grazing Lease Agreement (FLGLA) No.
542;

2. Recommending to the DENR to the immediate segregation of the


Three Hundred (300) hectares requested by complainants from the Nine
Hundred Twenty Three (923) Hectares;

3. Recommending to the DENR to declare the entire area of the Nine


Hundred Twenty Three (923) Hectares, the ancestral lands of the B’laans;

4. Recommending to the DENR after the Cancellation of FLGLA No.


542, to place in possession the petitioners in order to start cultivation and
plant crops for their food and solve the on-going famine and hunger being
experience[d] at present by the Lumads.[19]

In addition, the COSLAP made the following factual findings:

a) The subject land is the ancestral domain of the complainant indigenous people, whose
possession was merely interrupted by the forcible and violent takeover of outside
settlers.[20]

b) FLGLA No. 542 was issued by the DENR without giving due process to the indigenous
communities as oppositors and in violation of existing laws such as Presidential Decree
(P.D.) No. 410 and the Constitution.[21]

The COSLAP maintained that it had jurisdiction over the case by virtue of Executive Order (E.O.)
No. 561, the law creating the COSLAP, which provides:
Sec. 3. Powers and Functions. - The Commission shall have the following
powers and functions:
xxxx
2. Refer and follow-up for immediate action by the agency having appropriate
jurisdiction any land problem or dispute referred to the Commission: Provided, That the
385
Commission may, in the following cases, assume jurisdiction and resolve land problems or
disputes which are critical and explosive in nature considering, for instance, the large
number of the parties involved, the presence or emergence of social tension or unrest, or
other similar critical situations requiring immediate action:

(a) Between occupants/squatters and pasture lease agreement holders or


timber concessioners;
(b) Between occupants/squatters and government reservation grantees;
(c) Between occupants/squatters and public land claimants or applicants;
(d) Petitions for classification, release and/or subdivision of lands of the
public domain; and
(e) Other similar land problems of grave urgency and magnitude.[22]

Disagreeing with the ruling of COSLAP, petitioner filed a motion for reconsideration of the decision,
which COSLAP denied.

Petitioner then filed before the CA a petition[23] for certiorari under Rule 65 to question the decision
of the COSLAP. The CA, in its Decision dated June 22, 2000, affirmed in toto the decision of the
COSLAP.[24]

Aggrieved, petitioner filed a petition for review on certiorari before the Court, docketed as G.R. No.
145838.

The Court, in its Decision dated July 20, 2001, upheld the CA and the COSLAP, holding that a)
COSLAP had jurisdiction to decide the case; b) FLGLA No. 542 was issued in violation of the law, and; c)
the 923 hectares covered by FLGLA No. 542 were ancestral land of the private respondents.[25]

When the decision of the Court attained finality, private respondents filed a motion for execution
of the COSLAP'sdecision. Petitioner filed his opposition to the motion.

On July 29, 2002, the COSLAP issued a writ of execution of its decision, wherein it ordered the
Secretary of the DENR to implement the August 3, 1998 decision as affirmed by the Supreme Court.[26]

In a memorandum dated October 19, 2001, the Secretary of the DENR Heherson Alvarez (Sec.
Alvarez), upon receipt of the writ of execution and before cancelling FLGLA No. 542, ordered the Office of
the Regional Executive Director of DENR Region XII, in Koronadal City, to conduct a review and
investigation of FLGLA No. 542.[27] In compliance, the Officer in Charge (OIC)-Regional Executive Director
conducted an investigation and review of the lease under the said FLGLA. One of the participants in the
investigation was a representative of petitioner.[28] Following the investigation, the team released its

386
report,[29]dated February 13, 2002, which found violations by petitioner of the terms of the FLGLA, as
follows:

1. Failure to establish a food production area within the leased area;

2. Failure to undertake forage improvement within the leased area;

3. Failure to pay the full and or on time Annual Rental/User's Fee/


Government Share pursuant to section 28 and 29 of DAO No. 99-36 dated
August 10, 1999 Re: Revised Rules and Regulations Governing the
Administration, Management, Development and Disposition of Forest Lands Used
for Grazing Purposes. Instead the lessee pay (sic) a partial payment of
Php18,566 per O.R. [No.] 9640117 dated December 29, 2000 and Php147,680
per O.R. [No.] 9640246 dated February 1, 2001.

4. The 7-years (sic) Grazing Management Plan for CY 1987-1993 of the said
lessee was expired. During our investigation, the lessee had failed to present the
revised 7-years [sic] Grazing Management Plan for CY 1994-2000 and thereafter
pursuant to item No. 23 of the aforesaid contract.

5. Annual report for year 2001 submitted by the lessee revealed that cattle
stock of the leased area is only 249 heads; however, the investigation team
observed that there were an excess of cattle stock present in the grazing area.
The said excess cattle were (sic) allegedly came from [an] adjacent ranch own
(sic) by Alejandro Alcantara.

6. The team noticed the presence of squatters within the leased area by [a]
certain Asonto et al. and Jumawan et al.

7. FLGLA no. 542 having [sic] an area of 923 hectares which exceed to (sic)
the limit of 500 hectares for individual holder [sic] pursuant to Section 3 Article
XII of [the] 1987 Philippine Constitution as implemented by DAO No. 99-36
series of 1999.

8. Pursuant to Memorandum dated December 5, 2001 of the team leader


Wahid Amella of CLCSI No. 6 the 478.08 hectares out of the 923 hectares of the
leased area is portion of PMD 5338 reverting it to the category of Forest Land.
However, no Forestry Administrative Order issued. x x x[30]

Thus, on August 15, 2002, Sec. Alvarez issued an order cancelling FLGLA No. 542 and subjecting
the area under theDENR's authority pending final distribution to the concerned communities by the
National Commission on Indigenous Peoples (NCIP) or the COSLAP.[31]

Petitioner filed a motion for reconsideration of the order of cancellation. In an order


dated November 21, 2002,[32] Sec. Alvarez denied the motion for reconsideration and affirmed the order
of cancellation dated August 15, 2002.
387
On November 22, 2002, Sec. Alvarez issued a memorandum to the Regional Executive Director of
DENR Region XII, inKoronadal City, to implement the four recommendations of the COSLAP contained in
its Order dated August 3, 1998; and issue the corresponding survey authority.[33]

On November 26, 2002, Community Environment and Natural Resources Officer (CENRO) Andrew
B. Patricio Jr. sent a letter to petitioner, advising him to vacate and remove all improvements in the area
within 10 days from receipt of the letter.[34] On even date, CENRO Patricio sent another letter which
amended the first letter and advised petitioner to vacate the landimmediately, instead of within 10 days
as earlier advised.[35]

On November 27, 2002, CENRO Patricio issued an Installation Order, which directed the
immediate installation and occupation of the area, covered by the cancelled FLGLA No. 542, by the private
respondents’ indigenous communities.[36]

On December 3, 2002, petitioner filed a petition for certiorari before the CA, docketed as CA G.R.
SP No. 74166, praying for the annulment and setting aside of the orders of the public respondents,
enumerated as follows:

1) The Order dated August 15, 2002 by Sec. Alvarez, which cancelled the FLGLA
No. 542 issued to petitioner;

2) The Order dated November 21, 2002 by Sec. Alvarez denying petitioner's
motion for reconsideration of the order of cancellation;

3) The Memorandum dated November 22, 2002 by Sec. Alvarez which orders
Regional Office XII of the DENR to implementCOSLAP's recommendations and to
issue the corresponding survey authority;

4) The two Letters dated November 26, 2002 of CENRO Patricio ordering
petitioner to immediately vacate and remove improvements in the subject area.

5) The Installation Order dated November 27, 2002 of


CENRO Patricio authorizing the installation and occupation of the subject area by
private respondents.

On September 24, 2003, the CA rendered its decision, dismissing the petition filed by
petitioner Alcantara and ruling that the issues and arguments it raised had all been addressed squarely in
the Supreme Court's decision in G.R. No. 145838 which upheld the COSLAP's decision and which had long
become final and executory. The CA stated further that the petition was barred by the decision in that
case, as both shared the same parties, the same subject matter and the same cause of action.
388
Hence, herein petition.

Petitioner alleges that when he filed the petition for certiorari before the CA below (CA G.R. SP No
74166), questioning the orders of respondents DENR officials, he “did not seek to have the cancellation of
its FLGLA No. 542 reconsidered or reopened, precisely because such cancellation was already covered by
a final decision of the Supreme Court.” He insists that what he sought was to have a “clear determination
of his residual rights after such cancellation in the context of the provisions of the IPRA Law
x x x considering that the right to 'lands of the ancestral domain' arose only in view of the IPRA Law and
cultural minorities had priorly no right to recover their ancestral lands.”[37]

Petitioner's arguments are centered on the following two main issues:

Whether petitioner may continue his enjoyment of the land up to the expiration of
FLGA No. 542, or December 31, 2018, based on his alleged residual rights.

Whether respondents DENR officials committed grave abuse of discretion in


implementing the COSLAP's decision, which has been upheld by the Supreme Court.

The petition lacks merit.

Petitioner may not enjoy possession and use of the land


up to the expiration of FLGLA No. 542, or December 31,
2018, based on his alleged residual rights.

Petitioner’s claim that he has residual rights to remain on the property is based on Section 56 of
the IPRA, which states:

SEC. 56. Existing Property Rights Regimes. – Property rights within the ancestral
domains already existing and/or vested uponeffectivity of this Act, shall be recognized and
respected.
The contention of petitioner has no merit. As stated in the Court's decision in G.R. No.
145838,[38] the legal dispute surrounding petitioner's FLGLA No. 542 began in 1990, which was before
the IPRA's passage in 1997, and even before the FLGLA was renewed in 1993. Thus, the case is not
covered by IPRA, but by other laws existing at the time the COSLAP took cognizance of the case. IPRA
also did not cure the legal defects and infirmities of FLGLA No. 542, which were already the subject of
controversy by the time the law was passed.

Petitioner further calls for IPRA's application, since “the right to lands of the ancestral domain
arose only in view of the IPRA Law and cultural minorities had priorly no right to recover their ancestral

389
lands.”[39] Petitioner is utterly mistaken or misinformed. Before IPRA, the right of ICCs/IPs to recover
their ancestral land was governed by Presidential Decree (P.D.) No. 410,[40] which declared ancestral lands
of national cultural communities as alienable and disposable, and E.O. No. 561,[41]which created the
COSLAP. These laws were the bases of the Court's decision in G.R. No. 145838. That the rights of
mostICCs/IPs went largely unrecognized despite these laws was not due to the laws' inadequacies, but
due to government indifference and the political inertia in their implementation.[42]

It is also clear that when this Court, in G.R. No. 145838, declared FLGLA No. 542 as illegal and
upheld COSLAP'srecommendation of its cancellation, petitioner had no right to the land, and
consequently, had no right to remain in the use and possession of the subject land. Sec. Alvarez's
cancellation of FLGLA No. 542 merely conformed with the Court’s findings. The cancellation made by the
DENR merely sealed the fact that FLGLA No. 542 should not have been issued in favour of petitioner, in
the first place. The COSLAP decision has the force and effect of a regular administrative resolution;
hence, it must be implemented and is binding on all parties to the case.[43]

The question whether FLGLA No. 542 is valid has been settled conclusively in G.R. No. 145838 in
which the Court made the final finding that FLGLA No. 542 was issued illegally, and that it was made in
violation of prevailing laws; and that it was proper for it to be cancelled. The Court ruled, thus:

The Court of Appeals also stated that based on the records, the land area being
claimed by private respondents belongs to the B’laanindigenous cultural community since
they have been in possession of, and have been occupying and cultivating the same since
time immemorial, a fact which has not been disputed by petitioner. It was likewise declared
by the appellate court that FLGLA No. 542 granted to petitioner violated Section 1 of
Presidential Decree No. 410 which states that all unappropriated agricultural lands forming
part of the public domain are declared part of the ancestral lands of the indigenous cultural
groups occupying the same, and these lands are further declared alienable and
disposable, to be distributed exclusively among the members of the indigenous cultural
group concerned.

The Court finds no reason to depart from such finding by the appellate court, it
being a settled rule that findings of fact of the Court of Appeals are binding and conclusive
upon the Supreme Court absent any showing that such findings are not supported by the
evidence on record.[44] (Emphasis supplied)

Petitioner himself admits the finality of that decision, as he states in the petition that he does not “seek to
have the cancellation of FLGLA No. 542 reconsidered or reopened, x x x but a clear determination of his
residual rights after such cancellation in the context of the provisions of the IPRA Law.” However, it
appears from a reading of the entire petition that what petitioner means by his “residual rights” is for him
to continue enjoying exclusive use of the land until the expiration of FLGLA No. 542 onDecember 31,
2018.[45]
390
Again, the decision in G.R. No. 145838 brings out the futility of petitioner's arguments. In no
uncertain terms, that decision declared that FLGLA No. 542 was illegally issued. Therefore, from that
illegal issuance only flowed an invalid FLGLA, as it is axiomatic in our legal system that acts executed
against the laws are void,[46] and that administrative or executive acts, orders and regulations that are
contrary to the laws or the Constitution are invalid.[47] Petitioner has no right or interest to speak of,
because it is also axiomatic that no vested or acquired right can arise from illegal acts or those that
infringe upon the rights of others.[48]

Petitioner's proposition that despite the lengthy litigation that culminated in the invalidation of
FLGLA No. 542, he still has the “residual right” to enjoy use of the land until December 31, 2018 is
absolutely unacceptable. His stance invites anomaly at best, or ridicule at worst, for it asks this Court to
render useless its own final decision in G.R. No. 145838. It also solicits disrespect of all judicial decisions
and processes. Instead of ending the litigation, it mocks the painstaking process undertaken by the
courts and administrative agencies to arrive at the decision in that case. Petitioner’s alleged “residual
right” has no legal basis and contradicts his admission that FLGLA No. 542 has been declared invalid by
the Court in its decision in G.R. No. 145838. Petitioner has had no residue of any right and no entitlement
to the land, from the very beginning.

Petitioner's concern over his alleged rights under the IPRA have all been addressed in G.R. No.
145838. The IPRA was enacted on October 29, 1997. The decision in G.R. No. 145838 was promulgated
on July 20, 2001. On that later date, the Court was already aware of IPRA; and when it rendered the
decision, it could have expressly declared that petitioner had residual rights under that law if such was the
case.[49] The Court applied P.D. No. 410, the law in effect before the IPRA, in finding that FLGLA No. 542
was illegal. This finally disposes of petitioner's claim that he has rights under the IPRA.

In fact, the Court sees petitioner's filing of the present petition as outright forum-shopping, as it
seeks to revisit what has become a final and executory decision. As explained in earlier cases, the
hallmarks of forum-shopping are:

Forum-shopping exists where the elements of litis pendentia are present, and
where a final judgment in one case will amount to resjudicata in the other. Thus, there is
forum-shopping when, between an action pending before this Court and another one, there
exist: “a) identity of parties, or at least such parties as represent the same interests in both
actions, b) identity of rights asserted and relief prayed for, the relief being founded on the
same facts, and c) the identity of the two preceding particulars is such that any judgment
rendered in the other action, will, regardless of which party is successful amount
to res judicata in the action under consideration; said requisites also constitutive of the
requisites for auter action pendant or lis pendens.” Another case elucidates the
391
consequence of forum-shopping: “[W]here a litigant sues the same party against whom
another action or actions for the alleged violation of the same right and the enforcement of
the same relief is/are still pending, the defense of litis pendentia in one case is a bar to the
others; and, a final judgment in one would constitute res judicata and thus would cause the
dismissal of the rest.”[50]

Thus, when petitioner raised the issue on whether he should be allowed to remain on the subject
land until the expiration of FLGLA No. 542, based on his alleged residual rights, he re-opened an issue
already discussed and settled in an earlier case. His use of cleverly disguised language does not hide this
fact. Clearly, the Supreme Court decision, in G.R. No. 145838, is resjudicata in the present
case. Therefore, his filing of the present case despite the finality of an earlier identical case makes the
present one subject to dismissal.

It has been held that res judicata has two concepts: bar by prior judgment and conclusiveness of
judgement.[51] The elements under the first concept are the following:
(1) a former final judgment that was rendered on the merits;
(2) the court in the former judgment had jurisdiction over the subject matter and the
parties; and,
(3) identity of parties, subject matter and cause of action between the first and second
actions;[52]

On the other hand, for the second concept to operate, or for there to be conclusiveness of
judgment, there must be identity of parties and subject matter in the first and second cases, but no
identity of causes of action.[53] If a particular point or question is in issue in the second action, and the
judgment will depend on the determination of that particular point or question, a former judgment
between the same parties will be final and conclusive in the second if that same point or question was in
issue and adjudicated in the first suit; but the adjudication of an issue in the first case is not conclusive of
an entirely different and distinct issue arising in the second.[54] Under the doctrine of conclusiveness of
judgment, facts and issues actually and directly resolved in a former suit cannot again be raised in any
future case between the same parties, even if the latter suit may involve a different claim or cause of
action.[55]

Consequently, the present petition is already barred by res judicata under the first concept, since
the first and second cases share identical parties, subject matter and cause of action. The shared cause of
action is the alleged violation of petitioner's right to remain on the subject land until the expiry date of
FLGLA No. 542 on December 31, 2018. As this issue has been settled, there is no more reason to revisit
it in the present case. There is no reason for an illegal and cancelled FLGLA to continue in effect or confer
any rights on anyone until it expires on December 31, 2018.

392
Even if the Court accepts petitioner's contention that in the present case, he introduces another
cause of action, which is the alleged violation of his right to due process by the haphazard implementation
of the COSLAP decision by the respondent DENR officials, it is severely limited by the second concept
of res judicata, i.e., conclusiveness of judgment. Since it is now conclusive and binding in this case that
FLGLA No. 542 is illegal and should be cancelled, per the decision in G.R. No. 145838, petitioner could no
longer deny that the respondent DENR officials acted legally in cancelling FLGLA No. 542 and in ordering
petitioner to vacate the subject land. The public respondents merely acted to implement the COSLAP
decision as upheld by the Supreme Court.

Thus, petitioner is left to prove only whether the public respondents acted with grave abuse of
discretion in their execution of COSLAP's decision.

There was no grave abuse of discretion in public


respondents' implementation of the COSLAP decision.

The Court finds that no grave abuse of discretion was committed by respondent DENR officials in
their implementation of the COSLAP decision.

It must be emphasized that FLGLA No. 542 is a mere license or privilege granted by the State to
petitioner for the use or exploitation of natural resources and public lands over which the State has
sovereign ownership under the Regalian Doctrine.[56] Like timber or mining licenses, a forest land grazing
lease agreement is a mere permit which, by executive action, can be revoked, rescinded, cancelled,
amended or modified, whenever public welfare or public interest so requires.[57] The determination of
what is in the public interest is necessarily vested in the State as owner of the country's natural
resources.[58] Thus, a privilege or license is not in the nature of a contract that enjoys protection under
the due process and non-impairment clauses of the Constitution.[59] In cases in which the license or
privilege is in conflict with the people's welfare, the license or privilege must yield to the supremacy of the
latter, as well as to the police power of the State.[60] Such a privilege or license is not even a property or
property right, nor does it create a vested right; as such, no irrevocable rights are created in its
issuance.[61]

FLGLA No. 542 has not only been withdrawn by executive action to further the public welfare, it
has also been declared illegal or unlawful by judicial authorities for clearly violating actual provisions of
law. Thus, the DENR was under obligation to effect the cancellation accordingly.

We likewise find no irregularity in the procedure followed by respondent DENR officials in their
cancellation of FLGLA No. 542 and their orders for petitioner to vacate the subject land. Petitioner claims

393
that the public respondents were “haphazard” in their cancellation of the FLGLA, thus denying him due
process.[62] Contrary to the portrayals by the petitioner, however, the officials were not precipitate in their
cancellation of the license and in ordering petitioner to vacate the land. Instead of immediately cancelling
FLGLA No. 542, Sec. Alvarez first ordered the Regional Executive Director of DENR to conduct a review
and investigation of FLGLA No. 542.[63] Following that investigation, attended by petitioner's
representative, it was found that petitioner committed several violations of the terms of the FLGLA.[64] It
was only then that Sec. Alvarez issued the cancellation order.

It is clear from the investigation report that petitioner's FLGLA No. 542 is not only illegal per
se, for having been issued contrary to the provisions of P.D. No. 410; it has also been rendered illegal by
petitioner's blatant violations of DENR regulations and the FLGLA's very own terms and conditions. Thus,
the DENR had compelling reasons to cancel the FLGLA.

In conclusion, the Court, in G.R. No. 145838, recognized the inherent right of ICCs/IPs to recover
their ancestral land from outsiders and usurpers. Seen by many as a victory attained by the private
respondents only after a long and costly effort, the Court, as a guardian and instrument of social justice,
abhors a further delay in the resolution of this controversy and brings it to its fitting conclusion by denying
the petition.

WHEREFORE, the decision appealed from is AFFIRMED. Double costs against petitioner.

SO ORDERED.

MA. ALICIA AUSTRIA-MARTINEZ


Associate Justice

WE CONCUR:

CONSUELO YNARES-SANTIAGO
Associate Justice
Chairperson

MINITA V. CHICO-NAZARIO RUBEN T. REYES


Associate Justice Associate Justice
394
TERESITA J. LEONARDO-DE CASTRO
Associate Justice

ATTESTATION

I attest that the conclusions in the above Decision had been reached in consultation before the
case was assigned to the writer of the opinion of the Court’s Division.

CONSUELO YNARES-SANTIAGO
Associate Justice
Chairperson, Third Division

CERTIFICATION

Pursuant to Section 13, Article VIII of the Constitution, and the Division Chairperson’s Attestation,
it is hereby certified that the conclusions in the above Decision had been reached in consultation before
the case was assigned to the writer of the opinion of the Court’s Division.

REYNATO S. PUNO

Chief Justice

395
*
In lieu of Justice Antonio Eduardo B. Nachura, per Raffle dated July 21, 2008.
[1]
Penned by Associate Justice B.A. Adefuin-De la Cruz with the concurrence of Associate
Justices Eliezer R. de los Santos and Jose C. Mendoza; rollo, pp. 46-52.
[2]
Id. at 54.
[3]
Id. at 5.
[4]
CA rollo, p. 35.
[5]
Rollo, p. 128.
[6]
Memorandum of respondents Paglangan et al., pp. 7-11; Court of Appeals decision in CA-
G.R. SP No. 53159, June 22, 2000, pp. 2-4, CA rollo, pp. 143-145.
[7]
Id. at 52.
[8]
Rollo, p. 12.
[9]
Docketed as COSLAP Case No. 98-052.
[10]
Rollo, pp. 73, 128, 215; Alcantara v. Commission on the Settlement of Land
Problems, G.R. No. 145838, July 20, 2001, 361 SCRA 664.
[11]
Memorandum of respondents Paglangan, et al., pp. 7-9.
[12]
Id. at 8-9.
[13]
Id. at 9.
[14]
Quoted from the COSLAP Decision dated August 3, 1998, CA rollo, pp. 128-141, 134-135.
[15]
Id.
[16]
Rollo, pp. 73, 129.
[17]
Id. at 5, 129.
[18]
The provisions of the Constitution recognizing the rights of ICCs/IPs are: Article II, Sec.
22; Article VI, Sec. 5, par. 2; Article XII, Sec. 5; Article XIII, Sec. 6; Article XIV, Sec. 17; and
Article XVI, Sec. 12.
[19]
CA rollo, p. 56.
[20]
Id. at 53.
[21]
Id. at 55.
[22]
CA rollo, p. 55.
[23]
Docketed as CA-G.R. SP No. 53159.
[24]
CA rollo, pp. 142-152.
[25]
Alcantara v. Commission on the Settlement of Land Problems, supra note 10, at 670-671.
[26]
CA rollo, pp. 66-68.
[27]
Id. at 161.
[28]
Id. at 162.
[29]
Id. at 162-165.
[30]
CA rollo, pp. 162-165.
[31]
Id. at 35-36.
[32]
Id. at 39-41.
[33]
Id. at 42.
[34]
CA rollo, p.102.
[35]
Id. at 103.
[36]
Id. at 104.
[37]
Rollo, pp. 14-15.
[38]
Supra note 10, at 664.
[39]
Rollo, pp. 14-15.

396
[40]
Declaring Ancestral Lands Occupied and Cultivated by National Cultural Communities as
Alienable and Disposable, and for Other Purposes.
[41]
Creating the Commission on the Settlement of Land Problems; among the provisions of
this law are:
xxxx
WHEREAS, land problems are frequently a source of conflicts among small settlers,
landowners and members of cultural minorities;
xxxx
Sec. 3. Powers and Functions. - The Commission shall have the following powers and
functions:
1. Coordinate the activities, particularly the investigation work, of the various government
offices and agencies involved in the settlement of land problems or disputes, and streamline
administrative procedures to relieve small settlers and landholders and members of cultural
minorities of the expense and time-consuming delay attendant to the solution of such problems or
disputes; x x x.
[42]
Cruz v. Secretary of Environment and Natural Resources, G.R. No. 135385, December 6,
2000, 347 SCRA 128, Separate Opinion of Associate (now Chief) JusticeReynato S. Puno, p. 193.
[43]
Executive Order No. 561 (1979), Sec. 3, par. 2.
[44]
Supra note 10, at 670-671.
[45]
Rollo, pp. 11, 33.
[46]
CIVIL CODE, Art. 5.
[47]
CIVIL CODE, Art. 7.
[48]
CIVIL CODE, Art. 2254; Philippine National Bank v. Court of Appeals, G.R. No. 108870,
July 14, 1995, 246 SCRA 304; Heirs of Gabriel Zari v. Santos, Nos. L-21213 & L-21214, March 28,
1969, 27 SCRA 651.
[49]
The decision mentions the National Commission on Indigenous Peoples (NCIP), an agency
created under the IPRA, in discussing the jurisdiction of the COSLAP. Supra note 10, at 667.
[50]
Prubankers Association v. Prudential Bank and Trust Company, G.R. No. 131247, January
25, 1999, 302 SCRA 74, 83; First Philippine International Bank v. Court of Appeals, January 24,
1996, 252 SCRA 259.
[51]
Laperal v. Katigbak, No. L-16951, February 28, 1962, 4 SCRA 582, 590; Tiongson v.
Court of Appeals, No. L-35059, February 27, 1973, 49 SCRA 429, 434; Vda. deCruzo v. Carriaga,
Jr., G.R. Nos. 75109-10, June 28, 1989, 174 SCRA 330, 338; Nabus v. Court of Appeals, G.R. No.
91670, February 7, 1991; 190 SCRA 732, 739.
[52]
Cayana v. Court of Appeals, G.R. No. 125607, March 18, 2004, 426 SCRA 10, 20.
[53]
Republic v. Yu, G.R. No. 157557, March 10, 2006, 484 SCRA 416, 422.
[54]
Nabus v. Court of Appeals, supra note 51, at 739.
[55]
Rizal Surety and Insurance Company v. Court of Appeals, G.R. No. 112360, July 18,
2000, 336 SCRA 12, 21-22 citing Smith Bell and Company (Phils.), Inc. v. Court of Appeals, G.R.
No. 56294, May 20, 1991, 197 SCRA 201, 209.
[56]
CONSTITUTION, Art. XII, Sec. 2; United Paracale Mining Company, Inc. v. Dela Rosa,
G.R. Nos. 63786-87, April 7, 1993, 221 SCRA 108; Republic v. Court of Appeals, No. L-
43938, April 15, 1988, 160 SCRA 228, 239; Santa Rosa Mining Company, Inc. v. Leido, Jr., No. L-
49109, December 1, 1987, 156 SCRA 1, 8-9; La Bugal-B’LaanTribal Association, Inc. v. Ramos,
G.R. No. 127882, January 27, 2004, 421 SCRA 148.
[57]
Tan v. Director of Forestry, No. L-24548, October 27, 1983, 125 SCRA 302,
325; Oposa v. Factoran, Jr., G.R. No. 101083, July 30, 1993, 224 SCRA 792, 812; Alvarez
v.Picop Resources, Inc., G.R. No. 162243, November 29, 2006, 508 SCRA 498; Republic

397
v. Rosemoor Mining and Development Corporation, G.R. No. 149927, March 30, 2004, 426 SCRA
516, 530.
[58]
Republic v. Rosemoor Mining and Development Corporation, supra note 57.
[59]
Id.
[60]
Id.
[61]
Tan v. Director of Forestry, supra note 57.
[62]
Rollo, pp. 18-19.
[63]
Supra note 27.
[64]
Supra note 30.

398
Republic of the Philippines
Supreme Court
Manila

EN BANC

THE SECRETARY OF THE G.R. No. 167707


DEPARTMENT OF ENVIRONMENT
AND NATURAL RESOURCES, THE
REGIONAL EXECUTIVE Present:
DIRECTOR, DENR-REGION VI,
REGIONAL TECHNICAL PUNO, C.J.,
DIRECTOR FOR LANDS, QUISUMBING,
LANDS MANAGEMENT BUREAU, YNARES-SANTIAGO,
REGION VI PROVINCIAL CARPIO,
ENVIRONMENT AND NATURAL AUSTRIA-MARTINEZ,
RESOURCES OFFICER OF KALIBO, CORONA,*
AKLAN, REGISTER OF DEEDS, CARPIO MORALES,
DIRECTOR OF LAND AZCUNA,
REGISTRATION AUTHORITY, TINGA,
DEPARTMENT OF TOURISM CHICO-NAZARIO,
SECRETARY, DIRECTOR OF VELASCO, JR.,
PHILIPPINE TOURISM NACHURA,**
AUTHORITY, REYES,
Petitioners, LEONARDO-DE CASTRO, and
BRION, JJ.

- versus -

MAYOR JOSE S. YAP, LIBERTAD


TALAPIAN, MILA Y. SUMNDAD, and
ANICETO YAP, in their behalf and Promulgated:
in behalf of all those similarly situated,
Respondents. October 8, 2008

x--------------------------------------------------x

DR. ORLANDO SACAY and G.R. No. 173775


WILFREDO GELITO, joined by
THE LANDOWNERS OF
BORACAY SIMILARLY
SITUATED NAMED IN A LIST,
ANNEX “A” OF THIS PETITION,
Petitioners,

- versus -
399
THE SECRETARY OF THE
DEPARTMENT OF ENVIRONMENT
AND NATURAL RESOURCES, THE
REGIONAL TECHNICAL
DIRECTOR FOR LANDS, LANDS
MANAGEMENT BUREAU,
REGION VI, PROVINCIAL
ENVIRONMENT AND NATURAL
RESOURCES OFFICER, KALIBO,
AKLAN,
Respondents.

x--------------------------------------------------x

DECISION

REYES, R.T., J.:

AT stake in these consolidated cases is the right of the present occupants


of Boracay Island to secure titles over their occupied lands.

There are two consolidated petitions. The first is G.R. No. 167707, a petition for review
on certiorari of the Decision[1]of the Court of Appeals (CA) affirming that[2] of the Regional Trial
Court (RTC) in Kalibo, Aklan, which granted the petition for declaratory relief filed by
respondents-claimants Mayor Jose Yap, et al. and ordered the survey of Boracay for titling
purposes. The second is G.R. No. 173775, a petition for prohibition, mandamus, and nullification
of Proclamation No. 1064[3] issued by President Gloria Macapagal-Arroyo classifying Boracay into
reserved forest and agricultural land.

The Antecedents

G.R. No. 167707

Boracay Island in the Municipality of Malay, Aklan, with its powdery white sand beaches
and warm crystalline waters, is reputedly a premier Philippine tourist destination. The island is
also home to 12,003 inhabitants[4] who live in the bone-shaped island’s three barangays.[5]

400
On April 14, 1976, the Department of Environment and Natural Resources (DENR)
approved the National Reservation Survey of Boracay
Island,[6] which identified several lots as being occupied or claimed by named persons.[7]

On November 10, 1978, then President Ferdinand Marcos issued Proclamation


[8]
No. 1801 declaring Boracay Island, among other islands, caves and peninsulas in
the Philippines, as tourist zones and marine reserves under the administration of the Philippine
Tourism Authority (PTA). President Marcos later approved the issuance of PTA Circular 3-
82[9] datedSeptember 3, 1982, to implement Proclamation No. 1801.

Claiming that Proclamation No. 1801 and PTA Circular No 3-82 precluded them from filing
an application for judicial confirmation of imperfect title or survey of land for titling purposes,
respondents-claimants

401
Mayor Jose S. Yap, Jr., Libertad Talapian, Mila Y. Sumndad, and Aniceto Yap filed a petition
for declaratory relief with theRTC in Kalibo, Aklan.

In their petition, respondents-claimants alleged that Proclamation No. 1801


and PTA Circular No. 3-82 raised doubts on their right to secure titles over their occupied
lands. They declared that they themselves, or through their predecessors-in-interest, had been
in open, continuous, exclusive, and notorious possession and occupation in Boracay since June
12, 1945, or earlier since time immemorial. They declared their lands for tax purposes and paid
realty taxes on them.[10]

Respondents-claimants posited that Proclamation No. 1801 and its implementing Circular
did not place Boracay beyond the commerce of man. Since the Island was classified as a tourist
zone, it was susceptible of private ownership. Under Section 48(b) of Commonwealth Act (CA)
No. 141, otherwise known as the Public Land Act, they had the right to have the lots registered
in their names through judicial confirmation of imperfect titles.

The Republic, through the Office of the Solicitor General (OSG), opposed the petition for
declaratory relief. The OSGcountered that Boracay Island was an unclassified land of the public
domain. It formed part of the mass of lands classified as “public forest,” which was not available
for disposition pursuant to Section 3(a) of Presidential Decree (PD) No. 705 or the Revised
Forestry Code,[11] as amended.

The OSG maintained that respondents-claimants’ reliance on PD No. 1801


and PTA Circular No. 3-82 was misplaced. Their right to judicial confirmation of title was
governed by CA No. 141 and PD No. 705. Since Boracay Island had not been classified as
alienable and disposable, whatever possession they had cannot ripen into ownership.

During pre-trial, respondents-claimants and the OSG stipulated on the following facts: (1)
respondents-claimants were presently in possession of parcels of land in Boracay Island; (2)
these parcels of land were planted with coconut trees and other natural growing trees; (3) the
coconut trees had heights of more or less twenty (20) meters and were planted more or less
fifty (50) years ago; and (4) respondents-claimants declared the land they were occupying for
tax purposes.[12]

The parties also agreed that the principal issue for resolution was purely legal: whether
Proclamation No. 1801 posed any legal hindrance or impediment to the titling of the lands in

402
Boracay. They decided to forego with the trial and to submit the case for resolution upon
submission of their respective memoranda.[13]

The RTC took judicial notice[14] that certain parcels of land in Boracay Island, more
particularly Lots 1 and 30, PlanPSU-5344, were covered by Original Certificate of Title No. 19502
(RO 2222) in the name of the Heirs of Ciriaco S. Tirol. These lots were involved in Civil Case
Nos. 5222 and 5262 filed before the RTC of Kalibo, Aklan.[15] The titles were issued on
August 7, 1933.[16]

RTC and CA Dispositions

On July 14, 1999, the RTC rendered a decision in favor of respondents-claimants, with
a fallo reading:

WHEREFORE, in view of the foregoing, the Court declares that Proclamation


No. 1801 and PTA Circular No. 3-82 pose no legal obstacle to the petitioners and
those similarly situated to acquire title to their lands in Boracay, in accordance with
the applicable laws and in the manner prescribed therein; and to have their lands
surveyed and approved by respondent Regional Technical Director of Lands as the
approved survey does not in itself constitute a title to the land.

SO ORDERED.[17]

The RTC upheld respondents-claimants’ right to have their occupied lands titled in their
name. It ruled that neither Proclamation No. 1801 nor PTA Circular No. 3-82 mentioned that
lands in Boracay were inalienable or could not be the subject of disposition.[18] The Circular itself
recognized private ownership of lands.[19] The trial court cited Sections 87[20] and 53[21] of the
Public Land Act as basis for acknowledging private ownership of lands in Boracay and that only
those forested areas in public lands were declared as part of the forest reserve.[22]

The OSG moved for reconsideration but its motion was denied.[23] The Republic then
appealed to the CA.

On December 9, 2004, the appellate court affirmed in toto the RTC decision, disposing as
follows:

WHEREFORE, in view of the foregoing premises, judgment is hereby rendered


by us DENYING the appeal filed in this case and AFFIRMING the decision of the
lower court.[24]

403
The CA held that respondents-claimants could not be prejudiced by a declaration that the
lands they occupied since time immemorial were part of a forest reserve.

Again, the OSG sought reconsideration but it was similarly denied.[25] Hence, the present
petition under Rule 45.

G.R. No. 173775

On May 22, 2006, during the pendency of G.R. No. 167707, President Gloria Macapagal-
Arroyo issued Proclamation No. 1064[26] classifying Boracay Island into four hundred (400)
hectares of reserved forest land (protection purposes) and six hundred twenty-eight and 96/100
(628.96) hectares of agricultural land (alienable and disposable). The Proclamation likewise
provided for a fifteen-meter buffer zone on each side of the centerline of roads and trails,
reserved for right-of-way and which shall form part of the area reserved for forest land
protection purposes.

On August 10, 2006, petitioners-claimants Dr. Orlando Sacay,[27] Wilfredo Gelito,[28] and
other landowners[29] in Boracay filed with this Court an original petition for prohibition,
mandamus, and nullification of Proclamation No. 1064.[30] They allege that the Proclamation
infringed on their “prior vested rights” over portions of Boracay. They have been in continued
possession of their respective lots in Boracay since time immemorial. They have also invested
billions of pesos in developing their lands and building internationally renowned first class resorts
on their lots.[31]

Petitioners-claimants contended that there is no need for a proclamation reclassifying


Boracay into agricultural land. Being classified as neither mineral nor timber land, the island
is deemed agricultural pursuant to the Philippine Bill of 1902 and Act No. 926, known as the
first Public Land Act.[32] Thus, their possession in the concept of owner for the required period
entitled them to judicial confirmation of imperfect title.

Opposing the petition, the OSG argued that petitioners-claimants do not have a vested
right over their occupied portions in the island. Boracay is an unclassified public forest land
pursuant to Section 3(a) of PD No. 705. Being public forest, the claimed portions of the island

404
are inalienable and cannot be the subject of judicial confirmation of imperfect title. It is only the
executive department, not the courts, which has authority to reclassify lands of the public
domain into alienable and disposable lands. There is a need for a positive government act in
order to release the lots for disposition.

On November 21, 2006, this Court ordered the consolidation of the two petitions as they
principally involve the same issues on the land classification of Boracay Island.[33]

Issues

G.R. No. 167707

The OSG raises the lone issue of whether Proclamation No. 1801 and PTA Circular No. 3-
82 pose any legal obstacle for respondents, and all those similarly situated, to acquire title to
their occupied lands in Boracay Island.[34]

G.R. No. 173775

Petitioners-claimants hoist five (5) issues, namely:

I.
AT THE TIME OF THE ESTABLISHED POSSESSION OF PETITIONERS IN CONCEPT OF
OWNER OVER THEIR RESPECTIVE AREAS IN BORACAY, SINCE TIME IMMEMORIAL
OR AT THE LATEST SINCE 30 YRS. PRIOR TO THE FILING OF THE PETITION FOR
DECLARATORY RELIEF ON NOV. 19, 1997, WERE THE AREAS OCCUPIED BY THEM
PUBLIC AGRICULTURAL LANDS AS DEFINED BY LAWS THEN ON JUDICIAL
CONFIRMATION OF IMPERFECT TITLES OR PUBLIC FOREST AS DEFINED BY SEC.
3a, PD 705?

II.
HAVE PETITIONERS OCCUPANTS ACQUIRED PRIOR VESTED RIGHT OF PRIVATE
OWNERSHIP OVER THEIR OCCUPIED PORTIONS OF BORACAY LAND, DESPITE
THE FACT THAT THEY HAVE NOT APPLIED YET FOR JUDICIAL CONFIRMATION OF
IMPERFECT TITLE?

III.
IS THE EXECUTIVE DECLARATION OF THEIR AREAS AS
ALIENABLE AND DISPOSABLE UNDER SEC 6, CA 141 [AN] INDISPENSABLE PRE-
REQUISITE FOR PETITIONERS TO OBTAIN TITLE UNDER THE TORRENS SYSTEM?

405
IV.
IS THE ISSUANCE OF PROCLAMATION 1064 ON MAY 22, 2006, VIOLATIVE OF THE
PRIOR VESTED RIGHTS TO PRIVATE OWNERSHIP OF PETITIONERS OVER THEIR
LANDS IN BORACAY, PROTECTED BY THE DUE PROCESS CLAUSE OF THE
CONSTITUTION OR IS PROCLAMATION 1064 CONTRARY TO SEC. 8, CA 141,
OR SEC. 4(a) OF RA 6657.

V.
CAN RESPONDENTS BE COMPELLED BY MANDAMUS TO ALLOW THE
SURVEY AND TO APPROVE THE SURVEY PLANSFOR PURPOSES OF THE
APPLICATION FOR TITLING OF THE LANDS OF PETITIONERS IN
[35]
BORACAY? (Underscoring supplied)

In capsule, the main issue is whether private claimants (respondents-claimants in G.R.


No. 167707 and petitioners-claimants in G.R. No. 173775) have a right to secure titles over their
occupied portions in Boracay. The twin petitions pertain to their right, if any, to judicial
confirmation of imperfect title under CA No. 141, as amended. They do not involve their right to
secure title under other pertinent laws.

Our Ruling

Regalian Doctrine and power of the executive


to reclassify lands of the public domain

Private claimants rely on three (3) laws and executive acts in their bid for judicial
confirmation of imperfect title, namely: (a) Philippine Bill of 1902[36] in relation to Act No. 926,
later amended and/or superseded by Act No. 2874 and CA No. 141;[37] (b) Proclamation No.
1801[38] issued by then President Marcos; and (c) Proclamation No. 1064[39] issued by President
Gloria Macapagal-Arroyo. We shall proceed to determine their rights to apply for judicial
confirmation of imperfect title under these laws and executive acts.

But first, a peek at the Regalian principle and the power of the executive to reclassify
lands of the public domain.

The 1935 Constitution classified lands of the public domain into agricultural, forest or
timber.[40] Meanwhile, the 1973 Constitution provided the following divisions: agricultural,
industrial or commercial, residential, resettlement, mineral, timber or forest and grazing lands,
and such other classes as may be provided by law,[41] giving the government great leeway for
classification.[42] Then the 1987 Constitution reverted to the 1935 Constitution classification with
one addition: national parks.[43] Of these, only agricultural lands may be alienated.[44] Prior to

406
Proclamation No. 1064 of May 22, 2006, BoracayIsland had never been expressly and
administratively classified under any of these grand divisions. Boracay was an unclassified land
of the public domain.

The Regalian Doctrine dictates that all lands of the public domain belong to the State, that
the State is the source of any asserted right to ownership of land and charged with the
conservation of such patrimony.[45] The doctrine has been consistently adopted under the 1935,
1973, and 1987 Constitutions.[46]

All lands not otherwise appearing to be clearly within private ownership are presumed to
belong to the State.[47] Thus, all lands that have not been acquired from the government, either
by purchase or by grant, belong to the State as part of the inalienable public
[48]
domain. Necessarily, it is up to the State to determine if lands of the public domain will be
disposed of for private ownership. The government, as the agent of the state, is possessed of
the plenary power as the persona in law to determine who shall be the favored recipients of
public lands, as well as under what terms they may be granted such privilege, not excluding the
placing of obstacles in the way of their exercise of what otherwise would be ordinary acts of
ownership.[49]

Our present land law traces its roots to the Regalian Doctrine. Upon the Spanish conquest
of the Philippines, ownership of all lands, territories and possessions in the Philippines passed to
the Spanish Crown.[50] The Regalian doctrine was first introduced in the Philippines through
the Laws of the Indies and the Royal Cedulas, which laid the foundation that “all lands that were
not acquired from the Government, either by purchase or by grant, belong to the public
domain.”[51]

The Laws of the Indies was followed by the Ley Hipotecaria or the Mortgage Law of
1893. The Spanish Mortgage Law provided for the systematic registration of titles and deeds as
well as possessory claims.[52]

The Royal Decree of 1894 or the Maura Law[53] partly amended the Spanish Mortgage
Law and the Laws of the Indies. It established possessory information as the method of
legalizing possession of vacant Crown land, under certain conditions which were set forth in said
decree.[54] Under Section 393 of the Maura Law, an informacion posesoria or possessory
information title,[55] when duly inscribed in the Registry of Property, is converted into a title of
ownership only after the lapse of twenty (20) years of uninterrupted possession which must be

407
actual, public, and adverse,[56] from the date of its inscription.[57]However, possessory
information title had to be perfected one year after the promulgation of the Maura Law, or
until April 17, 1895. Otherwise, the lands would revert to the State.[58]

In sum, private ownership of land under the Spanish regime could only be founded on
royal concessions which took various forms, namely: (1) titulo real or royal grant;
(2) concesion especial or special grant; (3) composicion con el estado or adjustment title;
(4) titulo de compra or title by purchase; and (5) informacion posesoria or possessory
information title.[59]

The first law governing the disposition of public lands in the Philippines under American
rule was embodied in the Philippine Bill of 1902.[60] By this law, lands of the public domain in
the Philippine Islands were classified into three (3) grand divisions, to wit: agricultural, mineral,
and timber or forest lands.[61] The act provided for, among others, the disposal of mineral lands
by means of absolute grant (freehold system) and by lease (leasehold system).[62] It also
provided the definition by exclusion of “agricultural public lands.”[63] Interpreting the meaning of
“agricultural lands” under the Philippine Bill of 1902, the Court declared in Mapa v. Insular
Government:[64]

x x x In other words, that the phrase “agricultural land” as used in Act No.
926 means those public lands acquired from Spainwhich are not timber or mineral
lands. x x x[65] (Emphasis Ours)

On February 1, 1903, the Philippine Legislature passed Act No. 496, otherwise known as
the Land Registration Act. The act established a system of registration by which recorded title
becomes absolute, indefeasible, and imprescriptible. This is known as the Torrens system.[66]

Concurrently, on October 7, 1903, the Philippine Commission passed Act No. 926, which
was the first Public Land Act. The Act introduced the homestead system and made provisions for
judicial and administrative confirmation of imperfect titles and for the sale or lease of public
lands. It permitted corporations regardless of the nationality of persons owning the controlling
stock to lease or purchase lands of the public domain.[67] Under the Act, open, continuous,
exclusive, and notorious possession and occupation of agricultural lands for the next ten (10)
years preceding July 26, 1904 was sufficient for judicial confirmation of imperfect title.[68]

408
On November 29, 1919, Act No. 926 was superseded by Act No. 2874, otherwise known
as the second Public Land Act. This new, more comprehensive law limited the exploitation of
agricultural lands to Filipinos and Americans and citizens of other countries which gave Filipinos
the same privileges. For judicial confirmation of title, possession and occupation en concepto
dueño since time immemorial, or since July 26, 1894, was required.[69]

After the passage of the 1935 Constitution, CA No. 141 amended Act No. 2874
on December 1, 1936. To this day, CA No. 141, as amended, remains as the existing general
law governing the classification and disposition of lands of the public domain other than timber
and mineral lands,[70] and privately owned lands which reverted to the State.[71]

Section 48(b) of CA No. 141 retained the requirement under Act No. 2874 of possession
and occupation of lands of the public domain since time immemorial or since July 26,
1894. However, this provision was superseded by Republic Act (RA) No. 1942,[72] which
provided for a simple thirty-year prescriptive period for judicial confirmation of imperfect
title. The provision was last amended by PD No. 1073,[73] which now provides for possession
and occupation of the land applied forsince June 12, 1945, or earlier.[74]

The issuance of PD No. 892[75] on February 16, 1976 discontinued the use of Spanish titles
as evidence in land registration proceedings.[76] Under the decree, all holders of Spanish titles or
grants should apply for registration of their lands under Act No. 496 within six (6) months from
the effectivity of the decree on February 16, 1976. Thereafter, the recording of allunregistered
lands[77] shall be governed by Section 194 of the Revised Administrative Code, as amended by
Act No. 3344.

On June 11, 1978, Act No. 496 was amended and updated by PD No. 1529, known as the
Property Registration Decree. It was enacted to codify the various laws relative to registration of
property.[78] It governs registration of lands under theTorrens system as well as unregistered
lands, including chattel mortgages.[79]

A positive act declaring land as alienable and disposable is required. In keeping with the
presumption of State ownership, the Court has time and again emphasized that there must be
a positive act of the government, such as an official proclamation,[80] declassifying inalienable
public land into disposable land for agricultural or other purposes.[81] In fact, Section 8 of CA
No. 141 limits alienable or disposable lands only to those lands which have been “officially
delimited and classified.”[82]

409
The burden of proof in overcoming the presumption of State ownership of the lands of the
public domain is on the person applying for registration (or claiming ownership), who must prove
that the land subject of the application is alienable or disposable.[83] To overcome this
presumption, incontrovertible evidence must be established that the land subject of the
application (or claim) is alienable or disposable.[84] There must still be a positive act declaring
land of the public domain as alienable and disposable. To prove that the land subject of an
application for registration is alienable, the applicant must establish the existence of a positive
act of the government such as a presidential proclamation or an executive order; an
administrative action; investigation reports of Bureau of Lands investigators; and a legislative
act or a statute.[85] The applicant may also secure a certification from the government that the
land claimed to have been possessed for the required number of years is alienable and
disposable.[86]

In the case at bar, no such proclamation, executive order, administrative action, report,
statute, or certification was presented to the Court. The records are bereft of evidence showing
that, prior to 2006, the portions of Boracay occupied by private claimants were subject of a
government proclamation that the land is alienable and disposable. Absent such well-nigh
incontrovertible evidence, the Court cannot accept the submission that lands occupied by private
claimants were already open to disposition before 2006. Matters of land classification or
reclassification cannot be assumed. They call for proof.[87]

Ankron and De Aldecoa did not make the whole of Boracay Island, or portions of it,
agricultural lands. Private claimants posit that Boracay was already an agricultural land pursuant
to the old cases Ankron v. Government of the PhilippineIslands (1919)[88] and De Aldecoa v. The
Insular Government (1909).[89] These cases were decided under the provisions of the Philippine
Bill of 1902 and Act No. 926. There is a statement in these old cases that “in the absence of
evidence to the contrary, that in each case the lands are agricultural lands until the contrary is
shown.”[90]

Private claimants’ reliance on Ankron and De Aldecoa is misplaced. These cases did not
have the effect of converting the whole of Boracay Island or portions of it into agricultural
lands. It should be stressed that the Philippine Bill of 1902 and Act No. 926 merely provided the
manner through which land registration courts would classify lands of the public
domain. Whether the land would be classified as timber, mineral, or agricultural depended on
proof presented in each case.

410
Ankron and De Aldecoa were decided at a time when the President of the Philippines had
no power to classify lands of the public domain into mineral, timber, and agricultural. At that
time, the courts were free to make corresponding classifications in justiciable cases, or were
vested with implicit power to do so, depending upon the preponderance of the evidence.[91] This
was the Court’s ruling in Heirs of the Late Spouses Pedro S. Palanca and Soterranea Rafols Vda.
De Palanca v. Republic,[92] in which it stated, through Justice Adolfo Azcuna, viz.:

x x x Petitioners furthermore insist that a particular land need not be


formally released by an act of the Executive before it can be deemed open to
private ownership, citing the cases of Ramos v. Director of Lands and Ankron v.
Government of the Philippine Islands.

xxxx

Petitioner’s reliance upon Ramos v. Director of Lands and Ankron v.


Government is misplaced. These cases were decided under the Philippine Bill of
1902 and the first Public Land Act No. 926 enacted by the Philippine Commission
on October 7, 1926, under which there was no legal provision vesting in the Chief
Executive or President of the Philippines the power to classify lands of the public
domain into mineral, timber and agricultural so that the courts then were free to
make corresponding classifications in justiciable cases, or were vested with implicit
power to do so, depending upon the preponderance of the evidence.[93]

To aid the courts in resolving land registration cases under Act No. 926, it was then
necessary to devise a presumption on land classification. Thus evolved the dictum
in Ankron that “the courts have a right to presume, in the absence of evidence to the contrary,
that in each case the lands are agricultural lands until the contrary is shown.”[94]

But We cannot unduly expand the presumption in Ankron and De Aldecoa to an argument
that all lands of the public domain had been automatically reclassified as disposable and
alienable agricultural lands. By no stretch of imagination did the presumption convert all lands
of the public domain into agricultural lands.

If We accept the position of private claimants, the Philippine Bill of 1902 and Act No. 926
would have automatically made all lands in the Philippines, except those already classified as
timber or mineral land, alienable and disposable lands. That would take these lands out of

411
State ownership and worse, would be utterly inconsistent with and totally repugnant to the long-
entrenched Regalian doctrine.

The presumption in Ankron and De Aldecoa attaches only to land registration cases
brought under the provisions of Act No. 926, or more specifically those cases dealing with
judicial and administrative confirmation of imperfect titles. The presumption applies to an
applicant for judicial or administrative conformation of imperfect title under Act No. 926. It
certainly cannot apply to landowners, such as private claimants or their predecessors-in-interest,
who failed to avail themselves of the benefits of Act No. 926. As to them, their land remained
unclassified and, by virtue of the Regalian doctrine, continued to be owned by the State.

In any case, the assumption in Ankron and De Aldecoa was not absolute. Land
classification was, in the end, dependent on proof. If there was proof that the land was better
suited for non-agricultural uses, the courts could adjudge it as a mineral or timber land despite
the presumption. In Ankron, this Court stated:

In the case of Jocson vs. Director of Forestry (supra), the Attorney-General


admitted in effect that whether the particular land in question belongs to one class
or another is a question of fact. The mere fact that a tract of land has trees upon it
or has mineral within it is not of itself sufficient to declare that one is forestry land
and the other, mineral land. There must be some proof of the extent and present
or future value of the forestry and of the minerals. While, as we have just
said, many definitions have been given for “agriculture,” “forestry,” and “mineral”
lands, and that in each case it is a question of fact, we think it is safe to say that in
order to be forestry or mineral land the proof must show that it is more valuable for
the forestry or the mineral which it contains than it is for agricultural purposes.
(Sec. 7, Act No. 1148.) It is not sufficient to show that there exists some trees
upon the land or that it bears some mineral. Land may be classified as forestry or
mineral today, and, by reason of the exhaustion of the timber or mineral, be
classified as agricultural land tomorrow. And vice-versa, by reason of the rapid
growth of timber or the discovery of valuable minerals, lands classified as
agricultural today may be differently classified tomorrow. Each case must be
decided upon the proof in that particular case, having regard for its present or
future value for one or the other purposes. We believe, however, considering the
fact that it is a matter of public knowledge that a majority of the lands in the
Philippine Islands are agricultural lands that the courts have a right to presume, in
the absence of evidence to the contrary, that in each case the lands are agricultural
lands until the contrary is shown. Whatever the land involved in a particular land
registration case is forestry or mineral land must, therefore, be a matter of
proof. Its superior value for one purpose or the other is a question of fact to be
settled by the proof in each particular case. The fact that the land is a manglar
[mangrove swamp] is not sufficient for the courts to decide whether it is
agricultural, forestry, or mineral land. It may perchance belong to one or the other
of said classes of land. The Government, in the first instance, under the provisions
412
of Act No. 1148, may, by reservation, decide for itself what portions of public land
shall be considered forestry land, unless private interests have intervened before
such reservation is made. In the latter case, whether the land is agricultural,
forestry, or mineral, is a question of proof. Until private interests have intervened,
the Government, by virtue of the terms of said Act (No. 1148), may decide for itself
what portions of the “public domain” shall be set aside and reserved as forestry or
mineral land. (Ramos vs. Director of Lands, 39 Phil. 175; Jocson vs. Director of
Forestry, supra)[95] (Emphasis ours)

Since 1919, courts were no longer free to determine the classification of lands from the
facts of each case, except those that have already became private lands.[96] Act No. 2874,
promulgated in 1919 and reproduced in Section 6 of CA No. 141, gave the Executive
Department, through the President, the exclusive prerogative to classify or reclassify public
lands into alienable or disposable, mineral or forest.96-a Since then, courts no longer had the
authority, whether express or implied, to determine the classification of lands of the public
domain.[97]

Here, private claimants, unlike the Heirs of Ciriaco Tirol who were issued their title in
1933,[98] did not present a justiciable case for determination by the land registration court of the
property’s land classification. Simply put, there was no opportunity for the courts then to
resolve if the land the Boracay occupants are now claiming were agricultural lands. When Act
No. 926 was supplanted by Act No. 2874 in 1919, without an application for judicial confirmation
having been filed by private claimants or their predecessors-in-interest, the courts were no
longer authorized to determine the property’s land classification. Hence, private claimants
cannot bank on Act No. 926.

We note that the RTC decision[99] in G.R. No. 167707 mentioned Krivenko v. Register of
Deeds of Manila,[100] which was decided in 1947 when CA No. 141, vesting the Executive with the
sole power to classify lands of the public domain was already in effect. Krivenko cited the old
cases Mapa v. Insular Government,[101] De Aldecoa v. The Insular Government,[102]and Ankron v.
Government of the Philippine Islands.[103]

Krivenko, however, is not controlling here because it involved a totally different issue. The
pertinent issue in Krivenko was whether residential lots were included in the general
classification of agricultural lands; and if so, whether an alien could acquire a residential
lot. This Court ruled that as an alien, Krivenko was prohibited by the 1935 Constitution[104] from
acquiring agricultural land, which included residential lots. Here, the issue is whether
unclassified lands of the public domain are automatically deemed agricultural.

413
Notably, the definition of “agricultural public lands” mentioned in Krivenko relied on the
old cases decided prior to the enactment of Act No. 2874, including Ankron and De
[105]
Aldecoa. As We have already stated, those cases cannot apply here, since they were decided
when the Executive did not have the authority to classify lands as agricultural, timber, or
mineral.

Private claimants’ continued possession under Act No. 926 does not create a presumption
that the land is alienable. Private claimants also contend that their continued possession of
portions of Boracay Island for the requisite period of ten (10) years under Act No. 926[106] ipso
facto converted the island into private ownership. Hence, they may apply for a title in their
name.

A similar argument was squarely rejected by the Court in Collado v. Court of


Appeals.[107] Collado, citing the separate opinion of now Chief Justice Reynato S. Puno in Cruz v.
Secretary of Environment and Natural Resources,107-a ruled:

“Act No. 926, the first Public Land Act, was passed in pursuance
of the provisions of the Philippine Bill of 1902. The law governed the
disposition of lands of the public domain. It prescribed rules and
regulations for the homesteading, selling and leasing of portions of the
public domain of the Philippine Islands, and prescribed the terms and
conditions to enable persons to perfect their titles to public lands in
the Islands. It also provided for the “issuance of patents to certain
native settlers upon public lands,” for the establishment of town sites
and sale of lots therein, for the completion of imperfect titles, and for
the cancellation or confirmation of Spanish concessions and grants in
the Islands.” In short, the Public Land Act operated on the assumption
that title to public lands in the Philippine Islands remained in the
government; and that the government’s title to public land sprung
from the Treaty of Paris and other subsequent treaties between Spain
and the United States. The term “public land” referred to all lands of
the public domain whose title still remained in the government and are
thrown open to private appropriation and settlement, and excluded the
patrimonial property of the government and the friar lands.”

Thus, it is plain error for petitioners to argue that under the Philippine Bill of 1902
and Public Land Act No. 926, mere possession by private individuals of lands
creates the legal presumption that the lands are alienable and
[108]
disposable. (Emphasis Ours)

Except for lands already covered by existing titles, Boracay was an unclassified land of the
public domain prior to Proclamation No. 1064. Such unclassified lands are considered public
414
forest under PD No. 705. The DENR[109] and the National Mapping and Resource Information
Authority[110] certify that Boracay Island is an unclassified land of the public domain.

PD No. 705 issued by President Marcos categorized all unclassified lands of the public
domain as public forest. Section 3(a) of PD No. 705 defines a public forest as “a mass of lands
of the public domain which has not been the subject of the present system of classification for
the determination of which lands are needed for forest purpose and which are not.” Applying PD
No. 705, all unclassified lands, including those in Boracay Island, are ipso facto considered public
forests. PD No. 705, however, respects titles already existing prior to its effectivity.

The Court notes that the classification of Boracay as a forest land under PD No. 705 may
seem to be out of touch with the present realities in the island. Boracay, no doubt, has been
partly stripped of its forest cover to pave the way for commercial developments. As a premier
tourist destination for local and foreign tourists, Boracay appears more of a commercial island
resort, rather than a forest land.

Nevertheless, that the occupants of Boracay have built multi-million peso beach resorts on
the island;[111] that the island has already been stripped of its forest cover; or that the
implementation of Proclamation No. 1064 will destroy the island’s tourism industry,
do not negate its character as public forest.

Forests, in the context of both the Public Land Act and the Constitution[112] classifying
lands of the public domain into “agricultural, forest or timber, mineral lands, and national
parks,” do not necessarily refer to large tracts of wooded land or expanses covered by dense
growths of trees and underbrushes.[113] The discussion in Heirs of Amunategui v. Director of
Forestry[114] is particularly instructive:

A forested area classified as forest land of the public domain does not lose
such classification simply because loggers or settlers may have stripped it of its
forest cover. Parcels of land classified as forest land may actually be covered with
grass or planted to crops by kaingincultivators or other farmers. “Forest lands” do
not have to be on mountains or in out of the way places. Swampy areas covered
by mangrove trees, nipa palms, and other trees growing in brackish or sea water
may also be classified as forest land. The classification is descriptive of its legal
nature or status and does not have to be descriptive of what the land actually looks
like. Unless and until the land classified as “forest” is released in an official
proclamation to that effect so that it may form part of the disposable agricultural
lands of the public domain, the rules on confirmation of imperfect title do not
apply.[115] (Emphasis supplied)

415
There is a big difference between “forest” as defined in a dictionary and “forest or timber
land” as a classification of lands of the public domain as appearing in our statutes. One is
descriptive of what appears on the land while the other is a legal status, a classification for legal
purposes.[116] At any rate, the Court is tasked to determine the legal status of Boracay Island,
and not look into its physical layout. Hence, even if its forest cover has been replaced by beach
resorts, restaurants and other commercial establishments, it has not been automatically
converted from public forest to alienable agricultural land.

Private claimants cannot rely on Proclamation No. 1801 as basis for judicial confirmation
of imperfect title. The proclamation did not convert Boracay into an agricultural land. However,
private claimants argue that Proclamation No. 1801 issued by then President Marcos in 1978
entitles them to judicial confirmation of imperfect title. The Proclamation classified Boracay,
among other islands, as a tourist zone. Private claimants assert that, as a tourist spot, the
island is susceptible of private ownership.

Proclamation No. 1801 or PTA Circular No. 3-82 did not convert the whole of Boracay into
an agricultural land. There is nothing in the law or the Circular which made Boracay Island an
agricultural land. The reference in Circular No. 3-82 to “private lands”[117] and “areas declared
as alienable and disposable”[118] does not by itself classify the entire island as
agricultural. Notably, Circular No. 3-82 makes reference not only to private lands and areas but
also to public forested lands. Rule VIII, Section 3 provides:

No trees in forested private lands may be cut without prior authority from
the PTA. All forested areas in public lands are declared forest reserves. (Emphasis
supplied)

Clearly, the reference in the Circular to both private and public lands merely recognizes
that the island can be classified by the Executive department pursuant to its powers under CA
No. 141. In fact, Section 5 of the Circular recognizes the then Bureau of Forest Development’s
authority to declare areas in the island as alienable and disposable when it provides:

Subsistence farming, in areas declared as alienable and disposable by the


Bureau of Forest Development.

Therefore, Proclamation No. 1801 cannot be deemed the positive act needed to
classify Boracay Island as alienable and disposable land. If President Marcos intended to classify
the island as alienable and disposable or forest, or both, he would have identified the specific
416
limits of each, as President Arroyo did in Proclamation No. 1064. This was not done in
Proclamation No. 1801.

The Whereas clauses of Proclamation No. 1801 also explain the rationale behind the
declaration of Boracay Island, together with other islands, caves and peninsulas in the
Philippines, as a tourist zone and marine reserve to be administered by thePTA – to ensure the
concentrated efforts of the public and private sectors in the development of the areas’ tourism
potential with due regard for ecological balance in the marine environment. Simply put, the
proclamation is aimed at administering the islands for tourism and ecological purposes. It does
not address the areas’ alienability.[119]

More importantly, Proclamation No. 1801 covers not only Boracay Island, but sixty-four
(64) other islands, coves, and peninsulas in the Philippines, such as Fortune and Verde Islands in
Batangas, Port Galera in Oriental Mindoro, Panglao and Balicasag Islands in Bohol, Coron Island,
Puerto Princesa and surrounding areas in Palawan, Camiguin Island in Cagayan de Oro, and
Misamis Oriental, to name a few. If the designation of Boracay Island as tourist zone makes it
alienable and disposable by virtue of Proclamation No. 1801, all the other areas mentioned
would likewise be declared wide open for private disposition. That could not have been, and is
clearly beyond, the intent of the proclamation.

It was Proclamation No. 1064 of 2006 which positively declared part of Boracay as
alienable and opened the same to private ownership. Sections 6 and 7 of CA No.
141[120] provide that it is only the President, upon the recommendation of the proper department
head, who has the authority to classify the lands of the public domain into alienable or
disposable, timber and mineral lands.[121]

In issuing Proclamation No. 1064, President Gloria Macapagal-Arroyo merely exercised the
authority granted to her to classify lands of the public domain, presumably subject to existing
vested rights. Classification of public lands is the exclusive prerogative of the Executive
Department, through the Office of the President. Courts have no authority to do so.[122] Absent
such classification, the land remains unclassified until released and rendered open to
disposition.[123]

Proclamation No. 1064 classifies Boracay into 400 hectares of reserved forest land and
628.96 hectares of agricultural land. The Proclamation likewise provides for a 15-meter buffer
zone on each side of the center line of roads and trails, which are reserved for right of way and
which shall form part of the area reserved for forest land protection purposes.
417
Contrary to private claimants’ argument, there was nothing invalid or irregular, much less
unconstitutional, about the classification of Boracay Island made by the President through
Proclamation No. 1064. It was within her authority to make such classification, subject to
existing vested rights.

Proclamation No. 1064 does not violate the Comprehensive Agrarian Reform Law. Private
claimants further assert that Proclamation No. 1064 violates the provision of the Comprehensive
Agrarian Reform Law (CARL) or RA No. 6657 barring conversion of public forests into agricultural
lands. They claim that since Boracay is a public forest under PD No. 705, President Arroyo can
no longer convert it into an agricultural land without running afoul of Section 4(a) of RA No.
6657, thus:

SEC. 4. Scope. – The Comprehensive Agrarian Reform Law of 1988 shall


cover, regardless of tenurial arrangement and commodity produced, all public and
private agricultural lands as provided in Proclamation No. 131 and Executive Order
No. 229, including other lands of the public domain suitable for agriculture.

More specifically, the following lands are covered by the Comprehensive


Agrarian Reform Program:

(a) All alienable and disposable lands of the public domain devoted to
or suitable for agriculture. No reclassification of forest or mineral
lands to agricultural lands shall be undertaken after the approval
of this Act until Congress, taking into account ecological,
developmental and equity considerations, shall have determined
by law, the specific limits of the public domain.

That Boracay Island was classified as a public forest under PD No. 705 did not bar the
Executive from later converting it into agricultural land. Boracay Island still remained an
unclassified land of the public domain despite PD No. 705.

In Heirs of the Late Spouses Pedro S. Palanca and Soterranea Rafols v. Republic,[124] the
Court stated that unclassified lands are public forests.

While it is true that the land classification map does not categorically state
that the islands are public forests, the fact that they were unclassified lands leads
to the same result. In the absence of the classification as mineral or timber land,
the land remains unclassified land until released and rendered open to
disposition.[125] (Emphasis supplied)

418
Moreover, the prohibition under the CARL applies only to a “reclassification” of land. If
the land had never been previously classified, as in the case of Boracay, there can be no
prohibited reclassification under the agrarian law. We agree with the opinion of the Department
of Justice[126] on this point:

Indeed, the key word to the correct application of the prohibition in Section
4(a) is the word “reclassification.” Where there has been no previous classification
of public forest [referring, we repeat, to the mass of the public domain which has
not been the subject of the present system of classification for purposes of
determining which are needed for forest purposes and which are not] into
permanent forest or forest reserves or some other forest uses under the Revised
Forestry Code, there can be no “reclassification of forest lands” to speak of within
the meaning of Section 4(a).

Thus, obviously, the prohibition in Section 4(a) of the CARL against the
reclassification of forest lands to agricultural lands without a prior law delimiting the
limits of the public domain, does not, and cannot, apply to those lands of the public
domain, denominated as “public forest” under the Revised Forestry Code, which
have not been previously determined, or classified, as needed for forest purposes in
accordance with the provisions of the Revised Forestry Code.[127]

Private claimants are not entitled to apply for judicial confirmation of imperfect title under
CA No. 141. Neither do they have vested rights over the occupied lands under the said
law. There are two requisites for judicial confirmation of imperfect or incomplete title under CA
No. 141, namely: (1) open, continuous, exclusive, and notorious possession and occupation of
the subject land by himself or through his predecessors-in-interest under a bona fide claim of
ownership since time immemorial or from June 12, 1945; and (2) the classification of the land as
alienable and disposable land of the public domain.[128]

As discussed, the Philippine Bill of 1902, Act No. 926, and Proclamation No. 1801 did not
convert portions of BoracayIsland into an agricultural land. The island remained an unclassified
land of the public domain and, applying the Regalian doctrine, is considered State property.

Private claimants’ bid for judicial confirmation of imperfect title, relying on the Philippine
Bill of 1902, Act No. 926, and Proclamation No. 1801, must fail because of the absence of the
second element of alienable and disposable land. Their entitlement to a government grant
under our present Public Land Act presupposes that the land possessed and applied for is
already alienable and disposable. This is clear from the wording of the law itself.[129] Where the
land is not alienable and disposable, possession of the land, no matter how long, cannot confer
ownership or possessory rights.[130]

419
Neither may private claimants apply for judicial confirmation of imperfect title under
Proclamation No. 1064, with respect to those lands which were classified as agricultural
lands. Private claimants failed to prove the first element of open, continuous, exclusive, and
notorious possession of their lands in Boracay since June 12, 1945.

We cannot sustain the CA and RTC conclusion in the petition for declaratory relief that
private claimants complied with the requisite period of possession.

The tax declarations in the name of private claimants are insufficient to prove the first
element of possession. We note that the earliest of the tax declarations in the name of private
claimants were issued in 1993. Being of recent dates, the tax declarations are not sufficient to
convince this Court that the period of possession and occupation commenced on June 12, 1945.

Private claimants insist that they have a vested right in Boracay, having been in
possession of the island for a long time. They have invested millions of pesos in developing the
island into a tourist spot. They say their continued possession and investments give them a
vested right which cannot be unilaterally rescinded by Proclamation No. 1064.

The continued possession and considerable investment of private claimants do not


automatically give them a vested right in Boracay. Nor do these give them a right to apply for a
title to the land they are presently occupying. This Court is constitutionally bound to decide
cases based on the evidence presented and the laws applicable. As the law and jurisprudence
stand, private claimants are ineligible to apply for a judicial confirmation of title over their
occupied portions in Boracay even with their continued possession and considerable investment
in the island.

One Last Note

The Court is aware that millions of pesos have been invested for the development
of Boracay Island, making it a by-word in the local and international tourism industry. The Court
also notes that for a number of years, thousands of people have called the island their
home. While the Court commiserates with private claimants’ plight, We are bound to apply the
law strictly and judiciously. This is the law and it should prevail. Ito ang batas at ito ang dapat
umiral.

420
All is not lost, however, for private claimants. While they may not be eligible to apply for
judicial confirmation of imperfect title under Section 48(b) of CA No. 141, as amended, this does
not denote their automatic ouster from the residential, commercial, and other areas they
possess now classified as agricultural. Neither will this mean the loss of their substantial
investments on their occupied alienable lands. Lack of title does not necessarily mean lack of
right to possess.

For one thing, those with lawful possession may claim good faith as builders of
improvements. They can take steps to preserve or protect their possession. For another, they
may look into other modes of applying for original registration of title, such as by
homestead[131] or sales patent,[132] subject to the conditions imposed by law.

More realistically, Congress may enact a law to entitle private claimants to acquire title to
their occupied lots or to exempt them from certain requirements under the present land
laws. There is one such bill[133] now pending in the House of Representatives. Whether that bill
or a similar bill will become a law is for Congress to decide.

In issuing Proclamation No. 1064, the government has taken the step necessary to open
up the island to private ownership. This gesture may not be sufficient to appease some sectors
which view the classification of the island partially into a forest reserve as absurd. That the
island is no longer overrun by trees, however, does not becloud the vision to protect its
remaining forest cover and to strike a healthy balance between progress and ecology. Ecological
conservation is as important as economic progress.

To be sure, forest lands are fundamental to our nation’s survival. Their promotion and
protection are not just fancy rhetoric for politicians and activists. These are needs that become
more urgent as destruction of our environment gets prevalent and difficult to control. As aptly
observed by Justice Conrado Sanchez in 1968 in Director of Forestry v. Munoz:[134]

The view this Court takes of the cases at bar is but in adherence to public
policy that should be followed with respect to forest lands. Many have written
much, and many more have spoken, and quite often, about the pressing need for
forest preservation, conservation, protection, development and reforestation. Not
without justification. For, forests constitute a vital segment of any country's natural
resources. It is of common knowledge by now that absence of the necessary green
cover on our lands produces a number of adverse or ill effects of serious
proportions. Without the trees, watersheds dry up; rivers and lakes which they
supply are emptied of their contents. The fish disappear. Denuded areas become
dust bowls. As waterfalls cease to function, so will hydroelectric plants. With the
421
rains, the fertile topsoil is washed away; geological erosion results. With erosion
come the dreaded floods that wreak havoc and destruction to property – crops,
livestock, houses, and highways – not to mention precious human lives. Indeed,
the foregoing observations should be written down in a lumberman’s decalogue.[135]

WHEREFORE, judgment is rendered as follows:

1. The petition for certiorari in G.R. No. 167707 is GRANTED and the Court of Appeals
Decision in CA-G.R. CV No. 71118 REVERSED AND SET ASIDE.

2. The petition for certiorari in G.R. No. 173775 is DISMISSED for lack of merit.

SO ORDERED.

RUBEN T. REYES
Associate Justice

WE CONCUR:

REYNATO S. PUNO
Chief Justice

LEONARDO A. QUISUMBING CONSUELO YNARES-SANTIAGO


Associate Justice Associate Justice

ANTONIO T. CARPIO MA. ALICIA AUSTRIA-MARTINEZ


Associate Justice Associate Justice

422
(On official leave)
RENATO C. CORONA CONCHITA CARPIO MORALES
Associate Justice Associate Justice

ADOLFO S. AZCUNA DANTE O. TINGA


Associate Justice Associate Justice

MINITA V. CHICO-NAZARIO PRESBITERO J. VELASCO, JR.


Associate Justice Associate Justice

(No part)
ANTONIO EDUARDO B. NACHURA TERESITA J. LEONARDO-DE CASTRO
Associate Justice Associate Justice

ARTURO D. BRION
Associate Justice

CERTIFICATION

Pursuant to Section 13, Article VIII of the Constitution, I certify that the conclusions in the
above Decision had been reached in consultation before the case was assigned to the writer of
the opinion of the Court.

REYNATO S. PUNO
Chief Justice

*
On official leave per Special Order No. 520 dated September 19, 2008.

423
**
No part. Justice Nachura participated in the present case as Solicitor General.
[1]
Rollo (G.R. No. 167707), pp. 37-43. CA-G.R. CV No. 71118, promulgated on December 9,
2004. Penned by Associate Justice Isaias P. Dicdican, with Associate Justices Sesinando E.
Villon and Ramon M. Bato, Jr., concurring.
[2]
Id. at 47-54; Annex “C.” Spl. Civil Case No. 5403. Penned by Judge Niovady M. Marin, RTC,
Kalibo, Branch 5.
[3]
Rollo (G.R. No. 173775), pp. 101-114. Annex “F.” Classifying Boracay Island Situated in the
Municipality of Malay, Province of Aklan Into Forestland (Protection Purposes) and Into
Agricultural Land (Alienable and Disposable) Pursuant to Presidential Decreee No. 705 (Revised
Forestry Reform Code of the Philippines). Issued on May 22, 2006.
[4]
As of the year 2000. http://www.nscb.gov.ph/ru6/boracay.htm.
[5]
Manoc-Manoc, Balabag, and Yapak. http://www.nscb.gov.ph/ru6/boracay.htm.
[6]
Under Survey Plan No. NR-06-000001.
[7]
Rollo (G.R. No. 167707), p. 49.
[8]
Id. at 21-23; Annex “B.” Declaring Certain Islands, Coves, and Peninsulas in
the Philippines as Tourist Zones and Marine Reserves Under the Administration and Control of
the Philippine Tourism Authority.
[9]
Id. at 24-27. Rules and Regulations Governing Activities at Boracay Island Tourist Zone.
[10]
Records, pp. 13-32; Annexes “A” to “A-18.”
[11]
Issued on May 19, 1975.
[12]
Records, p. 148.
[13]
Id.
[14]
RULES OF COURT, Rule 129, Sec. 2.
[15]
Records, p. 148.
[16]
Id. at 177, 178.
[17]
Rollo (G.R. No. 167707), p. 54.
[18]
Id. at 51.
[19]
Id.; PTA Circular No. 3-82, Rule VIII, Sec. 1(3) states:
No trees in forested private lands may be cut without prior authority from
the PTA. All forested areas in public lands are declared forest reserves.
[20]
Sec. 87. If all the lands included in the proclamation of the President are not registered
under the Land Registration Act, the Solicitor-General, if requested to do so by the Secretary of
Agriculture and Natural Resources, shall proceed in accordance with the provisions of section
fifty-three of this Act.
[21]
Sec. 53. It shall be lawful for the Director of Lands, whenever in the opinion of the President
the public interests shall require it, to cause to be filed in the proper Court of First Instance,
through the Solicitor General or the officer acting in his stead, a petition against the holder,
claimant, possessor, or occupant of any land who shall not have voluntarily come in under the
provisions of this chapter or of the Land Registration Act, stating in substance that the title of
such holder, claimant, possessor, or occupant is open to discussion; or that the boundaries of
any such land which has not been brought into court as aforesaid are open to question; or that it
is advisable that the title to such land be settled and adjudicated, and praying that the title to
any such land or the boundaries thereof or the right to occupancy thereof be settled and
adjudicated. The judicial proceedings under this section shall be in accordance with the laws on
adjudication of title in cadastral proceedings.
[22]
Rollo (G.R. No. 167707), p. 51.
[23]
Id. at 211-121.
[24]
Id. at 42.
[25]
Id. at 45-46.

424
[26]
Supra note 3.
[27]
Owner of Waling-Waling Beach Resort and Chairman of the Board of Boracay Foundation,
Inc.
[28]
Owner of Willy’s Beach Resort.
[29]
Rollo (G.R. No. 173775), p. 20; Annex “A.”
[30]
Petitioners in G.R. No. 173775 claim that they are also petitioners in the declaratory case
filed in November 1997 before the RTC in Kalibo, Aklan, docketed as Sp. Civil Case No. 5403 and
now before this Court as G.R. No. 167707.
[31]
Rollo (G.R No. 173775), pp. 4-5.
[32]
Id. at 4.
[33]
Id. at 143.
[34]
Rollo (G.R. No. 167707), p. 26.
[35]
Rollo (G.R. No. 173775), pp. 280-281.
[36]
An Act Temporarily to Provide for the Administration of the Affairs of Civil Government in the
Philippine Islands, and for Other Purposes. Issued on July 1, 1902.
[37]
An Act to Amend and Compile the Laws Relative to Lands of the Public Domain. Approved
on December 1, 1936.
[38]
See note 8.
[39]
See note 3.
[40]
CONSTITUTION (1935), Art. XIII, Sec. 1.
[41]
CONSTITUTION (1973), Art. XIV, Sec. 10.
[42]
Bernas, S.J., The Intent of the 1986 Constitution Writers, 1995 ed., p. 830.
[43]
CONSTITUTION (1987), Art. XII, Sec. 3.
[44]
Id.
[45]
Zarate v. Director of Lands, G.R. No. 131501, July 14, 2004, 434 SCRA 322; Reyes v. Court
of Appeals, 356 Phil. 606, 624 (1998).
[46]
Chavez v. Public Estates Authority, G.R. No. 133250, July 9, 2002, 384 SCRA 152.
[47]
Zarate v. Director of Lands, supra; Collado v. Court of Appeals, G.R. No. 107764, October 4,
2002, 390 SCRA 343; Director of Lands v. Intermediate Appellate Court, G.R. No. 73246, March
2, 1993, 219 SCRA 339.
[48]
Republic v. Estonilo, G.R. No. 157306, November 25, 2005, 476 SCRA 265; Zarate v.
Director of Lands, supra.
[49]
De los Reyes v. Ramolete, G.R. No. L-47331, June 21, 1983, 122 SCRA 652, citing Gonzaga
v. Court of Appeals, G.R. No. L-27455, June 28, 1973, 51 SCRA 381.
[50]
Collado v. Court of Appeals, supra, citing Chavez v. Public Estates Authority, supra.
[51]
Id., citing separate opinion of then Justice Reynato S. Puno in Cruz v. Secretary of
Environment and Natural Resources, G.R. No. 135385, December 6, 2000, 347 SCRA 128,
andChavez v. Public Estates Authority, supra note 46.
[52]
Collado v. Court of Appeals, supra note 47.
[53]
Effective February 13, 1894.
[54]
De Aldecoa v. The Insular Government, 13 Phil. 159 (1909).
[55]
A valid title based upon adverse possession or a valid title based upon
prescription. Noblejas, A.H. and Noblejas, E.H., Registration of Land Titles and Deeds, 1986 ed.,
p. 39, citing Cruz v. De Leon, 21 Phil. 199 (1912).
[56]
Ten (10) years, according to Archbishop of Manila v. Arnedo, 30 Phil. 593 (1915).
[57]
Noblejas, A.H. and Noblejas, E.H., Registration of Land Titles and Deeds, supra at 8.
[58]
Id. at 9; Director of Forest Administration v. Fernandez, G.R. Nos. 36827, 56622 &
70076, December 10, 1990, 192 SCRA 121, 137.
[59]
Id. at 5-11.

425
[60]
See note 36.
[61]
Director of Forestry v. Villareal, G.R. No. L-32266, February 27, 1989, 170 SCRA 598, 601.
[62]
Noblejas, A.H. and Noblejas, E.H., Registration of Land Titles and Deeds, supra note 55, at
347.
[63]
The provisions relevant to the definition are:
Sec. 13. That the Government of the Philippine Islands, subject to the provisions
of this Act and except as herein provided, shall classify according to its agricultural
character and productiveness, and shall immediately make rules and regulations for the
lease, sale, or other disposition of the public lands other than timber or mineral lands,
but such rules and regulations shall not go into effect or have the force of law until they
have received the approval of the President, and when approved by the President they
shall be submitted by him to Congress at the beginning of the next ensuing session
thereof and unless disapproved or amended by Congress at said session they shall at the
close of such period have the force and effect of law in the Philippine Islands: Provided,
That a single homestead entry shall not exceed sixteen hectares in extent.
Sec. 14. That the Government of the Philippine Islands is hereby authorized and
empowered to enact rules and regulations and to prescribe terms and conditions to
enable persons to perfect their title to public lands in said Islands, who, prior to the
transfer of sovereignty from Spain to the United States, had fulfilled all or some of the
conditions required by the Spanish laws and royal decrees of the Kingdom of Spain for
the acquisition of legal title thereto, yet failed to secure conveyance of title; and the
Philippine Commission is authorized to issue patents, without compensation, to any
native of said Islands, conveying title to any tract of land not more than sixteen hectares
in extent, which were public lands and had been actually occupied by such native or his
ancestors prior to and on the thirteenth of August, eighteen hundred and ninety-eight.
Sec. 15. That the Government of the Philippine Islands is hereby authorized and
empowered, on such terms as it may prescribe, by general legislation, to provide for the
granting or sale and conveyance to actual occupants and settlers and other citizens of
said Islands such parts and portions of the public domain, other than timber and mineral
lands, of the United States in said Islands as it may deem wise, not exceeding sixteen
hectares to any one person and for the sale and conveyance of not more than one
thousand and twenty-four hectares to any corporation or association of persons:
Provided, That the grant or sale of such lands, whether the purchase price be paid at
once or in partial payments, shall be conditioned upon actual and continued occupancy,
improvement, and cultivation of the premises sold for a period of not less than five
years, during which time the purchaser or grantee can not alienate or encumber said
land or the title thereto; but such restriction shall not apply to transfers of rights and
title of inheritance under the laws for the distribution of the estates of decedents.
[64]
10 Phil. 175 (1908).
[65]
Id. at 182.
[66]
Collado v. Court of Appeals, supra note 47.
[67]
Noblejas, A.H. and Noblejas, E.H., Registration of Land Titles and Deeds, supra note 55.
[68]
Sec. 54, par. 6.
[69]
Sec. 45(b); Public Estates Authority v. Court of Appeals, G.R. No. 112172, November 20,
2000, 345 SCRA 96; Director of Lands v. Buyco, G.R. No. 91189, November 27, 1992, 216 SCRA
78.
[70]
Collado v. Court of Appeals, supra note 47, see separate opinion of Justice Puno in Cruz v.
Secretary of Environment and Natural Resources, supra note 51, and Chavez v. Public Estates
Authority, supra note 46.

426
[71]
Sec. 2.
[72]
An Act to Amend Subsection (b) of Section Forty-Eight of Commonwealth Act Numbered One
Hundred Forty-One, Otherwise Known as the Public Land Act. Approved onJune 22, 1957.
[73]
Extending the Period of Filing Applications for Administrative Legislation (Free Patent) and
Judicial Confirmation of Imperfect and Incomplete Titles to Alienable and Disposable Lands in the
Public Domain Under Chapter VII and Chapter VIII of Commonwealth Act No. 141, As Amended,
For Eleven (11) Years Commencing January 1, 1977. Approved onJanuary 25, 1977.
[74]
Republic v. Doldol, G.R. No. 132963, September 10, 1998, 295 SCRA 359.
[75]
Discontinuance of the Spanish Mortgage System of Registration and of the Use of Spanish
Titles as Evidence in Land Registration Proceedings (Issued – February 16, 1976).
[76]
Director of Forest Administration v. Fernandez, supra note 58, citing Director of Lands v.
Rivas, G.R. No. L-61539, February 14, 1986, 141 SCRA 329.
[77]
Lands which were not recorded under the Maura Law and were not yet covered
by Torrens titles.
[78]
Presidential Decree No. 1529, Preamble; Director of Lands v. Intermediate Appellate
Court, supra note 47.
[79]
Peña, N. and Peña, Jr., N., Registration of Land Titles and Deeds, 1988 ed., p. 9.
[80]
Republic v. Court of Appeals, G.R. No. 48227, August 21, 1991, 201 SCRA 1; Director of
Lands v. Court of Appeals, G.R. No. 83609, October 26, 1989, 178 SCRA 708.
[81]
Heirs of the Late Spouses Pedro S. Palanca and Soterranea Rafols Vda. De Palanca v.
Republic, G.R. No. 151312, August 30, 2006, 500 SCRA 209; Director of Lands v. Intermediate
Appellate Court, supra note 47, citing Director of Lands v. Aquino, G.R. No. 31688, December
17, 1990, 192 SCRA 296.
[82]
Chavez v. Public Estates Authority, supra note 46.
[83]
Republic v. Lao, G.R. No. 150413, July 1, 2003; 405 SCRA 291; Director of Lands v.
Intermediate Appellate Court, supra note 47, citing Director of Lands v. Aquino, supra.
[84]
Republic v. Lao, supra; Pagkatipunan v. Court of Appeals, 429 Phil. 377, 389-390 (2002).
[85]
Republic of the Philippines v. Muñoz, G.R. No. 151910, October 15, 2007.
[86]
Heirs of the Late Spouses Pedro S. Palanca and Soterranea Rafols Vda. De Palanca v.
Republic, supra; Gutierrez Hermanos v. Court of Appeals, G.R. Nos. 54472-77,September 28,
1989, 178 SCRA 37.
[87]
Republic v. Naguiat, G.R. No. 134209, January 24, 2006, 479 SCRA 585.
[88]
40 Phil. 10 (1919).
[89]
Supra note 54.
[90]
Ankron v. Government of the Philippine Islands, supra at 16.
[91]
Heirs of the Late Spouses Pedro S. Palanca and Soterranea Rafols Vda. De Palanca v.
Republic, supra note 81.
[92]
Id. at 76.
[93]
Id. at 219-223.
[94]
Ankron v. Government of the Philippine Islands, supra note 88, at 16.
[95]
Id. at 15-16.
[96]
Act No. 2874, Sec. 8; Republic v. Court of Appeals, G.R. No. 155450, August 6,
2008; Republic v. Court of Appeals, G.R. No. 127245, January 30, 2001.
96-a
Bureau of Forestry v. Court of Appeals, G.R. No. L-37995, August 31, 1987, 153 SCRA 351,
357.
[97]
Heirs of the Late Spouses Pedro S. Palanca and Soterranea Rafols Vda. De Palanca v.
Republic, supra note 81.
[98]
The records do not show the manner in which title was issued to the Heirs of Ciriaco Tirol.
[99]
Records, p. 179.

427
[100]
79 Phil. 461 (1947).
[101]
Supra note 64.
[102]
Supra note 54.
[103]
Supra note 88.
[104]
Art. XIII, Sec. 1.
[105]
Krivenko v. Register of Deeds of Manila, supra note 100, at 468-469.
[106]
Act No. 926, Sec. 54, par. 6 states:
SEC. 54. The following described persons or their legal successors in right,
occupying lands in the Philippines, or claiming to own any such land or interest therein
but whose titles to such land have not been perfected may apply to the Court of Land
Registration of the Philippine Islands for confirmation of their claims and the issuance of
a certificate of title therefor to wit –
xxxx
(6) All persons who by themselves or their predecessors in interest have been in
the open, continuous exclusive, and notorious possession and occupation of agricultural
public lands, as defined by said Act of Congress of July first, nineteen hundred and two,
under a bona fide claim of ownership except as against the Government, for a period of
ten years next preceding the taking effect of this act, except when prevented by war, or
force majeure, shall be conclusively presumed to have performed all the conditions
essential to a Government grant and to have received the same, and shall be entitled to
a certificate of title to such land under the provisions of this chapter.
[107]
Supra note 47.
107-a
G.R. No. 135385, December 6, 2000, 347 SCRA 128.
[108]
Collado v. Court of Appeals, id. at 356.
[109]
Records, p. 101; Annex “A.”
[110]
Id. at 106; Exhibit “1-a.”
[111]
Rollo (G.R. No. 173775), p. 5.
[112]
CONSTITUTION (1987), Art. XII, Sec. 3; CONSTITUTION (1973), Art. XIV, Sec. 10, as
amended; and CONSTITUTION (1935), Art. XIII, Sec. 1.
[113]
Republic v. Naguiat, supra note 87.
[114]
G.R. No. L-27873, November 29, 1983, 126 SCRA 69.
[115]
Heirs of Amunategui v. Director of Forestry, id. at 75.
[116]
Republic v. Court of Appeals, G.R. No. L-56948, September 30, 1987, 154 SCRA 476, 482-
483.
[117]
Sec. 3 provides:
Establishment of or low-density human settlements in private lands, or
subdivisions, if any, subject to prior approval by the Ministry of Human
Settlements, PTA and local building officials; Provided, that no structures shall be
constructed within 30 meters from the shorelines.
[118]
Sec. 5 states:
Subsistence farming, in areas declared as alienable and disposable by the Bureau
of Forest Development.
[119]
Pars. 3-4.
[120]
SEC. 6. The President, upon recommendation of the Secretary of Agriculture and Commerce
(now the Secretary of the Department of Environment and Natural Resources), shall from time
to time classify lands of the public domain into –
(a) Alienable or disposable,
(b) Timber, and
(c) Mineral lands,

428
And may at any time and in a like manner transfer such lands from one class to another, for the
purposes of their administration and disposition.
SEC. 7. For the purposes of administration and disposition of alienable or disposable public
lands, the President, upon recommendation by the Secretary of Agriculture and Commerce (now
the Secretary of the Department of Environment and Natural Resources), shall from time to time
declare what lands are open to disposition or concession under this Act.
[121]
Director of Lands v. Intermediate Appellate Court, supra note 47; Manalo v. Intermediate
Appellate Court, G.R. No. 64753, April 26, 1989, 172 SCRA 795.
[122]
Republic v. Register of Deeds of Quezon, G.R. No. 73974, May 31, 1995, 244 SCRA
537; Director of Lands v. Intermediate Appellate Court, supra note 47.
[123]
Director of Lands v. Intermediate Appellate Court, supra note 47, citing Yngson v. Secretary
of Agriculture and Natural Resources, G.R. No. L-36847, July 20, 1983, 123 SCRA 441; Republic
v. Court of Appeals, G.R. No. L-45202, September 11, 1980, 99 SCRA 742.
[124]
Supra note 81.
[125]
Heirs of the Late Spouses Pedro S. Palanca and Soterranea Rafols Vda. De Palanca v.
Republic, id. at 222-223.
[126]
Reconsideration of DOJ Opinion No. 169, s. 1993, on the DOJ affirmative stand on whether
the prohibition against the reclassification of forest lands applies to “unclassified public forest.”
[127]
Rollo (G.R. No. 173775), p. 139.
[128]
Del Rosario-Igtiben v. Republic, G.R. No. 158449, October 22, 2004, 441 SCRA
188; Republic v. Lao, supra note 83.
[129]
Public Land Act, Sec. 48(b).
[130]
Public Estates Authority v. Court of Appeals, supra note 69.
[131]
Commonwealth Act No. 141, Chapter IV.
[132]
Id., Chapter V.
[133]
House Bill No. 1109. Declaring Certain Parcels of the Public Domain Within Boracay Island,
Malay, Aklan as Agricultural Land Open to Disposition.
[134]
G.R. No. L-24796, June 28, 1968, 23 SCRA 1183, cited in Lepanto Consolidated Mining
Company v. Dumyung, G.R. Nos. L-31666-68, April 30, 1979, 89 SCRA 532.
[135]
Director of Forestry v. Muñoz, id. at 1214.

429
Republic of the Philippines
SUPREME COURT
Manila

EN BANC

METROPOLITAN MANILA G.R. Nos. 171947-48


DEVELOPMENT AUTHORITY,
DEPARTMENT OF ENVIRONMENT
AND NATURAL RESOURCES, Present:
DEPARTMENT OF EDUCATION,
CULTURE AND SPORTS,[1] PUNO, C.J.,
DEPARTMENT OF HEALTH, QUISUMBING,
DEPARTMENT OF AGRICULTURE, YNARES-SANTIAGO,
DEPARTMENT OF PUBLIC CARPIO,
WORKS AND HIGHWAYS, AUSTRIA-MARTINEZ,
DEPARTMENT OF BUDGET AND CORONA,
MANAGEMENT, PHILIPPINE CARPIO MORALES,
COAST GUARD, PHILIPPINE AZCUNA,
NATIONAL POLICE MARITIME TINGA,
GROUP, and DEPARTMENT OF CHICO-NAZARIO,
THE INTERIOR AND LOCAL VELASCO, JR.,
GOVERNMENT, NACHURA,
Petitioners, REYES,
LEONARDO-DE CASTRO, and
- versus - BRION, JJ.

CONCERNED RESIDENTS OF
MANILA BAY, represented and
joined by DIVINA V. ILAS,
SABINIANO ALBARRACIN,
MANUEL SANTOS, JR., DINAH
DELA PEÑA, PAUL DENNIS
QUINTERO, MA. VICTORIA
LLENOS, DONNA CALOZA,
FATIMA QUITAIN, VENICE
SEGARRA, FRITZIE TANGKIA,
SARAH JOELLE LINTAG,
HANNIBAL AUGUSTUS BOBIS,
FELIMON SANTIAGUEL, and Promulgated:
JAIME AGUSTIN R. OPOSA,
Respondents. December 18, 2008
x-----------------------------------------------------------------------------------------x

430
DECISION

VELASCO, JR., J.:

The need to address environmental pollution, as a cause of climate change, has of late
gained the attention of the international community. Media have finally trained their sights on
the ill effects of pollution, the destruction of forests and other critical habitats, oil spills, and the
unabated improper disposal of garbage. And rightly so, for the magnitude of environmental
destruction is now on a scale few ever foresaw and the wound no longer simply heals by
itself.[2] But amidst hard evidence and clear signs of a climate crisis that need bold action, the
voice of cynicism, naysayers, and procrastinators can still be heard.

This case turns on government agencies and their officers who, by the nature of their
respective offices or by direct statutory command, are tasked to protect and preserve, at the
first instance, our internal waters, rivers, shores, and seas polluted by human activities. To most
of these agencies and their official complement, the pollution menace does not seem to carry the
high national priority it deserves, if their track records are to be the norm. Their cavalier attitude
towards solving, if not mitigating, the environmental pollution problem, is a sad commentary on
bureaucratic efficiency and commitment.

At the core of the case is the Manila Bay, a place with a proud historic past, once
brimming with marine life and, for so many decades in the past, a spot for different contact
recreation activities, but now a dirty and slowly dying expanse mainly because of the abject
official indifference of people and institutions that could have otherwise made a difference.

This case started when, on January 29, 1999, respondents Concerned Residents of Manila
Bay filed a complaint before the Regional Trial Court (RTC) in Imus, Cavite against several
government agencies, among them the petitioners, for the cleanup, rehabilitation, and
protection of the Manila Bay. Raffled to Branch 20 and docketed as Civil Case No. 1851-99 of the
RTC, the complaint alleged that the water quality of the Manila Bay had fallen way below the
allowable standards set by law, specifically Presidential Decree No. (PD) 1152 or the Philippine
Environment Code. This environmental aberration, the complaint stated, stemmed from:

x x x [The] reckless, wholesale, accumulated and ongoing acts of omission


or commission [of the defendants] resulting in the clear and present danger to
public health and in the depletion and contamination of the marine life of Manila
Bay, [for which reason] ALL defendants must be held jointly and/or solidarily liable
and be collectively ordered to clean up Manila Bay and to restore its water quality
431
to class B waters fit for swimming, skin-diving, and other forms of contact
recreation.[3]

In their individual causes of action, respondents alleged that the continued neglect of
petitioners in abating the pollution of the Manila Bay constitutes a violation of, among others:

(1) Respondents’ constitutional right to life, health, and a balanced ecology;


(2) The Environment Code (PD 1152);
(3) The Pollution Control Law (PD 984);
(4) The Water Code (PD 1067);
(5) The Sanitation Code (PD 856);
(6) The Illegal Disposal of Wastes Decree (PD 825);
(7) The Marine Pollution Law (PD 979);
(8) Executive Order No. 192;
(9) The Toxic and Hazardous Wastes Law (Republic Act No. 6969);
(10) Civil Code provisions on nuisance and human relations;
(11) The Trust Doctrine and the Principle of Guardianship; and
(12) International Law

Inter alia, respondents, as plaintiffs a quo, prayed that petitioners be ordered to clean
the Manila Bay and submit to the RTC a concerted concrete plan of action for the purpose.

The trial of the case started off with a hearing at the Manila Yacht Club followed by an
ocular inspection of the Manila Bay. Renato T. Cruz, the Chief of the Water Quality Management
Section, Environmental Management Bureau, Department of Environment and Natural Resources
(DENR), testifying for petitioners, stated that water samples collected from different beaches
around the Manila Bay showed that the amount of fecal coliform content ranged from 50,000 to
80,000 most probable number (MPN)/ml when what DENR Administrative Order No. 34-90
prescribed as a safe level for bathing and other forms of contact recreational activities, or the
“SB” level, is one not exceeding 200 MPN/100 ml.[4]

Rebecca de Vera, for Metropolitan Waterworks and Sewerage System (MWSS) and in
behalf of other petitioners, testified about the MWSS’ efforts to reduce pollution along
the Manila Bay through the Manila Second Sewerage Project. For its part, the Philippine Ports
Authority (PPA) presented, as part of its evidence, its memorandum circulars on the study being
conducted on ship-generated waste treatment and disposal, and its Linis Dagat (Clean the
Ocean) project for the cleaning of wastes accumulated or washed to shore.

The RTC Ordered Petitioners to Clean Up and Rehabilitate Manila Bay

432
On September 13, 2002, the RTC rendered a Decision[5] in favor of respondents. The
dispositive portion reads:

WHEREFORE, finding merit in the complaint, judgment is hereby rendered


ordering the abovenamed defendant-government agencies, jointly and solidarily, to
clean up and rehabilitate Manila Bay and restore its waters to SB classification to
make it fit for swimming, skin-diving and other forms of contact recreation. To
attain this, defendant-agencies, with defendant DENR as the lead agency, are
directed, within six (6) months from receipt hereof, to act and perform their
respective duties by devising a consolidated, coordinated and concerted scheme of
action for the rehabilitation and restoration of the bay.
In particular:

Defendant MWSS is directed to install, operate and maintain adequate


[sewerage] treatment facilities in strategic places under its jurisdiction and increase
their capacities.

Defendant LWUA, to see to it that the water districts under its wings,
provide, construct and operate sewage facilities for the proper disposal of waste.

Defendant DENR, which is the lead agency in cleaning up Manila Bay, to


install, operate and maintain waste facilities to rid the bay of toxic and hazardous
substances.

Defendant PPA, to prevent and also to treat the discharge not only of ship-
generated wastes but also of other solid and liquid wastes from docking vessels
that contribute to the pollution of the bay.

Defendant MMDA, to establish, operate and maintain an adequate and


appropriate sanitary landfill and/or adequate solid waste and liquid disposal as well
as other alternative garbage disposal system such as re-use or recycling of wastes.

Defendant DA, through the Bureau of Fisheries and Aquatic Resources, to


revitalize the marine life in Manila Bay and restock its waters with indigenous fish
and other aquatic animals.

Defendant DBM, to provide and set aside an adequate budget solely for the
purpose of cleaning up and rehabilitation of Manila Bay.

Defendant DPWH, to remove and demolish structures and other nuisances


that obstruct the free flow of waters to the bay. These nuisances discharge solid
and liquid wastes which eventually end up in Manila Bay. As the construction and
engineering arm of the government, DPWH is ordered to actively participate in
removing debris, such as carcass of sunken vessels, and other non-biodegradable
garbage in the bay.

Defendant DOH, to closely supervise and monitor the operations of septic


and sludge companies and require them to have proper facilities for the treatment
and disposal of fecal sludge and sewage coming from septic tanks.
433
Defendant DECS, to inculcate in the minds and hearts of the people
through education the importance of preserving and protecting the environment.

Defendant Philippine Coast Guard and the PNP Maritime Group, to protect
at all costs the Manila Bay from all forms of illegal fishing.

No pronouncement as to damages and costs.

SO ORDERED.
The MWSS, Local Water Utilities Administration (LWUA), and PPA filed before the Court of
Appeals (CA) individual Notices of Appeal which were eventually consolidated and docketed as
CA-G.R. CV No. 76528.

On the other hand, the DENR, Department of Public Works and Highways (DPWH),
Metropolitan Manila Development Authority (MMDA), Philippine Coast Guard (PCG), Philippine
National Police (PNP) Maritime Group, and five other executive departments and agencies filed
directly with this Court a petition for review under Rule 45. The Court, in a Resolution
ofDecember 9, 2002, sent the said petition to the CA for consolidation with the consolidated
appeals of MWSS, LWUA, and PPA, docketed as CA-G.R. SP No. 74944.

Petitioners, before the CA, were one in arguing in the main that the pertinent provisions of
the Environment Code (PD 1152) relate only to the cleaning of specific pollution incidents and do
not cover cleaning in general. And apart from raising concerns about the lack of funds
appropriated for cleaning purposes, petitioners also asserted that the cleaning of
the Manila Bayis not a ministerial act which can be compelled by mandamus.

The CA Sustained the RTC

By a Decision[6] of September 28, 2005, the CA denied petitioners’ appeal and affirmed
the Decision of the RTC in toto,stressing that the trial court’s decision did not require petitioners
to do tasks outside of their usual basic functions under existing laws.[7]

Petitioners are now before this Court praying for the allowance of their Rule 45 petition on
the following ground and supporting arguments:
THE [CA] DECIDED A QUESTION OF SUBSTANCE NOT HERETOFORE PASSED
UPON BY THE HONORABLE COURT, I.E., IT AFFIRMED THE TRIAL COURT’S
DECISION DECLARING THAT SECTION 20 OF [PD] 1152 REQUIRES CONCERNED
GOVERNMENT AGENCIES TO REMOVE ALL POLLUTANTS SPILLED AND
DISCHARGED IN THE WATER SUCH AS FECAL COLIFORMS.
434
ARGUMENTS

I
[SECTIONS] 17 AND 20 OF [PD] 1152 RELATE ONLY TO THE CLEANING OF
SPECIFIC POLLUTION INCIDENTS AND [DO] NOT COVER CLEANING IN GENERAL

II
THE CLEANING OR REHABILITATION OF THE MANILA BAY IS NOT A
MINISTERIAL ACT OF PETITIONERS THAT CAN BE COMPELLED BY MANDAMUS.

The issues before us are two-fold. First, do Sections 17 and 20 of PD 1152 under the
headings, Upgrading of Water Quality and Clean-up Operations, envisage a cleanup in general or
are they limited only to the cleanup of specific pollution incidents? And second, can petitioners
be compelled by mandamus to clean up and rehabilitate the Manila Bay?

On August 12, 2008, the Court conducted and heard the parties on oral arguments.

Our Ruling

We shall first dwell on the propriety of the issuance of mandamus under the premises.

The Cleaning or Rehabilitation of Manila Bay


Can be Compelled by Mandamus

Generally, the writ of mandamus lies to require the execution of a ministerial duty.[8] A
ministerial duty is one that “requires neither the exercise of official discretion nor
[9]
judgment.” It connotes an act in which nothing is left to the discretion of the person executing
it. It is a “simple, definite duty arising under conditions admitted or proved to exist and imposed
by law.”[10] Mandamus is available to compel action, when refused, on matters involving
discretion, but not to direct the exercise of judgment or discretion one way or the other.

Petitioners maintain that the MMDA’s duty to take measures and maintain adequate solid
waste and liquid disposal systems necessarily involves policy evaluation and the exercise of
judgment on the part of the agency concerned. They argue that the MMDA, in carrying out its
mandate, has to make decisions, including choosing where a landfill should be located by
undertaking feasibility studies and cost estimates, all of which entail the exercise of discretion.

435
Respondents, on the other hand, counter that the statutory command is clear and that
petitioners’ duty to comply with and act according to the clear mandate of the law does not
require the exercise of discretion. According to respondents, petitioners, the MMDA in particular,
are without discretion, for example, to choose which bodies of water they are to clean up, or
which discharge or spill they are to contain. By the same token, respondents maintain that
petitioners are bereft of discretion on whether or not to alleviate the problem of solid and liquid
waste disposal; in other words, it is the MMDA’s ministerial duty to attend to such services.

We agree with respondents.

First off, we wish to state that petitioners’ obligation to perform their duties as defined by
law, on one hand, and how they are to carry out such duties, on the other, are two different
concepts. While the implementation of the MMDA’s mandated tasks may entail a decision-
making process, the enforcement of the law or the very act of doing what the law exacts to be
done is ministerial in nature and may be compelled by mandamus. We said so in Social Justice
Society v. Atienza[11] in which the Court directed the City of Manila to enforce, as a matter of
ministerial duty, its Ordinance No. 8027 directing the three big local oil players to cease and
desist from operating their business in the so-called “Pandacan Terminals” within six months
from the effectivity of the ordinance. But to illustrate with respect to the instant case, the
MMDA’s duty to put up an adequate and appropriate sanitary landfill and solid waste and liquid
disposal as well as other alternative garbage disposal systems is ministerial, its duty being a
statutory imposition. The MMDA’s duty in this regard is spelled out in Sec. 3(c) of Republic Act
No. (RA) 7924 creating the MMDA. This section defines and delineates the scope of the MMDA’s
waste disposal services to include:

Solid waste disposal and management which include formulation and


implementation of policies, standards, programs and projects for proper and
sanitary waste disposal. It shall likewise include the establishment and operation of
sanitary land fill and related facilities and the implementation of other alternative
programs intended to reduce, reuse and recycle solid waste. (Emphasis added.)

The MMDA is duty-bound to comply with Sec. 41 of the Ecological Solid Waste
Management Act (RA 9003) which prescribes the minimum criteria for the establishment of
sanitary landfills and Sec. 42 which provides the minimum operating requirements that each site
operator shall maintain in the operation of a sanitary landfill. Complementing Sec. 41 are Secs.
36 and 37 of RA 9003,[12] enjoining the MMDA and local government units, among others, after

436
the effectivity of the law on February 15, 2001, from using and operating open dumps for solid
waste and disallowing, five years after such effectivity, the use of controlled dumps.

The MMDA’s duty in the area of solid waste disposal, as may be noted, is set forth not
only in the Environment Code (PD 1152) and RA 9003, but in its charter as well. This duty of
putting up a proper waste disposal system cannot be characterized as discretionary, for, as
earlier stated, discretion presupposes the power or right given by law to public functionaries to
act officially according to their judgment or conscience.[13] A discretionary duty is one that
“allows a person to exercise judgment and choose to perform or not to perform.”[14] Any
suggestion that the MMDA has the option whether or not to perform its solid waste disposal-
related duties ought to be dismissed for want of legal basis.

A perusal of other petitioners’ respective charters or like enabling statutes and pertinent
laws would yield this conclusion: these government agencies are enjoined, as a matter of
statutory obligation, to perform certain functions relating directly or indirectly to the cleanup,
rehabilitation, protection, and preservation of the Manila Bay. They are precluded from choosing
not to perform these duties. Consider:

(1) The DENR, under Executive Order No. (EO) 192,[15] is the primary agency responsible
for the conservation, management, development, and proper use of the country’s environment
and natural resources. Sec. 19 of the Philippine Clean Water Act of 2004 (RA 9275), on the other
hand, designates the DENR as the primary government agency responsible for its enforcement
and implementation, more particularly over all aspects of water quality management. On water
pollution, the DENR, under the Act’s Sec. 19(k), exercises jurisdiction “over all aspects of water
pollution, determine[s] its location, magnitude, extent, severity, causes and effects and other
pertinent information on pollution, and [takes] measures, using available methods and
technologies, to prevent and abate such pollution.”
The DENR, under RA 9275, is also tasked to prepare a National Water Quality Status
Report, an Integrated Water Quality Management Framework, and a 10-year Water Quality
Management Area Action Plan which is nationwide in scope covering theManila Bay and adjoining
areas. Sec. 19 of RA 9275 provides:

Sec. 19 Lead Agency.––The [DENR] shall be the primary government


agency responsible for the implementation and enforcement of this Act x x x unless
otherwise provided herein. As such, it shall have the following functions, powers
and responsibilities:

437
a) Prepare a National Water Quality Status report within twenty-four (24)
months from the effectivity of this Act: Provided, That the Department shall
thereafter review or revise and publish annually, or as the need arises, said
report;

b) Prepare an Integrated Water Quality Management Framework within twelve


(12) months following the completion of the status report;

c) Prepare a ten (10) year Water Quality Management Area Action Plan within
12 months following the completion of the framework for each designated water
management area. Such action plan shall be reviewed by the water quality
management area governing board every five (5) years or as need arises.

The DENR has prepared the status report for the period 2001 to 2005 and is in the
process of completing the preparation of the Integrated Water Quality Management
[16]
Framework. Within twelve (12) months thereafter, it has to submit a final Water Quality
Management Area Action Plan.[17] Again, like the MMDA, the DENR should be made to accomplish
the tasks assigned to it under RA 9275.

Parenthetically, during the oral arguments, the DENR Secretary manifested that the
DENR, with the assistance of and in partnership with various government agencies and non-
government organizations, has completed, as of December 2005, the final draft of a
comprehensive action plan with estimated budget and time frame, denominated as Operation
Plan for the Manila Bay Coastal Strategy, for the rehabilitation, restoration, and rehabilitation of
the Manila Bay.

The completion of the said action plan and even the implementation of some of its
phases should more than ever prod the concerned agencies to fast track what are assigned them
under existing laws.

(2) The MWSS, under Sec. 3 of RA 6234,[18] is vested with jurisdiction, supervision, and
control over all waterworks and sewerage systems in the territory comprising what is now the
cities of Metro Manila and several towns of the provinces of Rizal and Cavite, and charged with
the duty:
(g) To construct, maintain, and operate such sanitary sewerages as may be
necessary for the proper sanitation and other uses of the cities and towns
comprising the System; x x x

438
(3) The LWUA under PD 198 has the power of supervision and control over local water
districts. It can prescribe the minimum standards and regulations for the operations of these
districts and shall monitor and evaluate local water standards. The LWUA can direct these
districts to construct, operate, and furnish facilities and services for the collection, treatment,
and disposal of sewerage, waste, and storm water. Additionally, under RA 9275, the LWUA, as
attached agency of the DPWH, is tasked with providing sewerage and sanitation facilities,
inclusive of the setting up of efficient and safe collection, treatment, and sewage disposal system
in the different parts of the country.[19] In relation to the instant petition, the LWUA is mandated
to provide sewerage and sanitation facilities in Laguna, Cavite, Bulacan, Pampanga,
and Bataan to prevent pollution in the ManilaBay.
(4) The Department of Agriculture (DA), pursuant to the Administrative Code of 1987 (EO
[20]
292), is designated as the agency tasked to promulgate and enforce all laws and issuances
respecting the conservation and proper utilization of agricultural and fishery resources.
Furthermore, the DA, under the Philippine Fisheries Code of 1998 (RA 8550), is, in coordination
with local government units (LGUs) and other concerned sectors, in charge of establishing a
monitoring, control, and surveillance system to ensure that fisheries and aquatic resources in
Philippine waters are judiciously utilized and managed on a sustainable basis.[21] Likewise under
RA 9275, the DA is charged with coordinating with the PCG and DENR for the enforcement of
water quality standards in marine waters.[22] More specifically, its Bureau of Fisheries and
Aquatic Resources (BFAR) under Sec. 22(c) of RA 9275 shall primarily be responsible for the
prevention and control of water pollution for the development, management, and conservation
of the fisheries and aquatic resources.

(5) The DPWH, as the engineering and construction arm of the national government, is
tasked under EO 292[23] to provide integrated planning, design, and construction services for,
among others, flood control and water resource development systems in accordance with
national development objectives and approved government plans and specifications.

In Metro Manila, however, the MMDA is authorized by Sec. 3(d), RA 7924 to perform
metro-wide services relating to “flood control and sewerage management which include the
formulation and implementation of policies, standards, programs and projects for an integrated
flood control, drainage and sewerage system.”

On July 9, 2002, a Memorandum of Agreement was entered into between the DPWH and
MMDA, whereby MMDA was made the agency primarily responsible for flood control in Metro
Manila. For the rest of the country, DPWH shall remain as the implementing agency for flood

439
control services. The mandate of the MMDA and DPWH on flood control and drainage services
shall include the removal of structures, constructions, and encroachments built along rivers,
waterways, and esteros (drainages) in violation of RA 7279, PD 1067, and other pertinent laws.

(6) The PCG, in accordance with Sec. 5(p) of PD 601, or the Revised Coast Guard Law of
1974, and Sec. 6 of PD 979,[24] or the Marine Pollution Decree of 1976, shall have the primary
responsibility of enforcing laws, rules, and regulations governing marine pollution within the
territorial waters of the Philippines. It shall promulgate its own rules and regulations in
accordance with the national rules and policies set by the National Pollution Control Commission
upon consultation with the latter for the effective implementation and enforcement of PD 979. It
shall, under Sec. 4 of the law, apprehend violators who:

a. discharge, dump x x x harmful substances from or out of any ship, vessel, barge,
or any other floating craft, or other man-made structures at sea, by any method,
means or manner, into or upon the territorial and inland navigable waters of the
Philippines;

b. throw, discharge or deposit, dump, or cause, suffer or procure to be thrown,


discharged, or deposited either from or out of any ship, barge, or other floating
craft or vessel of any kind, or from the shore, wharf, manufacturing establishment,
or mill of any kind, any refuse matter of any kind or description whatever other
than that flowing from streets and sewers and passing therefrom in a liquid state
into tributary of any navigable water from which the same shall float or be washed
into such navigable water; and

c. deposit x x x material of any kind in any place on the bank of any navigable
water or on the bank of any tributary of any navigable water, where the same shall
be liable to be washed into such navigable water, either by ordinary or high tides,
or by storms or floods, or otherwise, whereby navigation shall or may be impeded
or obstructed or increase the level of pollution of such water.

(7) When RA 6975 or the Department of the Interior and Local Government (DILG) Act of
1990 was signed into law onDecember 13, 1990, the PNP Maritime Group was tasked to
“perform all police functions over the Philippine territorial waters and rivers.” Under Sec. 86, RA
6975, the police functions of the PCG shall be taken over by the PNP when the latter acquires
the capability to perform such functions. Since the PNP Maritime Group has not yet attained the
capability to assume and perform the police functions of PCG over marine pollution, the PCG and
PNP Maritime Group shall coordinate with regard to the enforcement of laws, rules, and
regulations governing marine pollution within the territorial waters of the Philippines. This was
made clear in Sec. 124, RA 8550 or the Philippine Fisheries Code of 1998, in which both the PCG

440
and PNP Maritime Group were authorized to enforce said law and other fishery laws, rules, and
regulations.[25]

(8) In accordance with Sec. 2 of EO 513, the PPA is mandated “to establish, develop,
regulate, manage and operate a rationalized national port system in support of trade and
national development.”[26] Moreover, Sec. 6-c of EO 513 states that the PPA has police authority
within the

ports administered by it as may be necessary to carry out its powers and functions
and attain its purposes and objectives, without prejudice to the exercise of the
functions of the Bureau of Customs and other law enforcement bodies within the
area. Such police authority shall include the following:
xxxx

b) To regulate the entry to, exit from, and movement within the port, of persons
and vehicles, as well as movement within the port of watercraft.[27]

Lastly, as a member of the International Marine Organization and a signatory to the


International Convention for the Prevention of Pollution from Ships, as amended by MARPOL
73/78,[28] the Philippines, through the PPA, must ensure the provision of adequate reception
facilities at ports and terminals for the reception of sewage from the ships docking in Philippine
ports. Thus, the PPA is tasked to adopt such measures as are necessary to prevent the
discharge and dumping of solid and liquid wastes and other ship-generated wastes into
the Manila Bay waters from vessels docked at ports and apprehend the violators. When the
vessels are not docked at ports but within Philippine territorial waters, it is the PCG and PNP
Maritime Group that have jurisdiction over said vessels.

(9) The MMDA, as earlier indicated, is duty-bound to put up and maintain adequate
sanitary landfill and solid waste and liquid disposal system as well as other alternative garbage
disposal systems. It is primarily responsible for the implementation and enforcement of the
provisions of RA 9003, which would necessary include its penal provisions, within its area of
jurisdiction.[29]

Among the prohibited acts under Sec. 48, Chapter VI of RA 9003 that are frequently
violated are dumping of waste matters in public places, such as roads, canals or esteros, open
burning of solid waste, squatting in open dumps and landfills, open dumping, burying of
biodegradable or non- biodegradable materials in flood-prone areas, establishment or operation

441
of open dumps as enjoined in RA 9003, and operation of waste management facilities without an
environmental compliance certificate.

Under Sec. 28 of the Urban Development and Housing Act of 1992 (RA 7279), eviction or
demolition may be allowed “when persons or entities occupy danger areas such as esteros,
railroad tracks, garbage dumps, riverbanks, shorelines, waterways, and other public places such
as sidewalks, roads, parks and playgrounds.” The MMDA, as lead agency, in coordination with
the DPWH, LGUs, and concerned agencies, can dismantle and remove all structures,
constructions, and other encroachments built in breach of RA 7279 and other pertinent laws
along the rivers, waterways, and esteros in Metro Manila. With respect to rivers, waterways,
and esteros in Bulacan, Bataan, Pampanga, Cavite, and Laguna that discharge wastewater
directly or eventually into the Manila Bay, the DILG shall direct the concerned LGUs to
implement the demolition and removal of such structures, constructions, and other
encroachments built in violation of RA 7279 and other applicable laws in coordination with the
DPWH and concerned agencies.

(10) The Department of Health (DOH), under Article 76 of PD 1067 (the Water Code), is
tasked to promulgate rules and regulations for the establishment of waste disposal areas that
affect the source of a water supply or a reservoir for domestic or municipal use. And under Sec.
8 of RA 9275, the DOH, in coordination with the DENR, DPWH, and other concerned agencies,
shall formulate guidelines and standards for the collection, treatment, and disposal of sewage
and the establishment and operation of a centralized sewage treatment system. In areas not
considered as highly urbanized cities, septage or a mix sewerage-septage management system
shall be employed.

In accordance with Sec. 72[30] of PD 856, the Code of Sanitation of the Philippines, and
Sec. 5.1.1[31] of Chapter XVII of its implementing rules, the DOH is also ordered to ensure the
regulation and monitoring of the proper disposal of wastes by private sludge companies through
the strict enforcement of the requirement to obtain an environmental sanitation clearance of
sludge collection treatment and disposal before these companies are issued their environmental
sanitation permit.

(11) The Department of Education (DepEd), under the Philippine Environment Code (PD
1152), is mandated to integrate subjects on environmental education in its school curricula at all
levels.[32] Under Sec. 118 of RA 8550, the DepEd, in collaboration with the DA, Commission on
Higher Education, and Philippine Information Agency, shall launch and pursue a nationwide

442
educational campaign to promote the development, management, conservation, and proper use
of the environment. Under the Ecological Solid Waste Management Act (RA 9003), on the other
hand, it is directed to strengthen the integration of environmental concerns in school curricula at
all levels, with an emphasis on waste management principles.[33]

(12) The Department of Budget and Management (DBM) is tasked under Sec. 2, Title
XVII of the Administrative Code of 1987 to ensure the efficient and sound utilization of
government funds and revenues so as to effectively achieve the country’s development
objectives.[34]

One of the country’s development objectives is enshrined in RA 9275 or the Philippine


Clean Water Act of 2004. This law stresses that the State shall pursue a policy of economic
growth in a manner consistent with the protection, preservation, and revival of the quality of our
fresh, brackish, and marine waters. It also provides that it is the policy of the government,
among others, to streamline processes and procedures in the prevention, control, and
abatement of pollution mechanisms for the protection of water resources; to promote
environmental strategies and use of appropriate economic instruments and of control
mechanisms for the protection of water resources; to formulate a holistic national program of
water quality management that recognizes that issues related to this management cannot be
separated from concerns about water sources and ecological protection, water supply, public
health, and quality of life; and to provide a comprehensive management program for water
pollution focusing on pollution prevention.

Thus, the DBM shall then endeavor to provide an adequate budget to attain the noble
objectives of RA 9275 in line with the country’s development objectives.

All told, the aforementioned enabling laws and issuances are in themselves clear,
categorical, and complete as to what are the obligations and mandate of each agency/petitioner
under the law. We need not belabor the issue that their tasks include the cleanup of
the Manila Bay.

Now, as to the crux of the petition. Do Secs. 17 and 20 of the Environment Code
encompass the cleanup of water pollution in general, not just specific pollution incidents?

Secs. 17 and 20 of the Environment Code


Include Cleaning in General

443
The disputed sections are quoted as follows:

Section 17. Upgrading of Water Quality.––Where the quality of water has


deteriorated to a degree where its state will adversely affect its best usage, the
government agencies concerned shall take such measures as may be necessary to
upgrade the quality of such water to meet the prescribed water quality standards.

Section 20. Clean-up Operations.––It shall be the responsibility of the polluter to


contain, remove and clean-up water pollution incidents at his own expense. In case
of his failure to do so, the government agencies concerned shall undertake
containment, removal and clean-up operations and expenses incurred in said
operations shall be charged against the persons and/or entities responsible for such
pollution.

When the Clean Water Act (RA 9275) took effect, its Sec. 16 on the subject, Cleanup
Operations, amended the counterpart provision (Sec. 20) of the Environment Code (PD 1152).
Sec. 17 of PD 1152 continues, however, to be operational.

The amendatory Sec. 16 of RA 9275 reads:

SEC. 16. Cleanup Operations.––Notwithstanding the provisions of Sections


15 and 26 hereof, any person who causes pollution in or pollutes water bodies in
excess of the applicable and prevailing standards shall be responsible to contain,
remove and clean up any pollution incident at his own expense to the extent that
the same water bodies have been rendered unfit for utilization and beneficial use:
Provided, That in the event emergency cleanup operations are necessary and the
polluter fails to immediately undertake the same, the [DENR] in coordination with
other government agencies concerned, shall undertake containment, removal and
cleanup operations. Expenses incurred in said operations shall be reimbursed by the
persons found to have caused such pollution under proper administrative
determination x x x. Reimbursements of the cost incurred shall be made to the
Water Quality Management Fund or to such other funds where said disbursements
were sourced.

As may be noted, the amendment to Sec. 20 of the Environment Code is more apparent
than real since the amendment, insofar as it is relevant to this case, merely consists in the
designation of the DENR as lead agency in the cleanup operations.

Petitioners contend at every turn that Secs. 17 and 20 of the Environment Code concern
themselves only with the matter of cleaning up in specific pollution incidents, as opposed to
cleanup in general. They aver that the twin provisions would have to be read alongside the

444
succeeding Sec. 62(g) and (h), which defines the terms “cleanup operations” and “accidental
spills,” as follows:

g. Clean-up Operations [refer] to activities conducted in removing


the pollutants discharged or spilled in water to restore it to pre-spillcondition.

h. Accidental Spills [refer] to spills of oil or other hazardous substances


in water that result from accidents such as collisions and groundings.

Petitioners proffer the argument that Secs. 17 and 20 of PD 1152 merely direct the
government agencies concerned to undertake containment, removal, and cleaning operations of
a specific polluted portion or portions of the body of water concerned. They maintain that the
application of said Sec. 20 is limited only to “water pollution incidents,” which are situations that
presuppose the occurrence of specific, isolated pollution events requiring the corresponding
containment, removal, and cleaning operations. Pushing the point further, they argue that the
aforequoted Sec. 62(g) requires “cleanup operations” to restore the body of water to pre-spill
condition, which means that there must have been a specific incident of either intentional or
accidental spillage of oil or other hazardous substances, as mentioned in Sec. 62(h).

As a counterpoint, respondents argue that petitioners erroneously read Sec. 62(g) as


delimiting the application of Sec. 20 to the containment, removal, and cleanup operations for
accidental spills only. Contrary to petitioners’ posture, respondents assert that Sec. 62(g), in
fact, even expanded the coverage of Sec. 20. Respondents explain that without its Sec. 62(g),
PD 1152 may have indeed covered only pollution accumulating from the day-to-day operations
of businesses around the Manila Bay and other sources of pollution that slowly accumulated in
the bay. Respondents, however, emphasize that Sec. 62(g), far from being a delimiting
provision, in fact even enlarged the operational scope of Sec. 20, by including accidental spills as
among the water pollution incidents contemplated in Sec. 17 in relation to Sec. 20 of PD 1152.

To respondents, petitioners’ parochial view on environmental issues, coupled with their


narrow reading of their respective mandated roles, has contributed to the worsening water
quality of the Manila Bay. Assuming, respondents assert, that petitioners are correct in saying
that the cleanup coverage of Sec. 20 of PD 1152 is constricted by the definition of the phrase
“cleanup operations” embodied in Sec. 62(g), Sec. 17 is not hobbled by such limiting definition.
As pointed out, the phrases “cleanup operations” and “accidental spills” do not appear in said
Sec. 17, not even in the chapter where said section is found.

445
Respondents are correct. For one thing, said Sec. 17 does not in any way state that the
government agencies concerned ought to confine themselves to the containment, removal, and
cleaning operations when a specific pollution incident occurs. On the contrary, Sec. 17 requires
them to act even in the absence of a specific pollution incident, as long as water quality “has
deteriorated to a degree where its state will adversely affect its best usage.” This section, to
stress, commands concerned government agencies, when appropriate, “to take such measures
as may be necessary to meet the prescribed water quality standards.” In fine, the underlying
duty to upgrade the quality of water is not conditional on the occurrence of any pollution
incident.

For another, a perusal of Sec. 20 of the Environment Code, as couched, indicates that it is
properly applicable to a specific situation in which the pollution is caused by polluters who fail to
clean up the mess they left behind. In such instance, the concerned government agencies shall
undertake the cleanup work for the polluters’ account. Petitioners’ assertion, that they have to
perform cleanup operations in the Manila Bay only when there is a water pollution incident and
the erring polluters do not undertake the containment, removal, and cleanup operations, is quite
off mark. As earlier discussed, the complementary Sec. 17 of the Environment Code comes into
play and the specific duties of the agencies to clean up come in even if there are no pollution
incidents staring at them. Petitioners, thus, cannot plausibly invoke and hide behind Sec. 20 of
PD 1152 or Sec. 16 of RA 9275 on the pretext that their cleanup mandate depends on the
happening of a specific pollution incident. In this regard, what the CA said with respect to the
impasse over Secs. 17 and 20 of PD 1152 is at once valid as it is practical. The appellate court
wrote: “PD 1152 aims to introduce a comprehensive program of environmental protection and
management. This is better served by making Secs. 17 & 20 of general application rather than
limiting them to specific pollution incidents.”[35]

Granting arguendo that petitioners’ position thus described vis-à-vis the implementation
of Sec. 20 is correct, they seem to have overlooked the fact that the pollution of
the Manila Bay is of such magnitude and scope that it is well-nigh impossible to draw the line
between a specific and a general pollution incident. And such impossibility extends to pinpointing
with reasonable certainty who the polluters are. We note that Sec. 20 of PD 1152 mentions
“water pollution incidents” which may be caused by polluters in the waters of
the Manila Bay itself or by polluters in adjoining lands and in water bodies or waterways that
empty into the bay. Sec. 16 of RA 9275, on the other hand, specifically adverts to “any person
who causes pollution in or pollutes water bodies,” which may refer to an individual or an
establishment that pollutes the land mass near the Manila Bay or the waterways, such that the

446
contaminants eventually end up in the bay. In this situation, the water pollution incidents are so
numerous and involve nameless and faceless polluters that they can validly be categorized as
beyond the specific pollution incident level.

Not to be ignored of course is the reality that the government agencies concerned are so
undermanned that it would be almost impossible to apprehend the numerous polluters of
the Manila Bay. It may perhaps not be amiss to say that the apprehension, if any, of
the Manila Bay polluters has been few and far between. Hence, practically nobody has been
required to contain, remove, or clean up a given water pollution incident. In this kind of setting,
it behooves the Government to step in and undertake cleanup operations. Thus, Sec. 16 of RA
9275, previously Sec. 20 of PD 1152, covers for all intents and purposes a general cleanup
situation.

The cleanup and/or restoration of the Manila Bay is only an aspect and the initial stage of
the long-term solution. The preservation of the water quality of the bay after the rehabilitation
process is as important as the cleaning phase. It is imperative then that the wastes and
contaminants found in the rivers, inland bays, and other bodies of water be stopped from
reaching theManila Bay. Otherwise, any cleanup effort would just be a futile, cosmetic exercise,
for, in no time at all, the Manila Bay water quality would again deteriorate below the ideal
minimum standards set by PD 1152, RA 9275, and other relevant laws. It thus behooves the
Court to put the heads of the petitioner-department-agencies and the bureaus and offices under
them on continuing notice about, and to enjoin them to perform, their mandates and duties
towards cleaning up the Manila Bay and preserving the quality of its water to the ideal level.
Under what other judicial discipline describes as “continuing mandamus,”[36] the Court may,
under extraordinary circumstances, issue directives with the end in view of ensuring that its
decision would not be set to naught by administrative inaction or indifference. In India, the
doctrine of continuing mandamus was used to enforce directives of the court to clean up the
length of the Ganges River from industrial and municipal pollution.[37]

The Court can take judicial notice of the presence of shanties and other unauthorized
structures which do not have septic tanks along the Pasig-Marikina-San Juan Rivers, the National
Capital Region (NCR) (Parañaque-Zapote, Las Piñas) Rivers, the Navotas-Malabon-Tullahan-
Tenejeros Rivers, the Meycuayan-Marilao-Obando (Bulacan) Rivers, the Talisay (Bataan) River,
the Imus (Cavite) River, the Laguna De Bay, and other minor rivers and connecting waterways,
river banks, and esteros which discharge their waters, with all the accompanying filth, dirt, and
garbage, into the major rivers and eventually the Manila Bay. If there is one factor responsible

447
for the pollution of the major river systems and the Manila Bay, these unauthorized structures
would be on top of the list. And if the issue of illegal or unauthorized structures is not seriously
addressed with sustained resolve, then practically all efforts to cleanse these important bodies of
water would be for naught. The DENR Secretary said as much.[38]

Giving urgent dimension to the necessity of removing these illegal structures is Art. 51 of
PD 1067 or the Water Code,[39]which prohibits the building of structures within a given length
along banks of rivers and other waterways. Art. 51 reads:

The banks of rivers and streams and the shores of the seas and
lakes throughout their entire length and within a zone of three (3) meters in urban
areas, twenty (20) meters in agricultural areas and forty (40) meters in forest
areas, along their margins, are subject to the easement of public use in the interest
of recreation, navigation, floatage, fishing and salvage. No person shall be allowed
to stay in this zone longer than what is necessary for recreation, navigation,
floatage, fishing or salvage or to build structures of any kind. (Emphasis added.)

Judicial notice may likewise be taken of factories and other industrial establishments
standing along or near the banks of the Pasig River, other major rivers, and connecting
waterways. But while they may not be treated as unauthorized constructions, some of these
establishments undoubtedly contribute to the pollution of the Pasig River and waterways. The
DILG and the concerned LGUs, have, accordingly, the duty to see to it that non-complying
industrial establishments set up, within a reasonable period, the necessary waste water
treatment facilities and infrastructure to prevent their industrial discharge, including their
sewage waters, from flowing into the Pasig River, other major rivers, and connecting waterways.
After such period, non-complying establishments shall be shut down or asked to transfer their
operations.

At this juncture, and if only to dramatize the urgency of the need for petitioners-agencies
to comply with their statutory tasks, we cite the Asian Development Bank-commissioned study
on the garbage problem in Metro Manila, the results of which are embodied in the The Garbage
Book. As there reported, the garbage crisis in the metropolitan area is as alarming as it is
shocking. Some highlights of the report:

1. As early as 2003, three land-filled dumpsites in Metro Manila - the


Payatas, Catmon and Rodriquez dumpsites - generate an alarming quantity of lead
and leachate or liquid run-off. Leachate are toxic liquids that flow along the surface
and seep into the earth and poison the surface and groundwater that are used for
drinking, aquatic life, and the environment.

448
2. The high level of fecal coliform confirms the presence of a large amount of
human waste in the dump sites and surrounding areas, which is presumably
generated by households that lack alternatives to sanitation. To say
that Manila Bay needs rehabilitation is an understatement.

3. Most of the deadly leachate, lead and other dangerous contaminants and
possibly strains of pathogens seeps untreated into ground water and runs into
the Marikina and Pasig River systems and Manila Bay.[40]

Given the above perspective, sufficient sanitary landfills should now more than ever be
established as prescribed by the Ecological Solid Waste Management Act (RA 9003). Particular
note should be taken of the blatant violations by some LGUs and possibly the MMDA of Sec. 37,
reproduced below:
Sec. 37. Prohibition against the Use of Open Dumps for Solid Waste.––No
open dumps shall be established and operated, nor any practice or disposal of solid
waste by any person, including LGUs which [constitute] the use of open dumps for
solid waste, be allowed after the effectivity of this Act: Provided, further that no
controlled dumps shall be allowed (5) years following the effectivity of this Act.
(Emphasis added.)

RA 9003 took effect on February 15, 2001 and the adverted grace period of five (5) years
which ended on February 21, 2006 has come and gone, but no single sanitary landfill which
strictly complies with the prescribed standards under RA 9003 has yet been set up.

In addition, there are rampant and repeated violations of Sec. 48 of RA 9003, like
littering, dumping of waste matters in roads, canals, esteros, and other public places, operation
of open dumps, open burning of solid waste, and the like. Some sludge companies which do not
have proper disposal facilities simply discharge sludge into the Metro Manila sewerage system
that ends up in the Manila Bay. Equally unabated are violations of Sec. 27 of RA 9275, which
enjoins the pollution of water bodies, groundwater pollution, disposal of infectious wastes from
vessels, and unauthorized transport or dumping into sea waters of sewage or solid waste and of
Secs. 4 and 102 of RA 8550 which proscribes the introduction by human or machine of
substances to the aquatic environment including “dumping/disposal of waste and other marine
litters, discharge of petroleum or residual products of petroleum of carbonaceous
materials/substances [and other] radioactive, noxious or harmful liquid, gaseous or solid
substances, from any water, land or air transport or other human-made structure.”

In the light of the ongoing environmental degradation, the Court wishes to emphasize the
extreme necessity for all concerned executive departments and agencies to immediately act and
discharge their respective official duties and obligations. Indeed, time is of the essence; hence,
449
there is a need to set timetables for the performance and completion of the tasks, some of them
as defined for them by law and the nature of their respective offices and mandates.

The importance of the Manila Bay as a sea resource, playground, and as a historical
landmark cannot be over-emphasized. It is not yet too late in the day to restore
the Manila Bay to its former splendor and bring back the plants and sea life that once thrived in
its blue waters. But the tasks ahead, daunting as they may be, could only be accomplished if
those mandated, with the help and cooperation of all civic-minded individuals, would put their
minds to these tasks and take responsibility. This means that the State, through petitioners, has
to take the lead in the preservation and protection of the Manila Bay.

The era of delays, procrastination, and ad hoc measures is over. Petitioners must
transcend their limitations, real or imaginary, and buckle down to work before the problem at
hand becomes unmanageable. Thus, we must reiterate that different government agencies and
instrumentalities cannot shirk from their mandates; they must perform their basic functions in
cleaning up and rehabilitating the Manila Bay. We are disturbed by petitioners’ hiding behind two
untenable claims: (1) that there ought to be a specific pollution incident before they are required
to act; and (2) that the cleanup of the bay is a discretionary duty.

RA 9003 is a sweeping piece of legislation enacted to radically transform and improve


waste management. It implements Sec. 16, Art. II of the 1987 Constitution, which explicitly
provides that the State shall protect and advance the right of the people to a balanced and
healthful ecology in accord with the rhythm and harmony of nature.

So it was that in Oposa v. Factoran, Jr. the Court stated that the right to a balanced and
healthful ecology need not even be written in the Constitution for it is assumed, like other civil
and political rights guaranteed in the Bill of Rights, to exist from the inception of mankind and it
is an issue of transcendental importance with intergenerational implications.[41] Even assuming
the absence of a categorical legal provision specifically prodding petitioners to clean up the bay,
they and the men and women representing them cannot escape their obligation to future
generations of Filipinos to keep the waters of the Manila Bay clean and clear as humanly as
possible. Anything less would be a betrayal of the trust reposed in them.

WHEREFORE, the petition is DENIED. The September 28, 2005 Decision of the CA in CA-
G.R. CV No. 76528 and SP No. 74944 and the September 13, 2002 Decision of the RTC in Civil

450
Case No. 1851-99 are AFFIRMED but withMODIFICATIONS in view of subsequent developments
or supervening events in the case. The fallo of the RTC Decision shall now read:

WHEREFORE, judgment is hereby rendered ordering the abovenamed


defendant-government agencies to clean up, rehabilitate, and preserve Manila Bay,
and restore and maintain its waters to SB level (Class B sea waters per Water
Classification Tables under DENR Administrative Order No. 34 [1990]) to make
them fit for swimming, skin-diving, and other forms of contact recreation.

In particular:

(1) Pursuant to Sec. 4 of EO 192, assigning the DENR as the primary agency responsible
for the conservation, management, development, and proper use of the country’s environment
and natural resources, and Sec. 19 of RA 9275, designating the DENR as the primary
government agency responsible for its enforcement and implementation, the DENR is directed to
fully implement its Operational Plan for the Manila Bay Coastal Strategy for the rehabilitation,
restoration, and conservation of the Manila Bay at the earliest possible time. It is ordered to call
regular coordination meetings with concerned government departments and agencies to ensure
the successful implementation of the aforesaid plan of action in accordance with its indicated
completion schedules.

(2) Pursuant to Title XII (Local Government) of the Administrative Code of 1987 and Sec.
25 of the Local Government Code of 1991,[42] the DILG, in exercising the President’s power of
general supervision and its duty to promulgate guidelines in establishing waste management
programs under Sec. 43 of the Philippine Environment Code (PD 1152), shall direct all LGUs in
Metro Manila, Rizal, Laguna, Cavite, Bulacan, Pampanga, and Bataan to inspect all factories,
commercial establishments, and private homes along the banks of the major river systems in
their respective areas of jurisdiction, such as but not limited to the Pasig-Marikina-San Juan
Rivers, the NCR (Parañaque-Zapote, Las Piñas) Rivers, the Navotas-Malabon-Tullahan-Tenejeros
Rivers, the Meycauayan-Marilao-Obando (Bulacan) Rivers, the Talisay (Bataan) River, the Imus
(Cavite) River, the Laguna De Bay, and other minor rivers and waterways that eventually
discharge water into the Manila Bay; and the lands abutting the bay, to determine whether they
have wastewater treatment facilities or hygienic septic tanks as prescribed by existing laws,
ordinances, and rules and regulations. If none be found, these LGUs shall be ordered to require
non-complying establishments and homes to set up said facilities or septic tanks within a
reasonable time to prevent industrial wastes, sewage water, and human wastes from flowing
into these rivers, waterways, esteros, and the Manila Bay, under pain of closure or imposition of
fines and other sanctions.

451
(3) As mandated by Sec. 8 of RA 9275,[43] the MWSS is directed to provide, install,
operate, and maintain the necessary adequate waste water treatment facilities in Metro Manila,
Rizal, and Cavite where needed at the earliest possible time.

(4) Pursuant to RA 9275,[44] the LWUA, through the local water districts and in
coordination with the DENR, is ordered to provide, install, operate, and maintain sewerage and
sanitation facilities and the efficient and safe collection, treatment, and disposal of sewage in the
provinces of Laguna, Cavite, Bulacan, Pampanga, and Bataan where needed at the earliest
possible time.

(5) Pursuant to Sec. 65 of RA 8550,[45] the DA, through the BFAR, is ordered to improve
and restore the marine life of the Manila Bay. It is also directed to assist the LGUs in Metro
Manila, Rizal, Cavite, Laguna, Bulacan, Pampanga, and Bataan in developing, using recognized
methods, the fisheries and aquatic resources in the Manila Bay.

(6) The PCG, pursuant to Secs. 4 and 6 of PD 979, and the PNP Maritime Group, in
accordance with Sec. 124 of RA 8550, in coordination with each other, shall apprehend violators
of PD 979, RA 8550, and other existing laws and regulations designed to prevent marine
pollution in the Manila Bay.

(7) Pursuant to Secs. 2 and 6-c of EO 513[46] and the International Convention for the
Prevention of Pollution from Ships, the PPA is ordered to immediately adopt such measures to
prevent the discharge and dumping of solid and liquid wastes and other ship-generated wastes
into the Manila Bay waters from vessels docked at ports and apprehend the violators.

(8) The MMDA, as the lead agency and implementor of programs and projects for flood
control projects and drainage services in Metro Manila, in coordination with the DPWH, DILG,
affected LGUs, PNP Maritime Group, Housing and Urban Development Coordinating Council
(HUDCC), and other agencies, shall dismantle and remove all structures, constructions, and
other encroachments established or built in violation of RA 7279, and other applicable laws along
the Pasig-Marikina-San Juan Rivers, the NCR (Parañaque-Zapote, Las Piñas) Rivers, the
Navotas-Malabon-Tullahan-Tenejeros Rivers, and connecting waterways and esteros in Metro
Manila. The DPWH, as the principal implementor of programs and projects for flood control
services in the rest of the country more particularly in Bulacan, Bataan, Pampanga, Cavite, and
Laguna, in coordination with the DILG, affected LGUs, PNP Maritime Group, HUDCC, and other

452
concerned government agencies, shall remove and demolish all structures, constructions, and
other encroachments built in breach of RA 7279 and other applicable laws along the
Meycauayan-Marilao-Obando (Bulacan) Rivers, the Talisay (Bataan) River, the Imus (Cavite)
River, the Laguna De Bay, and other rivers, connecting waterways, and esteros that discharge
wastewater into the Manila Bay.

In addition, the MMDA is ordered to establish, operate, and maintain a sanitary landfill, as
prescribed by RA 9003, within a period of one (1) year from finality of this Decision. On matters
within its territorial jurisdiction and in connection with the discharge of its duties on the
maintenance of sanitary landfills and like undertakings, it is also ordered to cause the
apprehension and filing of the appropriate criminal cases against violators of the respective
penal provisions of RA 9003,[47] Sec. 27 of RA 9275 (the Clean Water Act), and other existing
laws on pollution.

(9) The DOH shall, as directed by Art. 76 of PD 1067 and Sec. 8 of RA 9275, within one
(1) year from finality of this Decision, determine if all licensed septic and sludge companies have
the proper facilities for the treatment and disposal of fecal sludge and sewage coming from
septic tanks. The DOH shall give the companies, if found to be non-complying, a reasonable time
within which to set up the necessary facilities under pain of cancellation of its environmental
sanitation clearance.

(10) Pursuant to Sec. 53 of PD 1152,[48] Sec. 118 of RA 8550, and Sec. 56 of RA


9003,[49] the DepEd shall integrate lessons on pollution prevention, waste management,
environmental protection, and like subjects in the school curricula of all levels to inculcate in the
minds and hearts of students and, through them, their parents and friends, the importance of
their duty toward achieving and maintaining a balanced and healthful ecosystem in the Manila
Bay and the entire Philippine archipelago.

(11) The DBM shall consider incorporating an adequate budget in the General
Appropriations Act of 2010 and succeeding years to cover the expenses relating to the cleanup,
restoration, and preservation of the water quality of the Manila Bay, in line with the country’s
development objective to attain economic growth in a manner consistent with the protection,
preservation, and revival of our marine waters.

(12) The heads of petitioners-agencies MMDA, DENR, DepEd, DOH, DA, DPWH, DBM,
PCG, PNP Maritime Group, DILG, and also of MWSS, LWUA, and PPA, in line with the principle of

453
“continuing mandamus,” shall, from finality of this Decision, each submit to the Court a quarterly
progressive report of the activities undertaken in accordance with this Decision.

No costs.

SO ORDERED.

PRESBITERO J. VELASCO, JR.


Associate Justice
WE CONCUR:

REYNATO S. PUNO
Chief Justice

LEONARDO A. QUISUMBING CONSUELO YNARES-SANTIAGO


Associate Justice Associate Justice

ANTONIO T. CARPIO MA. ALICIA AUSTRIA-MARTINEZ


Associate Justice Associate Justice

RENATO C. CORONA CONCHITA CARPIO MORALES Associate


Justice Associate Justice

ADOLFO S. AZCUNA DANTE O. TINGA


Associate Justice Associate Justice

MINITA V. CHICO-NAZARIO ANTONIO EDUARDO B. NACHURA


Associate Justice Associate Justice

454
RUBEN T. REYES TERESITA J. LEONARDO-DE CASTRO
Associate Justice Associate Justice

ARTURO D. BRION
Associate Justice

CERTIFICATION

Pursuant to Section 13, Article VIII of the Constitution, it is hereby certified that the
conclusions in the above Decision were reached in consultation before the case was assigned to
the writer of the opinion of the Court.

REYNATO S. PUNO
Chief Justice

[1]
Now the Department of Education (DepEd).
[2]
Gore, AN INCONVENIENT TRUTH 161.
[3]
Rollo, p. 74.
[4]
Id. at 53.
[5]
Id. at 109-123. Penned by Executive Judge Lucenito N. Tagle (now retired Court of
Appeals Justice).
[6]
Id. at 47-58. Penned by Associate Justice Eliezer R. De Los Santos and concurred
in by Associate Justices Eugenio S. Labitoria and Jose C. Reyes, Jr.
[7]
Id. at 52.
[8]
Angchangco, Jr. v. Ombudsman, G.R. No. 122728, February 13, 1997, 268 SCRA
301, 306.
[9]
BLACK’S LAW DICTIONARY (8th ed., 2004).
[10]
Lamb v. Phipps, 22 Phil. 456, 490 (1912).
[11]
G.R. No. 156052, March 7, 2007, 517 SCRA 657, as subsequently reiterated
on February 13, 2008.
[12]
RA 9003 was approved on January 26, 2001.
[13]
2 Feria Noche, CIVIL PROCEDURE ANNOTATED.
[14]
BLACK’S LAW DICTIONARY (8th ed., 2004).
[15]
“Providing for the Reorganization of the [DENR], Renaming it as the Department
of Environment and Natural Resources, and for Other Purposes.”

455
[16]
Per DENR Secretary Jose Atienza, the DENR is preparing an EO for the purpose. TSN of
oral arguments, p. 118.
[17]
Per information from the Water Quality Management Section, Environmental
Management Bureau, DENR, as validated by the DENR Secretary during the oral arguments.
TSN, pp. 119-120.
[18]
“An Act Creating the [MWSS] and Dissolving the National Waterworks and Sewerage
Authority [NAWASA]; and for Other Purposes.”
[19]
Sec. 22. Linkage Mechanism.––The [DENR] and its concerned attached agencies x x x
shall coordinate and enter into agreement with other government agencies, industrial sector and
other concerned sectors in the furtherance of the objectives of this Act. The following agencies
shall perform tile functions specified hereunder:
xxxx
b) DPWH through its attached agencies, such as the MWSS, LWUA, and including other
urban water utilities for the provision or sewerage and sanitation facilities and the efficient and
safe collection, treatment and disposal of sewage within their area of jurisdiction.
[20]
Book IV, Title IV, Sec. 2.
[21]
Sec. 14. Monitoring Control and Surveillance of the Philippine Waters.––A monitoring,
control and surveillance system shall be established by the [DA] in coordination with LGUs and
other agencies concerned to ensure that the fisheries and aquatic resources in the Philippine
waters are judiciously and wisely utilized and managed on a sustainable basis x x x.
[22]
Sec. 22. Linkage Mechanism.––x x x x
a) Philippine Coast Guard in coordination with DA and DENR shall enforce for the
enforcement of water quality standards in marine waters x x x specifically from offshore sources;
xxxx
c) DA, shall coordinate with the DENR, in the formulation of guidelines x x x for the
prevention, control and abatement of pollution from agricultural and aquaculture activities x x x
Provided, further, That the x x x BFAR of the DA shall be primarily responsible for the prevention
and control of water pollution for the development, management and conservation of the
fisheries and aquatic resources.
[23]
Book IV, Title V, Sec. 2. Mandate.––The [DPWH] shall be the State’s engineering arm
and is tasked to carry out the policy enumerated above [i.e., the planning, design, construction,
and maintenance of infrastructure facilities, especially x x x flood control and water resources
development systems].
Sec. 3. Powers and Functions.––The Department, in order to carry out its mandate, shall:
xxxx
(2) Develop and implement effective codes, standards, and reasonable guidelines to
ensure the safety of all public and private structures in the country and assure efficiency and
proper quality in the construction of public works;
(3) Ascertain that all public works plans and project implementation designs are
consistent with current standards and guidelines;
xxxx
(8) Provide an integrated planning for x x x flood control and water resource and water
resource development systems x x x.
[24]
Sec. 6. Enforcement and Implementation.—The [PCG] shall have the primary
responsibility of enforcing the laws, rules and regulations governing marine pollution. However,
it shall be the joint responsibility of the [PCG] and the National Pollution Control Commission to
coordinate and cooperate with each other in the enforcement of the provisions of this decree and
its implementing rules and regulations, and may call upon any other government office,
instrumentality or agency to extend every assistance in this respect.

456
[25]
Sec. 124. Persons and Deputies Authorized to Enforce this Code x x x.—The law
enforcements of the [DA], the Philippine Navy, [PCG, PNP], PNP-Maritime Command x x x are
hereby authorized to enforce this Code and other fishery laws x x x.
[26]
<http://www.ppa.com.ph> (visited November 20, 2008).
[27]
EO 513, “Reorganizing the Philippine Ports Authority,” Sec. 2 provides further:
Section 6 is hereby amended by adding a new paragraph to read as follows:
Sec. 6-c. Police Authority.—x x x Such police authority shall include the following:
xxxx
c) To maintain peace and order inside the port, in coordination with local police
authorities;
xxxx
e) To enforce rules and regulations promulgated by the Authority pursuant to law.
[28]
“International Convention for the Prevention of Marine Pollution from Ships, 1973
as modified by the Protocol of 1978 Relating Thereto.”
[29]
Sec. 10. Role of LGUs in Solid Waste Management.––Pursuant to the relevant
provisions of RA No. 7160, otherwise known as the Local Government Code, the LGUs shall be
primarily responsible for the implementation and enforcement of the provisions of this Act within
their respective jurisdictions.
[30]
Sec. 72. Scope of Supervision of the Department.––The approval of the Secretary
or his duly authorized representative is required in the following matters:
xxxx
(g) Method of disposal of sludge from septic tanks or other treatment plants.
[31]
Sec. 5.1.1.a. It shall be unlawful for any person, entity or firm to discharge
untreated effluent of septic tanks and/or sewage treatment plants to bodies of water without
obtaining approval from the Secretary of Health or his duly authorized representatives.
[32]
Sec. 53. Environmental Education.––The [DepEd] shall integrate subjects on
environmental education in its school curricula at all levels. It shall also endeavor to conduct
special community education emphasizing the relationship of man and nature as well as
environmental sanitation and practices.
[33]
Sec. 56. Environmental Education in the Formal and Nonformal Sectors.––The national
government, through the [DepEd] and in coordination with concerned government agencies,
NGOs and private institutions, shall strengthen the integration of environmental concerns in
school curricula at all levels, with particular emphasis on the theory and practice of waste
management principles like waste minimization, specifically resource conservation and recovery,
segregation at source, reduction, recycling, re-use, and composing, in order to promote
environmental awareness and action among the citizenry.
[34]
Title XVII, Sec. 1. Declaration of Policy.––The national budget shall be formulated and
implemented as an instrument of national development, reflective of national objectives and
plans; supportive of and consistent with the socio-economic development plans and oriented
towards the achievement of explicit objectives and expected results, to ensure that the
utilization of funds and operations of government entities are conducted effectively; formulated
within the context of a regionalized governmental structure and within the totality of revenues
and other receipts, expenditures and borrowings of all levels of government and of government-
owned or controlled corporations; and prepared within the context of the national long-term
plans and budget programs of the Government.
[35]
Rollo, p. 76.
[36]
Vineet Narain v. Union of India, 1 SCC 226 (1998).
[37]
M.C. Mehta v. Union of India, 4 SC 463 (1987).
[38]
TSN, p. 121.

457
[39]
Repealed Art. 638 of the CIVIL CODE. See E.L. Pineda, PROPERTY 399 (1999).
[40]
Asian Development Bank, THE GARBAGE BOOK 44-45 (November 2006).
[41]
G.R. No. 101083, July 30, 1993, 224 SCRA 792, 805.
[42]
Sec. 25. National Supervision over Local Government Units.––(a) Consistent with the
basic policy on local autonomy, the President shall exercise general supervision over local
government units to ensure that their acts are within the scope of their prescribed powers and
functions.
[43]
Sec. 8. Domestic Sewage Collection, Treatment and Disposal.––Within five (5) years
following the effectivity of this Act, the Agency vested to provide water supply and sewerage
facilities and/or concessionaires in Metro Manila and other highly urbanized cities (HUCs) as
defined in [RA] 7160, in coordination with LGUs, shall be required to connect the existing
sewage line found in all subdivisions, condominiums, commercial centers, hotels, sports and
recreational facilities, hospitals, market places, public buildings, industrial complex and other
similar establishments including households to available sewerage system. Provided, That the
said connection shall be subject to sewerage services charge/fees in accordance with existing
laws, rules or regulations unless the sources had already utilized their own sewerage system:
Provided, further, That all sources of sewage and septage shall comply with the requirements
herein.
[44]
Supra note 19.
[45]
Sec. 65. Functions of the Bureau of Fisheries and Aquatic Resources.––As a line
bureau, the BFAR shall have the following functions:
xxxx
q. assist the LGUs in developing their technical capability in the development,
management, regulation, conservation, and protection of fishery resources;
xxxx
s. perform such other related function which shall promote the development,
conservation, management, protection and utilization of fisheries and aquatic resources.
[46]
Supra notes 26 & 27.
[47]
Among the prohibited and penalized acts under Sec. 48 of RA 9003 are: (1) littering
and dumping of waste matters in public places; (2) open burning of solid wastes; (3) squatting
in open dumps and landfills; (4) transporting and dumping in bulk of collected domestic,
industrial, commercial and institutional wastes in areas other than centers and facilities
prescribed under the Act; (5) construction or operation of waste management facilities without
an Environmental Compliance Certificate; and (6) construction or operation of landfills or any
waste disposal facility on any aquifer, groundwater reservoir or watershed area.
[48]
Supra note 32.
[49]
Supra note 33.

458
EN BANC

NESTOR A. JACOT,
Petitioner, G.R. No. 179848
Present:

PUNO, C.J.,
QUISUMBING,
YNARES-SANTIAGO,
CARPIO,
AUSTRIA-MARTINEZ,
CORONA,
CARPIO MORALES,
- versus - AZCUNA,
TINGA,
CHICO-NAZARIO,
VELASCO, JR.,
NACHURA,
REYES,
DE CASTRO,* and
BRION,** JJ.

ROGEN T. DAL and COMMISSION ON Promulgated:


ELECTIONS,
November 27, 2008
Respondents.

x---------------------------- ---------------------x

DECISION

CHICO-NAZARIO, J.:

Petitioner Nestor A. Jacot assails the Resolution[1] dated 28 September 2007 of the
Commission on Elections (COMELEC) En Banc in SPA No. 07-361, affirming the Resolution dated
12 June 2007 of the COMELEC Second Division[2]disqualifying him from running for the position
of Vice-Mayor of Catarman, Camiguin, in the 14 May 2007 National and Local Elections, on the
ground that he failed to make a personal renouncement of his United States (US) citizenship.
459
Petitioner was a natural born citizen of the Philippines, who became a naturalized citizen
of the US on 13 December 1989.[3]

Petitioner sought to reacquire his Philippine citizenship under Republic Act No. 9225,
otherwise known as the Citizenship Retention and Re-Acquisition Act. He filed a request for the
administration of his Oath of Allegiance to the Republic of thePhilippines with the Philippine
Consulate General (PCG) of Los Angeles, California. The Los Angeles PCG issued on 19 June
2006 an Order of Approval[4] of petitioner’s request, and on the same day, petitioner took his
Oath of Allegiance to the Republic of the Philippines before Vice Consul Edward
C. Yulo. [5] On 27 September 2006, the Bureau of Immigration issued Identification Certificate
No. 06-12019 recognizing petitioner as a citizen of the Philippines.[6]

Six months after, on 26 March 2007, petitioner filed his Certificate of Candidacy for the
Position of Vice-Mayor of theMunicipality of Catarman, Camiguin. [7]

On 2 May 2007, respondent Rogen T. Dal filed a Petition for Disqualification[8] before the
COMELEC Provincial Office in Camiguin against petitioner, arguing that the latter failed to
renounce his US citizenship, as required under Section 5(2) of Republic Act No. 9225, which
reads as follows:

Section 5. Civil and Political Rights and Liabilities.—Those who retain or


reacquire Philippine citizenship under this Act shall enjoy full civil and political rights
and be subject to all attendant liabilities and responsibilities under existing laws of
the Philippines and the following conditions:

xxxx

(2) Those seeking elective public office in the Philippines shall meet the
qualifications for holding such public office as required by the Constitution and
existing laws and, at the time of the filing of the certificate of candidacy, make a
personal and sworn renunciation of any and all foreign citizenship before any public
officer authorized to administer an oath.

In his Answer[9] dated 6 May 2007 and Position Paper[10] dated 8 May 2007, petitioner
countered that his Oath of Allegiance to the Republic of the Philippines made before the Los
Angeles PCG and the oath contained in his Certificate of Candidacy operated as an effective
renunciation of his foreign citizenship.

460
In the meantime, the 14 May 2007 National and Local Elections were held. Petitioner
garnered the highest number of votes for the position of Vice Mayor.

On 12 June 2007, the COMELEC Second Division finally issued its


[11]
Resolution disqualifying the petitioner from running for the position of Vice-Mayor
of Catarman, Camiguin, for failure to make the requisite renunciation of his UScitizenship. The
COMELEC Second Division explained that the reacquisition of Philippine citizenship under
Republic Act No. 9225 does not automatically bestow upon any person the privilege to run for
any elective public office. It additionally ruled that the filing of a Certificate of Candidacy cannot
be considered as a renunciation of foreign citizenship. The COMELEC Second Division did not
consider Valles v. COMELEC[12] and Mercado v. Manzano[13] applicable to the instant case,
since Valles and Mercado were dual citizens since birth, unlike the petitioner who lost his Filipino
citizenship by means of naturalization. The COMELEC, thus, decreed in the aforementioned
Resolution that:

ACCORDINGLY, NESTOR ARES JACOT is DISQUALIFIED to run for the


position of Vice-Mayor of Catarman, Camiguin for the May 14, 2007 National and
Local Elections. If proclaimed, respondent cannot thus assume the Office of Vice-
Mayor of said municipality by virtue of such disqualification.[14]

Petitioner filed a Motion for Reconsideration on 29 June 2007 reiterating his position that
his Oath of Allegiance to the Republic of the Philippines before the Los Angeles PCG and his oath
in his Certificate of Candidacy sufficed as an effective renunciation of
his US citizenship. Attached to the said Motion was an “Oath of Renunciation of Allegiance to
the United Statesand Renunciation of Any and All Foreign Citizenship” dated 27 June 2007,
wherein petitioner explicitly renounced his UScitizenship.[15] The COMELEC en banc dismissed
petitioner’s Motion in a Resolution[16] dated 28 September 2007 for lack of merit.

Petitioner sought remedy from this Court via the present Special Civil Action
for Certiorari under Rule 65 of the Revised Rules of Court, where he presented for the first time
an “Affidavit of Renunciation of Allegiance to the United States and Any and All Foreign
Citizenship”[17] dated 7 February 2007. He avers that he executed an act of renunciation of
his US citizenship, separate from the Oath of Allegiance to the Republic of the Philippines he took
before the Los Angeles PCG and his filing of his Certificate of Candidacy, thereby changing his
theory of the case during the appeal. He attributes the delay in the presentation of the affidavit
to his former counsel, Atty. Marciano Aparte, who allegedly advised him that said piece of
evidence was unnecessary but who, nevertheless, made him execute an identical document
461
entitled “Oath of Renunciation of Allegiance to the United States and Renunciation of Any and All
Foreign Citizenship” on 27 June 2007 after he had already filed his Certificate of Candidacy.[18]

Petitioner raises the following issues for resolution of this Court:

WHETHER OR NOT PUBLIC RESPONDENT EXERCISED GRAVE ABUSE OF


DISCRETION WHEN IT HELD THAT PETITIONER FAILED TO COMPLY WITH THE
PROVISIONS OF R.A. 9225, OTHERWISE KNOWN AS THE “CITIZENSHIP
RETENTION AND RE-ACQUISITION ACT OF 2003,” SPECIFICALLY SECTION 5(2) AS
TO THE REQUIREMENTS FOR THOSE SEEKING ELECTIVE PUBLIC OFFICE;

II

WHETHER OR NOT PUBLIC RESPONDENT EXERCISED GRAVE ABUSE OF


DISCRETION WHEN IT HELD THAT PETITIONER FAILED TO COMPLY WITH THE
PROVISIONS OF THE COMELEC RULES OF PROCEDURE AS REGARDS THE PAYMENT
OF THE NECESSARY MOTION FEES; AND

III

WHETHER OR NOT UPHOLDING THE DECISION OF PUBLIC RESPONDENT WOULD


RESULT IN THE FRUSTRATION OF THE WILL OF THE PEOPLE OF CATARMAN,
CAMIGUIN.[19]

The Court determines that the only fundamental issue in this case is whether petitioner is
disqualified from running as a candidate in the 14 May 2007 local elections for his failure to
make a personal and sworn renunciation of his US citizenship.

This Court finds that petitioner should indeed be disqualified.

Contrary to the assertions made by petitioner, his oath of allegiance to the Republic of
the Philippines made before the Los Angeles PCG and his Certificate of Candidacy do not
substantially comply with the requirement of a personal and sworn renunciation of foreign
citizenship because these are distinct requirements to be complied with for different purposes.

Section 3 of Republic Act No. 9225 requires that natural-born citizens of the Philippines,
who are already naturalized citizens of a foreign country, must take the following oath of
allegiance to the Republic of the Philippines to reacquire or retain their Philippine citizenship:

462
SEC. 3. Retention of Philippine Citizenship.—Any provision of law to the
contrary notwithstanding, natural-born citizens of the Philippines who have lost
their Philippine citizenship by reason of their naturalization as citizens of a foreign
country are hereby deemed to have reacquired Philippine citizenship upon taking
the following oath of allegiance to the Republic:

“I __________ solemnly swear (or affirm) that I will support and defend the
Constitution of the Republic of the Philippines and obey the laws and legal orders
promulgated by the duly constituted authorities of the Philippines; and I hereby
declare that I recognize and accept the supreme authority of the Philippines and will
maintain true faith and allegiance thereto; and that I impose this obligation upon
myself voluntarily, without mental reservation or purpose of evasion.”

Natural-born citizens of the Philippines who, after the effectivity of this Act,
become citizens of a foreign country shall retain their Philippine citizenship upon
taking the aforesaid oath.

By the oath dictated in the afore-quoted provision, the Filipino swears allegiance to
the Philippines, but there is nothing therein on his renunciation of foreign citizenship. Precisely,
a situation might arise under Republic Act No. 9225 wherein said Filipino has dual citizenship by
also reacquiring or retaining his Philippine citizenship, despite his foreign citizenship.

The afore-quoted oath of allegiance is substantially similar to the one contained in


the Certificate of Candidacy which must be executed by any person who wishes to run for public
office in Philippine elections. Such an oath reads:

I am eligible for the office I seek to be elected. I will support and defend the
Constitution of the Philippines and will maintain true faith and allegiance thereto;
that I will obey the laws, legal orders and decrees promulgated by the duly
constituted authorities of the Republic of the Philippines; and that I impose this
obligation upon myself voluntarily, without mental reservation or purpose of
evasion. I hereby certify that the facts stated herein are true and correct of my
own personal knowledge.

Now, Section 5(2) of Republic Act No. 9225 specifically provides that:

Section 5. Civil and Political Rights and Liabilities.—Those who retain or


reacquire Philippine citizenship under this Act shall enjoy full civil and political rights
and be subject to all attendant liabilities and responsibilities under existing laws of
the Philippines and the following conditions:

xxxx
463
(2) Those seeking elective public office in the Philippines shall meet the
qualifications for holding such public office as required by the Constitution and
existing laws and, at the time of the filing of the certificate of candidacy, make a
personal and sworn renunciation of any and all foreign citizenship before any public
officer authorized to administer an oath.

The law categorically requires persons seeking elective public office, who either retained
their Philippine citizenship or those who reacquired it, to make a personal and sworn
renunciation of any and all foreign citizenship before a public officer authorized to administer an
oath simultaneous with or before the filing of the certificate of candidacy.[20]

Hence, Section 5(2) of Republic Act No. 9225 compels natural-born Filipinos, who have
been naturalized as citizens of a foreign country, but who reacquired or retained their Philippine
citizenship (1) to take the oath of allegiance under Section 3 of Republic Act No. 9225, and (2)
for those seeking elective public offices in the Philippines, to additionally execute a personal and
sworn renunciation of any and all foreign citizenship before an authorized public officer prior or
simultaneous to the filing of their certificates of candidacy, to qualify as candidates in Philippine
elections.

Clearly Section 5(2) of Republic Act No. 9225 (on the making of a personal and sworn
renunciation of any and all foreign citizenship) requires of the Filipinos availing themselves of the
benefits under the said Act to accomplish an undertaking other than that which they have
presumably complied with under Section 3 thereof (oath of allegiance to the Republic of the
Philippines). This is made clear in the discussion of the Bicameral Conference Committee on
Disagreeing Provisions of House Bill No. 4720 and Senate Bill No. 2130 held on 18 August 2003
(precursors of Republic Act No. 9225), where the Hon. Chairman Franklin Drilon and Hon.
Representative Arthur Defensor explained to Hon. Representative Exequiel Javier that the oath
of allegiance is different from the renunciation of foreign citizenship:

CHAIRMAN DRILON. Okay. So, No. 2. “Those seeking elective public office in
the Philippines shall meet the qualifications for holding such public office as
required by the Constitution and existing laws and, at the time of the filing of the
certificate of candidacy, make a personal and sworn renunciation of any and all
foreign citizenship before any public officer authorized to administer an oath.” I
think it’s very good, ha? No problem?

REP. JAVIER. … I think it’s already covered by the oath.

CHAIRMAN DRILON. Renouncing foreign citizenship.

464
REP. JAVIER. Ah… but he has taken his oath already.

CHAIRMAN DRILON. No…no, renouncing foreign citizenship.

xxxx

CHAIRMAN DRILON. Can I go back to No. 2. What’s your problem,


Boy? Those seeking elective office in the Philippines.

REP. JAVIER. They are trying to make him renounce his citizenship
thinking that ano…

CHAIRMAN DRILON. His American citizenship.

REP. JAVIER. To discourage him from running?

CHAIRMAN DRILON. No.

REP. A.D. DEFENSOR. No. When he runs he will only have one
citizenship. When he runs for office, he will have only one. (Emphasis ours.)

There is little doubt, therefore, that the intent of the legislators was not only for Filipinos
reacquiring or retaining their Philippine citizenship under Republic Act No. 9225 to take their
oath of allegiance to the Republic of the Philippines, but also to explicitly renounce their foreign
citizenship if they wish to run for elective posts in the Philippines. To qualify as a candidate in
Philippine elections, Filipinos must only have one citizenship, namely, Philippine citizenship.

By the same token, the oath of allegiance contained in the Certificate of Candidacy, which
is substantially similar to the one contained in Section 3 of Republic Act No. 9225, does not
constitute the personal and sworn renunciation sought under Section 5(2) of Republic Act No.
9225. It bears to emphasize that the said oath of allegiance is a general requirement for all
those who wish to run as candidates in Philippine elections; while the renunciation of foreign
citizenship is an additional requisite only for those who have retained or reacquired Philippine
citizenship under Republic Act No. 9225 and who seek elective public posts, considering their
special circumstance of having more than one citizenship.

Petitioner erroneously invokes the doctrine in Valles[21] and Mercado,[22] wherein the filing
by a person with dual citizenship of a certificate of candidacy, containing an oath of allegiance,
was already considered a renunciation of foreign citizenship. The ruling of this Court

465
in Valles and Mercado is not applicable to the present case, which is now specially governed by
Republic Act No. 9225, promulgated on 29 August 2003.

In Mercado, which was cited in Valles, the disqualification of therein private


respondent Manzano was sought under another law, Section 40(d) of the Local Government
Code, which reads:

SECTION 40. Disqualifications. The following persons are disqualified from


running for any elective local position:

xxxx

(d) Those with dual citizenship.

The Court in the aforesaid cases sought to define the term “dual citizenship” vis-à-vis the
concept of “dual allegiance.” At the time this Court decided the cases
of Valles and Mercado on 26 May 1999 and 9 August 2000, respectively, the more explicitly
worded requirements of Section 5(2) of Republic Act No. 9225 were not yet enacted by our
legislature.[23]

Lopez v. Commission on Elections[24] is the more fitting precedent for this case since they
both share the same factual milieu. In Lopez, therein petitioner Lopez was a natural-born
Filipino who lost his Philippine citizenship after he became a naturalized US citizen. He later
reacquired his Philippine citizenship by virtue of Republic Act No. 9225. Thereafter, Lopez filed
his candidacy for a local elective position, but failed to make a personal and sworn renunciation
of his foreign citizenship. This Court unequivocally declared that despite having garnered the
highest number of votes in the election, Lopez is nonetheless disqualified as a candidate for a
local elective position due to his failure to comply with the requirements of Section 5(2) of
Republic Act No. 9225.

Petitioner presents before this Court for the first time, in the instant Petition
for Certiorari, an “Affidavit of Renunciation of Allegiance to the United States and Any and All
Foreign Citizenship,”[25] which he supposedly executed on 7 February 2007, even before he filed
his Certificate of Candidacy on 26 March 2007. With the said Affidavit, petitioner puts forward in
the Petition at bar a new theory of his case—that he complied with the requirement of making a
personal and sworn renunciation of his foreign citizenship before filing his Certificate of
Candidacy. This new theory constitutes a radical change from the earlier position he took before

466
the COMELEC—that he complied with the requirement of renunciation by his oaths of allegiance
to the Republic of the Philippines made before the Los Angeles PCG and in his Certificate of
Candidacy, and that there was no more need for a separate act of renunciation.

As a rule, no question will be entertained on appeal unless it has been raised in the
proceedings below. Points of law, theories, issues and arguments not brought to the attention of
the lower court, administrative agency or quasi-judicial body need not be considered by a
reviewing court, as they cannot be raised for the first time at that late stage. Basic
considerations of fairness and due process impel this rule.[26] Courts have neither the time nor
the resources to accommodate parties who chose to go to trial haphazardly.[27]

Likewise, this Court does not countenance the late submission of evidence.[28] Petitioner
should have offered the Affidavit dated 7 February 2007 during the proceedings before the
COMELEC.

Section 1 of Rule 43 of the COMELEC Rules of Procedure provides that “In the absence of
any applicable provisions of these Rules, the pertinent provisions of the Rules of Court in
the Philippines shall be applicable by analogy or in suppletorycharacter and effect.” Section 34
of Rule 132 of the Revised Rules of Court categorically enjoins the admission of evidence not
formally presented:

SEC. 34. Offer of evidence. - The court shall consider no evidence which
has not been formally offered. The purpose for which the evidence is offered must
be specified.

Since the said Affidavit was not formally offered before the COMELEC, respondent had no
opportunity to examine and controvert it. To admit this document would be contrary to due
process. [29] Additionally, the piecemeal presentation of evidence is not in accord with orderly
justice.[30]

The Court further notes that petitioner had already presented before the COMELEC an
identical document, “Oath of Renunciation of Allegiance to the United States and Renunciation of
Any and All Foreign Citizenship” executed on 27 June 2007, subsequent to his filing of his
Certificate of Candidacy on 26 March 2007. Petitioner attached the said Oath of 27 June 2007 to
his Motion for Reconsideration with the COMELEC en banc. The COMELEC en banc eventually
refused to reconsider said document for being belatedly executed. What was extremely
perplexing, not to mention suspect, was that petitioner did not submit the Affidavit of 7 February
467
2007 or mention it at all in the proceedings before the COMELEC, considering that it could have
easily won his case if it was actually executed on and in existence before the filing of his
Certificate of Candidacy, in compliance with law.

The justification offered by petitioner, that his counsel had advised him against presenting
this crucial piece of evidence, is lame and unconvincing. If the Affidavit of 7 February 2007 was
in existence all along, petitioner’s counsel, and even petitioner himself, could have easily
adduced it to be a crucial piece of evidence to prove compliance with the requirements of
Section 5(2) of Republic Act No. 9225. There was no apparent danger for petitioner to submit as
much evidence as possible in support of his case, than the risk of presenting too little for which
he could lose.

And even if it were true, petitioner’s excuse for the late presentation of the Affidavit of 7
February 2007 will not change the outcome of petitioner’s case.

It is a well-settled rule that a client is bound by his counsel’s conduct, negligence, and
mistakes in handling the case, and the client cannot be heard to complain that the result might
have been different had his lawyer proceeded differently.[31] The only exceptions to the general
rule -- that a client is bound by the mistakes of his counsel -- which this Court finds acceptable
are when the reckless or gross negligence of counsel deprives the client of due process of law, or
when the application of the rule results in the outright deprivation of one’s property through a
technicality.[32] These exceptions are not attendant in this case.

The Court cannot sustain petitioner’s averment that his counsel was grossly negligent in
deciding against the presentation of the Affidavit of 7 February 2007 during the proceedings
before the COMELEC. Mistakes of attorneys as to the competency of a witness; the sufficiency,
relevancy or irrelevancy of certain evidence; the proper defense or the burden of proof, failure to
introduce evidence, to summon witnesses and to argue the case -- unless they prejudice the
client and prevent him from properly presenting his case -- do not constitute gross
incompetence or negligence, such that clients may no longer be bound by the acts of their
counsel.[33]

Also belying petitioner’s claim that his former counsel was grossly negligent was the fact
that petitioner continuously used his former counsel’s theory of the case. Even when the
COMELEC already rendered an adverse decision, he persistently argues even to this Court that
his oaths of allegiance to the Republic of the Philippines before the Los Angeles PCG and in his

468
Certificate of Candidacy amount to the renunciation of foreign citizenship which the law
requires. Having asserted the same defense in the instant Petition, petitioner only demonstrates
his continued reliance on and complete belief in the position taken by his former counsel, despite
the former’s incongruous allegations that the latter has been grossly negligent.

Petitioner himself is also guilty of negligence. If indeed he believed that his counsel was
inept, petitioner should have promptly taken action, such as discharging his counsel earlier
and/or insisting on the submission of his Affidavit of 7 February 2007 to the COMELEC, instead
of waiting until a decision was rendered disqualifying him and a resolution issued dismissing his
motion for reconsideration; and, thereupon, he could have heaped the blame on his former
counsel. Petitioner could not be so easily allowed to escape the consequences of his former
counsel’s acts, because, otherwise, it would render court proceedings indefinite, tentative, and
subject to reopening at any time by the mere subterfuge of replacing counsel. [34]

Petitioner cites De Guzman v. Sandiganbayan,[35] where therein petitioner De Guzman was


unable to present a piece of evidence because his lawyer proceeded to file a demurrer to
evidence, despite the Sandiganbayan’s denial of his prior leave to do so. The wrongful
insistence of the lawyer in filing a demurrer to evidence had totally deprived De Guzman of any
chance to present documentary evidence in his defense. This was certainly not the case in the
Petition at bar.

Herein, petitioner was in no way deprived of due process. His counsel actively defended
his suit by attending the hearings, filing the pleadings, and presenting evidence on petitioner’s
behalf. Moreover, petitioner’s cause was not defeated by a mere technicality, but because of a
mistaken reliance on a doctrine which is not applicable to his case. A case lost due to an
untenable legal position does not justify a deviation from the rule that clients are bound by the
acts and mistakes of their counsel.[36]

Petitioner also makes much of the fact that he received the highest number of votes for
the position of Vice-Mayor ofCatarman during the 2007 local elections. The fact that a
candidate, who must comply with the election requirements applicable to dual citizens and failed
to do so, received the highest number of votes for an elective position does not dispense with, or
amount to a waiver of, such requirement.[37] The will of the people as expressed through the
ballot cannot cure the vice of ineligibility, especially if they mistakenly believed that the
candidate was qualified. The rules on citizenship qualifications of a candidate must be strictly
applied. If a person seeks to serve the Republic of the Philippines, he must owe his loyalty to

469
this country only, abjuring and renouncing all fealty and fidelity to any other state.[38] The
application of the constitutional and statutory provisions on disqualification is not a matter of
popularity.[39]

WHEREFORE, the instant appeal is DISMISSED. The Resolution dated 28 September


2007 of the COMELEC en banc in SPA No. 07-361, affirming the Resolution dated 12 June
2007 of the COMELEC Second Division, is AFFIRMED. Petitioner is DISQUALIFIED to run for the
position of Vice-Mayor of Catarman, Camiguin in the 14 May 2007 National and Local Elections,
and if proclaimed, cannot assume the Office of Vice-Mayor of said municipality by virtue of such
disqualification. Costs against petitioner.

SO ORDERED.

MINITA V. CHICO-NAZARIO
Associate Justice
WE CONCUR :

REYNATO S. PUNO
Chief Justice

LEONARDO A. QUISUMBING CONSUELO YNARES-SANTIAGO


Associate Justice Associate Justice

ANTONIO T. CARPIO MA. ALICIA AUSTRIA-MARTINEZ


Associate Justice Associate Justice

470
RENATO C. CORONA CONCHITA CARPIO MORALES
Associate Justice Associate Justice

ADOLFO S. AZCUNA DANTE O. TINGA


Associate Justice Associate Justice

PRESBITERO J. VELASCO, JR. ANTONIO EDUARDO B. NACHURA


Associate Justice Associate Justice

ON OFFICIAL LEAVE
RUBEN T. REYES TERESITA J. LEONARDO-DE CASTRO
Associate Justice Associate Justice

ON LEAVE
ARTURO D. BRION
Associate Justice

CERTIFICATION

Pursuant to Article VIII, Section 13 of the Constitution, it is hereby certified that the
conclusions in the above Decision were reached in consultation before the case was assigned to
the writer of the opinion of the Court.

REYNATO S. PUNO
Chief Justice
471
* On official leave.
** On leave.
[1]
Per Curiam, with Chairman Benjamin S. Abalos, Sr.,
Commissioners Resurreccion Z. Borra, Florentino A. Tuason, Jr., Romeo A. Brawner, Rene
V. Sarmiento, andNicodemo T. Ferrer. Rollo, pp. 36-39.
[2]
Penned by Presiding Commissioner Florentino A. Tuason, Jr with Commissioners
Rene V. Sarmiento and Nicodemo T. Ferrer, concurring; Rollo, pp. 31-35.
[3]
Id. at 9.
[4]
Id. at 94.
[5]
Id. at 95.
[6]
Id. at 50.
[7]
Id. at 59.
[8]
Id. at 40-42.
[9]
Id. at 46-49.
[10]
Id. at 61-65.
[11]
Id. at 31-35.
[12]
392 Phil. 327 (2000).
[13]
367 Phil. 132 (1999).
[14]
Rollo, p. 35.
[15]
Id. at 74.
[16]
Id. at 36-39.
[17]
Id. at 96.
[18]
Id. at 11-13.
[19]
Id. at 188.
[20]
Lopez v. Commission on Elections, G.R. No. 182701, 23 July 2008.
[21]
Supra note 12 at 340.
[22]
Supra note 13 at 152-153.
[23]
Even if Republic Act No. 9225 had not been enacted, petitioner would still
not be able to rely on Valles and Mercado. The ruling in those cases was that when a
person who was merely a dual citizen, not a person with dual allegiance, files a certificate
of candidacy, this already constitutes as a renunciation of foreign citizenship. In these
cases, this Court made an important distinction between “dual citizenship” and “dual
allegiance.” Dual citizenship is the result of the application of the different laws of two
states, whereby a person is simultaneously considered a national by the said states. Dual
allegiance, on the other hand, arises when a person simultaneously owes her loyalty to
two or more states by undertaking a positive act. While dual citizenship is involuntary,
dual allegiance is the result of an individual’s volition. Thus, Article IV, Section 5 of the
Constitution provides that: “Dual allegiance of citizens is inimical to national interest and
shall be dealt with by law.” In both Valles and Mercado, the candidates whose
qualifications are being challenged were dual citizens: They became citizens of another
state without performing another act—both candidates, who have Filipino parents, became
citizens of the foreign state where they were born under the principal of jus soli and had
not taken an oath of allegiance to said foreign state. In contrast, herein petitioner has

472
dual allegiance since he acquired his US citizenship through the positive and voluntary act
of swearing allegiance to the US.
Other factual considerations need to be pointed out. It is significant to note that
in Valles, therein private respondent Lopez executed a Declaration of Renunciation of
Australian Citizenship which, consequently, led to the cancellation of her Australian
passport, even before she filed her Certificate of Candidacy. The issue in that case was
Lopez’s reacquisition of her citizenship, not her failure to renounce her foreign citizenship.
(Valles v. Commission on Elections, supra note 12 at 340-341.)
In Mercado, the Court took special notice of the fact that “private respondent’s oath
of allegiance to the Philippines, when considered with the fact that he has spent his youth
and adulthood, received his education, practiced his profession as an artist, and taken
part in past elections in this country, leaves no doubt of his election of Philippine
citizenship.” (Mercado v. Manzano, supra note 13 at 153.)
Herein petitioner’s situation is markedly different since he actively elected to
acquire a foreign citizenship and re-acquired his Filipino citizenship only a year before he
filed his candidacy for a local elective position.
[24]
Supra note 20.
[25]
Rollo, p. 96.
[26]
Tan and Commission on Elections, G.R. Nos. 166143-47 and 166891, 20 November
2006, 507 SCRA 352, 373-374; Vda de Gualberto v. Go, G.R. No. 139843, 21 July 2005,
463 SCRA 671, 678; Del Rosario v. Bonga, 402 Phil. 949, 957-958 (2001).
[27]
Villanueva v. Court of Appeals, G.R. No. 143286, 14 April 2004, 427 SCRA 439, 448.
[28]
Filipinas Systems, Inc. v. National Labor Relations Commission, 463 Phil. 813, 819
(2003)
[29]
Manongsong v. Estimo, 452 Phil. 862, 879-880 (2003).
[30]
Cansino v. Court of Appeals, 456 Phil. 686, 693 (2003).
[31]
People v. Kawasa, 327 Phil. 928, 933 (1996).
[32]
R Transport Corporation v. Philippine Hawk Transport Corporation, G.R. No. 155737,
19 October 2005, 473 SCRA 342, 347-348; Trust International Paper Corporation
v. Pelaez, G.R. No. 164871, 22 August 2006, 499 SCRA 552, 563.
[33]
Andrada v. People, G.R. No. 135222, 4 March 2005, 452 SCRA 685, 693-
694; Custodio v. Sandiganbayan, G.R. Nos. 96027-28, 8 March 2005, 453 SCRA 24,
45; People v. Mercado, 445 Phil. 813, 829 (2003); Tesoro v. Court of Appeals, 153 Phil.
580, 588-589 (1973); United States v. Umali, 15 Phil. 33, 35 (1910).
[34]
People v. Kawasa, supra note 31 at 934-935.
[35]
326 Phil. 184 (1996).
[36]
Espinosa v. Court of Appeals, G.R. No.128686, 28 May 2004, 430 SCRA 96, 105-
106.
[37]
Labo, Jr. v. Commission on Elections, G.R. Nos. 105111 and 105384, 3 July 1992,
211 SCRA 297, 308.
[38]
Frivaldo v. Commission on Elections, G.R. No. 87193, 23 June 1989, 174 SCRA 245,
255.
[39]
Lopez v. Commission on Elections, supra note 20.

473
EN BANC

SUZETTE NICOLAS y SOMBILON, G.R. No. 175888


Petitioner,

Present:

PUNO, C.J.,
QUISUMBING,
YNARES-SANTIAGO,
CARPIO,
AUSTRIA-MARTINEZ,
- versus - CORONA,
CARPIO MORALES,
AZCUNA,
TINGA,
CHICO-NAZARIO,
VELASCO, JR.,
NACHURA,
LEONARDO-DE CASTRO,
BRION, and
PERALTA, JJ.
A
B
E
R
T
O

R
O
M
U
L
O
,
Respondents.

X - - - - - - - - - - - - - - - - - - - - - - - - - - - - - - - - - - -X

JOVITO R. SALONGA, WIGBERTO G.R. No. 176051


E. TAÑADA, JOSE DE LA RAMA,
474
EMILIO C. CAPULONG, H. HARRY
L. ROQUE, JR., FLORIN HILBAY,
and BENJAMIN POZON,
Petitioners,

- versus -

DANIEL SMITH, SECRETARY RAUL GONZALEZ, PRESIDENTIAL


LEGAL COUNSEL SERGIO APOSTOL, SECRETARY RONALDO
PUNO, SECRETARY ALBERTO ROMULO, The Special
th
16 Division of the COURT OF APPEALS, and all persons acting
in their capacity,
Respondents.

X - - - - - - - - - - - - - - - - - - - - - - - - - - - - - - - - - - -X

Promulgated:

February 11, 2009

X ---------------------------------------------------------------------------------------- X

DECISION

AZCUNA, J.:

These are petitions for certiorari, etc. as special civil actions and/or for review of the
Decision of the Court of Appeals inLance Corporal Daniel J. Smith v. Hon. Benjamin T. Pozon, et
al., in CA-G.R. SP No. 97212, dated January 2, 2007.

The facts are not disputed.

Respondent Lance Corporal (L/CPL) Daniel Smith is a member of the United States Armed
Forces. He was charged with the crime of rape committed against a Filipina, petitioner herein,
sometime on November 1, 2005, as follows:

The undersigned accused LCpl. Daniel Smith, Ssgt. Chad Brian Carpentier,
Dominic Duplantis, Keith Silkwood and Timoteo L. Soriano, Jr. of the crime of Rape
under Article 266-A of the Revised Penal Code, as amended by Republic Act 8353,
upon a complaint under oath filed by Suzette S. Nicolas, which is attached hereto
and made an integral part hereof as Annex “A,” committed as follows:

475
“That on or about the First (1st) day of November 2005, inside
the Subic Bay Freeport Zone, Olongapo City and within the jurisdiction
of this Honorable Court, the above-named accused’s (sic), being then
members of the United States Marine Corps, except Timoteo L.
Soriano, Jr., conspiring, confederating together and mutually helping
one another, with lewd design and by means of force, threat and
intimidation, with abuse of superior strength and taking advantage of
the intoxication of the victim, did then and there willfully, unlawfully
and feloniously sexually abuse and have sexual intercourse with or
carnal knowledge of one Suzette S. Nicolas, a 22-year old unmarried
woman inside a Starex Van with Plate No. WKF-162, owned by
Starways Travel and Tours, with Office address at 8900 P. Victor St.,
Guadalupe, Makati City, and driven by accused Timoteo L. Soriano, Jr.,
against the will and consent of the said Suzette S. Nicolas, to her
damage and prejudice.

CONTRARY TO LAW.”[1]

Pursuant to the Visiting Forces Agreement (VFA) between the Republic of


the Philippines and the United States, entered into on February 10, 1998, the United States, at
its request, was granted custody of defendant Smith pending the proceedings.

During the trial, which was transferred from the Regional Trial Court (RTC) of Zambales to
the RTC of Makati for security reasons, the United States Government faithfully complied with its
undertaking to bring defendant Smith to the trial court every time his presence was required.

On December 4, 2006, the RTC of Makati, following the end of the trial, rendered its
Decision, finding defendant Smith guilty, thus:

WHEREFORE, premises considered, for failure of the prosecution to adduce


sufficient evidence against accused S/SGT. CHAD BRIAN CARPENTER, L/CPL. KEITH
SILKWOOD AND L/CPL. DOMINIC DUPLANTIS, all of the US Marine Corps assigned
at the USS Essex, are hereby ACQUITTED to the crime charged.

The prosecution having presented sufficient evidence against accused L/CPL.


DANIEL J. SMITH, also of the US Marine Corps at the USS Essex, this Court hereby
finds him GUILTY BEYOND REASONABLE DOUBT of the crime of RAPE defined
under Article 266-A, paragraph 1 (a) of the Revised Penal Code, as amended by
R.A. 8353, and, in accordance with Article 266-B, first paragraph thereof, hereby
sentences him to suffer the penalty of reclusion perpetua together with the
accessory penalties provided for under Article 41 of the same Code.

476
Pursuant to Article V, paragraph No. 10, of the Visiting Forces Agreement
entered into by the Philippines and the United States, accused L/CPL. DANIEL J.
SMITH shall serve his sentence in the facilities that shall, thereafter, be agreed
upon by appropriate Philippine and United States authorities. Pending agreement
on such facilities, accused L/CPL. DANIEL J. SMITH is hereby temporarily committed
to the Makati City Jail.

Accused L/CPL. DANIEL J. SMITH is further sentenced to indemnify


complainant SUZETTE S. NICOLAS in the amount ofP50,000.00 as compensatory
damages plus P50,000.00 as moral damages.

SO ORDERED.[2]

As a result, the Makati court ordered Smith detained at the Makati jail until further orders.

On December 29, 2006, however, defendant Smith was taken out of the Makati jail by a
contingent of Philippine law enforcement agents, purportedly acting under orders of the
Department of the Interior and Local Government, and brought to a facility for detention under
the control of the United States government, provided for under new agreements between the
Philippines and the United States, referred to as the Romulo-Kenney Agreement of December
19, 2006 which states:

The Government of the Republic of the Philippines and the Government of


the United States of America agree that, in accordance with the Visiting Forces
Agreement signed between our two nations, Lance Corporal Daniel J. Smith, United
States Marine Corps, be returned to U.S. military custody at the U.S. Embassy in
Manila.

(Sgd.) KRISTIE A. KENNEY (Sgd.) ALBERTO G. ROMULO


Representative of the United States Representative of the Republic
of America of the Philippines

DATE: 12-19-06 DATE: December 19, 2006__

and the Romulo-Kenney Agreement of December 22, 2006 which states:

The Department of Foreign Affairs of the Republic of the Philippines and the
Embassy of the United States of America agree that, in accordance with the Visiting
Forces Agreement signed between the two nations, upon transfer of Lance Corporal
Daniel J. Smith, United States Marine Corps, from the Makati City Jail, he will be
detained at the first floor, Rowe (JUSMAG) Building, U.S. Embassy Compound in a
room of approximately 10 x 12 square feet. He will be guarded round the clock
by U.S. military personnel. The Philippine police and jail authorities, under the

477
direct supervision of the Philippine Department of Interior and Local Government
(DILG) will have access to the place of detention to ensure the United States is in
compliance with the terms of the VFA.

The matter was brought before the Court of Appeals which decided on January 2, 2007, as
follows:

WHEREFORE, all the foregoing considered, we resolved to DISMISS the


petition for having become moot.[3]

Hence, the present actions.

The petitions were heard on oral arguments on September 19, 2008, after which the
parties submitted their memoranda.

Petitioners contend that the Philippines should have custody of defendant L/CPL Smith
because, first of all, the VFA is void and unconstitutional.

This issue had been raised before, and this Court resolved in favor of the constitutionality
of the VFA. This was in Bayan v. Zamora,[4] brought by Bayan, one of petitioners in the present
cases.

Against the barriers of res judicata vis-à-vis Bayan, and stare decisis vis-à-vis all the
parties, the reversal of the previous ruling is sought on the ground that the issue is of primordial
importance, involving the sovereignty of the Republic, as well as a specific mandate of the
Constitution.

The provision of the Constitution is Art. XVIII, Sec. 25 which states:

Sec. 25. After the expiration in 1991 of the Agreement between the
Philippines and the United States of America concerning Military Bases, foreign
military bases, troops, or facilities shall not be allowed in the Philippines except
under a treaty duly concurred in by the Senate and, when the Congress so requires,
ratified by a majority of the votes cast by the people in a national referendum held
for that purpose, and recognized as a treaty by the other contracting State.

The reason for this provision lies in history and the Philippine experience in regard to
the United States military bases in the country.

478
It will be recalled that under the Philippine Bill of 1902, which laid the basis for the
Philippine Commonwealth and, eventually, for the recognition of independence, the United
States agreed to cede to the Philippines all the territory it acquired from Spain under the Treaty
of Paris, plus a few islands later added to its realm, except certain naval ports and/or military
bases and facilities, which the United States retained for itself.

This is noteworthy, because what this means is that Clark and Subic and the other places
in the Philippines covered by the RP-US Military Bases Agreement of 1947 were not Philippine
territory, as they were excluded from the cession and retained by the US.

Accordingly, the Philippines had no jurisdiction over these bases except to the extent
allowed by the United States. Furthermore, the RP-US Military Bases Agreement was never
advised for ratification by the United States Senate, a disparity in treatment, because
the Philippines regarded it as a treaty and had it concurred in by our Senate.

Subsequently, the United States agreed to turn over these bases to the Philippines; and
with the expiration of the RP-US Military Bases Agreement in 1991, the territory covered by
these bases were finally ceded to the Philippines.

To prevent a recurrence of this experience, the provision in question was adopted in the
1987 Constitution.

The provision is thus designed to ensure that any agreement allowing the presence of
foreign military bases, troops or facilities in Philippine territory shall be equally binding on
the Philippines and the foreign sovereign State involved. The idea is to prevent a recurrence of
the situation in which the terms and conditions governing the presence of foreign armed forces
in our territory were binding upon us but not upon the foreign State.

Applying the provision to the situation involved in these cases, the question is whether or
not the presence of US Armed Forces in Philippine territory pursuant to the VFA is allowed
“under a treaty duly concurred in by the Senate xxx and recognized as a treaty by the other
contracting State.”

This Court finds that it is, for two reasons.

479
First, as held in Bayan v. Zamora,[5] the VFA was duly concurred in by the Philippine
Senate and has been recognized as a treaty by the United States as attested and certified by the
duly authorized representative of the United States government.

The fact that the VFA was not submitted for advice and consent of the United States
Senate does not detract from its status as a binding international agreement or treaty
recognized by the said State. For this is a matter of internal United Stateslaw. Notice can be
taken of the internationally known practice by the United States of submitting to its Senate for
advice and consent agreements that are policymaking in nature, whereas those that carry out or
further implement these policymaking agreements are merely submitted to Congress, under the
provisions of the so-called Case–Zablocki Act, within sixty days from ratification.[6]

The second reason has to do with the relation between the VFA and the RP-US Mutual
Defense Treaty of August 30, 1951. This earlier agreement was signed and duly ratified with the
concurrence of both the Philippine Senate and the United States Senate.

The RP-US Mutual Defense Treaty states:[7]

MUTUAL DEFENSE TREATY BETWEEN THE REPUBLIC OF THE PHILIPPINES AND


THE UNITED STATES OF AMERICA. Signed at Washington, August 30, 1951.

The Parties of this Treaty

Reaffirming their faith in the purposes and principles of the Charter of the
United Nations and their desire to live in peace with all peoples and all
governments, and desiring to strengthen the fabric of peace in the Pacific area.

Recalling with mutual pride the historic relationship which brought their two
peoples together in a common bond of sympathy and mutual ideals to fight side-by-
side against imperialist aggression during the last war.

Desiring to declare publicly and formally their sense of unity and their
common determination to defend themselves against external armed attack, so
that no potential aggressor could be under the illusion that either of them stands
alone in the Pacific area.

Desiring further to strengthen their present efforts for collective defense for
the preservation of peace and security pending the development of a more
comprehensive system of regional security in the Pacific area.

Agreeing that nothing in this present instrument shall be considered or


interpreted as in any way or sense altering or diminishing any existing agreements

480
or understandings between the Republic of the Philippines and the United States of
America.

Have agreed as follows:

ARTICLE I. The parties undertake, as set forth in the Charter of the United
Nations, to settle any international disputes in which they may be involved by
peaceful means in such a manner that international peace and security and justice
are not endangered and to refrain in their international relation from the threat or
use of force in any manner inconsistent with the purposes of the United Nations.

ARTICLE II. In order more effectively to achieve the objective of this


Treaty, the Parties separately and jointly by self-help and mutual aid will maintain
and develop their individual and collective capacity to resist armed attack.

ARTICLE III. The Parties, through their Foreign Ministers or their deputies,
will consult together from time to time regarding the implementation of this Treaty
and whenever in the opinion of either of them the territorial integrity, political
independence or security of either of the Parties is threatened by external armed
attack in the Pacific.

ARTICLE IV. Each Party recognizes that an armed attack in the Pacific area
on either of the parties would be dangerous to its own peace and safety and
declares that it would act to meet the common dangers in accordance with its
constitutional processes.

Any such armed attack and all measures taken as a result thereof shall be
immediately reported to the Security Council of the United Nations. Such measures
shall be terminated when the Security Council has taken the measures necessary to
restore and maintain international peace and security.

ARTICLE V. For the purpose of Article IV, an armed attack on either of the
Parties is deemed to include an armed attack on the metropolitan territory of either
of the Parties, or on the island territories under its jurisdiction in the Pacific Ocean,
its armed forces, public vessels or aircraft in the Pacific.

ARTICLE VI. This Treaty does not affect and shall not be interpreted as
affecting in any way the rights and obligations of the Parties under the Charter of
the United Nations or the responsibility of the United Nations for the maintenance of
international peace and security.

ARTICLE VII. This Treaty shall be ratified by the Republic of


the Philippines and the United Nations of America in accordance with their
respective constitutional processes and will come into force when instruments of
ratification thereof have been exchanged by them atManila.

ARTICLE VIII. This Treaty shall remain in force indefinitely. Either Party
may terminate it one year after notice has been given to the other party.

481
IN WITHNESS WHEREOF the undersigned Plenipotentiaries have signed this
Treaty.

DONE in duplicate at Washington this thirtieth day of August, 1951.

For the Republic of the Philippines:


(Sgd.) CARLOS P. ROMULO
(Sgd.) JOAQUIN M. ELIZALDE
(Sgd.) VICENTE J. FRANCISCO
(Sgd.) DIOSDADO MACAPAGAL

For the United States of America:

(Sgd.) DEAN ACHESON


(Sgd.) JOHN FOSTER DULLES
(Sgd.) TOM CONNALLY
(Sgd.) ALEXANDER WILEY[8]

Clearly, therefore, joint RP-US military exercises for the purpose of developing the
capability to resist an armed attack fall squarely under the provisions of the RP-US Mutual
Defense Treaty. The VFA, which is the instrument agreed upon to provide for the joint RP-US
military exercises, is simply an implementing agreement to the main RP-US Military Defense
Treaty. The Preamble of the VFA states:

The Government of the United States of America and the Government of the
Republic of the Philippines,

Reaffirming their faith in the purposes and principles of the Charter of the United
Nations and their desire to strengthen international and regional security in the
Pacific area;

Reaffirming their obligations under the Mutual Defense Treaty of August 30, 1951;

Noting that from time to time elements of the United States armed forces may visit
the Republic of the Philippines;

Considering that cooperation between the United States and the Republic of
the Philippines promotes their common security interests;

Recognizing the desirability of defining the treatment of United States personnel


visiting the Republic of the Philippines;

Have agreed as follows:[9]

482
Accordingly, as an implementing agreement of the RP-US Mutual Defense Treaty, it was
not necessary to submit the VFA to the US Senate for advice and consent, but merely to the US
Congress under the Case–Zablocki Act within 60 days of its ratification. It is for this reason that
the US has certified that it recognizes the VFA as a binding international agreement, i.e., a
treaty, and this substantially complies with the requirements of Art. XVIII, Sec. 25 of our
Constitution.[10]

The provision of Art. XVIII, Sec. 25 of the Constitution, is complied with by virtue of the
fact that the presence of the US Armed Forces through the VFA is a presence “allowed under”
the RP-US Mutual Defense Treaty. Since the RP-US Mutual Defense Treaty itself has been
ratified and concurred in by both the Philippine Senate and the US Senate, there is no violation
of the Constitutional provision resulting from such presence.

The VFA being a valid and binding agreement, the parties are required as a matter of
international law to abide by its terms and provisions.

The VFA provides that in cases of offenses committed by the members of the US Armed
Forces in the Philippines, the following rules apply:

Article V
Criminal Jurisdiction

xxx
6. The custody of any United States personnel over whom the Philippines is
to exercise jurisdiction shall immediately reside with United States military
authorities, if they so request, from the commission of the offense until completion
of all judicial proceedings. United Statesmilitary authorities shall, upon formal
notification by the Philippine authorities and without delay, make such personnel
available to those authorities in time for any investigative or judicial proceedings
relating to the offense with which the person has been charged. In extraordinary
cases, the Philippine Government shall present its position to the United States
Government regarding custody, which the United States Government shall take into
full account. In the event Philippine judicial proceedings are not completed within
one year, the United States shall be relieved of any obligations under this
paragraph. The one year period will not include the time necessary to
appeal. Also, the one year period will not include any time during which scheduled
trial procedures are delayed because United States authorities, after timely
notification by Philippine authorities to arrange for the presence of the accused, fail
to do so.

483
Petitioners contend that these undertakings violate another provision of the Constitution,
namely, that providing for the exclusive power of this Court to adopt rules of procedure for all
courts in the Philippines (Art. VIII, Sec. 5[5]). They argue that to allow the transfer of custody
of an accused to a foreign power is to provide for a different rule of procedure for that accused,
which also violates the equal protection clause of the Constitution (Art. III, Sec. 1.).

Again, this Court finds no violation of the Constitution.

The equal protection clause is not violated, because there is a substantial basis for a
different treatment of a member of a foreign military armed forces allowed to enter our territory
and all other accused.[11]

The rule in international law is that a foreign armed forces allowed to enter one’s territory
is immune from local jurisdiction, except to the extent agreed upon. The Status of Forces
Agreements involving foreign military units around the world vary in terms and conditions,
according to the situation of the parties involved, and reflect their bargaining power. But the
principle remains, i.e., the receiving State can exercise jurisdiction over the forces of the
sending State only to the extent agreed upon by the parties.[12]

As a result, the situation involved is not one in which the power of this Court to adopt
rules of procedure is curtailed or violated, but rather one in which, as is normally encountered
around the world, the laws (including rules of procedure) of one State do not extend or apply –
except to the extent agreed upon – to subjects of another State due to the recognition of
extraterritorial immunity given to such bodies as visiting foreign armed forces.

Nothing in the Constitution prohibits such agreements recognizing immunity from


jurisdiction or some aspects of jurisdiction (such as custody), in relation to long-recognized
subjects of such immunity like Heads of State, diplomats and members of the armed forces
contingents of a foreign State allowed to enter another State’s territory. On the contrary, the
Constitution states that the Philippines adopts the generally accepted principles of international
law as part of the law of the land. (Art. II, Sec. 2).

Applying, however, the provisions of VFA, the Court finds that there is a different
treatment when it comes to detention as against custody. The moment the accused has to be
detained, e.g., after conviction, the rule that governs is the following provision of the VFA:

Article V
484
Criminal Jurisdiction

xxx
Sec. 10. The confinement or detention by Philippine authorities of United
States personnel shall be carried out in facilities agreed on by
appropriate Philippines and United States authorities. United States personnel
serving sentences in the Philippines shall have the right to visits and material
assistance.

It is clear that the parties to the VFA recognized the difference between custody during
the trial and detention after conviction, because they provided for a specific arrangement to
cover detention. And this specific arrangement clearly states not only that the detention shall be
carried out in facilities agreed on by authorities of both parties, but also that the detention shall
be “by Philippine authorities.” Therefore, the Romulo-Kenney Agreements of December 19 and
22, 2006, which are agreements on the detention of the accused in the United States Embassy,
are not in accord with the VFA itself because such detention is not “by Philippine authorities.”

Respondents should therefore comply with the VFA and negotiate with representatives of
the United States towards an agreement on detention facilities under Philippine authorities as
mandated by Art. V, Sec. 10 of the VFA.

Next, the Court addresses the recent decision of the United States Supreme Court
in Medellin v. Texas ( 552 US ___ No. 06-984, March 25, 2008), which held that treaties entered
into by the United States are not automatically part of their domestic law unless these treaties
are self-executing or there is an implementing legislation to make them enforceable.

On February 3, 2009, the Court issued a Resolution, thus:

“G.R. No. 175888 (Suzette Nicolas y Sombilon v. Alberto Romulo, et al.); G.R. No.
176051 (Jovito R. Salonga, et al. v. Daniel Smith, et al.); and G.R. No.
176222 (Bagong Alyansang Makabayan [BAYAN], et al. v. President Gloria
Macapagal-Arroyo, et al.).

The parties, including the Solicitor General, are required to submit within
three (3) days a Comment/Manifestation on the following points:

1. What is the implication on the RP-US Visiting Forces Agreement of


the recent US Supreme Court decision in Jose Ernesto Medellin v. Texas,
dated March 25, 2008, to the effect that treaty stipulations that are not self-
executory can only be enforced pursuant to legislation to carry them into
effect; and that, while treaties may comprise international commitments,
they are not domestic law unless Congress has enacted implementing
485
statutes or the treaty itself conveys an intention that it be “self-executory”
and is ratified on these terms?

2. Whether the VFA is enforceable in the US as domestic law, either


because it is self-executory or because there exists legislation to implement
it.

3. Whether the RP-US Mutual Defense Treaty of August 30, 1951 was
concurred in by the US Senate and, if so, is there proof of the US Senate
advice and consent resolution? Peralta, J., no part.”

After deliberation, the Court holds, on these points, as follows:

First, the VFA is a self-executing Agreement, as that term is defined in Medellin itself,
because the parties intend its provisions to be enforceable, precisely because the Agreement is
intended to carry out obligations and undertakings under the RP-US Mutual Defense Treaty. As
a matter of fact, the VFA has been implemented and executed, with the US faithfully complying
with its obligation to produce L/CPL Smith before the court during the trial.

Secondly, the VFA is covered by implementing legislation, namely, the Case-Zablocki Act,
USC Sec. 112(b), inasmuch as it is the very purpose and intent of the US Congress that
executive agreements registered under this Act within 60 days from their ratification be
immediately implemented. The parties to these present cases do not question the fact that the
VFA has been registered under the Case-Zablocki Act.

In sum, therefore, the VFA differs from the Vienna Convention on Consular Relations and
the Avena decision of the International Court of Justice (ICJ), subject matter of
the Medellin decision. The Convention and the ICJ decision are not self-executing and are not
registrable under the Case-Zablocki Act, and thus lack legislative implementing authority.

Finally, the RP-US Mutual Defense Treaty was advised and consented to by the US Senate
on March 20, 1952, as reflected in the US Congressional Record, 82nd Congress, Second Session,
Vol. 98 – Part 2, pp. 2594-2595.

The framers of the Constitution were aware that the application of international law in
domestic courts varies from country to country.

486
As Ward N. Ferdinandusse states in his Treatise, DIRECT APPLICATION OF
INTERNATIONAL CRIMINAL LAW IN NATIONAL COURTS, some countries require legislation
whereas others do not.

It was not the intention of the framers of the 1987 Constitution, in adopting Article XVIII,
Sec. 25, to require the other contracting State to convert their system to achieve alignment and
parity with ours. It was simply required that the treaty be recognized as a treaty by the other
contracting State. With that, it becomes for both parties a binding international obligation and
the enforcement of that obligation is left to the normal recourse and processes under
international law.

Furthermore, as held by the US Supreme Court in Weinberger v. Rossi,[13] an executive


agreement is a “treaty” within the meaning of that word in international law and constitutes
enforceable domestic law vis-à-vis the United States. Thus, the US Supreme Court
in Weinberger enforced the provisions of the executive agreement granting preferential
employment to Filipinos in the US Bases here.

Accordingly, there are three types of treaties in the American system:

1. Art. II, Sec. 2 treaties – These are advised and consented to by the US
Senate in accordance with Art. II, Sec. 2 of the US Constitution.

2. Executive–Congressional Agreements: These are joint agreements of the


President and Congress and need not be submitted to the Senate.

3. Sole Executive Agreements. – These are agreements entered into by the


President. They are to be submitted to Congress within sixty (60) days of
ratification under the provisions of the Case-Zablocki Act, after which they are
recognized by the Congress and may be implemented.

As regards the implementation of the RP-US Mutual Defense Treaty, military aid or
assistance has been given under it and this can only be done through implementing
legislation. The VFA itself is another form of implementation of its provisions.

WHEREFORE, the petitions are PARTLY GRANTED, and the Court of Appeals’ Decision in
CA-G.R. SP No. 97212 dated January 2, 2007 is MODIFIED. The Visiting Forces Agreement
(VFA) between the Republic of the Philippines and the United States, entered into on February
10, 1998, is UPHELD as constitutional, but the Romulo-Kenney Agreements of December 19 and
487
22, 2006 are DECLARED not in accordance with the VFA, and respondent Secretary of Foreign
Affairs is hereby ordered to forthwith negotiate with the United States representatives for the
appropriate agreement on detention facilities under Philippine authorities as provided in Art. V,
Sec. 10 of the VFA, pending which the status quo shall be maintained until further orders by this
Court.

The Court of Appeals is hereby directed to resolve without delay the related matters
pending therein, namely, the petition for contempt and the appeal of L/CPL Daniel Smith from
the judgment of conviction.

No costs.

SO ORDERED.

ADOLFO S. AZCUNA
Associate Justice

WE CONCUR:

REYNATO S. PUNO
Chief Justice

LEONARDO A. QUISUMBING CONSUELO YNARES-SANTIAGO


Associate Justice Associate Justice

ANTONIO T. CARPIO MA. ALICIA AUSTRIA-MARTINEZ


Associate Justice Associate Justice

488
RENATO C. CORONA CONCHITA CARPIO MORALES
Associate Justice Associate Justice

DANTE O. TINGA MINITA V. CHICO-NAZARIO


Associate Justice Associate Justice

PRESBITERO J. VELASCO, JR. ANTONIO EDUARDO B. NACHURA


Associate Justice Associate Justice

TERESITA J. LEONARDO-DE CASTRO ARTURO D. BRION


Associate Justice Associate Justice

DIOSDADO M. PERALTA
Associate Justice

CERTIFICATION

Pursuant to Section 13, Article VIII of the Constitution, it is hereby certified that the
conclusions in the above Decision were reached in consultation before the case was assigned to
the writer of the opinion of the Court.

REYNATO S. PUNO
Chief Justice

489
[1]
Annex “B” of RTC Decision, CA rollo, p. 45.
[2]
Annex “B” of CA rollo, pp. 36-96.
[3]
Rollo, pp. 90-127.
[4]
G.R. No. 138570, October 10, 2000, 342 SCRA 449.
[5]
Supra, note 4.
[6]
The Case-Zablocki Act, 1 U.S.C. 112b (a) (1976 ed., Supp IV). See also Weinberger
v. Rossi, 456 U.S. 25 (1982), in which the U.S. Supreme Court sustained recognition as a
“treaty” of agreements not concurred in by the U.S. Senate.
[7]
The RP-US Mutual Defense Treaty was signed in Washington, D.C. on August 30,
1951. Its ratification was advised by the US Senate on March 20, 1952, and
the USPresident ratified the Treaty on April 15, 1952.
The Treaty was concurred in by the RP Senate, S.R. No. 84, May 12, 1952. The
Philippine instrument of ratification was signed by the RP President on August 27,
1952. The Agreement entered into force on August 27, 1952 upon the exchange of
ratification between the Parties.
This Agreement is published in II DFA TS No. 1, p. 13; 177 UNTS, p. 133; 3 UST
3847-3952. The RP Presidential proclamation of the Agreement, Proc. No. 341, S. 1952,
is published in 48 O.G. 4224 (Aug. 1952).
[8]
Emphasis supplied.
[9]
Emphasis supplied.
[10]
See Letter of Ambassador Thomas C. Hubbard quoted in Bayan, 342 SCRA 449, 491.
[11]
See, the summation of the rule on equal protection in ISAGANI A. CRUZ,
CONSTITUTIONAL LAW, pp. 123-139 (2007), and the authorities cited therein.
[12]
See Dieter Fleck, Ed., The HANDBOOK OF THE LAW OF VISITING FORCES , Oxford:
2001.
[13]
Supra, Note 6.

490
EN BANC

ATTY. OLIVER O. LOZANO G.R. No. 187883


and ATTY. EVANGELINE J.
LOZANO-
ENDRIANO, Petitioners,

- versus -

SPEAKER PROSPERO C.
NOGRALES, Representative,
Majority, House of Representatives,
Respondent.
x----------------------x

LOUIS “BAROK” C. BIRAOGO, G.R. No. 187910


Petitioner,
Present:

- versus - PUNO, C.J.,


QUISUMBING,
YNARES-SANTIAGO,
SPEAKER PROSPERO C. CARPIO,
NOGRALES, Speaker of the CORONA,
House of Representatives, CARPIO MORALES*,
Congress of the Philippines, CHICO-NAZARIO,
Respondent. VELASCO, JR.,
NACHURA,
LEONARDO-DE CASTRO,
BRION,
PERALTA, and
BERSAMIN, JJ.

Promulgated:

June 16, 2009

x- - - - - - - - - - - - - - - - - - - - - - - - - - - - - - - - - - - - - - - - - - - - - - - - - - - x

RESOLUTION

PUNO, C.J.:

This Court, so long as the fundamentals of republicanism continue to guide it, shall not
shirk its bounden duty to wield its judicial power to settle "actual controversies involving rights
which are legally demandable and enforceable, and to determine whether or not there has been
491
a grave abuse of discretion amounting to a lack or excess of jurisdiction on the part of any
branch or instrumentality of the government."[1] Be that as it may, no amount of exigency can
make this Court exercise a power where it is not proper.

The two petitions, filed by their respective petitioners in their capacities as concerned
citizens and taxpayers, prayed for the nullification of House Resolution No. 1109 entitled “A
Resolution Calling upon the Members of Congress to Convene for the Purpose of Considering
Proposals to Amend or Revise the Constitution, Upon a Three-fourths Vote of All the Members of
Congress.” In essence, both petitions seek to trigger a justiciable controversy that would
warrant a definitive interpretation by this Court of Section 1, Article XVII, which provides for the
procedure for amending or revising the Constitution. Unfortunately, this Court cannot indulge
petitioners’ supplications. While some may interpret petitioners’ moves as vigilance in
preserving the rule of law, a careful perusal of their petitions would reveal that they cannot
hurdle the bar of justiciability set by this Court before it will assume jurisdiction over cases
involving constitutional disputes.

It is well settled that it is the duty of the judiciary to say what the law is.[2] The
determination of the nature, scope and extent of the powers of government is the exclusive
province of the judiciary, such that any mediation on the part of the latter for the allocation of
constitutional boundaries would amount, not to its supremacy, but to its mere fulfillment of its
“solemn and sacred obligation” under the Constitution.[3] This Court’s power of review may be
awesome, but it is limited to actual cases and controversies dealing with parties having
adversely legal claims, to be exercised after full opportunity of argument by the parties, and
limited further to the constitutional question raised or the very lis mota presented.[4] The “case-
or-controversy” requirement bans this court from deciding “abstract, hypothetical or contingent
questions,”[5] lest the court give opinions in the nature of advice concerning legislative or
executive action.[6] In the illuminating words of the learned Justice Laurel in Angara v. Electoral
Commission[7]:

Any attempt at abstraction could only lead to dialectics and barren legal questions
and to sterile conclusions unrelated to actualities. Narrowed as its function is in this
manner, the judiciary does not pass upon questions of wisdom, justice or
expediency of legislation. More than that, courts accord the
presumption of constitutionality to legislative enactments, not
only because the legislature is presumed to abide by the Constitution but also
because the judiciary in the determination of actual cases and controversies must
reflect the wisdom and justice of the people as expressed through their
representatives in the executive and legislative departments of the government.

An aspect of the “case-or-controversy” requirement is the requisite of “ripeness.” In


the United States, courts are centrally concerned with whether a case involves uncertain

492
contingent future events that may not occur as anticipated, or indeed may not occur at
all.[8] Another approach is the evaluation of the twofold aspect of ripeness: first, the fitness of
the issues for judicial decision; and second, the hardship to the parties entailed by withholding
court consideration.[9] In our jurisdiction, the issue of ripeness is generally treated in terms of
actual injury to the plaintiff. Hence, a question is ripe for adjudication when the act being
challenged has had a direct adverse effect on the individual challenging it.[10] An alternative road
to review similarly taken would be to determine whether an action has already been
accomplished or performed by a branch of government before the courts may step in.[11]

In the present case, the fitness of petitioners’ case for the exercise of judicial review is
grossly lacking. In thefirst place, petitioners have not sufficiently proven any adverse injury or
hardship from the act complained of. In the second place, House Resolution No. 1109 only
resolved that the House of Representatives shall convene at a future time for the purpose of
proposing amendments or revisions to the Constitution. No actual convention has yet transpired
and no rules of procedure have yet been adopted. More importantly, no proposal has yet been
made, and hence, no usurpation of power or gross abuse of discretion has yet taken place. In
short, House Resolution No. 1109 involves a quintessential example of an uncertain contingent
future event that may not occur as anticipated, or indeed may not occur at all. The House has
not yet performed a positive act that would warrant an intervention from this Court.

Tan v. Macapagal presents a similar factual milieu. In said case, petitioners filed a petition
assailing the validity of the Laurel-Langley resolution, which dealt with the range of authority of
the 1971 Constitutional Convention. The court resolved the issue thus:

More specifically, as long as any proposed amendment is still unacted on by


it, there is no room for the interposition of judicial oversight. Only after it has made
concrete what it intends to submit for ratification may the appropriate case be
instituted. Until then, the courts are devoid of jurisdiction. That is the command of
the Constitution as interpreted by this Court. Unless and until such a doctrine loses
force by being overruled or a new precedent being announced, it is controlling. It is
implicit in the rule of law.[12]
Yet another requisite rooted in the very nature of judicial power is locus standi or
standing to sue. Thus, generally, a party will be allowed to litigate only when he can
demonstrate that (1) he has personally suffered some actual or threatened injury because of the
allegedly illegal conduct of the government; (2) the injury is fairly traceable to the challenged
action; and (3) the injury is likely to be redressed by the remedy being sought.[13] In the cases
at bar, petitioners have not shown the elemental injury in fact that would endow them with the
standing to sue. Locus standi requires a personal stake in the outcome of a controversy for
significant reasons. It assures adverseness and sharpens the presentation of issues for the
illumination of the Court in resolving difficult constitutional questions.[14] The lack of petitioners’

493
personal stake in this case is no more evident than in Lozano’s three-page petition that is devoid
of any legal or jurisprudential basis.

Neither can the lack of locus standi be cured by the claim of petitioners that they are
instituting the cases at bar as taxpayers and concerned citizens. A taxpayer’s suit requires that
the act complained of directly involves the illegal disbursement of public funds derived from
taxation.[15] It is undisputed that there has been no allocation or disbursement of public funds in
this case as of yet. To be sure, standing as a citizen has been upheld by this Court in cases
where a petitioner is able to craft an issue of transcendental importance or when paramount
public interest is involved.[16] While the Court recognizes the potential far-reaching implications
of the issue at hand, the possible consequence of House Resolution No.1109 is
yet unrealized and does not infuse petitioners with locus standi under the “transcendental
importance” doctrine.

The rule on locus standi is not a plain procedural rule but a constitutional requirement
derived from Section 1, Article VIII of the Constitution, which mandates courts of justice to
settle only "actual controversies involving rights which are legally demandable and
enforceable." As stated in Kilosbayan, Incorporated v. Guingona, Jr.,[17] viz.:
x x x [C]ourts are neither free to decide all kinds of cases dumped into their laps
nor are they free to open their doors to all parties or entities claiming a grievance.
The rationale for this constitutional requirement of locus standiis by no means trifle.
It is intended "to assure a vigorous adversary presentation of the case, and,
perhaps more importantly to warrant the judiciary's overruling the determination of
a coordinate, democratically elected organ of government." It thus goes to the
very essence of representative democracies.

xxxx

A lesser but not insignificant reason for screening the standing of persons who
desire to litigate constitutional issues is economic in character. Given the
sparseness of our resources, the capacity of courts to render efficient judicial
service to our people is severely limited. For courts to indiscriminately open their
doors to all types of suits and suitors is for them to unduly overburden their
dockets, and ultimately render themselves ineffective dispensers of justice. To be
sure, this is an evil that clearly confronts our judiciary today.

Moreover, while the Court has taken an increasingly liberal approach to the rule of locus
standi, evolving from the stringent requirements of “personal injury” to the broader
“transcendental importance” doctrine, such liberality is not to be abused. It is not an open
invitation for the ignorant and the ignoble to file petitions that prove nothing but their cerebral
deficit.

494
In the final scheme, judicial review is effective largely because it is not available simply at
the behest of a partisan faction, but is exercised only to remedy a particular, concrete
injury.[18] When warranted by the presence of indispensible minimums for judicial review, this
Court shall not shun the duty to resolve the constitutional challenge that may confront it.
IN VIEW WHEREOF, the petitions are dismissed.

SO ORDERED.

REYNATO S. PUNO
Chief Justice

WE CONCUR:

LEONARDO A. QUISUMBING
Associate Justice

CONSUELO YNARES-SANTIAGO ANTONIO T. CARPIO


Associate Justice Associate Justice

(on official leave)


RENATO C. CORONA CONCHITA CARPIO MORALES
Associate Justice Associate Justice

495
MINITA V. CHICO-NAZARIO PRESBITERO J. VELASCO, JR.
Associate Justice Associate Justice

ANTONIO EDUARDO B. NACHURA TERESITA J. LEONARDO-DE CASTRO


Associate Justice Associate Justice

ARTURO D. BRION DIOSDADO M. PERALTA


Associate Justice Associate Justice

LUCAS P. BERSAMIN
Associate Justice

CERTIFICATION

Pursuant to Section 13, Article VIII of the Constitution, I certify that the conclusions in
the above Resolution had been reached in consultation before the case was assigned to the
writer of the opinion of the Court.

REYNATO S. PUNO
Chief Justice

496
* On official leave.
[1]
Article VIII, Section 1, 1987 Constitution.
[2]
Marbury v. Madison , 1 Cranch 137, 2L. Ed. 60 [1803].
[3]
Angara v. Electoral Commission, 63 Phil. 139 (1936).
[4]
Ibid.
[5]
Alabama State Fed. of Labor v. McAdory, 325 U.S. 450 461 (1945).
[6]
Muskrat v. United States, 219 U.S. 346, 362 (1911).
[7]
Supra, see note 3.
[8]
Tribe, American Constitutional Law, 3d ed. 2000, p. 335.
[9]
Abbott Laboratories v. Gardner, 387 U.S. 136 (1967).
[10]
Guingona, Jr. v. Court of Appeals, 354 Phil. 415, 427-428 (1998).
[11]
Francisco, Jr. v. House of Representatives, 460 Phil. 830, 901-902 (2003).
[12]
G.R. No. L-34161, February 29, 1972, 43 SCRA 677, 682.
[13]
Tolentino v. COMELEC, 465 Phil. 385, 402 (2004).
[14]
Kilosbayan, Incorporated v. Morato, G.R. No. 118910, July 17, 1995, 246 SCRA 540.
[15]
Pascual v. Secretary of Public Works, 110 Phil. 331 (1960).
[16]
Integrated Bar of the Philippines v. Zamora, G.R. No. 141284, August 15, 2000, 338 SCRA
81.
[17]
See Dissent of then Associate Justice Reynato S. Puno, G.R. No. 113375, May 5, 1994, 232
SCRA 110.
[18]
Sierra Club v. Morton, 405 U.S. 727, 740-741, n. 16 (1972).

497
EN BANC

BARANGAY ASSOCIATION FOR G.R. No. 179271


NATIONAL ADVANCEMENT
AND TRANSPARENCY (BANAT),
Petitioner,

- versus -

COMMISSION ON ELECTIONS
(sitting as the National Board of
Canvassers),
Respondent.

ARTS BUSINESS AND SCIENCE


PROFESSIONALS,
Intervenor.

AANGAT TAYO,
Intervenor.

COALITION OF ASSOCIATIONS
OF SENIOR CITIZENS IN THE
PHILIPPINES, INC. (SENIOR
CITIZENS),
Intervenor.
x- - - - - - - - - - - - - - - - - - - - - - - - - - - - x
BAYAN MUNA, ADVOCACY FOR G.R. No. 179295
TEACHER EMPOWERMENT
THROUGH ACTION, COOPERATION Present:
AND HARMONY TOWARDS
EDUCATIONAL REFORMS, INC., PUNO, C.J.,
and ABONO, QUISUMBING,
Petitioners, YNARES-SANTIAGO,
CARPIO,
AUSTRIA-MARTINEZ,
CORONA,
498
- versus - CARPIO MORALES,
TINGA,
CHICO-NAZARIO,
VELASCO, JR.,

NACHURA,
LEONARDO-DE CASTRO,
BRION,
PERALTA, and
BERSAMIN, JJ.

COMMISSION ON ELECTIONS, Promulgated:


Respondent.
_______________________

x---------------------------------------------------x

DECISION

CARPIO, J.:

The Case

Petitioner in G.R. No. 179271 — Barangay Association for National Advancement and
Transparency (BANAT) — in a petition for certiorari and mandamus,[1] assails the
[2]
Resolution promulgated on 3 August 2007 by the Commission on Elections (COMELEC) in NBC
No. 07-041 (PL). The COMELEC’s resolution in NBC No. 07-041 (PL) approved the
recommendation of Atty. Alioden D. Dalaig, Head of the National Board of Canvassers (NBC)
Legal Group, to deny the petition of BANAT for being moot. BANAT filed before the COMELEC En
Banc, acting as NBC, a Petition to Proclaim the Full Number of Party-List Representatives
Provided by the Constitution.

499
The following are intervenors in G.R. No. 179271: Arts Business and Science
Professionals (ABS), Aangat Tayo (AT), and Coalition of Associations of Senior Citizens in the
Philippines, Inc. (Senior Citizens).

Petitioners in G.R. No. 179295 — Bayan Muna, Abono, and Advocacy for Teacher
Empowerment Through Action, Cooperation and Harmony Towards Educational Reforms (A
Teacher) — in a petition for certiorari with mandamus and prohibition,[3] assails NBC Resolution
No. 07-60[4] promulgated on 9 July 2007. NBC No. 07-60 made a partial proclamation of
parties, organizations and coalitions that obtained at least two percent of the total votes cast
under the Party-List System. The COMELEC announced that, upon completion of the canvass of
the party-list results, it would determine the total number of seats of each winning party,
organization, or coalition in accordance with Veterans Federation Party v.
[5]
COMELEC (Veterans).

Estrella DL Santos, in her capacity as President and First Nominee of the Veterans
Freedom Party, filed a motion to intervene in both G.R. Nos. 179271 and 179295.

The Facts

The 14 May 2007 elections included the elections for the party-list representatives. The
COMELEC counted 15,950,900 votes cast for 93 parties under the Party-List System.[6]
On 27 June 2002, BANAT filed a Petition to Proclaim the Full Number of Party-List
Representatives Provided by the Constitution, docketed as NBC No. 07-041 (PL) before the
NBC. BANAT filed its petition because “[t]he Chairman and the Members of the [COMELEC]
have recently been quoted in the national papers that the [COMELEC] is duty bound to and shall
implement the Veterans ruling, that is, would apply the Panganiban formula in allocating party-
list seats.”[7] There were no intervenors in BANAT’s petition before the NBC. BANAT filed a
memorandum on 19 July 2007.

On 9 July 2007, the COMELEC, sitting as the NBC, promulgated NBC Resolution No. 07-
60. NBC Resolution No. 07-60 proclaimed thirteen (13) parties as winners in the party-list
elections, namely: Buhay Hayaan Yumabong (BUHAY), Bayan Muna, Citizens’ Battle Against
Corruption (CIBAC), Gabriela’s Women Party (Gabriela), Association of Philippine Electric
Cooperatives (APEC), A Teacher, Akbayan! Citizen’s Action Party (AKBAYAN), Alagad, Luzon
Farmers Party (BUTIL), Cooperative-Natco Network Party (COOP-NATCCO), Anak Pawis, Alliance
of Rural Concerns (ARC), and Abono. We quote NBC Resolution No. 07-60 in its entirety below:

500
WHEREAS, the Commission on Elections sitting en banc as National Board
of Canvassers, thru its Sub-Committee for Party-List, as of 03 July 2007, had
officially canvassed, in open and public proceedings, a total of fifteen million two
hundred eighty three thousand six hundred fifty-nine (15,283,659) votes under the
Party-List System of Representation, in connection with the National and Local
Elections conducted last 14 May 2007;

WHEREAS, the study conducted by the Legal and Tabulation Groups of the
National Board of Canvassers reveals that the projected/maximum total party-list
votes cannot go any higher than sixteen million seven hundred twenty three
thousand one hundred twenty-one (16,723,121) votes given the following statistical
data:

Projected/Maximum Party-List Votes for May 2007 Elections

i. Total party-list votes already 15,283,659


canvassed/tabulated

ii. Total party-list votes remaining


uncanvassed/ untabulated (i.e. canvass 1,337,032
deferred)

iii. Maximum party-list votes (based on 100%


outcome) from areas not yet submitted for
canvass (Bogo, Cebu; Bais City; Pantar, Lanao
del Norte; and Pagalungan, Maguindanao) 102,430

Maximum Total Party-List Votes 16,723,121

WHEREAS, Section 11 of Republic Act No. 7941 (Party-List System Act)


provides in part:

The parties, organizations, and coalitions receiving at least two


percent (2%) of the total votes cast for the party-list system shall be
entitled to one seat each: provided, that those garnering more than two
percent (2%) of the votes shall be entitled to additional seats in
proportion to their total number of votes: provided, finally, that each
party, organization, or coalition shall be entitled to not more than three
(3) seats.

WHEREAS, for the 2007 Elections, based on the above projected total of
party-list votes, the presumptive two percent (2%) threshold can be pegged
at three hundred thirty four thousand four hundred sixty-two (334,462) votes;

WHEREAS, the Supreme Court, in Citizen’s Battle Against Corruption


(CIBAC) versus COMELEC, reiterated its ruling inVeterans Federation Party versus
501
COMELEC adopting a formula for the additional seats of each party, organization or
coalition receving more than the required two percent (2%) votes, stating that the
same shall be determined only after all party-list ballots have been completely
canvassed;

WHEREAS, the parties, organizations, and coalitions that have thus far
garnered at least three hundred thirty four thousand four hundred sixty-two
(334,462) votes are as follows:

RANK PARTY/ORGANIZATION/ VOTES


COALITION RECEIVED

1 BUHAY 1,163,218

2 BAYAN MUNA 972,730

3 CIBAC 760,260

4 GABRIELA 610,451

5 APEC 538,971

6 A TEACHER 476,036

7 AKBAYAN 470,872

8 ALAGAD 423,076

9 BUTIL 405,052

10 COOP-NATCO 390,029

11 BATAS 386,361

12 ANAK PAWIS 376,036

13 ARC 338,194

14 ABONO 337,046

WHEREAS, except for Bagong Alyansang Tagapagtaguyod ng Adhikaing


Sambayanan (BATAS), against which an URGENT PETITION FOR
CANCELLATION/REMOVAL OF REGISTRATION AND DISQUALIFICATION OF PARTY-
LIST NOMINEE (With Prayer for the Issuance of Restraining Order) has been filed
before the Commission, docketed as SPC No. 07-250, all the parties, organizations
and coalitions included in the aforementioned list are therefore entitled to at least
one seat under the party-list system of representation in the meantime.

NOW, THEREFORE, by virtue of the powers vested in it by the Constitution,


the Omnibus Election Code, Executive Order No. 144, Republic Act Nos. 6646,
7166, 7941, and other election laws, the Commission on Elections, sitting en

502
banc as the National Board of Canvassers, hereby RESOLVES to PARTIALLY
PROCLAIM, subject to certain conditions set forth below, the following parties,
organizations and coalitions participating under the Party-List System:

1 Buhay Hayaan Yumabong BUHAY

2 Bayan Muna BAYAN MUNA

3 Citizens Battle Against Corruption CIBAC

4 Gabriela Women’s Party GABRIELA

5 Association of Philippine Electric APEC


Cooperatives

6 Advocacy for Teacher Empowerment A TEACHER


Through Action, Cooperation and
Harmony Towards Educational
Reforms, Inc.

7 Akbayan! Citizen’s Action Party AKBAYAN

8 Alagad ALAGAD

9 Luzon Farmers Party BUTIL

10 Cooperative-Natco Network Party COOP-NATCCO

11 Anak Pawis ANAKPAWIS

12 Alliance of Rural Concerns ARC

13 Abono ABONO

This is without prejudice to the proclamation of other parties,


organizations, or coalitions which may later on be established to have obtained at
least two percent (2%) of the total actual votes cast under the Party-List System.

The total number of seats of each winning party, organization or coalition


shall be determined pursuant to Veterans Federation Party versus
COMELEC formula upon completion of the canvass of the party-list results.

The proclamation of Bagong Alyansang Tagapagtaguyod ng Adhikaing


Sambayanan (BATAS) is hereby deferred until final resolution of SPC No. 07-250, in
order not to render the proceedings therein moot and academic.

Finally, all proclamation of the nominees of concerned parties,


organizations and coalitions with pending disputes shall likewise be held in
abeyance until final resolution of their respective cases.

503
Let the Clerk of the Commission implement this Resolution, furnishing a
copy thereof to the Speaker of the House of Representatives of the Philippines.

SO ORDERED.[8] (Emphasis in the original)

Pursuant to NBC Resolution No. 07-60, the COMELEC, acting as NBC, promulgated NBC
Resolution No. 07-72, which declared the additional seats allocated to the appropriate
parties. We quote from the COMELEC’s interpretation of the Veteransformula as found in NBC
Resolution No. 07-72:

WHEREAS, on July 9, 2007, the Commission on Elections sitting en banc as


the National Board of Canvassers proclaimed thirteen (13) qualified parties,
organization[s] and coalitions based on the presumptive two percent (2%)
threshold of 334,462 votes from the projected maximum total number of party-list
votes of 16,723,121, and were thus given one (1) guaranteed party-list seat each;

WHEREAS, per Report of the Tabulation Group and Supervisory Committee


of the National Board of Canvassers, the projected maximum total party-list votes,
as of July 11, 2007, based on the votes actually canvassed, votes canvassed but
not included in Report No. 29, votes received but uncanvassed, and maximum
votes expected for Pantar, Lanao del Norte, is 16,261,369; and that the projected
maximum total votes for the thirteen (13) qualified parties, organizations and
coalition[s] are as follows:

Party-List Projected total number of


votes

1 BUHAY 1,178,747

2 BAYAN MUNA 977,476

3 CIBAC 755,964

4 GABRIELA 621,718

5 APEC 622,489

6 A TEACHER 492,369

7 AKBAYAN 462,674

8 ALAGAD 423,190

9 BUTIL 409,298

10 COOP-NATCO 412,920

504
11 ANAKPAWIS 370,165

12 ARC 375,846

13 ABONO 340,151

WHEREAS, based on the above Report, Buhay Hayaan Yumabong (Buhay)


obtained the highest number of votes among the thirteen (13) qualified parties,
organizations and coalitions, making it the “first party” in accordance with Veterans
Federation Party versus COMELEC, reiterated in Citizen’s Battle Against Corruption
(CIBAC) versus COMELEC;

WHEREAS, qualified parties, organizations and coalitions participating


under the party-list system of representation that have obtained one guaranteed
(1) seat may be entitled to an additional seat or seats based on the formula
prescribed by the Supreme Court in Veterans;

WHEREAS, in determining the additional seats for the “first party”, the
correct formula as expressed in Veterans, is:

Number of votes of first party Proportion of votes of first


--------------------- = party relative to total votes for
Total votes for party-list system party-list system

wherein the proportion of votes received by the first party (without rounding off)
shall entitle it to additional seats:

Proportion of votes received Additional seats


by the first party

Equal to or at least 6% Two (2) additional


seats

Equal to or greater than 4% but less One (1) additional


than 6% seat

Less than 4% No additional seat

WHEREAS, applying the above formula, Buhay obtained the following


percentage:

1,178,747
-------- = 0.07248 or 7.2%
16,261,369

which entitles it to two (2) additional seats.

505
WHEREAS, in determining the additional seats for the other qualified
parties, organizations and coalitions, the correct formula as expressed
in Veterans and reiterated in CIBAC is, as follows:

No. of votes of
concerned party No. of additional
Additional seats for = ------------------- x seats allocated to
a concerned party No. of votes of first party
first party

WHEREAS, applying the above formula, the results are as follows:

Party List Percentage Additional Seat

BAYAN MUNA 1.65 1

CIBAC 1.28 1

GABRIELA 1.05 1

APEC 1.05 1

A TEACHER 0.83 0

AKBAYAN 0.78 0

ALAGAD 0.71 0

BUTIL 0.69 0

COOP-NATCO 0.69 0

ANAKPAWIS 0.62 0

ARC 0.63 0

ABONO 0.57 0

NOW THEREFORE, by virtue of the powers vested in it by the Constitution,


Omnibus Election Code, Executive Order No. 144, Republic Act Nos. 6646, 7166,
7941 and other elections laws, the Commission on Elections en banc sitting as the
National Board of Canvassers, hereby RESOLVED, as it hereby RESOLVES, to
proclaim the following parties, organizations or coalitions as entitled to additional
seats, to wit:

Party List Additional Seats

BUHAY 2

506
BAYAN MUNA 1

CIBAC 1

GABRIELA 1

APEC 1

This is without prejudice to the proclamation of other parties, organizations


or coalitions which may later on be established to have obtained at least two per
cent (2%) of the total votes cast under the party-list system to entitle them to one
(1) guaranteed seat, or to the appropriate percentage of votes to entitle them to
one (1) additional seat.

Finally, all proclamation of the nominees of concerned parties,


organizations and coalitions with pending disputes shall likewise be held in
abeyance until final resolution of their respective cases.

Let the National Board of Canvassers Secretariat implement this


Resolution, furnishing a copy hereof to the Speaker of the House of Representatives
of the Philippines.

SO ORDERED.[9]

Acting on BANAT’s petition, the NBC promulgated NBC Resolution No. 07-88 on 3 August
2007, which reads as follows:

This pertains to the Petition to Proclaim the Full Number of Party-List


Representatives Provided by the Constitution filed by the Barangay Association for
National Advancement and Transparency (BANAT).

Acting on the foregoing Petition of the Barangay Association for National


Advancement and Transparency (BANAT) party-list, Atty. Alioden D. Dalaig, Head,
National Board of Canvassers Legal Group submitted his comments/observations
and recommendation thereon [NBC 07-041 (PL)], which reads:

COMMENTS / OBSERVATIONS:

Petitioner Barangay Association for National Advancement and


Transparency (BANAT), in its Petition to Proclaim the Full Number of
Party-List Representatives Provided by the Constitution prayed for the
following reliefs, to wit:

507
1. That the full number -- twenty percent (20%) -- of Party-List
representatives as mandated by Section 5, Article VI of the
Constitution shall be proclaimed.

2. Paragraph (b), Section 11 of RA 7941 which prescribes the


2% threshold votes, should be harmonized with Section 5, Article VI of
the Constitution and with Section 12 of the same RA 7941 in that it
should be applicable only to the first party-list representative seats to
be allotted on the basis of their initial/first ranking.

3. The 3-seat limit prescribed by RA 7941 shall be applied; and

4. Initially, all party-list groups shall be given the number of


seats corresponding to every 2% of the votes they received and the
additional seats shall be allocated in accordance with Section 12 of RA
7941, that is, in proportion to the percentage of votes obtained by
each party-list group in relation to the total nationwide votes cast in
the party-list election, after deducting the corresponding votes of those
which were allotted seats under the 2% threshold rule. In fine, the
formula/procedure prescribed in the “ALLOCATION OF PARTY-LIST
SEATS, ANNEX “A” of COMELEC RESOLUTION 2847 dated 25 June
1996, shall be used for [the] purpose of determining how many seats
shall be proclaimed, which party-list groups are entitled to
representative seats and how many of their nominees shall seat [sic].

5. In the alternative, to declare as unconstitutional Section 11 of


Republic Act No. 7941 and that the procedure in allocating seats for
party-list representative prescribed by Section 12 of RA 7941 shall be
followed.

RECOMMENDATION:

The petition of BANAT is now moot and academic.

The Commission En Banc in NBC Resolution No. 07-60


promulgated July 9, 2007 re “In the Matter of the Canvass of Votes
and Partial Proclamation of the Parties, Organizations and Coalitions
Participating Under the Party-List System During the May 14, 2007
National and Local Elections” resolved among others that the total
number of seats of each winning party, organization or coalition shall
be determined pursuant to the Veterans Federation
Party versus COMELEC formula upon completion of the canvass of the
party-list results.”

WHEREFORE, premises considered, the National Board of Canvassers


RESOLVED, as it hereby RESOLVES, to approve and adopt the recommendation of
Atty. Alioden D. Dalaig, Head, NBC Legal Group, to DENY the herein petition of
BANAT for being moot and academic.

508
Let the Supervisory Committee implement this resolution.

SO ORDERED.[10]

BANAT filed a petition for certiorari and mandamus assailing the ruling in NBC Resolution
No. 07-88. BANAT did not file a motion for reconsideration of NBC Resolution No. 07-88.

On 9 July 2007, Bayan Muna, Abono, and A Teacher asked the COMELEC, acting as NBC,
to reconsider its decision to use the Veterans formula as stated in its NBC Resolution No. 07-60
because the Veterans formula is violative of the Constitution and of Republic Act No. 7941 (R.A.
No. 7941). On the same day, the COMELEC denied reconsideration during the proceedings of
the NBC.[11]

Aside from the thirteen party-list organizations proclaimed on 9 July 2007, the COMELEC
proclaimed three other party-list organizations as qualified parties entitled to one guaranteed
seat under the Party-List System: Agricultural Sector Alliance of the Philippines, Inc.
(AGAP),[12] Anak Mindanao (AMIN),[13] and An Waray.[14] Per the certification[15] by COMELEC,
the following party-list organizations have been proclaimed as of 19 May 2008:

Party-List No. of Seat(s)

1.1 Buhay 3

1.2 Bayan Muna 2

1.3 CIBAC 2

1.4 Gabriela 2

1.5 APEC 2

1.6 A Teacher 1

1.7 Akbayan 1

1.8 Alagad 1

1.9 Butil 1

1.10 Coop-Natco [sic] 1

1.11 Anak Pawis 1

1.12 ARC 1

509
1.13 Abono 1

1.14 AGAP 1

1.15 AMIN 1

The proclamation of Bagong Alyansang Tagapagtaguyod ng Adhikaing Sambayanan (BATAS),


against which an Urgent Petition for Cancellation/Removal of Registration and Disqualification of
Party-list Nominee (with Prayer for the Issuance of Restraining Order) has been filed before the
COMELEC, was deferred pending final resolution of SPC No. 07-250.

Issues

BANAT brought the following issues before this Court:

1. Is the twenty percent allocation for party-list


representatives provided in Section 5(2), Article VI of the Constitution
mandatory or is it merely a ceiling?

2. Is the three-seat limit provided in Section 11(b) of RA


7941 constitutional?

3. Is the two percent threshold and “qualifier” votes prescribed by


the same Section 11(b) of RA 7941 constitutional?

4. How shall the party-list representatives be allocated?[16]

Bayan Muna, A Teacher, and Abono, on the other hand, raised the following issues in
their petition:

I. Respondent Commission on Elections, acting as National Board


of Canvassers, committed grave abuse of discretion amounting to lack or
excess of jurisdiction when it promulgated NBC Resolution No. 07-60 to
implement the First-Party Rule in the allocation of seats to qualified party-list
organizations as said rule:

A. Violates the constitutional principle of


proportional representation.

B. Violates the provisions of RA 7941 particularly:

1. The 2-4-6 Formula used by the First Party Rule


in allocating additional seats for the “First
510
Party” violates the principle of proportional
representation under RA 7941.

2. The use of two formulas in the allocation


of additional seats, one for the “First Party”
and another for the qualifying parties, violates
Section 11(b) of RA 7941.

3. The proportional relationships under the First


Party Rule are different from those required under
RA 7941;

C. Violates the “Four Inviolable Parameters” of the


Philippine party-list system as provided for under the same case
of Veterans Federation Party, et al. v. COMELEC.

II. Presuming that the Commission on Elections did not commit


grave abuse of discretion amounting to lack or excess of
jurisdiction when it implemented the First-Party Rule in the allocation of
seats to qualified party-list organizations, the same being merely
in consonance with the ruling in Veterans Federations Party, et al.
v. COMELEC, the instant Petition is a justiciable case as the issues involved
herein are constitutional in nature, involving the correct interpretation and
implementation of RA 7941, and are of transcendental importance to our nation.[17]

Considering the allegations in the petitions and the comments of the parties in these
cases, we defined the following issues in our advisory for the oral arguments set on 22 April
2008:

1. Is the twenty percent allocation for party-list representatives


in Section 5(2), Article VI of the Constitution mandatory or merely
a ceiling?

2. Is the three-seat limit in Section 11(b) of RA 7941 constitutional?

3. Is the two percent threshold prescribed in Section 11(b) of RA 7941 to


qualify for one seat constitutional?

4. How shall the party-list representative seats be allocated?

5. Does the Constitution prohibit the major political parties


from participating in the party-list elections? If not, can the major political
parties be barred from participating in the party-list elections?[18]

The Ruling of the Court

511
The petitions have partial merit. We maintain that a Philippine-style party-list election
has at least four inviolable parameters as clearly stated in Veterans. For easy reference, these
are:

First, the twenty percent allocation — the combined number of all party-list
congressmen shall not exceed twenty percent of the total membership of the House
of Representatives, including those elected under the party list;

Second, the two percent threshold — only those parties garnering a


minimum of two percent of the total valid votes cast for the party-list system are
“qualified” to have a seat in the House of Representatives;

Third, the three-seat limit — each qualified party, regardless of the number
of votes it actually obtained, is entitled to a maximum of three seats; that is, one
“qualifying” and two additional seats;

Fourth, proportional representation— the additional seats which a qualified


party is entitled to shall be computed “in proportion to their total number of
votes.”[19]

However, because the formula in Veterans has flaws in its mathematical interpretation of the
term “proportional representation,” this Court is compelled to revisit the formula for the
allocation of additional seats to party-list organizations.

Number of Party-List Representatives:


The Formula Mandated by the Constitution

Section 5, Article VI of the Constitution provides:

Section 5. (1) The House of Representatives shall be composed of not more


than two hundred and fifty members, unless otherwise fixed by law, who shall be
elected from legislative districts apportioned among the provinces, cities, and the
Metropolitan Manila area in accordance with the number of their respective
inhabitants, and on the basis of a uniform and progressive ratio, and those who, as
provided by law, shall be elected through a party-list system of registered national,
regional, and sectoral parties or organizations.

(2) The party-list representatives shall constitute twenty per centum of the
total number of representatives including those under the party-list. For three
consecutive terms after the ratification of this Constitution, one-half of the seats
allocated to party-list representatives shall be filled, as provided by law, by
selection or election from the labor, peasant, urban poor, indigenous cultural

512
communities, women, youth, and such other sectors as may be provided by law,
except the religious sector.

The first paragraph of Section 11 of R.A. No. 7941 reads:

Section 11. Number of Party-List Representatives. — The party-list


representatives shall constitute twenty per centum (20%) of the total number of
the members of the House of Representatives including those under the party-list.
xxx

Section 5(1), Article VI of the Constitution states that the “House of Representatives shall
be composed of not more than two hundred and fifty members, unless otherwise fixed by
law.” The House of Representatives shall be composed of district representatives and party-list
representatives. The Constitution allows the legislature to modify the number of the members of
the House of Representatives.

Section 5(2), Article VI of the Constitution, on the other hand, states the ratio of party-
list representatives to the total number of representatives. We compute the number of seats
available to party-list representatives from the number of legislative districts. On this point, we
do not deviate from the first formula in Veterans, thus:

Number of seats Number of seats available


available to legislative x .20 = to
districts party-list representatives

.80

This formula allows for the corresponding increase in the number of seats available for party-list
representatives whenever a legislative district is created by law. Since the 14th Congress of the
Philippines has 220 district representatives, there are 55 seats available to party-list
representatives.

220 x .20 = 55

.80

513
After prescribing the ratio of the number of party-list representatives to the total number
of representatives, the Constitution left the manner of allocating the seats available to party-list
representatives to the wisdom of the legislature.

Allocation of Seats for Party-List Representatives:


The Statutory Limits Presented by the Two Percent Threshold
and the Three-Seat Cap

All parties agree on the formula to determine the maximum number of seats reserved
under the Party-List System, as well as on the formula to determine the guaranteed seats to
party-list candidates garnering at least two-percent of the total party-list votes. However, there
are numerous interpretations of the provisions of R.A. No. 7941 on the allocation of “additional
seats”under the Party-List System. Veterans produced the First Party Rule,[20] and Justice
Vicente V. Mendoza’s dissent in Veteranspresented Germany’s Niemeyer formula[21] as an
alternative.

The Constitution left to Congress the determination of the manner of allocating the seats
for party-list representatives. Congress enacted R.A. No. 7941, paragraphs (a) and (b) of
Section 11 and Section 12 of which provide:

Section 11. Number of Party-List Representatives. — x x x

In determining the allocation of seats for the second vote,[22] the following
procedure shall be observed:

(a) The parties, organizations, and coalitions shall be ranked from the highest
to the lowest based on the number of votes they garnered during the elections.

(b) The parties, organizations, and coalitions receiving at least two percent
(2%) of the total votes cast for the party-list system shall be entitled to one seat
each: Provided, That those garnering more than two percent (2%) of the votes
shall be entitled to additional seats in proportion to their total number of
votes: Provided, finally, That each party, organization, or coalition shall be entitled
to not more than three (3) seats.

Section 12. Procedure in Allocating Seats for Party-List Representatives. —


The COMELEC shall tally all the votes for the parties, organizations, or coalitions on
a nationwide basis, rank them according to the number of votes received and
allocate party-list representatives proportionately according to the percentage of
votes obtained by each party, organization, or coalition as against the total
nationwide votes cast for the party-list system. (Emphasis supplied)
514
In G.R. No. 179271, BANAT presents two interpretations through three formulas to
allocate party-list representative seats.

The first interpretation allegedly harmonizes the provisions of Section 11(b) on the 2%
requirement with Section 12 of R.A. No. 7941. BANAT described this procedure as follows:

(a) The party-list representatives shall constitute twenty percent (20%) of the
total Members of the House of Representatives including those from the party-list
groups as prescribed by Section 5, Article VI of the Constitution, Section 11
(1st par.) of RA 7941 and Comelec Resolution No. 2847 dated 25 June 1996. Since
there are 220 District Representatives in the 14th Congress, there shall be 55 Party-
List Representatives. All seats shall have to be proclaimed.

(b) All party-list groups shall initially be allotted one (1) seat for every two per
centum (2%) of the total party-list votes they obtained; provided, that no party-list
groups shall have more than three (3) seats (Section 11, RA 7941).

(c) The remaining seats shall, after deducting the seats obtained by the party-
list groups under the immediately preceding paragraph and after deducting from
their total the votes corresponding to those seats, the remaining seats shall be
allotted proportionately to all the party-list groups which have not secured the
maximum three (3) seats under the 2% threshold rule, in accordance with Section
12 of RA 7941.[23]

Forty-four (44) party-list seats will be awarded under BANAT’s first interpretation.

The second interpretation presented by BANAT assumes that the 2% vote requirement is
declared unconstitutional, and apportions the seats for party-list representatives by following
Section 12 of R.A. No. 7941. BANAT states that the COMELEC:

(a) shall tally all the votes for the parties, organizations, or coalitions on a
nationwide basis;
(b) rank them according to the number of votes received; and,
(c) allocate party-list representatives proportionately according to
the percentage of votes obtained by each party, organization or coalition as
against the total nationwide votes cast for the party-list system.[24]

BANAT used two formulas to obtain the same results: one is based on the proportional
percentage of the votes received by each party as against the total nationwide party-list votes,
and the other is “by making the votes of a party-list with a median percentage of votes as the
515
divisor in computing the allocation of seats.”[25] Thirty-four (34) party-list seats will be awarded
under BANAT’s second interpretation.

In G.R. No. 179295, Bayan Muna, Abono, and A Teacher criticize both the COMELEC’s
original 2-4-6 formula and theVeterans formula for systematically preventing all the party-list
seats from being filled up. They claim that both formulas do not factor in the total number of
seats alloted for the entire Party-List System. Bayan Muna, Abono, and A Teacher reject the
three-seat cap, but accept the 2% threshold. After determining the qualified parties, a second
percentage is generated by dividing the votes of a qualified party by the total votes of all
qualified parties only. The number of seats allocated to a qualified party is computed by
multiplying the total party-list seats available with the second percentage. There will be a first
round of seat allocation, limited to using the whole integers as the equivalent of the number of
seats allocated to the concerned party-list. After all the qualified parties are given their seats, a
second round of seat allocation is conducted. The fractions, or remainders, from the whole
integers are ranked from highest to lowest and the remaining seats on the basis of this ranking
are allocated until all the seats are filled up.[26]

We examine what R.A. No. 7941 prescribes to allocate seats for party-list
representatives.

Section 11(a) of R.A. No. 7941 prescribes the ranking of the participating parties from the
highest to the lowest based on the number of votes they garnered during the elections.

Table 1. Ranking of the participating parties from the highest to the lowest based
on the number of votes garnered during the elections.[27]

Votes Votes
Rank Party Rank Party
Garnered Garnered

1 BUHAY 1,169,234 48 KALAHI 88,868

2 BAYAN 979,039 49 APOI 79,386


MUNA

3 CIBAC 755,686 50 BP 78,541

4 GABRIELA 621,171 51 AHONBAYAN 78,424

516
5 APEC 619,657 52 BIGKIS 77,327

6 A TEACHER 490,379 53 PMAP 75,200

7 AKBAYAN 466,112 54 AKAPIN 74,686

8 ALAGAD 423,149 55 PBA 71,544

9 COOP- 409,883 56 GRECON 62,220


NATCCO

10 BUTIL 409,160 57 BTM 60,993

11 BATAS 385,810 58 A SMILE 58,717

12 ARC 374,288 59 NELFFI 57,872

13 ANAKPAWIS 370,261 60 AKSA 57,012

14 ABONO 339,990 61 BAGO 55,846

15 AMIN 338,185 62 BANDILA 54,751

16 AGAP 328,724 63 AHON 54,522

17 AN WARAY 321,503 64 ASAHAN MO 51,722

18 YACAP 310,889 65 AGBIAG! 50,837

19 FPJPM 300,923 66 SPI 50,478

20 UNI-MAD 245,382 67 BAHANDI 46,612

21 ABS 235,086 68 ADD 45,624

22 KAKUSA 228,999 69 AMANG 43,062

23 KABATAAN 228,637 70 ABAY PARAK 42,282

24 ABA-AKO 218,818 71 BABAE KA 36,512

25 ALIF 217,822 72 SB 34,835

26 SENIOR 213,058 73 ASAP 34,098


CITIZENS

27 AT 197,872 74 PEP 33,938

28 VFP 196,266 75 ABA 33,903


ILONGGO

29 ANAD 188,521 76 VENDORS 33,691

517
30 BANAT 177,028 77 ADD-TRIBAL 32,896

31 ANG 170,531 78 ALMANA 32,255


KASANGGA

32 BANTAY 169,801 79 AANGAT KA 29,130


PILIPINO

33 ABAKADA 166,747 80 AAPS 26,271

34 1-UTAK 164,980 81 HAPI 25,781

35 TUCP 162,647 82 AAWAS 22,946

36 COCOFED 155,920 83 SM 20,744

37 AGHAM 146,032 84 AG 16,916

38 ANAK 141,817 85 AGING PINOY 16,729

39 ABANSE! 130,356 86 APO 16,421


PINAY

40 PM 119,054 87 BIYAYANG 16,241


BUKID

41 AVE 110,769 88 ATS 14,161

42 SUARA 110,732 89 UMDJ 9,445

43 ASSALAM 110,440 90 BUKLOD 8,915


FILIPINA

44 DIWA 107,021 91 LYPAD 8,471

45 ANC 99,636 92 AA-KASOSYO 8,406

46 SANLAKAS 97,375 93 KASAPI 6,221

47 ABC 90,058 TOTAL 15,950,900

The first clause of Section 11(b) of R.A. No. 7941 states that “parties, organizations, and
coalitions receiving at least two percent (2%) of the total votes cast for the party-list system
shall be entitled to one seat each.” This clause guarantees a seat to the two-percenters. In
Table 2 below, we use the first 20 party-list candidates for illustration purposes. The percentage
of votes garnered by each party is arrived at by dividing the number of votes garnered by each
party by 15,950,900, the total number of votes cast for all party-list candidates.
518
Table 2. The first 20 party-list candidates and their respective percentage of votes
garnered over the total votes for the party-list.[28]

Votes
Garnered over
Votes Guaranteed
Rank Party Total Votes for
Garnered Seat
Party-List, in
%

1 BUHAY 1,169,234 7.33% 1

2 BAYAN MUNA 979,039 6.14% 1

3 CIBAC 755,686 4.74% 1

4 GABRIELA 621,171 3.89% 1

5 APEC 619,657 3.88% 1

6 A TEACHER 490,379 3.07% 1

7 AKBAYAN 466,112 2.92% 1

8 ALAGAD 423,149 2.65% 1

9 COOP-NATCCO 409,883 2.57% 1

10 BUTIL 409,160 2.57% 1

11 BATAS[29] 385,810 2.42% 1

12 ARC 374,288 2.35% 1

13 ANAKPAWIS 370,261 2.32% 1

14 ABONO 339,990 2.13% 1

15 AMIN 338,185 2.12% 1

16 AGAP 328,724 2.06% 1

17 AN WARAY 321,503 2.02% 1

Total 17

18 YACAP 310,889 1.95% 0

19 FPJPM 300,923 1.89% 0

20 UNI-MAD 245,382 1.54% 0

519
From Table 2 above, we see that only 17 party-list candidates received at least 2% from
the total number of votes cast for party-list candidates. The 17 qualified party-list candidates,
or the two-percenters, are the party-list candidates that are “entitled to one seat each,” or the
guaranteed seat. In this first round of seat allocation, we distributed 17 guaranteed seats.
The second clause of Section 11(b) of R.A. No. 7941 provides that “those garnering more
than two percent (2%) of the votes shall be entitled to additional seats in proportion to their
total number of votes.” This is where petitioners’ and intervenors’ problem with the formula
in Veterans lies. Veterans interprets the clause “in proportion to their total number of votes” to
be in proportion to the votes of the first party. This interpretation is contrary to the express
language of R.A. No. 7941.

We rule that, in computing the allocation of additional seats, the continued operation of
the two percent threshold for the distribution of the additional seats as found in the second
clause of Section 11(b) of R.A. No. 7941 is unconstitutional. This Court finds that the two
percent threshold makes it mathematically impossible to achieve the maximum number of
available party list seats when the number of available party list seats exceeds 50. The
continued operation of the two percent threshold in the distribution of the additional seats
frustrates the attainment of the permissive ceiling that 20% of the members of the House of
Representatives shall consist of party-list representatives.

To illustrate: There are 55 available party-list seats. Suppose there are 50 million votes
cast for the 100 participants in the party list elections. A party that has two percent of the
votes cast, or one million votes, gets a guaranteed seat. Let us further assume that the first 50
parties all get one million votes. Only 50 parties get a seat despite the availability of 55
seats. Because of the operation of the two percent threshold, this situation will repeat itself
even if we increase the available party-list seats to 60 seats and even if we increase the votes
cast to 100 million. Thus, even if the maximum number of parties get two percent of the votes
for every party, it is always impossible for the number of occupied party-list seats to exceed 50
seats as long as the two percent threshold is present.

We therefore strike down the two percent threshold only in relation to the distribution of
the additional seats as found in the second clause of Section 11(b) of R.A. No. 7941. The two
percent threshold presents an unwarranted obstacle to the full implementation of Section 5(2),

520
Article VI of the Constitution and prevents the attainment of “the broadest possible
representation of party, sectoral or group interests in the House of Representatives.”[30]

In determining the allocation of seats for party-list representatives under Section 11 of


R.A. No. 7941, the following procedure shall be observed:

1. The parties, organizations, and coalitions shall be ranked from the highest to the
lowest based on the number of votes they garnered during the elections.

2. The parties, organizations, and coalitions receiving at least two percent (2%) of
the total votes cast for the party-list system shall be entitled to one guaranteed seat each.

3. Those garnering sufficient number of votes, according to the ranking in paragraph


1, shall be entitled to additional seats in proportion to their total number of votes until all the
additional seats are allocated.

4. Each party, organization, or coalition shall be entitled to not more than three (3)
seats.

In computing the additional seats, the guaranteed seats shall no longer be included
because they have already been allocated, at one seat each, to every two-percenter. Thus, the
remaining available seats for allocation as “additional seats” are the maximum seats reserved
under the Party List System less the guaranteed seats. Fractional seats are disregarded in the
absence of a provision in R.A. No. 7941 allowing for a rounding off of fractional seats.

In declaring the two percent threshold unconstitutional, we do not limit our allocation of
additional seats in Table 3 below to the two-percenters. The percentage of votes garnered by
each party-list candidate is arrived at by dividing the number of votes garnered by each party by
15,950,900, the total number of votes cast for party-list candidates. There are two steps in the
second round of seat allocation. First, the percentage is multiplied by the remaining available
seats, 38, which is the difference between the 55 maximum seats reserved under the Party-List
System and the 17 guaranteed seats of the two-percenters. The whole integer of the product of
the percentage and of the remaining available seats corresponds to a party’s share in the
remaining available seats. Second, we assign one party-list seat to each of the parties next in
rank until all available seats are completely distributed. We distributed all of the remaining 38

521
seats in the second round of seat allocation. Finally, we apply the three-seat cap to determine
the number of seats each qualified party-list candidate is entitled. Thus:

Table 3. Distribution of Available Party-List Seats

Votes Guaranteed Additional (B) plus Applying


Garnered Seat Seats (C), in the
over whole three
Total integers seat cap
Votes for
Votes
Rank Party Party
Garnered
List, in %
(First (Second
Round) Round)

(A) (B) (C) (D) (E)

1 BUHAY 1,169,234 7.33% 1 2.79 3 N.A.

2 BAYAN 979,039 6.14% 1 2.33 3 N.A.


MUNA

3 CIBAC 755,686 4.74% 1 1.80 2 N.A.

4 GABRIELA 621,171 3.89% 1 1.48 2 N.A.

5 APEC 619,657 3.88% 1 1.48 2 N.A.

6 A Teacher 490,379 3.07% 1 1.17 2 N.A.

7 AKBAYAN 466,112 2.92% 1 1.11 2 N.A.

8 ALAGAD 423,149 2.65% 1 1.01 2 N.A.

9[31] COOP- 409,883 2.57% 1 1 2 N.A.


NATCCO

522
10 BUTIL 409,160 2.57% 1 1 2 N.A.

11 BATAS 385,810 2.42% 1 1 2 N.A.

12 ARC 374,288 2.35% 1 1 2 N.A.

13 ANAKPAWIS 370,261 2.32% 1 1 2 N.A.

14 ABONO 339,990 2.13% 1 1 2 N.A.

15 AMIN 338,185 2.12% 1 1 2 N.A.

16 AGAP 328,724 2.06% 1 1 2 N.A.

17 AN WARAY 321,503 2.02% 1 1 2 N.A.

18 YACAP 310,889 1.95% 0 1 1 N.A.

19 FPJPM 300,923 1.89% 0 1 1 N.A.

20 UNI-MAD 245,382 1.54% 0 1 1 N.A.

21 ABS 235,086 1.47% 0 1 1 N.A.

22 KAKUSA 228,999 1.44% 0 1 1 N.A.

23 KABATAAN 228,637 1.43% 0 1 1 N.A.

24 ABA-AKO 218,818 1.37% 0 1 1 N.A.

25 ALIF 217,822 1.37% 0 1 1 N.A.

26 SENIOR 213,058 1.34% 0 1 1 N.A.


CITIZENS

27 AT 197,872 1.24% 0 1 1 N.A.

28 VFP 196,266 1.23% 0 1 1 N.A.

29 ANAD 188,521 1.18% 0 1 1 N.A.

30 BANAT 177,028 1.11% 0 1 1 N.A.

31 ANG 170,531 1.07% 0 1 1 N.A.


KASANGGA

32 BANTAY 169,801 1.06% 0 1 1 N.A.

33 ABAKADA 166,747 1.05% 0 1 1 N.A.

34 1-UTAK 164,980 1.03% 0 1 1 N.A.

35 TUCP 162,647 1.02% 0 1 1 N.A.

523
36 COCOFED 155,920 0.98% 0 1 1 N.A.

Total 17 55

Applying the procedure of seat allocation as illustrated in Table 3 above, there are 55
party-list representatives from the 36 winning party-list organizations. All 55 available party-list
seats are filled. The additional seats allocated to the parties with sufficient number of votes for
one whole seat, in no case to exceed a total of three seats for each party, are shown in column
(D).

Participation of Major Political Parties in Party-List Elections

The Constitutional Commission adopted a multi-party system that allowed all political
parties to participate in the party-list elections. The deliberations of the Constitutional
Commission clearly bear this out, thus:

MR. MONSOD. Madam President, I just want to say that we suggested or


proposed the party list system because we wanted to open up the political system
to a pluralistic society through a multiparty system. x x x We are for opening up
the system, and we would like very much for the sectors to be there. That is why
one of the ways to do that is to put a ceiling on the number of representatives from
any single party that can sit within the 50 allocated under the party list system. x x
x.

xxx

MR. MONSOD. Madam President, the candidacy for the 198 seats is not
limited to political parties. My question is this: Are we going to classify for example
Christian Democrats and Social Democrats as political parties? Can they run under
the party list concept or must they be under the district legislation side of it only?

MR. VILLACORTA. In reply to that query, I think these parties that the
Commissioner mentioned can field candidates for the Senate as well as for the
House of Representatives. Likewise, they can also field sectoral candidates for the
20 percent or 30 percent, whichever is adopted, of the seats that we are allocating
under the party list system.

MR. MONSOD. In other words, the Christian Democrats can field district
candidates and can also participate in the party list system?

MR. VILLACORTA. Why not? When they come to the party list system,
they will be fielding only sectoral candidates.

524
MR. MONSOD. May I be clarified on that? Can UNIDO participate in the
party list system?

MR. VILLACORTA. Yes, why not? For as long as they field candidates who
come from the different marginalized sectors that we shall designate in this
Constitution.

MR. MONSOD. Suppose Senator Tañada wants to run under BAYAN group
and says that he represents the farmers, would he qualify?

MR. VILLACORTA. No, Senator Tañada would not qualify.

MR. MONSOD. But UNIDO can field candidates under the party list system
and say Juan dela Cruz is a farmer. Who would pass on whether he is a farmer or
not?

MR. TADEO. Kay Commissioner Monsod, gusto ko lamang linawin


ito. Political parties, particularly minority political parties, are not prohibited to
participate in the party list election if they can prove that they are also organized
along sectoral lines.

MR. MONSOD. What the Commissioner is saying is that all political parties
can participate because it is precisely the contention of political parties that they
represent the broad base of citizens and that all sectors are represented in
them. Would the Commissioner agree?

MR. TADEO. Ang punto lamang namin, pag pinayagan mo ang UNIDO na
isang political party, it will dominate the party list at mawawalang saysay din yung
sector. Lalamunin mismo ng political parties ang party list system. Gusto ko
lamang bigyan ng diin ang “reserve.” Hindi ito reserve seat sa marginalized
sectors. Kung titingnan natin itong 198 seats, reserved din ito sa political parties.

MR. MONSOD. Hindi po reserved iyon kasi anybody can run there. But my
question to Commissioner Villacorta and probably also to Commissioner Tadeo is
that under this system, would UNIDO be banned from running under the party list
system?

MR. VILLACORTA. No, as I said, UNIDO may field sectoral candidates. On


that condition alone, UNIDO may be allowed to register for the party list system.

MR. MONSOD. May I inquire from Commissioner Tadeo if he shares that


answer?

MR. TADEO. The same.

MR. VILLACORTA. Puwede po ang UNIDO, pero sa sectoral lines.

xxxx

525
MR. OPLE. x x x In my opinion, this will also create the stimulus for
political parties and mass organizations to seek common ground. For example, we
have the PDP-Laban and the UNIDO. I see no reason why they should not be able
to make common goals with mass organizations so that the very leadership of
these parties can be transformed through the participation of mass
organizations. And if this is true of the administration parties, this will be true of
others like the Partido ng Bayan which is now being formed. There is no question
that they will be attractive to many mass organizations. In the opposition parties
to which we belong, there will be a stimulus for us to contact mass organizations so
that with their participation, the policies of such parties can be radically
transformed because this amendment will create conditions that will challenge both
the mass organizations and the political parties to come together. And the party
list system is certainly available, although it is open to all the parties. It is
understood that the parties will enter in the roll of the COMELEC the names of
representatives of mass organizations affiliated with them. So that we may, in
time, develop this excellent system that they have in Europe where labor
organizations and cooperatives, for example, distribute themselves either in the
Social Democratic Party and the Christian Democratic Party in Germany, and their
very presence there has a transforming effect upon the philosophies and the
leadership of those parties.

It is also a fact well known to all that in the United States, the AFL-CIO
always vote with the Democratic Party. But the businessmen, most of them,
always vote with the Republican Party, meaning that there is no reason at all why
political parties and mass organizations should not combine, reenforce, influence
and interact with each other so that the very objectives that we set in this
Constitution for sectoral representation are achieved in a wider, more lasting, and
more institutionalized way. Therefore, I support this [Monsod-Villacorta]
amendment. It installs sectoral representation as a constitutional gift, but at the
same time, it challenges the sector to rise to the majesty of being elected
representatives later on through a party list system; and even beyond that, to
become actual political parties capable of contesting political power in the wider
constitutional arena for major political parties.
[32]
xxx (Emphasis supplied)
R.A. No. 7941 provided the details for the concepts put forward by the Constitutional
Commission. Section 3 of R.A. No. 7941 reads:

Definition of Terms. (a) The party-list system is a mechanism of


proportional representation in the election of representatives to the House of
Representatives from national, regional and sectoral parties or organizations or
coalitions thereof registered with the Commission on Elections (COMELEC).
Component parties or organizations of a coalition may participate independently
provided the coalition of which they form part does not participate in the party-list
system.

(b) A party means either a political party or a sectoral party or a coalition


of parties.
526
(c) A political party refers to an organized group of citizens advocating an
ideology or platform, principles and policies for the general conduct of government
and which, as the most immediate means of securing their adoption, regularly
nominates and supports certain of its leaders and members as candidates for public
office.

It is a national party when its constituency is spread over the geographical


territory of at least a majority of the regions. It is a regional party when its
constituency is spread over the geographical territory of at least a majority of the
cities and provinces comprising the region.

(d) A sectoral party refers to an organized group of citizens belonging to


any of the sectors enumerated in Section 5 hereof whose principal advocacy
pertains to the special interests and concerns of their sector,

(e) A sectoral organization refers to a group of citizens or a coalition of


groups of citizens who share similar physical attributes or characteristics,
employment, interests or concerns.

(f) A coalition refers to an aggrupation of duly registered national, regional,


sectoral parties or organizations for political and/or election purposes.

Congress, in enacting R.A. No. 7941, put the three-seat cap to prevent any party from
dominating the party-list elections.

Neither the Constitution nor R.A. No. 7941 prohibits major political parties from
participating in the party-list system. On the contrary, the framers of the Constitution clearly
intended the major political parties to participate in party-list elections through their sectoral
wings. In fact, the members of the Constitutional Commission voted down, 19-22, any
permanent sectoral seats, and in the alternative the reservation of the party-list system to the
sectoral groups.[33] In defining a “party” that participates in party-list elections as either “a
political party or a sectoral party,” R.A. No. 7941 also clearly intended that major political
parties will participate in the party-list elections. Excluding the major political parties in party-
list elections is manifestly against the Constitution, the intent of the Constitutional Commission,
and R.A. No. 7941. This Court cannot engage in socio-political engineering and judicially
legislate the exclusion of major political parties from the party-list elections in patent violation of
the Constitution and the law.

Read together, R.A. No. 7941 and the deliberations of the Constitutional Commission
state that major political parties are allowed to establish, or form coalitions with, sectoral
organizations for electoral or political purposes. There should not be a problem if, for example,

527
the Liberal Party participates in the party-list election through the Kabataang Liberal ng Pilipinas
(KALIPI), its sectoral youth wing. The other major political parties can thus organize, or affiliate
with, their chosen sector or sectors. To further illustrate, the Nacionalista Party can establish a
fisherfolk wing to participate in the party-list election, and this fisherfolk wing can field its
fisherfolk nominees. Kabalikat ng Malayang Pilipino (KAMPI) can do the same for the urban
poor.

The qualifications of party-list nominees are prescribed in Section 9 of R.A. No. 7941:

Qualifications of Party-List Nominees. — No person shall be nominated as


party-list representative unless he is a natural born citizen of the Philippines, a
registered voter, a resident of the Philippines for a period of not less than one (1)
year immediately preceding the day of the elections, able to read and write, bona
fide member of the party or organization which he seeks to represent for at least
ninety (90) days preceding the day of the election, and is at least twenty-five (25)
years of age on the day of the election.

In case of a nominee of the youth sector, he must at least be twenty-five


(25) but not more than thirty (30) years of age on the day of the election. Any
youth sectoral representative who attains the age of thirty (30) during his
term shall be allowed to continue until the expiration of his term.

Under Section 9 of R.A. No. 7941, it is not necessary that the party-list organization’s nominee
“wallow in poverty, destitution and infirmity”[34] as there is no financial status required in the
law. It is enough that the nominee of the sectoral party/organization/coalition belongs to the
marginalized and underrepresented sectors,[35] that is, if the nominee represents the fisherfolk,
he or she must be a fisherfolk, or if the nominee represents the senior citizens, he or she must
be a senior citizen.

Neither the Constitution nor R.A. No. 7941 mandates the filling-up of the entire 20%
allocation of party-list representatives found in the Constitution. The Constitution, in paragraph
1, Section 5 of Article VI, left the determination of the number of the members of the House of
Representatives to Congress: “The House of Representatives shall be composed of not more
than two hundred and fifty members, unless otherwise fixed by law, x x x.” The 20% allocation
of party-list representatives is merely a ceiling; party-list representatives cannot be more than
20% of the members of the House of Representatives. However, we cannot allow the continued
existence of a provision in the law which will systematically prevent the constitutionally allocated
20% party-list representatives from being filled. The three-seat cap, as a limitation to the

528
number of seats that a qualified party-list organization may occupy, remains a valid statutory
device that prevents any party from dominating the party-list elections. Seats for party-list
representatives shall thus be allocated in accordance with the procedure used in Table 3 above.

However, by a vote of 8-7, the Court decided to continue the ruling


in Veterans disallowing major political parties from participating in the party-list elections,
directly or indirectly. Those who voted to continue disallowing major political parties from the
party-list elections joined Chief Justice Reynato S. Puno in his separate opinion. On the formula
to allocate party-list seats, the Court is unanimous in concurring with this ponencia.

WHEREFORE, we PARTIALLY GRANT the petition. We SET ASIDE the Resolution of the
COMELEC dated 3 August 2007 in NBC No. 07-041 (PL) as well as the Resolution dated 9 July
2007 in NBC No. 07-60. We declare unconstitutional the two percent threshold in the
distribution of additional party-list seats. The allocation of additional seats under the Party-List
System shall be in accordance with the procedure used in Table 3 of this Decision. Major
political parties are disallowed from participating in party-list elections. This Decision is
immediately executory. No pronouncement as to costs.

SO ORDERED.

ANTONIO T. CARPIO
Associate Justice

WE CONCUR:

REYNATO S. PUNO
Chief Justice

529
LEONARDO A. QUISUMBING CONSUELO YNARES-SANTIAGO
Associate Justice Associate Justice

MA. ALICIA AUSTRIA-MARTINEZ RENATO C. CORONA


Associate Justice Associate Justice

CONCHITA CARPIO MORALES DANTE O. TINGA


Associate Justice Associate Justice

MINITA V. CHICO-NAZARIO PRESBITERO J. VELASCO, JR.


Associate Justice Associate Justice

ANTONIO EDUARDO B. NACHURA TERESITA J. LEONARDO-DE CASTRO


Associate Justice Associate Justice

ARTURO D. BRION
Associate Justice

530
DIOSDADO M. PERALTA
Associate Justice

LUCAS P. BERSAMIN
Associate Justice

CERTIFICATION

Pursuant to Section 13, Article VIII of the Constitution, I certify that the conclusions in
the above Decision had been reached in consultation before the case was assigned to the writer
of the opinion of the Court.

REYNATO S. PUNO
Chief Justice

531
[1]
Under Rule 65 of the 1997 Rules of Civil Procedure.
[2]
Rollo (G.R. No. 179271), pp. 86-87. Signed by Chairman Benjamin S. Abalos,
Sr., Commissioners Resurreccion Z. Borra, Florentino A. Tuason, Jr., Romeo A.
Brawner, Rene V. Sarmiento, and Nicodemo T. Ferrer.
[3]
Under Rule 65 of the 1997 Rules of Civil Procedure.
[4]
Rollo (G.R. No. 179295), pp. 103-108. Signed by Chairman Benjamin S. Abalos,
Sr., Commissioners Resurreccion Z. Borra, Florentino A. Tuason, Jr., Romeo A. Brawner,
Rene V. Sarmiento, and Nicodemo T. Ferrer.
[5]
396 Phil. 419 (2000).
[6]
Rollo (G.R. No. 179271), pp. 969-986; rollo (G.R. No. 179295), pp. 798-815. Party-List
Canvass Report No. 32, as of 31 August 2007, 6:00 p.m.
[7]
Rollo (G.R. No. 179271), p. 70.
[8]
Rollo (G.R. No. 179271), pp. 88-92.
[9]
Id. at 150-153.
[10]
Id. at 86-87.
[11]
Rollo (G.R. No. 179295), p. 112.
[12]
Rollo (G.R. No. 179271), pp. 158-159. NBC Resolution No. 07-74, 24 July 2007.
[13]
Id. at 160-161. NBC Resolution No. 07-87, 3 August 2007.
[14]
NBC Resolution No. 07-97, 4 September 2007.
[15]
Rollo (G.R. No. 179295), pp. 816-817. This COMELEC certification should have included
An Waray, which was proclaimed on 4 September 2007 under NBC Resolution No. 07-97.
[16]
Rollo (G.R. No. 179271), p. 14.
[17]
Rollo (G.R. No. 179295), pp. 21-22.
[18]
Rollo (G.R. No. 179271), p. 553; rollo (G. R. No. 179295), p. 341.
[19]
Supra note 5 at 424.
[20]
Id. at 446-451. We quote below the discussion in Veterans explaining the First Party
Rule:

Formula for Determining


Additional Seats for the First Party

Now, how do we determine the number of seats the first party is entitled to?
The only basis given by the law is that a party receiving at least two percent of the total
votes shall be entitled to one seat. Proportionally, if the first party were to receive twice
the number of votes of the second party, it should be entitled to twice the latter’s number
of seats and so on. The formula, therefore, for computing the number of seats to which
the first party is entitled is as follows:

Number of votes
532
of first party Proportion of votes of
-------------------- = first party relative to
Total votes for total votes for party-list system
party -list system

If the proportion of votes received by the first party without rounding it off is
equal to at least six percent of the total valid votes cast for all the party list groups, then
the first party shall be entitled to two additional seats or a total of three seats overall. If
the proportion of votes without a rounding off is equal to or greater than four percent, but
less than six percent, then the first party shall have one additional or a total of two seats.
And if the proportion is less than four percent, then the first party shall not be entitled to
any additional seat.

We adopted this six percent bench mark, because the first party is not always
entitled to the maximum number of additional seats. Likewise, it would prevent the
allotment of more than the total number of available seats, such as in an extreme case
wherein 18 or more parties tie for the highest rank and are thus entitled to three seats
each. In such scenario, the number of seats to which all the parties are entitled may
exceed the maximum number of party-list seats reserved in the House of Representatives.

xxx

Note that the above formula will be applicable only in determining the number of
additional seats the first party is entitled to. It cannot be used to determine the number of
additional seats of the other qualified parties. As explained earlier, the use of the same
formula for all would contravene the proportional representation parameter. For example,
a second party obtains six percent of the total number of votes cast. According to the
above formula, the said party would be entitled to two additional seats or a total of three
seats overall. However, if the first party received a significantly higher amount of votes —
say, twenty percent — to grant it the same number of seats as the second party would
violate the statutory mandate of proportional representation, since a party getting only six
percent of the votes will have an equal number of representatives as the one obtaining
twenty percent. The proper solution, therefore, is to grant the first party a total of three
seats; and the party receiving six percent, additional seats in proportion to those of the
first party.

Formula for Additional


Seats of Other Qualified Parties

Step Three The next step is to solve for the number of additional seats that the
other qualified parties are entitled to, based on proportional representation. The formula
is encompassed by the following complex fraction:

No. of votes of
concerned party
------------------
Total no. of votes
533
Additional seats for party-list system No. of additional
for concerned = ----------------------- x seats allocated to
party No. of votes of the first party
first party
--------------
Total no. of votes
for party list system

In simplified form, it is written as follows:

No. of votes of
Additional seats concerned
party No. of additional
for concerned = ------------------ x seats allocated to
party No. of votes of the first party
first party

xxx

Incidentally, if the first party is not entitled to any additional seat, then the ratio
of the number of votes for the other party to that for the first one is multiplied by zero.
The end result would be zero additional seat for each of the other qualified parties as well.

The above formula does not give an exact mathematical representation of the
number of additional seats to be awarded since, in order to be entitled to one additional
seat, an exact whole number is necessary. In fact, most of the actual mathematical
proportions are not whole numbers and are not rounded off for the reasons explained
earlier. To repeat, rounding off may result in the awarding of a number of seats in excess
of that provided by the law. Furthermore, obtaining absolute proportional representation
is restricted by the three-seat-per-party limit to a maximum of two additional slots. An
increase in the maximum number of additional representatives a party may be entitled to
would result in a more accurate proportional representation. But the law itself has set the
limit: only two additional seats. Hence, we need to work within such extant parameter.
[21]
Id. at 475-481.
[22]
The second vote cast by a registered voter is for the party-list candidates as
provided in Section 10 of R.A. No. 7941.
[23]
Rollo (G.R. No. 179271), p. 47.
[24]
Id. at 48.
[25]
Id. at 1076.
[26]
Rollo (G.R. No. 179295), pp. 66-81.
[27]
Rollo (G.R. No. 179271), pp. 969-974; rollo (G.R. No. 179295), pp. 798-803. Party-List
Canvass Report No. 32, as of 31 August 2007, 6:00 p.m.
[28]
Id.
[29]
Proclamation deferred by COMELEC.
[30]
Section 2, R.A. No. 7941.
[31]
The product of the percentage and the remaining available seats of all parties ranked nine
and below is less than one.

534
[32]
II RECORD, CONSTITUTIONAL COMMISSION 256-257 (25 July 1986), 568 (1 August
1986).
[33]
Id. at 584 (1 August 1986). Dissenting opinion of Justice Jose C. Vitug in Ang Bagong
Bayani- OFW Labor Party v. COMELEC, 412 Phil. 308, 350 (2001).
[34]
Ang Bagong Bayani-OFW Labor Party v. COMELEC, 412 Phil. 308, 336 (2001).
[35]
Section 2, R.A. No. 7941.

535
EN BANC

ROSALINDA A. PENERA, G. R. No. 181613


Petitioner,
Present:

PUNO, C.J.,
QUISUMBING,
YNARES-SANTIAGO,
CARPIO,
CORONA,
CARPIO MORALES,
- versus - CHICO-NAZARIO,
VELASCO, JR.,
NACHURA,
LEONARDO-DE CASTRO,
BRION,
PERALTA,
BERSAMIN,
DEL CASTILLO, and
ABAD, JJ.

COMMISSION ON ELECTIONS and Promulgated:


EDGAR T. ANDANAR,
Respondents. September 11, 2009

x--------------------------------------------------x

DECISION

CHICO-NAZARIO, J.:

This Petition for Certiorari with Prayer for the Issuance of a Writ of Preliminary Injunction
and/or Temporary Restraining Order [1] under Rule 65, in relation to Rule 64 of the Rules of
Court, seeks the nullification of the Resolution[2] dated 30 January 2008 of the Commission on
Elections (COMELEC) en banc. Said Resolution denied the Motion for Reconsideration of the
earlier Resolution[3] dated 24 July 2007 of the COMELEC Second Division in SPA No. 07-224,

536
ordering the disqualification of herein petitioner Rosalinda A. Penera (Penera) as a candidate for
the position of mayor of the Municipality of Sta. Monica, Surigao del Norte (Sta. Monica) in the
2007 Synchronized National and Local Elections.

The antecedents of the case, both factual and procedural, are set forth hereunder:

Penera and private respondent Edgar T. Andanar (Andanar) were mayoralty candidates in
Sta. Monica during the 14 May 2007 elections.

On 2 April 2007, Andanar filed before the Office of the Regional Election Director (ORED),
Caraga Region (Region XIII),a Petition for Disqualification[4] against Penera, as well as the
candidates for Vice-Mayor and Sangguniang Bayan who belonged to her political party,[5] for
unlawfully engaging in election campaigning and partisan political activity prior to the
commencement of the campaign period. The petition was docketed as SPA No. 07-224.

Andanar claimed that on 29 March 2007 – a day before the start of the authorized
campaign period on 30 March 2007 – Penera and her partymates went around the
different barangays in Sta. Monica, announcing their candidacies and requesting the people to
vote for them on the day of the elections. Attached to the Petition were the Affidavits of
individuals[6] who witnessed the said incident.

Penera alone filed an Answer[7] to the Petition on 19 April 2007, averring that the charge
of premature campaigning was not true. Although Penera admitted that a motorcade did take
place, she explained that it was simply in accordance with the usual practice in nearby cities and
provinces, where the filing of certificates of candidacy (COCs) was preceded by a motorcade,
which dispersed soon after the completion of such filing. In fact, Penera claimed, in the
motorcade held by her political party, no person made any speech, not even any of the
candidates. Instead, there was only marching music in the background and “a grand standing
for the purpose of raising the hands of the candidates in the motorcade.” Finally, Penera
cited Barroso v. Ampig[8] in her defense, wherein the Court supposedly ruled that a motorcade
held by candidates during the filing of their COCs was not a form of political campaigning.

Also on 19 April 2007, Andanar and Penera appeared with their counsels before the
ORED-Region XIII, where they agreed to submit their position papers and other evidence in
support of their allegations.[9]

537
After the parties filed their respective Position Papers, the records of the case were
transmitted to the COMELEC main office in Manila for adjudication. It was subsequently raffled
to the COMELEC Second Division.

While SPA No. 07-224 was pending before the COMELEC Second Division, the 14 May
2007 elections took place and, as a result thereof, Penera was proclaimed the duly elected
Mayor of Sta. Monica. Penera soon assumed office on 2 July 2002.

On 24 July 2007, the COMELEC Second Division issued its Resolution in SPA No. 07-224,
penned by Commissioner Nicodemo T. Ferrer (Ferrer), which disqualified Penera from continuing
as a mayoralty candidate in Sta. Monica, for engaging in premature campaigning, in violation of
Sections 80 and 68 of the Omnibus Election Code.

The COMELEC Second Division found that:

On the afternoon of 29 March 2007, the 1st [sic] day to file the certificates of
candidacy for local elective positions and a day before the start of the campaign
period for the May 14, 2007 elections – [some of the members of the political party
Partido Padajon Surigao], headed by their mayoralty candidate “Datty” Penera, filed
their respective Certificates of Candidacy before the Municipal Election Officer of
Sta. Monica, Surigao del Norte.

Accompanied by a bevy of supporters, [Penera and her partymates] came to


the municipal COMELEC office on board a convoy of two (2) trucks and an
undetermined number of motorcycles, laden with balloons ad [sic] posters/banners
containing names and pictures and the municipal positions for which they were
seeking election. Installed with [sic] one of the trucks was a public speaker sound
subsystem which broadcast [sic] the intent the [sic] run in the coming
elections. The truck had the posters of Penera attached to it proclaiming his [sic]
candidacy for mayor. The streamer of [Mar Longos, a candidate for the position of
Board Member,] was proudly seen at the vehicle’s side. The group proceeded to
motorcade until the barangays of Bailan, Libertad and as afar [sic] as Mabini almost
nine (9) kilometers from Sta. Monica. [Penera and her partymates] were seen
aboard the vehicles and throwing candies to the residents and onlookers.

Various affidavits and pictures were submitted elucidating the above-


mentioned facts. The above facts were also admitted in the Answer, the Position
Paper and during the hearings conducted for this case, the only defense
propounded by [Penera] is that such acts allegedly do not constitute campaigning
and is therefore not proscribed by the pertinent election laws.

xxxx

538
What we however find disturbing is [Penera’s] reference to the Ampig
Case as the justification for the acts committed by [her]. There is really no
reference to the acts or similar acts committed by [Penera] as having been
considered as not constituting political campaign or partisan political activity. The
issue in that case is whether or not the defect of the lack of a certification against
non-forum [sic] shopping should result to the immediate dismissal of the election
cases filed in that case. There is nothing in said case justifying a motorcade during
the filing of certificates of candidacy. [Penera’s] reliance thereon is therefore
misplaced and of no potency at all.

xxxx

However, the photos submitted by [Andanar] only identified [Penera] and did
not have any notation identifying or indicating any of the other [candidates from
Penera’s party]. It cannot be conclusively proven that the other [candidates from
Penera’s party] were indeed with Penera during the Motorcade. More importantly,
the Answer and the Position Paper contain admissions referring only to
[Penera]. There is therefore no justification for a whole sale [sic] disqualification of
all the [candidates from Penera’s party], as even the petition failed to mention
particularly the participation of the other individual [party members].[10]

The afore-quoted findings of fact led the COMELEC Second Division to decree:

PREMISES CONSIDERED, this Commission resolves to disqualify [Penera]


but absolves the other [candidates from Penera’s party] from violation of section 80
and 68 of the Omnibus Elections [sic] Code.[11]

Commissioner Florentino A. Tuason, Jr. (Tuason) wrote a Separate Opinion[12] on the 24


July 2007 Resolution. Although Commissioner Tuason concurred with the ponente, he stressed
that, indeed, Penera should be made accountable for her actions after the filing of her COC
on 29 March 2007. Prior thereto, there was no candidate yet whose candidacy would have been
enhanced by the premature campaigning.

It was the third member of the COMELEC Second Division, Commissioner Rene V.
Sarmiento (Sarmiento) who put forth a Dissenting Opinion[13] on the 24 July
2007 Resolution. Commissioner Sarmiento believed that the pieces of evidence submitted by
Andanar did not sufficiently establish probable cause that Penera engaged in premature
campaigning, in violation of Sections 80 and 68 of the Omnibus Election Code. The two
photocopied pictures, purporting to be those of Penera, did not clearly reveal what was actually
happening in the truck or who were the passengers thereof. Likewise, the Affidavits seemed to

539
have been prepared and executed by one and the same person because they had similar
sentence construction and form, and they were sworn to before the same attesting officer.

Penera filed before the COMELEC en banc a Motion for Reconsideration[14] of the 24 July
2007 Resolution of the COMELEC Second Division, maintaining that she did not make any
admission on the factual matters stated in the appealed resolution. Penera also contended that
the pictures and Affidavits submitted by Andanar should not have been given any credence. The
pictures were mere photocopies of the originals and lacked the proper authentication, while the
Affidavits were taken ex parte, which would almost always make them incomplete and
inaccurate. Subsequently, Penera filed a Supplemental Motion for Reconsideration,[15] explaining
that supporters spontaneously accompanied Penera and her fellow candidates in filing their
COCs, and the motorcade that took place after the filing was actually part of the dispersal of said
supporters and their transportation back to their respective barangays.

In the Resolution dated 30 January 2008, the COMELEC en banc denied Penera’s Motion
for Reconsideration, disposing thus:

WHEREFORE, this Commission RESOLVES to DENY the instant Motion for


Reconsideration filed by [Penera] for UTTER LACK OF MERIT.[16]

The COMELEC en banc ruled that Penera could no longer advance the arguments set forth
in her Motion for Reconsideration and Supplemental Motion for Reconsideration, given that she
failed to first express and elucidate on the same in her Answer and Position Paper. Penera did
not specifically deny the material averments that the motorcade “went as far as Barangay
Mabini, announcing their candidacy and requesting the people to vote for them on Election Day,”
despite the fact that the same were clearly propounded by Andanar in his Petition for
Disqualification and Position Paper. Therefore, these material averments should be considered
admitted. Although the COMELEC en banc agreed that no undue importance should be given to
sworn statements or affidavits submitted as evidence, this did not mean that such affidavits
should not be given any evidentiary weight at all. Since Penera neither refuted the material
averments in Andanar’s Petition and the Affidavits attached thereto nor submitted countervailing
evidence, then said Affidavits, even if taken ex parte, deserve some degree of importance. The
COMELEC en banc likewise conceded that the pictures submitted by Andanar as evidence would
have been unreliable, but only if they were presented by their lonesome. However, said
pictures, together with Penera’s admissions and the Affidavits of Andanar’s witnesses,
constituted sufficient evidence to establish Penera’s violation of the rule against premature
campaigning. Lastly, the COMELEC en banc accused Penera of deliberately trying to mislead the
540
Commission by citing Barroso, given that the said case was not even remotely applicable to the
case at bar.

Consistent with his previous stand, Commissioner Sarmiento again dissented[17] from
the 30 January 2008 Resolution of the COMELEC en banc. He still believed that Andanar was not
able to adduce substantial evidence that would support the claim of violation of election
laws. Particularly, Commissioner Sarmiento accepted Penera’s explanation that the motorcade
conducted after the filing by Penera and the other candidates of their COCs was merely part of
the dispersal of the spontaneous gathering of their supporters. The incident was only in accord
with normal human social experience.

Still undeterred, Penera filed the instant Petition before us, praying that the Resolutions
dated 24 July 2007 and 30 January 2008 of the COMELEC Second Division and en banc,
respectively, be declared null and void for having been issued with grave abuse of discretion
amounting to lack or excess of jurisdiction.

In a Resolution[18] dated 4 March 2008, we issued a Temporary Restraining Order (TRO),


enjoining the COMELEC from implementing the assailed Resolutions, on the condition that
Penera post a bond in the amount of P5,000.00. We also directed COMELEC and Andanar to
comment on the instant Petition.

After the COMELEC, through the Office of the Solicitor General (OSG), and Andanar filed
their respective Comments[19]on the Petition at bar, we required Penera, in a
Resolution[20] dated 17 June 2008, to file a Reply. However, as no Reply was filed in due time,
we dismissed Penera’s Petition in a Resolution[21] dated 14 October 2008, in accordance with
Rule 56, Section 5(e) of the Rules of Court.[22] Penera subsequently filed an Ex Parte Motion to
Admit Reply,[23] which we treated as a Motion for Reconsideration of the Resolution dated 14
October 2008. On 11 November 2008, we issued another Resolution reinstating Penera’s
Petition.[24]

Penera presents the following issues for our consideration:

I.

Whether or not [Penera] has engaged in an election campaign or partisan political


activity outside the campaign period.

II.
541
Whether the contents of the complaint are deemed admitted for failure of [Penera]
to specifically deny the same.

III.

Whether or not [Andanar] has presented competent and substantial evidence to


justify a conclusion that [Penera] violated Section 80 and 68 of the Omnibus
Election Code.

IV.

Whether or not [the COMELEC] committed grave abuse of discretion amounting to


lack of or in excess of jurisdiction in finding that the act of [Penera] in conducting a
motorcade before the filing of her certificate of candidacy constitutes premature
campaigning.

V.

Whether or not [the COMELEC] committed grave abuse of discretion amounting to


lack of or in excess of jurisdiction when it resolves [sic] to disqualify [Penera]
despite the failure of [Andanar] to present competent, admissible and substantial
evidence to prove [the] violation of Section 68 and 80 of the Omnibus Election
Code.

Penera claims that the COMELEC exercised its discretion despotically, arbitrarily and
whimsically in disqualifying her as a mayoralty candidate in Sta. Monica on the ground that she
engaged in premature campaigning. She asserts that the evidence adduced by Andanar was
grossly insufficient to warrant the ruling of the COMELEC.

Penera insists that the COMELEC Second Division erred in its findings of fact, basically
adopting Andanar’s allegations which, contrary to the belief of the COMELEC Second Division,
Penera never admitted. Penera maintains that the motorcade was spontaneous and unplanned,
and the supporters merely joined Penera and the other candidates from her party along the way
to, as well as within the premises of, the office of the COMELEC Municipal Election
Officer. Andanar’s averments – that after Penera and the other candidates from her party filed
their COCs, they held a motorcade in the different barangays of Sta. Monica, waived their hands
to the public and threw candies to the onlookers – were not supported by competent substantial
evidence. Echoing Commissioner Sarmiento’s dissent from the assailed COMELEC Resolutions,
Penera argues that too much weight and credence were given to the pictures and Affidavits
submitted by Andanar. The declaration by the COMELEC that it was Penera in the pictures is
tenuous and erroneous, as the COMELEC has no personal knowledge of Penera’s identity, and

542
the said pictures do not clearly reveal the faces of the individuals and the contents of the posters
therein. In the same vein, theAffidavits of Andanar’s known supporters, executed almost a
month after Andanar filed his Petition for Disqualification before the ORED-Region XIII, were
obviously prepared and executed by one and the same person, because they have a similar
sentence construction, and computer font and form, and were even sworn to before the same
attesting officer on the same date.

We find no merit in the instant Petition.

The questions of fact

Crystal clear from the above arguments is that Penera is raising only questions of fact in
her Petition presently before us. We do not find any reason to pass upon the same, as this Court
is not a trier of facts. It is not the function of the Court to review, examine and evaluate or
weigh the probative value of the evidence presented. A question of fact would arise in such an
event.

The sole function of a writ of certiorari is to address issues of want of jurisdiction or grave
abuse of discretion, and it does not include a review of the tribunal’s evaluation of the
evidence.[25] Because of its fact-finding facilities and its knowledge derived from actual
experience, the COMELEC is in a peculiarly advantageous position to evaluate, appreciate and
decide on factual questions before it. Factual findings of the COMELEC, based on its own
assessments and duly supported by evidence, are conclusive on this Court, more so in the
absence of a grave abuse of discretion, arbitrariness, fraud, or error of law in the questioned
resolutions. Unless any of these causes are clearly substantiated, the Court will not interfere
with the findings of fact of the COMELEC.[26]

Grave abuse of discretion is such capricious and whimsical exercise of judgment


equivalent to lack of jurisdiction. Mere abuse of discretion is not enough. It must be grave, as
when it is exercised arbitrarily or despotically by reason of passion or personal hostility. The
abuse must be so patent and so gross as to amount to an evasion of a positive duty or to a
virtual refusal to perform the duty enjoined or to act at all in contemplation of law.[27]

We find no grave abuse of discretion amounting to lack or excess of jurisdiction on the


part of the COMELEC Second Division in disqualifying Penera as a mayoralty candidate in Sta.
Monica in the Resolution dated 24 July 2007; and also on the part of the COMELEC en banc in

543
denying Penera’s Motion for Reconsideration on the Resolution dated 30 January 2008. Said
Resolutions are sufficiently supported by substantial evidence, meaning, such evidence as a
reasonable mind might accept as adequate to support a conclusion.[28]

The prohibited act of premature campaigning is defined under Section 80 of the Omnibus
Election Code, to wit:

SECTION 80. Election campaign or partisan political activity outside campaign


period. — It shall be unlawful for any person, whether or not a voter or candidate,
or for any party, or association of persons, to engage in an election campaign or
partisan political activity except during the campaign period: Provided, That political
parties may hold political conventions or meetings to nominate their official
candidates within thirty days before the commencement of the campaign period
and forty-five days for Presidential and Vice-Presidential election. (Emphasis
ours.)

If the commission of the prohibited act of premature campaigning is duly proven, the
consequence of the violation is clearly spelled out in Section 68 of the said Code, which reads:

SECTION. 68. Disqualifications. - Any candidate who, in an action or protest in


which he is a party is declared by final decision of a competent court guilty of, or
found by the Commission of having xxx (e) violated any of Sections 80, 83, 85, 86
and 261, paragraphs d, e, k, v, and cc, subparagraph 6, shall be disqualified from
continuing as a candidate, or if he has been elected, from holding the office. Any
person who is a permanent resident of or an immigrant to a foreign country shall
not be qualified to run for any elective office under this Code, unless said person
has waived his status as permanent resident or immigrant of a foreign country in
accordance with the residence requirement provided for in the election
laws. (Emphases ours.)

In the case at bar, it had been sufficiently established, not just by Andanar’s evidence, but
also those of Penera herself, that Penera and her partymates, after filing their COCs on 29 March
2007, participated in a motorcade which passed through the different barangays of Sta. Monica,
waived their hands to the public, and threw candies to the onlookers.

Indeed, Penera expressly admitted in her Position Paper that:

Respondents actually had a motorcade of only two (2) jeppneys [sic] and ten
(10) motorcycles after filing their Certificate of Candidacy at 3:00 P.M., March 29,
2007 without any speeches made and only one streamer of a board member
Candidate and multi-colored balloons attached to the jeppneys [sic] and
motorcycles.[29] (Emphasis ours.)
544
Additionally, the Joint Affidavit of Marcial Dolar, Allan Llatona, and Renante Platil, attached
to Penera’s Position Paper, gave an even more straightforward account of the events, thus:

1. That on March 29, 2007 at 3:00 P.M. at Sta. Monica, Surigao del Norte,
Mayoralty Candidates Rosalinda CA. Penera [sic] and her parties of four (4)
kagawads filed their certificate of candidacy at the COMELEC Office;

2. That their [sic] was a motorcade consisting of two jeppneys [sic] and 10
motorcycles after actual registration with the COMELEC with jeeps decorated
with balloons and a streamer of Margarito Longos, Board Member Candidate;

3. That the motorcade proceeded to three (3) barangays out of the 11


barangays while supporters were throwing sweet candies to the crowd;

4. That there was merriment and marching music without mention of any name
of the candidates more particularly lead-candidate Rosalinda CA. Penera [sic];

5. That we were in the motorcade on that afternoon only riding in one of the
jeepneys.[30] (Emphases ours.)

In view of the foregoing admissions by Penera and her witnesses, Penera cannot now be
allowed to adopt a conflicting position.

More importantly, the conduct of a motorcade is a form of election campaign or partisan


political activity, falling squarely within the ambit of Section 79(b)(2) of the Omnibus Election
Code, on “[h]olding political caucuses, conferences, meetings, rallies, parades, or other similar
assemblies, for the purpose of soliciting votes and/or undertaking any campaign or propaganda
for or against a candidate[.]” A motorcade is a procession or parade of automobiles or other
motor vehicles.[31] The conduct thereof during election periods by the candidates and their
supporters is a fact that need not be belabored due to its widespread and pervasive
practice. The obvious purpose of the conduct of motorcades is to introduce the candidates and
the positions, to which they seek to be elected, to the voting public; or to make them more
visible so as to facilitate the recognition and recollection of their names in the minds of the
voters come election time. Unmistakably, motorcades are undertaken for no other purpose than
to promote the election of a particular candidate or candidates.

In the instant Petition, Penera never denied that she took part in the conduct of the
motorcade after she filed her COC on the day before the start of the campaign period. She
merely claimed that the same was not undertaken for campaign purposes. Penera proffered the
545
excuse that the motorcade was already part of the dispersal of the supporters who
spontaneously accompanied Penera and her partymates in filing their COCs. The said supporters
were already being transported back to their respective barangays after the COC filing. Penera
stressed that no speech was made by any person, and there was only background marching
music and a “grand standing for the purpose of raising the hands of the candidates in the
motorcade.

We are not convinced.

As we previously noted, Penera and her witnesses admitted that the vehicles, consisting
of two jeepneys and ten motorcycles, were festooned with multi-colored balloons; the motorcade
went around three barangays in Sta. Monica; and Penera and her partymates waved their hands
and threw sweet candies to the crowd. With vehicles, balloons, and even candies on hand,
Penera can hardly persuade us that the motorcade was spontaneous and unplanned.

For violating Section 80 of the Omnibus Election Code, proscribing election campaign or
partisan political activity outside the campaign period, Penera must be disqualified from holding
the office of Mayor of Sta. Monica.

The questions of law

The dissenting opinion, however, raises the legal issue that Section 15 of Republic Act No.
8436, as amended by Republic Act No. 9369, provides a new definition of the term “candidate,”
as a result of which, premature campaigning may no longer be committed.

Under Section 79(a) of the Omnibus Election Code, a candidate is “any person aspiring
for or seeking an elective public office, who has filed a certificate of candidacy by himself or
through an accredited political party, aggroupment, or coalition of parties.”

Republic Act No. 8436,[32] enacted on 22 December 1997, authorized the COMELEC to use
an automated election system for the process of voting, counting of votes, and
canvassing/consolidating the results of the national and local elections. The statute also
mandated the COMELEC to acquire automated counting machines, computer equipment, devices
and materials; and to adopt new electoral forms and printing materials. In particular, Section
11 of Republic Act No. 8436 provided for the specifications of the official ballots to be used in the

546
automated election system and the guidelines for the printing thereof, the relevant portions of
which state:

SECTION 11. Official ballot. - The Commission shall prescribe the size and
form of the official ballot which shall contain the titles of the positions to be filled
and/or the propositions to be voted upon in an initiative, referendum or plebiscite.
Under each position, the names of candidates shall be arranged alphabetically by
surname and uniformly printed using the same type size. A fixed space where the
chairman of the Board of Election inspectors shall affix his/her signature to
authenticate the official ballot shall be provided.

Both sides of the ballots may be used when necessary.

For this purpose, the deadline for the filing of certificate of


candidacy/petition for registration/manifestation to participate in the election shall
not be later than one hundred twenty (120) days before the elections: Provided,
That, any elective official, whether national or local, running for any office other
than the one which he/she is holding in a permanent capacity, except for president
and vice-president, shall be deemed resigned only upon the start of the campaign
period corresponding to the position for which he/she is running:Provided, further,
That, unlawful acts or omissions applicable to a candidate shall take effect upon the
start of the aforesaid campaign period: Provided, finally, That, for purposes of the
May 11, 1998 elections, the deadline for filing of the certificate of candidacy for the
positions of President, Vice President, Senators and candidates under the Party-List
System as well as petitions for registration and/or manifestation to participate in
the Party-List System shall be on February 9, 1998 while the deadline for the filing
of certificate of candidacy for other positions shall be on March 27,
1998. (Emphases ours.)

On 10 February 2007, Republic Act No. 9369[33] took effect. Section 13 of Republic Act
No. 9369 amended Section 11 of Republic Act No. 8436 and renumbered the same as the new
Section 15 of Republic Act No. 8436. The pertinent portions of Section 15 of Republic Act No.
8436, as amended by Republic Act No. 9369, now read:
SECTION.15. Official Ballot. - The Commission shall prescribe the format of the
electronic display and/or the size and form of the official ballot, which shall contain
the titles of the position to be filled and/or the proposition to be voted upon in an
initiative, referendum or plebiscite. Where practicable, electronic displays must be
constructed to present the names of all candidates for the same position in the
same page or screen, otherwise, the electronic displays must be constructed to
present the entire ballot to the voter, in a series of sequential pages, and to ensure
that the voter sees all of the ballot options on all pages before completing his or her
vote and to allow the voter to review and change all ballot choices prior to
completing and casting his or her ballot. Under each position to be filled, the names
of candidates shall be arranged alphabetically by surname and uniformly indicated
using the same type size. The maiden or married name shall be listed in the official
ballot, as preferred by the female candidate. Under each proposition to be vote
upon, the choices should be uniformly indicated using the same font and size.
547
A fixed space where the chairman of the board of election inspector shall affix
her/her signature to authenticate the official ballot shall be provided.
For this purpose, the Commission shall set the deadline for the filing of certificate of
candidacy/petition of registration/manifestation to participate in the election. Any
person who files his certificate of candidacy within this period shall only be
considered as a candidate at the start of the campaign period for which he filed his
certificate of candidacy: Provided, That, unlawful acts or omissions applicable to a
candidate shall effect only upon the start of the aforesaid campaign
period: Provided, finally, That any person holding a public appointive office or
position, including active members of the armed forces, and officers, and
employees in government-owned or-controlled corporations, shall be
considered ipso factor resigned from his/her office and must vacate the same at the
start of the day of the filing of his/her certification of candidacy. (Emphases ours.)

In view of the third paragraph of Section 15 of Republic Act No. 8436, as amended, the
Dissenting Opinion argues that Section 80 of the Omnibus Election Code can not be applied to
the present case since, as the Court held in Lanot v. Commission on Elections,[34] the election
campaign or partisan activity, which constitute the prohibited premature campaigning, should be
designed to promote the election or defeat of a particular candidate or candidates. Under
present election laws, while a person may have filed his/her COC within the prescribed period for
doing so, said person shall not be considered a candidate until the start of the campaign
period. Thus, prior to the start of the campaign period, there can be no election campaign or
partisan political activity designed to promote the election or defeat of a particular candidate to
public office because there is no candidate to speak of.

According to the Dissenting Opinion, even if Penera’s acts before the start of the campaign
period constitute election campaigning or partisan political activities, these are not punishable
under Section 80 of the Omnibus Election Code given that she was not yet a candidate at that
time. On the other hand, Penera’s acts, if committed within the campaign period, when she was
already a candidate, are likewise not covered by Section 80 as this provision punishes only acts
outside the campaign period.

The Dissenting Opinion ultimately concludes that because of Section 15 of Republic Act
No. 8436, as amended, the prohibited act of premature campaigning in Section 80 of the
Omnibus Election Code, is practically impossible to commit at any time.

We disagree. Section 80 of the Omnibus Election Code remains relevant and applicable
despite Section 15 of Republic Act No. 8436, as amended.

548
A close reading of the entire Republic Act No. 9369, which amended Republic Act No.
8436, would readily reveal that that it did not contain an express repeal of Section 80 of the
Omnibus Election Code. An express repeal is one wherein a statute declares, usually in its
repealing clause, that a particular and specific law, identified by its number or title, is
repealed.[35] Absent this specific requirement, an express repeal may not be presumed.

Although the title of Republic Act No. 9369 particularly mentioned the amendment of
Batas Pambansa Blg. 881, or the Omnibus Election Code, to wit:

An Act Amending Republic Act No. 8436, Entitled "An Act Authorizing the
Commission on Elections to Use an Automated Election System x x x, Amending for
the Purpose Batas Pambansa Blg. 881, As Amended x x x. (Emphasis ours.),

said title explicitly mentions, not the repeal, but the amendment of Batas Pambansa Blg.
881. Such fact is indeed very material. Repeal of a law means its complete abrogation by the
enactment of a subsequent statute, whereas the amendment of a statute means an alteration in
the law already existing, leaving some part of the original still standing.[36] Section 80 of the
Omnibus Election Code is not even one of the specific provisions of the said code that were
expressly amended by Republic Act No. 9369.

Additionally, Section 46,[37] the repealing clause of Republic Act No. 9369, states that:

Sec. 46. Repealing Clause. – All laws, presidential decrees, executive


orders, rules and regulations or parts thereof inconsistent with the provisions of this
Act are hereby repealed or modified accordingly.

Section 46 of Republic Act No. 9369 is a general repealing clause. It is a clause which
predicates the intended repeal under the condition that a substantial conflict must be found in
existing and prior acts. The failure to add a specific repealing clause indicates that the intent
was not to repeal any existing law, unless an irreconcilable inconsistency and repugnancy exist
in the terms of the new and old laws. This latter situation falls under the category of an implied
repeal.[38]

Well-settled is the rule in statutory construction that implied repeals are disfavored. In
order to effect a repeal by implication, the later statute must be so irreconcilably inconsistent
and repugnant with the existing law that they cannot be made to reconcile and stand
together. The clearest case possible must be made before the inference of implied repeal may
be drawn, for inconsistency is never presumed. There must be a showing of repugnance clear

549
and convincing in character. The language used in the later statute must be such as to render it
irreconcilable with what had been formerly enacted. An inconsistency that falls short of that
standard does not suffice.[39]

Courts of justice, when confronted with apparently conflicting statutes, should endeavor
to reconcile the same instead of declaring outright the invalidity of one as against the
other. Such alacrity should be avoided. The wise policy is for the judge toharmonize them if
this is possible, bearing in mind that they are equally the handiwork of the same legislature, and
so give effect to both while at the same time also according due respect to a coordinate
department of the government.[40]

To our mind, there is no absolute and irreconcilable incompatibility between Section 15 of


Republic Act No. 8436, as amended, and Section 80 of the Omnibus Election Code, which defines
the prohibited act of premature campaigning. It is possible to harmonize and reconcile these
two provisions and, thus, give effect to both.

The following points are explanatory:

First, Section 80 of the Omnibus Election Code, on premature campaigning, explicitly


provides that “[i]t shall be unlawful for any person, whether or not a voter or candidate, or for
any party, or association of persons, to engage in an election campaign or partisan political
activity, except during the campaign period.” Very simply, premature campaigning may be
committed even by a person who is not a candidate.

For this reason, the plain declaration in Lanot that “[w]hat Section 80 of the Omnibus
Election Code prohibits is ‘an election campaign or partisan political activity’ by a
[41]
‘candidate’ ‘outside’ of the campaign period,” is clearly erroneous.

Second, Section 79(b) of the Omnibus Election Code defines election campaign or
partisan political activity in the following manner:

SECTION 79. Definitions. - As used in this Code:

xxxx

(b) The term "election campaign" or "partisan political activity" refers to an


act designed to promote the election or defeat of a particular candidate or
candidates to a public office which shall include:

550
(1) Forming organizations, associations, clubs, committees or other groups of
persons for the purpose of soliciting votes and/or undertaking any campaign for or
against a candidate;

(2) Holding political caucuses, conferences, meetings, rallies, parades, or


other similar assemblies, for the purpose of soliciting votes and/or undertaking any
campaign or propaganda for or against a candidate;

(3) Making speeches, announcements or commentaries, or holding interviews


for or against the election of any candidate for public office;

(4) Publishing or distributing campaign literature or materials designed to


support or oppose the election of any candidate; or

(5) Directly or indirectly soliciting votes, pledges or support for or against a


candidate.

True, that pursuant to Section 15 of Republic Act No. 8436, as amended, even after the
filing of the COC but before the start of the campaign period, a person is not yet officially
considered a candidate. Nevertheless, a person, upon the filing of his/her
COC, already explicitly declares his/her intention to run as a candidate in the coming
elections. The commission by such a person of any of the acts enumerated under Section 79(b)
of the Omnibus Election Code (i.e., holding rallies or parades, making speeches, etc.) can, thus,
be logically and reasonably construed as for the purpose of promoting his/her intended
candidacy.

When the campaign period starts and said person proceeds with his/her
candidacy, his/her intent turning into actuality, we can already consider his/her acts, after the
filing of his/her COC and prior to the campaign period, as the promotion of his/her election as a
candidate, hence, constituting premature campaigning, for which he/she may be
disqualified. Also, conversely, if said person, for any reason, withdraws his/her COC before the
campaign period, then there is no point to view his/her acts prior to said period as acts for the
promotion of his/her election as a candidate. In the latter case, there can be no premature
campaigning as there is no candidate, whose disqualification may be sought, to begin with.[42]

Third, in connection with the preceding discussion, the line in Section 15 of Republic Act
No. 8436, as amended, which provides that “any unlawful act or omission applicable to a
candidate shall take effect only upon the start of the campaign period,” does not mean that the
acts constituting premature campaigning can only be committed, for which the offender may be
disqualified, during the campaign period. Contrary to the pronouncement in the

551
dissent, nowhere in the said proviso was it stated that campaigning before the start of the
campaign period is lawful, such that the offender may freely carry out the same with impunity.

As previously established, a person, after filing his/her COC but prior to his/her becoming
a candidate (thus, prior to the start of the campaign period), can already commit the acts
described under Section 79(b) of the Omnibus Election Code as election campaign or partisan
political activity. However, only after said person officially becomes a candidate, at the
beginning of the campaign period, can said acts be given effect as premature campaigning under
Section 80 of the Omnibus Election Code. Only after said person officially becomes a candidate,
at the start of the campaign period, can his/her disqualification be sought for acts constituting
premature campaigning. Obviously, it is only at the start of the campaign period, when the
person officially becomes a candidate, that the undue and iniquitous advantages of his/her prior
acts, constituting premature campaigning, shall accrue to his/her benefit. Compared to the
other candidates who are only about to begin their election campaign, a candidate who had
previously engaged in premature campaigning already enjoys an unfair headstart in promoting
his/her candidacy.

As can be gleaned from the foregoing disquisition, harmony in the provisions of Sections
80 and 79 of the Omnibus Election Code, as well as Section 15 of Republic Act No. 8436, as
amended, is not only very possible, but in fact desirable, necessary and consistent with the
legislative intent and policy of the law.

The laudable and exemplary intention behind the prohibition against premature
campaigning, as declared in Chavez v. Commission on Elections,[43] is to level the playing field
for candidates of public office, to equalize the situation between the popular or rich candidates,
on one hand, and lesser-known or poorer candidates, on the other, by preventing the former
from enjoying undue advantage in exposure and publicity on account of their resources and
popularity. The intention for prohibiting premature campaigning, as explained in Chavez, could
not have been significantly altered or affected by Republic Act No. 8436, as amended by
Republic Act No. 9369, the avowed purpose of which is to carry-on the automation of the
election system. Whether the election would be held under the manual or the automated
system, the need for prohibiting premature campaigning – to level the playing field between the
popular or rich candidates, on one hand, and the lesser-known or poorer candidates, on the
other, by allowing them to campaign only within the same limited period – remains.

552
We cannot stress strongly enough that premature campaigning is a pernicious act that is
continuously threatening to undermine the conduct of fair and credible elections in our country,
no matter how great or small the acts constituting the same are. The choice as to who among
the candidates will the voting public bestow the privilege of holding public office should not be
swayed by the shrewd conduct, verging on bad faith, of some individuals who are able to spend
resources to promote their candidacies in advance of the period slated for campaign activities.

Verily, the consequences provided for in Section 68[44] of the Omnibus Election Code for
the commission of the prohibited act of premature campaigning are severe: the candidate who is
declared guilty of committing the offense shall be disqualified from continuing as a candidate, or,
if he/she has been elected, from holding office. Not to mention that said candidate also faces
criminal prosecution for an election offense under Section 262 of the same Code.

The Dissenting Opinion, therefore, should not be too quick to pronounce the
ineffectiveness or repeal of Section 80 of the Omnibus Election Code just because of a change in
the meaning of candidate by Section 15 of Republic Act No. 8436, as amended, primarily, for
administrative purposes. An interpretation should be avoided under which a statute or provision
being construed is defeated, or as otherwise expressed, nullified, destroyed, emasculated,
repealed, explained away, or rendered insignificant, meaningless, inoperative, or
[45]
nugatory. Indeed, not only will the prohibited act of premature campaigning be officially
decriminalized, the value and significance of having a campaign period before the conduct of
elections would also be utterly negated. Any unscrupulous individual with the deepest of
campaign war chests could then afford to spend his/her resources to promote his/her candidacy
well ahead of everyone else. Such is the very evil that the law seeks to prevent. Our
lawmakers could not have intended to cause such an absurd situation.

The Dissenting Opinion attempts to brush aside our preceding arguments by contending
that there is no room for statutory construction in the present case since Section 15 of Republic
Act No. 8436,[46] as amended by Section 13 of Republic Act No. 9369,[47] is crystal clear in its
meaning. We disagree. There would only be no need for statutory construction if there is a
provision in Republic Act No. 8436 or Republic Act No. 9369 that explicitly states that there shall
be no more premature campaigning. But absent the same, our position herein, as well as that
of the Dissenting Opinion, necessarily rest on our respective construction of the legal provisions
involved in this case.

553
Notably, while faulting us for resorting to statutory construction to resolve the instant
case, the Dissenting Opinion itself cites a rule of statutory construction, particularly, that penal
laws should be liberally construed in favor of the offender. The Dissenting Opinion asserts that
because of the third paragraph in Section 15 of Republic Act No. 8436, as amended, the election
offense described in Section 80 of the Omnibus Election Code is practically impossible to commit
at any time and that this flaw in the law, which defines a criminal act, must be construed in
favor of Penera, the offender in the instant case.

The application of the above rule is uncalled for. It was acknowledged in Lanot that a
disqualification case has two aspects: one, electoral;[48] the other, criminal.[49] The instant case
concerns only the electoral aspect of the disqualification case. Any discussion herein on the
matter of Penera’s criminal liability for premature campaigning would be nothing more
thanobiter dictum. More importantly, as heretofore already elaborated upon, Section 15 of
Republic Act No. 8436, as amended, did not expressly or even impliedly repeal Section 80 of the
Omnibus Election Code, and these two provisions, based on legislative intent and policy, can be
harmoniously interpreted and given effect. Thus, there is no flaw created in the law, arising
from Section 15 of Republic Act No. 8436, as amended, which needed to be construed in
Penera’s favor.

The Dissenting Opinion further expresses the fear that pursuant to our “theory,” all the
politicians with “infomercials” prior to the filing of their COCs would be subject to disqualification,
and this would involve practically all the prospective presidential candidates who are now leading
in the surveys.

This fear is utterly unfounded. It is the filing by the person of his/her COC through which
he/she explicitly declares his/her intention to run as a candidate in the coming elections. It is
such declaration which would color the subsequent acts of said person to be election
campaigning or partisan political activities as described under Section 79(b) of the Omnibus
Election Code. It bears to point out that, at this point, no politician has yet submitted his/her
COC. Also, the plain solution to this rather misplaced apprehension is for the politicians
themselves to adhere to the letter and intent of the law and keep within the bounds of fair play
in the pursuit of their candidacies. This would mean that after filing their COCs, the prudent and
proper course for them to take is to wait for the designated start of the campaign period before
they commence their election campaign or partisan political activities. Indeed, such is the only
way for them to avoid disqualification on the ground of premature campaigning. It is not for us

554
to carve out exceptions to the law, much more to decree away the repeal thereof, in order to
accommodate any class of individuals, where no such exception or repeal is warranted.

Lastly, as we have observed at the beginning, Penera’s Petition is essentially grounded on


questions of fact. Penera’s defense against her disqualification, before the COMELEC and this
Court, rests on the arguments that she and her partymates did not actually hold a motorcade;
that their supporters spontaneously accompanied Penera and the other candidates from her
political party when they filed their certificates of candidacy; that the alleged motorcade was
actually the dispersal of the supporters of Penera and the other candidates from her party as
said supporters were dropped off at their respective barangays; and that Andanar was not able
to present competent, admissible, and substantial evidence to prove that Penera committed
premature campaigning. Penera herself never raised the argument that she can no longer be
disqualified for premature campaigning under Section 80, in relation to Section 68, of the
Omnibus Election Code, since the said provisions have already been, in the words of the
Dissenting Opinion, rendered “inapplicable,” “repealed,” and “done away with” by Section 15 of
Republic Act No. 8436, as amended. This legal argument was wholly raised by the Dissenting
Opinion.

As a rule, a party who deliberately adopts a certain theory upon which the case is tried
and decided by the lower court will not be permitted to change theory on appeal. Points of law,
theories, issues, and arguments not brought to the attention of the lower court need not be, and
ordinarily will not be, considered by a reviewing court, as these cannot be raised for the first
time at such late stage. Basic considerations of due process underlie this rule.[50] If we do not
allow and consider the change in theory of a case by a party on appeal, should we not also
refrain from motu proprio adopting a theory which none of the parties even raised before us?

Nonetheless, the questions of fact raised by Penera and questions of law raised by the
Dissenting Opinion must all be resolved against Penera. Penera should be disqualified from
holding office as Mayor of Sta. Monica for having committed premature campaigning when, right
after she filed her COC, but still a day before the start of the campaign period, she took part in a
motorcade, which consisted of two jeepneys and ten motorcycles laden with multi-colored
balloons that went around severalbarangays of Sta. Monica, and gave away candies to the
crowd.

Succession

555
Despite the disqualification of Penera, we cannot grant Andanar’s prayer to be allowed to
assume the position of Mayor of Sta. Monica. The well-established principle is that the
ineligibility of a candidate receiving majority votes does not entitle the candidate receiving the
next highest number of votes to be declared elected.[51]

In this case, the rules on succession under the Local Government Code shall apply, to
wit:

SECTION 44. Permanent Vacancies in the Offices of the Governor, Vice-


Governor, Mayor, and Vice-Mayor. – If a permanent vacancy occurs in the office of
the xxx mayor, the x x x vice-mayor concerned shall become the x x x mayor.

xxxx

For purposes of this Chapter, a permanent vacancy arises when an elective


local official fills a higher vacant office, refuses to assume office, fails to qualify or is
removed from office, voluntarily resigns, or is otherwise permanently incapacitated
to discharge the functions of his office. (Emphases ours.)

Considering Penera’s disqualification from holding office as Mayor of Sta. Monica, the
proclaimed Vice-Mayor shall then succeed as Mayor.

WHEREFORE, premises considered, the instant Petition for Certiorari is


hereby DISMISSED. The Resolutions dated24 July 2007 and 30 January 2008 of the COMELEC
Second Division and en banc, respectively, in SPA No. 07-224 are herebyAFFIRMED. In view of
the disqualification of petitioner Rosalinda A. Penera from running for the office of Mayor of Sta.
Monica, Surigao del Norte, and the resulting permanent vacancy therein, it is
hereby DECLARED that the proclaimed Vice-Mayor is the rightful successor to said office. The
Temporary Restraining Order issued on 4 March 2008 is hereby ORDEREDlifted. Costs against
the petitioner.

SO ORDERED.

MINITA V. CHICO-NAZARIO
Associate Justice

556
WE CONCUR:

REYNATO S. PUNO
Chief Justice

LEONARDO A. QUISUMBING CONSUELO YNARES-SANTIAGO


Associate Justice Associate Justice

ANTONIO T. CARPIO RENATO C. CORONA


Associate Justice Associate Justice

CONCHITA CARPIO MORALES PRESBITERO J. VELASCO, JR.


Associate Justice Associate Justice

ANTONIO EDUARDO B. NACHURA TERESITA J. LEONARDO-DE CASTRO


Associate Justice Associate Justice

557
ARTURO D. BRION DIOSDADO M. PERALTA
Associate Justice Associate Justice

LUCAS P. BERSAMIN MARIANO C. DEL CASTILLO


Associate Justice Associate Justice

ROBERTO A. ABAD
Associate Justice

CERTIFICATION

Pursuant to Article VIII, Section 13 of the Constitution, it is hereby certified that the
conclusions in the above Decision were reached in consultation before the case was assigned to
the writer of the opinion of the Court.

REYNATO S. PUNO
Chief Justice

[1]
Rollo, pp. 3-28.

558
[2]
Penned by Commissioner Nicodemo T. Ferrer with Acting Chairman Resurreccion Z.
Borra and Commissioners Romeo A. Brawner, Florentino A. Tuason, Jr., and Moslemen T.
Macarambon, Sr., concurring, and Commissioner Rene V. Sarmiento, dissenting; rollo, pp.
41-52.
[3]
Penned by Commissioner Nicodemo T. Ferrer with Commissioner Florentino A.
Tuason, Jr., concurring, and Commissioner Rene V. Sarmiento, dissenting; id. at 29-40.
[4]
Id. at 53-54.
[5]
Arcelito Petallo, Renato Virtudazo, Glorina Aparente, Silverio Tajos, Jose Platil,
Medardo Sunico, Edelito Lerio and Sensualito Febra.
[6]
Loreta Billona, Hermilo Botona and Victorino Florendo; rollo, pp. 55-57.
[7]
Id. at 58-59.
[8]
385 Phil. 237 (2000).
[9]
Rollo, p. 127.
[10]
Id. at 30-33.
[11]
Id. at 33.
[12]
Id. at 34-36.
[13]
Id. at 37-40.
[14]
Id. at 97-108.
[15]
Id. at 112-126.
[16]
Id. at 48.
[17]
Id. at 49-52.
[18]
Id. at 138.
[19]
Id. at 161-165, 190-208.
[20]
Id. at 210.
[21]
Id. at 215.
[22]
Sec. 5. Grounds for dismissal of appeal. – The appeal may be dismissed motu
proprio or on motion of the respondent on the following grounds:
xxxx
(e) Failure to comply with any circular, directive or order of the
Supreme Court without justifiable cause;
[23]
Rollo, pp. 217-225.
[24]
Id. at 227-228.
[25]
Bantay Republic Act or BA-RA 7941 v. Commission on Elections, G.R. No. 177271, 4
May 2007, 523 SCRA 11, cited in Cadangen v. Commission on Elections, G.R. No. 177179,
5 June 2009.
[26]
Alvarez v. Commission on Elections, 405 Phil. 950, 959 (2001).
[27]
Cantoria v. Commission on Elections, G.R. No. 162035, 26 November 2004, 444
SCRA 538, 543, cited in Basmala v. Commission on Elections, G.R. No. 176724, 6 October
2008, 567 SCRA 664, 668.
[28]
Doruelo v. Commission on Elections, 218 Phil. 346 (1984).
[29]
Rollo, p. 76.
[30]
Id. at 77.
[31]
Motorcade. Dictionary.com. Dictionary.com Unabridged (v 1.1). Random House, Inc.
http://dictionary.reference.com/browse/motorcade (accessed: July 16, 2009).
[32]
AN ACT AUTHORIZING THE COMMISSION ON ELECTIONS TO USE AN AUTOMATED
ELECTION SYSTEM IN THE MAY 11, 1998 NATIONAL OR LOCAL ELECTIONS AND IN
SUBSEQUENT NATIONAL AND LOCAL ELECTORAL EXERCISES, PROVIDING FUNDS
THEREFOR AND FOR OTHER PURPOSES.

559
[33]
Republic Act No. 9369 is entitled “AN ACT AMENDING REPUBLIC ACT NO. 8436,
ENTITLED ‘AN ACT AUTHORIZING THE COMMISSION ON ELECTIONS TO USE AN
AUTOMATED ELECTION SYSTEM IN THE MAY 11, 1998 NATIONAL OR LOCAL ELECTIONS
AND IN SUBSEQUENT NATIONAL AND LOCAL ELECTORAL EXERCISES, TO ENCOURAGE
TRANSPARENCY, CREDIBILITY, FAIRNESS AND ACCURACY OF ELECTIONS, AMENDING
FOR THE PURPOSE BATAS PAMPANSA BLG. 881, AS AMEMDED, REPUBLIC ACT NO. 7166
AND OTHER RELATED ELECTIONS LAWS, PROVIDING FUNDS THEREFOR AND FOR OTHER
PURPOSES.’" It was published in the newspapers Malaya (26 January 2007) and Business
Mirror (26-27 January 2007). It thus took effect fifteen (15) days after its publication or
on 10 February 2007.
[34]
G.R. No. 164858, 16 November 2006, 507 SCRA 114.
[35]
Mecano v. Commission on Audit, G.R. No. 103982, 11 December 1992, 216 SCRA
500, 504.
[36]
Black’s Law Dictionary (6th Ed [1990]), p. 1299.
[37]
Erroneously cited as Section 47 in the Revised Dissenting Opinion.
[38]
Intia, Jr. v. Commission on Audit, 366 Phil. 273, 290 (1999), citing Mecano v.
Commission on Audit, supra note 35.
[39]
Agujetas v. Court of Appeals, G.R. No. 106560, 23 August 1996, 261 SCRA 17, 34-
35.
[40]
Ty v. Trampe, G.R. No. 117577, 1 December 1995, 250 SCRA 500, 514-515,
citing Gordon v. Veridiano, 11 December 1992, 216 SCRA 500, 505-506.
[41]
G.R. No. 164858, 16 November 2006, 507 SCRA 114, 146.
[42]
This same reasoning holds true for a person (who is neither a candidate nor a voter)
who commits any of the acts described under Section 79(b) of the Omnibus Election Code
for the promotion of the election of another person who has already filed a certificate of
candidacy; the former shall be prosecuted for the election offense of premature
campaigning only in the event that the latter actually continues with his/her candidacy
after the start of the campaign period.
[43]
480 Phil. 915 (2004).
[44]
Sec. 68. Disqualifications. - Any candidate who, in an action or protest in which he is
a party is declared by final decision of a competent court guilty of, or found by the
Commission of having xxx (e) violated any of Sections 80, 83, 85, 86 and 261,
paragraphs d, e, k, v, and cc, subparagraph 6, shall be disqualified from continuing as a
candidate, or if he has been elected, from holding the office. x x x (Emphasis ours.)
[45]
Paras v. Commission on Elections, 332 Phil. 56, 64 (1996).
[46]
AN ACT AUTHORIZING THE COMMISSION ON ELECTIONS TO USE AN AUTOMATED
ELECTION SYSTEM IN THE MAY 11, 1998 NATIONAL OR LOCAL ELECTIONS AND IN
SUBSEQUENT NATIONAL AND LOCAL ELECTORAL EXERCISES, PROVIDING FUNDS
THEREFOR AND FOR OTHER PURPOSES.
[47]
AN ACT AMENDING REPUBLIC ACT NO. 8436, ENTITLED "AN ACT AUTHORIZING THE
COMMISSION ON ELECTIONS TO USE AN AUTOMATED ELECTION SYSTEM IN THE MAY 11,
1998 NATIONAL OR LOCAL ELECTIONS AND IN SUBSEQUENT NATIONAL AND LOCAL
ELECTORAL EXERCISES, TO ENCOURAGE TRANSPARENCY, CREDIBILITY, FAIRNESS AND
ACCURACY OF ELECTIONS, AMENDING FOR THE PURPOSE BATAS PAMBANSA BLG. 881,
AS AMENDED, REPUBLIC ACT NO. 7166 AND OTHER RELATED ELECTIONS LAWS,
PROVIDING FUNDS THEREFOR AND FOR OTHER PURPOSES.”
[48]
The electoral aspect of a disqualification case determines whether the offender
should be disqualified from being a candidate or from holding office. Proceedings are

560
summary in character and require only clear preponderance of evidence. An erring
candidate may be disqualified even without prior determination of probable cause in a
preliminary investigation. The electoral aspect may proceed independently of the criminal
aspect, and vice-versa. (Lanot v. Commission on Elections, supra note 34.)
[49]
The criminal aspect of a disqualification case determines whether there is probable
cause to charge a candidate for an election offense. The prosecutor is the COMELEC,
through its Law Department, which determines whether probable cause exists. If there is
probable cause, the COMELEC, through its Law Department, files the criminal information
before the proper court. Proceedings before the proper court demand a full-blown hearing
and require proof beyond reasonable doubt to convict. A criminal conviction shall result in
the disqualification of the offender, which may even include disqualification from holding a
future public office. (Lanot v. Commission on Elections, supra note 34.)
[50]
Spouses Pasco v. Pison-Arceo Agricultural and Development Corporation, G.R. No.
165501, 28 March 2006, 485 SCRA 514, 523.
[51]
Labo, Jr. v. Commission on Elections, 211 Phil. 297, 312 (1992).

561
EN BANC

KILUSANG MAYO UNO, G.R. No. 167798

NATIONAL FEDERATION OF

LABOR UNIONS-KILUSANG

MAYO UNO (NAFLU-KMU),

JOSELITO V. USTAREZ,

EMILIA P. DAPULANG,

SALVADOR T. CARRANZA,

MARTIN T. CUSTODIO, JR. and

ROQUE M. TAN,

Petitioners,

- versus -

THE DIRECTOR-GENERAL,

NATIONAL ECONOMIC

DEVELOPMENT AUTHORITY,

and THE SECRETARY,

DEPARTMENT OF BUDGET and

MANAGEMENT,

Respondents.

x- - - - - - - - - - - - - - - - - - - - - - - - - - - - - -x

562
BAYAN MUNA Representatives G.R. No. 167930

SATUR C. OCAMPO, TEODORO


A. CASIÑO, and JOEL G. VIRADOR, Present:

GABRIELA WOMEN’S PARTY


Representative LIZA L. MAZA, PANGANIBAN, C.J.,

ANAKPAWIS Representatives PUNO,


RAFAEL V. MARIANO QUISUMBING,

and CRISPIN B. BELTRAN, YNARES-SANTIAGO,

Rep. FRANCIS G. ESCUDERO, SANDOVAL-GUTIERREZ,

Rep. EDUARDO C. ZIALCITA, CARPIO,

Rep. LORENZO R. TAÑADA III, AUSTRIA-MARTINEZ,

DR. CAROL PAGADUAN-ARAULLO CORONA,

and RENATO M. REYES, JR. CARPIO-MORALES,

of BAYAN, MARIE HILAO-ENRIQUEZ CALLEJO, SR.,

of KARAPATAN, ANTONIO L. TINIO AZCUNA,

of ACT, FERDINAND GAITE TINGA,

of COURAGE, GIOVANNI A. TAPANG CHICO-NAZARIO,

of AGHAM, WILFREDO MARBELLA GARCIA, and


of KMP, LANA LINABAN of GABRIELA, VELASCO, Jr., JJ.

AMADO GAT INCIONG,

RENATO CONSTANTINO, JR.,

DEAN PACIFICO H. AGABIN,

SHARON R. DUREMDES of the

NATIONAL COUNCIL OF CHURCHES

IN THE PHILIPPINES, and


BRO. EDMUNDO L. FERNANDEZ (FSC)
563
of the ASSOCIATION OF MAJOR

RELIGIOUS SUPERIORS OF THE

PHILIPPINES (AMRSP),

Petitioners,

- versus -

EDUARDO ERMITA, in his capacity as

Executive Secretary, ROMULO NERI,

in his capacity as Director-General

of the NATIONAL ECONOMIC and

DEVELOPMENT AUTHORITY (NEDA)

and the Administrator of the Promulgated:

NATIONAL STATISTICS OFFICE (NSO),

Respondents. April 19, 2006

x-----------------------------------------------------x

DECISION

CARPIO, J.:

564
This case involves two consolidated petitions for certiorari, prohibition, and mandamus

under Rule 65 of the Rules of Court, seeking the nullification of Executive Order No. 420 (EO

420) on the ground that it is unconstitutional.

EO 420, issued by President Gloria Macapagal-Arroyo on 13 April 2005, reads:

REQUIRING ALL GOVERNMENT AGENCIES AND GOVERNMENT-OWNED AND


CONTROLLED CORPORATIONS TO STREAMLINE AND HARMONIZE THEIR
IDENTIFICATION (ID) SYSTEMS, AND AUTHORIZING FOR SUCH PURPOSE THE
DIRECTOR-GENERAL, NATIONAL ECONOMIC AND DEVELOPMENT AUTHORITY TO
IMPLEMENT THE SAME, AND FOR OTHER PURPOSES

WHEREAS, good governance is a major thrust of this Administration;

WHEREAS, the existing multiple identification systems in government have


created unnecessary and costly redundancies and higher costs to government,
while making it inconvenient for individuals to be holding several identification
cards;

WHEREAS, there is urgent need to streamline and integrate the processes


and issuance of identification cards in government to reduce costs and to provide
greater convenience for those transacting business with government;

WHEREAS, a unified identification system will facilitate private businesses,


enhance the integrity and reliability of government-issued identification cards in
private transactions, and prevent violations of laws involving false names and
identities.

NOW, THEREFORE, I, GLORIA MACAPAGAL-ARROYO, President of the


Republic of the Philippines by virtue of the powers vested in me by law, do hereby
direct the following:

Section 1. Adoption of a unified multi-purpose identification (ID) system for


government. – All government agencies, including government-owned and
controlled corporations, are hereby directed to adopt a unified multi-purpose ID
system to ensure the attainment of the following objectives:

a. To reduce costs and thereby lessen the financial burden on both the
government and the public brought about by the use of multiple ID
cards and the maintenance of redundant database containing the same
or related information;
b. To ensure greater convenience for those transacting business with
the government and those availing of government services;
c. To facilitate private businesses and promote the wider use of
the unified ID card as provided under this executive order;
565
d. To enhance the integrity and reliability of government-issued
ID cards; and
e. To facilitate access to and delivery of quality and effective
government service.

Section 2. Coverage – All government agencies and government-owned and


controlled corporations issuing ID cards to their members or constituents shall be
covered by this executive order.

Section 3. Data requirement for the unified ID system – The data to be


collected and recorded by the participating agencies shall be limited to the
following:

Name

Home Address

Sex

Picture

Signature

Date of Birth

Place of Birth

Marital Status

Names of Parents

Height

Weight

Two index fingers and two thumbmarks

Any prominent distinguishing features like moles and others

Tax Identification Number (TIN)

Provided that a corresponding ID number issued by the participating agency and a common
reference number shall form part of the stored ID data and, together with at least the first five
items listed above, including the print of the right thumbmark, or any of the fingerprints as
collected and stored, shall appear on the face or back of the ID card for visual verification
purposes.

566
Section 4. Authorizing the Director-General, National Economic and
Development Authority, to Harmonize All Government Identification Systems. – The
Director-General, National Economic Development Authority, is hereby authorized
to streamline and harmonize all government ID systems.

Section 5. Functions and responsibilities of the Director-General, National


Economic and Development Authority. – In addition to his organic functions and
responsibilities, the Director-General, National Economic and Development
Authority, shall have the following functions and responsibilities:

a. Adopt within sixty (60) days from the effectivity of this executive
order a unified government ID system containing only such data and
features, as indicated in Section 3 above, to validly establish the
identity of the card holder:
b. Enter into agreements with local governments, through their
respective leagues of governors or mayors, the Commission on
Elections (COMELEC), and with other branches or instrumentalities of
the government, for the purpose of ensuring government-wide
adoption of and support to this effort to streamline the ID systems in
government;
b. Call on any other government agency or institution, or create
sub–committees or technical working groups, to provide such
assistance as may be necessary or required for the effective
performance of its functions; and
d. Promulgate such rules or regulations as may be necessary in
pursuance of the objectives of this executive order.

Section 6. Safeguards. – The Director-General, National Economic and


Development Authority, and the pertinent agencies shall adopt such safeguard as
may be necessary and adequate to ensure that the right to privacy of an individual
takes precedence over efficient public service delivery. Such safeguards shall, as a
minimum, include the following:

a. The data to be recorded and stored, which shall be used only for
purposes of establishing the identity of a person, shall be limited to
those specified in Section 3 of this executive order;
b. In no case shall the collection or compilation of other data in
violation of a person’s right to privacy shall be allowed or tolerated
under this order;
c. Stringent systems of access control to data in the identification
system shall be instituted;
d. Data collected and stored for this purpose shall be kept and treated
as strictly confidential and a personal or written authorization of the
Owner shall be required for access and disclosure of data;
e. The identification card to be issued shall be protected by advanced
security features and cryptographic technology; and

567
f. A written request by the Owner of the identification card shall be
required for any correction or revision of relevant data, or under such
conditions as the participating agency issuing the identification card
shall prescribe.

Section 7. Funding. – Such funds as may be recommended by the


Department of Budget and Management shall be provided to carry out the
objectives of this executive order.

Section 8. Repealing clause. – All executive orders or issuances, or portions


thereof, which are inconsistent with this executive order, are hereby revoked,
amended or modified accordingly.

Section 9. Effectivity. – This executive order shall take effect fifteen (15)
days after its publication in two (2) newspapers of general circulation.

DONE in the City of Manila, this 13th day of April, in the year of Our Lord,
Two Thousand and Five.

Thus, under EO 420, the President directs all government agencies and government-
owned and controlled corporations to adopt a uniform data collection and format for their
existing identification (ID) systems.

Petitioners in G.R. No. 167798 allege that EO 420 is unconstitutional because it


constitutes usurpation of legislative functions by the executive branch of the government.
Furthermore, they allege that EO 420 infringes on the citizen’s right to privacy.[1]

Petitioners in G.R. No. 167930 allege that EO 420 is void based on the following grounds:

1. EO 420 is contrary to law. It completely disregards and violates the


decision of this Honorable Court in Ople v. Torres et al., G.R. No. 127685,
July 23, 1998. It also violates RA 8282 otherwise known as the Social
Security Act of 1997.

2. The Executive has usurped the legislative power of Congress as she has no

power to issue EO 420. Furthermore, the implementation of the EO will use

public funds not appropriated by Congress for that purpose.

568
3. EO 420 violates the constitutional provisions on the right to privacy

(i) It allows access to personal confidential data without the owner’s


consent.

(ii) EO 420 is vague and without adequate safeguards or


penalties for any violation of its provisions.

(iii) There are no compelling reasons that will legitimize the necessity
of EO 420.

4. Granting without conceding that the President may issue EO 420, the
Executive Order was issued without public hearing.

5. EO 420 violates the Constitutional provision on equal protection of laws and


results in the discriminatory treatment of and penalizes those without
ID.[2]

Issues

Essentially, the petitions raise two issues. First, petitioners claim that EO 420 is a
usurpation of legislative power by the President. Second, petitioners claim that EO 420 infringes
on the citizen’s right to privacy.

Respondents question the legal standing of petitioners and the ripeness of the
petitions. Even assuming that petitioners are bereft of legal standing, the Court considers the
issues raised under the circumstances of paramount public concern or of transcendental
significance to the people. The petitions also present a justiciable controversy ripe for judicial
determination because all government entities currently issuing identification cards are

569
mandated to implement EO 420, which petitioners claim is patently unconstitutional. Hence, the
Court takes cognizance of the petitions.

The Court’s Ruling

The petitions are without merit.

On the Alleged Usurpation of Legislative Power

Section 2 of EO 420 provides, “Coverage. – All government agencies and government-


owned and controlled corporations issuing ID cards to their members or constituents shall be
covered by this executive order.” EO 420 applies only to government entities that issue ID
cards as part of their functions under existing laws. These government entities have already
been issuing ID cards even prior to EO 420. Examples of these government entities are the
[3] [4] [5] [6]
GSIS, SSS, Philhealth, Mayor’s Office, LTO,[7] PRC,[8] and similar government entities.

Section 1 of EO 420 directs these government entities to “adopt a unified multi-purpose


ID system.” Thus, all government entities that issue IDs as part of their functions under existing
laws are required to adopt a uniform data collection and formatfor their IDs. Section 1 of EO
420 enumerates the purposes of the uniform data collection and format, namely:

a. To reduce costs and thereby lessen the financial burden on both the
government and the public brought about by the use of multiple ID cards and
the maintenance of redundant database containing the same or related
information;

b. To ensure greater convenience for those transacting business with


the government and those availing of government services;

570
c. To facilitate private businesses and promote the wider use of the
unified ID card as provided under this executive order;

d. To enhance the integrity and reliability of government-issued ID


cards; and

e. To facilitate access to and delivery of quality and effective


government service.

In short, the purposes of the uniform ID data collection and ID format are to reduce costs,
achieve efficiency and reliability, insure compatibility, and provide convenience to the people
served by government entities.

Section 3 of EO 420 limits the data to be collected and recorded under the uniform ID
system to only 14 specific items, namely: (1) Name; (2) Home Address; (3) Sex; (4) Picture;
(5) Signature; (6) Date of Birth; (7) Place of Birth; (8) Marital Status; (9) Name of Parents;
(10) Height; (11) Weight; (12) Two index fingers and two thumbmarks; (13) Any prominent
distinguishing features like moles or others; and (14) Tax Identification Number.

These limited and specific data are the usual data required for personal identification by
government entities, and even by the private sector. Any one who applies for or renews a
driver’s license provides to the LTO all these 14 specific data.

At present, government entities like LTO require considerably more data from applicants
for identification purposes. EO 420 will reduce the data required to be collected and recorded in
the ID databases of the government entities. Government entities cannot collect or record data,
for identification purposes, other than the 14 specific data.

Various laws allow several government entities to collect and record data for their ID
systems, either expressly or impliedly by the nature of the functions of these government

571
entities. Under their existing ID systems, some government entities collect and record more
data than what EO 420 allows. At present, the data collected and recorded by government
entities are disparate, and the IDs they issue are dissimilar.

In the case of the Supreme Court,[9] the IDs that the Court issues to all its employees,
including the Justices, contain 15 specific data, namely: (1) Name; (2) Picture; (3) Position;
(4) Office Code Number; (5) ID Number; (6) Height; (7) Weight; (8) Complexion; (9) Color of
Hair; (10) Blood Type; (11) Right Thumbmark; (12) Tax Identification Number; (13) GSIS
Policy Number; (14) Name and Address of Person to be Notified in Case of Emergency; and (15)
Signature. If we consider that the picture in the ID can generally also show the sex of the
employee, the Court’s ID actually contains 16 data.

In contrast, the uniform ID format under Section 3 of EO 420 requires only “the first five
items listed” in Section 3, plus the fingerprint, agency number and the common reference
number, or only eight specific data. Thus, at present, the Supreme Court’s ID contains far more
data than the proposed uniform ID for government entities under EO 420. The nature of the
data contained in the Supreme Court ID is also far more financially sensitive, specifically the Tax
Identification Number.

Making the data collection and recording of government entities unified, and making their
ID formats uniform, will admittedly achieve substantial benefits. These benefits are savings in
terms of procurement of equipment and supplies, compatibility in systems as to hardware and
software, ease of verification and thus increased reliability of data, and the user-friendliness of a
single ID format for all government entities.

There is no dispute that government entities can individually limit the collection and
recording of their data to the 14 specific items in Section 3 of EO 420. There is also no dispute
that these government entities can individually adopt the ID format as specified in Section 3 of
EO 420. Such an act is certainly within the authority of the heads or governing boards of the
government entities that are already authorized under existing laws to issue IDs.

A unified ID system for all these government entities can be achieved in either of two
ways. First, the heads of these existing government entities can enter into a memorandum of
572
agreement making their systems uniform. If the government entities can individually adopt a
format for their own ID pursuant to their regular functions under existing laws, they can also
adopt by mutual agreement a uniform ID format, especially if the uniform format will result in
substantial savings, greater efficiency, and optimum compatibility. This is purely an
administrative matter, and does not involve the exercise of legislative power.

Second, the President may by executive or administrative order direct the government
entities under the Executive department to adopt a uniform ID data collection and
format. Section 17, Article VII of the 1987 Constitution provides that the “President shall have
control of all executive departments, bureaus and offices.” The same Section also mandates the
President to “ensure that the laws be faithfully executed.”

Certainly, under this constitutional power of control the President can direct all
government entities, in the exercise of their functions under existing laws, to adopt a uniform ID
data collection and ID format to achieve savings, efficiency, reliability, compatibility, and
convenience to the public. The President’s constitutional power of control is self-executing and
does not need any implementing legislation.

Of course, the President’s power of control is limited to the Executive branch of


government and does not extend to the Judiciary or to the independent constitutional
commissions. Thus, EO 420 does not apply to the Judiciary, or to the COMELEC which under
existing laws is also authorized to issue voter’s ID cards.[10] This only shows that EO 420 does
not establish a national ID system because legislation is needed to establish a single ID system
that is compulsory for all branches of government.

The Constitution also mandates the President to ensure that the laws are faithfully
executed. There are several laws mandating government entities to reduce costs, increase
efficiency, and in general, improve public services.[11] The adoption of a uniform ID data
collection and format under EO 420 is designed to reduce costs, increase efficiency, and in
general, improve public services. Thus, in issuing EO 420, the President is simply performing
the constitutional duty to ensure that the laws are faithfully executed.

573
Clearly, EO 420 is well within the constitutional power of the President to
promulgate. The President has not usurped legislative power in issuing EO 420. EO 420 is an
exercise of Executive power – the President’s constitutional power of control over the Executive
department. EO 420 is also compliance by the President of the constitutional duty to ensure
that the laws are faithfully executed.

Legislative power is the authority to make laws and to alter or repeal them. In issuing
EO 420, the President did not make, alter or repeal any law but merely implemented and
executed existing laws. EO 420 reduces costs, as well as insures efficiency, reliability,
compatibility and user-friendliness in the implementation of current ID systems of government
entities under existing laws. Thus, EO 420 is simply an executive issuance and not an act of
legislation.

The act of issuing ID cards and collecting the necessary personal data for imprinting on
the ID card does not require legislation. Private employers routinely issue ID cards to their
employees. Private and public schools also routinely issue ID cards to their students. Even
private clubs and associations issue ID cards to their members. The purpose of all these ID
cards is simply to insure the proper identification of a person as an employee, student, or
member of a club. These ID cards, although imposed as a condition for exercising a privilege,
are voluntary because a person is not compelled to be an employee, student or member of a
club.

What require legislation are three aspects of a government maintained ID card


system. First, when the implementation of an ID card system requires a special appropriation
because there is no existing appropriation for such purpose. Second, when the ID card system
is compulsory on all branches of government, including the independent constitutional
commissions, as well as compulsory on all citizens whether they have a use for the ID card or
not. Third, when the ID card system requires the collection and recording of personal data
beyond what is routinely or usually required for such purpose, such that the citizen’s right to
privacy is infringed.

In the present case, EO 420 does not require any special appropriation because the
existing ID card systems of government entities covered by EO 420 have the proper
574
appropriation or funding. EO 420 is not compulsory on all branches of government and is not
compulsory on all citizens. EO 420 requires a very narrow and focused collection and recording
of personal data while safeguarding the confidentiality of such data. In fact, the data collected
and recorded under EO 420 are far less than the data collected and recorded under the ID
systems existing prior to EO 420.

EO 420 does not establish a national ID card system. EO 420 does not compel all citizens
to have an ID card. EO 420 applies only to government entities that under existing laws are
already collecting data and issuing ID cards as part of their governmental functions. Every
government entity that presently issues an ID card will still issue its own ID card under its own
name. The only difference is that the ID card will contain only the five data specified in Section 3
of EO 420, plus the fingerprint, the agency ID number, and the common reference number
which is needed for cross-verification to ensure integrity and reliability of identification.

This Court should not interfere how government entities under the Executive department
should undertake cost savings, achieve efficiency in operations, insure compatibility of
equipment and systems, and provide user-friendly service to the public. The collection of ID data
and issuance of ID cards are day-to-day functions of many government entities under existing
laws. Even the Supreme Court has its own ID system for employees of the Court and all first and
second level courts. The Court is even trying to unify its ID system with those of the appellate
courts, namely the Court of Appeals, Sandiganbayan and Court of Tax Appeals.

There is nothing legislative about unifying existing ID systems of all courts within the
Judiciary. The same is true for government entities under the Executive department. If
government entities under the Executive department decide to unify theirexisting ID data
collection and ID card issuance systems to achieve savings, efficiency, compatibility and
convenience, such act does not involve the exercise of any legislative power. Thus, the issuance
of EO 420 does not constitute usurpation of legislative power.

On the Alleged Infringement of the Right to Privacy

575
All these years, the GSIS, SSS, LTO, Philhealth and other government entities have been
issuing ID cards in the performance of their governmental functions. There have been no
complaints from citizens that the ID cards of these government entities violate their right to
privacy. There have also been no complaints of abuse by these government entities in the
collection and recording of personal identification data.

In fact, petitioners in the present cases do not claim that the ID systems of government
entities prior to EO 420 violate their right to privacy. Since petitioners do not make such claim,
they even have less basis to complain against the unified ID system under EO 420. The data
collected and stored for the unified ID system under EO 420 will be limited to only 14 specific
data, and the ID card itself will show only eight specific data. The data collection, recording and
ID card system under EO 420 will even require less data collected, stored and revealed than
under the disparate systems prior to EO 420.

Prior to EO 420, government entities had a free hand in determining the kind, nature and
extent of data to be collected and stored for their ID systems. Under EO 420, government
entities can collect and record only the 14 specific data mentioned in Section 3 of EO 420. In
addition, government entities can show in their ID cards only eight of these specific data, seven
less data than what the Supreme Court’s ID shows.

Also, prior to EO 420, there was no executive issuance to government entities prescribing
safeguards on the collection, recording, and disclosure of personal identification data to protect
the right to privacy. Now, under Section 5 of EO 420, the following safeguards are instituted:

a. The data to be recorded and stored, which shall be used only for
purposes of establishing the identity of a person, shall be limited to those
specified in Section 3 of this executive order;

b. In no case shall the collection or compilation of other data in violation


of a person’s right to privacy be allowed or tolerated under this order;

576
c. Stringent systems of access control to data in the identification
system shall be instituted;

d. Data collected and stored for this purpose shall be kept and treated
as strictly confidential and a personal or written authorization of the Owner
shall be required for access and disclosure of data;

e. The identification card to be issued shall be protected by advanced


security features and cryptographic technology;

f. A written request by the Owner of the identification card shall be


required for any correction or revision of relevant data, or under such
conditions as the participating agency issuing the identification card shall
prescribe.

On its face, EO 420 shows no constitutional infirmity because it even narrowly limits the
data that can be collected, recorded and shown compared to the existing ID systems of
government entities. EO 420 further provides strict safeguards to protect the confidentiality of
the data collected, in contrast to the prior ID systems which are bereft of strict administrative
safeguards.

The right to privacy does not bar the adoption of reasonable ID systems by government
entities. Some one hundred countries have compulsory national ID systems, including
democracies such as Spain, France, Germany, Belgium, Greece, Luxembourg, and
Portugal. Other countries which do not have national ID systems, like the United States,
Canada, Australia, New Zealand, Ireland, the Nordic Countries and Sweden, have sectoral cards
for health, social or other public services.[12] Even with EO 420, the Philippines will still fall under
the countries that do not have compulsory national ID systems but allow only sectoral cards for
social security, health services, and other specific purposes.

577
Without a reliable ID system, government entities like GSIS, SSS, Philhealth, and LTO
cannot perform effectively and efficiently their mandated functions under existing laws. Without
a reliable ID system, GSIS, SSS, Philhealth and similar government entities stand to suffer
substantial losses arising from false names and identities. The integrity of the LTO’s licensing
system will suffer in the absence of a reliable ID system.

The dissenting opinion cites three American decisions on the right to privacy,
namely, Griswold v. Connecticut,[13] U.S.Justice Department v. Reporters Committee for
[14] [15]
Freedom of the Press, and Whalen v. Roe. The last two decisions actually support the
validity of EO 420, while the first is inapplicable to the present case.

In Griswold, the U.S. Supreme Court declared unconstitutional a state law that prohibited
the use and distribution of contraceptives because enforcement of the law would allow the police
entry into the bedrooms of married couples. Declared the U.S. Supreme Court: “Would we allow
the police to search the sacred precincts of the marital bedrooms for telltale signs of the use of
contraceptives? The very idea is repulsive to the notions of privacy surrounding the marriage
relationship.” Because the facts and the issue involved in Griswold are materially different from
the present case, Griswold has no persuasive bearing on the present case.

In U.S. Justice Department, the issue was not whether the State could collect and store
information on individuals from public records nationwide but whether the State could withhold
such information from the press. The premise of the issue inU.S. Justice Department is that the
State can collect and store in a central database information on citizens gathered from public
records across the country. In fact, the law authorized the Department of Justice to collect and
preserve fingerprints and other criminal identification records nationwide. The law also
authorized the Department of Justice to exchange such information with “officials of States,
cities and other institutions.” The Department of Justice treated such information as
confidential. A CBS news correspondent and the Reporters Committee demanded the criminal
records of four members of a family pursuant to the Freedom of Information Act. The U.S.
Supreme Court ruled that the Freedom of Information Act expressly exempts release of
information that would “constitute an unwarranted invasion of personal privacy,” and the
information demanded falls under that category of exempt information.

578
With the exception of the 8 specific data shown on the ID card, the personal data
collected and recorded under EO 420 are treated as “strictly confidential” under Section 6(d) of
EO 420. These data are not only strictly confidential but alsopersonal matters. Section 7,
Article III of the 1987 Constitution grants the “right of the people to information on matters of
public concern.” Personal matters are exempt or outside the coverage of the people’s right to
information on matters of public concern. The data treated as “strictly confidential” under EO
420 being private matters and not matters of public concern, these data cannot be released to
the public or the press. Thus, the ruling in U.S. Justice Department does not collide with EO
420 but actually supports the validity EO
420.

Whalen v. Roe is the leading American case on the constitutional protection for control
over information. InWhalen, the U.S. Supreme Court upheld the validity of a New York law that
required doctors to furnish the government reports identifying patients who received prescription
drugs that have a potential for abuse. The government maintained a central computerized
database containing the names and addresses of the patients, as well as the identity of the
prescribing doctors. The law was assailed because the database allegedly infringed the right to
privacy of individuals who want to keep their personal matters confidential. The U.S. Supreme
Court rejected the privacy claim, and declared:

Disclosures of private medical information to doctors, to hospital personnel, to


insurance companies, and to public health agencies are often an essential part of
modern medical practice even when the disclosure may reflect unfavorably on the
character of the patient. Requiring such disclosures to representatives of the State
having responsibility for the health of the community does not automatically amount
to an impermissible invasion of privacy. (Emphasis supplied)

Compared to the personal medical data required for disclosure to the New York State
in Whalen, the 14 specific data required for disclosure to the Philippine government under EO
420 are far less sensitive and far less personal. In fact, the 14 specific data required under EO
420 are routine data for ID systems, unlike the sensitive and potentially embarrassing medical
records of patients taking prescription drugs. Whalen, therefore, carries persuasive force for
upholding the constitutionality of EO 420 as non-violative of the right to privacy.

579
Subsequent U.S. Supreme Court decisions have reiterated Whalen. In Planned
[16]
Parenthood of Central Missouri v. Danforth, the U.S. Supreme Court upheld the validity of a
law that required doctors performing abortions to fill up forms, maintain records for seven years,
and allow the inspection of such records by public health officials. The U.S. Supreme Court ruled
that “recordkeeping and reporting requirements that are reasonably directed to the preservation
of maternal health and that properly respect a patient’s confidentiality and privacy are
permissible.”

Again, in Planned Parenthood of Southeastern Pennsylvania v. Casey,[17] the U.S.


Supreme Court upheld a law that required doctors performing an abortion to file a report to the
government that included the doctor’s name, the woman’s age, the number of prior pregnancies
and abortions that the woman had, the medical complications from the abortion, the weight of
the fetus, and the marital status of the woman. In case of state-funded institutions, the law
made such information publicly available. In Casey, the U.S. Supreme Court stated: “The
collection of information with respect to actual patients is a vital element of medical research,
and so it cannot be said that the requirements serve no purpose other than to make abortion
more difficult.”

Compared to the disclosure requirements of personal data that the U.S. Supreme Court
have upheld in Whalen, Danforthand Casey as not violative of the right to privacy, the
disclosure requirements under EO 420 are far benign and cannot therefore constitute violation of
the right to privacy. EO 420 requires disclosure of 14 personal data that are routine for ID
purposes, data that cannot possibly embarrass or humiliate anyone.

Petitioners have not shown how EO 420 will violate their right to privacy. Petitioners
cannot show such violation by a mere facial examination of EO 420 because EO 420 narrowly
draws the data collection, recording and exhibition while prescribing comprehensive
[18]
safeguards. Ople v. Torres is not authority to hold that EO 420 violates the right to privacy
because in that case the assailed executive issuance, broadly drawn and devoid of safeguards,
was annulled solely on the ground that the subject matter required legislation. As then
Associate Justice, now Chief Justice Artemio V. Panganiban noted in his concurring opinion
in Ople v. Torres, “The voting is decisive only on the need for appropriate legislation, and it is
only on this ground that the petition is granted by this Court.”

580
EO 420 applies only to government entities that already maintain ID systems and issue ID
cards pursuant to their regular functions under existing laws. EO 420 does not grant such
government entities any power that they do not already possess under existing laws. In
contrast, the assailed executive issuance in Ople v. Torres sought to establish a
[19]
“National ComputerizedIdentification Reference System,” a national ID system that did not
exist prior to the assailed executive issuance. Obviously, a national ID card system requires
legislation because it creates a new national data collection and card issuance system where
none existed before.

In the present case, EO 420 does not establish a national ID system but makes the
existing sectoral card systems of government entities like GSIS, SSS, Philhealth and LTO less
costly, more efficient, reliable and user-friendly to the public. Hence, EO 420 is a proper
subject of executive issuance under the President’s constitutional power of control over
government entities in the Executive department, as well as under the President’s constitutional
duty to ensure that laws are faithfully executed.

WHEREFORE, the petitions are DISMISSED. Executive Order No. 420 is declared VALID.

SO ORDERED.

ANTONIO T. CARPIO

Associate Justice

WE CONCUR:

ARTEMIO V. PANGANIBAN

581
Chief Justice

(On leave)

REYNATO S. PUNO LEONARDO A. QUISUMBING


Associate Justice
Associate Justice

CONSUELO YNARES-SANTIAGO ANGELINA SANDOVAL-GUTIERREZ

Associate Justice Associate Justice

MA. ALICIA AUSTRIA-MARTINEZ RENATO C. CORONA


Associate Justice
Associate Justice

CONCHITA CARPIO MORALES ROMEO J. CALLEJO, SR.


Associate Justice
Associate Justice

582
ADOLFO S. AZCUNA DANTE O. TINGA
Associate Justice Associate Justice

MINITA V. CHICO-NAZARIO CANCIO C. GARCIA


Associate Justice Associate Justice

PRESBITERO J. VELASCO, JR.


Associate Justice

CERTIFICATION

Pursuant to Section 13, Article VIII of the Constitution, I certify that the conclusions in
the above Decision had been reached in consultation before the case was assigned to the writer
of the opinion of the Court.

ARTEMIO V. PANGANIBAN

Chief Justice

583
[1]
Rollo, pp. 6-7.
[2]
Rollo, pp. 15-16.
[3]
Government Service Insurance System.
[4]
Social Security System.
[5]
Philippine Health Insurance Corporation. Section 8 of RA No. 7875 (National Health
Insurance Act) provides: “SECTION 8. Health Insurance ID Card. — In conjunction with
the enrollment provided above, the Corporation through its local office shall issue a health
insurance ID which shall be used for purposes of identification, eligibility verification, and
utilization recording. The issuance of this ID card shall be accompanied by a clear
explanation to the enrollee of his rights, privileges and obligations as a member. A list of
health care providers accredited by the Local Health Insurance Office shall likewise be
attached thereto.”
[6]
Section 4(m) of RA No. 7432 (Senior Citizens Act), as expanded by RA No. 9257,
provides:
“In the availment of the privileges mentioned above, the senior citizen or elderly person
may submit as proof of his/her entitlement thereto any of the following:
(a) an ID issued by the city or municipal mayor or of the barangay captain of
the place where the senior citizen or the elderly resides;
(b) the passport of the elderly person or senior citizen concerned; and
x x x.”
[7]
Land Transportation Office. Section 24 of RA No. 4136 (Land Transportation and
Traffic Code, as amended) provides: “SECTION 24. Use of Driver’s License and
Identification Card. — Every license issued under the provisions of this Act to any driver
shall entitle the holder thereof, while the same is valid and effective, to operate motor
vehicles described in such license: Provided, however, That every licensed professional
driver, before operating a public utility vehicle registered under classification (b) of
Section seven hereof, as amended by Batas Pambansa Bilang 74, shall secure from the
Director, upon payment of the sum of five pesos, a driver’s identification card which he
shall, at all times while so operating a public utility vehicle, display in plain sight in the
vehicle being operated. The identification card shall be issued simultaneously with the
license.
[8]
Professional Regulation Commission. Section 19 of RA No. 9292 (Electronics
Engineering Law of 2004) provides: “SECTION 19. Issuance of the Certificate of
Registration and Professional Identification Card. — x x x
A Professional Identification Card bearing the registration number, date of registration,
duly signed by the Chairperson of the Commission, shall likewise be issued to every
registrant who has paid the prescribed fee. This identification card will serve as evidence
that the holder thereof is duly registered with the Commission.” See also Section 19
of RA No. 9200 (Philippine Geodetic Engineering Act of 1998).

584
[9]
Like GSIS and SSS, there is no express provision of law authorizing the Supreme
Court to issue ID cards to its employees. However, any employer necessarily must issue
ID cards to its employees for several purposes. First, an ID card is necessary to identify
those who may enter the premises of the employer, especially in areas where non-
employees are prohibited. Second, an ID or reference number is necessary for a
computerized payroll system. Third, an ID card is necessary to identify those who can
withdraw stock or borrow property of the employer. In the case of GSIS and SSS, they
issue ID cards not only to their employees but also to their members. Like any mutual
association, GSIS and SSS can issue membership cards to their members who contribute
to the trust funds they administer and who are entitled to the corresponding benefits.
[10]
Sections 126 and 128 of the Omnibus Election Code (BP Blg. 881)
provide: “SECTION 126. Registration of voters. — On the seventh and sixth Saturdays
before a regular election or on the second Saturday following the day of the proclamation
calling for a new special election, plebiscite or referendum, any person desiring to be
registered as a voter shall accomplish in triplicate before the board of election inspectors a
voter’s affidavit in which shall be stated the following data:
(a) Name, surname, middle name, maternal surname;
(b) Date and place of birth;
(c) Citizenship;
(d) Periods of residence in the Philippines and in the place of registration;
(e) Exact address with the name of the street and house number or in case there is
none, a brief description of the locality and the place;
(f) A statement that the applicant has not been previously registered, otherwise he shall
be required to attach a sworn application for cancellation of his previous registration;
and
(g) Such other information or data which may be required by the Commission.
The voter’s affidavit shall also contain three specimens of the applicant’s signature and
clear and legible prints of his left and right hand thumbmarks and shall be sworn to and
filed together with four copies of the latest identification photograph to be supplied by the
applicant.
The oath of the applicant shall include a statement that he does not have any of the
disqualifications of a voter and that he has not been previously registered in the precinct
or in any other precinct.
Before the applicant accomplishes his voter’s affidavit, the board of election inspectors
shall appraise the applicant of the qualifications and disqualifications prescribed by law for
a voter. It shall also see to it that the accomplished voter's affidavit contains all the data
therein required and that the applicant's specimen signatures, the prints of his left and
right hand thumbmarks and his photograph are properly affixed in each of the voter’s
affidavit.
xxx
SECTION 128. Voter’s identification. — The identification card issued to the voter shall
serve and be considered as a document for the identification of each registered voter:
Provided, however, That if the voter’s identity is challenged on election day and he cannot
present his voter identification card, his identity may be established by the specimen
signatures, the photograph or the fingerprints in his voter’s affidavit in the book of voters.
No extra or duplicate copy of the voter identification card shall be prepared and issued
except upon authority of the Commission.
Each identification card shall bear the name and the address of the voter, his date of
birth, sex, civil status, occupation, his photograph, thumbmark, the city or municipality

585
and number of the polling place where he is registered, his signature, his voter serial
number and the signature of the chairman of the board of election inspectors.
Any voter previously registered under the provisions of Presidential Decree Numbered
1896 who desires to secure a voter identification card shall, on any registration day,
provide four copies of his latest identification photograph to the board of election
inspectors which upon receipt thereof shall affix one copy thereof to the voter’s affidavit in
the book of voters, one copy to the voter identification card to be issued to the voter and
transmit through the election registrar, one copy each to the provincial election supervisor
and the Commission to be respectively attached to the voter's affidavit in their respective
custody.”
[11]
Section 48, Chapter 5, Book VI of the Revised Administrative Code of 1987 provides:
“SECTION 48. Cost Reduction. — Each head of a department, bureau, office or agency
shall implement a cost reduction program for his department, bureau, office or agency for
the purpose of reducing cost of operations and shall submit to the President reports on
the results of the implementation thereof. The Department of Budget shall provide
technical and other necessary assistance in the design and implementation of cost
reduction activities. An incentive award not exceeding one month’s salary may be granted
to any official or employee whose suggestion for cost reduction has been adopted and
shall have actually resulted in cost reduction, payable from the savings resulting
therefrom.

Similarly, Section 54 of PD No. 1177 (Budget Reform Decree of 1977)


provides: “SECTION 54. Cost Reduction. — Each head of department, bureau, office or
agency shall implement a cost reduction program for his department, bureau, office or
agency for the purpose of reducing cost of operations and shall submit to the President
reports on the results of the implementation thereof. The Budget Commission shall
provide technical and other necessary assistance in the design and implementation of cost
reduction activities. An incentive award not exceeding one month's salary may be granted
to any official or employee whose suggestion for cost reduction has been adopted and
shall have actually resulted in cost reduction, payable from the savings resulting
therefrom.

In addition, the annual General Appropriations Act contains similar provisions mandating
cost reduction in all government offices.

Moreover, Section (a) of RA No. 6713 (Code of Conduct and Ethical Standards for Public
Officials and Employees) also provides: “Commitment to public interest. – x x x All
government resources and powers of their respective offices must be employed and used
efficiently, effectively, honestly and economically, particularly to avoid wastage in public
funds and revenues.” (Emphasis supplied)
[12]
Identity Cards, Privacy International,
http://www.privacy.org/pi/activities/idcard/idcard_faq.html.
[13]
381 U.S. 479 (1965).
[14]
489 U.S. 749 (1989).
[15]
429 U.S. 589 (1977).
[16]
428 U.S. 52 (1976).
[17]
505 U.S. 833 (1992).
[18]
354 Phil. 948 (1998).

586
[19]
Section 1 of Administrative Order No. 308 dated 12 December 1996 states: “SEC
1. Establishment of a National Computerized Identification Reference System. – A
decentralized Identification Reference System among the key basic services and social
security providers is hereby established.”

587
Republic of the Philippines
Supreme Court
Manila

EN BANC

PHARMACEUTICAL and HEALTH G.R. NO. 173034


CARE ASSOCIATION of the
PHILIPPINES,
Petitioner,
Present:

PUNO, C.J.
QUISUMBING,
YNARES-SANTIAGO,
SANDOVAL-GUTIERREZ,
CARPIO,
- versus - AUSTRIA-MARTINEZ,
CORONA,
CARPIO-MORALES,
AZCUNA,
TINGA,
CHICO-NAZARIO,
GARCIA,
VELASCO, JR.,
NACHURA, and
REYES, JJ.

HEALTH SECRETARY
FRANCISCO T. DUQUE III;
HEALTH UNDERSECRETARIES
DR. ETHELYN P. NIETO,
DR. MARGARITA M. GALON,
ATTY. ALEXANDER A. PADILLA,
& DR. JADE F. DEL MUNDO; and
ASSISTANT SECRETARIES
DR. MARIO C. VILLAVERDE,
DR. DAVID J. LOZADA, AND
DR. NEMESIO T. GAKO, Promulgated:
Respondents. October 9, 2007
x- - - - - - - - - - - - - - - - - - - - - - - - - - - - - - - - - - - - - - - - - - - - - - - - - - - x
DECISION

588
AUSTRIA-MARTINEZ, J.:

The Court and all parties involved are in agreement that the best nourishment for an
infant is mother's milk. There is nothing greater than for a mother to nurture her beloved child
straight from her bosom. The ideal is, of course, for each and every Filipino child to enjoy the
unequaled benefits of breastmilk. But how should this end be attained?

Before the Court is a petition for certiorari under Rule 65 of the Rules of Court, seeking to
nullify Administrative Order (A.O.) No. 2006-0012 entitled, Revised Implementing Rules and
Regulations of Executive Order No. 51, Otherwise Known as The “Milk Code,” Relevant
International Agreements, Penalizing Violations Thereof, and for Other
Purposes(RIRR). Petitioner posits that the RIRR is not valid as it contains provisions that are not
constitutional and go beyond the law it is supposed to implement.

Named as respondents are the Health Secretary, Undersecretaries, and Assistant


Secretaries of the Department of Health (DOH). For purposes of herein petition, the DOH is
deemed impleaded as a co-respondent since respondents issued the questioned RIRR in their
capacity as officials of said executive agency.[1]

Executive Order No. 51 (Milk Code) was issued by President Corazon Aquino on October
28, 1986 by virtue of the legislative powers granted to the president under the Freedom
Constitution. One of the preambular clauses of the Milk Code states that the law seeks to give
effect to Article 11[2] of the International Code of Marketing of Breastmilk Substitutes (ICMBS), a
code adopted by the World Health Assembly (WHA) in 1981. From 1982 to 2006, the WHA
adopted several Resolutions to the effect that breastfeeding should be supported, promoted and
protected, hence, it should be ensured that nutrition and health claims are not permitted
for breastmilk substitutes.

In 1990, the Philippines ratified the International Convention on the Rights of the
Child. Article 24 of said instrument provides that State Parties should take appropriate
measures to diminish infant and child mortality, and ensure that all segments of society,
specially parents and children, are informed of the advantages of breastfeeding.

On May 15, 2006, the DOH issued herein assailed RIRR which was to take effect on July 7,
2006.

589
However, on June 28, 2006, petitioner, representing its members that are manufacturers
of breastmilk substitutes, filed the present Petition for Certiorari and Prohibition with Prayer for
the Issuance of a Temporary Restraining Order (TRO) or Writ of Preliminary Injunction.

The main issue raised in the petition is whether respondents officers of the DOH acted
without or in excess of jurisdiction, or with grave abuse of discretion amounting to lack or excess
of jurisdiction, and in violation of the provisions of the Constitution in promulgating the RIRR.[3]

On August 15, 2006, the Court issued a Resolution granting a TRO enjoining respondents
from implementing the questioned RIRR.

After the Comment and Reply had been filed, the Court set the case for oral arguments
on June 19, 2007. The Court issued an Advisory (Guidance for Oral Arguments) dated June 5,
2007, to wit:

The Court hereby sets the following issues:

1. Whether or not petitioner is a real party-in-interest;

2. Whether Administrative Order No. 2006-0012 or the Revised


Implementing Rules and Regulations (RIRR) issued by the Department of
Health (DOH) is not constitutional;

2.1 Whether the RIRR is in accord with the provisions of Executive Order
No. 51 (Milk Code);

2.2 Whether pertinent international agreements1 entered into by


the Philippines are part of the law of the land and may be implemented by
the DOH through the RIRR; If in the affirmative, whether the RIRR is in
accord with the international agreements;

2.3 Whether Sections 4, 5(w), 22, 32, 47, and 52 of the RIRR violate the
due process clause and are in restraint of trade; and

2.4 Whether Section 13 of the RIRR on Total Effect provides sufficient


standards.
_____________
1 (1) United Nations Convention on the Rights of
the Child; (2) the WHO and Unicef “2002 Global Strategy on Infant
and Young Child Feeding;” and (3) various World Health Assembly (WHA)
Resolutions.

The parties filed their respective memoranda.


590
The petition is partly imbued with merit.

On the issue of petitioner's standing

With regard to the issue of whether petitioner may prosecute this case as the real party-
in-interest, the Court adopts the view enunciated in Executive Secretary v. Court
[4]
of Appeals, to wit:
The modern view is that an association has standing to complain of injuries to
its members. This view fuses the legal identity of an association with that of its
members. An association has standing to file suit for its workers despite its lack of
direct interest if its members are affected by the action. An organization has
standing to assert the concerns of its constituents.

xxxx

x x x We note that, under its Articles of Incorporation, the respondent was


organized x x x to act as the representative of any individual, company, entity or
association on matters related to the manpower recruitment industry, and to
perform other acts and activities necessary to accomplish the purposes embodied
therein. The respondent is, thus, the appropriate party to assert the rights of its
members, because it and its members are in every practical sense
identical. x x x The respondent [association] is but the medium through
which its individual members seek to make more effective the expression of their
voices and the redress of their grievances. [5](Emphasis supplied)

which was reasserted in Purok Bagong Silang Association, Inc. v. Yuipco,[6] where the Court
ruled that an association has the legal personality to represent its members because the results
of the case will affect their vital interests.[7]

Herein petitioner's Amended Articles of Incorporation contains a similar provision just like
in Executive Secretary, that the association is formed “to represent directly or through approved
representatives the pharmaceutical and health care industry before the Philippine Government
and any of its agencies, the medical professions and the general public.”[8] Thus, as an
organization, petitioner definitely has an interest in fulfilling its avowed purpose of representing
members who are part of the pharmaceutical and health care industry. Petitioner is duly
authorized[9] to take the appropriate course of action to bring to the attention of government
agencies and the courts any grievance suffered by its members which are directly affected by
the RIRR. Petitioner, which is mandated by its Amended Articles of Incorporation to represent
the entire industry, would be remiss in its duties if it fails to act on governmental action that
would affect any of its industry members, no matter how few or numerous they are. Hence,
591
petitioner, whose legal identity is deemed fused with its members, should be considered as a
real party-in-interest which stands to be benefited or injured by any judgment in the present
action.

On the constitutionality of the provisions of the RIRR

First, the Court will determine if pertinent international instruments adverted to by


respondents are part of the law of the land.

Petitioner assails the RIRR for allegedly going beyond the provisions of the Milk Code,
thereby amending and expanding the coverage of said law. The defense of the DOH is that the
RIRR implements not only the Milk Code but also various international instruments[10] regarding
infant and young child nutrition. It is respondents' position that said international instruments
are deemed part of the law of the land and therefore the DOH may implement them through the
RIRR.

The Court notes that the following international instruments invoked by respondents,
namely: (1) The United Nations Convention on the Rights of the Child; (2) The International
Covenant on Economic, Social and Cultural Rights; and (3) the Convention on the Elimination of
All Forms of Discrimination Against Women, only provide in general terms that steps must be
taken by State Parties to diminish infant and child mortality and inform society of the
advantages of breastfeeding, ensure the health and well-being of families, and ensure that
women are provided with services and nutrition in connection with pregnancy and
lactation. Said instruments do not contain specific provisions regarding the use or marketing
of breastmilk substitutes.

The international instruments that do have specific provisions


regarding breastmilk substitutes are the ICMBS and various WHA Resolutions.

Under the 1987 Constitution, international law can become part of the sphere of domestic
law either by transformation orincorporation.[11] The transformation method requires that an
international law be transformed into a domestic law through a constitutional mechanism such
as local legislation. The incorporation method applies when, by mere constitutional declaration,
international law is deemed to have the force of domestic law.[12]

592
Treaties become part of the law of the land through transformation pursuant to Article VII,
Section 21 of the Constitution which provides that “[n]o treaty or international agreement shall
be valid and effective unless concurred in by at least two-thirds of all the members of the
Senate.” Thus, treaties or conventional international law must go through a process prescribed
by the Constitution for it to be transformed into municipal law that can be applied to domestic
conflicts.[13]

The ICMBS and WHA Resolutions are not treaties as they have not been concurred in by at
least two-thirds of all members of the Senate as required under Section 21, Article VII of the
1987 Constitution.

However, the ICMBS which was adopted by the WHA in 1981 had been transformed into
domestic law through local legislation, the Milk Code. Consequently, it is the Milk Code that has
the force and effect of law in this jurisdiction and not the ICMBS per se.

The Milk Code is almost a verbatim reproduction of the ICMBS, but it is well to emphasize
at this point that the Code did not adopt the provision in the ICMBS absolutely prohibiting
advertising or other forms of promotion to the general public of products within the scope of the
ICMBS. Instead, the Milk Code expressly provides that advertising, promotion, or other
marketing materials may be allowed if such materials are duly authorized and approved by the
Inter-Agency Committee (IAC).

On the other hand, Section 2, Article II of the 1987 Constitution, to wit:

SECTION 2. The Philippines renounces war as an instrument of national


policy, adopts the generally accepted principles of international law as part of the
law of the land and adheres to the policy of peace, equality, justice, freedom,
cooperation and amity with all nations. (Emphasis supplied)

embodies the incorporation method.[14]

In Mijares v. Ranada,[15] the Court held thus:


593
[G]enerally accepted principles of international law, by virtue of the
incorporation clause of the Constitution, form part of the laws of the land even if
they do not derive from treaty obligations. The classical formulation in international
law sees those customary rules accepted as binding result from the combination
[of] two elements: the established, widespread, and consistent practice on the part
of States; and a psychological element known as
the opinion juris sive necessitates (opinion as to law or necessity). Implicit in the
latter element is abelief that the practice in question is rendered obligatory by the
existence of a rule of law requiring it.[16] (Emphasis supplied)
“Generally accepted principles of international law” refers to norms of general or
customary international law which are binding on all states,[17] i.e., renunciation of war as an
instrument of national policy, the principle of sovereign immunity,[18] a person's right to life,
liberty and due process,[19] and pacta sunt servanda,[20] among others. The concept
of “generally accepted principles of law” has also been depicted in this wise:

Some legal scholars and judges look upon certain “general principles of
law” as a primary source of international law becausethey have the “character of jus
rationale” and are “valid through all kinds of human societies.” (Judge Tanaka in
his dissenting opinion in the 1966 South West Africa Case, 1966 I.C.J.
296). O'Connell holds that certain priniciples are part of international law
becausethey are “basic to legal systems generally” and hence part of
the jus gentium. These principles, he believes, are established by a process of
reasoning based on the common identity of all legal systems. If there should be
doubt or disagreement, one must look to state practice and determine whether the
municipal law principle provides a just and acceptable
solution. x x x [21] (Emphasis supplied)

Fr. Joaquin G. Bernas defines customary international law as follows:

Custom or customary international law means “a general and


consistent practice of states followed by them from a sense of legal
obligation [opinio juris].” (Restatement) This statement contains the two basic
elements of custom: the material factor, that is, how states behave, and the
psychological or subjective factor, that is, why they behave the way they do.

xxxx

The initial factor for determining the existence of custom is the actual
behavior of states. This includes several elements: duration, consistency, and
generality of the practice of states.

The required duration can be either short or long. xx x

xxxx

594
Duration therefore is not the most important element. More important is
the consistency and the generality of the practice. x x x

xxxx

Once the existence of state practice has been established, it becomes


necessary to determine why states behave the way they do. Do states behave the
way they do because they consider it obligatory to behave thus or do they do it
only as a matter of courtesy? Opiniojuris, or the belief that a certain form of
behavior is obligatory, is what makes practice an international rule. Without it,
practice is not law.[22] (Underscoring and Emphasis supplied)

Clearly, customary international law is deemed incorporated into our domestic


[23]
system.

WHA Resolutions have not been embodied in any local legislation. Have they attained the
status of customary law and should they then be deemed incorporated as part of the law of the
land?

The World Health Organization (WHO) is one of the international specialized agencies
allied with the United Nations (UN) by virtue of Article 57,[24] in relation to Article 63[25] of the UN
Charter. Under the 1946 WHO Constitution, it is the WHA which determines the policies of the
WHO,[26] and has the power to adopt regulations concerning “advertising and labeling of
biological, pharmaceutical and similar products moving in international commerce,”[27] and to
“make recommendations to members with respect to any matter within the competence of the
Organization.”[28] The legal effect of its regulations, as opposed to recommendations, is quite
different.

Regulations, along with conventions and agreements, duly adopted by the WHA bind
member states thus:

Article 19. The Health Assembly shall have authority to adopt conventions or
agreements with respect to any matter within the competence of the Organization.
A two-thirds vote of the Health Assembly shall be required for the adoption of
such conventions or agreements, whichshall come into force for each Member when
accepted by it in accordance with its constitutional processes.

Article 20. Each Member undertakes that it will, within eighteen months after the
adoption by the Health Assembly of a convention or agreement, take action relative
to the acceptance of such convention or agreement. Each Member shall notify the
Director-General of the action taken, and if it does not accept such convention or

595
agreement within the time limit, it will furnish a statement of the reasons for non-
acceptance. In case of acceptance, each Member agrees to make an annual report
to the Director-General in accordance with Chapter XIV.

Article 21. The Health Assembly shall have authority to adopt regulations
concerning: (a) sanitary and quarantine requirements and other procedures
designed to prevent the international spread of disease; (b) nomenclatures with
respect to diseases, causes of death and public health practices; (c) standards with
respect to diagnostic procedures for international use; (d) standards with respect to
the safety, purity and potency of biological, pharmaceutical and similar products
moving in international commerce; (e) advertising and labeling of biological,
pharmaceutical and similar products moving in international commerce.

Article 22. Regulations adopted pursuant to Article 21 shall come into force for all
Members after due notice has been given of their adoption by the Health Assembly
except for such Members as may notify the Director-General of rejection or
reservations within the period stated in the notice. (Emphasis supplied)

On the other hand, under Article 23, recommendations of the WHA do not come into
force for members, in the same way that conventions or agreements under Article 19
and regulations under Article 21 come into force. Article 23 of the WHO Constitution reads:

Article 23. The Health Assembly shall have authority to make


recommendations to Members with respect to any matter within the competence of
the Organization. (Emphasis supplied)

The absence of a provision in Article 23 of any mechanism by which the recommendation


would come into force for member states is conspicuous.

The former Senior Legal Officer of WHO, Sami Shubber, stated that WHA
recommendations are generally not binding, but they “carry moral and political weight, as they
constitute the judgment on a health issue of the collective membership of the highest
international body in the field of health.”[29] Even the ICMBS itself was adopted as a mere
recommendation, as WHA Resolution No. 34.22 states:

“The Thirty-Fourth World Health Assembly x x x adopts, in the sense


of Article 23 of the Constitution, the International Code of Marketing
of Breastmilk Substitutes annexed to the present resolution.” (Emphasis supplied)

The Introduction to the ICMBS also reads as follows:

596
In January 1981, the Executive Board of the World Health Organization at its
sixty-seventh session, considered the fourth draft of the code, endorsed it, and
unanimously recommended to the Thirty-fourth World Health Assembly the text of
a resolution by which it would adopt the code in the form of a recommendation
rather than a regulation. x x x (Emphasis supplied)

The legal value of WHA Resolutions as recommendations is summarized in Article 62 of


the WHO Constitution, to wit:

Art. 62. Each member shall report annually on the action taken with
respect to recommendations made to it by the Organization, and with respect to
conventions, agreements and regulations.

Apparently, the WHA Resolution adopting the ICMBS and subsequent WHA Resolutions urging
member states to implement the ICMBS are merely recommendatory and legally non-
binding. Thus, unlike what has been done with the ICMBS whereby the legislature enacted most
of the provisions into law which is the Milk Code, the subsequent WHA Resolutions,[30]specifically
providing for exclusive breastfeeding from 0-6 months, continued breastfeeding up to 24
months, and absolutely prohibiting advertisements and promotions of breastmilk substitutes,
have not been adopted as a domestic law.

It is propounded that WHA Resolutions may constitute “soft law” or non-binding norms,
principles and practices that influence state behavior.[31]

“Soft law” does not fall into any of the categories of international law set forth in Article
38, Chapter III of the 1946 Statute of the International Court of Justice.[32] It is, however, an
expression of non-binding norms, principles, and practices that influence state
[33]
behavior. Certain declarations and resolutions of the UN General Assembly fall under this
[34]
category. The most notable is the UN Declaration of Human Rights, which this Court has
enforced in various cases, specifically, Government ofHongkong Special Administrative Region
v. Olalia,[35] Mejoff v. Director of Prisons,[36] Mijares v. Rañada[37] andShangri-la International
Hotel Management, Ltd. v. Developers Group of Companies, Inc..[38]

The World Intellectual Property Organization (WIPO), a specialized agency attached to


the UN with the mandate to promote and protect intellectual property worldwide, has resorted to
597
soft law as a rapid means of norm creation, in order “to reflect and respond to the changing
needs and demands of its constituents.”[39] Other international organizations which have
resorted to soft law include the International Labor Organization and the Food and Agriculture
Organization (in the form of theCodex Alimentarius).[40]

WHO has resorted to soft law. This was most evident at the time of the Severe Acute
Respiratory Syndrome (SARS) and Avian flu outbreaks.

Although the IHR Resolution does not create new international law binding
on WHO member states, it provides an excellent example of the power of "soft law"
in international relations. International lawyers typically distinguish binding rules of
international law-"hard law"-from non-binding norms, principles, and practices that
influence state behavior-"soft law." WHO has during its existence generated many
soft law norms, creating a "soft law regime" in international governance for
public health.

The "soft law" SARS and IHR Resolutions represent significant steps in
laying the political groundwork for improved international cooperation on infectious
diseases. These resolutions clearly define WHO member states' normative duty to
cooperate fully with other countries and with WHO in connection with infectious
disease surveillance and response to outbreaks.

This duty is neither binding nor enforceable, but, in the wake of the SARS
epidemic, the duty is powerful politically for two reasons. First, the SARS outbreak
has taught the lesson that participating in, and enhancing, international
cooperation on infectious disease controls is in a country's self-interest x x x if this
warning is heeded, the "soft law" in the SARS and IHR Resolution could inform the
development of general and consistent state practice on infectious disease
surveillance and outbreak response, perhaps crystallizing eventually into customary
international law on infectious disease prevention and control.[41]

In the Philippines, the executive department implemented certain measures recommended


by WHO to address the outbreaks of SARS and Avian flu by issuing Executive Order (E.O.) No.
201 on April 26, 2003 and E.O. No. 280 on February 2, 2004, delegating to various departments
broad powers to close down schools/establishments, conduct health surveillance and monitoring,
and ban importation of poultry and agricultural products.

It must be emphasized that even under such an international emergency, the duty of a
state to implement the IHR Resolution was still considered not binding or enforceable, although
said resolutions had great political influence.

598
As previously discussed, for an international rule to be considered as customary law, it
must be established that such rule is being followed by states because they consider it
obligatory to comply with such rules (opinio juris). Respondents have not presented any
evidence to prove that the WHA Resolutions, although signed by most of the member states,
were in fact enforced or practiced by at least a majority of the member states; neither have
respondents proven that any compliance by member states with said WHA Resolutions was
obligatory in nature.

Respondents failed to establish that the provisions of pertinent WHA Resolutions are
customary international law that may be deemed part of the law of the land.

Consequently, legislation is necessary to transform the provisions of the WHA Resolutions


into domestic law. The provisions of the WHA Resolutions cannot be considered as part of the
law of the land that can be implemented by executive agencies without the need of a law
enacted by the legislature.

Second, the Court will determine whether the DOH may implement the provisions of the
WHA Resolutions by virtue of its powers and functions under the Revised Administrative Code
even in the absence of a domestic law.

Section 3, Chapter 1, Title IX of the Revised Administrative Code of 1987 provides that the
DOH shall define the national health policy and implement a national health plan within the
framework of the government's general policies and plans, and issue orders and regulations
concerning the implementation of established health policies.

It is crucial to ascertain whether the absolute prohibition on advertising and other forms of
promotion of breastmilksubstitutes provided in some WHA Resolutions has been adopted as part
of the national health policy.

Respondents submit that the national policy on infant and young child feeding is embodied
in A.O. No. 2005-0014, datedMay 23, 2005. Basically, the Administrative Order declared the
following policy guidelines: (1) ideal breastfeeding practices, such as early initiation of
breastfeeding, exclusive breastfeeding for the first six months, extended breastfeeding up to two
years and beyond; (2) appropriate complementary feeding, which is to start at age six months;
(3) micronutrient supplementation; (4) universal salt iodization; (5) the exercise of other
feeding options; and (6) feeding in exceptionally difficult circumstances. Indeed, the primacy of

599
breastfeeding for children is emphasized as a national health policy. However, nowhere in A.O.
No. 2005-0014 is it declared that as part of such health policy, the advertisement or promotion
of breastmilk substitutes should be absolutely prohibited.

The national policy of protection, promotion and support of breastfeeding cannot


automatically be equated with a total ban on advertising for breastmilk substitutes.
In view of the enactment of the Milk Code which does not contain a total ban on the
advertising and promotion ofbreastmilk substitutes, but instead, specifically creates an IAC
which will regulate said advertising and promotion, it follows that a total ban policy could be
implemented only pursuant to a law amending the Milk Code passed by the constitutionally
authorized branch of government, the legislature.

Thus, only the provisions of the Milk Code, but not those of subsequent WHA
Resolutions, can be validly implemented by the DOH through the subject RIRR.

Third, the Court will now determine whether the provisions of the RIRR are in accordance
with those of the Milk Code.

In support of its claim that the RIRR is inconsistent with the Milk Code, petitioner alleges
the following:

1. The Milk Code limits its coverage to children 0-12 months old, but the RIRR extended
its coverage to “young children” or those from ages two years old and beyond:

MILK CODE RIRR


WHEREAS, in order to ensure that Section 2. Purpose – These Revised Rules and
safe and adequate nutrition for Regulations are hereby promulgated to ensure
infants is provided, there is a need to the provision of safe and adequate nutrition for
protect and promote breastfeeding infants and young children by the promotion,
and to inform the public about the protection and support of breastfeeding and by
proper use ofbreastmilk substitutes ensuring the proper use
and supplements and related of breastmilk substitutes,breastmilk supplements
products through adequate, and related products when these are medically
consistent and objective information indicated and only when necessary, on the basis
and appropriate regulation of the of adequate information and through appropriate
marketing and distribution of the said marketing and distribution.
substitutes, supplements and related
products; Section 5(ff). “Young Child” means a person
from the age of more than twelve (12) months
SECTION 4(e). “Infant” means a up to the age of three (3) years (36 months).
person falling within the age bracket
600
of 0-12 months.

2. The Milk Code recognizes that infant formula may be a proper and possible substitute
for breastmilk in certain instances; but the RIRR provides “exclusive breastfeeding for
infants from 0-6 months” and declares that “there is no substitute nor replacement
for breastmilk”:

MILK CODE RIRR


WHEREAS, in order to ensure that Section 4. Declaration of Principles –
safe and adequate nutrition The following are the underlying
for infants is provided, there is a principles from which the revised
need to protect and promote rules and regulations are premised
breastfeeding and to inform the upon:
public about the proper use
ofbreastmilk substitutes and a. Exclusive breastfeeding is for
supplements and related infants from 0 to six (6) months.
products through adequate,
consistent and objective information b. There is no substitute or
and appropriate regulation of the replacement forbreastmilk.
marketing and distribution of the said
substitutes, supplements and related
products;

3. The Milk Code only regulates and does not impose unreasonable requirements for
advertising and promotion; RIRR imposes an absolute ban on such activities
for breastmilk substitutes intended for infants from 0-24 months old or beyond, and
forbids the use of health and nutritional claims. Section 13 of the RIRR, which provides
for a “total effect” in the promotion of products within the scope of the Code, is vague:

MILK CODE RIRR


SECTION 6. The General Public and Section 4. Declaration of Principles –
Mothers. – The following are the underlying
(a) No advertising, promotion or principles from which the revised
other marketing materials, whether rules and regulations are premised
written, audio or visual, for products upon:
within the scope of this Codeshall be
printed, published, distributed, xxxx
exhibited and broadcast unless such
materials are duly authorized and f. Advertising, promotions, or
approved by an inter-agency sponsor-ships of infant
committee created herein pursuant to formula, breastmilk substitutes and
the applicable standards provided for other related products are prohibited.
in this Code.
601
Section 11. Prohibition – No
advertising, promotions,
sponsorships, or marketing materials
and
activities for breastmilk substitutes
intendedfor infants and young
children up to twenty-four (24)
months, shall be allowed, because
they tend to convey or give subliminal
messages or impressions that
undermine breastmilk and
breastfeeding or otherwise
exaggerate breastmilksubstitutes
and/or replacements, as well as
related products covered within the
scope of this Code.

Section 13. “Total Effect” - Promotion


of products within the scope of this
Code must be objective and should
not equate or make the product
appear to be as good or equal
tobreastmilk or breastfeeding in the
advertising concept. It must not in
any case underminebreastmilk or
breastfeeding. The “total effect”
should not directly or indirectly
suggest that buying their product
would produce better individuals, or
resulting in greater love, intelligence,
ability, harmony or in any manner
bring better health to the baby or
other such exaggerated and
unsubstantiated claim.

Section 15. Content of Materials. -


The following shall not be included in
advertising, promotional and
marketing materials:

a. Texts, pictures, illustrations or


information which discourage or tend
to undermine the benefits or
superiority of breastfeeding or which
idealize the use
of breastmilk substitutes and milk
supplements. In this connection, no
pictures of babies and children
together with their mothers, fathers,

602
siblings, grandparents, other relatives
or caregivers (or yayas) shall be used
in any advertisements for infant
formula and breastmilksupplements;

b. The term “humanized,”


“maternalized,” “close to mother's
milk” or similar words in
describingbreastmilk substitutes or
milk supplements;

c. Pictures or texts that idealize the


use of infant and milk formula.

Section 16. All health and nutrition


claims for products within the scope
of the Code are absolutely
prohibited. For this purpose, any
phrase or words that connotes to
increase emotional, intellectual
abilities of the infant and young child
and other like phrases shall not be
allowed.

4. The RIRR imposes additional labeling requirements not found in the Milk Code:

MILK CODE RIRR


SECTION 10. Containers/Label. – Section 26. Content – Each
(a) Containers and/or labels container/label shall contain such
shall be designed to provide message, in both Filipino and English
the necessary information languages, and which message
about the appropriate use of cannot be readily
the products, and in such a separated therefrom, relative the
way as not to discourage following points:
breastfeeding. (a) The words or phrase “Important
(b) Each container shall have a Notice” or “Government
clear, conspicuous and easily Warning” or their equivalent;
readable and understandable (b) A statement of the superiority of
message in Pilipino or English breastfeeding;
printed on it, or on a label, (c) A statement that there is no
which message can not readily substitute forbreastmilk;
become separated from it, and (d) A statement that the product
which shall include the shall be used only on the advice
following points: of a health worker as to the
(i) the words “Important need for its use and the proper
Notice” or their methods of use;
equivalent; (e) Instructions for
603
(ii) a statement of the appropriate prepara-tion, and a
superiority of warning against the health
breastfeeding; hazards of inappropriate
(iii) a statement that the preparation; and
product shall be used only (f) The health hazards of
on the advice of a health unnecessary or improper use of
worker as to the need for infant formula and other related
its use and the proper products including information
methods of use; and that powdered infant formula
(iv) instructions for may contain pathogenic
appropriate preparation, microorganisms and must be
and a warning against the prepared and used
health hazards of appropriately.
inappropriate preparation.

5. The Milk Code allows dissemination of information on infant formula to health


professionals; the RIRR totally prohibits such activity:

MILK CODE RIRR


SECTION 7. Health Care System. – Section 22. No manufacturer,
(b) No facility of the health care distributor, or representatives of
system shall be used for the purpose products covered by the Code shall
of promoting infant formula or other be allowed to conduct or be involved
products within the scope of this in any activity on breastfeeding
Code. This Code does not, however, promotion, education and production
preclude the dissemination of of Information, Education and
information to health professionals as Communication (IEC) materials on
provided in Section 8(b). breastfeeding, holding of or
participating as speakers in classes or
SECTION 8. Health Workers. - seminars for women and children
(b) Information provided by activities and to avoid the use of
manufacturers and distributors to these venues to market their brands
health professionals regarding or company names.
products within the scope of this
Code shall be restricted to scientific SECTION 16. All health and nutrition
and factual matters and such claims for products within the scope
information shall not imply or create of the Code are absolutely
a belief that bottle-feeding is prohibited. For this purpose, any
equivalent or superior to phrase or words that connotes to
breastfeeding. It shall also include increase emotional, intellectual
the information specified in Section abilities of the infant and young child
5(b). and other like phrases shall not be
allowed.

604
6. The Milk Code permits milk manufacturers and distributors to extend assistance in
research and continuing education of health professionals; RIRR absolutely forbids the
same.

MILK CODE RIRR


SECTION 8. Health Workers – Section 4. Declaration of Principles –
(e) Manufacturers and distributors of The following are the underlying
products within the scope of this principles from which the revised
Code may assist in the research, rules and regulations are premised
scholarships and continuing upon:
education, of health professionals, in i. Milk companies, and their
accordance with the rules and representatives, should not
regulations promulgated by the form part of any policymaking
Ministry of Health. body or entity in relation to the
advancement ofbreasfeeding.

SECTION 22. No manufacturer,


distributor, or representatives of
products covered by the Code shall
be allowed to conduct or be involved
in any activity on breastfeeding
promotion, education and production
of Information, Education and
Communication (IEC) materials on
breastfeeding, holding of or
participating as speakers in classes or
seminars for women and children
activities and to avoid the use of
these venues to market their brands
or company names.

SECTION 32. Primary Responsibility


of Health Workers - It is the primary
responsibility of the health workers to
promote, protect and support
breastfeeding and appropriate infant
and young child feeding. Part of this
responsibility is to continuously
update their knowledge and skills on
breastfeeding. No assistance,
support, logistics or training from
milk companies shall be permitted.

7. The Milk Code regulates the giving of donations; RIRR absolutely prohibits it.

MILK CODE RIRR

605
SECTION 6. The General Public and Section 51. Donations Within the
Mothers. – Scope of This Code - Donations of
(f) Nothing herein contained shall products, materials, defined and
prevent donations from covered under the Milk Code and
manufacturers and distributors of these implementing rules and
products within the scope of this regulations, shall be strictly
Code upon request by or with the prohibited.
approval of the Ministry of Health.
Section 52. Other Donations By Milk
Companies Not Covered by this
Code. - Donations of products,
equipments, and the like, not
otherwise falling within the scope of
this Code or these Rules, given by
milk companies and their agents,
representatives, whether in kind or in
cash, may only be coursed through
the Inter Agency Committee (IAC),
which shall determine whether such
donation be accepted or otherwise.

8. The RIRR provides for administrative sanctions not imposed by the Milk Code.

MILK CODE RIRR


Section 46. Administrative
Sanctions. – The following
administrative sanctions shall be
imposed upon any person, juridical or
natural, found to have violated the
provisions of the Code and its
implementing Rules and Regulations:
a) 1st violation – Warning;
b) 2nd violation – Administrative
fine of a minimum of Ten
Thousand (P10,000.00) to Fifty
Thousand (P50,000.00) Pesos,
depending on the gravity and
extent of the violation, including
the recall of the offending
product;
c) 3rd violation – Administrative
Fine of a minimum of Sixty
Thousand (P60,000.00) to One
Hundred Fifty Thousand
(P150,000.00) Pesos,
depending on the gravity and
extent of the violation, and in
addition thereto, the recall of
606
the offending product, and
suspension of the Certificate of
Product Registration (CPR);
d) 4th violation –Administrative
Fine of a minimum of Two
Hundred Thousand
(P200,000.00) to Five Hundred
(P500,000.00) Thousand Pesos,
depending on the gravity and
extent of the violation; and in
addition thereto, the recall of
the product, revocation of the
CPR, suspension of the License
to Operate (LTO) for one year;
e) 5th and succeeding repeated
violations – Administrative Fine
of One Million (P1,000,000.00)
Pesos, the recall of the
offending product, cancellation
of the CPR, revocation of the
License to Operate (LTO) of the
company concerned, including
the blacklisting of the company
to be furnished the Department
of Budget and Management
(DBM) and the Department of
Trade and Industry (DTI);
f) An additional penalty of Two
Thou-sand Five Hundred
(P2,500.00) Pesos per day shall
be made for every day the
violation continues after having
received the order from the IAC
or other such appropriate body,
notifying and penalizing the
company for the infraction.
For purposes of determining
whether or not there is “repeated”
violation, each product violation
belonging or owned by a company,
including those of their subsidiaries,
are deemed to be violations of the
concerned milk company and shall
not be based on the specific violating
product alone.

9. The RIRR provides for repeal of existing laws to the contrary.


607
The Court shall resolve the merits of the allegations of petitioner seriatim.

1. Petitioner is mistaken in its claim that the Milk Code's coverage is limited
only to children 0-12 months old. Section 3 of the Milk Code states:

SECTION 3. Scope of the Code – The Code applies to the marketing, and
practices related thereto, of the following products:breastmilk substitutes, including
infant formula; other milk products, foods and beverages, including bottle-fed
complementary foods, when marketed or otherwise represented to be suitable, with
or without modification, for use as a partial or total replacement of breastmilk;
feeding bottles and teats. It also applies to their quality and availability, and to
information concerning their use.

Clearly, the coverage of the Milk Code is not dependent on the age of the child but on the kind of
product being marketed to the public. The law treats infant formula, bottle-fed complementary
food, and breastmilk substitute as separate and distinct product categories.

Section 4(h) of the Milk Code defines infant formula as “a breastmilk substitute x x x to
satisfy the normal nutritional requirements of infants up to between four to six months of age,
and adapted to their physiological characteristics”; while under Section 4(b), bottle-fed
complementary food refers to “any food, whether manufactured or locally prepared, suitable as
a complement to breastmilk or infant formula, when either becomes insufficient to satisfy the
nutritional requirements of theinfant.” An infant under Section 4(e) is a person falling within
the age bracket 0-12 months. It is the nourishment of this group of infants or children aged 0-
12 months that is sought to be promoted and protected by the Milk Code.

But there is another target group. Breastmilk substitute is defined under Section 4(a)
as “any food being marketed or otherwise presented as a partial or total replacement for
breastmilk, whether or not suitable for that purpose.” This section conspicuously lacks reference
to any particular age-group of children. Hence, the provision of the Milk Code cannot be
considered exclusive for children aged 0-12 months. In other words, breastmilk substitutes may
also be intended for young children more than 12 months of age. Therefore, by regulating
breastmilk substitutes, the Milk Code also intends to protect and promote the nourishment
of children more than 12 months old.

608
Evidently, as long as what is being marketed falls within the scope of the Milk Code as
provided in Section 3, then it can be subject to regulation pursuant to said law, even if the
product is to be used by children aged over 12 months.

There is, therefore, nothing objectionable with Sections 2[42] and 5(ff)[43] of the RIRR.

2. It is also incorrect for petitioner to say that the RIRR, unlike the Milk Code,
does not recognize that breastmilksubstitutes may be a proper and possible substitute
for breastmilk.

The entirety of the RIRR, not merely truncated portions thereof, must
[44]
be considered and construed together. As held in De Luna v. Pascual, “[t]he particular
words, clauses and phrases in the Rule should not be studied as detached and isolated
expressions, but the whole and every part thereof must be considered in fixing the meaning of
any of its parts and in order to produce a harmonious whole.”
Section 7 of the RIRR provides that “when medically indicated and only when
necessary, the use of breastmilksubstitutes is proper if based on complete and updated
information.” Section 8 of the RIRR also states that information and educational materials
should include information on the proper use of infant formula when the use thereof is needed.

Hence, the RIRR, just like the Milk Code, also recognizes that in certain cases, the use
of breastmilk substitutes may be proper.

3. The Court shall ascertain the merits of allegations 3[45] and 4[46] together as
they are interlinked with each other.

To resolve the question of whether the labeling requirements and advertising regulations
under the RIRR are valid, it is important to deal first with the nature, purpose, and depth of the
regulatory powers of the DOH, as defined in general under the 1987 Administrative
[47]
Code, and as delegated in particular under the Milk Code.

Health is a legitimate subject matter for regulation by the DOH (and certain other
administrative agencies) in exercise of police powers delegated to it. The sheer span of
jurisprudence on that matter precludes the need to further discuss it..[48] However, health
information, particularly advertising materials on apparently non-toxic products

609
like breastmilk substitutes and supplements, is a relatively new area for regulation by the
DOH.[49]
As early as the 1917 Revised Administrative Code of the Philippine Islands,[50] health
information was already within the ambit of the regulatory powers of the predecessor of
DOH.[51] Section 938 thereof charged it with the duty to protect the health of the people, and
vested it with such powers as “(g) the dissemination of hygienic information among the people
and especially the inculcation of knowledge as to the proper care of infants and the methods of
preventing and combating dangerous communicable diseases.”

Seventy years later, the 1987 Administrative Code tasked respondent DOH to carry out
the state policy pronounced under Section 15, Article II of the 1987 Constitution, which is “to
protect and promote the right to health of the people and instill health consciousness among
them.”[52] To that end, it was granted under Section 3 of the Administrative Code the power to
“(6) propagate health information and educate the population on important health, medical and
environmental matters which have health implications.”[53]

When it comes to information regarding nutrition of infants and young children, however,
the Milk Code specifically delegated to the Ministry of Health (hereinafter referred to as DOH)
the power to ensure that there is adequate, consistent and objective information on
breastfeeding and use of breastmilk substitutes, supplements and related products; and the
power tocontrol such information. These are expressly provided for in Sections 12 and 5(a), to
wit:

SECTION 12. Implementation and Monitoring –

xxxx

(b) The Ministry of Health shall be principally responsible for the


implementation and enforcement of the provisions of this Code. For this
purpose, the Ministry of Health shall have the following powers and functions:

(1) To promulgate such rules and regulations as are necessary


or proper for the implementation of this Code and the
accomplishment of its purposes and objectives.

xxxx

(4) To exercise such other powers and functions as may be


necessary for or incidental to the attainment of the purposes
and objectives of this Code.
610
SECTION 5. Information and Education –

(a) The government shall ensure that objective and


consistent information is provided on infant feeding, for use by families
and those involved in the field of infant nutrition. This responsibility shall
cover the planning, provision, design and dissemination of information, and
the control thereof, on infant nutrition. (Emphasis supplied)

Further, DOH is authorized by the Milk Code to control the content of any information
on breastmilk vis-à-vis breastmilksubstitutes, supplement and related products, in the following
manner:

SECTION 5. xxx

(b) Informational and educational materials, whether written, audio, or


visual, dealing with the feeding of infants and intended to reach pregnant
women and mothers of infants, shall include clear information on all the
following points: (1) the benefits and superiority of breastfeeding; (2)
maternal nutrition, and the preparation for and maintenance of
breastfeeding; (3) the negative effect on breastfeeding of introducing
partial bottlefeeding; (4) the difficulty of reversing the decision not to
breastfeed; and (5) where needed, the proper use of infant formula, whether
manufactured industrially or home-prepared. When such materials contain
information about the use of infant formula, they shall include the social and
financial implications of its use; the health hazards of inappropriate foods or
feeding methods; and, in particular, the health hazards of unnecessary or
improper use of infant formula and other breastmilk substitutes. Such
materials shall not use any picture or text which may idealize the use
of breastmilksubstitutes.

SECTION 8. Health Workers –

xxxx

(b) Information provided by manufacturers and distributors to health


professionals regarding products within the scope of this Codeshall be
restricted to scientific and factual matters, and such information shall not
imply or create a belief that bottlefeedingis equivalent or superior to
breastfeeding. It shall also include the information specified in Section 5(b).

SECTION 10. Containers/Label –

(a) Containers and/or labels shall be designed to provide the necessary


information about the appropriate use of the products, and in such a way as
not to discourage breastfeeding.
611
xxxx

(d) The term “humanized,” “maternalized” or similar terms shall not


be used. (Emphasis supplied)

The DOH is also authorized to control the purpose of the information and to whom such
information may be disseminated under Sections 6 through 9 of the Milk Code[54] to ensure that
the information that would reach pregnant women, mothers of infants, and health professionals
and workers in the health care system is restricted to scientific and factual matters and
shall notimply or create a belief that bottlefeeding is equivalent or superior to breastfeeding.

It bears emphasis, however, that the DOH's power under the Milk Code
to control information regarding breastmilk vis-a-vis breastmilk substitutes is not absolute as the
power to control does not encompass the power to absolutely prohibit the advertising,
marketing, and promotion of breastmilk substitutes.

The following are the provisions of the Milk Code that unequivocally indicate that the
control over information given to the DOH is not absolute and that absolute prohibition is not
contemplated by the Code:

a) Section 2 which requires adequate information and appropriate marketing and


distribution of breastmilk substitutes, to wit:

SECTION 2. Aim of the Code – The aim of the Code is to contribute to the
provision of safe and adequate nutrition for infants by the protection and promotion
of breastfeeding and by ensuring the proper use of breastmilk substitutes
and breastmilk supplements when these are necessary, on the basis of adequate
information and through appropriate marketing and distribution.

b) Section 3 which specifically states that the Code applies to the marketing of and
practices related to breastmilksubstitutes, including infant formula, and to information
concerning their use;

c) Section 5(a) which provides that the government shall ensure that objective and
consistent information is provided on infant feeding;

612
d) Section 5(b) which provides that written, audio or visual informational and educational
materials shall not use any picture or text which may idealize the use of breastmilk substitutes
and should include information on the health hazards of unnecessary or improper use of said
product;

e) Section 6(a) in relation to Section 12(a) which creates and empowers the IAC to review
and examine advertising, promotion, and other marketing materials;

f) Section 8(b) which states that milk companies may provide information to health
professionals but such information should be restricted to factual and scientific matters and shall
not imply or create a belief that bottlefeeding is equivalent or superior to breastfeeding; and

g) Section 10 which provides that containers or labels should not contain information that
would discourage breastfeeding and idealize the use of infant formula.

It is in this context that the Court now examines the assailed provisions of the RIRR
regarding labeling and advertising.

Sections 13[55] on “total effect” and 26[56] of Rule VII of the RIRR contain some labeling
requirements, specifically: a) that there be a statement that there is no substitute to breastmilk;
and b) that there be a statement that powdered infant formula may contain pathogenic
microorganisms and must be prepared and used appropriately. Section 16[57] of the RIRR
prohibits all health and nutrition claims for products within the scope of the Milk Code, such as
claims of increased emotional and intellectual abilities of the infant and young child.

These requirements and limitations are consistent with the provisions of Section 8 of the
Milk Code, to wit:

SECTION 8. Health workers -

xxxx

(b) Information provided by manufacturers and distributors to health


professionals regarding products within the scope of this Code shall
be restricted to scientific and factual matters, and such information shall
not imply or create a belief that bottlefeeding is equivalent or superior to
breastfeeding. It shall also include the information specified in Section
5.[58] (Emphasis supplied)

613
and Section 10(d)[59] which bars the use on containers and labels of the terms “humanized,”
“maternalized,” or similar terms.

These provisions of the Milk Code expressly forbid information that would imply or create a
belief that there is any milk product equivalent to breastmilk or which is humanized or
maternalized, as such information would be inconsistent with the superiority of breastfeeding.

It may be argued that Section 8 of the Milk Code refers only to information given to health
workers regarding breastmilk substitutes, not to containers and labels thereof. However, such
restrictive application of Section 8(b) will result in the absurd situation in which milk companies
and distributors are forbidden to claim to health workers that their products are substitutes or
equivalents of breastmilk, and yet be allowed to display on the containers and labels of their
products the exact opposite message. That askewed interpretation of the Milk Code is precisely
what Section 5(a) thereof seeks to avoid by mandating that all information regarding
breastmilk vis-a-vis breastmilk substitutes be consistent, at the same time giving the
government control over planning, provision, design, and dissemination of information on infant
feeding.

Thus, Section 26(c) of the RIRR which requires containers and labels to state that the
product offered is not a substitute for breastmilk, is a reasonable means of enforcing Section
8(b) of the Milk Code and deterring circumvention of the protection and promotion of
breastfeeding as embodied in Section 2[60] of the Milk Code.

Section 26(f)[61] of the RIRR is an equally reasonable labeling requirement. It implements


Section 5(b) of the Milk Code which reads:

SECTION 5. x x x

xxxx

(b) Informational and educational materials, whether written, audio, or


visual, dealing with the feeding of infants and intended to reach pregnant
women and mothers of infants, shall include clear information on all the
following points: x x x (5) where needed, the proper use of infant formula,
whether manufactured industrially or home-prepared. When such materials
contain information about the use of infant formula, they shall include the
social and financial implications of its use; the health hazards of
inappropriate foods or feeding methods; and, in particular, the health
hazards of unnecessary or improper use of infant formula and other

614
breastmilk substitutes. Such materials shall not use any picture or text which
may idealize the use of breastmilk substitutes. (Emphasis supplied)

The label of a product contains information about said product intended for the buyers
thereof. The buyers of breastmilksubstitutes are mothers of infants, and Section 26 of the RIRR
merely adds a fair warning about the likelihood of pathogenic microorganisms being present in
infant formula and other related products when these are prepared and used inappropriately.

Petitioner’s counsel has admitted during the hearing on June 19, 2007 that formula milk is
prone to contaminations and there is as yet no technology that allows production of powdered
infant formula that eliminates all forms of contamination.[62]

Ineluctably, the requirement under Section 26(f) of the RIRR for the label to contain the
message regarding health hazards including the possibility of contamination with pathogenic
microorganisms is in accordance with Section 5(b) of the Milk Code.

The authority of DOH to control information regarding breastmilk vis-a-


vis breastmilk substitutes and supplements and related products cannot be questioned. It is its
intervention into the area of advertising, promotion, and marketing that is being assailed by
petitioner.

In furtherance of Section 6(a) of the Milk Code, to wit:

SECTION 6. The General Public and Mothers. –

(a) No advertising, promotion or other marketing materials, whether


written, audio or visual, for products within the scope of this Code shall be printed,
published, distributed, exhibited and broadcast unless such materials are duly
authorized and approved by an inter-agency committee created herein pursuant to
the applicable standards provided for in this Code.

the Milk Code invested regulatory authority over advertising, promotional and marketing
materials to an IAC, thus:

SECTION 12. Implementation and Monitoring -

(a) For purposes of Section 6(a) of this Code, an inter-


agency committee composed of the following members is hereby created:

Minister of Health -------------------------------------------- Chairman


Minister of Trade and Industry ---------------------------- Member
615
Minister of Justice -------------------------------------------- Member
Minister of Social Services and Development ----------- Member

The members may designate their duly authorized representative to every


meeting of the Committee.

The Committee shall have the following powers and functions:

(1) To review and examine all advertising. promotion or other marketing


materials, whether written, audio or visual, on products within the scope of
this Code;

(2) To approve or disapprove, delete objectionable portions from and prohibit


the printing, publication, distribution, exhibition and broadcast of, all
advertising promotion or other marketing materials, whether written, audio
or visual, on products within the scope of this Code;
(3) To prescribe the internal and operational procedure for the exercise of its
powers and functions as well as the performance of its duties and
responsibilities; and

(4) To promulgate such rules and regulations as are necessary or proper for
the implementation of Section 6(a) of this Code. x x x (Emphasis supplied)

However, Section 11 of the RIRR, to wit:

SECTION 11. Prohibition – No advertising, promotions, sponsorships, or


marketing materials and activities for breastmilk substitutes intended for infants
and young children up to twenty-four (24) months, shall be allowed, because they
tend to convey or give subliminal messages or impressions that
undermine breastmilk and breastfeeding or otherwise
exaggerate breastmilk substitutes and/or replacements, as well as related products
covered within the scope of this Code.

prohibits advertising, promotions, sponsorships or marketing materials and activities


for breastmilk substitutes in line with theRIRR’s declaration of principle under Section 4(f), to
wit:

SECTION 4. Declaration of Principles –

xxxx

(f) Advertising, promotions, or sponsorships of infant


formula, breastmilk substitutes and other related products are prohibited.

616
The DOH, through its co-respondents, evidently arrogated to itself not only the regulatory
authority given to the IAC but also imposed absolute prohibition on advertising, promotion, and
marketing.

Yet, oddly enough, Section 12 of the RIRR reiterated the requirement of the Milk
Code in Section 6 thereof for prior approval by IAC of all advertising, marketing and promotional
materials prior to dissemination.

Even respondents, through the OSG, acknowledged the authority of IAC, and
repeatedly insisted, during the oral arguments on June 19, 2007, that the prohibition under
Section 11 is not actually operational, viz:

SOLICITOR GENERAL DEVANADERA:

xxxx

x x x Now, the crux of the matter that is being questioned by Petitioner is


whether or not there is an absolute prohibition on advertising making AO 2006-12
unconstitutional. We maintained that what AO 2006-12 provides is not an absolute
prohibition because Section 11 while it states and it is entitled prohibition it states
that no advertising, promotion, sponsorship or marketing materials and activities
for breast milk substitutes intended for infants and young children up to 24 months
shall be allowed because this is the standard they tend to convey or give subliminal
messages or impression undermine that breastmilk or breastfeeding x x x.

We have to read Section 11 together with the other Sections because the
other Section, Section 12, provides for the inter agency committee that is
empowered to process and evaluate all the advertising and promotion materials.

xxxx

What AO 2006-12, what it does, it does not prohibit the sale and
manufacture, it simply regulates the advertisement and the promotions of
breastfeeding milk substitutes.
xxxx

Now, the prohibition on advertising, Your Honor, must be taken together


with the provision on the Inter-Agency Committee that processes and evaluates
because there may be some information dissemination that are straight forward
information dissemination. What the AO 2006 is trying to prevent is any material
that will undermine the practice of breastfeeding, Your Honor.

xxxx

617
ASSOCIATE JUSTICE SANTIAGO:

Madam Solicitor General, under the Milk Code, which body has authority or
power to promulgate Rules and Regulations regarding the Advertising, Promotion
and Marketing of Breastmilk Substitutes?

SOLICITOR GENERAL DEVANADERA:

Your Honor, please, it is provided that the Inter-Agency


Committee, Your Honor.

xxxx

ASSOCIATE JUSTICE SANTIAGO:

x x x Don't you think that the Department of Health overstepped its rule
making authority when it totally banned advertising and promotion under Section
11 prescribed the total effect rule as well as the content of materials under Section
13 and 15 of the rules and regulations?

SOLICITOR GENERAL DEVANADERA:

Your Honor, please, first we would like to stress that there is no total
absolute ban. Second, the Inter-Agency Committee is under the Department of
Health, Your Honor.

xxxx

ASSOCIATE JUSTICE NAZARIO:

x x x Did I hear you correctly, Madam Solicitor, that there is no absolute


ban on advertising of breastmilk substitutes in the Revised Rules?

SOLICITOR GENERAL DEVANADERA:

Yes, your Honor.

ASSOCIATE JUSTICE NAZARIO:

But, would you nevertheless agree that there is an absolute ban on


advertising of breastmilk substitutes intended for children two (2) years old and
younger?

SOLICITOR GENERAL DEVANADERA:

It's not an absolute ban, Your Honor, because we have the Inter-Agency
Committee that can evaluate some advertising and promotional materials, subject

618
to the standards that we have stated earlier, which are- they should not undermine
breastfeeding, Your Honor.

xxxx

x x x Section 11, while it is titled Prohibition, it must be taken in relation


with the other Sections, particularly 12 and 13 and 15, Your Honor, because it is
recognized that the Inter-Agency Committee has that power to evaluate
promotional materials, Your Honor.

ASSOCIATE JUSTICE NAZARIO:

So in short, will you please clarify there's no absolute ban on advertisement


regarding milk substitute regarding infants two (2) years below?

SOLICITOR GENERAL DEVANADERA:

We can proudly say that the general rule is that there is a prohibition,
however, we take exceptions and standards have been set. One of which is that,
the Inter-Agency Committee can allow if the advertising and promotions will not
undermine breastmilk and breastfeeding, Your Honor.[63]

Sections 11 and 4(f) of the RIRR are clearly violative of the Milk Code.

However, although it is the IAC which is authorized to promulgate rules and


regulations for the approval or rejection of advertising, promotional, or other marketing
materials under Section 12(a) of the Milk Code, said provision must be related to Section 6
thereof which in turn provides that the rules and regulations must be “pursuant to the applicable
standards provided for in this Code.” Said standards are set forth in Sections 5(b), 8(b), and 10
of the Code, which, at the risk of being repetitious, and for easy reference, are quoted
hereunder:

SECTION 5. Information and Education –

xxxx

(b) Informational and educational materials, whether written, audio, or


visual, dealing with the feeding of infants and intended to reach pregnant
women and mothers of infants, shall include clear information on all the
following points: (1) the benefits and superiority of
breastfeeding; (2) maternal nutrition, and the preparation for and
maintenance of breastfeeding; (3) the negative effect on breastfeeding of
introducing partial bottlefeeding; (4) the difficulty of reversing the decision
not to breastfeed; and (5) where needed, the proper use of infant formula,
whether manufactured industrially or home-prepared. When such materials
619
contain information about the use of infant formula, they shall include the
social and financial implications of its use; the health hazards of
inappropriate foods of feeding methods; and, in particular, the health
hazards of unnecessary or improper use of infant formula and
other breastmilksubstitutes. Such materials shall not use any picture or text
which may idealize the use of breastmilk substitutes.

xxxx

SECTION 8. Health Workers. –

xxxx

(b) Information provided by manufacturers and distributors to health


professionals regarding products within the scope of this Code shall be
restricted to scientific and factual matters and such information shall not
imply or create a belief that bottle feeding is equivalent or superior to
breastfeeding. It shall also include the information specified in Section 5(b).

xxxx

SECTION 10. Containers/Label –

(a) Containers and/or labels shall be designed to provide the necessary


information about the appropriate use of the products, and in such a way as
not to discourage breastfeeding.

(b) Each container shall have a clear, conspicuous and easily readable
and understandable message in Pilipino or English printed on it, or on a label,
which message can not readily become separated from it, and which shall
include the following points:

(i) the words “Important Notice” or their equivalent;


(ii) a statement of the superiority of breastfeeding;
(iii) a statement that the product shall be used only on
the advice of a health worker as to the need for its
use and the proper methods of use; and
(iv) instructions for appropriate preparation, and
a warning against the health hazards of
inappropriate preparation.

Section 12(b) of the Milk Code designates the DOH as


the principal implementing agency for the enforcement of the provisions of the Code. In
relation to such responsibility of the DOH, Section 5(a) of the Milk Code states that:

SECTION 5. Information and Education –

620
(a) The government shall ensure that objective and consistent information is
provided on infant feeding, for use by families and those involved in the field
of infant nutrition. This responsibility shall cover the planning, provision,
design and dissemination of information, and the control thereof, on infant
nutrition. (Emphasis supplied)

Thus, the DOH has the significant responsibility to translate into operational terms the standards
set forth in Sections 5, 8, and 10 of the Milk Code, by which the IAC shall screen advertising,
promotional, or other marketing materials.

It is pursuant to such responsibility that the DOH correctly provided for Section 13
in the RIRR which reads as follows:

SECTION 13. “Total Effect” - Promotion of products within the scope of this
Code must be objective and should not equate or make the product appear to be as
good or equal to breastmilk or breastfeeding in the advertising concept. It must
not in any case underminebreastmilk or breastfeeding. The “total effect” should not
directly or indirectly suggest that buying their product would produce better
individuals, or resulting in greater love, intelligence, ability, harmony or in any
manner bring better health to the baby or other such exaggerated and
unsubstantiated claim.

Such standards bind the IAC in formulating its rules and regulations on advertising, promotion,
and marketing. Through that single provision, the DOH exercises control over the information
content of advertising, promotional and marketing materials onbreastmilk vis-a-
vis breastmilk substitutes, supplements and other related products. It also sets a viable
standard against which the IAC may screen such materials before they are made public.

In Equi-Asia Placement, Inc. vs. Department of Foreign Affairs,[64] the Court held:

x x x [T]his Court had, in the past, accepted as sufficient standards the


following: “public interest,” “justice and equity,” “public convenience and welfare,”
and “simplicity, economy and welfare.”[65]

In this case, correct information as to infant feeding and nutrition is infused with public
interest and welfare.

4. With regard to activities for dissemination of information to health professionals, the


Court also finds that there is no inconsistency between the provisions of the Milk Code and the
RIRR. Section 7(b)[66] of the Milk Code, in relation to Section 8(b)[67] of the same Code, allows

621
dissemination of information to health professionals but such information is restricted to
scientific and factual matters.

Contrary to petitioner's claim, Section 22 of the RIRR does not prohibit the giving of
information to health professionals on scientific and factual matters. What it prohibits is the
involvement of the manufacturer and distributor of the products covered by the Code in
activities for the promotion, education and production of Information, Education and
Communication (IEC) materials regarding breastfeeding that are intended for women and
children. Said provision cannot be construed to encompass even the dissemination of
information to health professionals, as restricted by the Milk Code.

5. Next, petitioner alleges that Section 8(e)[68] of the Milk Code permits milk
manufacturers and distributors to extend assistance in research and in the continuing education
of health professionals, while Sections 22 and 32 of the RIRR absolutely forbid the
same. Petitioner also assails Section 4(i)[69] of the RIRR prohibiting milk manufacturers' and
distributors' participation in any policymaking body in relation to the advancement of
breastfeeding.

Section 4(i) of the RIRR provides that milk companies and their representatives should not
form part of any policymaking body or entity in relation to the advancement of
breastfeeding. The Court finds nothing in said provisions which contravenes the Milk Code. Note
that under Section 12(b) of the Milk Code, it is the DOH which shall be principally
responsible for theimplementation and enforcement of the provisions of said Code. It is entirely
up to the DOH to decide which entities to call upon or allow to be part of policymaking bodies on
breastfeeding. Therefore, the RIRR's prohibition on milk companies’ participation in any
policymaking body in relation to the advancement of breastfeeding is in accord with the Milk
Code.

Petitioner is also mistaken in arguing that Section 22 of the RIRR prohibits milk companies
from giving reasearchassistance and continuing education to health professionals. Section
22[70] of the RIRR does not pertain to research assistance to or the continuing education
of health professionals; rather, it deals with breastfeeding promotion andeducation for women
and children. Nothing in Section 22 of the RIRR prohibits milk companies from giving
assistance for research or continuing education to health professionals; hence, petitioner's
argument against this particular provision must be struck down.

622
It is Sections 9[71] and 10[72] of the RIRR which govern research assistance. Said sections
of the RIRR provide thatresearch assistance for health workers and researchers may be allowed
upon approval of an ethics committee, and with certain disclosure requirements imposed on the
milk company and on the recipient of the research award.

The Milk Code endows the DOH with the power to determine how such research or
educational assistance may be given by milk companies or under what conditions health workers
may accept the assistance. Thus, Sections 9 and 10 of the RIRR imposing limitations on the
kind of research done or extent of assistance given by milk companies are completely in accord
with the Milk Code.

Petitioner complains that Section 32[73] of the RIRR prohibits milk companies from giving
assistance, support, logistics or training to health workers. This provision is within the
prerogative given to the DOH under Section 8(e)[74] of the Milk Code,which provides that
manufacturers and distributors of breastmilk substitutes may assist in researches, scholarships
and the continuing education, of health professionals in accordance with the rules and
regulations promulgated by the Ministry of Health, now DOH.

6. As to the RIRR's prohibition on donations, said provisions are also consistent with the
Milk Code. Section 6(f) of the Milk Code provides that donations may be made by manufacturers
and distributors of breastmilk substitutes upon the request or with the approval of the DOH. The
law does not proscribe the refusal of donations. The Milk Code leaves it purely to the discretion
of the DOH whether to request or accept such donations. The DOH then appropriately exercised
its discretion through Section 51[75] of the RIRR which sets forth its policy not to request
or approve donations from manufacturers and distributors of breastmilk substitutes.
It was within the discretion of the DOH when it provided in Section 52 of the RIRR that
any donation from milk companies not covered by the Code should be coursed through the IAC
which shall determine whether such donation should be accepted or refused. As reasoned out by
respondents, the DOH is not mandated by the Milk Code to accept donations. For that matter,
no person or entity can be forced to accept a donation. There is, therefore, no real inconsistency
between the RIRR and the law because the Milk Code does not prohibit the DOH from refusing
donations.

7. With regard to Section 46 of the RIRR providing for administrative sanctions that are
not found in the Milk Code, theCourt upholds petitioner's objection thereto.

623
Respondent's reliance on Civil Aeronautics Board v. Philippine Air Lines, Inc.[76] is
misplaced. The glaring difference in said case and the present case before the Court is that, in
the Civil Aeronautics Board, the Civil Aeronautics Administration (CAA) was expressly granted by
the law (R.A. No. 776) the power to impose fines and civil penalties, while the Civil Aeronautics
Board (CAB) was granted by the same law the power to review on appeal the order or decision
of the CAA and to determine whether to impose, remit, mitigate, increase or compromise such
fine and civil penalties. Thus, the Court upheld the CAB's Resolution imposing administrative
fines.

In a more recent case, Perez v. LPG Refillers Association of the Philippines, Inc.,[77] the
Court upheld the Department of Energy (DOE) Circular No. 2000-06-10
implementing Batas Pambansa (B.P.) Blg. 33. The circular provided for fines for the commission
of prohibited acts. The Court found that nothing in the circular contravened the law because the
DOE was expressly authorized by B.P. Blg. 33 and R.A. No. 7638 to impose fines or penalties.

In the present case, neither the Milk Code nor the Revised Administrative Code grants the
DOH the authority to fix or impose administrative fines. Thus, without any express grant of
power to fix or impose such fines, the DOH cannot provide for those fines in the RIRR. In this
regard, the DOH again exceeded its authority by providing for such fines or sanctions in Section
46 of the RIRR. Said provision is, therefore, null and void.

The DOH is not left without any means to enforce its rules and regulations. Section 12(b)
(3) of the Milk Code authorizes the DOH to “cause the prosecution of the violators of this Code
and other pertinent laws on products covered by this Code.” Section 13 of the Milk Code
provides for the penalties to be imposed on violators of the provision of the Milk Code or the
rules and regulations issued pursuant to it, to wit:

SECTION 13. Sanctions –

(a) Any person who violates the provisions of this Code or the rules and
regulations issued pursuant to this Code shall, upon conviction, be punished by a
penalty of two (2) months to one (1) year imprisonment or a fine of not less than
One Thousand Pesos (P1,000.00) nor more than Thirty Thousand Pesos
(P30,000.00) or both. Should the offense be committed by a juridical person, the
chairman of the Board of Directors, the president, general manager, or the partners
and/or the persons directly responsible therefor, shall be penalized.

(b) Any license, permit or authority issued by any government agency to


any health worker, distributor, manufacturer, or marketing firm or personnel for the
practice of their profession or occupation, or for the pursuit of their business, may,
624
upon recommendation of the Ministry of Health, be suspended or revoked in the
event of repeated violations of this Code, or of the rules and regulations issued
pursuant to this Code. (Emphasis supplied)

8. Petitioner’s claim that Section 57 of the RIRR repeals existing laws that are contrary to
the RIRR is frivolous.
Section 57 reads:

SECTION 57. Repealing Clause - All orders, issuances, and rules and
regulations or parts thereof inconsistent with these revised rules and implementing
regulations are hereby repealed or modified accordingly.

Section 57 of the RIRR does not provide for the repeal of laws but only orders, issuances
and rules and regulations. Thus, said provision is valid as it is within the DOH's rule-making
power.

An administrative agency like respondent possesses quasi-legislative or rule-making


power or the power to make rules and regulations which results in delegated legislation that is
within the confines of the granting statute and the Constitution, and subject to the doctrine of
non-delegability and separability of powers.[78] Such express grant of rule-
making power necessarily includes the power to amend, revise, alter, or repeal the
[79]
same. This is to allow administrative agencies flexibility in formulating and adjusting the
details and manner by which they are to implement the provisions of a law,[80] in order to make
it more responsive to the times. Hence, it is a standard provision in administrative rules that
prior issuances of administrative agencies that are inconsistent therewith are declared repealed
or modified.

In fine, only Sections 4(f), 11 and 46 are ultra vires, beyond the authority of the DOH to
promulgate and in contravention of the Milk Code and, therefore, null and void. The rest of the
provisions of the RIRR are in consonance with the Milk Code.

Lastly, petitioner makes a “catch-all” allegation that:

x x x [T]he questioned RIRR sought to be implemented by the Respondents


is unnecessary and oppressive, and is offensive to the due process clause of the
Constitution, insofar as the same is in restraint of trade and because a provision
therein is inadequate to provide the public with a comprehensible basis to
determine whether or not they have committed a violation.[81] (Emphasis supplied)

625
Petitioner refers to Sections 4(f),[82] 4(i),[83] 5(w),[84] 11,[85] 22,[86] 32,[87] 46,[88] and 52[89] as the
provisions that suppress the trade of milk and, thus, violate the due process clause of the
Constitution.

The framers of the constitution were well aware that trade must be subjected to some
form of regulation for the public good. Public interest must be upheld over business
interests.[90] In Pest Management Association of the Philippines v. Fertilizer and Pesticide
[91]
Authority, it was held thus:

x x x Furthermore, as held in Association of Philippine Coconut


Desiccators v. Philippine Coconut Authority, despite the fact that “our present
Constitution enshrines free enterprise as a policy, it nonetheless reserves to the
government the power to intervene whenever necessary to promote the general
welfare.” There can be no question that the unregulated use or proliferation of
pesticides would be hazardous to our environment. Thus, in the aforecited case,
the Court declared that “free enterprise does not call for removal of ‘protective
regulations’.” x x x It must be clearly explained and proven by competent
evidence just exactly how such protective regulation would result in the restraint of
trade. [Emphasis and underscoring supplied]

In this case, petitioner failed to show that the proscription of milk manufacturers’
participation in any policymaking body (Section 4(i)), classes and seminars for women and
children (Section 22); the giving of assistance, support and logistics or training (Section 32);
and the giving of donations (Section 52) would unreasonably hamper the trade
of breastmilk substitutes. Petitioner has not established that the proscribed activities are
indispensable to the trade of breastmilk substitutes. Petitioner failed to demonstrate that the
aforementioned provisions of the RIRR are unreasonable and oppressive for being in restraint of
trade.

Petitioner also failed to convince the Court that Section 5(w) of the RIRR is unreasonable
and oppressive. Said section provides for the definition of the term “milk company,” to wit:

SECTION 5 x x x. (w) “Milk Company” shall refer to the owner,


manufacturer, distributor of infant formula, follow-up milk, milk formula, milk
supplement, breastmilk substitute or replacement, or by any other description of
such nature, including their representatives who promote or otherwise advance their
commercial interests in marketing those products;

626
On the other hand, Section 4 of the Milk Code provides:
(d) “Distributor” means a person, corporation or any other entity in the
public or private sector engaged in the business (whether directly or
indirectly) of marketing at the wholesale or retail level a product within
the scope of this Code. A “primary distributor” is a manufacturer's sales
agent, representative, national distributor or broker.

xxxx

(j) “Manufacturer” means a corporation or other entity in the public or


private sector engaged in the business or function (whether directly or
indirectly or through an agent or and entity controlled by or under
contract with it) of manufacturing a products within the scope of this
Code.

Notably, the definition in the RIRR merely merged together under the term “milk company” the
entities defined separately under the Milk Code as “distributor” and “manufacturer.” The RIRR
also enumerated in Section 5(w) the products manufactured or distributed by an entity that
would qualify it as a “milk company,” whereas in the Milk Code, what is used is the phrase
“products within the scope of this Code.” Those are the only differences between the definitions
given in the Milk Code and the definition as re-stated in the RIRR.

Since all the regulatory provisions under the Milk Code apply equally to both
manufacturers and distributors, the Court sees no harm in the RIRR providing for just one term
to encompass both entities. The definition of “milk company” in the RIRR and the definitions of
“distributor” and “manufacturer” provided for under the Milk Code are practically the same.

The Court is not convinced that the definition of “milk company” provided in the RIRR
would bring about any change in the treatment or regulation of “distributors” and
“manufacturers” of breastmilk substitutes, as defined under the Milk Code.
Except Sections 4(f), 11 and 46, the rest of the provisions of the RIRR are in consonance
with the objective, purpose and intent of the Milk Code, constituting reasonable regulation of an
industry which affects public health and welfare and, as such, the rest of the RIRR do not
constitute illegal restraint of trade nor are they violative of the due process clause of the
Constitution.

WHEREFORE, the petition is PARTIALLY GRANTED. Sections 4(f), 11 and 46 of


Administrative Order No. 2006-0012 dated May 12, 2006 are declared NULL and VOID for

627
being ultra vires. The Department of Health and respondents arePROHIBITED from implementing
said provisions.

The Temporary Restraining Order issued on August 15, 2006 is LIFTED insofar as the rest
of the provisions of Administrative Order No. 2006-0012 is concerned.

SO ORDERED.

MA. ALICIA AUSTRIA-MARTINEZ


Associate Justice

WE CONCUR:

REYNATO S. PUNO
Chief Justice

LEONARDO A. QUISUMBING CONSUELO YNARES-SANTIAGO


Associate Associate
Justice Justice

ANGELINA SANDOVAL-GUTIERREZ ANTONIO T. CARPIO


Associate Justice Associate Justice

RENATO C. CORONA CONCHITA CARPIO-MORALES


Associate Justice Associate
Justice

ADOLFO S. AZCUNA DANTE O. TINGA


Associate Justice Associate Justice

MINITA V. CHICO-NAZARIO CANCIO C. GARCIA


Associate Justice Associate Justice

PRESBITERO J. VELASCO, JR. ANTONIO EDUARDO B. NACHURA


Associate Justice Associate Justice

628
RUBEN T. REYES
Associate Justice

CERTIFICATION

Pursuant to Section 13, Article VIII of the Constitution, it is hereby certified that the
conclusions in the above Decision had been reached in consultation before the case was
assigned to the writer of the opinion of the Court.

REYNATO S. PUNO
Chief Justice

[1]
Section 11, Rule 3, 1997 Rules of Civil Procedure which provides:
Section 11. Misjoinder and non-joinder of parties. - Neither misjoinder nor
non-joinder of parties is ground for dismissal of an action. Parties may be dropped or
added by order of the court on motion of any party or on its own initiative at any stage of
the action and on such terms as are just. x x x (Emphasis supplied)
[2]
Article 11. Implementation and monitoring
11.1 Governments should take action to give effect to the principles
and aim of this Code, as appropriate to their social and legislative framework, including
the adoption of national legislation, regulations or other suitable measures. For this
purpose, governments should seek, when necessary, the cooperation ofWHO, UNICEF and
other agencies of the United Nations system. National policies and measures, including
laws and regulations, which are adopted to give effect to the principles and aim of this
Code should be publicly stated, and should apply on the same basis to all those involved
in the manufacture and marketing of products within the scope of this Code.
xxxx
[3]
Petition, rollo, p. 12.
[4]
G.R. No. 131719, May 25, 2004, 429 SCRA 81.
[5]
Id. at 96-97.
[6]
G.R. No. 135092, May 4, 2006, 489 SCRA 382.
[7]
Id. at 396.
[8]
Annex “G”, Petitioner's Memorandum dated July 19, 2007.
[9]
Annexes “H”, “I”, and “J” of Petitioner's Memorandum executed
by Wyeth Philippines, Inc., Bristol Myers Squibb (Phil.), Inc., and Abbott Laboratories,
Inc., respectively.
[10]
a) The UN Convention on the Rights of the Child (CRC); b) the International Code
of Marketing Breastmilk Substitutes (ICMBS); c) the International Covenant on
Economic, Social and Cultural Rights (CSCR); d) the Convention on the Elimination of
All Forms of Discrimination Against Women (CEDAW); e) the Global Strategy for Infant

629
and Young Child Nutrition (Global Strategy); and f) various resolutions adopted by the
World Health Assembly.
[11]
Joaquin G. Bernas, S.J., Constitutional Structure and Powers of Government (Notes
and Cases) Part I ( 2005).
[12]
Id.
[13]
Joaquin G. Bernas, S.J., An Introduction to Public International Law, 2002 Ed., p. 57.
[14]
According to Fr. Bernas, the Austrian Constitution (Art. 9) and the Constitution of the
Federal Republic of Germany (Art. 25) also use the incorporation method.
[15]
G.R. No. 139325, April 12, 2005, 455 SCRA 397.
[16]
Id. at 421.
[17]
Merlin M. Magallona, Fundamentals of Public International Law, 2005 Ed., p. 526.
[18]
Id. at 525.
[19]
Government of Hong Kong Special Administrative Region v. Olalia, G.R. No.
153675, April 19, 2007.
[20]
Tañada v. Angara, 338 Phil. 546, 592 (1997).
[21]
Louis Henkin, Richard C. Pugh, Oscar Schachter, Hans Smit, International Law,
Cases and Materials, 2nd Ed., p. 96.
[22]
Supra note 13, at 10-13.
[23]
Minucher v. Court of Appeals, 445 Phil. 250, 269 (2003).
[24]
Article 57. The various specialized agencies, established by intergovernmental
agreement and having wide international responsibilities, as defined in their basic
instruments, in economic, social, cultural, educational, health, and related fields, shall be
brought into relationship with the United Nations in accordance with the provisions of
Article 63.
Such agencies thus brought into relationship with the United Nations are
hereinafter referred to as specialized agencies.
[25]
Article 63. The Economic and Social Council may enter into agreements with any of
the agencies referred to in Article 57, defining the terms on which the agency
concerned shall be brought into relationship with the United Nations. Such agreements
shall be subject to approval by the General Assembly.
It may coordinate the activities of the specialized agencies through
consultation with and recommendations to such agencies and through
recommendations to the General Assembly and to the Members of the United Nations.
[26]
Article 18. The functions of the Health Assembly shall be: (a) to determine the
policies of the Organization x x x. (Emphasis supplied)
[27]
Article 21. The Health Assembly shall have authority to adopt regulations
concerning: x x x (e) advertising and labeling of biological, pharmaceutical and
similar products moving in international commerce. (Emphasis supplied)
[28]
Article 23. The Health Assembly shall have authority to make recommendations to
Members with respect to any matter within the competence of the Organization.
(Emphasis supplied)
[29]
See David Fidler, Developments Involving SARS, International Law, and Infectious
Disease Control at the Fifty-Sixth Meeting of the World Health Assembly, June 2003,
ASIL.
[30]
In Resolution No. 34.22 (May 21, 1981), the WHA, acting under Article 23 of the
WHO Constitution, adopted the ICBMS.

(a) In Resolution No. 35.26 (May 1982), the WHA urged member
states to implement the ICBMS as a “minimum requirement”.
630
(b) In Resolution No. 39.28 (May 16, 1986), the WHA requested
the WHO Director General to direct the attention of member states to the fact
that any food or drink given before complementary feeding is nutritionally
required may interfere with the initiation or maintenance of breastfeeding
and therefore should neither be promoted nor encouraged for us by infants
during this period.

(c) In Resolution No. 43.3 (May 14, 1990), the WHA urged member
states to protect and promote breastfeeding as an essential component of
nutrition policies so as to enable infants to be exclusively breastfed during
the first four to six months of life.

(d) In Resolution No. 45.34 (May 14, 1992), the WHA urged
member states to implement the targets of the Innocenti Declaration
specifically, to give effect to the ICMBS.

(e) In Resolution No. 46.7 (May 10, 1993), the WHA urged member
states to strive to eliminate under-nutrition, malnutrition and nutritional
deficiency among children.

(f) In Resolution No. 47.5 (May 9, 1994), the WHA urged member
states to ensure that there are no donations of supplies
of breastmilk substitutes and other products covered by the ICMBS in any
part of the health care system.

(g) In Resolution No. 49.15 (May 25, 1996), the WHA urged
member states to ensure that complementary foods are not marketed for or
used in ways that undermine exclusive and sustained breastfeeding.

(h) In Resolution No. 54.2 (May 2002), the WHA, noting that
“despite the fact that the International Code of Marketing
of Breastmilk Substitutes and relevant subsequent World Health Assembly
resolutions state that there should be no advertising or other forms of
promotion of products within its scope, new modern communication
methods including electronic means, are currently increasingly being used to
promote such products; and conscious of the need for the
Codex Alimentarius Commission to take the International Code and
subsequent relevant Health Assembly resolutions into consideration in
dealing with health claims in the development of food standards and
guidelines x x x,” urged member states to develop new approaches to
protect, promote and support exclusive breastfeeding for six months as a
global public health recommendation.

(i) In Resolution No. 55.25 (May 15, 2002), the WHA requested
the Codex Alimentarius Commission to ensure that labelling of processed
foods for infants and young children be consistent with the WHO policy under
the ICBMS.

631
(j) In Resolution No. 58.32 (May 25, 2005), the WHA urged
member states to continue to protect and promote exclusive breastfeeding
for six months.

(k) In Resolution No. 59.21 (May 27, 2006), the WHA reiterated its
support for the Gobal strategy for Infant and Young Child Feeding.
[31]
David Fidler, supra note 29.
[32]
Article 38. 1. The Court, whose function is to decide in accordance with international
law such disputes as are submitted to it, shall apply: a) international conventions, whether
general or particular, establishing rules expressly recognized by the contesting states;
b) international custom, as evidence of a general practice accepted as law; c) the
general principles of law recognized by civilized nations; d) subject to the provisions of
Article 59, judicial decisions and the teachings of the most highly qualified publicists
of the various nations, as subsidiary means for the determination of rules of law.
[33]
Supra note 29.
[34]
Louis Henkin, et al., International Law, Cases and Materials, 2nd Ed., supra note 21,
at 114-136.
[35]
Supra note 19.
[36]
90 Phil. 70 (1951).
[37]
Supra note 15.
[38]
G.R. No. 159938, March 31, 2006, 486 SCRA 405.
[39]
Edward Kwakwa, Some Comments on Rulemaking at the World Intellectual Property
Organization, www.law.duke.edu/shell/cite; September 13, 2007, 12:33, citing the 1999
WIPO Resolution Concerning Provisions on the Protection of Well-Known Marks, 2000
WIPO Recommendation Concerning Trademark Licenses, and 2001 WIPO
Recommendation Concerning Provisions on the Protection of Marks and other Industrial
Property Rights in Signs on the Internet.
[40]
Id.
[41]
Supra note 29.
[42]
Section 2. Purpose – These Revised Rules and Regulations are hereby promulgated to
ensure the provision of safe and adequate nutrition for infants and young children by
the promotion, protection and support of breastfeeding and by ensuring the proper use
of breastmilk substitutes, breastmilk supplements and related products when these are
medically indicated and only when necessary, on the basis of adequate information
and through appropriate marketing and distribution. (Underscoring supplied)
[43]
Section 5(ff). “Young Child” means a person from the age of more than twelve (12)
months up to the age of three (3) years (36 months). (Underscoring supplied)
[44]
G.R. No. 144218, July 14, 2006, 495 SCRA 42, 55.
[45]
See pp. 19-21.
[46]
See p. 21.
[47]
Executive Order No. 292, made effective on November 23, 1989 by Proclamation No.
495.
[48]
Jacobson v. Massachusetts, 197 US 11 (1905); Beltran v. Secretary of Health G.R.
No. 133640, November 25, 2005, 476 SCRA 168, 196; St. Lukes’s Medical Center
Employees Association- AFW v. National Labor Relations Commission, G.R. No. 162053,
March 7, 2007; Tablarin v. Gutierrez, G.R. No. L-78164, July 31, 1987, 152 SCRA 730,
741; Pollution Adjudication Board v. Court of Appeals, G.R. No. 93891, March 11, 1991,
195 SCRA 112, 123-124; Rivera v. Campbell, 34 Phil. 348, 353-354 (1916);Lorenzo v.
Director of Health, 50 Phil. 595, 597 (1927).
632
[49]
As early as People v. Pomar, 46 Phil. 440, 445 (1924), we already
noted that “advancing civilization is bringing within the scope of police power of the state
today things which were not thought of as being with in such power yesterday. The
development of civilization, the rapidly increasing population, the growth of public opinion,
with [an increasing] desire on the part of the masses and of the government to look after
and care for the interests of the individuals of the state, have brought within the police
power of the state many questions for regulation which formerly were not so considered.”
[50]
Act No. 2711, approved on March 10, 1917.
[51]
Known then as Public Health Service
[52]
Section 1, Chapter I, Title IX, Executive Order No. 292.
[53]
Id. at Section 3.
[54]
SECTION 6. The General Public and Mothers –
(a) No advertising, promotion or other marketing materials, whether written,
audio or visual, for products within the scope of this Code shall be printed,
published, distributed, exhibited and broadcast unless such materials are duly
authorized and approved by an inter-agency committee created herein pursuant
to the applicable standards provided for in this Code.
(b) Manufacturers and distributors shall not be permitted to give, directly or
indirectly, samples and supplies of products within the scope of this Code or gifts
of any sort to any member of the general public, including members of their
families, to hospitals and other health institutions, as well as to personnel within
the health care system, save as otherwise provided in this Code.
(c) There shall be no point-of-sale advertising, giving of samples or any other
promotion devices to induce sales directly to the consumers at the retail level,
such as special displays, discount coupons, premiums, special sales, bonus and
tie-in sales for the products within the scope of this Code. This provision shall
not restrict the establishment of pricing policies and practices intended to
provide products at lower prices on a long-term basis.
(d) Manufactures and distributors shall not distribute to pregnant women or
mothers of infants any gifts or articles or utensils which may promote the use
ofbreastmilk substitutes or bottlefeeding, nor shall any other groups, institutions
or individuals distribute such gifts, utensils or products to the general public and
mothers.
(e) Marketing personnel shall be prohibited from advertising or promoting in any
other manner the products covered by this Code, either directly or indirectly, to
pregnant women or with mother of infants, except as otherwise provided by this
Code.
(f) Nothing herein contained shall prevent donations from manufacturers and
distributors or products within the scope of this Code upon request by or with
the approval of the Ministry of Health.
SECTION 7. Health Care System –
(a) The Ministry of Health shall take appropriate measures to encourage and
promote breastfeeding. It shall provide objective and consistent information,
training and advice to health workers on infant nutrition, and on their obligations
under this Code.
(b) No facility of the health care system shall be used for the purpose of
promoting infant formula or other products within the scope of this Code. This
Code does not, however, preclude the dissemination of information to health
professionals as provided in Section 8(b).

633
(c) Facilities of the health care system shall not be used for the display of
products within the scope of this Code, or for placards or posters concerning
such products.
(d) The use by the health care system of “professional service” representatives,
“mothercraft nurses” or similar personnel, provided or paid for by manufacturers
or distributors, shall not be permitted.

(e) In health education classes for mothers and the general public, health
workers and community workers shall emphasize the hazards and risks of the
improper use of breastmilk substitutes particularly infant formula. Feeding with
infant formula shall be demonstrated only to mothers who may not be able to
breastfeed for medical or other legitimate reasons.
SECTION 8. Health Workers –
(a) Health workers shall encourage and promote breastfeeding and shall make
themselves familiar with objectives and consistent information on maternal and
infant nutrition, and with their responsibilities under this Code.
(b) Information provided by manufacturers and distributors to health
professionals regarding products within the scope of this Code shall be restricted
to scientific and factual matters and such information shall not imply or create a
belief that bottlefeeding is equivalent or superior to breastfeeding. It shall also
include the information specified in Section 5(b).
(c) No financial or material inducements to promote products within the scope of
this Code shall be offered by manufacturers or distributors to health workers or
members of their families, nor shall these be accepted by the health workers or
members of their families, except as otherwise provided in Section 8(e).
(d) Samples of infant formula or other products within the scope of this Code, or
of equipment or utensils for their preparation or use, shall not be provided to
health workers except when necessary for the purpose of professional evaluation
or research in accordance with the rules and regulations promulgated by the
Ministry of Health. No health workers shall give samples of infant formula to
pregnant women and mothers of infants or members of their families.
(e) Manufacturers and distributors of products within the scope of this Code may
assist in the research, scholarships and continuing education, of health
professionals, in accordance with the rules and regulations promulgated by the
Ministry of Health.
SECTION 9. Persons employed by Manufacturers and Distributors – Personnel employed
in marketing products within the scope of this Code shall not, as part of their job
responsibilities, perform educational functions in relation to pregnant women or mothers
of infants.
[55]
See p. 20.
[56]
See p. 21.
[57]
SECTION 16. All health and nutrition claims for products within the scope of the
Code are absolutely prohibited. For this purpose, any phrase or words that
connotes to increase emotional, intellectual abilities of the infant and young child
and other like phrases shall not be allowed.
[58]
See p. 30.
[59]
SECTION 10. Containers/Label –
xxxx

634
(d) The term “humanized”, “maternalized” or similar terms shall not be used.
[60]
SECTION 2. Aim of the Code – The aim of the Code is to contribute to the provision
of safe and adequate nutrition for infants by the protection and promotion of breastfeeding
and by ensuring the proper use of breastmilk substitutes
and breastmilk supplements when these are necessary, on the basis of adequate
information and through appropriate marketing and distribution.
[61]
SECTION 26. Content – Each container/label shall contain such message, in both
Filipino and English languages, and which message cannot be readily separatedtherefrom,
relative the following points:
xxxx
(f) The health hazards of unnecessary or improper use of infant formula and other
related products including information that powdered infant formula may contain
pathogenic microorganisms and must be prepared and used appropriately.
[62]
TSN of the hearing of June 19, 2007, pp. 114-120.
[63]
TSN of June 19, 2007 hearing, pp. 193-194, 198, 231, 237-240, 295-300.
[64]
G.R. No. 152214, September 19, 2006, 502 SCRA 295.
[65]
Id. at 314.
[66]
SECTION 7. Health Care System –
xxxx
(b) No facility of the health care system shall be used for the purpose of promoting
infant formula or other products within the scope of this Code. This Code does not,
however, preclude the dissemination of information to health professionals as
provided in Section 8(b).
[67]
SECTION 8. Health Workers. -
xxxx
(b) Information provided by manufacturers and distributors to health professionals
regarding products within the scope of this Code shall be restricted to scientific and
factual matters and such information shall not imply or create a belief that bottlefeeding is
equivalent or superior to breastfeeding. It shall also include the information specified
in Section 5(b).
[68]
SECTION 8. Health Workers -
xxxx
(e) Manufacturers and distributors of products within the scope of this Code may
assist in the research, scholarships and continuing education, of health professionals, in
accordance with the rules and regulations promulgated by the Ministry of Health.
[69]
SECTION 4. Declaration of Principles – The following are the underlying principles from
which the revised rules and regulations are premised upon:
xxxx
(i) Milk companies, and their representatives, should not form part of any policymaking
body or entity in relation to the advancement of breastfeeding.
[70]
SECTION 22. No manufacturer, distributor, or representatives of products covered by
the Code shall be allowed to conduct or be involved in any activity on breastfeeding
promotion, education and production of Information, Education and Communication (IEC)
materials on breastfeeding, holding of or participating as speakers in classes or seminars
for women and children activities and to avoid the use of these venues to market their
brands or company names.
[71]
SECTION 9. Research, Ethics Committee, Purpose - The DOH shall ensure that
research conducted for public policy purposes, relating to infant and young child
feeding should, at all times, be free form any commercial influence/bias; accordingly, the

635
health worker or researcher involved in such must disclose any actual or potential conflict
of interest with the company/person funding the research. In any event, such research
and its findings shall be subjected to independent peer review. x x x.
[72]
SECTION 10. Public Disclosure – For transparency purposes, a disclosure and/or
disclaimer of the sponsoring company should be done by the company itself, health worker,
researcher involved through verbal declaration during the public presentation of the research
and in print upon publication.
[73]
SECTION 32. Primary Responsibility of Health Workers – It is the primary
responsibility of the health workers to promote, protect and support breastfeeding and
appropriate infant and young child feeding. Part of this responsibility is to continuously
update their knowledge and skills on breastfeeding. No assistance, support, logistics or
training from milk companies shall be permitted.
[74]
Supra note 68.
[75]
SECTION 51. Donations Within the Scope of This Code - Donations of products,
materials, defined and covered under the Milk Code and these implementing rules and
regulations, shall be strictly prohibited.
[76]
159-A Phil. 142 (1975).
[77]
G.R. No. 159149, June 26, 2006, 492 SCRA 638.
[78]
Smart Communications, Inc. v. National Telecommunications Commission, 456 Phil.
145, 155-156 (2003).
[79]
Yazaki Torres Manufacturing, Inc. v. Court of Appeals, G.R. No. 130584, June 27,
2006, 493 SCRA 86, 97.
[80]
Supra note 78, at 156.
[81]
Petitioner's Memorandum.
[82]
SECTION 4. Declaration of Principles – The following are the underlying principles from
which the revised rules and regulations are premised upon:
xxxx
(f) Advertising, promotions, or sponsorships of infant formula, breastmilk substitutes
and other related products are prohibited.
[83]
SECTION 4. Declaration of Principles – x x x
(i) Milk companies, and their representatives, should not form part of any policymaking
body or entity in relation to the advancement of breastfeeding.
[84]
SECTION 5. x x x x (w) “Milk Company” shall refer to the owner, manufacturer,
distributor, of infant formula, follow-up milk, milk formula, milk
supplement, breastmilksubstitute or replacement, or by any other description of
such nature, including their representatives who promote or otherwise advance their
commercial interests in marketing those products; x x x.
[85]
SECTION 11. Prohibition – No advertising, promotions, sponsorships, or marketing
materials and activities for breastmilk substitutes intended for infants and young
children up to twenty-four (24) months, shall be allowed, because they tend to convey or
give subliminal messages or impressions that undermine breastmilk and breastfeeding or
otherwise exaggerate breastmilk substitutes and/or replacements, as well as related products
covered within the scope of this Code.
[86]
Supra note 70.
[87]
Supra note 73.
[88]
SECTION 46. Administrative Sanctions. – The following administrative sanctions shall
be imposed upon any person, juridical or natural, found to have violated the provisions
of the Code and its implementing Rules and Regulations:
a) 1st violation – Warning;

636
b) 2nd violation – Administrative fine of a minimum of Ten Thousand
(P10,000.00) to Fifty Thousand (P50,000.00) Pesos, depending on the gravity and
extent of the violation, including the recall of the offending product;
(c) 3rd violation – Administrative Fine of a minimum of Sixty Thousand
(P60,000.00) to One Hundred Fifty Thousand (P150,000.00) Pesos, depending on the
gravity and extent of the violation, and in addition thereto, the recall of the offending
product, and suspension of the Certificate of Product Registration (CPR);
(d) 4th violation –Administrative Fine of a minimum of Two Hundred
Thousand (P200,000.00) to Five Hundred (P500,000.00) Thousand Pesos, depending on
the gravity and extent of the violation; and in addition thereto, the recall of the product,
revocation of the CPR, suspension of the License to Operate (LTO) for one year;
(e) 5th and succeeding repeated violations – Administrative Fine of One
Million (P1,000,000.00) Pesos, the recall of the offending product, cancellation of the CPR,
revocation of the License to Operate (LTO) of the company concerned, including the
blacklisting of the company to be furnished the Department of Budget and Management
(DBM) and the Department of Trade and Industry (DTI);
(f) An additional penalty of Two Thou-sand Five Hundred (P2,500.00)
Pesos per day shall be made for every day the violation continues after having received
the order from the IAC or other such appropriate body, notifying and penalizing the
company for the infraction.
For purposes of determining whether or not there is “repeated” violation, each
product violation belonging or owned by a company, including those of their subsidiaries,
are deemed to be violations of the concerned milk company and shall not be based on the
specific violating product alone.
[89]
SECTION 52. Other Donations By Milk Companies Not Covered by this Code -
Donations of products, equipments, and the like, not otherwise falling within the scope of
this Code or these Rules, given by milk companies and their agents, representatives,
whether in kind or in cash, may only be coursed through the Inter Agency
Committee (IAC), which shall determine whether such donation be accepted or
otherwise.
[90]
Eastern Assurance & Surety Corporation v. Land Transportation Franchising and
Regulatory Board, 459 Phil. 395, 399 (2003).
[91]
G.R. No. 156041, February 21, 2007.

637
Republic of the Philippines
Supreme Court
Manila

EN BANC

ROLEX SUPLICO, G.R. No. 178830

Petitioner,

Present:

– versus –

PUNO, C.J.,

QUISUMBING,

NATIONAL ECONOMIC ANDDEVELOPMENT YNARES-SANTIAGO,


AUTHORITY, represented by NEDA
SECRETARY ROMULO L. NERI, and the CARPIO,
NEDA-INVESTMENT COORDINATION
AUSTRIA-MARTINEZ,
COMMITTEE, DEPARTMENT OF
TRANSPORTATION ANDCOMMUNICATIONS CORONA,
(DOTC), represented by DOTC SECRETARY
LEANDRO MENDOZA, including the CARPIO MORALES,
COMMISSION ON
INFORMATIONAND COMMUNICATIONS AZCUNA,
TECHNOLOGY, headed by its Chairman,
TINGA,
RAMON P. SALES, THE
TELECOMMUNICATIONS CHICO-NAZARIO,*
OFFICE, BIDS AND AWARDS FOR
INFORMATION ANDCOMMUNICATIONS VELASCO, JR.,
TECHNOLOGY (ICT), headed by DOTC
ASSISTANT SECRETARY ELMER A. SONEJA NACHURA,
as Chairman, and the TECHNICAL
REYES,
WORKING GROUP FOR ICT, AND DOTC
ASSISTANT SECRETARY LORENZO LEONARDO-DE CASTRO, and
FORMOSO, AND ALLOTHER OPERATING
UNITS OF THE DOTC FOR BRION, JJ.
INFORMATIONAND COMMUNICATIONS
TECHNOLOGY, and ZTE CORPORATION,
AMSTERDAM HOLDINGS,
INC., AND ALLPERSONS ACTING IN THEIR
BEHALF, Promulgated

638
Respondents.

July 14, 2008

x------------------------------------------------------------------------------------------- x

AMSTERDAM HOLDINGS, INC., and NATHANIEL G.R. No. 179317


SAUZ,

Petitioners,

– versus –

DEPARTMENT OF
TRANSPORTATION ANDCOMMUNICATIONS,
SECRETARY LEANDRO MENDOZA, COMMISSION
ON INFORMATION ANDCOMMUNICATIONS
TECHNOLOGY, and ASSISTANT SECRETARY
LORENZO FORMOSO III,

Respondents.

x------------------------------------------------------------------------------------------- x

GALELEO P. ANGELES, VICENTE C. G.R. No. 179613


ANGELES, JOB FLORANTE L. CASTILLO, TRINI
ANNE G. NIEVA, ROY ALLAN T. ARELLANO,
CARLO MAGNO M. REONAL, ETHEL B. REGADIO,
RAENAN B. MALIG, ANDVINALYN M. POTOT,
639
TOGETHER WITH LAWYERS AND ADVOCATES
FOR ACCOUNTABILITY, TRANSPARENCY,
INTEGRITY ANDGOOD GOVERNANCE (LATIGO),

Petitioners,

– versus –

DEPARTMENT OF
TRANSPORTATION ANDCOMMUNICATIONS
(DOTC), represented by DOTC SECRETARY
LEANDRO MENDOZA, and
ZHONGXING EQUIPMENT
(ZTE) COMPANY,LTD., AND ANY AND ALL PERSONS
ACTING ON THEIR BEHALF,

Respondents.

x------------------------------------------------------------------------------------------- x

RESOLUTION

REYES, R.T., J.:

Under consideration is the Manifestation and Motion[1] dated October 26, 2007 of the
Office of the Solicitor General (OSG) which states:

640
The Office of the Solicitor General (OSG) respectfully avers that in an
Indorsement dated October 24, 2007, the Legal Service of the Department of
Transportation and Communications (DOTC) has informed it of the Philippine
Government’s decision not to continue with the ZTE National Broadband Network
Project (see attachment[2]). That said, there is no more justiciable controversy for
this Honorable Court to resolve. WHEREFORE, public respondents respectfully pray
that the present petitions be DISMISSED.

On November 13, 2007, the Court noted the OSG’s manifestation and motion and
required petitioners in G.R. Nos. 178830, 179317, and 179613 to comment.

On December 6, 2007, Rolex Suplico, petitioner in G.R. No. 178830, filed his Consolidated
Reply and Opposition,[3]opposing the aforequoted OSG Manifestation and Motion, arguing that:

66. Aside from the fact that the Notes of the Meeting Between
President Gloria Macapagal-Arroyo and Chinese President Hu Jintao held 2 October
2007 were not attached to the 26 October 2007 Manifestation and Motion – thus
depriving petitioners of the opportunity to comment thereon – a mere verbally
requested 1st Indorsement is not sufficient basis for the conclusion that the ZTE-
DOTC NBN deal has been permanently scrapped.

67. Suffice to state, said 1st Indorsement is glaringly self-serving,


especially without the Notes of the Meeting Between President Gloria Macapagal-
Arroyo and Chinese President Hu Jintao to support its allegations or other proof of
the supposed decision to cancel the ZTE-DOTC NBN deal. Public respondents can
certainly do better than that.[4]

Petitioner Suplico further argues that:

641
79. Assuming arguendo that some aspects of the present Petition have
been rendered moot (which is vehemently denied), this Honorable Court, consistent
with well-entrenched jurisprudence, may still take cognizance thereof.[5]

Petitioner Suplico cites this Court’s rulings in Gonzales v. Chavez,[6] Rufino v.


[7] [8]
Endriga, and Alunan III v. Mirasol that despite their mootness, the Court nevertheless took
cognizance of these cases and ruled on the merits due to the Court’s symbolic function of
educating the bench and the bar by formulating guiding and controlling principles, precepts,
doctrines, and rules.

On January 31, 2008, Amsterdam Holdings, Inc. (AHI) and Nathaniel Sauz, petitioners in
G.R. No. 179317, also filed their comment expressing their sentiments, thus:

3. First of all, the present administration has never been known


for candor. The present administration has a very nasty habit of not keeping its
word. It says one thing, but does another.

4. This being the case, herein petitioners are unable to bring


themselves to feel even a bit reassured that the government, in the event that the
above-captioned cases are dismissed, will not backtrack, re-transact, or even
resurrect the now infamous NBN-ZTE transaction. This is especially relevant since
what was attached to the OSG’s Manifestation and Motion was a mere one (1) page
written communication sent by the Department of Transportation and
Communications (DOTC) to the OSG, allegedly relaying that the Philippine
Government has decided not to continue with the NBN project “x x x due to several
reasons and constraints.”

Petitioners AHI and Sauz further contend that because of the transcendental importance
of the issues raised in the petition, which among others, included the President’s use of the
power to borrow, i.e., to enter into foreign loan agreements, this Court should take cognizance
of this case despite its apparent mootness.

642
On January 15, 2008, the Court required the OSG to file respondents’ reply to petitioners’
comments on its manifestation and motion.

On April 18, 2008, the OSG filed respondents’ reply, reiterating their position that for a
court to exercise its power of adjudication, there must be an actual case or controversy – one
which involves a conflict of legal rights, an assertion of opposite legal claims susceptible of
judicial resolution; the case must not be moot or academic or based on extra-legal or other
similar considerations not cognizable by a court of justice.[9]

Respondents also insist that there is no perfected contract in this case that would
prejudice the government or public interest. Explaining the nature of the NBN Project as an
executive agreement, respondents stress that it remained in the negotiation stage. The
conditions precedent[10] for the agreement to become effective have not yet been complied with.

Respondents further oppose petitioners’ claim of the right to information, which they
contend is not an absolute right. They contend that the matters raised concern executive policy,
a political question which the judicial branch of government would generally hesitate to pass
upon.

On July 2, 2008, the OSG filed a Supplemental Manifestation and Motion. Appended to it
is the Highlights from the Notes of Meeting between President Gloria Macapagal-Arroyo and
Chinese President Hu Jintao, held in XI Jiao Guesthouse, Shanghai,China, on October 2,
2007. In the Notes of Meeting, the Philippine Government conveyed its decision not to continue
with the ZTE National Broadband Network Project due to several constraints. The same Notes
likewise contained President Hu Jintao’s expression of understanding of the Philippine
Government decision.

We resolve to grant the motion.

643
Firstly, the Court notes the triple petitions to be for certiorari, prohibition and mandamus,
with application for the issuance of a Temporary Restraining Order (TRO) and/or Preliminary
Injunction. The individual prayers in each of the three (3) consolidated petitions are:

G.R. No. 178830

WHEREFORE, it is respectfully prayed of this Honorable Court:

1. Upon the filing of this Petition, pursuant to the second


paragraph of Rule 58, Section 5 of the Rules of Court,
issueforthwith an ex parte temporary restraining order enjoining
respondents, their subordinates, agents, representatives and any and
all persons acting on their behalf from pursuing, entering into
indebtedness, disbursing funds, and implementing the ZTE-DOTC
Broadband Deal;

2. Compel respondents, upon Writ of


Mandamus, to forthwith produce and furnish petitioner or his
undersigned counsel a certified true copy of the contract or agreement
covering the NBN project as agreed upon with ZTE Corporation;

3. Schedule Oral Arguments in the present case pursuant to


Rule 49 in relation to Section 2, Rule 56 of the revised Rules of Court;
and,

4. Annul and set aside the award of the ZTE-DOTC


Broadband Deal, and compel public respondents to forthwith comply
with pertinent provisions of law regarding procurement of government
ICT contracts and public bidding for the NBN contract.[11] (Emphasis
supplied)

G.R. No. 179317

WHEREFORE, petitioners Amsterdam Holdings, Inc., and Nathaniel Sauz


respectfully pray as follows:

A. upon the filing of this Petition for Mandamus and conditioned


upon the posting of a bond in such amount as the Honorable Court
may fix, a temporary restraining order and/or writ of preliminary
injunction be issued directing the Department of Transportation and
Communication, the Commission on Information and Communications
Technology, all other government agencies and instrumentalities, their
officers, employees, and/or other persons acting for and on their
behalf to desist during the pendency of the instant Petition for
Mandamus from entering into any other agreements and from
commencing with any kind, sort, or specie of activity in connection
with the National Broadband Network Project;
644
B. the instant Petition for Mandamus be given due course;
and,

C. after due consideration of all relevant issues, judgment


be rendered directing respondents to allow herein petitioners access to
all agreements entered into with the Government of China, the ZTE
Corporation, and/or other entities, government instrumentalities,
and/or individuals with regard to the National Broadband Network
Project.[12] (Emphasis supplied)

G.R. No. 179613

WHEREFORE, it is respectfully prayed of this Honorable Court to:

1. Compel respondents, upon Writ of


Mandamus, to forthwith produce and furnish petitioner or his
undersigned counsel a certified true copy of the contract or agreement
covering the NBN project as agreed upon with ZTE Corporation;

2. Schedule Oral Arguments in the present case pursuant to


Rule 49 in relation to Section 2, Rule 56 of the Revised Rules of Court;

3. Annul and set aside the award of the contract for the
national broadband network to respondent ZTE Corporation, upon the
ground that said contract, as well as the procedures resorted to
preparatory to the execution thereof, is contrary to the Constitution, to
law and to public policy;

4. Compel public respondent to forthwith comply with pertinent


provisions of law regarding procurement of government infrastructure
projects, including public bidding for said contract to undertake the
construction of the national broadband network.[13] (Emphasis
supplied)

On September 11, 2007, the Court issued a TRO[14] in G.R. No. 178830, enjoining the
parties from “pursuing, entering into indebtedness, disbursing funds, and implementing the ZTE-
DOTC Broadband Deal and Project” as prayed for. Pertinent parts of the said Order read:

WHEREAS, the Supreme Court, on 11 September 2007, adopted a


resolution in the above-entitled case, to wit:

645
“G.R. No. 178830 (Rolex Suplico vs. National Economic and
Development Authority, represented by NEDA Secretary Romulo L.
Neri, and the NEDA Investment Coordination Committee, Department
of Transportation and Communications (DOTC), represented by DOTC
Secretary Leandro Mendoza, including the Commission on Information
and Communications Technology, headed by its Chairman, Ramon P.
Sales, The Telecommunications Office, Bids and Awards for
Information and Communications Technology Committee (ICT),
headed by DOTC Assistant Secretary Elmer A. Soneja as Chairman,
and The Technical Working Group for ICT, and DOTC Assistant
Secretary Lorenzo Formoso, and All Other Operating Units of the DOTC
for Information and Communications Technology, and ZTE
Corporation, Amsterdam Holdings, Inc., and ARESCOM, Inc.—Acting
on the instant petition with prayer for temporary restraining order
and/or writ of preliminary injunction, the Court Resolved, without
giving due course to the petition, to

xxxx

(d) Issue a TEMPORARY RESTRAINING


ORDER, effective immediately and continuing until further
orders from this Court, enjoining the (i) National
Economic and Development Authority, (ii) NEDA-
Investment Coordination Committee, (iii) Department of
Transportation and Communications, Commission on
Information and Communications Technology, (iv)
Telecommunications Office, Bids and Awards for
Information and Communications Technology Committee
(ICT), (v) Technical Working Group for ICT, and all other
Operating Units of the DOTC for Information and
Communications Technology, (vi) ZTE Corporation; (vii)
Amsterdam Holdings, Inc., and (viii) ARESCOM, Inc., and
any and all persons acting on their behalf from ‘pursuing,
entering into indebtedness, disbursing funds, and
implementing the ZTE-DOTC Broadband Deal and Project’
as prayed for.”

NOW THEREFORE, effective immediately and continuing until further orders


from this Court, You, Respondents (i) National Economic and Development
646
Authority, (ii) NEDA-Investment Coordination Committee, (iii) Department of
Transportation and Communications, Commission on Information and
Communications Technology, (iv) Telecommunications Office, Bids and
Awards for Information and Communications Technology Committee (ICT), (v)
Technical Working Group for ICT, and all other Operating Units of the DOTC for
Information and Communications Technology, (vi) ZTE Corporation; (vii)
Amsterdam Holdings, Inc., and (viii) ARESCOM, Inc., and any and all persons acting
on their behalf are hereby ENJOINED from “pursuing, entering into indebtedness,
disbursing funds, and implementing the ZTE-DOTC Broadband Deal and Project” as
prayed for.[15] (Emphasis supplied.)

Petitioners in G.R. Nos. 178830 and 179613 pray that they be furnished certified true
copies of the “contract or agreement covering the NBN project as agreed upon with ZTE
Corporation.” It appears that during one of the Senate hearings on the NBN project, copies of
the supply contract[16] were readily made available to petitioners.[17] Evidently, the said prayer
has been complied with and is, thus, mooted.

When President Gloria Macapagal-Arroyo, acting in her official capacity during the
meeting held on October 2, 2007 in China, informed China’s President Hu Jintao that the
Philippine Government had decided not to continue with the ZTE-National Broadband Network
(ZTE-NBN) Project due to several reasons and constraints, there is no doubt that all the other
principal prayers in the three petitions (to annul, set aside, and enjoin the implementation of the
ZTE-NBN Project) had also become moot.

Contrary to petitioners’ contentions that these declarations made by officials belonging to


the executive branch on the Philippine Government’s decision not to continue with the ZTE-NBN
Project are self-serving, hence, inadmissible, the Court has no alternative but to take judicial
notice of this official act of the President of the Philippines.

Section 1, Rule 129 of the Rules of Court provides:

SECTION 1. Judicial Notice, when mandatory. – A court shall take judicial


notice, without introduction of evidence, of the existence and territorial extent of
states, their political history, forms of government and symbols of nationality, the
law of nations, the admiralty and maritime courts of the world and their seals, the
political constitution and history of the Philippines, the official acts of
the legislative,executive and judicial departments of the Philippines, the laws of
nature, the measure of time, and the geographical divisions. (Emphasis supplied)

647
Under the rules, it is mandatory and the Court has no alternative but to take judicial
notice of the official acts of the President of the Philippines, who heads the executive branch of
our government. It is further provided in the above-quoted rule that the court shall take judicial
notice of the foregoing facts without introduction of evidence. Since we consider the act of
cancellation by President Macapagal-Arroyo of the proposed ZTE-NBN Project during the meeting
of October 2, 2007 with the Chinese President in China as an official act of the executive
department, the Court must take judicial notice of such official act without need of evidence.

In David v. Macapagal-Arroyo,[18] We took judicial notice of the announcement by the


Office of the President banning all rallies and canceling all permits for public assemblies following
the issuance of Presidential Proclamation No. 1017 and General Order No. 5.

In Estrada v. Desierto,[19] the Court also resorted to judicial notice in resolving the factual
ingredient of the petition.

Moreover, under Section 2, paragraph (m) of Rule 131 of the Rules of Court, the official
duty of the executive officials[20] of informing this Court of the government’s decision not to
continue with the ZTE-NBN Project is also presumed tohave been regularly performed, absent
proof to the contrary. Other than petitioner AHI’s unsavory insinuation in its comment, the
Court finds no factual or legal basis to disregard this disputable presumption in the present
instance.

Concomitant to its fundamental task as the ultimate citadel of justice and legitimacy is the
judiciary’s role of strengthening political stability indispensable to progress and national
development. Pontificating on issues which no longer legitimately constitute an actual case or
controversy will do more harm than good to the nation as a whole. Wise exercise of judicial
discretion militates against resolving the academic issues, as petitioners want this Court to
do. This is especially true where, as will be further discussed, the legal issues raised cannot be
resolved without previously establishing the factual basis or antecedents.

Judicial power presupposes actual controversies, the very antithesis of mootness. In


the absence of actual justiciable controversies or disputes, the Court generally opts to refrain
from deciding moot issues. Where there is no more live subject of controversy, the Court ceases
to have a reason to render any ruling or make any pronouncement.

648
Kapag wala nang buhay na kaso, wala nang dahilan para magdesisyon ang Husgado.

In Republic Telecommunications Holdings, Inc. v. Santiago,[21] the lone issue tackled by


the Court of Appeals (CA) was whether the Securities Investigation and Clearing Department
(SICD) and Securities and Exchange Commission (SEC) en banc committed reversible error in
issuing and upholding, respectively, the writ of preliminary injunction. The writ enjoined the
execution of the questioned agreements between Qualcomm, Inc. and Republic
Telecommunications Holdings, Inc. (RETELCOM). The implementation of the agreements was
restrained through the assailed orders of the SICD and the SEC en banc which, however, were
nullified by the CA decision. Thus, RETELCOM elevated the matter to this Court praying for the
reinstatement of the writ of preliminary injunction of the SICD and the SEC en banc. However,
before the matter was finally resolved, Qualcomm, Inc. withdrew from the negotiating table. Its
withdrawal had thwarted the execution and enforcement of the contracts. Thus, the resolution
of whether the implementation of said agreements should be enjoined became no longer
necessary.

Equally applicable to the present case is the Court ruling in the above-cited Republic
Telecommunications. There We held, thus:

Indeed, the instant petition, insofar as it assails the Court of Appeals’


Decision nullifying the orders of the SEC en banc and the SICD, has been rendered
moot and academic. To rule, one way or the other, on the correctness of the
questioned orders of the SEC en banc and the SICD will be indulging in a theoretical
exercise that has no practical worth in view of the supervening event.

The rule is well-settled that for a court to exercise its power of adjudication,
there must be an actual case or controversy – one which involves a conflict of legal
rights, an assertion of opposite legal claims susceptible of judicial resolution; the
case must not be moot or academic or based on extra-legal or other similar
considerations not cognizable by a court of justice. Where the issue has become
moot and academic, there is no justiciable controversy, and an adjudication thereon
would be of no practical use or value as courts do not sit to adjudicate mere
academic questions to satisfy scholarly interest, however intellectually challenging.
649
In the ultimate analysis, petitioners are seeking the reinstatement of the writ
of injunction to prevent the concerned parties from pushing through with
transactions with Qualcomm, Inc. Given that Qualcomm, Inc. is no longer
interested in pursuing the contracts, there is no actual substantial relief to which
petitioners would be entitled and which would be negated by the dismissal of the
petition.

The Court likewise finds it unnecessary to rule whether the assailed Court of
Appeals’ Decision had the effect of overruling the Court’s Resolution dated 29
January 1999, which set aside the TRO issued by the appellate court.

A ruling on the matter practically partakes of a mere advisory opinion, which


falls beyond the realm of judicial review. The exercise of the power of judicial
review is limited to actual cases and controversies. Courts have no authority to
pass upon issues through advisory opinions or to resolve hypothetical or feigned
problems.

While there were occasions when the Court passed upon issues although
supervening events had rendered those petitions moot and academic, the instant
case does not fall under the exceptional cases. In those cases, the Court was
persuaded to resolve moot and academic issues to formulate guiding and
controlling constitutional principles, precepts, doctrines or rules for future guidance
of both bench and bar.

In the case at bar, the resolution of whether a writ of preliminary injunction


may be issued to prevent the implementation of the assailed contracts calls for an
appraisal of factual considerations which are peculiar only to the transactions and
parties involved in this controversy. Except for the determination of whether
petitioners are entitled to a writ of preliminary injunction which is now moot, the
issues raised in this petition do not call for a clarification of any constitutional
principle or the interpretation of any statutory provision.[22]

Secondly, even assuming that the Court will choose to disregard the foregoing
considerations and brush aside mootness, the Court cannot completely rule on the merits of the
case because the resolution of the three petitions involves settling factual issues which definitely
650
requires reception of evidence. There is not an iota of doubt that this may not be done by this
Court in the first instance because, as has been stated often enough, this Court is not a trier of
facts.

Ang pagpapasiya sa tatlong petisyon ay nangangailangan ng paglilitis na hindi gawain ng


Hukumang ito.

Respondent ZTE, in its Comment in G.R. No. 178830,[23] correctly pointed out that since
petitioner Suplico filed his petition directly with this Court, without prior factual findings made by
any lower court, a determination of pertinent and relevant facts is needed. ZTE enumerated
some of these factual issues, to wit:

(1) Whether an executive agreement has been reached between the


Philippine and Chinese governments over the NBN Project;

(2) Whether the ZTE Supply Contract was entered into by the Republic of
the Philippines, through the DOTC, and ZTE International pursuant to, and as
an integral part of, the executive agreement;

(3) Whether a loan agreement for the NBN Project has actually been
executed;

(4) Whether the Philippine government required that the NBN Project be
completed under a Build-Operate-and-Transfer Scheme;

(5) Whether the AHI proposal complied with the requirements for an
unsolicited proposal under the BOT Law;

(6) Whether the Philippine government has actually earmarked public


finds for disbursement under the ZTE Supply Contract; and

(7) Whether the coverage of the NBN Project to be supplied under the
ZTE Supply Contract is more extensive than that under the AHIproposal or
such other proposal submitted therefor.[24]

Definitely, some very specific reliefs prayed for in both G.R. Nos. 178830 and 179613
require prior determination of facts before pertinent legal issues could be resolved and specific
reliefs granted.

651
In G.R. No. 178830, petitioner seeks to annul and set aside the award of the ZTE-DOTC
Broadband Deal and compel public respondents to forthwith comply with pertinent provisions of
law regarding procurement of government ICT contracts and public bidding for the NBN contract.

In G.R. No. 179613, petitioners also pray that the Court annul and set aside the award of
the contract for the national broadband network to respondent ZTE Corporation, upon the
ground that said contract, as well as the procedures resorted to preparatory to the execution
thereof, is contrary to the Constitution, to law and to public policy. They also ask the Court
tocompel public respondent to forthwith comply with pertinent provisions of law regarding
procurement of government infrastructure projects, including public bidding for said contract to
undertake the construction of the national broadband network.

It is simply impossible for this Court “to annul and set aside the award of the ZTE-DOTC
Broadband Deal” without any evidence to support a prior factual finding pointing to any violation
of law that could lead to such annulment order. For sure, the Supreme Court is not the proper
venue for this factual matter to be threshed out.

Thirdly, petitioner Suplico in G.R. No. 178830 prayed that this Court order “public
respondents to forthwith comply with pertinent provisions of law regarding procurement of
government ICT contracts and public bidding for the NBN contract.”[25] It would be too
presumptuous on the part of the Court to summarily compel public respondents to comply with
pertinent provisions of law regarding procurement of government infrastructure projects without
any factual basis or prior determination of very particular violations committed by specific
government officials of the executive branch. For the Court to do so would amount to a breach
of the norms of comity among co-equal branches of government. A perceived error cannot be
corrected by committing another error. Without proper evidence, the Court cannot just presume
that the executive did not comply with procurement laws. Should the Court allow itself to fall
into this trap, it would plainly commit grave error itself.

Magiging kapangahasan sa Hukumang ito na pilitin ang mga pinipetisyon na tumalima sa


batas sa pangongontrata ng pamahalaan kung wala pang pagtitiyak o angkop na ebidensiya ng
nagawang paglabag dito.

652
Let it be clarified that the Senate investigation in aid of legislation cannot be the basis of
Our decision which requires ajudicial finding of facts.

Justice Antonio T. Carpio takes the view that the National Broadband Network Project
should be declared null and void. The foregoing threefold reasons would suffice to address the
concern of Our esteemed colleague.

The Court is, therefore, constrained to dismiss the petitions and deny them due course
because of mootness and because their resolution requires reception of evidence which cannot
be done in an original petition brought before the Supreme Court.

WHEREFORE, the petitions are DISMISSED. The Temporary Restraining Order issued
on September 11, 2007 isDISSOLVED.

SO ORDERED.

RUBEN T. REYES

Associate Justice

WE CONCUR:

653
REYNATO S. PUNO

Chief Justice

LEONARDO A. QUISUMBING CONSUELO YNARES-SANTIAGO

Associate Justice Associate Justice

ANTONIO T. CARPIO MA. ALICIA AUSTRIA-MARTINEZ

Associate Justice Associate Justice

RENATO C. CORONA CONCHITA CARPIO MORALES

Associate Justice Associate Justice

654
ADOLFO S. AZCUNA DANTE O. TINGA

Associate Justice Associate Justice

(On official leave)

MINITA V. CHICO-NAZARIO PRESBITERO J. VELASCO, JR.

Associate Justice Associate Justice

ANTONIO EDUARDO B. NACHURA TERESITA J. LEONARDO-DE CASTRO

Associate Justice Associate Justice

ARTURO D. BRION

Associate Justice

655
CERTIFICATION

Pursuant to Section 13, Article VIII of the Constitution, I certify that the conclusions in the
above Resolution had been reached in consultation before the case was assigned to the writer of
the opinion of the Court.

REYNATO S. PUNO

Chief Justice

*
On official leave per Special Order No. 508 dated June 25, 2008.
[1]
Rollo (G.R. No. 178830), p. 1093.
[2]
1st Indorsement dated October 24, 2007 from the DOTC signed by Atty. Raquel Desiderio,
Director III, Legal Service states:
Respectfully indorsed to SOLICITOR GENERAL AGNES VST DEVANADERA (Attention:
ASSISTANT SOLICITOR GENERAL AMPARO M. CABOTAJE-TANG), herein copy of
the Highlights From the Notes of the Meeting Between President Gloria Macapagal-Arroyo
and Chinese President Hu Jintao which was held in Xi Jiao Guesthouse, Shanghai, The
People’s Republic of China on 02 October 2007 as transmitted from the Office of the
President as provided by the Department of Foreign Affairs (DFA).
As per verbal request from your honorable office we are furnishing you a copy of the
record of the said meeting which states in sum the Philippine Government’s decision not
to continue with the ZTE National Broadband Network Project due to several reasons and
constraints. It is the understanding of the DOTC that this document will form part of the
evidence that will be submitted to the Honorable Supreme Court in connection with the
cases filed against the DOTC in relation to the NBN Project.

656
Kindly refer to the attached document and respectfully request appropriate action on the
same. Thank you very much for your continued support and assistance to the
Department of Transportation and Communications.
[3]
Rollo (G.R. No. 178830), p. 1124.
[4]
Id. at 1157.
[5]
Id. at 1160.
[6]
G.R. No. 97351, February 4, 1992, 205 SCRA 816.
[7]
G.R. No. 139554, July 21, 2006, 496 SCRA 13.
[8]
G.R. No. 108399, July 31, 1997, 276 SCRA 501.
[9]
Citing Republic v. Tan, G.R. No. 145255, March 30, 2004, 426 SCRA 485, 492-493.
[10]
(a) Issuance of a Forward Obligation Authority (FOA) by the Department of Budget and
Management (DBM) of the Government of the Republic of the Philippines;
(b) Conclusion of the Loan Agreement between the Export-Import Bank of China and the
Department of Finance (DOF) of the Government of the Republic of the Philippines;
(c) Legal Opinion on the procurement process by the Department of Justice of the
Government of the Republic of the Philippines;
(d) The ratification by the Government of the Republic of the Philippines and the People’s
Republic of China of the Executive Agreement evidenced by the letter dated 02 December 2006
of Chinese Ambassador Li Jinjun to Presidential Chief of Staff Michael T. Defensor relating to the
NBN project and the letter of the NEDA Secretary dated 20 April 2007 addressed to Honorable
Minister Bo Xilai, Ministry of Commerce and Honorable Li Rougu, Chairman and President of the
Export-Import Bank of China, People’s Republic of China nominating the NBN Project.
[11]
Rollo (G.R. No. 178830), pp. 127-128.
[12]
Rollo (G.R. No. 179317), pp. 35-36.
[13]
Rollo (G.R. No. 179613), pp. 77-78.
[14]
Rollo (G.R. No. 178830), p. 232.
[15]
Id. at 233-235.
[16]
Also attached to public respondents’ Comment in G.R. No. 178830 as Annex “LL.” Id. at 537.
[17]
Id. at 589-590; Annex “OO.” Letter of Sec. Leandro Mendoza, DOTC, to Sen. Allan Peter
Cayetano dated September 25, 2007. In response to a request of the Senate Blue Ribbon
Committee to be furnished with the copy of the supply contract, DOTC Secretary Mendoza
informed Sen. Allan Peter Cayetano that the pertinent documents were transmitted as publicly
requested, and the same were distributed to guests who requested a copy.
[18]
G.R. No. 171396, May 3, 2006, 489 SCRA 160.
[19]
G.R. No. 146710, March 2, 2001, 353 SCRA 452.
[20]
The Highlights from the notes of the meeting between President Gloria Macapagal-Arroyo and
Chinese President Hu Jintao which was held in the Xi Jiao Guesthouse, Shanghai, China
on October 2, 2007 was transmitted by the Office of the President through the Department of
Foreign Affairs (DFA) to the Department of Transportation and Communications (DOTC), which
in turn transmitted the communication through 1st Indorsement dated October 24,
2007 (Rollo [G.R. No. 178830], p. 1097) to the Office of the Solicitor General, which in informed
this Court, through its Manifestation and Motion dated October 26, 2007 (Id. at 1093).
[21]
G.R. No. 140338, August 7, 2007, 529 SCRA 232.
[22]
Republic Telecommunications Holdings, Inc. v. Santiago, id. at 242-244.
[23]
Rollo (G.R. No. 178830), p. 676.
[24]
Id. at 720-721.
[25]
Id. at 127-128.

657
EN BANC

GOVERNMENT OF HONG KONG SPECIAL G.R. No. 153675


ADMINISTRATIVE REGION, represented by the
Philippine Department of
Justice, Petitioner, Present:

PUNO, C.J.,

QUISUMBING,
YNARES-SANTIAGO,

SANDOVAL-GUTIERREZ,

CARPIO,

AUSTRIA-MARTINEZ,

CORONA,

- versus - CARPIO MORALES,

CALLEJO, SR.,

AZCUNA,

TINGA,

CHICO-NAZARIO,

GARCIA,

VELASCO, JR., and

NACHURA, JJ.

HON. FELIXBERTO T. OLALIA, JR. and JUAN


ANTONIO MUÑOZ,
Promulgated:
Respondents.

April 19, 2007

658
x-------------------------------------------------------------------------------------x

DECISION

SANDOVAL-GUTIERREZ, J.:

For our resolution is the instant Petition for Certiorari under Rule 65 of the 1997 Rules of
Civil Procedure, as amended, seeking to nullify the two Orders of the Regional Trial Court (RTC),
Branch 8, Manila (presided by respondent Judge Felixberto T. Olalia, Jr.) issued in Civil Case No.
99-95773. These are: (1) the Order dated December 20, 2001 allowing Juan Antonio Muñoz,
private respondent, to post bail; and (2) the Order dated April 10, 2002 denying the motion to
vacate the said Order ofDecember 20, 2001 filed by the Government of Hong Kong Special
Administrative Region, represented by the Philippine Department of Justice (DOJ),
petitioner. The petition alleges that both Orders were issued by respondent judge with grave
abuse of discretion amounting to lack or excess of jurisdiction as there is no provision in the
Constitution granting bail to a potential extraditee.

The facts are:

On January 30, 1995, the Republic of the Philippines and the then British Crown Colony
of Hong Kong signed an “Agreement for the Surrender of Accused and Convicted Persons.” It
took effect on June 20, 1997.

On July 1, 1997, Hong Kong reverted back to the People’s Republic of China and became
the Hong Kong Special Administrative Region.

Private respondent Muñoz was charged before the Hong Kong Court with three (3) counts
of the offense of “accepting an advantage as agent,” in violation of Section 9 (1) (a) of the
Prevention of Bribery Ordinance, Cap. 201 of Hong Kong. He also faces seven (7) counts of the
offense of conspiracy to defraud, penalized by the common law of Hong Kong. On August 23,

659
1997 and October 25, 1999, warrants of arrest were issued against him. If convicted, he faces a
jail term of seven (7) to fourteen (14) years for each charge.

On September 13, 1999, the DOJ received from the Hong Kong Department of Justice a
request for the provisional arrest of private respondent. The DOJ then forwarded the request to
the National Bureau of Investigation (NBI) which, in turn, filed with the RTC of Manila, Branch 19
an application for the provisional arrest of private respondent.

On September 23, 1999, the RTC, Branch 19, Manila issued an Order of Arrest against
private respondent. That same day, the NBI agents arrested and detained him.

On October 14, 1999, private respondent filed with the Court of Appeals a petition
for certiorari, prohibition andmandamus with application for preliminary mandatory injunction
and/or writ of habeas corpus questioning the validity of the Order of Arrest.

On November 9, 1999, the Court of Appeals rendered its Decision declaring the Order of
Arrest void.

On November 12, 1999, the DOJ filed with this Court a petition for review on certiorari,
docketed as G.R. No. 140520, praying that the Decision of the Court of Appeals be reversed.

On December 18, 2000, this Court rendered a Decision granting the petition of the DOJ
and sustaining the validity of the Order of Arrest against private respondent. The Decision
became final and executory on April 10, 2001.

Meanwhile, as early as November 22, 1999, petitioner Hong Kong Special Administrative
Region filed with the RTC of Manila a petition for the extradition of private respondent, docketed
as Civil Case No. 99-95733, raffled off to Branch 10, presided by Judge Ricardo Bernardo,

660
Jr. For his part, private respondent filed, in the same case,- a petition for bail which was
opposed by petitioner.

After hearing, or on October 8, 2001, Judge Bernardo, Jr. issued an Order denying the
petition for bail, holding that there is no Philippine law granting bail in extradition cases and that
private respondent is a high “flight risk.”

On October 22, 2001, Judge Bernardo, Jr. inhibited himself from further hearing Civil
Case No. 99-95733. It was then raffled off to Branch 8 presided by respondent judge.

On October 30, 2001, private respondent filed a motion for reconsideration of the Order
denying his application for bail. This was granted by respondent judge in an Order
dated December 20, 2001 allowing private respondent to post bail, thus:

In conclusion, this Court will not contribute to accused’s further erosion of


civil liberties. The petition for bail is granted subject to the following conditions:

1. Bail is set at Php750,000.00 in cash with the condition that accused hereby
undertakes that he will appear and answer the issues raised in these
proceedings and will at all times hold himself amenable to orders and processes
of this Court, will further appear for judgment. If accused fails in this
undertaking, the cash bond will be forfeited in favor of the government;

2. Accused must surrender his valid passport to this Court;

3. The Department of Justice is given immediate notice and discretion of filing


its own motion for hold departure order before this Court even in extradition
proceeding; and

661
4. Accused is required to report to the government prosecutors handling this
case or if they so desire to the nearest office, at any time and day of the week;
and if they further desire, manifest before this Court to require that all the
assets of accused, real and personal, be filed with this Court soonest, with the
condition that if the accused flees from his undertaking, said assets be forfeited
in favor of the government and that the corresponding lien/annotation be noted
therein accordingly.

SO ORDERED.

On December 21, 2001, petitioner filed an urgent motion to vacate the above Order, but
it was denied by respondent judge in his Order dated April 10, 2002.

Hence, the instant petition. Petitioner alleged that the trial court committed grave abuse
of discretion amounting to lack or excess of jurisdiction in admitting private respondent to bail;
that there is nothing in the Constitution or statutory law providing that a potential extraditee has
a right to bail, the right being limited solely to criminal proceedings.

In his comment on the petition, private respondent maintained that the right to bail
guaranteed under the Bill of Rights extends to a prospective extraditee; and that extradition is a
harsh process resulting in a prolonged deprivation of one’s liberty.

Section 13, Article III of the Constitution provides that the right to bail shall not be
impaired, thus:

Sec. 13. All persons, except those charged with offenses punishable
by reclusion perpetua when evidence of guilt is strong, shall, before conviction, be
bailable by sufficient sureties, or be released on recognizance as may be provided
by law. The right to bail shall not be impaired even when the privilege of the writ
of habeas corpus is suspended. Excessive bail shall not be required.

662
Jurisprudence on extradition is but in its infancy in this jurisdiction. Nonetheless, this is
not the first time that this Court has an occasion to resolve the question of whether a
prospective extraditee may be granted bail.

In Government of United States of America v. Hon. Guillermo G. Purganan, Presiding


Judge, RTC of Manila, Branch 42, and Mark B. Jimenez, a.k.a. Mario Batacan Crespo,[1] this
Court, speaking through then Associate Justice Artemio V. Panganiban, later Chief Justice, held
that the constitutional provision on bail does not apply to extradition proceedings. It is
“available only in criminal proceedings,” thus:

x x x. As suggested by the use of the word “conviction,” the constitutional


provision on bail quoted above, as well as Section 4, Rule 114 of the Rules of
Court, applies only when a person has been arrested and detained for violation of
Philippine criminal laws. It does not apply to extradition proceedings because
extradition courts do not render judgments of conviction or acquittal.

Moreover, the constitutional right to bail “flows from the presumption of


innocence in favor of every accused who should not be subjected to the loss of
freedom as thereafter he would be entitled to acquittal, unless his guilt be proved
beyond reasonable doubt” (De la Camara v. Enage, 41 SCRA 1, 6, September 17,
1971, per Fernando, J., later CJ). It follows that the constitutional provision on
bail will not apply to a case like extradition, where the presumption of innocence is
not at issue.

The provision in the Constitution stating that the “right to bail shall not be
impaired even when the privilege of the writ of habeas corpusis suspended” does
not detract from the rule that the constitutional right to bail is available only in
criminal proceedings. It must be noted that the suspension of the privilege of the
writ of habeas corpus finds application “only to persons judicially charged for
rebellion or offenses inherent in or directly connected with invasion” (Sec. 18, Art.
VIII, Constitution). Hence, the second sentence in the constitutional provision on
bail merely emphasizes the right to bail in criminal proceedings for the

663
aforementioned offenses. It cannot be taken to mean that the right is available
even in extradition proceedings that are not criminal in nature.

At first glance, the above ruling applies squarely to private respondent’s case. However,
this Court cannot ignore the following trends in international law: (1) the growing importance
of the individual person in public international law who, in the 20th century, has gradually
attained global recognition; (2) the higher value now being given to human rights in the
international sphere; (3) the corresponding duty of countries to observe these universal human
rights in fulfilling their treaty obligations; and (4) the duty of this Court to balance the rights of
the individual under our fundamental law, on one hand, and the law on extradition, on the other.

The modern trend in public international law is the primacy placed on the worth of the
individual person and the sanctity of human rights. Slowly, the recognition that the individual
person may properly be a subject of international law is now taking root. The vulnerable
doctrine that the subjects of international law are limited only to states was dramatically eroded
towards the second half of the past century. For one, the Nuremberg and Tokyo trials after
World War II resulted in the unprecedented spectacle of individual defendants for acts
characterized as violations of the laws of war, crimes against peace, and crimes against
humanity. Recently, under the Nuremberg principle, Serbian leaders have been persecuted for
war crimes and crimes against humanity committed in the former Yugoslavia. These significant
events show that the individual person is now a valid subject of international law.

On a more positive note, also after World War II, both international organizations and
states gave recognition and importance to human rights. Thus, on December 10, 1948, the
United Nations General Assembly adopted the Universal Declaration of Human Rights in which
the right to life, liberty and all the other fundamental rights of every person were
proclaimed. While not a treaty, the principles contained in the said Declaration are now
recognized as customarily binding upon the members of the international community. Thus,
in Mejoff v. Director of Prisons,[2] this Court, in granting bail to a prospective deportee, held that
under the Constitution,[3] the principles set forth in that Declaration are part of the law of the
land. In 1966, the UN General Assembly also adopted the International Covenant on Civil and
664
Political Rights which the Philippines signed and ratified. Fundamental among the rights
enshrined therein are the rights of every person to life, liberty, and due process.

The Philippines, along with the other members of the family of nations, committed to
uphold the fundamental human rights as well as value the worth and dignity of every
person. This commitment is enshrined in Section II, Article II of our Constitution which
provides: “The State values the dignity of every human person and guarantees full respect for
human rights.” The Philippines, therefore, has the responsibility of protecting and promoting the
right of every person to liberty and due process, ensuring that those detained or arrested can
participate in the proceedings before a court, to enable it to decide without delay on the legality
of the detention and order their release if justified. In other words, the Philippine authorities are
under obligation to make available to every person under detention such remedies which
safeguard their fundamental right to liberty. These remedies include the right to be admitted to
bail. While this Court in Purganan limited the exercise of the right to bail to criminal
proceedings, however, in light of the various international treaties giving recognition and
protection to human rights, particularly the right to life and liberty, a reexamination of this
Court’s ruling in Purganan is in order.

First, we note that the exercise of the State’s power to deprive an individual of his liberty
is not necessarily limited to criminal proceedings. Respondents in administrative proceedings,
such as deportation and quarantine,[4] have likewise been detained.

Second, to limit bail to criminal proceedings would be to close our eyes to our
jurisprudential history. Philippine jurisprudence has not limited the exercise of the right to bail
to criminal proceedings only. This Court has admitted to bail persons who are not involved in
criminal proceedings. In fact, bail has been allowed in this jurisdiction to persons in detention
during the pendency of administrative proceedings, taking into cognizance the obligation of
the Philippinesunder international conventions to uphold human rights.

The 1909 case of US v. Go-Sioco[5] is illustrative. In this case, a Chinese facing


deportation for failure to secure the necessary certificate of registration was granted bail
pending his appeal. After noting that the prospective deportee had committed no crime, the
Court opined that “To refuse him bail is to treat him as a person who has committed the most
serious crime known to law;” and that while deportation is not a criminal proceeding, some of
the machinery used “is the machinery of criminal law.” Thus, the provisions relating to bail was
applied to deportation proceedings.

In Mejoff v. Director of Prisons[6] and Chirskoff v. Commission of Immigration,[7] this Court


ruled that foreign nationals against whom no formal criminal charges have been filed may be
released on bail pending the finality of an order of deportation. As previously stated, the Court
in Mejoff relied upon the Universal declaration of Human Rights in sustaining the detainee’s right
to bail.

665
If bail can be granted in deportation cases, we see no justification why it should not also
be allowed in extradition cases. Likewise, considering that the Universal Declaration of Human
Rights applies to deportation cases, there is no reason why it cannot be invoked in extradition
cases. After all, both are administrative proceedings where the innocence or guilt of the person
detained is not in issue.

Clearly, the right of a prospective extraditee to apply for bail in this jurisdiction must be
viewed in the light of the various treaty obligations of the Philippines concerning respect for the
promotion and protection of human rights. Under these treaties, the presumption lies in favor of
human liberty. Thus, the Philippines should see to it that the right to liberty of every individual
is not impaired.

Section 2(a) of Presidential Decree (P.D.) No. 1069 (The Philippine Extradition Law)
defines “extradition” as “the removal of an accused from the Philippines with the object of
placing him at the disposal of foreign authorities to enable the requesting state or government to
hold him in connection with any criminal investigation directed against him or the execution of a
penalty imposed on him under the penal or criminal law of the requesting state or government.”

Extradition has thus been characterized as the right of a foreign power, created by treaty,
to demand the surrender of one accused or convicted of a crime within its territorial jurisdiction,
and the correlative duty of the other state to surrender him to the demanding state.[8] It is not
a criminal proceeding.[9] Even if the potential extraditee is a criminal, an extradition proceeding
is not by its nature criminal, for it is not punishment for a crime, even though such punishment
may follow extradition.[10] It is sui generis, tracing its existence wholly to treaty obligations
[11]
between different nations. It is not a trial to determine the guilt or innocence of the potential
extraditee.[12] Nor is it a full-blown civil action, but one that is merely administrative in
[13]
character. Its object is to prevent the escape of a person accused or convicted of a crime and
to secure his return to the state from which he fled, for the purpose of trial or punishment.[14]

But while extradition is not a criminal proceeding, it is characterized by the


following: (a) it entails a deprivation of liberty on the part of the potential extraditee and
(b) the means employed to attain the purpose of extradition is also “the machinery of criminal
law.” This is shown by Section 6 of P.D. No. 1069 (The Philippine Extradition Law) which
mandates the “immediate arrest and temporary detention of the accused” if such “will best serve
the interest of justice.” We further note that Section 20 allows the requesting state “in case of
urgency” to ask for the “provisional arrest of the accused, pending receipt of the request for
666
extradition;” and that release from provisional arrest “shall not prejudice re-arrest and
extradition of the accused if a request for extradition is received subsequently.”

Obviously, an extradition proceeding, while ostensibly administrative, bears all earmarks


of a criminal process. A potential extraditee may be subjected to arrest, to a prolonged restraint
of liberty, and forced to transfer to the demanding state following the proceedings. “Temporary
detention” may be a necessary step in the process of extradition, but the length of time of the
detention should be reasonable.

Records show that private respondent was arrested on September 23, 1999, and
remained incarcerated until December 20, 2001, when the trial court ordered his admission to
bail. In other words, he had been detained for over two (2) years without having been convicted
of any crime. By any standard, such an extended period of detention is a serious deprivation of
his fundamental right to liberty. In fact, it was this prolonged deprivation of liberty which
prompted the extradition court to grant him bail.

While our extradition law does not provide for the grant of bail to an extraditee, however,
there is no provision prohibiting him or her from filing a motion for bail, a right to due process
under the Constitution.

The applicable standard of due process, however, should not be the same as that in
criminal proceedings. In the latter, the standard of due process is premised on the presumption
of innocence of the accused. As Purganan correctly points out, it is from this major premise that
the ancillary presumption in favor of admitting to bail arises. Bearing in mind the purpose of
extradition proceedings, the premise behind the issuance of the arrest warrant and the
“temporary detention” is the possibility of flight of the potential extraditee. This is based on the
assumption that such extraditee is a fugitive from justice.[15] Given the foregoing, the
prospective extraditee thus bears the onus probandi of showing that he or she is not a flight risk
and should be granted bail.

667
The time-honored principle of pacta sunt servanda demands that the Philippines honor its
obligations under the Extradition Treaty it entered into with the Hong Kong Special
Administrative Region. Failure to comply with these obligations is a setback in our foreign
relations and defeats the purpose of extradition. However, it does not necessarily mean that in
keeping with its treaty obligations, the Philippines should diminish a potential extraditee’s rights
to life, liberty, and due process. More so, where these rights are guaranteed, not only by our
Constitution, but also by international conventions, to which the Philippines is a party. We
should not, therefore, deprive an extraditee of his right to apply for bail, provided that a certain
standard for the grant is satisfactorily met.

An extradition proceeding being sui generis, the standard of proof required in granting or
denying bail can neither be the proof beyond reasonable doubt in criminal cases nor the
standard of proof of preponderance of evidence in civil cases. While administrative in character,
the standard of substantial evidence used in administrative cases cannot likewise apply given the
object of extradition law which is to prevent the prospective extraditee from fleeing our
jurisdiction. In his Separate Opinion inPurganan, then Associate Justice, now Chief Justice
Reynato S. Puno, proposed that a new standard which he termed “clear and convincing
evidence” should be used in granting bail in extradition cases. According to him, this standard
should be lower than proof beyond reasonable doubt but higher than preponderance of
evidence. The potential extraditee must prove by “clear and convincing evidence” that he is not
a flight risk and will abide with all the orders and processes of the extradition court.

In this case, there is no showing that private respondent presented evidence to show that
he is not a flight risk. Consequently, this case should be remanded to the trial court to determine
whether private respondent may be granted bail on the basis of “clear and convincing
evidence.”

WHEREFORE, we DISMISS the petition. This case is REMANDED to the trial court to
determine whether private respondent is entitled to bail on the basis of “clear and convincing
evidence.” If not, the trial court should order the cancellation of his bail bond and his immediate
detention; and thereafter, conduct the extradition proceedings with dispatch.

668
SO ORDERED.

ANGELINA SANDOVAL-GUTIERREZ

Associate Justice

WE CONCUR:

REYNATO S. PUNO
Chief Justice

LEONARDO A. QUISUMBING CONSUELO YNARES-SANTIAGO

Associate Justice Associate Justice

ANTONIO T. CARPIO MA. ALICIA AUSTRIA-MARTINEZ

Associate Justice Associate Justice

RENATO C. CORONA CONCHITA CARPIO MORALES

Associate Justice Associate Justice

ROMEO J. CALLEJO, SR. ADOLFO S. AZCUNA

669
Associate Justice Associate Justice

MINITA V. CHICO-NAZARIO DANTE O. TINGA

Associate Justice Associate Justice

CANCIO C. GARCIA PRESBITERO J. VELASCO, JR.

Associate Justice Associate Justice

ANTONIO EDUARDO B. NACHURA

Associate Justice

CERTIFICATION

Pursuant to Section 13, Article VIII of the Constitution, it is hereby certified that the
conclusions in the above Decision were reached in consultation before the case was assigned to
the writer of the opinion of the Court.

REYNATO S. PUNO

Chief Justice

670
[1]
G.R. No. 148571, September 24, 2002, 389 SCRA 623, 664.
[2]
90 Phil. 70 (1951).
[3]
Sec. 2, Art. II states “The Philippines renounces war as an instrument of national
policy, adopts the generally accepted principles of international law as part of the law of
the land and adheres to the policy of peace, equality, justice, freedom, cooperation, and
amity with all nations.”
[4]
In cases involving quarantine to prevent the spread of communicable diseases, bail
is not available. See State v. Hutchinson, 18 So.2d. 723, 246 Ala. 48; Varholy v.
Sweat,15 So.2d. 267, 153 Fla. 571, Baker v. Strautz, 54 NE2d. 441, 386 lll. 360.
[5]
12 Phil. 490 (1909).
[6]
Supra, footnote 2.
[7]
90 Phil. 256 (1951).
[8]
Factor v. Laubenheimer, 290 US 276, 78 L. Ed. 315, 54 S. Ct. 101; Terlindon
v. Ames, 184 US 270, 46 L.Ed. 534, 22 S.Ct. 484; Fong Yue Ting v. US, 149 US 698, 37
L.Ed. 905, 13 S.Ct. 1016; Fitzpatrick v. Williams, 46 F2d. 40; US v. Godwin, 97 F. Supp.
252, affd. 191 F2d. 932; Dominguez v. State, 234 SW 701, 90 Tex. Crim. 92.

[9]
Secretary of Justice v. Lantion, G.R. No. 139465, October 17, 2000, 343 SCRA 377.
[10]
US ex rel Oppenheim v. Hecht, 16 F2d. 955, cert den. 273 US 969, 71 L. Ed. 883,
47 S. Ct. 572.
[11]
State v. Chase, 107 So. 541, 91 Fla. 413; State v. Quigg, 108 So. 409, 91 Fla. 197.
[12]
Benson v. McMahon, 127 US 457, 32 L. Ed. 234, 8 S. Ct. 1240; Jimenez v.
Aristequieta, 311 F2d. 547, stay den. 314 F2d. 649.
[13]
Spatola v. US, 741 F. Supp. 362, Affd. 925 F2d. 615.
[14]
Re Henderson, 145 NW 574, 27 ND 155; State ex rel Tresoder v. Remann, 4 P2d.
866, 165 Wash. 92.
[15]
Beaulieu v. Hartigan, 554 F.2d 1.

671
EN BANC

IN THE MATTER OF THE PETITION FOR G.R. No. 174340


ISSUANCE OF WRIT OF HABEAS CORPUS
OF CAMILO L. SABIO,

Petitioner,

J. ERMIN ERNEST LOUIE R. MIGUEL,

Petitioner-Relator,

- versus -

HONORABLE SENATOR RICHARD GORDON,


in his capacity as Chairman, and the
HONORABLE MEMBERS OF THE
COMMITTEE ON GOVERNMENT
CORPORATIONS AND PUBLIC
ENTERPRISES and THE COMMITTEE ON
PUBLIC SERVICES of the Senate,
HONORABLE SENATOR JUAN PONCE-
ENRILE, in his official capacity as Member,
HONORABLE MANUEL VILLAR, Senate
President, SENATE SERGEANT-AT-ARMS,
and the SENATE OF THE PHILIPPINES,

Respondents.

x ----------------------------------------------

672
-- x

PRESIDENTIAL COMMISSION ON GOOD


GOVERNMENT (PCGG) and CAMILO L.
SABIO, Chairman, NARCISO S. NARIO,
RICARDO M. ABCEDE, TERESO L. JAVIER
and NICASIO A. CONTI, Commissioners,
MANUEL ANDAL and JULIO JALANDONI,
PCGG nominees to Philcomsat Holdings
Corporation,

Petitioners,

G.R. No. 174318


- versus -

RICHARD GORDON, in his capacity as


Chairman, and MEMBERS OF THE
COMMITTEE ON GOVERNMENT
CORPORATIONS AND PUBLIC
ENTERPRISES, MEMBERS OF THE
COMMITTEE ON PUBLIC SERVICES,
SENATOR JUAN PONCE-ENRILE, in his
capacity as member of both said
Committees, MANUEL VILLAR, Senate
President, THE SENATE SERGEANT-AT-
ARMS, and SENATE OF THE PHILIPPINES,

Respondents.

x----------------------------------------------

---x

PHILCOMSAT HOLDINGS CORPORATIONS,

PHILIP G. BRODETT, LUIS K. LOKIN, JR.,

ROBERTO V. SAN JOSE, DELFIN P.

ANGCAO, ROBERTO L. ABAD, ALMA

KRISTINA ALOBBA, and JOHNNY TAN,

Petitioners,

673
- versus -

G.R. No. 174177

Present:

SENATE COMMITTEE ON GOVERNMENT PANGANIBAN, C.J.


CORPORATIONS and PUBLIC
ENTERPRISES, its MEMBERS and PUNO,
CHAIRMAN, the HONORABLE SENATOR
QUISUMBING,
RICHARD GORDON and SENATE YNARES-SANTIAGO,
COMMITTEE ON PUBLIC SERVICES, its
Members and Chairman, the HONORABLE SANDOVAL-GUTIERREZ,
SENATOR JOKER P. ARROYO,
CARPIO,
Respondents.
AUSTRIA-MARTINEZ,

CORONA,

CARPIO MORALES,

CALLEJO, SR.,

AZCUNA,

TINGA,

NAZARIO,

GARCIA, and

VELASCO,JJ.

674
Promulgated:

October 17, 2006

x------------------------------------------------------------------------------------------------------------
-x

DECISION

SANDOVAL-GUTIERREZ, J.:

Two decades ago, on February 28, 1986, former President Corazon C.

Aquino installed her regime by issuing Executive Order (E.O.) No. 1,[1] creating the Presidential
Commission on Good Government (PCGG). She entrusted upon this Commission
the herculean task of recovering the ill-gotten wealth accumulated by the deposed President
Ferdinand E. Marcos, his family, relatives, subordinates and close associates.[2] Section 4 (b) of
E.O. No. 1 provides that: “No member or staff of the Commission shall be required to testify or
produce evidence in any judicial, legislative or administrative proceeding concerning matters
within its official cognizance.” Apparently, the purpose is to ensure PCGG’s unhampered
performance of its task.[3]

Today, the constitutionality of Section 4(b) is being questioned on the ground that it
tramples upon the Senate’s power to conduct legislative inquiry under Article VI, Section 21 of
the 1987 Constitution, which reads:

The Senate or the House of Representatives or any of its respective


committees may conduct inquiries in aid of legislation in accordance with its duly
published rules of procedure. The rights of persons appearing in or affected by
such inquiries shall be respected.

The facts are undisputed.

675
On February 20, 2006, Senator Miriam Defensor Santiago introduced Philippine Senate
Resolution No. 455 (Senate Res. No. 455),[4] “directing an inquiry in aid of legislation on the
anomalous losses incurred by the Philippines Overseas Telecommunications Corporation
(POTC), Philippine Communications Satellite Corporation (PHILCOMSAT), and PHILCOMSAT
Holdings Corporation (PHC) due to the alleged improprieties in their operations by their
respective Board of Directors.”

The pertinent portions of the Resolution read:

WHEREAS, in the last quarter of 2005, the representation and entertainment


expense of the PHC skyrocketed to P4.3 million, as compared to the previous year’s
mere P106 thousand;

WHEREAS, some board members established wholly owned PHC subsidiary


called Telecommunications Center, Inc. (TCI), where PHC funds are allegedly
siphoned; in 18 months, over P73 million had been allegedly advanced to TCI
without any accountability report given to PHC and PHILCOMSAT;

WHEREAS, the Philippine Star, in its 12 February 2002 issue reported that
the executive committee of Philcomsat has precipitately released P265 million and
granted P125 million loan to a relative of an executive committee member; to date
there have been no payments given, subjecting the company to an estimated
interest income loss of P11.25 million in 2004;

WHEREAS, there is an urgent need to protect the interest of the Republic of


the Philippines in the PHC, PHILCOMSAT, and POTC from any anomalous
transaction, and to conserve or salvage any remaining value of the government’s
equity position in these corporations from any abuses of power done by their
respective board of directors;

WHEREFORE, be it resolved that the proper Senate Committee shall conduct


an inquiry in aid of legislation, on the anomalous losses incurred by the Philippine
Overseas Telecommunications Corporation (POTC), Philippine Communications
Satellite Corporation (PHILCOMSAT), and Philcomsat Holdings Corporations (PHC)
due to the alleged improprieties in the operations by their respective board of
directors.

676
Adopted.

(Sgd) MIRIAM DEFENSOR SANTIAGO

On the same date, February 20, 2006, Senate Res. No. 455 was submitted to the Senate
and referred to the Committee on Accountability of Public Officers and
Investigations and Committee on Public Services. However, on March 28, 2006, upon motion of
Senator Francis N. Pangilinan, it was transferred to the Committee on Government Corporations
and Public Enterprises.[5]

On May 8, 2006, Chief of Staff Rio C. Inocencio, under the authority of Senator Richard J.
Gordon, wrote ChairmanCamilo L. Sabio of the PCGG, one of the herein petitioners, inviting him
to be one of the resource persons in the public meeting jointly conducted by the Committee on
Government Corporations and Public Enterprises and Committee on Public Services. The purpose
of the public meeting was to deliberate on Senate Res. No. 455.[6]

On May 9, 2006, Chairman Sabio declined the invitation because of prior


[7]
commitment. At the same time, he invoked Section 4(b) of E.O. No. 1 earlier quoted.

On August 10, 2006, Senator Gordon issued a Subpoena Ad Testificandum,[8] approved by


Senate President ManuelVillar, requiring Chairman Sabio and PCGG
Commissioners Ricardo Abcede, Nicasio Conti, Tereso Javier and NarcisoNario to appear in the
public hearing scheduled on August 23, 2006 and testify on what they know relative to the
matters specified in Senate Res. No. 455. Similar subpoenae were issued against the directors
and officers of Philcomsat Holdings Corporation, namely: Benito V. Araneta, Philip J. Brodett,
Enrique L. Locsin, Manuel D. Andal, Roberto L. Abad, Luis K.Lokin, Jr., Julio J. Jalandoni,
Roberto V. San Jose, Delfin P. Angcao, Alma Kristina Alloba and Johnny Tan.[9]

Again, Chairman Sabio refused to appear. In his letter to Senator Gordon dated August
18, 2006, he reiterated his earlier position, invoking Section 4(b) of E.O. No. 1. On the other
hand, the directors and officers of Philcomsat Holdings Corporation relied on the position paper
they previously filed, which raised issues on the propriety of legislative inquiry.

Thereafter, Chief of Staff Ma. Carissa O. Coscolluela, under the authority of Senator
Gordon, sent another notice[10] to Chairman Sabio requiring him to appear and testify on the

677
same subject matter set on September 6, 2006. The notice was issued “under the same
authority of the Subpoena Ad Testificandum previously served upon (him) last 16 August 2006.”

Once more, Chairman Sabio did not comply with the notice. He sent a
[11]
letter dated September 4, 2006 to Senator Gordon reiterating his reason for declining to
appear in the public hearing.

This prompted Senator Gordon to issue an Order dated September 7, 2006 requiring
Chairman Sabio and CommissionersAbcede, Conti, Javier and Nario to show cause why they
should not be cited in contempt of the Senate. On September 11, 2006, they submitted to the
Senate their Compliance and Explanation,[12] which partly reads:

Doubtless, there are laudable intentions of the subject inquiry in aid of


legislation. But the rule of law requires that even the best intentions must be
carried out within the parameters of the Constitution and the law. Verily, laudable
purposes must be carried out by legal methods. (Brillantes, Jr., et al. v.
Commission on Elections, En Banc [G.R. No. 163193, June 15, 2004])

On this score, Section 4(b) of E.O. No. 1 should not be ignored as it explicitly
provides:

No member or staff of the Commission shall be required to


testify or produce evidence in any judicial legislative or administrative
proceeding concerning matters within its official cognizance.

With all due respect, Section 4(b) of E.O. No. 1 constitutes a limitation on the
power of legislative inquiry, and a recognition by the State of the need to provide
protection to the PCGG in order to ensure the unhampered performance of its
duties under its charter. E.O. No. 1 is a law, Section 4(b) of which had not been
amended, repealed or revised in any way.

To say the least, it would require both Houses of Congress and Presidential
fiat to amend or repeal the provision in controversy. Until then, it stands to be
respected as part of the legal system in this jurisdiction. (As held in People
v. Veneracion, G.R. Nos. 119987-88,October 12, 1995: Obedience to the rule of

678
law forms the bedrock of our system of justice. If judges, under the guise of
religious or political beliefs were allowed to roam unrestricted beyond boundaries
within which they are required by law to exercise the duties of their office, then law
becomes meaningless. A government of laws, not of men excludes the exercise of
broad discretionary powers by those acting under its authority. Under this system,
judges are guided by the Rule of Law, and ought to ‘protect and enforce it without
fear or favor,’ 4 [Act of Athens (1955)] resist encroachments by governments,
political parties, or even the interference of their own personal beliefs.)

x x x x x x

Relevantly, Chairman Sabio’s letter to Sen. Gordon dated August 19, 2006
pointed out that the anomalous transactions referred to in the P.S. Resolution No.
455 are subject of pending cases before the regular courts, the Sandiganbayan and
the Supreme Court (Pending cases include: a. Samuel Divina v. Manuel
Nieto, Jr., et al., CA-G.R. No. 89102; b. Philippine Communications Satellite
Corporation v. Manuel Nieto, et al.; c. Philippine Communications Satellite
Corporation v. Manuel D. Andal, Civil Case No. 06-095, RTC, Branch 61, Makati
City; d. Philippine Communications Satellite Corporation v. PHILCOMSAT Holdings
Corporation, et al., Civil Case No. 04-1049) for which reason they may not be able
to testify thereon under the principle of sub judice. The laudable objectives of
thePCGG’s functions, recognized in several cases decided by the Supreme Court, of
the PCGG will be put to naught if its recovery efforts will be unduly impeded by a
legislative investigation of cases that are already pending before
the Sandiganbayan and trial courts.

In Bengzon v. Senate Blue Ribbon Committee, (203 SCRA 767, 784 [1991])
the Honorable Supreme Court held:

“…[T]he issues sought to be investigated by the respondent


Committee is one over which jurisdiction had been acquired by
the Sandiganbayan. In short, the issue has been pre-empted by that
court. To allow the respondent Committee to conduct its own
investigation of an issue already before the Sandigabayan would not
only pose the possibility of conflicting judgments between a legislative
committee and a judicial tribunal, but if the Committee’s judgment
were to be reached before that of theSandiganbayan, the possibility of

679
its influence being made to bear on the ultimate judgment of
the Sandiganbayan can not be discounted.

x x x x x x

IT IS IN VIEW OF THE FOREGOING CONSIDERATIONS that the Commission


decided not to attend the Senate inquiry to testify and produce evidence thereat.

Unconvinced with the above Compliance and Explanation, the Committee on Government
Corporations and Public Enterprises and the Committee on Public Services issued an
Order[13] directing Major General Jose Balajadia (Ret.), Senate Sergeant-At-Arms, to place
Chairman Sabio and his Commissioners under arrest for contempt of the Senate. The Order
bears the approval of Senate President Villar and the majority of the Committees’ members.

On September 12, 2006, at around 10:45 a.m., Major General Balajadia arrested
Chairman Sabio in his office at IRCBuilding, No. 82 EDSA, Mandaluyong City and brought him to
the Senate premises where he was detained.

Hence, Chairman Sabio filed with this Court a petition for habeas corpus against the
Senate Committee on Government Corporations and Public Enterprises and Committee on Public
Services, their Chairmen, Senators Richard Gordon and Joker P. Arroyo and Members. The case
was docketed as G.R. No. 174340.

Chairman Sabio, Commissioners Abcede, Conti, Nario, and Javier, and


the PCGG’s nominees to Philcomsat Holdings Corporation, Manuel Andal and Julio Jalandoni,
likewise filed a petition for certiorari and prohibition against the same respondents, and also
against Senate President Manuel Villar, Senator Juan Ponce Enrile, the Sergeant-at-Arms, and
the entire Senate. The case was docketed as G.R. No. 174318.

Meanwhile, Philcomsat Holdings Corporation and its officers and directors, namely: Philip
G. Brodett, Luis K. Lokin, Jr., Roberto V. San Jose, Delfin P. Angcao, Roberto L. Abad, Alma
Kristina Alobba and Johnny Tan filed a petition for certiorari and prohibition against the
Senate Committees on Government Corporations and Public Enterprises and Public
Services, their Chairmen, Senators Gordon and Arroyo, and Members. The case was docketed
as G.R. No. 174177.

680
In G.R. No. 174340 (for habeas corpus) and G.R. No. 174318 (for certiorari and
prohibition) Chairman Sabio, Commissioners Abcede, Conti, Nario, and Javier; and
the PCGG’s nominees Andal and Jalandoni alleged: first, respondent Senate Committees
disregarded Section 4(b) of E.O. No. 1 without any justifiable reason; second, the inquiries
conducted by respondent Senate Committees are not in aid of legislation; third, the inquiries
were conducted in the absence of duly publishedSenate Rules of Procedure Governing Inquiries
in Aid of Legislation; and fourth, respondent Senate Committees are not vested with the power
of contempt.

In G.R. No. 174177, petitioners Philcomsat Holdings Corporation and its directors and
officers alleged: first, respondent Senate Committees have no jurisdiction over the subject
matter stated in Senate Res. No. 455; second, the same inquiry is not in accordance with the
Senate’s Rules of Procedure Governing Inquiries in Aid of
Legislation; third, the subpoenae against the individual petitioners are void for having been
issued without authority; fourth, the conduct of legislative inquiry pursuant to Senate Res. No.
455 constitutes undue encroachment by respondents into justiciable controversies over which
several courts and tribunals have already acquired jurisdiction; and fifth, the subpoenae violated
petitioners’ rights to privacy and against self-incrimination.

In their Consolidated Comment, the above-named respondents countered: first, the


issues raised in the petitions involve political questions over which this Court has no
jurisdiction; second, Section 4(b) has been repealed by the Constitution; third,respondent
Senate Committees are vested with contempt power; fourth, Senate’s Rules of Procedure
Governing Inquiries in Aid of Legislation have been duly published; fifth, respondents have not
violated any civil right of the individual petitioners, such as their (a) right to privacy;
and (b) right against self-incrimination; and sixth, the inquiry does not constitute undue
encroachment into justiciable controversies.

During the oral arguments held on September 21, 2006, the parties were directed to
submit simultaneously their respective memoranda within a non-extendible period of fifteen (15)
days from date. In the meantime, per agreement of the parties, petitioner Chairman Sabio was
allowed to go home. Thus, his petition for habeas corpus has become moot. The parties also
agreed that the service of the arrest warrants issued against all petitioners and the proceedings
before the respondent Senate Committees are suspended during the pendency of the instant
cases.[14]

Crucial to the resolution of the present petitions is the fundamental issue of


whether Section 4(b) of E.O. No. 1 is repealed by the 1987 Constitution. On this lone issue
hinges the merit of the contention of Chairman Sabio and his Commissioners that their refusal to
appear before respondent Senate Committees is justified. With the resolution of this issue, all
the other issues raised by the parties have become inconsequential.

681
Perched on one arm of the scale of justice is Article VI, Section 21 of the 1987
Constitution granting respondent Senate Committees the power of legislative inquiry. It reads:

The Senate or the House of Representatives or any of its respective


committees may conduct inquiries in aid of legislation in accordance with its duly
published rules of procedure. The rights of persons appearing in or affected by such
inquiries shall be respected.

On the other arm of the scale is Section 4(b) of E.O. No.1 limiting such power of
legislative inquiry by exempting all PCGG members or staff from testifying in any judicial,
legislative or administrative proceeding, thus:

No member or staff of the Commission shall be required to testify or produce


evidence in any judicial, legislative or administrative proceeding concerning matters
within its official cognizance.

To determine whether there exists a clear and unequivocal repugnancy between the two
quoted provisions that warrants a declaration that Section 4(b) has been repealed by the 1987
Constitution, a brief consideration of the Congress’ power of inquiry is imperative.

The Congress’ power of inquiry has been recognized in foreign jurisdictions long before it
reached our shores throughMcGrain v. Daugherty,[15] cited in Arnault v. Nazareno.[16] In those
earlier days, American courts considered the power of inquiry as inherent in the power to
legislate. The 1864 case of Briggs v. MacKellar[17] explains the breath and basis of the power,
thus:

Where no constitutional limitation or restriction exists, it is competent for


either of the two bodies composing the legislature to do, in their separate capacity,
whatever may be essential to enable them to legislate….It is well-established
principle of this parliamentary law, that either house may institute any
investigation having reference to its own organization, the conduct or qualification
of its members, its proceedings, rights, or privileges or any matter affecting the
public interest upon which it may be important that it should have exact
information, and in respect to which it would be competent for it to legislate. The

682
right to pass laws, necessarily implies the right to obtain information upon any
matter which may become the subject of a law. It is essential to the full and
intelligent exercise of the legislative function….In American legislatures the
investigation of public matters before committees, preliminary to legislation, or with
the view of advising the house appointing the committee is, as a parliamentary
usage, well established as it is in England, and the right of either house to compel
witnesses to appear and testify before its committee, and to punish for
disobedience has been frequently enforced….The right of inquiry, I think, extends to
other matters, in respect to which it may be necessary, or may be deemed
advisable to apply for legislative aid.

Remarkably, in Arnault, this Court adhered to a similar theory. Citing McGrain, it


recognized that the power of inquiry is “an essential and appropriate auxiliary to the legislative
function,” thus:

Although there is no provision in the “Constitution expressly investing either


House of Congress with power to make investigations and exact testimony to the
end that it may exercise its legislative functions advisedly and effectively, such
power is so far incidental to the legislative function as to be implied. In other
words, the power of inquiry – with process to enforce it – is an essential and
appropriate auxiliary to the legislative function. A legislative body cannot legislate
wisely or effectively in the absence of information respecting the conditions which
the legislation is intended to affect or change; and where the legislation body does
not itself possess the requisite information – which is not infrequently true –
recourse must be had to others who possess it.”

Dispelling any doubt as to the Philippine Congress’ power of inquiry, provisions on


such power made their maiden appearance in Article VIII, Section 12 of the 1973
Constitution.[18] Then came the 1987 Constitution incorporating the present Article VI,
Section 12. What was therefore implicit under the 1935 Constitution, as influenced by
American jurisprudence, became explicit under the 1973 and 1987 Constitutions.[19]

Notably, the 1987 Constitution recognizes the power of investigation, not just of
Congress, but also of “any of its committee.” This is significant because it constitutes a direct
conferral of investigatory power upon the committees and it means that the mechanisms which
the Houses can take in order to effectively perform its investigative function are also available to
the committees.[20]

It can be said that the Congress’ power of inquiry has gained more solid existence and
expansive construal. The Court’s high regard to such power is rendered more evident

683
in Senate v. Ermita,[21] where it categorically ruled that “the power of inquiry is broad enough to
cover officials of the executive branch.” Verily, the Court reinforced the doctrine
in Arnault that “the operation of government, being a legitimate subject for legislation, is a
proper subject for investigation” and that “the power of inquiry is co-extensive with the power to
legislate.”

Considering these jurisprudential instructions, we find Section 4(b) directly repugnant with
Article VI, Section 21. Section 4(b) exempts the PCGG members and staff from the Congress’
power of inquiry. This cannot be countenanced. Nowhere in the Constitution is any provision
granting such exemption. The Congress’ power of inquiry, being broad, encompasses
everything that concerns the administration of existing laws as well as proposed or possibly
needed statutes.[22] It even extends “to government agencies created by Congress and officers
whose positions are within the power of Congress to regulate or even abolish.”[23] PCGG belongs
to this class.

Certainly, a mere provision of law cannot pose a limitation to the broad power of
Congress, in the absence of any constitutional basis.

Furthermore, Section 4(b) is also inconsistent with Article XI, Section 1 of the
Constitution stating that: “Public office is a public trust. Public officers and employees must at all
times be accountable to the people, serve them with utmost responsibility, integrity, loyalty, and
efficiency, act with patriotism and justice, and lead modest lives.”

The provision presupposes that since an incumbent of a public office is invested with
certain powers and charged with certain duties pertinent to sovereignty, the powers so
delegated to the officer are held in trust for the people and are to be exercised in behalf of the
government or of all citizens who may need the intervention of the officers. Such trust extends
to all matters within the range of duties pertaining to the office. In other words, public officers
are but the servants of the people, and not their rulers.[24]

Section 4(b), being in the nature of an immunity, is inconsistent with the principle of
public accountability. It places the PCGG members and staff beyond the reach of courts,
Congress and other administrative bodies. Instead of encouraging public accountability, the
same provision only institutionalizes irresponsibility and non-accountability. In Presidential
Commission on Good Government v. Peña,[25] Justice Florentino P. Feliciano characterized as
“obiter” the portion of the majority opinion barring, on the basis of Sections 4(a) and (b) of E.O.
No. 1, a civil case for damages filed against the PCGG and its Commissioners. He eloquently
opined:

684
The above underscored portions are, it is respectfully submitted,
clearly obiter. It is important to make clear that the Court is not here interpreting,
much less upholding as valid and constitutional, the literal terms of Section 4 (a),
(b) of Executive Order No.1. If Section 4 (a) were given its literal import as
immunizing the PCGG or any member thereof from civil liability “for anything done
or omitted in the discharge of the task contemplated by this Order,” the
constitutionality of Section 4 (a) would, in my submission, be open to most serious
doubt. For so viewed, Section 4 (a) would institutionalize the irresponsibility and
non-accountability of members and staff of the PCGG, a notion that is clearly
repugnant to both the 1973 and 1987 Constitution and a privileged status not
claimed by any other official of the Republic under the 1987 Constitution. x x x.

x x x x x x

It would seem constitutionally offensive to suppose that a member or staff member


of the PCGG could not be required to testify before the Sandiganbayan or that such
members were exempted from complying with orders of this Court.

Chavez v. Sandiganbayan[26] reiterates the same view. Indeed, Section 4(b) has been
frowned upon by this Court even before the filing of the present petitions.

Corollarily, Section 4(b) also runs counter to the following constitutional provisions
ensuring the people’s access to information:

Article II, Section 28

Subject to reasonable conditions prescribed by law, the State adopts and


implements a policy of full public disclosure of all its transactions involving public
interest.

Article III, Section 7

The right of the people to information on matters of public concern shall be


recognized. Access to official records, and to documents, and papers pertaining to
685
official acts, transactions, or decisions, as well as to government research data used
as basis for policy development, shall be afforded the citizen, subject to such
limitations as may be provided by law.

These twin provisions of the Constitution seek to promote transparency in policy-making


and in the operations of the government, as well as provide the people sufficient information to
enable them to exercise effectively their constitutional rights. Armed with the right information,
citizens can participate in public discussions leading to the formulation of government policies
and their effective implementation. In Valmonte v. Belmonte, Jr.[27] the Court explained that
an informed citizenry is essential to the existence and proper functioning of any democracy,
thus:

An essential element of these freedoms is to keep open a continuing dialogue


or process of communication between the government and the people. It is in the
interest of the State that the channels for free political discussion be maintained to
the end that the government may perceive and be responsive to the people’s
will. Yet, this open dialogue can be effective only to the extent that the citizenry is
informed and thus able to formulate its will intelligently. Only when the participants
in the discussion are aware of the issues and have access to information relating
thereto can such bear fruit.

Consequently, the conduct of inquiries in aid of legislation is not only intended to benefit
Congress but also the citizenry. The people are equally concerned with this proceeding and have
the right to participate therein in order to protect their interests. The extent of their participation
will largely depend on the information gathered and made known to them. In other words, the
right to information really goes hand-in-hand with the constitutional policies of full public
disclosure and honesty in the public service. It is meant to enhance the widening role of the
citizenry in governmental decision-making as well as in checking abuse in the
government.[28] The cases of Tañada v. Tuvera[29] and Legaspi v. Civil Service
[30]
Commission have recognized a citizen’s interest and personality to enforce a public duty and
to bring an action to compel public officials and employees to perform that duty.

Section 4(b) limits or obstructs the power of Congress to secure from PCGG members and
staff information and other data in aid of its power to legislate. Again, this must not be
countenanced. In Senate v. Ermita,[31] this Court stressed:

686
To the extent that investigations in aid of legislation are generally conducted
in public, however, any executive issuance tending to unduly limit disclosures of
information in such investigations necessarily deprives the people of information
which, being presumed to be in aid of legislation, is presumed to be a matter of
public concern. The citizens are thereby denied access to information which they
can use in formulating their own opinions on the matter before Congress – opinions
which they can then communicate to their representatives and other government
officials through the various legal means allowed by their freedom of expression.

A statute may be declared unconstitutional because it is not within the legislative power to
enact; or it creates or establishes methods or forms that infringe constitutional principles; or its
purpose or effect violates the Constitution or its basic principles.[32] As shown in the above
discussion, Section 4(b) is inconsistent with Article VI, Section 21 (Congress’ power of
inquiry), Article XI, Section 1 (principle of public accountability), Article II, Section 28 (policy of
full disclosure) and Article III, Section 7 (right to public information).

Significantly, Article XVIII, Section 3 of the Constitution provides:

All existing laws, decrees, executive orders, proclamations, letters of


instructions, and other executive issuances not inconsistent with this Constitution
shall remain operative until amended, repealed, or revoked.

The clear import of this provision is that all existing laws, executive orders,
proclamations, letters of instructions and other executive issuances inconsistent or repugnant to
the Constitution are repealed.

Jurisprudence is replete with decisions invalidating laws, decrees, executive orders,


proclamations, letters of instructions and other executive issuances inconsistent with the
Constitution. In Pelaez v. Auditor General,[33] the Court considered repealed Section 68 of the
Revised Administrative Code of 1917 authorizing the Executive to change the seat of the
government of any subdivision of local governments, upon the approval of the 1935
Constitution. Section 68 was adjudged incompatible and inconsistent with the Constitutional
grant of limited executive supervision over local governments. In Islamic Da’wah Council of the
Philippines, Inc., v. Office of the Executive Secretary,[34] the Court declared Executive Order No.
46, entitled “Authorizing the Office on Muslim Affairs to Undertake
Philippine Halal Certification,” void for encroaching on the religious freedom of Muslims. In The
Province of Batangas v. Romulo,[35] the Court declared some provisions of the General
Appropriations Acts of 1999, 2000 and 2001 unconstitutional for violating the Constitutional
precept on local autonomy. And inOple v. Torres,[36] the Court likewise declared unconstitutional

687
Administrative Order No. 308, entitled “Adoption of a National Computerized Identification
Reference System,” for being violative of the right to privacy protected by the Constitution.

These Decisions, and many others, highlight that the Constitution is the highest law of the
land. It is “the basic and paramount law to which all other laws must conform and to which all
persons, including the highest officials of the land, must defer. No act shall be valid, however
noble its intentions, if it conflicts with the Constitution.”[37] Consequently, this Court has no
recourse but to declare Section 4(b) of E.O. No. 1 repealed by the 1987 Constitution.

Significantly, during the oral arguments on September 21, 2006,


Chairman Sabio admitted that should this Court rule that Section 4(b) is unconstitutional or that
it does not apply to the Senate, he will answer the questions of the Senators, thus:

CHIEF JUSTICE PANGANIBAN:

Okay. Now, if the Supreme Court rules that Sec. 4(b) is


unconstitutional or that it does not apply to the Senate, will you answer the
questions of the Senators?

CHAIRMAN SABIO:

Your Honor, my father was a judge, died being a judge. I was here
in the Supreme Court as Chief of Staff of Justice Feria. I would definitely
honor the Supreme Court and the rule of law.

CHIEF JUSTICE PANGANIBAN:

You will answer the questions of the Senators if we say that?

CHAIRMAN SABIO:

688
Yes, Your Honor. That is the law already as far as I am concerned.

With his admission, Chairman Sabio is not fully convinced that he and his Commissioners
are shielded from testifying before respondent Senate Committees by Section 4(b) of E.O. No.
1. In effect, his argument that the said provision exempts him and his co-respondent
Commissioners from testifying before respondent Senate Committees concerning Senate Res.
No. 455 utterly lacks merit.

Incidentally, an argument repeated by Chairman Sabio is that respondent Senate


Committees have no power to punish him and his Commissioners for contempt of the Senate.

The argument is misleading.

Article VI, Section 21 provides:

The Senate or the House of Representatives or any of its respective


committees may conduct inquiries in aid of legislation in accordance with its duly
published rules of procedure. The rights of persons appearing in or affected by such
inquiries shall be respected.

It must be stressed that the Order of Arrest for “contempt of Senate Committees and the
Philippine Senate” wasapproved by Senate President Villar and signed by fifteen (15) Senators.
From this, it can be concluded that the Order is under the authority, not only of the respondent
Senate Committees, but of the entire Senate.

At any rate, Article VI, Section 21 grants the power of inquiry not only to the Senate and
the House of Representatives, but also to any of their respective committees. Clearly, there is
a direct conferral of power to the committees. Father Bernas, in his Commentary on the 1987
Constitution, correctly pointed out its significance:

689
It should also be noted that the Constitution explicitly recognizes the power
of investigation not just of Congress but also of “any of its committees.” This is
significant because it constitutes a direct conferral of investigatory power upon the
committees and it means that the means which the Houses can take in order to
effectively perform its investigative function are also available to the
Committees.[38]

This is a reasonable conclusion. The conferral of the legislative power of inquiry upon any
committee of Congress must carry with it all powers necessary and proper for its effective
discharge. Otherwise, Article VI, Section 21 will be meaningless. The indispensability and
usefulness of the power of contempt in a legislative inquiry is underscored in a catena of cases,
foreign and local.

In the 1821 case of Anderson v. Dunn,[39] the function of the Houses of Congress
with respect to the contempt power was likened to that of a court, thus:

…But the court in its reasoning goes beyond this, and though the grounds of
the decision are not very clearly stated, we take them to be:that there is in some
cases a power in each House of Congress to punish for contempt; that this power is
analogous to that exercised by courts of justice, and that it being the well
established doctrine that when it appears that a prisoner is held under the order of
a court of general jurisdiction for a contempt of its authority, no other court will
discharge the prisoner or make further inquiry into the cause of his
commitment. That this is the general rule…as regards the relation of one court to
another must be conceded.

In McGrain,[40] the U.S. Supreme Court held: “Experience has shown that mere requests
for such information are often unavailing, and also that information which is volunteered is not
always accurate or complete; so some means of compulsion is essential to obtain what is
needed.” The Court, in Arnault v. Nazareno,[41] sustained the Congress’ power of contempt on
the basis of this observation.

In Arnault v. Balagtas,[42] the Court further explained that the contempt power of
Congress is founded upon reason and policy and that the power of inquiry will not be complete if
for every contumacious act, Congress has to resort to judicial interference, thus:

690
The principle that Congress or any of its bodies has the power to punish
recalcitrant witnesses is founded upon reason and policy. Said power must be
considered implied or incidental to the exercise of legislative power. How could a
legislative body obtain the knowledge and information on which to base intended
legislation if it cannot require and compel the disclosure of such knowledge and
information if it is impotent to punish a defiance of its power and authority? When
the framers of the Constitution adopted the principle of separation of powers,
making each branch supreme within the realm of its respective authority, it must
have intended each department’s authority to be full and complete, independently
of the other’s authority or power. And how could the authority and power become
complete if for every act of refusal, every act of defiance, every act of contumacy
against it, the legislative body must resort to the judicial department for the
appropriate remedy, because it is impotent by itself to punish or deal therewith,
with the affronts committed against its authority or dignity.[43]

In Negros Oriental II Electric Cooperative, Inc. v.


Sangguniang Panlungsod of Dumaguete,[44] the Court characterized contempt power as a
matter of self-preservation, thus:

The exercise by the legislature of the contempt power is a matter of self-


preservation as that branch of the government vested with the legislative power,
independently of the judicial branch, asserts its authority and
punishes contempts thereof. The contempt power of the legislature is,
therefore, sui generis x x x.

Meanwhile, with respect to G.R. No. 174177, the petition of Philcomsat Holdings
Corporation and its directors and officers, this Court holds that the respondent Senate
Committees’ inquiry does not violate their right to privacy and right against self-incrimination.

One important limitation on the Congress’ power of inquiry is that “the rights of persons
appearing in or affected by such inquiries shall be respected.” This is just another way of saying
that the power of inquiry must be “subject to the limitations placed by the Constitution on
government action.” As held in Barenblatt v. United States,[45] “the Congress, in common with all
the other branches of the Government, must exercise its powers subject to the limitations placed
by the Constitution on governmental action, more particularly in the context of this case, the
relevant limitations of the Bill of Rights.”

691
First is the right to privacy.

Zones of privacy are recognized and protected in our laws.[46] Within these zones, any
form of intrusion is impermissible unless excused by law and in accordance with customary
legal process. The meticulous regard we accord to these zones arises not only from our
conviction that the right to privacy is a “constitutional right” and “the right most valued by
civilized men,”[47] but also from our adherence to the Universal Declaration of Human Rights
which mandates that, “no one shall be subjected to arbitrary interference with his privacy”
and “everyone has the right to the protection of the law against such interference or
attacks.”[48]

Our Bill of Rights, enshrined in Article III of the Constitution, provides at least two
guarantees that explicitly create zones of privacy. It highlights a person’s “right to be let alone”
or the “right to determine what, how much, to whom and when information about himself shall
be disclosed.”[49] Section 2 guarantees “the right of the people to be secure in their
persons, houses, papers and effects against unreasonable searches and seizures of whatever
nature and for any purpose.” Section 3 renders inviolable the “privacy of communication
and correspondence” and further cautions that “any evidence obtained in violation of this or
the preceding section shall be inadmissible for any purpose in any proceeding.”

In evaluating a claim for violation of the right to privacy, a court must determine whether
a person has exhibited a reasonable expectation of privacy and, if so, whether that expectation
has been violated by unreasonable government intrusion.[50] Applying this determination to
these cases, the important inquiries are: first, did the directors and officers
ofPhilcomsat Holdings Corporation exhibit a reasonable expectation of privacy?; and second, did
the government violate such expectation?

The answers are in the negative. Petitioners were invited in the Senate’s public hearing to
deliberate on Senate Res. No. 455, particularly “on the anomalous losses incurred by the
Philippine Overseas Telecommunications Corporation (POTC), Philippine Communications
Satellite Corporation (PHILCOMSAT), and Philcomsat Holdings Corporations (PHC) due to the
alleged improprieties in the operations by their respective board of directors.” Obviously, the
inquiry focus on petitioners’ acts committed in the discharge of their duties as officers and
directors of the said corporations, particularly Philcomsat Holdings Corporation. Consequently,
they have no reasonable expectation of privacy over matters involving their offices in a
corporation where the government has interest. Certainly, such matters are of public concern
and over which the people have the right to information.

This goes to show that the right to privacy is not absolute where there is an
overriding compelling state interest. InMorfe v. Mutuc,[51] the Court, in line with Whalen v.
Roe,[52] employed the rational basis relationship test when it held that there was no infringement
692
of the individual’s right to privacy as the requirement to disclosure information is for a valid
purpose, i.e., to curtail and minimize the opportunities for official corruption, maintain a
standard of honesty in public service, and promote morality in public
administration.[53] In Valmonte v. Belmonte,[54] the Court remarked that as public figures, the
Members of the former Batasang Pambansa enjoy a more limited right to privacy as compared
to ordinary individuals, and their actions are subject to closer scrutiny. Taking this into
consideration, the Court ruled that the right of the people to access information on matters of
public concern prevails over the right to privacy of financial transactions.

Under the present circumstances, the alleged anomalies in the PHILCOMSAT, PHC and
POTC, ranging in millions of pesos, and the conspiratorial participation of the PCGG and its
officials are compelling reasons for the Senate to exact vital information from the directors and
officers of Philcomsat Holdings Corporations, as well as from Chairman Sabio and his
Commissioners to aid it in crafting the necessary legislation to prevent corruption and formulate
remedial measures and policy determination regarding PCGG’s efficacy. There being no
reasonable expectation of privacy on the part of those directors and officers over the subject
covered by Senate Res. No. 455, it follows that their right to privacy has not been violated by
respondent Senate Committees.

Anent the right against self-incrimination, it must be emphasized that this right maybe
invoked by the said directors and officers of Philcomsat Holdings Corporation only when the
incriminating question is being asked, since they have no way of knowing in advance the nature
or effect of the questions to be asked of them.”[55] That this right may possibly be violated or
abused is no ground for denying respondent Senate Committees their power of inquiry. The
consolation is that when this power is abused, such issue may be presented before the courts. At
this juncture, what is important is that respondent Senate Committees have sufficient Rules to
guide them when the right against self-incrimination is invoked. Sec. 19 reads:

Sec. 19. Privilege Against Self-Incrimination

A witness can invoke his right against self-incrimination only when a question
tends to elicit an answer that will incriminate him is propounded to him. However,
he may offer to answer any question in an executive session.
No person can refuse to testify or be placed under oath or affirmation or
answer questions before an incriminatory question is asked. His invocation of such
right does not by itself excuse him from his duty to give testimony.
In such a case, the Committee, by a majority vote of the members present
there being a quorum, shall determine whether the right has been properly
invoked. If the Committee decides otherwise, it shall resume its investigation and
the question or questions previously refused to be answered shall be repeated to
the witness. If the latter continues to refuse to answer the question, the Committee
may punish him for contempt for contumacious conduct.

693
The same directors and officers contend that the Senate is barred from inquiring into the
same issues being litigated before the Court of Appeals and the Sandiganbayan. Suffice it to
state that the Senate Rules of Procedure Governing Inquiries in Aid of Legislation provide that
the filing or pendency of any prosecution of criminal or administrative action should not stop or
abate any inquiry to carry out a legislative purpose.

Let it be stressed at this point that so long as the constitutional rights of witnesses, like
Chairman Sabio and his Commissioners, will be respected by respondent Senate Committees, it
their duty to cooperate with them in their efforts to obtain the facts needed for intelligent
legislative action. The unremitting obligation of every citizen is to respond to subpoenae, to
respect the dignity of the Congress and its Committees, and to testify fully with respect to
matters within the realm of proper investigation.

In fine, PCGG Chairman Camilo Sabio and Commissioners Ricardo Abcede,


Narciso Nario, Nicasio Conti, and TeresoJavier; and Manuel Andal and
Julio Jalandoni, PCGG’s nominees to Philcomsat Holdings Corporation, as well as its directors and
officers, must comply with the Subpoenae Ad Testificandum issued by respondent Senate
Committees directing them to appear and testify in public hearings relative to Senate Resolution
No. 455.

WHEREFORE, the petition in G.R. No. 174340 for habeas corpus is DISMISSED, for being
moot. The petitions in G.R Nos. 174318 and 174177 are likewise DISMISSED.

Section 4(b) of E.O. No. 1 is declared REPEALED by the 1987 Constitution. Respondent
Senate Committees’ power of inquiry relative to Senate Resolution 455 is upheld. PCGG
Chairman Camilo L. Sabio and Commissioners Ricardo Abcede, Narciso Nario, Nicasio Conti
and Tereso Javier; and Manuel Andal and Julio Jalandoni, PCGG’s nominees
to PhilcomsatHoldings Corporation, as well as its directors and officers, petitioners in G.R. No.
174177, are ordered to comply with theSubpoenae Ad Testificandum issued by respondent
Senate Committees directing them to appear and testify in public hearings relative to Senate
Resolution No. 455.

SO ORDERED.

ANGELINA SANDOVAL-GUTIERREZ

Associate Justice

694
WE CONCUR:

ARTEMIO V. PANGANIBAN

Chief Justice

REYNATO S. PUNO LEONARDO A. QUISUMBING

Associate Justice Associate Justice

CONSUELO YNARES-SANTIAGO ANTONIO T. CARPIO

Associate Justice Associate Justice

MA. ALICIA AUSTRIA-MARTINEZ RENATO C. CORONA

Associate Justice Associate Justice

CONCHITA CARPIO MORALES ROMEO J. CALLEJO, SR.

Associate Justice Associate Justice

695
ADOLFO S. AZCUNA MINITA CHICO-NAZARIO

Associate Justice Associate Justice

DANTE O. TINGA CANCIO C. GARCIA

Associate Justice Associate Justice

PRESBITERO J. VELASCO

Associate Justice

CERTIFICATION

Pursuant to Section 13, Article VIII of the Constitution, it is hereby certified that the
conclusions in the above Decision were reached in consultation before the case was assigned to
the writer of the opinion of the Court.

ARTEMIO V. PANGANIBAN

Chief Justice

696
[1]
E.O. No. 1 was issued by Former President Aquino in the exercise of her legislative
power under the Provisional (Freedom) Constitution. Thus, it is of the same category and
has the same binding force as a statute. (Agpalo, Statutory Construction, 1998
citing Legaspi v. Ministry of Finance, 115 SCRA 418 [1982]; Garcia-Padilla v. Ponce Enrile,
G.R. No. 61388, April 20, 1983; Aquino v. Commission on Elections, 62 SCRA 275
[1975] )

[2]
Section 2 (a), Executive Order No.1.

[3]
See Presidential Commission on Good Government v. Pena, April 12, 1988, 159
SCRA 558
[4]
Annex “E” of the Petition in G.R. No. 174318.
[5]
Id.
[6]
Annex “F” of the Petition in G.R. No. 174318.
[7]
Annex “G” of the Petition in G.R. No. 174318.
[8]
Annex “A” of the Petition in G.R. No. 174318.
[9]
Petition in G.R. No. 174177 at p. 15.

[10]
Annex “B” of the Petition in G.R. No. 174318.
[11]
Annex “I” of the Petition in G.R. No. 174318.
[12]
Annex “J” of the Petition in G.R. No. 174318.
[13]
Annex “D” of the petition in G.R. No. 174318.
[14]
En Banc Resolution dated September 21, 2006.
[15]
273 U.S. 135, 47 S. Ct. 319, 71 L. Ed. 580, 50 A.L.R. 1 (1927).
[16]
No. L- 3820, 87 Phil. 29 (1950).
[17]
2 Abb. Pr. 30 (N.Y. 1864).
[18]
Puno, Lecture on Legislative Investigations and the Right to Privacy, at p. 22.
[19]
Bernas S.J., The 1987 Constitution of the Republic of the Philippines, 2003 Ed. at
p.737.
[20]
Bernas S.J., The 1987 Constitution of the Republic of the Philippines, 2003 Ed. at
p.739.
[21]
G.R. No. 169777, April 20, 2006.
[22]
Watkins v. United States, 354 U.S. 178 (1957), pp. 194-195.
[23]
Senate v. Ermita, Id.
[24]
De Leon, De Leon, Jr. The Law on Public Officers and Election Law, p. 2.
[25]
No. L-77663, April 12, 1988, 159 SCRA 558.

[26]
193 SCRA 282 (1991).
[27]
G.R. No. 74930, February 13, 1989, 170 SCRA 256.

[28]
Valmonte v. Belmonte, Jr., supra.
[29]
136 SCRA 27.
[30]
150 SCRA 530.
[31]
Supra.

697
[32]
Agpalo, Statutory Construction, 1998 citing In re Cunanan, 94 Phil. 534 (1954).
[33]
No. L-23825, December 24, 1965, 15 SCRA 569.
[34]
G.R. No. 153888, July 9, 2003, 405 SCRA 497.
[35]
G.R. No. 152774, May 27, 2004, 429 SCRA 736.
[36]
293 SCRA 141 (1998).
[37]
Cruz, Constitutional Law, 2003, p. 4.
[38]
Bernas, S.J., The 1987 Constitution of the Republic of the Philippines A Commentary,
p. 678.
[39]
19 U.S. [6 Wheat.] 204 (1821) cited in Justice Puno, Legislative Investigations and
Right to Privacy.

[40]
Supra.
[41]
Supra.
[42]
97 Phil. 358 [1955].
[43]
Id.
[44]
No. L-72492, November 5, 1987, 155 SCRA 421.
[45]
360 U.S. 109 (1959).

[46]
Marquez v. Desierto, G.R. No. 135882, June 27, 2001, 359 SCRA 772.
[47]
See Morfe v. Mutuc No. L-20387, January 31, 1968, 22 SCRA 424.
[48]
Article 12 of the Universal Declaration of Human Rights. See also Article 17 (1) and
(2) of the International Covenant on Civil and Political Rights.
[49]
Constitutional and Legal Systems of ASEAN Countries, Sison, Academy of ASEAN
Law and Jurisprudence, 1990, at 221, citing I.R. Cortes, The Constitutional Foundations of
Privacy, 7 (1970).
[50]
Burrows v. Superior Court of San Bernardino County, 13 Cal. 3d 238, 529 P 2d 590
(1974). See Katz v. United states (1967), 389 U.S. 347, 350-352, 88 S. Ct. 507, 19 L.
Ed. 2d 576; People v. Krivda (1971) 5 Cal. 3d 357, 364, 96 Cal. Rptr. 62, 486 P. 2d 1262;
8 Cal. 3d 623-624,105 Cal. Rptr. 521, 504 P. 2d 457. INSERT Herrera’s Handbook on
Arrest, Search and Seizure.
[51]
Supra.
[52]
429 U.S. 589 (1977).
[53]
Justice Puno, Lecture on Legislative Inquiry and Right to Privacy, p. 60.
[54]
170 SCRA 256 (1989)
[55]
Cruz, Constitutional Law, 2003, p. 307.

698
EN BANC

PROF. RANDOLF S. DAVID, LORENZO G.R. No. 171396


TAÑADA III, RONALD LLAMAS, H. HARRY L.
ROQUE, JR., JOEL RUIZ BUTUYAN, ROGER R. Present:
RAYEL, GARY S. MALLARI, ROMEL
REGALADO BAGARES, CHRISTOPHER F.C. PANGANIBAN, C.J.,
*
BOLASTIG, PUNO,
Petitioners, QUISUMBING,
YNARES-SANTIAGO,
- versus - SANDOVAL-GUTIERREZ,
CARPIO,
AUSTRIA-MARTINEZ,
GLORIA MACAPAGAL-ARROYO, AS CORONA,
PRESIDENT AND COMMANDER-IN-CHIEF, CARPIO MORALES,
EXECUTIVE SECRETARY EDUARDO ERMITA, CALLEJO, SR.,
HON. AVELINO CRUZ II, SECRETARY OF AZCUNA,
NATIONAL DEFENSE, GENERAL GENEROSO TINGA,
SENGA, CHIEF OF STAFF, ARMED FORCES OF CHICO-NAZARIO,
THE PHILIPPINES, DIRECTOR GENERAL GARCIA, and
ARTURO LOMIBAO, CHIEF, PHILIPPINE VELASCO, JJ.
NATIONAL POLICE,
Respondents. Promulgated:
x-------------------------------------------------
x May 3, 2006
NIÑEZ CACHO-OLIVARES AND TRIBUNE
PUBLISHING CO., INC.,
Petitioners, G.R. No. 171409

- versus -

HONORABLE SECRETARY EDUARDO ERMITA


AND HONORABLE DIRECTOR GENERAL
ARTURO C. LOMIBAO,
Respondents.
x-------------------------------------------------
x
FRANCIS JOSEPH G. ESCUDERO, JOSEPH A.
SANTIAGO, TEODORO A. CASINO, AGAPITO G.R. No. 171485
A. AQUINO, MARIO J. AGUJA, SATUR C.
OCAMPO, MUJIV S. HATAMAN, JUAN
EDGARDO ANGARA, TEOFISTO DL.
GUINGONA III, EMMANUEL JOSEL J.
VILLANUEVA, LIZA L. MAZA, IMEE R.
MARCOS, RENATO B. MAGTUBO, JUSTIN
MARC SB. CHIPECO, ROILO GOLEZ, DARLENE

699
ANTONINO-CUSTODIO, LORETTA ANN P.
ROSALES, JOSEL G. VIRADOR, RAFAEL V.
MARIANO, GILBERT C. REMULLA, FLORENCIO
G. NOEL, ANA THERESIA HONTIVEROS-
BARAQUEL, IMELDA C. NICOLAS, MARVIC
M.V.F. LEONEN, NERI JAVIER COLMENARES,
MOVEMENT OF CONCERNED CITIZENS FOR
CIVIL LIBERTIES REPRESENTED BY AMADO
GAT INCIONG,
Petitioners,

- versus -

EDUARDO R. ERMITA, EXECUTIVE


SECRETARY, AVELINO J. CRUZ, JR.,
SECRETARY, DND RONALDO V. PUNO,
SECRETARY, DILG, GENEROSO SENGA, AFP
CHIEF OF STAFF, ARTURO LOMIBAO, CHIEF
PNP,
Respondents.
x-------------------------------------------------
x
KILUSANG MAYO UNO, REPRESENTED BY ITS
CHAIRPERSON ELMER C. LABOG AND
SECRETARY GENERAL JOEL MAGLUNSOD,
NATIONAL FEDERATION OF LABOR UNIONS –
KILUSANG MAYO UNO (NAFLU-KMU),
REPRESENTED BY ITS NATIONAL PRESIDENT, G.R. No. 171483
JOSELITO V. USTAREZ, ANTONIO C.
PASCUAL, SALVADOR T. CARRANZA, EMILIA
P. DAPULANG, MARTIN CUSTODIO, JR., AND
ROQUE M. TAN,
Petitioners,

- versus -

HER EXCELLENCY, PRESIDENT GLORIA


MACAPAGAL-ARROYO, THE HONORABLE
EXECUTIVE SECRETARY, EDUARDO ERMITA,
THE CHIEF OF STAFF, ARMED FORCES OF
THE PHILIPPINES, GENEROSO SENGA, AND
THE PNP DIRECTOR GENERAL, ARTURO

700
LOMIBAO,
Respondents.
x-------------------------------------------------
x
ALTERNATIVE LAW GROUPS, INC. (ALG),
Petitioner,
- versus -

EXECUTIVE SECRETARY EDUARDO R.


ERMITA, LT. GEN. GENEROSO SENGA, AND G.R. No. 171400
DIRECTOR GENERAL ARTURO
LOMIBAO,
Respondents.
x-------------------------------------------------
x
JOSE ANSELMO I. CADIZ, FELICIANO M.
BAUTISTA, ROMULO R. RIVERA, JOSE AMOR
M. AMORADO, ALICIA A. RISOS-VIDAL,
FELIMON C. ABELITA III, MANUEL P.
LEGASPI, J.B. JOVY C. BERNABE, BERNARD L.
DAGCUTA, ROGELIO V. GARCIA AND
INTEGRATED BAR OF THE PHILIPPINES (IBP),
Petitioners,

- versus -
G.R. No. 171489

HON. EXECUTIVE SECRETARY EDUARDO


ERMITA, GENERAL GENEROSO SENGA, IN
HIS CAPACITY AS AFP CHIEF OF STAFF, AND
DIRECTOR GENERAL ARTURO LOMIBAO, IN
HIS CAPACITY AS PNP CHIEF,
Respondents.
x-------------------------------------------------
x
LOREN B. LEGARDA,
Petitioner,

- versus -

GLORIA MACAPAGAL-ARROYO, IN HER


CAPACITY AS PRESIDENT AND COMMANDER-
IN-CHIEF; ARTURO LOMIBAO, IN HIS
CAPACITY AS DIRECTOR-GENERAL OF THE
PHILIPPINE NATIONAL POLICE (PNP);
GENEROSO SENGA, IN HIS CAPACITY AS

701
CHIEF OF STAFF OF THE ARMED FORCES OF G.R. No. 171424
THE PHILIPPINES (AFP); AND EDUARDO
ERMITA, IN HIS CAPACITY AS EXECUTIVE
SECRETARY,
Respondents.

x---------------------------------------------------------------------------------------------x

DECISION

SANDOVAL-GUTIERREZ, J.:

All powers need some restraint; practical adjustments rather than rigid formula are
necessary.[1] Superior strength – the use of force – cannot make wrongs into rights. In this
regard, the courts should be vigilant in safeguarding the constitutional rights of the citizens,
specifically their liberty.

Chief Justice Artemio V. Panganiban’s philosophy of liberty is thus most relevant. He


said: “In cases involving liberty, the scales of justice should weigh heavily against government
and in favor of the poor, the oppressed, the marginalized, the dispossessed and the
weak.” Laws and actions that restrict fundamental rights come to the courts “with a heavy
presumption against their constitutional validity.”[2]

These seven (7) consolidated petitions for certiorari and prohibition allege that in issuing
Presidential Proclamation No. 1017 (PP 1017) and General Order No. 5 (G.O. No. 5), President
Gloria Macapagal-Arroyo committed grave abuse of discretion. Petitioners contend that
respondent officials of the Government, in their professed efforts to defend and preserve
democratic institutions, are actually trampling upon the very freedom guaranteed and protected
by the Constitution. Hence, such issuances are void for being unconstitutional.

Once again, the Court is faced with an age-old but persistently modern problem. How
does the Constitution of a free people combine the degree of liberty, without which, law becomes
tyranny, with the degree of law, without which, liberty becomes license?[3]

On February 24, 2006, as the nation celebrated the 20th Anniversary of the Edsa People
Power I, President Arroyo issued PP 1017 declaring a state of national emergency, thus:
702
NOW, THEREFORE, I, Gloria Macapagal-Arroyo, President of the Republic of
the Philippines and Commander-in-Chief of the Armed Forces of the Philippines, by
virtue of the powers vested upon me by Section 18, Article 7 of the Philippine
Constitution which states that: “The President. . . whenever it becomes necessary, .
. . may call out (the) armed forces to prevent or suppress. . .rebellion. . .,” and in
my capacity as their Commander-in-Chief, do hereby command the Armed Forces
of the Philippines, to maintain law and order throughout the Philippines, prevent or
suppress all forms of lawless violence as well as any act of insurrection or rebellion
and to enforce obedience to all the laws and to all decrees, orders and regulations
promulgated by me personally or upon my direction; and as provided in Section 17,
Article 12 of the Constitution do hereby declare a State of National Emergency.

She cited the following facts as bases:

WHEREAS, over these past months, elements in the political opposition have
conspired with authoritarians of the extreme Left represented by the NDF-CPP-NPA
and the extreme Right, represented by military adventurists – the historical
enemies of the democratic Philippine State – who are now in a tactical alliance and
engaged in a concerted and systematic conspiracy, over a broad front, to bring
down the duly constituted Government elected in May 2004;

WHEREAS, these conspirators have repeatedly tried to bring down the


President;

WHEREAS, the claims of these elements have been recklessly magnified by


certain segments of the national media;

WHEREAS, this series of actions is hurting the Philippine State – by


obstructing governance including hindering the growth of the economy and
sabotaging the people’s confidence in government and their faith in the future of
this country;

WHEREAS, these actions are adversely affecting the economy;

WHEREAS, these activities give totalitarian forces of both the extreme Left
and extreme Right the opening to intensify their avowed aims to bring down the
democratic Philippine State;

WHEREAS, Article 2, Section 4 of the our Constitution makes the defense and
preservation of the democratic institutions and the State the primary duty of
Government;

WHEREAS, the activities above-described, their consequences, ramifications


and collateral effects constitute a clear and present danger to the safety and the
integrity of the Philippine State and of the Filipino people;

703
On the same day, the President issued G. O. No. 5 implementing PP 1017, thus:

WHEREAS, over these past months, elements in the political opposition have
conspired with authoritarians of the extreme Left, represented by the NDF-CPP-NPA
and the extreme Right, represented by military adventurists - the historical
enemies of the democratic Philippine State – and who are now in a tactical alliance
and engaged in a concerted and systematic conspiracy, over a broad front, to bring
down the duly-constituted Government elected in May 2004;
WHEREAS, these conspirators have repeatedly tried to bring down our
republican government;

WHEREAS, the claims of these elements have been recklessly magnified by


certain segments of the national media;

WHEREAS, these series of actions is hurting the Philippine State by


obstructing governance, including hindering the growth of the economy and
sabotaging the people’s confidence in the government and their faith in the future
of this country;

WHEREAS, these actions are adversely affecting the economy;

WHEREAS, these activities give totalitarian forces; of both the extreme Left
and extreme Right the opening to intensify their avowed aims to bring down the
democratic Philippine State;

WHEREAS, Article 2, Section 4 of our Constitution makes the defense and


preservation of the democratic institutions and the State the primary duty of
Government;

WHEREAS, the activities above-described, their consequences, ramifications


and collateral effects constitute a clear and present danger to the safety and the
integrity of the Philippine State and of the Filipino people;

WHEREAS, Proclamation 1017 date February 24, 2006 has been issued
declaring a State of National Emergency;

NOW, THEREFORE, I GLORIA MACAPAGAL-ARROYO, by virtue of the powers


vested in me under the Constitution as President of the Republic of the Philippines,
and Commander-in-Chief of the Republic of the Philippines, and pursuant to
Proclamation No. 1017 dated February 24, 2006, do hereby call upon the Armed
Forces of the Philippines (AFP) and the Philippine National Police (PNP), to prevent
and suppress acts of terrorism and lawless violence in the country;

I hereby direct the Chief of Staff of the AFP and the Chief of the PNP, as
well as the officers and men of the AFP and PNP, to immediately carry out the
necessary and appropriate actions and measures to suppress and prevent acts of
terrorism and lawless violence.

704
On March 3, 2006, exactly one week after the declaration of a state of national emergency
and after all these petitions had been filed, the President lifted PP 1017. She issued
Proclamation No. 1021 which reads:

WHEREAS, pursuant to Section 18, Article VII and Section 17, Article XII of
the Constitution, Proclamation No. 1017 dated February 24, 2006, was issued
declaring a state of national emergency;

WHEREAS, by virtue of General Order No.5 and No.6 dated February 24,
2006, which were issued on the basis of Proclamation No. 1017, the Armed Forces
of the Philippines (AFP) and the Philippine National Police (PNP), were directed to
maintain law and order throughout the Philippines, prevent and suppress all form
of lawless violence as well as any act of rebellion and to undertake such action as
may be necessary;

WHEREAS, the AFP and PNP have effectively prevented, suppressed and
quelled the acts lawless violence and rebellion;

NOW, THEREFORE, I, GLORIA MACAPAGAL-ARROYO, President of the


Republic of the Philippines, by virtue of the powers vested in me by law,
hereby declare that the state of national emergency has ceased to exist.

In their presentation of the factual bases of PP 1017 and G.O. No. 5, respondents stated
that the proximate cause behind the executive issuances was the conspiracy among some
military officers, leftist insurgents of the New People’s Army (NPA), and some members of the
political opposition in a plot to unseat or assassinate President Arroyo.[4] They considered the
aim to oust or assassinate the President and take-over the reigns of government as a clear and
present danger.

During the oral arguments held on March 7, 2006, the Solicitor General specified the facts
leading to the issuance of PP 1017 and G.O. No. 5. Significantly, there was no
refutation from petitioners’ counsels.

The Solicitor General argued that the intent of the Constitution is to give full discretionary
powers to the President in determining the necessity of calling out the armed forces. He
emphasized that none of the petitioners has shown that PP 1017 was without factual
bases. While he explained that it is not respondents’ task to state the facts behind the
questioned Proclamation, however, they are presenting the same, narrated hereunder, for the
elucidation of the issues.

705
On January 17, 2006, Captain Nathaniel Rabonza and First Lieutenants
Sonny Sarmiento, Lawrence San Juan and Patricio Bumidang, members of the Magdalo Group
indicted in the Oakwood mutiny, escaped their detention cell in Fort Bonifacio, Taguig City. In a
public statement, they vowed to remain defiant and to elude arrest at all costs. They called
upon the people to “show and proclaim our displeasure at the sham regime. Let us demonstrate
our disgust, not only by going to the streets in protest, but also by wearing red bands on our left
arms.” [5]

On February 17, 2006, the authorities got hold of a document entitled “Oplan Hackle I ”
which detailed plans for bombings and attacks during the Philippine Military Academy Alumni
Homecoming in Baguio City. The plot was to assassinate selected targets including some
cabinet members and President Arroyo herself.[6] Upon the advice of her security, President
Arroyo decided not to attend the Alumni Homecoming. The next day, at the height of the
celebration, a bomb was found and detonated at the PMA parade ground.

On February 21, 2006, Lt. San Juan was recaptured in a communist safehouse in
Batangas province. Found in his possession were two (2) flash disks containing minutes of the
meetings between members of the Magdalo Group and the National People’s Army (NPA), a tape
recorder, audio cassette cartridges, diskettes, and copies of subversive documents.[7] Prior to
his arrest, Lt. San Juan announced through DZRH that the “Magdalo’s D-Day would be on
February 24, 2006, the 20th Anniversary of Edsa I.”

On February 23, 2006, PNP Chief Arturo Lomibao intercepted information that members of
the PNP- Special Action Force were planning to defect. Thus, he immediately ordered SAF
Commanding General Marcelino Franco, Jr. to “disavow”any defection. The latter promptly
obeyed and issued a public statement: “All SAF units are under the effective control of
responsible and trustworthy officers with proven integrity and unquestionable loyalty.”

On the same day, at the house of former Congressman Peping Cojuangco, President Cory
Aquino’s brother, businessmen and mid-level government officials plotted moves to bring down
the Arroyo administration. Nelly Sindayen of TIME Magazine reported that Pastor Saycon,
longtime Arroyo critic, called a U.S. government official about his group’s plans if President
Arroyo is ousted. Saycon also phoned a man code-named Delta. Saycon identified him as
B/Gen. Danilo Lim, Commander of the Army’s elite Scout Ranger. Lim said “it was all systems
go for the planned movement against Arroyo.”[8]

706
B/Gen. Danilo Lim and Brigade Commander Col. Ariel Querubin confided to Gen. Generoso
Senga, Chief of Staff of the Armed Forces of the Philippines (AFP), that a huge number of
soldiers would join the rallies to provide a critical mass and armed component to the Anti-Arroyo
protests to be held on February 24, 2005. According to these two (2) officers, there was no
way they could possibly stop the soldiers because they too, were breaking the chain of command
to join the forces foist to unseat the President. However, Gen. Senga has remained faithful to
his Commander-in-Chief and to the chain of command. He immediately took custody of B/Gen.
Lim and directed Col. Querubin to return to the Philippine Marines Headquarters in Fort
Bonifacio.

Earlier, the CPP-NPA called for intensification of political and revolutionary work within the
military and the police establishments in order to forge alliances with its members and key
officials. NPA spokesman Gregorio “Ka Roger” Rosal declared: “The Communist Party and
revolutionary movement and the entire people look forward to the possibility in the coming year
of accomplishing its immediate task of bringing down the Arroyo regime; of rendering it to
weaken and unable to rule that it will not take much longer to end it.”[9]

On the other hand, Cesar Renerio, spokesman for the National Democratic Front (NDF) at
North Central Mindanao, publicly announced: “Anti-Arroyo groups within the military and police
are growing rapidly, hastened by the economic difficulties suffered by the families of AFP officers
and enlisted personnel who undertake counter-insurgency operations in the field.” He claimed
that with the forces of the national democratic movement, the anti-Arroyo conservative political
parties, coalitions, plus the groups that have been reinforcing since June 2005, it is probable
that the President’s ouster is nearing its concluding stage in the first half of 2006.

Respondents further claimed that the bombing of telecommunication towers and cell sites
in Bulacan and Bataan was also considered as additional factual basis for the issuance of PP
1017 and G.O. No. 5. So is the raid of an army outpost in Benguet resulting in the death of
three (3) soldiers. And also the directive of the Communist Party of the Philippines ordering its
front organizations to join 5,000 Metro Manila radicals and 25,000 more from the provinces in
mass protests.[10]

By midnight of February 23, 2006, the President convened her security advisers and
several cabinet members to assess the gravity of the fermenting peace and order
situation. She directed both the AFP and the PNP to account for all their men and ensure that
the chain of command remains solid and undivided. To protect the young students from any

707
possible trouble that might break loose on the streets, the President suspended classes in all
levels in the entire National Capital Region.

For their part, petitioners cited the events that followed after the issuance of PP 1017 and
G.O. No. 5.

Immediately, the Office of the President announced the cancellation of all programs and
activities related to the 20thanniversary celebration of Edsa People Power I; and revoked the
permits to hold rallies issued earlier by the local governments. Justice Secretary Raul Gonzales
stated that political rallies, which to the President’s mind were organized for purposes of
destabilization, are cancelled. Presidential Chief of Staff Michael Defensor announced that
“warrantless arrests and take-over of facilities, including media, can already be
[11]
implemented.”

Undeterred by the announcements that rallies and public assemblies would not be
allowed, groups of protesters (members of Kilusang Mayo Uno [KMU] and National Federation of
Labor Unions-Kilusang Mayo Uno [NAFLU-KMU]), marched from various parts of Metro Manila
with the intention of converging at the EDSA shrine. Those who were already near the EDSA
site were violently dispersed by huge clusters of anti-riot police. The well-trained policemen
used truncheons, big fiber glass shields, water cannons, and tear gas to stop and break up the
marching groups, and scatter the massed participants. The same police action was used against
the protesters marching forward to Cubao, Quezon City and to the corner of Santolan Street and
EDSA. That same evening, hundreds of riot policemen broke up an EDSA celebration rally held
along Ayala Avenue and Paseo de Roxas Street in Makati City.[12]

According to petitioner Kilusang Mayo Uno, the police cited PP 1017 as the ground for the
dispersal of their assemblies.

During the dispersal of the rallyists along EDSA, police arrested (without warrant)
petitioner Randolf S. David, a professor at the University of the Philippines and newspaper
columnist. Also arrested was his companion, Ronald Llamas, president of party-list Akbayan.

At around 12:20 in the early morning of February 25, 2006, operatives of the Criminal
Investigation and Detection Group (CIDG) of the PNP, on the basis of PP 1017 and G.O. No. 5,
raided the Daily Tribune offices in Manila. The raiding team confiscated news stories by
reporters, documents, pictures, and mock-ups of the Saturday issue. Policemen from Camp

708
Crame in Quezon City were stationed inside the editorial and business offices of the newspaper;
while policemen from the Manila Police District were stationed outside the building.[13]

A few minutes after the search and seizure at the Daily Tribune offices, the police
surrounded the premises of another pro-opposition paper, Malaya, and its sister publication, the
tabloid Abante.

The raid, according to Presidential Chief of Staff Michael Defensor, is “meant to show a
‘strong presence,’ to tell media outlets not to connive or do anything that would help the rebels
in bringing down this government.” The PNP warned that it would take over any media
organization that would not follow “standards set by the government during the state of national
emergency.” Director General Lomibao stated that “if they do not follow the standards – and
the standards are - if they would contribute to instability in the government, or if they do not
subscribe to what is in General Order No. 5 and Proc. No. 1017 – we will recommend a
‘takeover.’” National Telecommunications’ Commissioner Ronald Solis urged television and radio
networks to “cooperate” with the government for the duration of the state of national
emergency. He asked for“balanced reporting” from broadcasters when covering the events
surrounding the coup attempt foiled by the government. He warned that his agency will not
hesitate to recommend the closure of any broadcast outfit that violates rules set out for media
coverage when the national security is threatened.[14]

Also, on February 25, 2006, the police arrested Congressman Crispin Beltran,
representing the Anakpawis Party and Chairman of Kilusang Mayo Uno (KMU), while leaving his
farmhouse in Bulacan. The police showed a warrant for his arrest dated 1985. Beltran’s lawyer
explained that the warrant, which stemmed from a case of inciting to rebellion filed during the
Marcos regime, had long been quashed. Beltran, however, is not a party in any of these
petitions.

When members of petitioner KMU went to Camp Crame to visit Beltran, they were told
they could not be admitted because of PP 1017 and G.O. No. 5. Two members were arrested
and detained, while the rest were dispersed by the police.

Bayan Muna Representative Satur Ocampo eluded arrest when the police went after him
during a public forum at the Sulo Hotel in Quezon City. But his two drivers, identified as Roel
and Art, were taken into custody.

709
Retired Major General Ramon Montaño, former head of the Philippine Constabulary, was
arrested while with his wife and golfmates at the Orchard Golf and Country Club in Dasmariñas,
Cavite.

Attempts were made to arrest Anakpawis Representative Satur Ocampo, Representative


Rafael Mariano, Bayan MunaRepresentative Teodoro Casiño and Gabriela Representative Liza
Maza. Bayan Muna Representative Josel Virador was arrested at the PAL Ticket Office in Davao
City. Later, he was turned over to the custody of the House of Representatives where the
“Batasan 5” decided to stay indefinitely.

Let it be stressed at this point that the alleged violations of the rights of Representatives
Beltran, Satur Ocampo, et al., are not being raised in these petitions.

On March 3, 2006, President Arroyo issued PP 1021 declaring that the state of national
emergency has ceased to exist.

In the interim, these seven (7) petitions challenging the constitutionality of PP 1017 and
G.O. No. 5 were filed with this Court against the above-named respondents. Three (3) of these
petitions impleaded President Arroyo as respondent.

In G.R. No. 171396, petitioners Randolf S. David, et al. assailed PP 1017 on the grounds
that (1) it encroaches on the emergency powers of Congress; (2) it is a subterfuge to avoid the
constitutional requirements for the imposition of martial law; and (3) it violates the
constitutional guarantees of freedom of the press, of speech and of assembly.

In G.R. No. 171409, petitioners Ninez Cacho-Olivares and Tribune Publishing Co.,
Inc. challenged the CIDG’s act of raiding the Daily Tribune offices as a clear case of “censorship”
or “prior restraint.” They also claimed that the term “emergency” refers only to tsunami,
typhoon, hurricane and similar occurrences, hence, there is “absolutely no emergency” that
warrants the issuance of PP 1017.

In G.R. No. 171485, petitioners herein are Representative Francis Joseph G. Escudero,
and twenty one (21) other members of the House of Representatives, including Representatives
Satur Ocampo, Rafael Mariano, Teodoro Casiño, Liza Maza, and Josel Virador. They asserted
that PP 1017 and G.O. No. 5 constitute “usurpation of legislative powers”; “violation of freedom
of expression” and “a declaration of martial law.” They alleged that President Arroyo “gravely

710
abused her discretion in calling out the armed forces without clear and verifiable factual basis of
the possibility of lawless violence and a showing that there is necessity to do so.”

In G.R. No. 171483, petitioners KMU, NAFLU-KMU, and their members averred that PP
1017 and G.O. No. 5 are unconstitutional because (1) they arrogate unto President Arroyo the
power to enact laws and decrees; (2) their issuance was without factual basis; and (3) they
violate freedom of expression and the right of the people to peaceably assemble to redress their
grievances.

In G.R. No. 171400, petitioner Alternative Law Groups, Inc. (ALGI) alleged that PP 1017
and G.O. No. 5 are unconstitutional because they violate (a) Section 4[15] of Article
[16] [17] [18] [19]
II, (b) Sections 1, 2, and 4 of Article III, (c)Section 23 of Article
[20]
VI, and (d) Section 17 of Article XII of the Constitution.

In G.R. No. 171489, petitioners Jose Anselmo I. Cadiz et al., alleged that PP 1017 is an
“arbitrary and unlawful exercise by the President of her Martial Law powers.” And assuming
that PP 1017 is not really a declaration of Martial Law, petitioners argued that “it amounts to an
exercise by the President of emergency powers without congressional approval.” In addition,
petitioners asserted that PP 1017 “goes beyond the nature and function of a proclamation as
defined under the Revised Administrative Code.”

And lastly, in G.R. No. 171424, petitioner Loren B. Legarda maintained that PP 1017 and
G.O. No. 5 are “unconstitutional for being violative of the freedom of expression, including its
cognate rights such as freedom of the press and the right to access to information on matters of
public concern, all guaranteed under Article III, Section 4 of the 1987 Constitution.” In this
regard, she stated that these issuances prevented her from fully prosecuting her election protest
pending before the Presidential Electoral Tribunal.

In respondents’ Consolidated Comment, the Solicitor General countered that: first, the
petitions should be dismissed for being moot; second, petitioners in G.R. Nos. 171400
(ALGI), 171424 (Legarda), 171483 (KMU et al.), 171485 (Escudero et al.) and 171489 (Cadiz et
al.) have no legal standing; third, it is not necessary for petitioners to implead President Arroyo
as respondent; fourth, PP 1017 has constitutional and legal basis; and fifth, PP 1017 does not
violate the people’s right to free expression and redress of grievances.

711
On March 7, 2006, the Court conducted oral arguments and heard the parties on the
above interlocking issues which may be summarized as follows:

A. PROCEDURAL:
1) Whether the issuance of PP 1021 renders the petitions moot and
academic.
2) Whether petitioners in 171485 (Escudero et al.), G.R. Nos.
171400 (ALGI), 171483 (KMU et al.),171489 (Cadiz et al.), and 171424 (Legarda)
have legal standing.
B. SUBSTANTIVE:
1) Whether the Supreme Court can review the factual bases of PP 1017.
2) Whether PP 1017 and G.O. No. 5 are unconstitutional.
a. Facial Challenge
b. Constitutional Basis
c. As Applied Challenge

A. PROCEDURAL

First, we must resolve the procedural roadblocks.

I- Moot and Academic Principle

One of the greatest contributions of the American system to this country is the concept of
judicial review enunciated inMarbury v. Madison.[21] This concept rests on the extraordinary
simple foundation --

The Constitution is the supreme law. It was ordained by the people, the
ultimate source of all political authority. It confers limited powers on the national
government. x x x If the government consciously or unconsciously oversteps these
limitations there must be some authority competent to hold it in control, to thwart
its unconstitutional attempt, and thus to vindicate and preserve inviolate the will of
the people as expressed in the Constitution. This power the courts exercise. This is
the beginning and the end of the theory of judicial review.[22]

But the power of judicial review does not repose upon the courts a “self-starting
capacity.”[23] Courts may exercise such power only when the following requisites are
present: first, there must be an actual case or controversy; second, petitioners have to raise a
question of constitutionality; third, the constitutional question must be raised at the earliest
opportunity; and fourth, the decision of the constitutional question must be necessary to the
determination of the case itself.[24]
712
Respondents maintain that the first and second requisites are absent, hence, we shall
limit our discussion thereon.

An actual case or controversy involves a conflict of legal right, an opposite legal claims
susceptible of judicial resolution. It is “definite and concrete, touching the legal relations of
parties having adverse legal interest;” a real and substantial controversy admitting of specific
[25]
relief. The Solicitor General refutes the existence of such actual case or controversy,
contending that the present petitions were rendered “moot and academic” by President Arroyo’s
issuance of PP 1021.

Such contention lacks merit.

A moot and academic case is one that ceases to present a justiciable controversy by
virtue of supervening events,[26] so that a declaration thereon would be of no practical use or
value.[27] Generally, courts decline jurisdiction over such case[28] or dismiss it on ground of
mootness.[29]

The Court holds that President Arroyo’s issuance of PP 1021 did not render the present
petitions moot and academic. During the eight (8) days that PP 1017 was operative, the police
officers, according to petitioners, committed illegal acts in implementing it. Are PP 1017 and
G.O. No. 5 constitutional or valid? Do they justify these alleged illegal acts? These are the vital
issues that must be resolved in the present petitions. It must be stressed that “an
unconstitutional act is not a law, it confers no rights, it imposes no duties, it affords no
protection; it is in legal contemplation, inoperative.”[30]

The “moot and academic” principle is not a magical formula that can automatically
dissuade the courts in resolving a case. Courts will decide cases, otherwise moot and academic,
if: first, there is a grave violation of the Constitution;[31] second, the exceptional character of
the situation and the paramount public interest is involved;[32] third, when constitutional issue
raised requires formulation of controlling principles to guide the bench, the bar, and the
public;[33] and fourth, the case is capable of repetition yet evading review.[34]

All the foregoing exceptions are present here and justify this Court’s assumption of
jurisdiction over the instant petitions. Petitioners alleged that the issuance of PP 1017 and G.O.
No. 5 violates the Constitution. There is no question that the issues being raised affect the
public’s interest, involving as they do the people’s basic rights to freedom of expression, of

713
assembly and of the press. Moreover, the Court has the duty to formulate guiding and
controlling constitutional precepts, doctrines or rules. It has the symbolic function of educating
the bench and the bar, and in the present petitions, the military and the police, on the extent of
the protection given by constitutional guarantees.[35] And lastly, respondents’ contested actions
are capable of repetition. Certainly, the petitions are subject to judicial review.

In their attempt to prove the alleged mootness of this case, respondents cited Chief
Justice Artemio V. Panganiban’s Separate Opinion in Sanlakas v. Executive
[36]
Secretary. However, they failed to take into account the Chief Justice’s very statement that
an otherwise “moot” case may still be decided “provided the party raising it in a proper case has
been and/or continues to be prejudiced or damaged as a direct result of its issuance.” The
present case falls right within this exception to the mootness rule pointed out by the Chief
Justice.

II- Legal Standing

In view of the number of petitioners suing in various personalities, the Court deems it
imperative to have a more than passing discussion on legal standing or locus standi.

Locus standi is defined as “a right of appearance in a court of justice on a given


question.”[37] In private suits, standing is governed by the “real-parties-in interest” rule as
contained in Section 2, Rule 3 of the 1997 Rules of Civil Procedure, as amended. It provides that
“every action must be prosecuted or defended in the name of the real party in
interest.” Accordingly, the “real-party-in interest” is “the party who stands to be benefited or
injured by the judgment in the suit or the party entitled to the avails of the suit.”[38] Succinctly
put, the plaintiff’s standing is based on his own right to the relief sought.

The difficulty of determining locus standi arises in public suits. Here, the plaintiff who
asserts a “public right” in assailing an allegedly illegal official action, does so as a representative
of the general public. He may be a person who is affected no differently from any other
person. He could be suing as a “stranger,” or in the category of a “citizen,” or ‘taxpayer.” In
either case, he has to adequately show that he is entitled to seek judicial protection. In other
words, he has to make out a sufficient interest in the vindication of the public order and the
securing of relief as a “citizen” or “taxpayer.

714
Case law in most jurisdictions now allows both “citizen” and “taxpayer” standing in public
actions. The distinction was first laid down in Beauchamp v. Silk,[39] where it was held that the
plaintiff in a taxpayer’s suit is in a different category from the plaintiff in a citizen’s suit. In the
former, the plaintiff is affected by the expenditure of public funds, while in the latter, he is but
the mere instrument of the public concern. As held by the New York Supreme Court in People
[40]
ex rel Case v. Collins: “In matter of mere public right, however…the people are the real
parties…It is at least the right, if not the duty, of every citizen to interfere and see that a public
offence be properly pursued and punished, and that a public grievance be remedied.” With
[41]
respect to taxpayer’s suits, Terr v. Jordan held that “the right of a citizen and a taxpayer to
maintain an action in courts to restrain the unlawful use of public funds to his injury cannot be
denied.”

However, to prevent just about any person from seeking judicial interference in any
official policy or act with which he disagreed with, and thus hinders the activities of
governmental agencies engaged in public service, the United State Supreme Court laid down the
more stringent “direct injury” test in Ex Parte Levitt,[42] later reaffirmed in Tileston v.
[43]
Ullman. The same Court ruled that for a private individual to invoke the judicial power to
determine the validity of an executive or legislative action, he must show that he has sustained
a direct injury as a result of that action, and it is not sufficient that he has a general interest
common to all members of the public.

This Court adopted the “direct injury” test in our jurisdiction. In People v. Vera,[44] it
held that the person who impugns the validity of a statute must have “a personal and substantial
interest in the case such that he has sustained, or will sustain direct injury as a
result.” The Vera doctrine was upheld in a litany of cases, such as, Custodio v. President of the
Senate,[45] Manila Race Horse Trainers’ Association v. De la Fuente,[46] Pascual v. Secretary of
Public Works[47] and Anti-Chinese League of the Philippines v. Felix.[48]

However, being a mere procedural technicality, the requirement of locus standi may be
waived by the Court in the exercise of its discretion. This was done in the 1949 Emergency
Powers Cases, Araneta v. Dinglasan,[49] where the “transcendental importance” of the cases
prompted the Court to act liberally. Such liberality was neither a rarity nor
[50]
accidental. In Aquino v. Comelec, this Court resolved to pass upon the issues raised due to
the “far-reaching implications” of the petition notwithstanding its categorical statement that
petitioner therein had no personality to file the suit. Indeed, there is a chain of cases where this
liberal policy has been observed, allowing ordinary citizens, members of Congress, and civic

715
organizations to prosecute actions involving the constitutionality or validity of laws, regulations
and rulings.[51]

Thus, the Court has adopted a rule that even where the petitioners have failed to show
direct injury, they have been allowed to sue under the principle of “transcendental importance.”
Pertinent are the following cases:
(1) Chavez v. Public Estates Authority,[52] where the Court ruled that the
enforcement of the constitutional right to information and the equitable diffusion of
natural resources are matters of transcendental importance which clothe the
petitioner with locus standi;

(2) Bagong Alyansang Makabayan v. Zamora,[53] wherein the Court held


that “given the transcendental importance of the issues involved, the Court may
relax the standing requirements and allow the suit to prosper despite the lack of
direct injury to the parties seeking judicial review” of the Visiting Forces
Agreement;

(3) Lim v. Executive Secretary,[54] while the Court noted that the petitioners
may not file suit in their capacity as taxpayers absent a showing that “Balikatan 02-
01” involves the exercise of Congress’ taxing or spending powers,
it reiterated its ruling in Bagong Alyansang Makabayan v.
Zamora,[55] that in cases of transcendental importance, the cases must be settled
promptly and definitely and standing requirements may be relaxed.

By way of summary, the following rules may be culled from the cases decided by this
Court. Taxpayers, voters, concerned citizens, and legislators may be accorded standing to sue,
provided that the following requirements are met:

(1) the cases involve constitutional issues;

(2) for taxpayers, there must be a claim of illegal disbursement of public


funds or that the tax measure is unconstitutional;

(3) for voters, there must be a showing of obvious interest in the validity of
the election law in question;

(4) for concerned citizens, there must be a showing that the issues raised are
of transcendental importance which must be settled early; and

716
(5) for legislators, there must be a claim that the official action complained
of infringes upon their prerogatives as legislators.

Significantly, recent decisions show a certain toughening in the Court’s attitude toward
legal standing.

In Kilosbayan, Inc. v. Morato,[56] the Court ruled that the status of Kilosbayan as a
people’s organization does not give it the requisite personality to question the validity of the on-
line lottery contract, more so where it does not raise any issue of constitutionality. Moreover, it
cannot sue as a taxpayer absent any allegation that public funds are being misused. Nor can it
sue as a concerned citizen as it does not allege any specific injury it has suffered.

In Telecommunications and Broadcast Attorneys of the Philippines, Inc. v.


[57]
Comelec, the Court reiterated the “direct injury” test with respect to concerned citizens’
cases involving constitutional issues. It held that “there must be a showing that the citizen
personally suffered some actual or threatened injury arising from the alleged illegal official act.”

In Lacson v. Perez,[58] the Court ruled that one of the petitioners, Laban ng
Demokratikong Pilipino (LDP), is not a real party-in-interest as it had not demonstrated any
injury to itself or to its leaders, members or supporters.

In Sanlakas v. Executive Secretary,[59] the Court ruled that only the petitioners who are
members of Congress have standing to sue, as they claim that the President’s declaration of a
state of rebellion is a usurpation of the emergency powers of Congress, thus impairing their
legislative powers. As to petitioners Sanlakas, Partido Manggagawa, and Social Justice Society,
the Court declared them to be devoid of standing, equating them with the LDP in Lacson.

Now, the application of the above principles to the present petitions.

The locus standi of petitioners in G.R. No. 171396, particularly David and Llamas, is
beyond doubt. The same holds true with petitioners in G.R. No. 171409, Cacho-Olivares
and Tribune Publishing Co. Inc. They alleged “direct injury” resulting from “illegal arrest” and
“unlawful search” committed by police operatives pursuant to PP 1017. Rightly so, the Solicitor
General does not question their legal standing.

717
In G.R. No. 171485, the opposition Congressmen alleged there was usurpation of
legislative powers. They also raised the issue of whether or not the concurrence of Congress is
necessary whenever the alarming powers incident to Martial Law are used. Moreover, it is in
the interest of justice that those affected by PP 1017 can be represented by their Congressmen
in bringing to the attention of the Court the alleged violations of their basic rights.

In G.R. No. 171400, (ALGI), this Court applied the liberality rule in Philconsa v.
Enriquez,[60] Kapatiran Ng Mga Naglilingkod sa Pamahalaan ng Pilipinas, Inc. v.
[61]
Tan, Association of Small Landowners in the Philippines, Inc. v. Secretary of Agrarian
Reform,[62] Basco v. Philippine Amusement and Gaming Corporation,[63] and Tañada v.
Tuvera,[64] that when the issue concerns a public right, it is sufficient that the petitioner is a
citizen and has an interest in the execution of the laws.

In G.R. No. 171483, KMU’s assertion that PP 1017 and G.O. No. 5 violated its right to
peaceful assembly may be deemed sufficient to give it legal standing. Organizations may be
granted standing to assert the rights of their members.[65] We take judicial notice of the
announcement by the Office of the President banning all rallies and canceling all permits for
public assemblies following the issuance of PP 1017 and G.O. No. 5.

In G.R. No. 171489, petitioners, Cadiz et al., who are national officers of the Integrated
Bar of the Philippines (IBP) have no legal standing, having failed to allege any direct or potential
injury which the IBP as an institution or its members may suffer as a consequence of the
issuance of PP No. 1017 and G.O. No. 5. In Integrated Bar of the Philippines v. Zamora,[66] the
Court held that the mere invocation by the IBP of its duty to preserve the rule of law and
nothing more, while undoubtedly true, is not sufficient to clothe it with standing in this
case. This is too general an interest which is shared by other groups and the whole
citizenry. However, in view of the transcendental importance of the issue, this Court declares
that petitioner have locus standi.

In G.R. No. 171424, Loren Legarda has no personality as a taxpayer to file the instant
petition as there are no allegations of illegal disbursement of public funds. The fact that she is
a former Senator is of no consequence. She can no longer sue as a legislator on the allegation
that her prerogatives as a lawmaker have been impaired by PP 1017 and G.O. No. 5. Her
claim that she is a media personality will not likewise aid her because there was no showing that
the enforcement of these issuances prevented her from pursuing her occupation. Her
submission that she has pending electoral protest before the Presidential Electoral Tribunal is

718
likewise of no relevance. She has not sufficiently shown that PP 1017 will affect the proceedings
or result of her case. But considering once more the transcendental importance of the issue
involved, this Court may relax the standing rules.

It must always be borne in mind that the question of locus standi is but corollary to the
bigger question of proper exercise of judicial power. This is the underlying legal tenet of the
“liberality doctrine” on legal standing. It cannot be doubted that the validity of PP No. 1017 and
G.O. No. 5 is a judicial question which is of paramount importance to the Filipino people. To
paraphrase Justice Laurel, the whole of Philippine society now waits with bated breath the ruling
of this Court on this very critical matter. The petitions thus call for the application of the
“transcendental importance” doctrine, a relaxation of the standing requirements for the
petitioners in the “PP 1017 cases.”

This Court holds that all the petitioners herein have locus standi.

Incidentally, it is not proper to implead President Arroyo as respondent. Settled is the


doctrine that the President, during his tenure of office or actual incumbency,[67] may not be sued
in any civil or criminal case, and there is no need to provide for it in the Constitution or law. It
will degrade the dignity of the high office of the President, the Head of State, if he can be
dragged into court litigations while serving as such. Furthermore, it is important that he be
freed from any form of harassment, hindrance or distraction to enable him to fully attend to the
performance of his official duties and functions. Unlike the legislative and judicial branch, only
one constitutes the executive branch and anything which impairs his usefulness in the discharge
of the many great and important duties imposed upon him by the Constitution necessarily
impairs the operation of the Government. However, this does not mean that the President is not
accountable to anyone. Like any other official, he remains accountable to the people[68] but he
may be removed from office only in the mode provided by law and that is by impeachment.[69]

B. SUBSTANTIVE

I. Review of Factual Bases

Petitioners maintain that PP 1017 has no factual basis. Hence, it was not “necessary” for
President Arroyo to issue such Proclamation.

719
The issue of whether the Court may review the factual bases of the President’s exercise of
his Commander-in-Chief power has reached its distilled point - from the indulgent days
of Barcelon v. Baker[70] and Montenegro v. Castaneda[71] to the volatile era
of Lansang v. Garcia,[72] Aquino, Jr. v. Enrile,[73] and Garcia-Padilla v. Enrile.[74] The
tug-of-war always cuts across the line defining “political questions,” particularly those questions
“in regard to which full discretionary authority has been delegated to the legislative or executive
branch of the government.”[75] Barcelon and Montenegro were in unison in declaring that
the authority to decide whether an exigency has arisen belongs to the President and his decision
is final and conclusive on the courts. Lansang took the opposite view. There, the members of
the Court were unanimous in the conviction that the Court has the authority to inquire into the
existence of factual bases in order to determine their constitutional sufficiency. From the
principle of separation of powers, it shifted the focus to the system of checks and balances,
“under which the President is supreme, x x x only if and when he acts within the sphere allotted
to him by the Basic Law, and the authority to determine whether or not he has so acted
is vested in the Judicial Department, which in this respect, is,
in turn, constitutionally supreme.”[76] In 1973, the unanimous Court
[77]
of Lansang was divided in Aquino v. Enrile. There, the Court was almost evenly
divided on the issue of whether the validity of the imposition of Martial Law is a
political or justiciable question.[78] Then came Garcia-Padilla v. Enrile which greatly
diluted Lansang. It declared that there is a need to re-examine the latter case, ratiocinating that
“in times of war or national emergency, the President must be given absolute control for the
very life of the nation and the government is in great peril. The President, it intoned, is
answerable only to his conscience, the People, and God.”[79]

The Integrated Bar of the Philippines v. Zamora[80] -- a recent case most pertinent to
these cases at bar -- echoed a principle similar to Lansang. While the Court considered the
President’s “calling-out” power as a discretionary power solely vested in his wisdom, it stressed
that “this does not prevent an examination of whether such power was exercised within
permissible constitutional limits or whether it was exercised in a manner constituting grave
abuse of discretion.” This ruling is mainly a result of the Court’s reliance on Section 1, Article
VIII of 1987 Constitution which fortifies the authority of the courts to determine in an
appropriate action the validity of the acts of the political departments. Under the new definition
of judicial power, the courts are authorized not only “to settle actual controversies involving
rights which are legally demandable and enforceable,” but also “to determine whether or not
there has been a grave abuse of discretion amounting to lack or excess of jurisdiction on the
part of any branch or instrumentality of the government.” The latter part of the authority

720
represents a broadening of judicial power to enable the courts of justice to review what was
before a forbidden territory, to wit, the discretion of the political departments of the
government.[81] It speaks of judicial prerogative not only in terms of powerbut also of duty.[82]

As to how the Court may inquire into the President’s exercise of power, Lansang adopted
the test that “judicial inquiry can go no further than to satisfy the Court not that the President’s
decision is correct,” but that “the President did not actarbitrarily.” Thus, the standard laid down
is not correctness, but arbitrariness.[83] In Integrated Bar of the Philippines, this Court further
ruled that “it is incumbent upon the petitioner to show that the President’s decision is totally
bereft of factual basis” and that if he fails, by way of proof, to support his assertion, then “this
Court cannot undertake an independent investigation beyond the pleadings.”

Petitioners failed to show that President Arroyo’s exercise of the calling-out power, by
issuing PP 1017, is totally bereft of factual basis. A reading of the Solicitor General’s
Consolidated Comment and Memorandum shows a detailed narration of the events leading to
the issuance of PP 1017, with supporting reports forming part of the records. Mentioned are the
escape of the Magdalo Group, their audacious threat of the Magdalo D-Day, the defections in the
military, particularly in the Philippine Marines, and the reproving statements from the communist
leaders. There was also the Minutes of the Intelligence Report and Security Group of the
Philippine Army showing the growing alliance between the NPA and the military. Petitioners
presented nothing to refute such events. Thus, absent any contrary allegations, the Court is
convinced that the President was justified in issuing PP 1017 calling for military aid.

Indeed, judging the seriousness of the incidents, President Arroyo was not expected to
simply fold her arms and do nothing to prevent or suppress what she believed was lawless
violence, invasion or rebellion. However, the exercise of such power or duty must not stifle
liberty.

II. Constitutionality of PP 1017 and G.O. No. 5

Doctrines of Several Political Theorists


on the Power of the President
in Times of Emergency

721
This case brings to fore a contentious subject -- the power of the President in times of
emergency. A glimpse at the various political theories relating to this subject provides an
adequate backdrop for our ensuing discussion.

John Locke, describing the architecture of civil government, called upon the English
doctrine of prerogative to cope with the problem of emergency. In times of danger to the
nation, positive law enacted by the legislature might be inadequate or even a fatal obstacle to
the promptness of action necessary to avert catastrophe. In these situations, the Crown
retained a prerogative “power to act according to discretion for the public good, without the
proscription of the law and sometimes even against it.”[84] But Locke recognized that this moral
restraint might not suffice to avoid abuse of prerogative powers. Who shall judge the need for
resorting to the prerogative and how may its abuse be avoided? Here, Locke readily admitted
defeat, suggesting that “the people have no other remedy in this, as in all other cases where
they have no judge on earth, but to appeal to Heaven.”[85]

Jean-Jacques Rousseau also assumed the need for temporary suspension of democratic
processes of government in time of emergency. According to him:

The inflexibility of the laws, which prevents them from adopting themselves
to circumstances, may, in certain cases, render them disastrous and make them
bring about, at a time of crisis, the ruin of the State…

It is wrong therefore to wish to make political institutions as strong as to


render it impossible to suspend their operation. Even Sparta allowed its law to
lapse...

If the peril is of such a kind that the paraphernalia of the laws are an
obstacle to their preservation, the method is to nominate a supreme lawyer, who
shall silence all the laws and suspend for a moment the sovereign authority. In
such a case, there is no doubt about the general will, and it clear that the people’s
first intention is that the State shall not perish.[86]

Rosseau did not fear the abuse of the emergency dictatorship or “supreme magistracy” as
he termed it. For him, it would more likely be cheapened by “indiscreet use.” He was unwilling
to rely upon an “appeal to heaven.” Instead, he relied upon a tenure of office of prescribed
duration to avoid perpetuation of the dictatorship.[87]
722
John Stuart Mill concluded his ardent defense of representative government: “I am far
from condemning, in cases of extreme necessity, the assumption of absolute power in the form
of a temporary dictatorship.”[88]

Nicollo Machiavelli’s view of emergency powers, as one element in the whole scheme of
limited government, furnished an ironic contrast to the Lockean theory of prerogative. He
recognized and attempted to bridge this chasm in democratic political theory, thus:

Now, in a well-ordered society, it should never be necessary to resort to


extra –constitutional measures; for although they may for a time be beneficial, yet
the precedent is pernicious, for if the practice is once established for good objects,
they will in a little while be disregarded under that pretext but for evil purposes.
Thus, no republic will ever be perfect if she has not by law provided for everything,
having a remedy for every emergency and fixed rules for applying it.[89]

Machiavelli – in contrast to Locke, Rosseau and Mill – sought to incorporate into the
constitution a regularized system of standby emergency powers to be invoked with suitable
checks and controls in time of national danger. He attempted forthrightly to meet the problem
of combining a capacious reserve of power and speed and vigor in its application in time of
emergency, with effective constitutional restraints.[90]

Contemporary political theorists, addressing themselves to the problem of response to


emergency by constitutional democracies, have employed the doctrine of constitutional
dictatorship.[91] Frederick M. Watkins saw “no reason why absolutism should not be used as a
means for the defense of liberal institutions,” provided it “serves to protect established
institutions from the danger of permanent injury in a period of temporary emergency and is
followed by a prompt return to the previous forms of political life.”[92] He recognized the two (2)
key elements of the problem of emergency governance, as well as all constitutional
governance: increasing administrative powers of the executive, while at the same
[93]
time “imposing limitation upon that power.” Watkins placed his real faith in a scheme of
constitutional dictatorship. These are the conditions of success of such a dictatorship: “The
period of dictatorship must be relatively short…Dictatorship should always be strictly legitimate
in character…Final authority to determine the need for dictatorship in any given case must never

723
rest with the dictator himself…”[94] and the objective of such an emergency dictatorship should
be “strict political conservatism.”

Carl J. Friedrich cast his analysis in terms similar to those of Watkins.[95] “It is a problem
of concentrating power – in a government where power has consciously been divided – to cope
with… situations of unprecedented magnitude and gravity. There must be a broad grant of
powers, subject to equally strong limitations as to who shall exercise such powers, when, for
how long, and to what end.”[96] Friedrich, too, offered criteria for judging the adequacy of any of
scheme of emergency powers, to wit: “The emergency executive must be appointed by
constitutional means – i.e., he must be legitimate; he should not enjoy power to determine the
existence of an emergency; emergency powers should be exercised under a strict time
limitation; and last, the objective of emergency action must be the defense of the constitutional
order.”[97]

Clinton L. Rossiter, after surveying the history of the employment of emergency powers in
Great Britain, France, Weimar, Germany and the United States, reverted to a description of a
scheme of “constitutional dictatorship” as solution to the vexing problems presented by
emergency.[98] Like Watkins and Friedrich, he stated a priori the conditions of success
of the “constitutional dictatorship,” thus:

1) No general regime or particular institution of constitutional


dictatorship should be initiated unless it is necessary or even indispensable to
the preservation of the State and its constitutional order…

2) …the decision to institute a constitutional dictatorship should never


be in the hands of the man or men who will constitute the dictator…

3) No government should initiate a constitutional dictatorship without


making specific provisions for its termination…

4) …all uses of emergency powers and all readjustments in the


organization of the government should be effected in pursuit of
constitutional or legal requirements…

5) … no dictatorial institution should be adopted, no right invaded, no


regular procedure altered any more than is absolutely necessary for the
conquest of the particular crisis . . .

6) The measures adopted in the prosecution of the a constitutional


dictatorship should never be permanent in character or effect…

724
7) The dictatorship should be carried on by persons representative of
every part of the citizenry interested in the defense of the existing
constitutional order. . .

8) Ultimate responsibility should be maintained for every action taken


under a constitutional dictatorship. . .

9) The decision to terminate a constitutional dictatorship, like the


decision to institute one should never be in the hands of the man or men who
constitute the dictator. . .

10) No constitutional dictatorship should extend beyond the


termination of the crisis for which it was instituted…

11) …the termination of the crisis must be followed by a complete


return as possible to the political and governmental conditions existing prior
to the initiation of the constitutional dictatorship…[99]

Rossiter accorded to legislature a far greater role in the oversight exercise of emergency powers
than did Watkins. He would secure to Congress final responsibility for declaring the existence
or termination of an emergency, and he places great faith in the effectiveness of congressional
investigating committees.[100]
Scott and Cotter, in analyzing the above contemporary theories in light of recent
experience, were one in saying that, “the suggestion that democracies surrender the control of
government to an authoritarian ruler in time of grave danger to the nation is not based upon
sound constitutional theory.” To appraise emergency power in terms of constitutional
dictatorship serves merely to distort the problem and hinder realistic analysis. It matters not
whether the term “dictator” is used in its normal sense (as applied to authoritarian rulers) or is
employed to embrace all chief executives administering emergency powers. However used,
“constitutional dictatorship” cannot be divorced from the implication of suspension of the
processes of constitutionalism. Thus, they favored instead the “concept of constitutionalism”
articulated by Charles H. McIlwain:

A concept of constitutionalism which is less misleading in the analysis of


problems of emergency powers, and which is consistent with the findings of this
study, is that formulated by Charles H. McIlwain. While it does not by any means
necessarily exclude some indeterminate limitations upon the substantive powers of
government, full emphasis is placed upon procedural limitations, and political
responsibility. McIlwain clearly recognized the need to repose adequate power in
government. And in discussing the meaning of constitutionalism, he insisted that
the historical and proper test of constitutionalism was the existence of adequate
processes for keeping government responsible. He refused to equate
725
constitutionalism with the enfeebling of government by an exaggerated emphasis
upon separation of powers and substantive limitations on governmental power. He
found that the really effective checks on despotism have consisted not in the
weakening of government but, but rather in the limiting of it; between which there
is a great and very significant difference. In associating constitutionalism with
“limited” as distinguished from “weak” government, McIlwain meant government
limited to the orderly procedure of law as opposed to the processes of force. The
two fundamental correlative elements of constitutionalism for which all lovers of
liberty must yet fight are the legal limits to arbitrary power and a complete political
responsibility of government to the governed.[101]

In the final analysis, the various approaches to emergency of the above political theorists
–- from Lock’s “theory of prerogative,” to Watkins’ doctrine of “constitutional dictatorship” and,
eventually, to McIlwain’s “principle of constitutionalism” --- ultimately aim to solve one real
problem in emergency governance, i.e., that of allotting increasing areas of discretionary power
to the Chief Executive, while insuring that such powers will be exercised with a sense of political
responsibility and under effective limitations and checks.

Our Constitution has fairly coped with this problem. Fresh from the fetters of a repressive
regime, the 1986 Constitutional Commission, in drafting the 1987 Constitution, endeavored to
create a government in the concept of Justice Jackson’s “balanced power
[102]
structure.” Executive, legislative, and judicial powers are dispersed to the President, the
Congress, and the Supreme Court, respectively. Each is supreme within its own sphere. But
none has the monopoly of power in times of emergency. Each branch is given a role to serve as
limitation or check upon the other. This system does not weaken the
President, it just limits his power, using the language of McIlwain. In other words, in times
of emergency, our Constitution reasonably demands that we repose a certain amount of faith in
the basic integrity and wisdom of the Chief Executive but, at the same time, it obliges him to
operate within carefully prescribed procedural limitations.

a. “Facial Challenge”

Petitioners contend that PP 1017 is void on its face because of its “overbreadth.” They
claim that its enforcement encroached on both unprotected and protected rights under Section
4, Article III of the Constitution and sent a “chilling effect” to the citizens.

A facial review of PP 1017, using the overbreadth doctrine, is uncalled for.

726
First and foremost, the overbreadth doctrine is an analytical tool developed for testing
“on their faces” statutes in free speech cases, also known under the American Law as First
Amendment cases.[103]

A plain reading of PP 1017 shows that it is not primarily directed to speech or even
speech-related conduct. It is actually a call upon the AFP to prevent or suppress all forms
of lawless violence. In United States v. Salerno,[104] the US Supreme Court held that “we have
not recognized an ‘overbreadth’ doctrine outside the limited context of the First Amendment”
(freedom of speech).

Moreover, the overbreadth doctrine is not intended for testing the validity of a law that
“reflects legitimate state interest in maintaining comprehensive control over harmful,
constitutionally unprotected conduct.” Undoubtedly, lawless violence, insurrection and rebellion
are considered “harmful” and “constitutionally unprotected conduct.” In Broadrick v.
[105]
Oklahoma, it was held:

It remains a ‘matter of no little difficulty’ to determine when a law may


properly be held void on its face and when ‘such summary action’ is
inappropriate. But the plain import of our cases is, at the very least, that facial
overbreadth adjudication is an exception to our traditional rules of practice and that
its function, a limited one at the outset, attenuates as the otherwise unprotected
behavior that it forbids the State to sanction moves from ‘pure speech’ toward
conduct and that conduct –even if expressive – falls within the scope of otherwise
valid criminal laws that reflect legitimate state interests in maintaining
comprehensive controls over harmful, constitutionally unprotected conduct.

Thus, claims of facial overbreadth are entertained in cases involving statutes which, by
their terms, seek to regulate only “spoken words” and again, that “overbreadth claims, if
entertained at all, have been curtailed when invoked against ordinary criminal laws that are
sought to be applied to protected conduct.”[106] Here, the incontrovertible fact remains that PP
1017 pertains to a spectrum of conduct, not free speech, which is manifestly subject to state
regulation.

Second, facial invalidation of laws is considered as “manifestly strong medicine,” to be


used “sparingly and only as a last resort,” and is “generally disfavored;”[107] The reason for this
is obvious. Embedded in the traditional rules governing constitutional adjudication is the
principle that a person to whom a law may be applied will not be heard to challenge a law on the
727
ground that it may conceivably be applied unconstitutionally to others, i.e., in other situations
not before the Court.[108] A writer and scholar in Constitutional Law explains further:

The most distinctive feature of the overbreadth technique is that it marks an


exception to some of the usual rules of constitutional litigation. Ordinarily, a
particular litigant claims that a statute is unconstitutional as applied to him or her;
if the litigant prevails, the courts carve away the unconstitutional aspects of the law
by invalidating its improper applications on a case to case basis. Moreover,
challengers to a law are not permitted to raise the rights of third parties and can
only assert their own interests. In overbreadth analysis, those rules give way;
challenges are permitted to raise the rights of third parties; and the court
invalidates the entire statute “on its face,” not merely “as applied for” so that the
overbroad law becomes unenforceable until a properly authorized court construes it
more narrowly. The factor that motivates courts to depart from the normal
adjudicatory rules is the concern with the “chilling;” deterrent effect of the
overbroad statute on third parties not courageous enough to bring suit. The Court
assumes that an overbroad law’s “very existence may cause others not before the
court to refrain from constitutionally protected speech or expression.” An
overbreadth ruling is designed to remove that deterrent effect on the speech of
those third parties.

In other words, a facial challenge using the overbreadth doctrine will require the Court to
examine PP 1017 and pinpoint its flaws and defects, not on the basis of its actual operation to
petitioners, but on the assumption or prediction that its very existence may cause others not
before the Court to refrain from constitutionally protected speech or expression. In Younger v.
Harris,[109] it was held that:

[T]he task of analyzing a proposed statute, pinpointing its deficiencies, and


requiring correction of these deficiencies before the statute is put into effect, is
rarely if ever an appropriate task for the judiciary. The combination of the relative
remoteness of the controversy, theimpact on the legislative process of the relief
sought, and above all the speculative and amorphous nature of the required line-
by-line analysis of detailed statutes,...ordinarily results in a kind of case that
is wholly unsatisfactory for deciding constitutional questions, whichever way they
might be decided.

And third, a facial challenge on the ground of overbreadth is the most difficult challenge to
mount successfully, since the challenger must establish that there can be no instance when the
assailed law may be valid. Here, petitioners did not even attempt to show whether this situation
exists.

728
Petitioners likewise seek a facial review of PP 1017 on the ground of vagueness. This,
too, is unwarranted.

Related to the “overbreadth” doctrine is the “void for vagueness doctrine” which holds
that “a law is facially invalid if men of common intelligence must necessarily guess at its
meaning and differ as to its application.”[110] It is subject to the same principles governing
overbreadth doctrine. For one, it is also an analytical tool for testing “on their faces” statutes in
free speech cases. And like overbreadth, it is said that a litigant may challenge a statute on its
face only if it is vague in all its possible applications. Again, petitioners did not even attempt to
show that PP 1017 is vague in all its application. They also failed to establish that men of
common intelligence cannot understand the meaning and application of PP 1017.

b. Constitutional Basis of PP 1017

Now on the constitutional foundation of PP 1017.

The operative portion of PP 1017 may be divided into three important provisions, thus:

First provision:

“by virtue of the power vested upon me by Section 18, Artilce VII … do
hereby command the Armed Forces of the Philippines, to maintain law and order
throughout the Philippines, prevent or suppress all forms of lawless violence as well
any act of insurrection or rebellion”

Second provision:

“and to enforce obedience to all the laws and to all decrees, orders and
regulations promulgated by me personally or upon my direction;”

Third provision:

729
“as provided in Section 17, Article XII of the Constitution do hereby declare a
State of National Emergency.”

First Provision: Calling-out Power

The first provision pertains to the President’s calling-out power. In Sanlakas v. Executive
Secretary,[111] this Court, through Mr. Justice Dante O. Tinga, held that Section 18, Article VII of
the Constitution reproduced as follows:

Sec. 18. The President shall be the Commander-in-Chief of all armed forces
of the Philippines and whenever it becomes necessary, he may call out such armed
forces to prevent or suppress lawless violence, invasion or rebellion. In case of
invasion or rebellion, when the public safety requires it, he may, for a period not
exceeding sixty days, suspend the privilege of the writ of habeas corpus or place
the Philippines or any part thereof under martial law. Within forty-eight hours from
the proclamation of martial law or the suspension of the privilege of the writ
of habeas corpus, the President shall submit a report in person or in writing to the
Congress. The Congress, voting jointly, by a vote of at least a majority of all its
Members in regular or special session, may revoke such proclamation or
suspension, which revocation shall not be set aside by the President. Upon the
initiative of the President, the Congress may, in the same manner, extend such
proclamation or suspension for a period to be determined by the Congress, if the
invasion or rebellion shall persist and public safety requires it.

The Congress, if not in session, shall within twenty-four hours following


such proclamation or suspension, convene in accordance with its rules without need
of a call.

The Supreme Court may review, in an appropriate proceeding filed by any


citizen, the sufficiency of the factual bases of the proclamation of martial law or the
suspension of the privilege of the writ or the extension thereof, and must
promulgate its decision thereon within thirty days from its filing.

A state of martial law does not suspend the operation of the Constitution,
nor supplant the functioning of the civil courts or legislative assemblies, nor
authorize the conferment of jurisdiction on military courts and agencies over
civilians where civil courts are able to function, nor automatically suspend the
privilege of the writ.

730
The suspension of the privilege of the writ shall apply only to persons
judicially charged for rebellion or offenses inherent in or directly connected with
invasion.

During the suspension of the privilege of the writ, any person thus
arrested or detained shall be judicially charged within three days, otherwise he shall
be released.

grants the President, as Commander-in-Chief, a “sequence” of graduated powers. From the


most to the least benign, these are: the calling-out power, the power to suspend the privilege of
the writ of habeas corpus, and the power to declare Martial Law. Citing Integrated Bar of the
Philippines v. Zamora,[112] the Court ruled that the only criterion for the exercise of the calling-
out power is that “whenever it becomes necessary,” the President may call the armed forces “to
prevent or suppress lawless violence, invasion or rebellion.” Are these conditions present in the
instant cases? As stated earlier, considering the circumstances then prevailing, President Arroyo
found it necessary to issue PP 1017. Owing to her Office’s vast intelligence network, she is in
the best position to determine the actual condition of the country.

Under the calling-out power, the President may summon the armed forces to aid him in
suppressing lawless violence, invasion and rebellion. This involves ordinary police action. But
every act that goes beyond the President’s calling-out power is considered illegal or ultra
vires. For this reason, a President must be careful in the exercise of his powers. He cannot
invoke a greater power when he wishes to act under a lesser power. There lies the wisdom of
our Constitution, the greater the power, the greater are the limitations.

It is pertinent to state, however, that there is a distinction between the President’s


authority to declare a “state of rebellion” (in Sanlakas) and the authority to proclaim a state of
national emergency. While President Arroyo’s authority to declare a “state of rebellion”
emanates from her powers as Chief Executive, the statutory authority cited in Sanlakas was
Section 4, Chapter 2, Book II of the Revised Administrative Code of 1987, which provides:

SEC. 4. – Proclamations. – Acts of the President fixing a date or


declaring a status or condition of public moment or interest, upon the
existence of which the operation of a specific law or regulation is made to
depend, shall be promulgated in proclamations which shall have the force of
an executive order.

President Arroyo’s declaration of a “state of rebellion” was merely an act declaring a status
or condition of public moment or interest, a declaration allowed under Section 4 cited
731
above. Such declaration, in the words of Sanlakas, is harmless, without legal significance, and
deemed not written. In these cases, PP 1017 is more than that. In declaring a state of national
emergency, President Arroyo did not only rely on Section 18, Article VII of the Constitution, a
provision calling on the AFP to prevent or suppress lawless violence, invasion or rebellion. She
also relied on Section 17, Article XII, a provision on the State’s extraordinary power to take over
privately-owned public utility and business affected with public interest. Indeed, PP 1017 calls
for the exercise of an awesome power. Obviously, such Proclamation cannot be deemed
harmless, without legal significance, or not written, as in the case of Sanlakas.

Some of the petitioners vehemently maintain that PP 1017 is actually a declaration of


Martial Law. It is no so. What defines the character of PP 1017 are its wordings. It is plain
therein that what the President invoked was her calling-out power.

The declaration of Martial Law is a “warn[ing] to citizens that the military power has been
called upon by the executive to assist in the maintenance of law and order, and that, while the
emergency lasts, they must, upon pain of arrest and punishment, not commit any acts which will
in any way render more difficult the restoration of order and the enforcement of law.”[113]

In his “Statement before the Senate Committee on Justice” on March 13, 2006, Mr.
Justice Vicente V. Mendoza,[114] an authority in constitutional law, said that of the three powers
of the President as Commander-in-Chief, the power to declare Martial Law poses the most
severe threat to civil liberties. It is a strong medicine which should not be resorted to lightly. It
cannot be used to stifle or persecute critics of the government. It is placed in the keeping of the
President for the purpose of enabling him to secure the people from harm and to restore order
so that they can enjoy their individual freedoms. In fact, Section 18, Art. VII, provides:

A state of martial law does not suspend the operation of the Constitution, nor
supplant the functioning of the civil courts or legislative assemblies, nor authorize
the conferment of jurisdiction on military courts and agencies over civilians where
civil courts are able to function, nor automatically suspend the privilege of the writ.

Justice Mendoza also stated that PP 1017 is not a declaration of Martial Law. It is no
more than a call by the President to the armed forces to prevent or suppress lawless
violence. As such, it cannot be used to justify acts that only under a valid declaration of Martial
Law can be done. Its use for any other purpose is a perversion of its nature and scope, and any
act done contrary to its command is ultra vires.

732
Justice Mendoza further stated that specifically, (a) arrests and seizures without judicial
warrants; (b) ban on public assemblies; (c) take-over of news media and agencies and press
censorship; and (d) issuance of Presidential Decrees, are powers which can be exercised by the
President as Commander-in-Chief only where there is a valid declaration of Martial Law or
suspension of the writ of habeas corpus.

Based on the above disquisition, it is clear that PP 1017 is not a declaration of Martial
Law. It is merely an exercise of President Arroyo’s calling-out power for the armed forces to
assist her in preventing or suppressing lawless violence.

Second Provision: “Take Care” Power

The second provision pertains to the power of the President to ensure that the laws be
faithfully executed. This is based on Section 17, Article VII which reads:

SEC. 17. The President shall have control of all the executive departments,
bureaus, and offices. He shall ensure that the laws be faithfully executed.

As the Executive in whom the executive power is vested,[115] the primary function of the
President is to enforce the laws as well as to formulate policies to be embodied in existing
laws. He sees to it that all laws are enforced by the officials and employees of his
department. Before assuming office, he is required to take an oath or affirmation to the effect
that as President of the Philippines, he will, among others, “execute its laws.”[116] In the
exercise of such function, the President, if needed, may employ the powers attached to his office
as the Commander-in-Chief of all the armed forces of the country,[117] including the Philippine
National Police[118] under the Department of Interior and Local Government.[119]

Petitioners, especially Representatives Francis Joseph G. Escudero, Satur Ocampo, Rafael


Mariano, Teodoro Casiño, Liza Maza, and Josel Virador argue that PP 1017 is unconstitutional as
it arrogated upon President Arroyo the power to enact laws and decrees in violation of Section 1,
Article VI of the Constitution, which vests the power to enact laws in Congress. They assail the

733
clause “to enforce obedience to all the laws and to all decrees, orders and regulations
promulgated by me personally or upon my direction.”

Petitioners’ contention is understandable. A reading of PP 1017 operative clause shows


that it was lifted[120] from Former President Marcos’ Proclamation No. 1081, which partly reads:

NOW, THEREFORE, I, FERDINAND E. MARCOS, President of the Philippines by


virtue of the powers vested upon me by Article VII, Section 10, Paragraph (2) of
the Constitution, do hereby place the entire Philippines as defined in Article 1,
Section 1 of the Constitution under martial law and, in my capacity as their
Commander-in-Chief, do hereby command the Armed Forces of the Philippines, to
maintain law and order throughout the Philippines, prevent or suppress all forms of
lawless violence as well as any act of insurrection or rebellion and to enforce
obedience to all the laws and decrees, orders and regulations promulgated by me
personally or upon my direction.

We all know that it was PP 1081 which granted President Marcos legislative power. Its
enabling clause states: “to enforce obedience to all the laws and decrees, orders and
regulations promulgated by me personally or upon my direction.” Upon the other hand, the
enabling clause of PP 1017 issued by President Arroyo is: to enforce obedience to all the laws
and to all decrees, orders and regulations promulgated by me personally or upon my direction.”

Is it within the domain of President Arroyo to promulgate “decrees”?

PP 1017 states in part: “to enforce obedience to all the laws and decrees x x
x promulgated by me personally or upon my direction.”

The President is granted an Ordinance Power under Chapter 2, Book III of Executive Order
No. 292 (Administrative Code of 1987). She may issue any of the following:

Sec. 2. Executive Orders. — Acts of the President providing for rules of a


general or permanent character in implementation or execution of constitutional or
statutory powers shall be promulgated in executive orders.

734
Sec. 3. Administrative Orders. — Acts of the President which relate to
particular aspect of governmental operations in pursuance of his duties as
administrative head shall be promulgated in administrative orders.
Sec. 4. Proclamations. — Acts of the President fixing a date or declaring a
status or condition of public moment or interest, upon the existence of which the
operation of a specific law or regulation is made to depend, shall be promulgated in
proclamations which shall have the force of an executive order.
Sec. 5. Memorandum Orders. — Acts of the President on matters of
administrative detail or of subordinate or temporary interest which only concern a
particular officer or office of the Government shall be embodied in memorandum
orders.
Sec. 6. Memorandum Circulars. — Acts of the President on matters relating
to internal administration, which the President desires to bring to the attention of
all or some of the departments, agencies, bureaus or offices of the Government, for
information or compliance, shall be embodied in memorandum circulars.
Sec. 7. General or Special Orders. — Acts and commands of the President in
his capacity as Commander-in-Chief of the Armed Forces of the Philippines shall be
issued as general or special orders.

President Arroyo’s ordinance power is limited to the foregoing issuances. She cannot
issue decrees similar to those issued by Former President Marcos under PP 1081. Presidential
Decrees are laws which are of the same category and binding force as statutes because they
were issued by the President in the exercise of his legislative power during the period of Martial
Law under the 1973 Constitution.[121]

This Court rules that the assailed PP 1017 is unconstitutional insofar as it grants President
Arroyo the authority to promulgate “decrees.” Legislative power is peculiarly within the province
of the Legislature. Section 1, Article VI categorically states that “[t]he legislative power shall be
vested in the Congress of the Philippines which shall consist of a Senate and a House of
Representatives.” To be sure, neither Martial Law nor a state of rebellion nor a state of
emergency can justify President Arroyo’s exercise of legislative power by issuing decrees.

Can President Arroyo enforce obedience to all decrees and laws through the military?

As this Court stated earlier, President Arroyo has no authority to enact decrees. It follows
that these decrees are void and, therefore, cannot be enforced. With respect to “laws,” she
cannot call the military to enforce or implement certain laws, such as customs laws, laws
governing family and property relations, laws on obligations and contracts and the like. She can

735
only order the military, under PP 1017, to enforce laws pertinent to its duty to suppress lawless
violence.

Third Provision: Power to Take Over

The pertinent provision of PP 1017 states:

x x x and to enforce obedience to all the laws and to all decrees,


orders, and regulations promulgated by me personally or upon my
direction; and as provided in Section 17, Article XII of the
Constitution do hereby declare a state of national emergency.

The import of this provision is that President Arroyo, during the state of national
emergency under PP 1017, can call the military not only to enforce obedience “to all the laws
and to all decrees x x x” but also to act pursuant to the provision of Section 17, Article XII which
reads:

Sec. 17. In times of national emergency, when the public interest so


requires, the State may, during the emergency and under reasonable terms
prescribed by it, temporarily take over or direct the operation of any privately-
owned public utility or business affected with public interest.

What could be the reason of President Arroyo in invoking the above provision when she
issued PP 1017?

The answer is simple. During the existence of the state of national emergency, PP 1017
purports to grant the President, without any authority or delegation from Congress, to take over
or direct the operation of any privately-owned public utility or business affected with public
interest.

This provision was first introduced in the 1973 Constitution, as a product of the “martial
law” thinking of the 1971 Constitutional Convention.[122] In effect at the time of its approval was
President Marcos’ Letter of Instruction No. 2 dated September 22, 1972 instructing the Secretary

736
of National Defense to take over “the management, control and operation of the Manila Electric
Company, the Philippine Long Distance Telephone Company, the National Waterworks
and Sewerage Authority, the Philippine National Railways, the Philippine Air Lines, Air Manila
(and) Filipinas Orient Airways . . . for the successful prosecution by the Government of its effort
to contain, solve and end the present national emergency.”

Petitioners, particularly the members of the House of Representatives, claim that President
Arroyo’s inclusion of Section 17, Article XII in PP 1017 is an encroachment on the legislature’s
emergency powers.

This is an area that needs delineation.

A distinction must be drawn between the President’s authority to declare “a state of


national emergency” and toexercise emergency powers. To the first, as elucidated by the
Court, Section 18, Article VII grants the President such power, hence, no legitimate
constitutional objection can be raised. But to the second, manifold constitutional issues arise.

Section 23, Article VI of the Constitution reads:

SEC. 23. (1) The Congress, by a vote of two-thirds of both Houses in joint
session assembled, voting separately, shall have the sole power to declare the
existence of a state of war.

(2) In times of war or other national emergency, the Congress may, by law,
authorize the President, for a limited period and subject to such restrictions as it
may prescribe, to exercise powers necessary and proper to carry out a declared
national policy. Unless sooner withdrawn by resolution of the Congress, such
powers shall cease upon the next adjournment thereof.

It may be pointed out that the second paragraph of the above provision refers not only to
war but also to “other national emergency.” If the intention of the Framers of our Constitution
was to withhold from the President the authority to declare a “state of national emergency”
pursuant to Section 18, Article VII (calling-out power) and grant it to Congress (like the
declaration of the existence of a state of war), then the Framers could have provided
so. Clearly, they did not intend that Congress should first authorize the President before he can
declare a “state of national emergency.” The logical conclusion then is that President Arroyo
could validly declare the existence of a state of national emergency even in the absence of a
Congressional enactment.
737
But the exercise of emergency powers, such as the taking over of privately owned public
utility or business affected with public interest, is a different matter. This requires a delegation
from Congress.

Courts have often said that constitutional provisions in pari materia are to be construed
together. Otherwise stated, different clauses, sections, and provisions of a constitution which
relate to the same subject matter will be construed together and considered in the light of each
other.[123] Considering that Section 17 of Article XII and Section 23 of Article VI, previously
quoted, relate to national emergencies, they must be read together to determine the limitation
of the exercise of emergency powers.

Generally, Congress is the repository of emergency powers. This is evident in the tenor of
Section 23 (2), Article VI authorizing it to delegate such powers to the President. Certainly, a
body cannot delegate a power not reposed upon it. However, knowing that during grave
emergencies, it may not be possible or practicable for Congress to meet and exercise its powers,
the Framers of our Constitution deemed it wise to allow Congress to grant emergency powers to
the President, subject to certain conditions, thus:

(1) There must be a war or other emergency.

(2) The delegation must be for a limited period only.

(3) The delegation must be subject to such restrictions as the Congress may
prescribe.
(4) The emergency powers must be exercised to carry out a national
policy declared by Congress.[124]

Section 17, Article XII must be understood as an aspect of the emergency powers
clause. The taking over of private business affected with public interest is just another facet of
the emergency powers generally reposed upon Congress. Thus, when Section 17 states that the
“the State may, during the emergency and under reasonable terms prescribed by it, temporarily
take over or direct the operation of any privately owned public utility or business affected with
public interest,” it refers to Congress, not the President. Now, whether or not the President
may exercise such power is dependent on whether Congress may delegate it to him pursuant to
a law prescribing the reasonable terms thereof. Youngstown Sheet & Tube Co. et al. v.
[125]
Sawyer, held:
738
It is clear that if the President had authority to issue the order he did, it must
be found in some provision of the Constitution. And it is not claimed that express
constitutional language grants this power to the President. The contention is that
presidential power should be implied from the aggregate of his powers under the
Constitution. Particular reliance is placed on provisions in Article II which say that
“The executive Power shall be vested in a President . . . .;” that “he shall take Care
that the Laws be faithfully executed;” and that he “shall be Commander-in-Chief of
the Army and Navy of the United States.

The order cannot properly be sustained as an exercise of the President’s


military power as Commander-in-Chief of the Armed Forces. The Government
attempts to do so by citing a number of cases upholding broad powers in military
commanders engaged in day-to-day fighting in a theater of war. Such cases need
not concern us here. Even though “theater of war” be an expanding concept, we
cannot with faithfulness to our constitutional system hold that the Commander-in-
Chief of the Armed Forces has the ultimate power as such to take possession of
private property in order to keep labor disputes from stopping production. This is a
job for the nation’s lawmakers, not for its military authorities.

Nor can the seizure order be sustained because of the several constitutional
provisions that grant executive power to the President. In the framework of our
Constitution, the President’s power to see that the laws are faithfully executed
refutes the idea that he is to be a lawmaker. The Constitution limits his functions in
the lawmaking process to the recommending of laws he thinks wise and the vetoing
of laws he thinks bad. And the Constitution is neither silent nor equivocal about
who shall make laws which the President is to execute. The first section of the first
article says that “All legislative Powers herein granted shall be vested in a Congress
of the United States. . .”[126]

Petitioner Cacho-Olivares, et al. contends that the term “emergency” under Section 17,
Article XII refers to “tsunami,” “typhoon,” “hurricane” and “similar occurrences.” This is a
limited view of “emergency.”

Emergency, as a generic term, connotes the existence of conditions suddenly intensifying


the degree of existing danger to life or well-being beyond that which is accepted as
normal. Implicit in this definitions are the elements of intensity, variety, and
[127]
perception. Emergencies, as perceived by legislature or executive in the United Sates since
1933, have been occasioned by a wide range of situations, classifiable under three (3) principal
heads: a) economic,[128] b) natural disaster,[129] and c)national security.[130]

739
“Emergency,” as contemplated in our Constitution, is of the same breadth. It may include
rebellion, economic crisis, pestilence or epidemic, typhoon, flood, or other similar catastrophe of
nationwide proportions or effect.[131] This is evident in the Records of the Constitutional
Commission, thus:

MR. GASCON. Yes. What is the Committee’s definition of “national emergency”


which appears in Section 13, page 5? It reads:

When the common good so requires, the State may temporarily take over or
direct the operation of any privately owned public utility or business affected with
public interest.

MR. VILLEGAS. What I mean is threat from external aggression, for


example, calamities or natural disasters.

MR. GASCON. There is a question by Commissioner de los Reyes. What about


strikes and riots?

MR. VILLEGAS. Strikes, no; those would not be covered by the term “national
emergency.”

MR. BENGZON. Unless they are of such proportions such that they would
paralyze government service.[132]

x x x x x x

MR. TINGSON. May I ask the committee if “national emergency” refers


to military national emergency or could this be economic emergency?”

MR. VILLEGAS. Yes, it could refer to both military or economic dislocations.

MR. TINGSON. Thank you very much.[133]

It may be argued that when there is national emergency, Congress may not be able to
convene and, therefore, unable to delegate to the President the power to take over privately-
owned public utility or business affected with public interest.

In Araneta v. Dinglasan,[134] this Court emphasized that legislative power, through which
extraordinary measures are exercised, remains in Congress even in times of crisis.

“x x x
740
After all the criticisms that have been made against the efficiency of
the system of the separation of powers, the fact remains that the
Constitution has set up this form of government, with all its defects and
shortcomings, in preference to the commingling of powers in one man or
group of men. The Filipino people by adopting parliamentary government
have given notice that they share the faith of other democracy-loving
peoples in this system, with all its faults, as the ideal. The point is, under
this framework of government, legislation is preserved for Congress all the
time, not excepting periods of crisis no matter how serious. Never in the
history of the United States, the basic features of whose Constitution have
been copied in ours, have specific functions of the legislative branch of
enacting laws been surrendered to another department – unless we regard
as legislating the carrying out of a legislative policy according to prescribed
standards; no, not even when that Republic was fighting a total war, or when
it was engaged in a life-and-death struggle to preserve the Union. The truth
is that under our concept of constitutional government, in times of extreme
perils more than in normal circumstances ‘the various branches, executive,
legislative, and judicial,’ given the ability to act, are called upon ‘to perform
the duties and discharge the responsibilities committed to them
respectively.”

Following our interpretation of Section 17, Article XII, invoked by President Arroyo in
issuing PP 1017, this Court rules that such Proclamation does not authorize her during the
emergency to temporarily take over or direct the operation of any privately owned public utility
or business affected with public interest without authority from Congress.

Let it be emphasized that while the President alone can declare a state of national
emergency, however, without legislation, he has no power to take over privately-owned
public utility or business affected with public interest. The President cannot decide whether
exceptional circumstances exist warranting the take over of privately-owned public
utility or business affected with public interest. Nor can he determine when such exceptional
circumstances have ceased. Likewise, without legislation, the President has no power to point
out the types of businesses affected with public interest that should be taken over. In short,
the President has no absolute authority to exercise all the powers of the State under Section 17,
Article VII in the absence of an emergency powers act passed by Congress.

c. “AS APPLIED CHALLENGE”

741
One of the misfortunes of an emergency, particularly, that which pertains to security, is
that military necessity and the guaranteed rights of the individual are often not
compatible. Our history reveals that in the crucible of conflict, many rights are curtailed and
trampled upon. Here, the right against unreasonable search and seizure; the right against
warrantless arrest; and the freedom of speech, of expression, of the press, and of
assembly under the Bill of Rights suffered the greatest blow.

Of the seven (7) petitions, three (3) indicate “direct injury.”

In G.R. No. 171396, petitioners David and Llamas alleged that, on February 24, 2006,
they were arrested without warrants on their way to EDSA to celebrate the 20th Anniversary
of People Power I. The arresting officers cited PP 1017 as basis of the arrest.

In G.R. No. 171409, petitioners Cacho-Olivares and Tribune Publishing Co., Inc. claimed
that on February 25, 2006, the CIDG operatives “raided and ransacked without warrant” their
office. Three policemen were assigned to guard their office as a possible “source of
destabilization.” Again, the basis was PP 1017.

And in G.R. No. 171483, petitioners KMU and NAFLU-KMU et al. alleged that their
members were “turned away and dispersed” when they went to EDSA and later, to Ayala
Avenue, to celebrate the 20th Anniversary of People Power I.

A perusal of the “direct injuries” allegedly suffered by the said petitioners shows that they
resulted from theimplementation, pursuant to G.O. No. 5, of PP 1017.

Can this Court adjudge as unconstitutional PP 1017 and G.O. No 5 on the basis of these
illegal acts? In general, does the illegal implementation of a law render it unconstitutional?

Settled is the rule that courts are not at liberty to declare statutes invalid although they
may be abused and misabused[135] and may afford an opportunity for abuse in the manner of
application.[136] The validity of a statute or ordinance is to be determined from its general
purpose and its efficiency to accomplish the end desired, not from its effects in a particular
case.[137] PP 1017 is merely an invocation of the President’s calling-out power. Its general
purpose is to command the AFP to suppress all forms of lawless violence, invasion or
rebellion. It had accomplished the end desired which prompted President Arroyo to issue PP

742
1021. But there is nothing in PP 1017 allowing the police, expressly or impliedly, to conduct
illegal arrest, search or violate the citizens’ constitutional rights.

Now, may this Court adjudge a law or ordinance unconstitutional on the ground that its
implementor committed illegal acts? The answer is no. The criterion by which the validity of the
statute or ordinance is to be measured is the essential basis for the exercise of power, and not a
mere incidental result arising from its exertion.[138] This is logical. Just imagine the absurdity of
situations when laws maybe declared unconstitutional just because the officers implementing
them have acted arbitrarily. If this were so, judging from the blunders committed by policemen
in the cases passed upon by the Court, majority of the provisions of the Revised Penal Code
would have been declared unconstitutional a long time ago.

President Arroyo issued G.O. No. 5 to carry into effect the provisions of PP 1017. General
orders are “acts and commands of the President in his capacity as Commander-in-Chief of the
Armed Forces of the Philippines.” They are internal rules issued by the executive officer to his
subordinates precisely for the proper and efficient administration of law. Such rules and
regulations create no relation except between the official who issues them and the official who
receives them.[139] They are based on and are the product of, a relationship in which power is
their source, and obedience, their object.[140] For these reasons, one requirement for these rules
to be valid is that they must be reasonable, not arbitrary or capricious.

G.O. No. 5 mandates the AFP and the PNP to immediately carry out the “necessary and
appropriate actions and measures to suppress and prevent acts of terrorism and
lawless violence.”

Unlike the term “lawless violence” which is unarguably extant in our statutes and the
Constitution, and which is invariably associated with “invasion, insurrection or rebellion,” the
phrase “acts of terrorism” is still an amorphous and vague concept. Congress has yet to enact a
law defining and punishing acts of terrorism.

743
In fact, this “definitional predicament” or the “absence of an agreed definition of
terrorism” confronts not only our country, but the international community as well. The
following observations are quite apropos:

In the actual unipolar context of international relations, the “fight against


terrorism” has become one of the basic slogans when it comes to the justification of
the use of force against certain states and against groups operating
internationally. Lists of states “sponsoring terrorism” and of terrorist organizations
are set up and constantly being updated according to criteria that are not always
known to the public, but are clearly determined by strategic interests.

The basic problem underlying all these military actions – or threats of the use
of force as the most recent by the United States against Iraq – consists in the
absence of an agreed definition of terrorism.

Remarkable confusion persists in regard to the legal categorization of acts of


violence either by states, by armed groups such as liberation movements, or by
individuals.

The dilemma can by summarized in the saying “One country’s terrorist is


another country’s freedom fighter.” The apparent contradiction or lack of
consistency in the use of the term “terrorism” may further be demonstrated by the
historical fact that leaders of national liberation movements such as Nelson Mandela
in South Africa, Habib Bourgouiba in Tunisia, or Ahmed Ben Bella in Algeria, to
mention only a few, were originally labeled as terrorists by those who controlled the
territory at the time, but later became internationally respected statesmen.

What, then, is the defining criterion for terrorist acts – the differentia
specifica distinguishing those acts from eventually legitimate acts of national
resistance or self-defense?

Since the times of the Cold War the United Nations Organization has been
trying in vain to reach a consensus on the basic issue of definition. The
organization has intensified its efforts recently, but has been unable to bridge the
gap between those who associate “terrorism” with any violent act by non-state
groups against civilians, state functionaries or infrastructure or military
installations, and those who believe in the concept of the legitimate use of force
when resistance against foreign occupation or against systematic oppression of
ethnic and/or religious groups within a state is concerned.

The dilemma facing the international community can best be illustrated by


reference to the contradicting categorization of organizations and movements such
as Palestine Liberation Organization (PLO) – which is a terrorist group for Israel and
a liberation movement for Arabs and Muslims – the Kashmiri resistance groups –
who are terrorists in the perception of India, liberation fighters in that of Pakistan –
the earlier Contras in Nicaragua – freedom fighters for the United States, terrorists
for the Socialist camp – or, most drastically, the Afghani Mujahedeen (later to
become the Taliban movement): during the Cold War period they were a group of
744
freedom fighters for the West, nurtured by the United States, and a terrorist gang
for the Soviet Union. One could go on and on in enumerating examples of
conflicting categorizations that cannot be reconciled in any way – because of
opposing political interests that are at the roots of those perceptions.

How, then, can those contradicting definitions and conflicting perceptions and
evaluations of one and the same group and its actions be explained? In our
analysis, the basic reason for these striking inconsistencies lies in the divergent
interest of states. Depending on whether a state is in the position of an occupying
power or in that of a rival, or adversary, of an occupying power in a given territory,
the definition of terrorism will “fluctuate” accordingly. A state may eventually see
itself as protector of the rights of a certain ethnic group outside its territory and will
therefore speak of a “liberation struggle,” not of “terrorism” when acts of violence
by this group are concerned, and vice-versa.

The United Nations Organization has been unable to reach a decision on the
definition of terrorism exactly because of these conflicting interests of sovereign
states that determine in each and every instance how a particular armed movement
(i.e. a non-state actor) is labeled in regard to the terrorists-freedom fighter
dichotomy. A “policy of double standards” on this vital issue of international affairs
has been the unavoidable consequence.

This “definitional predicament” of an organization consisting of sovereign


states – and not of peoples, in spite of the emphasis in the Preamble to the United
Nations Charter! – has become even more serious in the present global power
constellation: one superpower exercises the decisive role in the Security Council,
former great powers of the Cold War era as well as medium powers are increasingly
being marginalized; and the problem has become even more acute since the
terrorist attacks of 11 September 2001 I the United States.[141]

The absence of a law defining “acts of terrorism” may result in abuse and oppression on
the part of the police or military. An illustration is when a group of persons are merely engaged
in a drinking spree. Yet the military or the police may consider the act as an act of terrorism
and immediately arrest them pursuant to G.O. No. 5. Obviously, this is abuse and oppression
on their part. It must be remembered that an act can only be considered a crime if there is a
law defining the same as such and imposing the corresponding penalty thereon.

So far, the word “terrorism” appears only once in our criminal laws, i.e., in P.D. No. 1835
dated January 16, 1981 enacted by President Marcos during the Martial Law regime. This decree
is entitled “Codifying The Various Laws on Anti-Subversion and Increasing The Penalties for
Membership in Subversive Organizations.” The word “terrorism” is mentioned in the following
provision: “That one who conspires with any other person for the purpose of overthrowing the

745
Government of the Philippines x x x by force, violence, terrorism, x x x shall be punished
by reclusion temporal x x x.”

P.D. No. 1835 was repealed by E.O. No. 167 (which outlaws the Communist Party of the
Philippines) enacted by President Corazon Aquino on May 5, 1985. These two (2) laws,
however, do not define “acts of terrorism.” Since there is no law defining “acts of terrorism,” it
is President Arroyo alone, under G.O. No. 5, who has the discretion to determine what acts
constitute terrorism. Her judgment on this aspect is absolute, without
restrictions. Consequently, there can be indiscriminate arrest without warrants, breaking into
offices and residences, taking over the media enterprises, prohibition and dispersal of all
assemblies and gatherings unfriendly to the administration. All these can be effected in the
name of G.O. No. 5. These acts go far beyond the calling-out power of the President. Certainly,
they violate the due process clause of the Constitution. Thus, this Court declares that the “acts
of terrorism” portion of G.O. No. 5 is unconstitutional.

Significantly, there is nothing in G.O. No. 5 authorizing the military or police to commit
acts beyond what are necessary and appropriate to suppress and prevent lawless violence, the
limitation of their authority in pursuing the Order. Otherwise, such acts are considered illegal.

We first examine G.R. No. 171396 (David et al.)

The Constitution provides that “the right of the people to be secured in their persons,
houses, papers and effects against unreasonable search and seizure of whatever nature and for
any purpose shall be inviolable, and no search warrant or warrant of arrest shall issue except
upon probable cause to be determined personally by the judge after examination under oath or
affirmation of the complainant and the witnesses he may produce, and particularly describing
the place to be searched and the persons or things to be seized.”[142] The plain import of the
746
language of the Constitution is that searches, seizures and arrests are normally unreasonable
unless authorized by a validly issued search warrant or warrant of arrest. Thus, the fundamental
protection given by this provision is that between person and police must stand the protective
authority of a magistrate clothed with power to issue or refuse to issue search warrants or
warrants of arrest.[143]

In the Brief Account[144] submitted by petitioner David, certain facts are


established: first, he was arrested without warrant; second, the PNP operatives arrested him on
the basis of PP 1017; third, he was brought at Camp Karingal, Quezon City where he
was fingerprinted, photographed and booked like a criminal suspect; fourth, he was treated
brusquely by policemen who “held his head and tried to push him” inside an unmarked
car; fifth, he was charged with Violation of Batas Pambansa Bilang No. 880[145] and Inciting
to Sedition; sixth, he was detained for seven (7) hours; and seventh, he was eventually
released for insufficiency of evidence.

Section 5, Rule 113 of the Revised Rules on Criminal Procedure provides:

Sec. 5. Arrest without warrant; when lawful. - A peace officer or a


private person may, without a warrant, arrest a person:

(a) When, in his presence, the person to be arrested has committed, is


actually committing, or is attempting to commit an offense.

(b) When an offense has just been committed and he has probable
cause to believe based on personal knowledge of facts or circumstances that
the person to be arrested has committed it; and

x x x.

Neither of the two (2) exceptions mentioned above justifies petitioner


David’s warrantless arrest. During the inquest for the charges of inciting to
sedition and violation of BP 880, all that the arresting officers could invoke was their
observation that some rallyists were wearing t-shirts with the invective “Oust Gloria

747
Now” and their erroneous assumption that petitioner David was the leader of the
[146]
rally. Consequently, the Inquest Prosecutor ordered his immediate release on the ground of
insufficiency of evidence. He noted that petitioner David was not wearing the subject t-shirt and
even if he was wearing it, such fact is insufficient to charge him with inciting to
sedition. Further, he also stated that there is insufficient evidence for the charge of violation of
BP 880 as it was not even known whether petitioner David was the leader of the rally.[147]

But what made it doubly worse for petitioners David et al. is that not only was their right
against warrantless arrest violated, but also their right to peaceably assemble.

Section 4 of Article III guarantees:

No law shall be passed abridging the freedom of speech, of expression, or of


the press, or the right of the people peaceably to assemble and petition the
government for redress of grievances.

“Assembly” means a right on the part of the citizens to meet peaceably for consultation in
respect to public affairs. It is a necessary consequence of our republican institution and
complements the right of speech. As in the case of freedom of expression, this right is not to be
limited, much less denied, except on a showing of a clear and present danger of a substantive
evil that Congress has a right to prevent. In other words, like other rights embraced in the
freedom of expression, the right to assemble is not subject to previous restraint or
censorship. It may not be conditioned upon the prior issuance of a permit or authorization from
the government authorities except, of course, if the assembly is intended to be held in a public
place, a permit for the use of such place, and not for the assembly itself, may be validly
required.

The ringing truth here is that petitioner David, et al. were arrested while they were
exercising their right to peaceful assembly. They were not committing any crime, neither was
there a showing of a clear and present danger that warranted the limitation of that right. As
can be gleaned from circumstances, the charges of inciting to sedition and violation of BP
880were mere afterthought. Even the Solicitor General, during the oral argument, failed to
748
justify the arresting officers’ conduct. InDe Jonge v. Oregon,[148] it was held that peaceable
assembly cannot be made a crime, thus:

Peaceable assembly for lawful discussion cannot be made a crime. The


holding of meetings for peaceable political action cannot be proscribed. Those who
assist in the conduct of such meetings cannot be branded as criminals on that
score. The question, if the rights of free speech and peaceful assembly are not to
be preserved, is not as to the auspices under which the meeting was held but as to
its purpose; not as to the relations of the speakers, but whether their utterances
transcend the bounds of the freedom of speech which the Constitution protects. If
the persons assembling have committed crimes elsewhere, if they have formed or
are engaged in a conspiracy against the public peace and order, they may be
prosecuted for their conspiracy or other violations of valid laws. But it is a different
matter when the State, instead of prosecuting them for such offenses, seizes upon
mere participation in a peaceable assembly and a lawful public discussion as the
basis for a criminal charge.

On the basis of the above principles, the Court likewise considers the dispersal and arrest
of the members of KMU et al.(G.R. No. 171483) unwarranted. Apparently, their dispersal was
done merely on the basis of Malacañang’s directive canceling all permits previously issued by
local government units. This is arbitrary. The wholesale cancellation of all permits to rally is a
blatant disregard of the principle that “freedom of assembly is not to be limited, much less
denied, except on a showing of a clear and present danger of a substantive evil that the State
has a right to prevent.”[149] Tolerance is the rule and limitation is the exception. Only upon a
showing that an assembly presents a clear and present danger that the State may deny the
citizens’ right to exercise it. Indeed, respondents failed to show or convince the Court that the
rallyists committed acts amounting to lawless violence, invasion or rebellion. With the blanket
revocation of permits, the distinction between protected and unprotected assemblies was
eliminated.

Moreover, under BP 880, the authority to regulate assemblies and rallies is lodged with
the local government units. They have the power to issue permits and to revoke such
permits after due notice and hearing on the determination of the presence of clear and present
danger. Here, petitioners were not even notified and heard on the revocation of their
permits.[150] The first time they learned of it was at the time of the dispersal. Such absence of
notice is a fatal defect. When a person’s right is restricted by government action, it behooves a
democratic government to see to it that the restriction is fair, reasonable, and according to
procedure.
749
G.R. No. 171409, (Cacho-Olivares, et al.) presents another facet of freedom of speech
i.e., the freedom of the press. Petitioners’ narration of facts, which the Solicitor General failed to
refute, established the following: first, the Daily Tribune’soffices were searched without
warrant; second, the police operatives seized several materials for publication; third, the search
was conducted at about 1:00 o’ clock in the morning of February 25, 2006; fourth, the search
was conducted in the absence of any official of the Daily Tribune except the security guard of the
building; and fifth, policemen stationed themselves at the vicinity of the Daily Tribune offices.

Thereafter, a wave of warning came from government officials. Presidential Chief of Staff
Michael Defensor was quoted as saying that such raid was “meant to show a ‘strong presence,’
to tell media outlets not to connive or do anything that would help the rebels in bringing down
this government.” Director General Lomibao further stated that “if they do not follow the
standards –and the standards are if they would contribute to instability in the government, or if
they do not subscribe to what is in General Order No. 5 and Proc. No. 1017 – we will recommend
a ‘takeover.’” National Telecommunications Commissioner Ronald Solis urged television and
radio networks to “cooperate” with the government for the duration of the state of national
emergency. He warned that his agency will not hesitate to recommend the closure of any
broadcast outfit that violates rules set out for media coverage during times when the national
security is threatened.[151]

The search is illegal. Rule 126 of The Revised Rules on Criminal Procedure lays down the
steps in the conduct of search and seizure. Section 4 requires that a search warrant be issued
upon probable cause in connection with one specific offence to be determined personally by the
judge after examination under oath or affirmation of the complainant and the witnesses he may
produce. Section 8 mandates that the search of a house, room, or any other premise be
made in the presence of the lawful occupant thereof or any member of his family or in the
absence of the latter, in the presence of two (2) witnesses of sufficient age and
discretion residing in the same locality. And Section 9 states that the warrant must direct that
it be served in thedaytime, unless the property is on the person or in the place ordered to be
searched, in which case a direction may be inserted that it be served at any time of the day or
night. All these rules were violated by the CIDG operatives.

750
Not only that, the search violated petitioners’ freedom of the press. The best gauge of a
free and democratic society rests in the degree of freedom enjoyed by its media. In the Burgos
v. Chief of Staff[152] this Court held that --

As heretofore stated, the premises searched were the business and printing
offices of the "Metropolitan Mail" and the "We Forum” newspapers. As a
consequence of the search and seizure, these premises were padlocked and sealed,
with the further result that the printing and publication of said newspapers were
discontinued.

Such closure is in the nature of previous restraint or censorship abhorrent to


the freedom of the press guaranteed under the fundamental law, and constitutes a
virtual denial of petitioners' freedom to express themselves in print. This state of
being is patently anathematic to a democratic framework where a free, alert and
even militant press is essential for the political enlightenment and growth of the
citizenry.

While admittedly, the Daily Tribune was not padlocked and sealed like the “Metropolitan
Mail” and “We Forum” newspapers in the above case, yet it cannot be denied that the CIDG
operatives exceeded their enforcement duties. The search and seizure of materials for
publication, the stationing of policemen in the vicinity of the The Daily Tribune offices, and the
arrogant warning of government officials to media, are plain censorship. It is that officious
functionary of the repressive government who tells the citizen that he may speak only if allowed
to do so, and no more and no less than what he is permitted to say on pain of punishment
should he be so rash as to disobey.[153] Undoubtedly, the The Daily Tribune was subjected to
these arbitrary intrusions because of its anti-government sentiments. This Court cannot
tolerate the blatant disregard of a constitutional right even if it involves the most defiant of our
citizens. Freedom to comment on public affairs is essential to the vitality of a representative
democracy. It is the duty of the courts to be watchful for the constitutional rights of the citizen,
and against any stealthy encroachments thereon. The motto should always be obsta
principiis.[154]

Incidentally, during the oral arguments, the Solicitor General admitted that the search of
the Tribune’s offices and the seizure of its materials for publication and other papers are illegal;
and that the same are inadmissible “for any purpose,” thus:
751
JUSTICE CALLEJO:

You made quite a mouthful of admission when you said that


the policemen, when inspected the Tribune for the purpose of
gathering evidence and you admitted that the policemen were
able to get the clippings. Is that not in admission of the
admissibility of these clippings that were taken from the
Tribune?

SOLICITOR GENERAL BENIPAYO:

Under the law they would seem to be, if they were illegally
seized, I think and I know, Your Honor, and these are
inadmissible for any purpose.[155]

xxx xxx xxx

SR. ASSO. JUSTICE PUNO:

These have been published in the past issues of the Daily


Tribune; all you have to do is to get those past issues. So why
do you have to go there at 1 o’clock in the morning and without
any search warrant? Did they become suddenly part of the
evidence of rebellion or inciting to sedition or what?

SOLGEN BENIPAYO:

Well, it was the police that did that, Your Honor. Not upon my
instructions.

SR. ASSO. JUSTICE PUNO:

Are you saying that the act of the policeman is illegal, it is not
based on any law, and it is not based on Proclamation 1017.

SOLGEN BENIPAYO:

It is not based on Proclamation 1017, Your Honor, because


there is nothing in 1017 which says that the police could go and
inspect and gather clippings from Daily Tribune or any other
newspaper.

SR. ASSO. JUSTICE PUNO:

Is it based on any law?

SOLGEN BENIPAYO:

752
As far as I know, no, Your Honor, from the facts, no.

SR. ASSO. JUSTICE PUNO:

So, it has no basis, no legal basis whatsoever?

SOLGEN BENIPAYO:

Maybe so, Your Honor. Maybe so, that is why I said, I don’t
know if it is premature to say this, we do not condone this. If
the people who have been injured by this would want to sue
them, they can sue and there are remedies for this.[156]

Likewise, the warrantless arrests and seizures executed by the police were, according to
the Solicitor General, illegal and cannot be condoned, thus:

CHIEF JUSTICE PANGANIBAN:

There seems to be some confusions if not contradiction in your


theory.

SOLICITOR GENERAL BENIPAYO:

I don’t know whether this will clarify. The acts, the supposed
illegal or unlawful acts committed on the occasion of 1017, as I said, it
cannot be condoned. You cannot blame the President for, as you said,
a misapplication of the law. These are acts of the police officers, that
is their responsibility.[157]

The Dissenting Opinion states that PP 1017 and G.O. No. 5 are constitutional in every
aspect and “should result in no constitutional or statutory breaches if applied according to their
letter.”

The Court has passed upon the constitutionality of these issuances. Its ratiocination has
been exhaustively presented. At this point, suffice it to reiterate that PP 1017 is limited to the
calling out by the President of the military to prevent or suppress lawless violence, invasion or
rebellion. When in implementing its provisions, pursuant to G.O. No. 5, the military and the

753
police committed acts which violate the citizens’ rights under the Constitution, this Court has to
declare such acts unconstitutional and illegal.

In this connection, Chief Justice Artemio V. Panganiban’s concurring opinion, attached


hereto, is considered an integral part of this ponencia.

SUMMATION

In sum, the lifting of PP 1017 through the issuance of PP 1021 – a supervening event –
would have normally rendered this case moot and academic. However, while PP 1017 was still
operative, illegal acts were committed allegedly in pursuance thereof. Besides, there is no
guarantee that PP 1017, or one similar to it, may not again be issued. Already, there have
been media reports on April 30, 2006 that allegedly PP 1017 would be reimposed “if the May 1
rallies” become “unruly and violent.” Consequently, the transcendental issues raised by the
parties should not be “evaded;” they must now be resolved to prevent future constitutional
aberration.

The Court finds and so holds that PP 1017 is constitutional insofar as it constitutes a call
by the President for the AFP to prevent or suppress lawless violence. The proclamation is
sustained by Section 18, Article VII of the Constitution and the relevant jurisprudence discussed
earlier. However, PP 1017’s extraneous provisions giving the President express or implied power
(1) to issue decrees; (2) to direct the AFP to enforce obedience to all laws even those not related
to lawless violence as well as decrees promulgated by the President; and (3) to impose
standards on media or any form of prior restraint on the press, are ultra
vires and unconstitutional. The Court also rules that under Section 17, Article XII of the
Constitution, the President, in the absence of a legislation, cannot take over privately-owned
public utility and private business affected with public interest.

In the same vein, the Court finds G.O. No. 5 valid. It is an Order issued by the President
– acting as Commander-in-Chief – addressed to subalterns in the AFP to carry out the provisions
of PP 1017. Significantly, it also provides a valid standard – that the military and the police
should take only the “necessary and appropriate actions and measures to suppress and prevent
acts of lawless violence.” But the words “acts of terrorism” found in G.O. No. 5 have not been

754
legally defined and made punishable by Congress and should thus be deemed deleted from the
said G.O. While “terrorism” has been denounced generally in media, no law has been enacted to
guide the military, and eventually the courts, to determine the limits of the AFP’s authority in
carrying out this portion of G.O. No. 5.

On the basis of the relevant and uncontested facts narrated earlier, it is also pristine clear
that (1) the warrantless arrest of petitioners Randolf S. David and Ronald Llamas; (2) the
dispersal of the rallies and warrantless arrest of the KMU and NAFLU-KMU members; (3) the
imposition of standards on media or any prior restraint on the press; and (4) the warrantless
search of theTribune offices and the whimsical seizures of some articles for publication and
other materials, are not authorized by the Constitution, the law and jurisprudence. Not even by
the valid provisions of PP 1017 and G.O. No. 5.

Other than this declaration of invalidity, this Court cannot impose any civil, criminal or
administrative sanctions on the individual police officers concerned. They have not been
individually identified and given their day in court. The civil complaints or causes of action
and/or relevant criminal Informations have not been presented before this Court. Elementary
due process bars this Court from making any specific pronouncement of civil, criminal or
administrative liabilities.

It is well to remember that military power is a means to an end and substantive civil
rights are ends in themselves. How to give the military the power it needs to protect the
Republic without unnecessarily trampling individual rights is one of the eternal balancing tasks of
a democratic state. During emergency, governmental action may vary in breadth and intensity
from normal times, yet they should not be arbitrary as to unduly restrain our people’s liberty.

Perhaps, the vital lesson that we must learn from the theorists who studied the various
competing political philosophies is that, it is possible to grant government the authority to cope
with crises without surrendering the two vital principles of constitutionalism: the maintenance of
legal limits to arbitrary power, and political responsibility of the government to the governed.[158]

755
WHEREFORE, the Petitions are partly granted. The Court rules that PP 1017
is CONSTITUTIONAL insofar as it constitutes a call by President Gloria Macapagal-Arroyo on the
AFP to prevent or suppress lawless violence. However, the provisions of PP 1017 commanding
the AFP to enforce laws not related to lawless violence, as well as decrees promulgated by the
President, are declared UNCONSTITUTIONAL. In addition, the provision in PP 1017 declaring
national emergency under Section 17, Article VII of the Constitution is CONSTITUTIONAL, but
such declaration does not authorize the President to take over privately-owned public utility or
business affected with public interest without prior legislation.

G.O. No. 5 is CONSTITUTIONAL since it provides a standard by which the AFP and the PNP
should implement PP 1017, i.e. whatever is “necessary and appropriate actions and measures to
suppress and prevent acts of lawless violence.” Considering that “acts of terrorism” have not
yet been defined and made punishable by the Legislature, such portion of G.O. No. 5 is
declared UNCONSTITUTIONAL.

The warrantless arrest of Randolf S. David and Ronald Llamas; the dispersal and
warrantless arrest of the KMU and NAFLU-KMU members during their rallies, in the absence of
proof that these petitioners were committing acts constituting lawless violence, invasion or
rebellion and violating BP 880; the imposition of standards on media or any form of prior
restraint on the press, as well as the warrantless search of the Tribune offices and whimsical
seizure of its articles for publication and other materials, are declared UNCONSTITUTIONAL.

No costs.

SO ORDERED.

ANGELINA SANDOVAL-GUTIERREZ
Associate Justice

WE CONCUR:

756
ARTEMIO V. PANGANIBAN
Chief Justice

(On leave)
REYNATO S. PUNO LEONARDO A. QUISUMBING
Associate Justice Associate Justice

CONSUELO YNARES-SANTIAGO ANTONIO T. CARPIO


Associate Justice Associate Justice

MA. ALICIA AUSTRIA-MARTINEZ RENATO C. CORONA


Associate Justice Associate Justice

CONCHITA CARPIO MORALES ROMEO J. CALLEJO, SR.


Associate Justice Associate Justice

ADOLFO S. AZCUNA DANTE O. TINGA


Associate Justice Associate Justice

MINITA V. CHICO-NAZARIO CANCIO C. GARCIA


Associate Justice Associate Justice

PRESBITERO J. VELASCO, JR.


Associate Justice

CERTIFICATION

Pursuant to Section 13, Article VIII of the Constitution, it is hereby certified that the
conclusions in the above Decision were reached in consultation before the case was assigned to
the writer of the opinion of the Court.

757
ARTEMIO V. PANGANIBAN
Chief Justice

*
On leave.
[1]
Law and Disorder, The Franklin Memorial Lectures, Justice Tom C. Clark – Lecturer, Volume
XIX, 1971, p. 29.
[2]
Chief Justice Artemio V. Panganiban, Liberty and Prosperity, February 15, 2006.
[3]
Articulated in the writings of the Greek philosopher, Heraclitus of Ephesus, 540-480 B.C.,
who propounded universal impermanence and that all things, notably opposites are
interrelated.
[4]
Respondents’ Comment dated March 6, 2006.
[5]
Ibid.
[6]
Ibid.
[7]
Minutes of the Intelligence Report and Security Group, Philippine Army, Annex “I” of
Respondents’ Consolidated Comment.
[8]
Respondents’ Consolidated Comment.
[9]
Ibid.
[10]
Ibid.
[11]
Petition in G.R. No. 171396, p. 5.
[12]
Police action in various parts of Metro Manila and the reactions of the huge crowds being
dispersed were broadcast as “breaking news” by the major television stations of this country.
[13]
Petition in G.R. No. 171400, p. 11.
[14]
Ibid.
[15]
The prime duty of the Government is to serve and protect the people. The Government
may call upon the people to defend the State and, in the fulfillment thereof, all citizens may
be required, under conditions provided by law, to render personal military or civil service.
[16]
No person shall be deprived of life, liberty, or property without due process of law, nor
shall any person be denied the equal protection of the laws.
[17]
The right of the people to be secure in their persons, houses, papers, and effects against
unreasonable searches and seizures of whatever nature and for any purpose shall be
inviolable, and no search warrant or warrant of arrest shall issue except upon probable cause
to be determined personally by the judge after examination under oath or affirmation of the
complainant and the witnesses he may produce, and particularly describing the place to be
searched and the persons or things to be seized.

758
[18]
No law shall be passed abridging the freedom of speech, of expression, or of the press, or
the right of the people peaceably to assemble and petition the Government for redress of
grievances.
[19]
(1) The Congress, by a vote of two-thirds of both Houses in joint session assembled, voting
separately, shall have the sole power to declare the existence of a state of war.
(2) In times of war or other national emergency, the Congress may, by law, authorize the
President, for a limited period and subject to such restrictions as it may prescribe, to exercise
powers necessary and proper to carry out a declared national policy. Unless sooner
withdrawn by resolution of the Congress, such powers shall cease upon the next adjournment
thereof.
[20]
In times of national emergency, when the public interest so requires, the State may,
during the emergency and under reasonable terms prescribed by it, temporarily take over or
direct the operation of any privately owned public utility or business affected with public
interest.
[21]
1 Cranch 137 [1803].
[22]
Howard L. MacBain, “Some Aspects of Judicial Review,” Bacon Lectures on the Constitution
of the United States (Boston: Boston University Heffernan Press, 1939), pp. 376-77.
[23]
The Court has no self-starting capacity and must await the action of some litigant so
aggrieved as to have a justiciable case. (Shapiro and Tresolini, American Constitutional Law,
Sixth Edition, 1983, p. 79).
[24]
Cruz, Philippine Political Law, 2002 Ed., p. 259.
[25]
Ibid.
[26]
Province of Batangas v. Romulo, G.R. No. 152774, May 27, 2004, 429 SCRA 736.
[27]
Banco Filipino Savings and Mortgage Bank v. Tuazon, Jr., G.R. No. 132795, March 10,
2004, 425 SCRA 129; Vda. De Dabao v. Court of Appeals, G.R. No. 1165, March 23, 2004,
426 SCRA 91; and Paloma v. Court of Appeals, G.R. No. 145431, November 11, 2003, 415
SCRA 590.
[28]
Royal Cargo Corporation v. Civil Aeronautics Board, G.R. Nos. 103055-56, January 26,
2004, 421 SCRA 21; Vda. De Dabao v. Court of Appeals, supra.
[29]
Lacson v. Perez, G.R. No. 147780, May 10, 2001, 357 SCRA 756.
[30]
Cruz, Philippine Political Law, 2002, p. 268 citing Norton v. Shelby, 118 U.S. 425.
[31]
Province of Batangas v. Romulo, supra.
[32]
Lacson v. Perez, supra.
[33]
Province of Batangas v. Romulo, supra.
[34]
Albaña v. Commission on Elections, G.R. No. 163302, July 23, 2004, 435 SCRA 98, Acop
v. Guingona, Jr., G.R. No. 134855, July 2, 2002, 383 SCRA 577, Sanlakas v. Executive
Secretary, G.R. No. 159085, February 3, 2004, 421 SCRA 656.
[35]
Salonga v. Cruz Paño, et al., No. L- 59524, February 18, 1985, 134 SCRA 438.
[36]
G.R. No. 159085, February 3, 2004, 421 SCRA 656.

759
[37]
Black’s Law Dictionary, 6th Ed. 1991, p. 941.
[38]
Salonga v. Warner Barnes & Co., 88 Phil. 125 (1951).
[39]
275 Ky 91, 120 SW2d 765 (1938).
[40]
19 Wend. 56 (1837).
[41]
232 NC 48, 59 SE2d 359 (1950).
[42]
302 U.S. 633.
[43]
318 U.S. 446.
[44]
65 Phil. 56 (1937).
[45]
G.R. No. 117, November 7, 1945 (Unreported).
[46]
G.R. No. 2947, January 11, 1959 (Unreported).
[47]
110 Phil. 331 (1960).
[48]
77 Phil. 1012 (1947).
[49]
84 Phil. 368 (1949) The Court held: “Above all, the transcendental importance to the public
of these cases demands that they be settled promptly and definitely, brushing aside, if we
must, technicalities of procedure.”
[50]
L-No. 40004, January 31, 1975, 62 SCRA 275.
[51]
Tañada v. Tuvera, G.R. No. 63915, April 24, 1985, 136 SCRA 27, where the Court held
that where the question is one of public duty and the enforcement of a public right, the
people are the real party in interest, and it is sufficient that the petitioner is a citizen
interested in the execution of the law;
Legaspi v. Civil Service Commission, G.R. No. 72119, May 29, 1987, 150 SCRA 530,
where the Court held that in cases involving an assertion of a public right, the requirement of
personal interest is satisfied by the mere fact that the petitioner is a citizen and part of the
general public which possesses the right.
Kapatiran ng mga Naglilingkod sa Pamahalaan ng Pilipinas, Inc. v. Tan, L. No. 81311,
June 30, 1988, 163 SCRA 371, where the Court held that objections to taxpayers’ lack of
personality to sue may be disregarded in determining the validity of the VAT law;
Albano v. Reyes, G.R. No. 83551, July 11, 1989, 175 SCRA 264, where the Court held
that while no expenditure of public funds was involved under the questioned contract,
nonetheless considering its important role in the economic development of the country and
the magnitude of the financial consideration involved, public interest was definitely involved
and this clothed petitioner with the legal personality under the disclosure provision of the
Constitution to question it.
Association of Small Landowners in the Philippines, Inc. v. Sec. of Agrarian Reform, G.R.
No. 78742, July 14, 1989, 175 SCRA 343, where the Court ruled that while petitioners are
strictly speaking, not covered by the definition of a “proper party,” nonetheless, it has the
discretion to waive the requirement, in determining the validity of the implementation of the
CARP.
Gonzales v. Macaraig, Jr., G.R. No. 87636, November 19, 1990, 191 SCRA 452, where the
Court held that it enjoys the open discretion to entertain taxpayer’s suit or not and that a
member of the Senate has the requisite personality to bring a suit where a constitutional
issue is raised.
760
Maceda v. Macaraig, Jr., G.R. No. 88291, May 31, 1991, 197 SCRA 771, where the Court
held that petitioner as a taxpayer, has the personality to file the instant petition, as the
issues involved, pertains to illegal expenditure of public money;
Osmeña v. Comelec, G.R. No. 100318, 100308, 100417,100420, July 30, 1991, 199
SCRA 750, where the Court held that where serious constitutional questions are involved,
the “transcendental importance” to the public of the cases involved demands that they be
settled promptly and definitely, brushing aside technicalities of procedures;
De Guia v. Comelec, G.R. No. 104712, May 6, 1992, 208 SCRA 420, where the Court
held that the importance of the issues involved concerning as it does the political exercise of
qualified voters affected by the apportionment, necessitates the brushing aside of the
procedural requirement of locus standi.
[52]
G.R. No. 133250, July 9, 2002, 384 SCRA 152.
[53]
G.R. Nos. 138570, 138572, 138587, 138680, 138698, October 10, 2000, 342 SCRA 449.
[54]
G.R. No. 151445, April 11, 2002, 380 SCRA 739.
[55]
Supra.
[56]
G.R. No. 118910, November 16, 1995, 250 SCRA 130.
[57]
G.R. No. 132922, April 21, 1998, 289 SCRA 337.
[58]
G.R. No. 147780, 147781, 147799, 147810, May 10, 2001, 357 SCRA 756.
[59]
G.R. No. 159085, February 3, 2004, 421 SCRA 656.
[60]
235 SCRA 506 (1994).
[61]
Supra.
[62]
Supra.
[63]
197 SCRA 52, 60 (1991).
[64]
Supra.
[65]
See NAACP v. Alabama, 357 U.S. 449 (1958).
[66]
G.R. No. 141284, August 15, 2000, 338 SCRA 81.
[67]
From the deliberations of the Constitutional Commission, the intent of the framers is clear
that the immunity of the President from suit is concurrent only with his tenure and not his
term. (De Leon, Philippine Constitutional Law, Vol. 2, 2004 Ed., p. 302).
[68]
Section 1, Article XI of the Constitution provides: Public Office is a public trust. Public
officers and employees must at all times be accountable to the people, serve them with
utmost responsibility, integrity, loyalty and efficiency, act with patriotism and justice, and
lead modest lives.
[69]
Ibid., Sec. 2.

761
[70]
No. 2908, September 30, 2005, 471 SCRA 87.
[71]
91 Phil. 882 (1952).
[72]
No. L-33964, December 11, 1971, 42 SCRA 448.
[73]
No. L-35546, September 17, 1974, 59 SCRA 183.
[74]
No. L-61388, April 20, 1983, 121 SCRA 472.
[75]
Tañada v. Cuenco, 103 Phil. 1051 (1957).
[76]
Lansang v. Garcia, supra, pp. 473 and 481.
[77]
Supra.
[78]
“Five Justices – Antonio, Makasiar, Esguerra, Fernandez, and Aquino – took the position
that the proclamation of martial law and the arrest and detention orders accompanying the
proclamation posed a “political question” beyond the jurisdiction of the Court. Justice
Antonio, in a separate opinion concurred in by Makasiar, Fernandez, and Aquino, argued that
the Constitution had deliberately set up a strong presidency and had concentrated powers in
times of emergency in the hands of the President and had given him broad authority and
discretion which the Court was bound to respect. He made reference to the decision
in Lansang v. Garcia but read it as in effect upholding the “political question”
position. Fernandez, in a separate opinion, also argued Lansang, even understood as giving
a narrow scope of review authority to the Court, affirmed the impossible task of ‘checking’
the action taken by the President. Hence, he advocated a return to Barcelon v.
Baker. Similarly, Esguerra advocated the abandonment ofLansang and a return
to Barcelon. And, although Justices Castro, Fernando, Muñoz- Palma, and, implicitly,
Teehankee, lined up on the side of justiciability as enunciated inLansang, x x x Barredo,
however, wanted to have the best of both worlds and opted for the view that “political
questions are not per se beyond the Court’s jurisdiction ... but that as a matter of policy
implicit in the Constitution itself the Court should abstain from interfering with the Executive’s
Proclamation.” (Bernas, The 1987 Constitution of the Republic of the Philippines: A
Commentary, 1996 Edition, p. 794.)
[79]
See Separate Opinion of J. Puno in Integrated Bar of the Philippines v. Zamora, supra.
[80]
Supra.
[81]
Cruz, Philippine Political Law, 2002 Ed., p. 247.
[82]
Santiago v. Guingona, Jr., G.R. No. 134577, November 18, 1998, 298 SCRA 756.
[83]
Supra, 481-482.
[84]
Smith and Cotter, Powers of the President during Crises, 1972, p. 6.
[85]
Ibid.
[86]
The Social Contract (New York: Dutton, 1950), pp. 123-124.
[87]
Smith and Cotter, Powers of the President during Crises, 1972, pp. 6-7.
[88]
Representative Government, New York, Dutton, 1950, pp. 274, 277-78.
[89]
The Discourses, Bk. 1, Ch. XXXIV.
[90]
Smith and Cotter, Powers of the President During Crises, 1972. p. 8.

762
[91]
Ibid.
[92]
See The Problem of Constitutional Dictatorship, p. 328.
[93]
Ibid., p. 353.
[94]
Ibid., pp. 338-341.
[95]
Smith and Cotter, Powers of the President During Crises, 1972, p. 9.
[96]
Constitutional Government and Democracy, Ch. XXVI, rev. ed., Boston: Ginn & Co., 1949,
p. 580.
[97]
Ibid, pp. 574-584.
[98]
Smith and Cotter, Powers of the President During Crises, 1972, p. 10.
[99]
Rossiter, Constitutional Dictatorship, Princeton: Princeton University Press, 1948, pp. 298-
306.
[100]
Smith and Cotter, Powers of the President During Crises, 1972, p. 11.
[101]
Smith and Cotter, Powers of the President During Crises, 1972, p. 12.
[102]
Youngstown Sheet and Tube Co. v. Sawyer, 343 U.S. 579; 72 Sup. Ct. 863; 96 L. Ed. 1153
(1952), See Concurring Opinion J. Jackson.
[103]
See Concurring Opinion of Justice Mendoza in Estrada v. Sandiganbayan, G.R. No.
148560, November 19, 2001, 369 SCRA 393.
[104]
481 U.S. 739, 95 L. Ed. 2d 697 (1987).
[105]
Supra.
[106]
See Concurring Opinion of Justice Mendoza in Estrada v. Sandiganbayan, supra.
[107]
Broadrick v. Oklahoma, 413 U.S. 601 (1973).
[108]
Ibid.
[109]
401 U.S. 37, 52-53, 27 L.Ed.2d 669, 680 (1971), United States v. Raines, 362 U.S. 17, 4
L.Ed.2d 524 (1960); Board of Trustees, State Univ. of N.Y v. Fox, 492 U.S. 469, 106 L.Ed.2d
388 (1989).
[110]
Ermita-Malate Hotel and Motel Operators Association v. City Mayor, No. L-24693, July
31, 1967, 20 SCRA 849 (1967).
[111]
G.R. No. 159085, February 3, 2004, 421 SCRA 656, wherein this Court sustained
President Arroyo’s declaration of a “state of rebellion” pursuant to her calling-out power.

[112]
Supra.
[113]
Westel Willoughby, Constitutional Law of the United States 1591 [2d Ed. 1929, quoted
in Aquino v. Ponce Enrile, 59 SCRA 183 (1974), (Fernando, J., concurring)].
[114]
Retired Associate Justice of the Supreme Court.

763
[115]
Section 1, Article VII of the Constitution.
[116]
Section 5, Article VII of the Constitution.
[117]
Section 18, Article VII of the Constitution.
[118]
Section 6, Article XVI of the Constitution.
[119]
See Republic Act No. 6975.
[120]
Ironically, even the 7th Whereas Clause of PP 1017 which states that “Article 2, Section 4 of
our Constitution makes the defense and preservation of the democratic institutions and
the State the primary duty of Government” replicates more closely Section 2, Article 2 of
the 1973 Constitution than Section 4, Article 2 of the 1987 Constitution which provides
that, “[t[he prime duty of the Government is to serve and protect the people.”
[121]
Agpalo, Statutory Construction, Fourth Edition, 1998, p. 1, citing Legaspi v. Ministry of
Finance, 115 SCRA 418 (1982); Garcia-Padilla v. Ponce-Enrile, supra. Aquino v. Commission
on Election, supra.
[122]
Section 17, Article XIV of the 1973 Constitution reads: “In times of national emergency
when the public interest so requires, the State may temporarily take over or direct the
operation of any privately owned public utility or business affected with public interest.”
[123]
Antieau, Constitutional Construction, 1982, p.21.
[124]
Cruz, Philippine Political Law, 1998, p. 94.
[125]
343 U.S. 579; 72 Sup. Ct. 863; 96 L. Ed. 1153 (1952).
[126]
Tresolini, American Constitutional Law, 1959, Power of the President, pp. 255-257.
[127]
Smith and Cotter, Powers of the President During Crises, 1972, p. 14
[128]
The Federal Emergency Relief Act of 1933 opened with a declaration that the economic
depression created a serious emergency, due to wide-spread unemployment and the
inadequacy of State and local relief funds, . . . making it imperative that the Federal
Government cooperate more effectively with the several States and Territories and the
District of Columbia in furnishing relief to their needy and distressed people. President
Roosevelt in declaring a bank holiday a few days after taking office in 1933 proclaimed that
“heavy and unwarranted withdrawals of gold and currency from … banking institutions for the
purpose of hoarding; ... resulting in “sever drains on the Nation’s stocks of gold … have
created a national emergency,” requiring his action. Enacted within months after Japan’s
attack on Pearl Harbor, the Emergency Price Control Act of 1942 was designed to
prevent economic dislocations from endangering the national defense and security and the
effective prosecution of the war. (Smith and Cotter, Powers of the President During
Crises, 1972, p.18)
[129]
The Emergency Appropriation Act for Fiscal 1935 appropriated fund to meet the emergency
and necessity for relief in stricken agricultural areas and in another section referred to “the
764
present drought emergency.”[129] The India Emergency Food Aid Act of 1951 provided for
emergency shipments of food to India to meet famine conditions then ravaging the great
Asian sub-continent. The Communication Act of 1934 and its 1951 amendment grant the
President certain powers in time of “public peril or disaster.” The other statutes provide for
existing or anticipated emergencies attributable to earthquake, flood, tornado, cyclone,
hurricane, conflagration an landslides.[129] There is also a Joint Resolution of April 1937. It
made “funds available for the control of incipient or emergency outbreaks of insect pests or
plant diseases, including grasshoppers, Mormon crickets, and chinch bugs. (66 Stat 315, July
1, 1952, Sec. 2 [a]) Supra.
[130]
National Security may be cataloged under the heads of (1) Neutrality, (2) Defense, (3) Civil
Defense, and (4) Hostilities or War. (p. 22) The Federal Civil Defense Act of
1950 contemplated an attack or series of attacks by an enemy of the United States which
conceivably would cause substantial damage or injury to civilian property or persons in the
United States by any one of several means; sabotage, the use of bombs, shellfire, or atomic,
radiological, chemical, bacteriological means or other weapons or processes. Such an
occurrence would cause a “National Emergency for Civil Defense Purposes,” or “a state of
civil defense emergency,” during the term which the Civil Defense Administrator would have
recourse to extraordinary powers outlined in the Act. TheNew York-New Jersey Civil Defense
Compact supplies an illustration in this context for emergency cooperation. “Emergency” as
used in this compact shall mean and include invasion, or otherhostile
action, disaster, insurrection or imminent danger thereof. ( Id., p.15-16)
[131]
Cruz, Philippine Political Law, 1998, p. 95.
[132]
Record of the Constitutional Commission, Vol. III, pp. 266-267.
[133]
Record of the Constitutional Convention, pp. 648-649.
[134]
84 Phil. 368 (1949).
[135]
Uren v Bagley, 118 Or 77, 245 P 1074, 46 ALR 1173.
[136]
Gutierrez v. Middle Rio Grande Conservancy Dist., 34 NM 346, 282 P 1, 70 ALR 1261, cert
den 280 US 610, 74 L ed 653, 50 S Ct 158.
[137]
Sanitation Dist. V. Campbell (Ky), 249 SW 2d 767; Rochester v. Gutberlett, 211 NY 309,
105 NE 548.
[138]
Hammond Packing Co. v. Arkansas, 212 US 322, 53 L ed 530, 29 S Ct 370.
[139]
De Leon and De Leon Jr., Administrative Law, Text and Cases, 2001 Ed., p. 115.
[140]
Ibid.
[141]
In a Lecture delivered on March 12, 2002 as part of the Supreme Court Centenary
Lecture Series, Hans Koechler, Professor of Philosophy at the University of Innsbruck
(Austria) and President of the International Progress Organization, speaking on “The United
Nations, The International Rule of Law and Terrorism” cited in the Dissenting Opinion of
Justice Kapunan in Lim v. Executive Secretary, G.R. No. 151445, April 11, 2002, 380 SCRA
739.
[142]
Section 2, Article III of the 1987 Constitution.
[143]
Bernas, The 1987 Constitution of the Republic of the Philippines, A Reviewer-Primer, p. 51.
765
[144]
Annex “A” of the Memorandum in G.R. No. 171396, pp. 271-273.
[145]
An Act Ensuring the Free Exercise by the People of their Right Peaceably to Assemble and
Petition the Government for Other Purposes.

[146]
Annex “A” of the Memorandum in G.R. No. 171396, pp. 271-273.
[147]
Ibid.
[148]
299 U.S. 353, 57 S. Ct. 255, 81 L. Ed. 278.
[149]
Reyes v. Bagatsing, No. L-65366, November 9, 1983, 125 SCRA 553.
[150]
Section 5. Application requirements - All applications for a permit shall comply with the
following guidelines:

x x x x x x

(c) If the mayor is of the view that there is imminent and grave danger of
a substantive evil warranting the denial or modification of the permit, he shall
immediately inform the applicant who must be heard on the matter.

[151]
Petition in G.R. No. 171400, p. 11.
[152]
No. L-64161, December 26, 1984, 133 SCRA 816.
[153]
Dissenting Opinion, J. Cruz, National Press Club v. Commission on Elections, G.R. Nos.
102653, 102925 & 102983, March 5, 1992, 207 SCRA 1.
[154]
Boyd v. United States, 116 U.S. 616 (1886).
[155]
Transcript of Stenographic Notes, Oral Arguments, March 7, 2006, p. 470.
[156]
Ibid., pp. 432-433.
[157]
Ibid, pp. 507-508.
[158]
Smith and Cotter, Powers of the President During Crisis, 1972, p. 146.

766
EN BANC

ELEAZAR P. QUINTO and G.R. No. 189698


GERINO A. TOLENTINO, JR.,
Petitioners, Present:

PUNO, C.J.,
CARPIO,
CORONA,
CARPIO MORALES,
CHICO-NAZARIO,
VELASCO, JR.,
- versus - NACHURA,
LEONARDO-DE CASTRO,
BRION,
PERALTA,
BERSAMIN,
DEL CASTILLO,
ABAD, and
VILLARAMA, JR., JJ.

COMMISSION ON ELECTIONS, Promulgated:


Respondent.
December 1, 2009
x-----------------------------------------------------------------------------------------x

DECISION

NACHURA, J.:

“In our predisposition to discover the ‘original intent’ of a statute, courts become the
unfeeling pillars of the status quo. Little do we realize that statutes or even constitutions are
bundles of compromises thrown our way by their framers. Unless we exercise vigilance, the
statute may already be out of tune and irrelevant to our day.”[1] It is in this light that we should
address the instant case.

Before the Court is a petition for prohibition and certiorari, with prayer for the issuance of
a temporary restraining order and a writ of preliminary injunction, assailing Section 4(a) of
Resolution No. 8678 of the Commission on Elections (COMELEC). In view of pressing
contemporary events, the petition begs for immediate resolution.

The Antecedents

767
This controversy actually stems from the law authorizing the COMELEC to use an
automated election system (AES).

On December 22, 1997, Congress enacted Republic Act (R.A.) No. 8436, entitled “AN ACT
AUTHORIZING THE COMMISSION ON ELECTIONS TO USE AN AUTOMATED ELECTION SYSTEM IN
THE MAY 11, 1998 NATIONAL OR LOCAL ELECTIONS AND IN SUBSEQUENT NATIONAL AND
LOCAL ELECTORAL EXERCISES, PROVIDING FUNDS THEREFOR AND FOR OTHER PURPOSES.”
Section 11 thereof reads:

SEC. 11. Official Ballot.—The Commission shall prescribe the size and form of
the official ballot which shall contain the titles of the positions to be filled and/or the
propositions to be voted upon in an initiative, referendum or plebiscite. Under each
position, the names of candidates shall be arranged alphabetically by surname and
uniformly printed using the same type size. A fixed space where the chairman of
the Board of Election inspectors shall affix his/her signature to authenticate the
official ballot shall be provided.

Both sides of the ballots may be used when necessary.

For this purpose, the deadline for the filing of certificate of candidacy/petition
for registration/manifestation to participate in the election shall not be later than
one hundred twenty (120) days before the elections: Provided, That, any elective
official, whether national or local, running for any office other than the one which
he/she is holding in a permanent capacity, except for president and vice president,
shall be deemed resigned only upon the start of the campaign period corresponding
to the position for which he/she is running: Provided, further, That, unlawful acts or
omissions applicable to a candidate shall take effect upon the start of the aforesaid
campaign period: Provided, finally, That, for purposes of the May 11, 1998
elections, the deadline for filing of the certificate of candidacy for the positions of
President, Vice President, Senators and candidates under the Party-List System as
well as petitions for registration and/or manifestation to participate in the Party-List
System shall be on February 9, 1998 while the deadline for the filing of certificate
of candidacy for other positions shall be on March 27, 1998.

The official ballots shall be printed by the National Printing Office and/or
the Bangko Sentral ng Pilipinas at the price comparable with that of private printers
under proper security measures which the Commission shall adopt. The
Commission may contract the services of private printers upon certification by the
National Printing Office/Bangko Sentral ng Pilipinas that it cannot meet the printing
requirements. Accredited political parties and deputized citizens' arms of the
Commission may assign watchers in the printing, storage and distribution of official
ballots.

To prevent the use of fake ballots, the Commission through the Committee
shall ensure that the serial number on the ballot stub shall be printed in magnetic
ink that shall be easily detectable by inexpensive hardware and shall be impossible
to reproduce on a photocopying machine and that identification marks, magnetic
768
strips, bar codes and other technical and security markings, are provided on the
ballot.

The official ballots shall be printed and distributed to each city/municipality at


the rate of one (1) ballot for every registered voter with a provision of additional
four (4) ballots per precinct.[2]

Almost a decade thereafter, Congress amended the law on January 23, 2007 by enacting
R.A. No. 9369, entitled “AN ACT AMENDING REPUBLIC ACT NO. 8436, ENTITLED ‘AN ACT
AUTHORIZING THE COMMISSION ON ELECTIONS TO USE AN AUTOMATED ELECTION SYSTEM IN
THE MAY 11, 1998 NATIONAL OR LOCAL ELECTIONS AND IN SUBSEQUENT NATIONAL AND
LOCAL ELECTORAL EXERCISES, TO ENCOURAGE TRANSPARENCY, CREDIBILITY, FAIRNESS AND
ACCURACY OF ELECTIONS, AMENDING FOR THE PURPOSE BATAS PAMPANSA BLG. 881, AS
AMEMDED, REPUBLIC ACT NO. 7166 AND OTHER RELATED ELECTION LAWS, PROVIDING FUNDS
THEREFOR AND FOR OTHER PURPOSES.’” Section 13 of the amendatory law modified Section 11
of R.A. No. 8436, thus:

SEC. 13. Section 11 of Republic Act No. 8436 is hereby amended to read as
follows:

“Section 15. Official Ballot.—The Commission shall prescribe the format of


the electronic display and/or the size and form of the official ballot, which shall
contain the titles of the position to be filled and/or the propositions to be voted
upon in an initiative, referendum or plebiscite. Where practicable, electronic
displays must be constructed to present the names of all candidates for the same
position in the same page or screen, otherwise, the electronic displays must be
constructed to present the entire ballot to the voter, in a series of sequential pages,
and to ensure that the voter sees all of the ballot options on all pages before
completing his or her vote and to allow the voter to review and change all ballot
choices prior to completing and casting his or her ballot. Under each position to be
filled, the names of candidates shall be arranged alphabetically by surname and
uniformly indicated using the same type size. The maiden or married name shall be
listed in the official ballot, as preferred by the female candidate. Under each
proposition to be vote upon, the choices should be uniformly indicated using the
same font and size.

“A fixed space where the chairman of the board of election inspectors shall
affix his/her signature to authenticate the official ballot shall be provided.

“For this purpose, the Commission shall set the deadline for the filing of
certificate of candidacy/petition of registration/manifestation to participate in the
election. Any person who files his certificate of candidacy within this period shall
only be considered as a candidate at the start of the campaign period for which he
filed his certificate of candidacy: Provided, That, unlawful acts or omissions
applicable to a candidate shall take effect only upon the start of the aforesaid
769
campaign period:Provided, finally, That any person holding a public appointive
office or position, including active members of the armed forces, and officers and
employees in government-owned or -controlled corporations, shall be
considered ipso facto resigned from his/her office and must vacate the same at the
start of the day of the filing of his/her certificate of candidacy.

“Political parties may hold political conventions to nominate their official


candidates within thirty (30) days before the start of the period for filing a
certificate of candidacy.

“With respect to a paper-based election system, the official ballots shall be


printed by the National Printing Office and/or the Bangko Sentral ng Pilipinas at the
price comparable with that of private printers under proper security measures
which the Commission shall adopt. The Commission may contract the services of
private printers upon certification by the National Printing Office/Bangko Sentral ng
Pilipinasthat it cannot meet the printing requirements. Accredited political parties
and deputized citizens’ arms of the Commission shall assign watchers in the
printing, storage and distribution of official ballots.

“To prevent the use of fake ballots, the Commission through the Committee
shall ensure that the necessary safeguards, such as, but not limited to, bar codes,
holograms, color shifting ink, microprinting, are provided on the ballot.

“The official ballots shall be printed and distributed to each city/municipality


at the rate of one ballot for every registered voter with a provision of additional
three ballots per precinct.”[3]

Pursuant to its constitutional mandate to enforce and administer election laws, COMELEC
issued Resolution No. 8678,[4]the Guidelines on the Filing of Certificates of Candidacy (CoC) and
Nomination of Official Candidates of Registered Political Parties in Connection with the May 10,
2010 National and Local Elections. Sections 4 and 5 of Resolution No. 8678 provide:

SEC. 4. Effects of Filing Certificates of Candidacy.—a) Any person holding a


public appointive office or position including active members of the Armed Forces of
the Philippines, and other officers and employees in government-owned or
controlled corporations, shall be considered ipso facto resigned from his office upon
the filing of his certificate of candidacy.

b) Any person holding an elective office or position shall not be considered


resigned upon the filing of his certificate of candidacy for the same or any other
elective office or position.

770
SEC. 5. Period for filing Certificate of Candidacy.—The certificate of candidacy
shall be filed on regular days, from November 20 to 30, 2009, during office hours,
except on the last day, which shall be until midnight.

Alarmed that they will be deemed ipso facto resigned from their offices the moment they
file their CoCs, petitioners Eleazar P. Quinto and Gerino A. Tolentino, Jr., who hold appointive
positions in the government and who intend to run in the coming elections,[5] filed the instant
petition for prohibition and certiorari, seeking the declaration of the afore-quoted Section 4(a) of
Resolution No. 8678 as null and void.

The Petitioners’ Contention

Petitioners contend that the COMELEC gravely abused its discretion when it issued the
assailed Resolution. They aver that the advance filing of CoCs for the 2010 elections is intended
merely for the purpose of early printing of the official ballots in order to cope with time
limitations. Such advance filing does not automatically make the person who filed the CoC a
candidate at the moment of filing. In fact, the law considers him a candidate only at the start of
the campaign period. Petitioners then assert that this being so, they should not be deemed ipso
facto resigned from their government offices when they file their CoCs, because at such time
they are not yet treated by law as candidates. They should be considered resigned from their
respective offices only at the start of the campaign period when they are, by law, already
considered as candidates.[6]

Petitioners also contend that Section 13 of R.A. No. 9369, the basis of the assailed
COMELEC resolution, contains two conflicting provisions. These must be harmonized or
reconciled to give effect to both and to arrive at a declaration that they are not ipso
facto resigned from their positions upon the filing of their CoCs.[7]

Petitioners further posit that the provision considering them as ipso facto resigned from
office upon the filing of their CoCs is discriminatory and violates the equal protection clause in
the Constitution.[8]

The Respondent’s Arguments

On the procedural aspect of the petition, the Office of the Solicitor General (OSG),
representing respondent COMELEC, argues that petitioners have no legal standing to institute
the suit. Petitioners have not yet filed their CoCs, hence, they are not yet affected by the
assailed provision in the COMELEC resolution. The OSG further claims that the petition is
premature or unripe for judicial determination. Petitioners have admitted that they are merely
771
planning to file their CoCs for the coming 2010 elections. Their interest in the present
controversy is thus merely speculative and contingent upon the filing of the same. The OSG
likewise contends that petitioners availed of the wrong remedy. They are questioning an
issuance of the COMELEC made in the exercise of the latter’s rule-making
[9]
power. Certiorari under Rule 65 is then an improper remedy.

On the substantive aspect, the OSG maintains that the COMELEC did not gravely abuse
its discretion in phrasing Section 4(a) of Resolution No. 8678 for it merely copied what is in the
law. The OSG, however, agrees with petitioners that there is a conflict in Section 13 of R.A. No.
9369 that should be resolved. According to the OSG, there seems to be no basis to consider
appointive officials as ipso facto resigned and to require them to vacate their positions on the
same day that they file their CoCs, because they are not yet considered as candidates at that
time. Further, this “deemed resigned” provision existed in Batas Pambansa Bilang (B.P. Blg.)
881, and no longer finds a place in our present election laws with the innovations brought about
by the automated system.[10]

Our Ruling

I.

At first glance, the petition suffers from an incipient procedural defect. What petitioners
assail in their petition is a resolution issued by the COMELEC in the exercise of its quasi-
legislative power. Certiorari under Rule 65, in relation to Rule 64, cannot be availed of, because
it is a remedy to question decisions, resolutions and issuances made in the exercise of a judicial
or quasi-judicial function.[11] Prohibition is also an inappropriate remedy, because what
petitioners actually seek from the Court is a determination of the proper construction of a
statute and a declaration of their rights thereunder. Obviously, their petition is one for
declaratory relief,[12] over which this Court does not exercise original jurisdiction.[13]

However, petitioners raise a challenge on the constitutionality of the questioned provisions


of both the COMELEC resolution and the law. Given this scenario, the Court may step in and
resolve the instant petition.

The transcendental nature and paramount importance of the issues raised and the
compelling state interest involved in their early resolution—the period for the filing of CoCs for
the 2010 elections has already started and hundreds of civil servants intending to run for
elective offices are to lose their employment, thereby causing imminent and irreparable damage
to their means of livelihood and, at the same time, crippling the government’s manpower—
772
further dictate that the Court must, for propriety, if only from a sense of obligation, entertain the
petition so as to expedite the adjudication of all, especially the constitutional, issues.

In any event, the Court has ample authority to set aside errors of practice or technicalities
of procedure and resolve the merits of a case. Repeatedly stressed in our prior decisions is the
principle that the Rules were promulgated to provide guidelines for the orderly administration of
justice, not to shackle the hand that dispenses it. Otherwise, the courts would be consigned to
being mere slaves to technical rules, deprived of their judicial discretion.[14]

II.

To put things in their proper perspective, it is imperative that we trace the brief history of
the assailed provision. Section 4(a) of COMELEC Resolution No. 8678 is a reproduction of the
second proviso in the third paragraph of Section 13 of R.A. No. 9369, which for ready reference
is quoted as follows:

For this purpose, the Commission shall set the deadline for the filing of
certificate of candidacy/petition for registration/manifestation to participate in the
election. Any person who files his certificate of candidacy within this period shall
only be considered as a candidate at the start of the campaign period for which he
filed his certificate of candidacy: Provided, That, unlawful acts or omissions
applicable to a candidate shall take effect only upon the start of the aforesaid
campaign period: Provided, finally, That any person holding a public appointive
office or position, including active members of the armed forces, and officers and
employees in government-owned or -controlled corporations, shall be
considered ipso facto resigned from his/her office and must vacate the same at the
start of the day of the filing of his/her certificate of candidacy.[15]

Notably, this proviso is not present in Section 11 of R.A. No. 8436, the law amended by
R.A. No. 9369. The proviso was lifted from Section 66 of B.P. Blg. 881 or the Omnibus Election
Code (OEC) of the Philippines, which reads:

Sec. 66. Candidates holding appointive office or position.—Any person


holding a public appointive office or position, including active members of the
Armed Forces of the Philippines, and officers and employees in government-owned
or controlled corporations, shall be considered ipso facto resigned from his office
upon the filing of his certificate of candidacy.

It may be recalled—in inverse chronology—that earlier, Presidential Decree No. 1296, or


the 1978 Election Code, contained a similar provision, thus—

773
SECTION 29. Candidates holding appointive office or position. — Every
person holding a public appointive office or position, including active members of
the Armed Forces of the Philippines, and officers and employees in government-
owned or controlled corporations, shall ipso facto cease in his office or position on
the date he files his certificate of candidacy. Members of the Cabinet shall continue
in the offices they presently hold notwithstanding the filing of certificate of
candidacy, subject to the pleasure of the President of the Philippines.

Much earlier, R.A. No. 6388, or the Election Code of 1971, likewise stated in its Section 23
the following:

SECTION 23. Candidates Holding Appointive Office or Position. — Every


person holding a public appointive office or position, including active members of
the Armed Forces of the Philippines and every officer or employee in government-
owned or controlled corporations, shall ipso facto cease in his office or position on
the date he files his certificate of candidacy: Provided, That the filing of a certificate
of candidacy shall not affect whatever civil, criminal or administrative liabilities
which he may have incurred.

Going further back in history, R.A. No. 180, or the Revised Election Code approved on
June 21, 1947, also provided that—

SECTION 26. Automatic cessation of appointive officers and employees who


are candidates. — Every person holding a public appointive office or position
shall ipso facto cease in his office or position on the date he files his certificate of
candidacy.

During the Commonwealth era, Commonwealth Act (C.A.) No. 725, entitled “AN ACT TO
PROVIDE FOR THE NEXT ELECTION FOR PRESIDENT AND VICE-PRESIDENT OF THE
PHILIPPINES, SENATORS AND MEMBERS OF THE HOUSE OF REPRESENTATIVES, AND
APPROPRIATING THE NECESSARY FUNDS THEREFOR,” approved on January 5, 1946, contained,
in the last paragraph of its Section 2, the following:

A person occupying any civil office by appointment in the government or any of its
political subdivisions or agencies or government-owned or controlled corporations,
whether such office by appointive or elective, shall be considered to have resigned
from such office from the moment of the filing of such certificate of candidacy.

Significantly, however, C.A. No. 666, entitled “AN ACT TO PROVIDE FOR THE FIRST ELECTION
FOR PRESIDENT AND VICE-PRESIDENT OF THE PHILIPPINES, SENATORS, AND MEMBERS OF
THE HOUSE OF REPRESENTATIVES, UNDER THE CONSTITUTION AND THE AMENDMENTS

774
THEREOF,” enacted without executive approval on June 22, 1941, the precursor of C.A. No. 725,
only provided for automatic resignation of elective, but not appointive, officials.

Nevertheless, C.A. No. 357, or the Election Code approved on August 22, 1938, had, in its
Section 22, the same verbatim provision as Section 26 of R.A. No. 180.

The earliest recorded Philippine law on the subject is Act No. 1582, or the Election Law
enacted by the Philippine Commission in 1907, the last paragraph of Section 29 of which reads:

Sec. 29. Penalties upon officers.— x x x.

No public officer shall offer himself as a candidate for election, nor shall he be
eligible during the time that he holds said public office to election, at any municipal,
provincial or Assembly election, except for reelection to the position which he may
be holding, and no judge of the Court of First Instance, justice of the peace,
provincial fiscal, or officer or employee of the Bureau of Constabulary or of the
Bureau of Education shall aid any candidate or influence in any manner or take any
part in any municipal, provincial, or Assembly election under penalty of being
deprived of his office and being disqualified to hold any public office whatever for a
term of five years: Provided, however, That the foregoing provisions shall not be
construed to deprive any person otherwise qualified of the right to vote at any
election.

From this brief historical excursion, it may be gleaned that the second proviso in the third
paragraph of Section 13 of R.A. No. 9369—that any person holding a public appointive office or
position, including active members of the armed forces, and officers, and employees in
government-owned or controlled corporations, shall be considered ipso facto resigned from
his/her office and must vacate the same at the start of the day of the filing of his/her certificate
of candidacy—traces its roots to the period of the American occupation.

In fact, during the deliberations of Senate Bill No. 2231, the bill later to be consolidated
with House Bill No. 5352 and enacted as R.A. No. 9369, Senator Richard Gordon, the principal
author of the bill, acknowledged that the said proviso in the proposed legislative measure is an
old provision which was merely copied from earlier existing legislation, thus—

Senator Osmeña. May I just opine here and perhaps obtain the opinion of
the good Sponsor. This reads like, “ANY PERSON HOLDING [means currently] A
PUBLIC APPOINTIVE POSITION… SHALL BE CONSIDERED IPSO FACTO RESIGNED”
[which means that the prohibition extends only to appointive officials] “INCLUDING
ACTIVE MEMBERS OF THE ARMED FORCES, OFFICERS AND EMPLOYEES”… This is a
prohibition, Mr. President. This means if one is chairman of SSS or PDIC, he is

775
deemed ipso facto resigned when he files his certificate of candidacy. Is that the
intention?

Senator Gordon. This is really an old provision, Mr. President.

Senator Osmeña. It is in bold letters, so I think it was a Committee


amendment.

Senator Gordon. No, it has always been there.

Senator Osmeña. I see.

Senator Gordon. I guess the intention is not to give them undue


advantage, especially certain people.

Senator Osmeña. All right.[16]

In that Senate deliberation, however, Senator Miriam Defensor-Santiago expressed her


concern over the inclusion of the said provision in the new law, given that the same would be
disadvantageous and unfair to potential candidates holding appointive positions, while it grants a
consequent preferential treatment to elective officials, thus—

Senator Santiago. On page 15, line 31, I know that this is a losing cause,
so I make this point more as a matter of record than of any feasible hope that it
can possibly be either accepted or if we come to a division of the House, it will be
upheld by the majority.

I am referring to page 15, line 21. The proviso begins: “PROVIDED


FINALLY, THAT ANY PERSON HOLDING A PUBLIC APPOINTIVE OFFICE…SHALL BE
CONSIDERED IPSO FACTO RESIGNED FROM HIS/HER OFFICE.”

The point that I made during the appropriate debate in the past in this Hall
is that there is, for me, no valid reason for exempting elective officials from this
inhibition or disqualification imposed by the law. If we are going to consider
appointive officers of the government, including AFP members and officers of
government-owned and controlled corporations, or any other member of the
appointive sector of the civil service, why should it not apply to the elective sector
for, after all, even senators and congressmen are members of the civil service as
well?

Further, it is self-serving for the Senate, or for the Congress in general, to


give an exception to itself which is not available to other similarly situated officials
of government. Of course, the answer is, the reason why we are special is that we
are elected. Since we are imposing a disqualification on all other government
officials except ourselves, I think, it is the better part of delicadeza to inhibit
ourselves as well, so that if we want to stay as senators, we wait until our term
expires. But if we want to run for some other elective office during our term, then

776
we have to be considered resigned just like everybody else. That is my proposed
amendment. But if it is unacceptable to the distinguished Sponsor, because of
sensitivity to the convictions of the rest of our colleagues, I will understand.

Senator Gordon. Mr. President, I think the suggestion is well-thought of. It


is a good policy. However, this is something that is already in the old law which
was upheld by the Supreme court in a recent case that the rider was not upheld
and that it was valid.[17]

The obvious inequality brought about by the provision on automatic resignation of


appointive civil servants must have been the reason why Senator Recto proposed the inclusion
of the following during the period of amendments: “ANY PERSON WHO FILES HIS CERTIFICATE
OF CANDIDACY WITHIN THIS PERIOD SHALL ONLY BE CONSIDERED AS A CANDIDATE AT THE
START OF THE CAMPAIGN PERIOD FOR WHICH HE FILED HIS COC.”[18] The said proviso seems
to mitigate the situation of disadvantage afflicting appointive officials by considering persons
who filed their CoCs as candidates only at the start of the campaign period, thereby, conveying
the tacit intent that persons holding appointive positions will only be considered as resigned at
the start of the campaign period when they are already treated by law as candidates.

Parenthetically, it may be remembered that Section 67 of the OEC and Section 11 of R.A.
No. 8436 contained a similar provision on automatic resignation of elective officials upon the
filing of their CoCs for any office other than that which they hold in a permanent capacity or for
President or Vice-President. However, with the enactment of R.A. No. 9006, or the Fair Election
Act,[19] in 2001, this provision was repealed by Section 14[20] of the said act. There was, thus,
created a situation of obvious discrimination against appointive officials who were deemed ipso
facto resigned from their offices upon the filing of their CoCs, while elective officials were not.

This situation was incidentally addressed by the Court in Fariñas v. The Executive
Secretary[21] when it ruled that—

Section 14 of Rep. Act No. 9006


Is Not Violative of the Equal
Protection Clause of the Constitution

The petitioners’ contention, that the repeal of Section 67 of the Omnibus


Election Code pertaining to elective officials gives undue benefit to such officials as
against the appointive ones and violates the equal protection clause of the
constitution, is tenuous.

The equal protection of the law clause in the Constitution is not absolute, but
is subject to reasonable classification. If the groupings are characterized by
777
substantial distinctions that make real differences, one class may be treated and
regulated differently from the other. The Court has explained the nature of the
equal protection guarantee in this manner:

The equal protection of the law clause is against undue favor


and individual or class privilege, as well as hostile discrimination or the
oppression of inequality. It is not intended to prohibit legislation which
is limited either in the object to which it is directed or by territory
within which it is to operate. It does not demand absolute equality
among residents; it merely requires that all persons shall be treated
alike, under like circumstances and conditions both as to privileges
conferred and liabilities enforced. The equal protection clause is not
infringed by legislation which applies only to those persons falling
within a specified class, if it applies alike to all persons within such
class, and reasonable grounds exist for making a distinction between
those who fall within such class and those who do not.

Substantial distinctions clearly exist between elective officials and appointive


officials. The former occupy their office by virtue of the mandate of the electorate.
They are elected to an office for a definite term and may be removed therefrom
only upon stringent conditions. On the other hand, appointive officials hold their
office by virtue of their designation thereto by an appointing authority. Some
appointive officials hold their office in a permanent capacity and are entitled to
security of tenure while others serve at the pleasure of the appointing authority.

Another substantial distinction between the two sets of officials is that under
Section 55, Chapter 8, Title I, Subsection A. Civil Service Commission, Book V of
the Administrative Code of 1987 (Executive Order No. 292), appointive officials, as
officers and employees in the civil service, are strictly prohibited from engaging in
any partisan political activity or take part in any election except to vote. Under the
same provision, elective officials, or officers or employees holding political offices,
are obviously expressly allowed to take part in political and electoral activities.

By repealing Section 67 but retaining Section 66 of the Omnibus Election


Code, the legislators deemed it proper to treat these two classes of officials
differently with respect to the effect on their tenure in the office of the filing of the
certificates of candidacy for any position other than those occupied by
them. Again, it is not within the power of the Court to pass upon or look into the
wisdom of this classification.

Since the classification justifying Section 14 of Rep. Act No. 9006, i.e.,
elected officials vis-a-vis appointive officials, is anchored upon material and
significant distinctions and all the persons belonging under the same classification
are similarly treated, the equal protection clause of the Constitution is, thus, not
infringed.[22]

However, it must be remembered that the Court, in Fariñas, was intently focused on the
main issue of whether the repealing clause in the Fair Election Act was a constitutionally
778
proscribed rider, in that it unwittingly failed to ascertain with stricter scrutiny the impact of the
retention of the provision on automatic resignation of persons holding appointive positions
(Section 66) in the OEC, vis-à-vis the equal protection clause. Moreover, the Court’s vision
in Fariñas was shrouded by the fact that petitioners therein, Fariñas et al., never posed a direct
challenge to the constitutionality of Section 66 of the OEC. Fariñas et al. rather merely
questioned, on constitutional grounds, the repealing clause, or Section 14 of the Fair Election
Act. The Court’s afore-quoted declaration in Fariñas may then very well be considered as
an obiter dictum.

III.

The instant case presents a rare opportunity for the Court, in view of the constitutional
challenge advanced by petitioners, once and for all, to settle the issue of whether the second
proviso in the third paragraph of Section 13 of R.A. No. 9369, a reproduction of Section 66 of
the OEC, which, as shown above, was based on provisions dating back to the American
occupation, is violative of the equal protection clause.

But before delving into the constitutional issue, we shall first address the issues on legal
standing and on the existence of an actual controversy.

Central to the determination of locus standi is the question of whether a party has alleged
such a personal stake in the outcome of the controversy as to assure that concrete adverseness
which sharpens the presentation of issues upon which the court so largely depends for
illumination of difficult constitutional questions.[23] In this case, petitioners allege that they will
be directly affected by COMELEC Resolution No. 8678 for they intend, and they all have the
qualifications, to run in the 2010 elections. The OSG, for its part, contends that since petitioners
have not yet filed their CoCs, they are not yet candidates; hence, they are not yet directly
affected by the assailed provision in the COMELEC resolution.

The Court, nevertheless, finds that, while petitioners are not yet candidates, they have
the standing to raise the constitutional challenge, simply because they are qualified voters. A
restriction on candidacy, such as the challenged measure herein, affects the rights of voters to
choose their public officials. The rights of voters and the rights of candidates do not lend
themselves to neat separation; laws that affect candidates always have at least some
theoretical, correlative effect on voters.[24]The Court believes that both candidates and voters
may challenge, on grounds of equal protection, the assailed measure because of its impact on
voting rights.[25]

779
In any event, in recent cases, this Court has relaxed the stringent direct injury test and
has observed a liberal policy allowing ordinary citizens, members of Congress, and civil
organizations to prosecute actions involving the constitutionality or validity of laws, regulations
and rulings.[26]

We have also stressed in our prior decisions that the exercise by this Court of judicial
power is limited to the determination and resolution of actual cases and controversies.[27] The
Court, in this case, finds that an actual case or controversy exists between the petitioners and
the COMELEC, the body charged with the enforcement and administration of all election laws.
Petitioners have alleged in a precise manner that they would engage in the very acts that would
trigger the enforcement of the provision—they would file their CoCs and run in the 2010
elections. Given that the assailed provision provides for ipso factoresignation upon the filing of
the CoC, it cannot be said that it presents only a speculative or hypothetical obstacle to
petitioners’ candidacy.[28]

IV.

Having hurdled what the OSG posed as obstacles to judicial review, the Court now delves
into the constitutional challenge.

It is noteworthy to point out that the right to run for public office touches on two
fundamental freedoms, those of expression and of association. This premise is best explained
in Mancuso v. Taft,[29] viz.:

Freedom of expression guarantees to the individual the opportunity to write a


letter to the local newspaper, speak out in a public park, distribute handbills
advocating radical reform, or picket an official building to seek redress of
grievances. All of these activities are protected by the First Amendment if done in a
manner consistent with a narrowly defined concept of public order and safety. The
choice of means will likely depend on the amount of time and energy the individual
wishes to expend and on his perception as to the most effective method of
projecting his message to the public. But interest and commitment are evolving
phenomena. What is an effective means for protest at one point in time may not
seem so effective at a later date. The dilettante who participates in a picket line
may decide to devote additional time and resources to his expressive activity. As
his commitment increases, the means of effective expression changes, but the
expressive quality remains constant. He may decide to lead the picket line, or to
publish the newspaper. At one point in time he may decide that the most effective
way to give expression to his views and to get the attention of an appropriate
audience is to become a candidate for public office-means generally considered
among the most appropriate for those desiring to effect change in our
governmental systems. He may seek to become a candidate by filing in a general
election as an independent or by seeking the nomination of a political party. And in
780
the latter instance, the individual's expressive activity has two dimensions: besides
urging that his views be the views of the elected public official, he is also
attempting to become a spokesman for a political party whose substantive program
extends beyond the particular office in question. But Cranston has said that a
certain type of its citizenry, the public employee, may not become a candidate and
may not engage in any campaign activity that promotes himself as a candidate for
public office. Thus the city has stifled what may be the most important expression
an individual can summon, namely that which he would be willing to effectuate, by
means of concrete public action, were he to be selected by the voters.

It is impossible to ignore the additional fact that the right to run for office
also affects the freedom to associate. In Williams v. Rhodes,supra, the Court used
strict review to invalidate an Ohio election system that made it virtually impossible
for third parties to secure a place on the ballot. The Court found that the First
Amendment protected the freedom to associate by forming and promoting a
political party and that that freedom was infringed when the state effectively denied
a party access to its electoral machinery. The Cranston charter provision before us
also affects associational rights, albeit in a slightly different way. An individual may
decide to join or participate in an organization or political party that shares his
beliefs. He may even form a new group to forward his ideas. And at some juncture
his supporters and fellow party members may decide that he is the ideal person to
carry the group's standard into the electoral fray. To thus restrict the options
available to political organization as the Cranston charter provision has done is to
limit the effectiveness of association; and the freedom to associate is intimately
related with the concept of making expression effective. Party access to the ballot
becomes less meaningful if some of those selected by party machinery to carry the
party's programs to the people are precluded from doing so because those
nominees are civil servants.

Whether the right to run for office is looked at from the point of view of
individual expression or associational effectiveness, wide opportunities exist for the
individual who seeks public office. The fact of candidacy alone may open previously
closed doors of the media. The candidate may be invited to discuss his views on
radio talk shows; he may be able to secure equal time on television to elaborate his
campaign program; the newspapers may cover his candidacy; he may be invited to
debate before various groups that had theretofore never heard of him or his views.
In short, the fact of candidacy opens up a variety of communicative possibilities
that are not available to even the most diligent of picketers or the most loyal of
party followers. A view today, that running for public office is not an interest
protected by the First Amendment, seems to us an outlook stemming from an
earlier era when public office was the preserve of the professional and the
wealthy. Consequently we hold that candidacy is both a protected First Amendment
right and a fundamental interest. Hence any legislative classification that
significantly burdens that interest must be subjected to strict equal protection
review.[30]

Here, petitioners’ interest in running for public office, an interest protected by Sections 4
and 8 of Article III of the Constitution, is breached by the proviso in Section 13 of R.A. No. 9369.
781
It is now the opportune time for the Court to strike down the said proviso for being violative of
the equal protection clause and for being overbroad.

In considering persons holding appointive positions as ipso facto resigned from their posts
upon the filing of their CoCs, but not considering as resigned all other civil servants, specifically
the elective ones, the law unduly discriminates against the first class. The fact alone that there is
substantial distinction between those who hold appointive positions and those occupying elective
posts, does not justify such differential treatment.

In order that there can be valid classification so that a discriminatory governmental act
may pass the constitutional norm of equal protection, it is necessary that the four (4) requisites
of valid classification be complied with, namely:

(1) It must be based upon substantial distinctions;


(2) It must be germane to the purposes of the law;
(3) It must not be limited to existing conditions only; and
(4) It must apply equally to all members of the class.

The first requirement means that there must be real and substantial differences between
the classes treated differently. As illustrated in the fairly recent Mirasol v. Department of Public
Works and Highways,[31] a real and substantial distinction exists between a motorcycle and other
motor vehicles sufficient to justify its classification among those prohibited from plying the toll
ways. Not all motorized vehicles are created equal—a two-wheeled vehicle is less stable and
more easily overturned than a four-wheel vehicle.

Nevertheless, the classification would still be invalid if it does not comply with the second
requirement—if it is not germane to the purpose of the law. Justice Isagani A. Cruz (Ret.), in his
treatise on constitutional law, explains,

The classification, even if based on substantial distinctions, will still be


invalid if it is not germane to the purpose of the law. To illustrate, the accepted
difference in physical stamina between men and women will justify the prohibition
of the latter from employment as miners or stevedores or in other heavy and
strenuous work. On the basis of this same classification, however, the law cannot
provide for a lower passing average for women in the bar examinations because
physical strength is not the test for admission to the legal profession. Imported cars
may be taxed at a higher rate than locally assembled automobiles for the protection
of the national economy, but their difference in origin is no justification for treating
them differently when it comes to punishing violations of traffic regulations. The
source of the vehicle has no relation to the observance of these rules.[32]

782
The third requirement means that the classification must be enforced not only for the
present but as long as the problem sought to be corrected continues to exist. And, under the last
requirement, the classification would be regarded as invalid if all the members of the class are
not treated similarly, both as to rights conferred and obligations imposed.[33]

Applying the four requisites to the instant case, the Court finds that the differential
treatment of persons holding appointive offices as opposed to those holding elective ones is not
germane to the purposes of the law.

The obvious reason for the challenged provision is to prevent the use of a governmental
position to promote one’s candidacy, or even to wield a dangerous or coercive influence on the
electorate. The measure is further aimed at promoting the efficiency, integrity, and discipline of
the public service by eliminating the danger that the discharge of official duty would be
motivated by political considerations rather than the welfare of the public.[34] The restriction is
also justified by the proposition that the entry of civil servants to the electoral arena, while still
in office, could result in neglect or inefficiency in the performance of duty because they would be
attending to their campaign rather than to their office work.

If we accept these as the underlying objectives of the law, then the assailed provision
cannot be constitutionally rescued on the ground of valid classification. Glaringly absent is the
requisite that the classification must be germane to the purposes of the law. Indeed, whether
one holds an appointive office or an elective one, the evils sought to be prevented by the
measure remain. For example, the Executive Secretary, or any Member of the Cabinet for that
matter, could wield the same influence as the Vice-President who at the same time is appointed
to a Cabinet post (in the recent past, elected Vice-Presidents were appointed to take charge of
national housing, social welfare development, interior and local government, and foreign affairs).
With the fact that they both head executive offices, there is no valid justification to treat them
differently when both file their CoCs for the elections. Under the present state of our law, the
Vice-President, in the example, running this time, let us say, for President, retains his position
during the entire election period and can still use the resources of his office to support his
campaign.

As to the danger of neglect, inefficiency or partisanship in the discharge of the functions of


his appointive office, the inverse could be just as true and compelling. The public officer who
files his certificate of candidacy would be driven by a greater impetus for excellent performance
to show his fitness for the position aspired for.

783
Mancuso v. Taft,[35] cited above, explains that the measure on automatic resignation,
which restricts the rights of civil servants to run for office—a right inextricably linked to their
freedom of expression and association, is not reasonably necessary to the satisfaction of the
state interest. Thus, in striking down a similar measure in the United States, Mancuso succinctly
declares—

In proceeding to the second stage of active equal protection review,


however, we do see some contemporary relevance of the Mitchelldecision. National
Ass'n of Letter Carriers, supra. In order for the Cranston charter provision to
withstand strict scrutiny, the city must show that the exclusion of all government
employees from candidacy is necessary to achieve a compelling state interest. And,
as stated in Mitchell and other cases dealing with similar
statutes, see Wisconsin State Employees, supra; Broadrick, supra, government at
all levels has a substantial interest in protecting the integrity of its civil service. It is
obviously conceivable that the impartial character of the civil service would be
seriously jeopardized if people in positions of authority used their discretion to
forward their electoral ambitions rather than the public welfare. Similarly if a public
employee pressured other fellow employees to engage in corrupt practices in return
for promises of post-election reward, or if an employee invoked the power of the
office he was seeking to extract special favors from his superiors, the civil service
would be done irreparable injury. Conversely, members of the public, fellow-
employees, or supervisors might themselves request favors from the candidate or
might improperly adjust their own official behavior towards him. Even if none of
these abuses actually materialize, the possibility of their occurrence might seriously
erode the public's confidence in its public employees. For the reputation of
impartiality is probably as crucial as the impartiality itself; the knowledge that a
clerk in the assessor's office who is running for the local zoning board has access to
confidential files which could provide “pressure” points for furthering his campaign
is destructive regardless of whether the clerk actually takes advantage of his
opportunities. For all of these reasons we find that the state indeed has a
compelling interest in maintaining the honesty and impartiality of its public work
force.

We do not, however, consider the exclusionary measure taken by Cranston-a


flat prohibition on office-seeking of all kinds by all kinds of public employees-as
even reasonably necessary to satisfaction of this state interest. As Justice Marshall
pointed out in Dunn v. Blumstein,“[s]tatutes affecting constitutional rights must be
drawn with ‘precision.’” For three sets of reasons we conclude that
the Cranston charter provision pursues its objective in a far too heavy-handed
manner and hence must fall under the equal protection clause. First, we think the
nature of the regulation-a broad prophylactic rule-may be unnecessary to fulfillment
of the city's objective. Second, even granting some sort of prophylactic rule may be
required, the provision here prohibits candidacies for all types of public office,
including many which would pose none of the problems at which the law is aimed.
Third, the provision excludes the candidacies of all types of public employees,
without any attempt to limit exclusion to those employees whose positions make
them vulnerable to corruption and conflicts of interest.
784
There is thus no valid justification to treat appointive officials differently from the elective
ones. The classification simply fails to meet the test that it should be germane to the purposes
of the law. The measure encapsulated in the second proviso of the third paragraph of Section
13 of R.A. No. 9369 and in Section 66 of the OEC violates the equal protection clause.

V.

The challenged provision also suffers from the infirmity of being overbroad.

First, the provision pertains to all civil servants holding appointive posts without
distinction as to whether they occupy high positions in government or not. Certainly, a utility
worker in the government will also be considered as ipso facto resigned once he files his CoC for
the 2010 elections. This scenario is absurd for, indeed, it is unimaginable how he can use his
position in the government to wield influence in the political world.

While it may be admitted that most appointive officials who seek public elective office are
those who occupy relatively high positions in government, laws cannot be legislated for them
alone, or with them alone in mind. For the right to seek public elective office is universal, open
and unrestrained, subject only to the qualification standards prescribed in the Constitution and
in the laws. These qualifications are, as we all know, general and basic so as to allow the widest
participation of the citizenry and to give free rein for the pursuit of one’s highest aspirations to
public office. Such is the essence of democracy.

Second, the provision is directed to the activity of seeking any and all public
offices, whether they be partisan or nonpartisan in character, whether they be in the national,
municipal or barangay level. Congress has not shown a compelling state interest to restrict the
fundamental right involved on such a sweeping scale.[36]

Specific evils require specific treatments, not through overly broad measures that unduly
restrict guaranteed freedoms of the citizenry. After all, sovereignty resides in the people, and all
governmental power emanates from them.

Mancuso v. Taft,[37] on this point, instructs—

As to approaches less restrictive than a prophylactic rule, there exists the


device of the leave of absence. Some system of leaves of absence would permit the
public employee to take time off to pursue his candidacy while assuring him his old
job should his candidacy be unsuccessful. Moreover, a leave of absence policy
785
would eliminate many of the opportunities for engaging in the questionable
practices that the statute is designed to prevent. While campaigning, the candidate
would feel no conflict between his desire for election and his publicly entrusted
discretion, nor any conflict between his efforts to persuade the public and his
access to confidential documents. But instead of adopting a reasonable leave of
absence policy, Cranston has chosen a provision that makes the public employee
cast off the security of hard-won public employment should he desire to compete
for elected office.

The city might also promote its interest in the integrity of the civil service by
enforcing, through dismissal, discipline, or criminal prosecution, rules or statutes
that treat conflict of interests, bribery, or other forms of official corruption. By thus
attacking the problem directly, instead of using a broad prophylactic rule, the city
could pursue its objective without unduly burdening the First Amendment rights of
its employees and the voting rights of its citizens. Last term in Dunn v. Blumstein,
the Supreme Court faced an analogous question when the State
ofTennessee asserted that the interest of “ballot box purity” justified its imposition
of one year and three month residency requirements before a citizen could vote.
Justice Marshall stated, inter alia, that Tennessee had available a number of
criminal statutes that could be used to punish voter fraud without unnecessary
infringement on the newcomer's right to vote. Similarly, it appears from the record
in this case that the Cranstoncharter contains some provisions that might be used
against opportunistic public employees.

Even if some sort of prophylactic rule is necessary, we cannot say


that Cranston has put much effort into tailoring a narrow provision that attempts to
match the prohibition with the problem. The charter forbids a Cranston public
employee from running for any office, anywhere. The prohibition is not limited
to the local offices of Cranston, but rather extends to statewide offices and even to
national offices. It is difficult for us to see that a public employee running for the
United States Congress poses quite the same threat to the civil service as would the
same employee if he were running for a local office where the contacts and
information provided by his job related directly to the position he was seeking, and
hence where the potential for various abuses was greater. Nor does
the Cranston charter except the public employee who works in Cranstonbut aspires
to office in another local jurisdiction, most probably his town of residence. Here
again the charter precludes candidacies which can pose only a remote threat to the
civil service. Finally, the charter does not limit its prohibition to partisan office-
seeking, but sterilizes also those public employees who would seek nonpartisan
elective office. The statute reviewed in Mitchell was limited to partisan political
activity, and since that time other courts have found the partisan-nonpartisan
distinction a material one. See Kinnear, supra; Wisconsin State Employees,
supra; Gray v. Toledo, supra. While the line between nonpartisan and partisan can
often be blurred by systems whose true characters are disguised by the names
given them by their architects, it seems clear that the concerns of a truly partisan
office and the temptations it fosters are sufficiently different from those involved in
an office removed from regular party politics to warrant distinctive treatment in a
charter of this sort.

786
The third and last area of excessive and overinclusive coverage of
the Cranston charter relates not to the type of office sought, but to the type of
employee seeking the office. As Justice Douglas pointed out in his dissent
in Mitchell, 330 U.S. at 120-126, 67 S.Ct. 556, restrictions on administrative
employees who either participate in decision-making or at least have some access
to information concerning policy matters are much more justifiable than restrictions
on industrial employees, who, but for the fact that the government owns the plant
they work in, are, for purposes of access to official information, identically situated
to all other industrial workers. Thus, a worker in the Philadelphia mint could be
distinguished from a secretary in an office of the Department of Agriculture; so also
could a janitor in the public schools of Cranston be distinguished from an assistant
comptroller of the same city. A second line of distinction that focuses on the type of
employee is illustrated by the cases of Kinnear and Minielly, supra. In both of these
cases a civil service deputy decided to run for the elected office of sheriff. The
courts in both cases felt that the no-candidacy laws in question were much too
broad and indicated that perhaps the only situation sensitive enough to justify a flat
rule was one in which an inferior in a public office electorally challenged his
immediate superior. Given all these considerations, we think Cranston has not
given adequate attention to the problem of narrowing the terms of its charter to
deal with the specific kinds of conflict-of-interest problems it seeks to avoid.

We also do not find convincing the arguments that after-hours campaigning


will drain the energy of the public employee to the extent that he is incapable of
performing his job effectively and that inevitable on-the-job campaigning and
discussion of his candidacy will disrupt the work of others. Although it is
indisputable that the city has a compelling interest in the performance of official
work, the exclusion is not well-tailored to effectuate that interest. Presumably the
city could fire the individual if he clearly shirks his employment responsibilities or
disrupts the work of others. Also, the efficiency rationale common to both
arguments is significantly underinclusive. It applies equally well to a number of
non-political, extracurricular activities that are not prohibited by
the Cranston charter. Finally, the connection between after-hours campaigning and
the state interest seems tenuous; in many cases a public employee would be able
to campaign aggressively and still continue to do his job well.[38]

Incidentally, Clements v. Fashing[39] sustained as constitutional a provision on the


automatic resignation of District Clerks, County Clerks, County Judges, County Treasurers,
Criminal District Attorneys, County Surveyors, Inspectors of Hides and Animals, County
Commissioners, Justices of the Peace, Sheriffs, Assessors and Collectors of Taxes, District
Attorneys, County Attorneys, Public Weighers, and Constables if they announce their candidacy
or if they become candidates in any general, special or primary election.

In Clements, it may be readily observed that a provision treating differently particular


officials, as distinguished from all others, under a classification that is germane to the purposes
of the law, merits the stamp of approval from American courts. Not, however, a general and
787
sweeping provision, and more so one violative of the second requisite for a valid classification,
which is on its face unconstitutional.

On a final note, it may not be amiss to state that the Americans, from whom we copied
the provision in question, had already stricken down a similar measure for being
unconstitutional. It is high-time that we, too, should follow suit and, thus, uphold fundamental
liberties over age-old, but barren, restrictions to such freedoms.

WHEREFORE, premises considered, the petition is GRANTED. The second proviso in the
third paragraph of Section 13 of Republic Act No. 9369, Section 66 of the Omnibus Election Code
and Section 4(a) of COMELEC Resolution No. 8678 are declared as UNCONSTITUTIONAL.

SO ORDERED.

ANTONIO EDUARDO B. NACHURA


Associate Justice

WE CONCUR:

REYNATO S. PUNO
Chief Justice

ANTONIO T. CARPIO RENATO C. CORONA


Associate Justice Associate Justice

CONCHITA CARPIO MORALES MINITA V. CHICO-NAZARIO


Associate Justice Associate Justice

PRESBITERO J. VELASCO, JR. TERESITA J. LEONARDO-DE CASTRO


Associate Justice Associate Justice

788
ARTURO D. BRION DIOSDADO M. PERALTA
Associate Justice Associate Justice

LUCAS P. BERSAMIN MARIANO C. DEL CASTILLO


Associate Justice Associate Justice

ROBERTO A. ABAD MARTIN S. VILLARAMA, JR.


Associate Justice Associate Justice

CERTIFICATION

Pursuant to Section 13, Article VIII of the Constitution, I certify that the conclusions in
the above decision had been reached in consultation before the case was assigned to the writer
of the opinion of the Court.

REYNATO S. PUNO
Chief Justice

[1]
Salvacion v. Central Bank of the Philippines, G.R. No. 94723, August 21, 1997, 278
SCRA 27, 28.
[2]
Emphasis supplied.
[3]
Emphasis supplied.
[4]
Promulgated on October 6, 2009.
[5]
Petitioner Eleazar P. Quinto is the Undersecretary for Field Operations of the
Department of Environment and Natural Resources (DENR). He intends to run for Representative
in the 4th Congressional District of Pangasinan. Petitioner Gerino A. Tolentino, Jr. is the OIC-
Director of the Land Management Bureau of the DENR. He likewise desires to run for City
Councilor in the 4th District of Manila. (Rollo, pp. 8-9.)
[6]
Rollo, pp. 10-13.
[7]
Id. at 11.
[8]
Id. at 12-13.
[9]
Comment of the OSG, pp. 11-26.

789
[10]
Id. at 27-40.
[11]
The first paragraph of Sec. 1 of Rule 65 provides:
SECTION 1. Petition for certiorari.—When any tribunal, board or officer exercising
judicial or quasi-judicial functions has acted without or in excess of its or his jurisdiction, or with
grave abuse of discretion amounting to lack or excess of jurisdiction, and there is no appeal, nor
any plain, speedy, and adequate remedy in the ordinary course of law, a person aggrieved
thereby may file a verified petition in the proper court, alleging the facts with certainty and
praying that judgment be rendered annulling or modifying the proceedings of such tribunal,
board or officer, and granting such incidental reliefs as law and justice may
require. (See Patalinghug v. Commission on Elections, G.R. No. 178767, January 30, 2008, 543
SCRA 175, 184-185.)
[12]
The first paragraph of Sec. 1 of Rule 63 provides:
SECTION 1. Who may file petition.—Any person interested under a deed, will,
contract or other written instrument, or whose rights are affected by a statute, executive order
or regulation, ordinance, or any other governmental regulation may, before breach or violation
thereof, bring an action in the appropriate Regional Trial Court to determine any question of
construction or validity arising, and for a declaration of his rights or duties,
thereunder. (See Almeda v. Bathala Marketing Industries, Inc., G.R. No. 150806, January 28,
2008, 542 SCRA 470, 478-479; John Hay Peoples Alternative Coalition v. Lim, G.R. No. 119775,
October 24, 2003, 414 SCRA 356, 369.)
[13]
Salvacion v. Central Bank of the Philippines, supra note 1, at 39.
[14]
MCC Industrial Sales Corporation v. Ssangyong Corporation, G.R. No. 170633,
October 17, 2007, 536 SCRA 408, 433.
[15]
Emphasis supplied.
[16]
Record of the Senate, Vol. III, Session No. 29, September 27, 2006, pp. 69-70.
[17]
Record of the Senate, Vol. III, Session No. 12, August 16, 2006, pp. 71-72.
[18]
Senate Records and Archives, 13th CP, 3rd Regular Session, Vol. III, August 1, 2006,
p. 25.
[19]
Entitled “AN ACT TO ENHANCE THE HOLDING OF FREE, ORDERLY, HONEST,
PEACEFUL AND CREDIBLE ELECTIONS THROUGH FAIR ELECTION PRACTICES,” approved on
February 12, 2001.
[20]
Sec. 14 of R.A. No. 9006 provides:
SEC. 14 Repealing Clause.—Sections 67 and 85 of the Omnibus Election Code (Batas
Pambansa Blg. 881) and Sections 10 and 11 of Republic Act No. 6646 are hereby repealed. As a
consequence, the first proviso in the third paragraph of Section 11 of Republic Act No. 8436 is
rendered ineffective. All laws, presidential decrees, executive orders, rules and regulations, or
any part thereof inconsistent with the provisions of this Act are hereby repealed or modified or
amended accordingly.
[21]
463 Phil. 179, 205-208 (2003).
[22]
Citations omitted.
[23]
Province of Batangas v. Romulo, G.R. No. 152774, May 27, 2004, 429 SCRA 736,
755.
[24]
Bullock v. Carter, 405 U.S. 134, 143 (1972).
[25]
Mancuso v. Taft, 476 F.2d 187, 190 (1973).
[26]
David v. Macapagal-Arroyo, G.R. Nos. 171396, 171409, 171485, 171483, 171400,
171489 and 171424, May 3, 2006, 489 SCRA 160, 218.
[27]
Dumlao v. COMELEC, G.R. No. L-52245, January 22, 1980, 95 SCRA 392, 401. This
case explains the standards that have to be followed in the exercise of the power of judicial

790
review, namely: (1) the existence of an appropriate case; (2) an interest personal and
substantial by the party raising the constitutional question; (3) the plea that the function be
exercised at the earliest opportunity; and (4) the necessity that the constitutional question be
passed upon in order to decide the case.
[28]
Clements v. Fashing, 457 U.S. 957, 960; 102 S.Ct. 2836, 2843 (1982).
[29]
Supra note 25, at 195-196.
[30]
Citations omitted.
[31]
G.R. No. 158793, June 8, 2006, 490 SCRA 318, 351-352.
[32]
Cruz, Constitutional Law (1998 ed.), p. 131.
[33]
Id. at 131-132.
[34]
Fort v. Civil Service Commission of the County of Alameda, 61 Cal.2d 331, 336; 392
P.2d 385, 388; 38 Cal.Rptr. 625, 628 (1964).

[35]
Supra note 25, at 198-199.
[36]
Kinnear v. City and County of San Francisco, 61 Cal.2d 341, 343; 392 P.2d 391,
392; 38 Cal.Rptr. 631, 632 (1964).
[37]
Supra note 25, at 199-201.
[38]
Citations omitted.
[39]
Supra note 28.

791

Das könnte Ihnen auch gefallen